You are on page 1of 627

2019

LSMU FINAL EXAM Q&A

3/27/2019
3/27/2019 LSMUSIS

Anatomy
I type tasks. Choose only one best answer

29.1 Structural components of the long bone in the case of splintery fracture are a cause of the fat embolism:
A. Periosteum
B. Red bone morrow
C. Yellow bone morrow
D. Spongy substance of the long bone

29.2 The facial canal fund in the:


A. Temporal bone
B. Parietal bone
C. Sphenoid bone
D. Ethmoid bone

29.5 Most movable synovial joint in the human body:


A. Temporomandibular joint
B. Shoulder joint
C. Hip joint
D. Metacarpophalangeal joints

29.8 Correct statement about the cranial nerves:


A. They are 31 pair
B. They consist of motor fibres which originate outside of the CNS
C. All cranial nerves contain afferent nerve fibres
D. Radixes (roots) of the cranial nerves emerging from brain stem are distributed on the ventral and
ventrolateral surfaces of brain stem (except the trochlear nerve)
E. All of them are mixt

29.9 Abductor muscles of the arm are innervated:


A. Axillar and thoracodorsal nerves
B. Suprascapular and axillar nerves
C. Musculocutaneus and median nerves
D. Axillar nerve
E. Radial nerve

29.10 Taste buds in the mucosa of the apex and body of the tongue are innervated by:
A. Glossopharyngeal nerve
B. Mandibular nerve
C. Vagus nerve
D. Facial nerve
E. Lingual nerve

29.11 Parasympathetic postganglionic fibres innervating palate mucosa glands originate in:
A. Geniculate ganglion
B. Otic ganglion
C. Pterygopalatine ganglion
D. Submandibular ganglion
E. Ciliary ganglion

29.12 Muscle not innervated by facial nerve:


A. M. orbicularis oculi
B. M. orbicularis oris
C. M. occipitofrontalis
D. M. levator anguli oris
E. M. levator palpebrae superioris

29.13 False statement concerning chorda tympani:


A. It innervates mucosa of the radix of the tongue
B. It passes through petrotympanic fissure
C. Part of fibres of the chorda tympani join the lingual nerve

lsmusis.lsmuni.lt/Klausimai/Spausdinti?Length=0?Kalba=EN&KategorijaId=131&Kalbos_input=EN&Kalbos=EN&KategorijaEn_input=Anatomy&Kateg… 1/23
3/27/2019 LSMUSIS
D. It contents specific taste fibres
E. It contents preganglionic parasympathetic fibres

29.14 Preganglionic parasympathetic fibres are not found in:


A. Vagus nerve
B. Facial nerve
C. Trigeminal nerve
D. Glossopharyngeal nerve
E. Oculomotor nerve

29.15 Parasympathetic nuclei are found in the spinal cord segments:


A. L 4,5 - S 1
B. S 2,3,4
C. L 2,3,4
D. S 1-5
E. Co 1

29.16 Correct answer about posterior crus of internal capsule:


A. It is between caudate nucleus and putamen
B. It is between thalamus and putamen
C. It is between thalamus and hypothalamus
D. It is between thalamus and globus pallidus

29.17 False answer about cortical centres:


A. The vision centre is found in the region of the calcarine sulcus
B. The hearing centre is in the gyri of insula
C. The general sensation centre is in gyrus postcentralis
D. The olfactory centre is in the uncus
E. Somatomotor centre is in the precentral gyrus

29.18 White matter band located between the head of caudate nucleus and putamen seen in the frontal section of
hemisphere is named:
A. Capsula externa
B. Genu of the capsula interna
C. Capsula extrema
D. Anterior crus of the capsula interna
E. Posterior crus of the capsula interna

29.19 Deep sensation impulses from ipsilateral side of the body arise to cerebellum in the:
A. Anterior spinocerebellar tract
B. Spinothalamic tract
C. Medial lemniscus
D. Posterior spinocerebellar tract

29.20 Spinal cord in newborn ends at:


A. L₅
B. L₂
C. L₃
D. S₃
E. S₅

29.21 False statement about symptoms after the different lesions:


A. Lesion of funiculus lateralis of spinal cord causes the loss of contralateral side superficial sensation
B. Lesion of the funiculus posterior of the spinal cord causes the loss of contralateral side superficial and
deep sensation
C. Lesion of the hypoglossal nerve contributes to tongue deviation to the side of involvement
D. Lesion of pedunculus cerebellaris causes chromatolysis in the ipsilateral nucleus thoracicus of the spinal
cord

29.22 Anatomical structure does not belong to the auditory afferent system:
A. Spiral ganglion
B. Trapezoid body
lsmusis.lsmuni.lt/Klausimai/Spausdinti?Length=0?Kalba=EN&KategorijaId=131&Kalbos_input=EN&Kalbos=EN&KategorijaEn_input=Anatomy&Kateg… 2/23
3/27/2019 LSMUSIS
C. Medial geniculate body
D. Medial lemniscus

29.23 False statement about the eyeball structure:


A. Photoreceptor cells are found in all parts of the retina
B. The aqueous humour of the eyeball from the posterior chamber drains to the anterior chamber and via
trabecular meshwork located at the limbus cornea and flows to scleral venous sinus (Schlemm's canal)
C. Light pass through the ganglionic and bipolar neurons of the retina
D. Nervous impulse spreads from the photoreceptors (cones and rods) to the bipolar neurons and to the
ganglionic neurons of the retina

29.24 False statement about the falx cerebri:


A. It is falciform dural matter derivation
B. Plates of cerebral falx compose the superior sagittal sinus
C. It originates from crista galli
D. It fasten posteriorly to protuberantia occipitalis externa
E. It is found in longitudinal cerebral fissure

29.25 False statement about the tentorium cerebelli:


A. It is a horizontal plate of the dura matter encephali
B. It divides cerebral hemisphera
C. It stabilize occipital lobes of the cerebral hemisphera
D. It forms the roof of the posterior cranial fossa

29.26 Correct statement about the pia matter covering:


A. It surrounds cerebrum and is intimately adherent to it
B. It does not surround neural radices in the subarachnoidal cavity
C. Below medullar conus it continues as terminal filum till S1
D. It is thin membrane without blood vessels

29.27 Venous blood from the sinus cavernous drains directly into the internal jugular vein through:
A. Sigmoid sinus
B. Rectus sinus
C. Occipital sinus
D. Inferior petrosal sinus
E. Inferior sagittal sinus

29.28 False statement concerning cerebrospinal fluid:


A. It drains to the venous system via the arachnoidal granulations
B. It circulates in the subarachnoid space
C. From the lateral ventricle of the brain it is drained to third ventricle and via interventricular foramina to the
fourth ventricle
D. Via medial openings in the roof of the fourth ventricle it flows to cerebellomedullar cisterna

29.29 Just after the ovulation oocyte from the ovary pass on:
A. Uterine tube
B. Uterus
C. Abdominal cavity
D. Peritoneal cavity

29.30 The fissure of the glottis, rima glottidis narrows after injury of nerve:
A. Phrenic nerve
B. Hypoglossal nerve loop
C. Cervical part of sympathetic trunk
D. Recurrens laryngeal nerve

29.31 The narrowest part of the male urethra:


A. Vesical (intramural) part
B. Prostatic part
C. Membranous part
D. Spongy part

29.32 After perforation of the ulcer of the posterior gastric wall content of the stomach effuse to:
lsmusis.lsmuni.lt/Klausimai/Spausdinti?Length=0?Kalba=EN&KategorijaId=131&Kalbos_input=EN&Kalbos=EN&KategorijaEn_input=Anatomy&Kateg… 3/23
3/27/2019 LSMUSIS
A. Omental bursa
B. Right mesenteric sinus
C. Subphrenic recess
D. Left mesenteric sinus

29.33 False statement about duodenum:


A. Originates on the left upper side of the abdominal cavity
B. Ends on the right upper side of the abdominal cavity
C. Length of the duodenum is about 15 cm
D. It has three parts

29.34 Part of the intestine lack of the mesentery:


A. Proximal part of jejunum
B. Distal part of ileum
C. Ascending colon
D. Transverse colon
E. Sigmoid colon

29.35 False statement about peritoneal cavity:


A. It contains small amount of the serous fluid
B. It boundaries of two layers of the peritoneum
C. Female peritoneal cavity is closed serous sac
D. It forms general peritoneal cavity and the omental bursa

29.36 Which of the following parts of the alimentary tract are adherent to the gallbladder and are common sites of
the cholecystenteric fistula, i.e. ulcerated junction between the gallbladder and intestine:
A. Inferior part of duodenum
B. Superior part of duodenum and transverse colon
C. Sigmoid colon
D. Descending colon
E. Rectum

29.37 Each of the following statements about a pleural cavity is correct except:
A. It is a large serous cavity when the lung is deflated
B. During respiration an air flow freely in and out of to the pleural cavity
C. It is a potential space between the layers of pleura
D. It contains a small amount of the serous fluid
E. The right and left pleural cavities are separated and closed

29.38 Part of respiratory pathway without cartilaginous tissue in the wall:


A. Principal bronchus
B. Lobar bronchi
C. Lobular bronchi
D. Terminal bronchioli

29.39 The deepest part of the costodiaphragmatic recess of the pleural cavity is beside the posterior and
midaxillary lines and here we can acomplish thoracocentesis (a puncture of the pleural cavity) at the level of:
A. VI intercostal space
B. VII intercostal space
C. VIII intercostal space
D. IX intercostal space

29.40 The narrowest part of the uterine tube where the fertilized ovum may lodge is:
A. The ampular part of the uterine tube
B. The infundibular part of the uterine tube
C. The uterine (intramural) part of the uterine tube
D. The isthmus of the uterine tube

29.41 Testosterone a male sex hormone in the testis is produced by:


A. The spermatogene epithelial cells of the convoluted seminiferous tubules of the testis
B. The supporting cells of seminiferous tubules (Sertoli cells)
C. The cells of the wall of the canal of the epididymis (ductus epididymidis)

lsmusis.lsmuni.lt/Klausimai/Spausdinti?Length=0?Kalba=EN&KategorijaId=131&Kalbos_input=EN&Kalbos=EN&KategorijaEn_input=Anatomy&Kateg… 4/23
3/27/2019 LSMUSIS
D. The specific interstitial tissue cells of the testis, located among the seminiferous tubules (Leydig cells)

29.42 During laparatomy the surgeon inserted a finger into the opening of the lesser sac of the peritoneum
(foramen epiploicum) .Firstly a finger enters into:
A. Into the vestibule of the omental bursa
B. Into the cavity of the greater omentum
C. Into the hepatic recess of the peritoneal cavity
D. Into the retroduodenal fatty tissue layer

29.43 The false answer about the nasal part of the pharynx is:
A. It is the widest part of the pharynx
B. Its walls do not oppose (collapse)
C. It has few local accumulations of lymphoid tissue
D. Its mucous membrane is covered with a stratified squamous epithelium

29.44 The following muscles of the pharynx receive their motor innervation from the pharyngeal plexus via the
cranial part of the accessory nerve, except the:
A. Superior constrictor muscle
B. Palatopharyngeus muscle
C. Stylopharyngeus muscle
D. Middle constrictor muscle
E. Salpingopharyngeus muscle

29.45 A patient unable to feel taste on the anterior part of the tongue. Which cranial nerve is likely to have a lesion:
A. Hypoglossal nerve
B. Vagus nerve
C. Glossopharyngeal nerve
D. Facial nerve
E. Maxillary division of trigeminal nerve

29.46 Which cranial nerve is likely to be damaged when in speech the uvula is seen to be drawn upward to the
right?
A. Left glossopharyngeal nerve
B. Right hypoglossal nerve
C. Left vagus nerve
D. Right vagus nerve
E. Right trigeminal nerve

29.47 A skin on the tip of the nose is supplied by the:


A. Zygomatic branch of the facial nerve
B. Maxillary division of the trigeminal nerve
C. Ophthalmic division of the trigeminal nerve
D. Buccal branch of the mandibular division of the trigeminal nerve

29.49 The following statements concerning the chorda tympani are correct, except:
A. It contains parasympathetic postganglionic fibers
B. It contains special sensory (taste) fibers
C. It joins the lingual nerve in the infratemporal fossa
D. It is a branch of the facial nerve in the temporal bone
E. It carries secretomotor fibers to the submandibular and sublingual salivary glands

29.50 Herniation of the intervertebral disc between the fifth and sixth cervical vertebrae will compress the:
A. Fourth cervical nerve root
B. Sixth cervical nerve root
C. Fifth cervical nerve root
D. Seventh and eighth cervical nerve roots
E. Seventh cervical nerve root

29.51 The subarachnoid space in adult ends at the level of:


A. The coccyx
B. The lower border of L1
C. S 2-3

lsmusis.lsmuni.lt/Klausimai/Spausdinti?Length=0?Kalba=EN&KategorijaId=131&Kalbos_input=EN&Kalbos=EN&KategorijaEn_input=Anatomy&Kateg… 5/23
3/27/2019 LSMUSIS
D. S5
E. The promontory of the sacrum

29.52 Correct answer about structures of the ear and organ of Corti:
A. The scala tympani and scala vestibuli are filled with endolymph
B. The spiral organ (organ of Corti) is composed exclusively of sensory neuroepithelial cells
C. The peripheral process of the bipolar neurons of the spiral ganglion ends in the spiral organ and central
process passes as the cochlear nerve
D. The stapes is attached in the round window of the scala tympani

29.53 Bilateral injury of the roots of facial nerves results in:


A. Hyperacusis because of impaired stapedius muscle activity
B. Loss of taste on the posterior one-third of the tongue
C. Loss of the secretion of all salivary glands
D. Ptosis of both eye eyelids.

29.54 Lesion of the efferent tract between the cortex and ventral portion of the facial nucleus causes:
A. Loss of the sense of taste of the anterior side of the tongue
B. Loss of the corneal reflex of the eye
C. Loss of frowning of forehead
D. Paralysis of lower facial muscles

29.55 The majority of the muscles of the pharyx are innervated by cranial nerve:
A. Glossopharyngeal nerve
B. Vagus nerve
C. Accessory nerve
D. Hypoglossal nerve

29.56 Parasympathetic fibers innervating a sphincter pupillae muscle do not pass through the:
A. Short ciliary nerves
B. Oculomotor nerve
C. Long ciliary nerves
D. Cavernous plexus

29.57 The common carotid artery divides:


A. At the level of the cricoid cartilage
B. At the level of the tracheal bifurcation
C. At the upper border of the thyroid cartilage
D. Over the body of the hyoid bone

29.58 Correct statement concerning the heart:


A. The fossa ovalis is on the interatrial septum
B. The atrioventricular groove extends longitudinally in the middle of both ventricles, separating them
C. During ventricular systole due to contraction of the papillary muscles the atrioventricular valves are
opened
D. The valvula of the inferior vena cava encloses aperture of the the vena cava inferior to the atrium
dextrum

29.59 Pulsating of the posterior tibial artery palpable:


A. Behind and below the medial malleolus of the tibia
B. In front of the medial malleolus of the tibia
C. Behind the lateral malleolus of the fibula
D. Between the tendo calcaneus and the lateral malleolus of the fibula

29.60 A collateral circulation of clinical significance between the subclavian and axillary arteries is formed by union
of:
A. The subscapular artery and the suprascapular artery
B. The acromiothoracic artery and the superior thoracic artery
C. The lateral thoracic artery and the transverse cervical artery
D. The costocervical trunk and the lateral thoracic artery

29.61 The profunda brachii artery is accompanied by:


A. The median nerve
lsmusis.lsmuni.lt/Klausimai/Spausdinti?Length=0?Kalba=EN&KategorijaId=131&Kalbos_input=EN&Kalbos=EN&KategorijaEn_input=Anatomy&Kateg… 6/23
3/27/2019 LSMUSIS
B. The ulnar nerve
C. Radialis nerve
D. Musculocutaneus nerve

29.62 The quadrangular space is traversed by:


A. The anterior circumflex humeral artery and the axillary nerve
B. The circumflex scapular artery and the axillary nerve
C. The posterior circumflex humeral artery and the axillary nerve
D. The axillary artery and the axillary nerve

29.63 The sinusoidal (type) capillaries are absent in organ:


A. The bone marrow
B. The spleen
C. The kidneys
D. The liver

29.64 Pulsation of the dorsalis pedis artery appreciable in the foot:


A. On the medial side of the tendon of the tibialis anterior muscle
B. Between the tendon of the tibialis anterior muscle and the tendon of the extensor hallucis longus muscle
C. Between the tendon of the extensor hallucis longus muscle and the tendon of the extensor digitorum
longus muscle
D. Between the tendon of the extensor digitorum longus muscle and tendon of the peroneus tertius muscle

29.65 The posterior group of the thigh muscles supplied by artery:


A. The obturator artery
B. The perforating branches of the profunda femoris artery
C. The lateral circumflex femoral artery and the medial circumflex femoral artery
D. Branches of the superior gluteal artery

29.66 The thoracic duct passes the diaphragm:


A. Through the aortic opening of the diaphragm
B. Through the oesophageal opening of the diaphragm
C. Extending in the space between the costal and lumbar parts of the diaphragm
D. Through the vena cava opening of the diaphragm

29.67 An arterial anastomosis in the greater curvature of the stomach is formed by:
A. The splenic artery and the common hepatic artery branches
B. The splenic artery and the superior mesenteric artery branches
C. The gastroduodenal artery and the superior mesenteric artery branches
D. The proper hepatic artery and the splenic artery branches

29.68 Which arterial branches compose a “corona mortis” in the inguinal region:
A. The arteria epigastrica inferior and the arteria obturatoria
B. The lateral sacral artery and the inferior epigastric artery
C. The inferior gluteal artery and the inferior vesical artery
D. The iliolumbar artery and the superior gluteal artery

29.69 Regarding the extrauterine pregnancy, uterine tube rupture and bleeding happen the following arteries are
ligated:
A. The uterine artery and the ovarian artery
B. The uterine artery and the obturator artery
C. The ovarian artery and the internal pudendal artery
D. The inferior vesical artery and the internal pudendal artery

29.70 In the operation of thyroid gland it is necessary to remember that arterial network of the this gland is formed
of four thyroid arteries. Which artery additionally takes part in forming of this network?
A. The thyroidea ima artery
B. The thyrocervical trunk branches
C. The costocervical trunk branches
D. The inferior laryngeal arteries

29.71 Vertebral bodies are joined by:


A. Ligamenta flava
lsmusis.lsmuni.lt/Klausimai/Spausdinti?Length=0?Kalba=EN&KategorijaId=131&Kalbos_input=EN&Kalbos=EN&KategorijaEn_input=Anatomy&Kateg… 7/23
3/27/2019 LSMUSIS
B. Ligamenta interspinalia
C. Articulationes intervertebrales
D. Ligamentum longitudinale posterius

29.72 Which of the following sutures generally does not persist in adulthood:
A. Sagittal
B. Lambdoid
C. Metopic (frontal)
D. Coronal

29.73 The fontanelles of the skull are composed of:


A. Osseous tissue
B. Cartilaginous tissue
C. Fibrous connective tissue
D. The fontanelles are made of the skin

29.74 The forming new bone after injury is capability of the:


A. Compact bone
B. Spongy bone
C. Epiphyseal plate
D. Periosteum

29.75 The ligament, which prevents hyperextension of the vertebral column:


A. Ligamentum longitudinale posterius
B. Ligamentum supraspinale
C. Ligamentum longitudinale anterius
D. Ligamenta flava

29.76 The maxillary sinus open:


A. Into the nasopharynx
B. Into the upper meatus of the nasal cavity
C. Into the middle meatus of the nasal cavity
D. Into the lower meatus of the nasal cavity

29.77 The pterygopalatine fossa communicte with the middle cranial fossa through:
A. The foramen ovale
B. The foramen spinosum
C. The foramen rotundum
D. The foramen lacerum

29.78 The pterygopalatine fossa communicate with the nasal cavity through:
A. The foramen rotundum
B. The foramen ovale
C. The sphenopalatine foramen
D. The foramen lacerum

29.3 The function of the epiphyseal plate is to:


A. regenerate the bone
B. allow the bone increase in diameter
C. allow the bone increase in length
D. shock absorbtion

29.4 Which type of junctions join the vertebral arches:


A. Intervertebral discs
B. Synovial joints
C. Interosseous membrane
D. Ligamenta flava

29.6 Main components of the synovial joint are:


A. Articular surfaces, joint cavity, meniscs
B. Articular surfaces, articular ligaments, joint capsule
C. Articular surfaces, joint cavity, joint capsule
D. Articular surfaces, joint capsule, joint cavity, ligaments
lsmusis.lsmuni.lt/Klausimai/Spausdinti?Length=0?Kalba=EN&KategorijaId=131&Kalbos_input=EN&Kalbos=EN&KategorijaEn_input=Anatomy&Kateg… 8/23
3/27/2019 LSMUSIS
29.7 Transverse openings are found in:
A. Vertebrae cervicales
B. Vertebrae thoracicae
C. Vertebrae lumbales
D. Vertebrae sacrales

II type tasks. For each numbered item,selct the one lettered heading that is most closely asssciated with it

29.79 Movements of the laryngeal cartilagines and vocal cords are accomplished by:
1 - The aryepiglottic muscle
2 - The posterior cricoarytenoid muscle
3 - The cricothyroid muscle
4 - The vocalis muscle
A. Tenses vocal fold
B. Relaxes vocal fold
C. Closes the inlet of the larynx
D. Widens the rima glottidis, tenses vocal folds

29.80 The uterus and the ovary are fixed in the lesser pelvis:
1 - The broad ligament of the uterus
2 - The round ligament of the uterus
3 - The ligament of the ovary
4 - The suspensory (or infundibulopelvic) ligament of the ovary
A. Fixes the uterus, prevents retroflexion
B. Fixes the uterus, prevents side movements
C. Joins uterus and ovary
D. Fixes the ovary. The ovarial vessels and nerves are included in the ligament

29.81 The indicated fibers of the cranial nerve originates:


1 - Sensory fibers of the facial n.
2 - Parasympathetic fibers of the glossopharyngeal n.
3 - Postganglionic parasympathetic fibers of the oculomotor n.
A. Trigeminal ganglion
B. Otic ganglion
C. Geniculate ganglion
D. Inferior salivatory nucleus
E. Ciliary ganglion

29.82 The …… joint is:


1 - Shoulder joint
2 - Talocrural joint
3 - Radiocarpal joint
4 - Carpometacarpal joint of the thumb
A. Saddle joint
B. Hinge joint
C. Ball- and socket joint
D. Ellipsoid joint

29.83 The main function of the:


1 - The biceps brachii muscle
2 - The pectoralis major muscle
3 - The triceps brachii muscle
4 - The deltoid muscle
A. Abducts the arm from the trunk
B. Flexes the forearm at the elbow joint
C. Extends the forearm at the elbow joint
D. Adduct the arm

29.84 The main function of the:


1 - Quadriceps femoris muscle
2 - The biceps femoris muscle
3 - The pectineus muscle
lsmusis.lsmuni.lt/Klausimai/Spausdinti?Length=0?Kalba=EN&KategorijaId=131&Kalbos_input=EN&Kalbos=EN&KategorijaEn_input=Anatomy&Kateg… 9/23
3/27/2019 LSMUSIS
4 - The iliopsoas muscle
A. Flexes the thigh at the hip joint
B. Extends the leg at the knee joint
C. Flexes the leg at the knee joint
D. Adducts the thigh

29.85 By what kind of junctions are the following parts of the vertebrae joined to each other:
1 - The bodies of the vertebrae
2 - The vertebral arches
3 - The articular proccesses
4 - The sacral vertebrae of the adult man
A. Ligaments
B. Cartilaginous discs
C. Synovial joints
D. Bony fusion

29.86 Which nuclei of the brain stem perform the following functions:
1 - The superior salivatory nucleus
2 - The spinal nucleus of the trigeminal nerve
3 - The nucleus ambiguus
4 - The nucleus of the tractus solitarius
A. Sensation of pain and temperature of the face skin
B. Visceral sensory
C. Somatic motor
D. Parasympathetic secretory

29.87 Join the cranial nerve with it's function:


1 - Hypoglossal nerve
2 - Vagus nerve
3 - Auriculotemporal nerve
4 - Lingual nerve
5 - Glossopharyngeal nerve
A. Provides motor innervation to the intrinsic muscles of the larynx
B. Carries general sensation from the anterior two-thirds of the tongue
C. Is the source of parasympathetic innervation for the parotid gland
D. Provides motor innervation to the intrinsic muscles of the tongue
E. Carries sensory information from baroreceptors in the carotid sinus

29.88 Cranial nerve ……:


1 - N. hypoglosus
2 - N. lingualis
3 - N. tympanicus
A. Is responsible for the sensory innervation of the total mucosa of the tongue
B. Carries preganglionic parasympathetic fibers
C. Carries motor fibers for striated muscles of the tongue
D. Contains general sensation and taste fibers from the anterior two-thirds of the tongue

29.89 To which sensory systems do the indicated neural structures belong:


1 - Somatosensory
2 - Auditory
3 - Visual
4 - Gustatory
A. The geniculate ganglion
B. The brachium of superior colliculus
C. The medial lemniscus
D. The brachium of inferior colliculus and medial geniculate body

29.90 The indicated holes of the skull transmits ……:


1 - Foramen ovale
2 - Foramen spinosum
3 - Foramen rotundum
4 - Foramen lacerum
lsmusis.lsmuni.lt/Klausimai/Spausdinti?Length=0?Kalba=EN&KategorijaId=131&Kalbos_input=EN&Kalbos=EN&KategorijaEn_input=Anatomy&Kate… 10/23
3/27/2019 LSMUSIS
5 - Superior orbital fissure
A. Transmits the ophthalmic nerve
B. Related to the internal carotid artery
C. Transmits the middle meningeal artery
D. Injury to the nerve that passes through this opening leads to a general loss of sensation of the
maxillary teeth
E. Injury to the nerve that passes through this opening leads to a loss of sensation of the
temporomandibular joint

29.91 In which parts of CNS are located the nuclei of the indicated below nerves:
1 - In the spinal cord
2 - In the medulla oblongata
3 - In the pons
4 - In the midbrain
A. The oculomotor and trochlear nerves
B. The abducent and facial nerves
C. The hypoglossal nerve
D. The accessory nerve

29.92 In which parts of the upper and lower extremities are there the following intermuscular canals:
1 - The thigh
2 - The upper arm
3 - The pelvis
4 - Anterior wall of the abdomen
A. The radial canal
B. The adductorius canal
C. The obturator canal
D. The inguinal canal

29.93 Match the statement with the correct nerve below:


1 - Hyperextension of the proximal phalanges of the little and ring fingers (i.e., claw hand) can result from
damage to the … nerve
2 - Wristdrop can result from damage to the … nerve
3 - An inability to oppose the thumb to the little finger can result from damage to the … nerve
A. Ulnar nerve
B. Axillary nerve
C. Radial nerve
D. Median nerve

29.94 Which of the following organs is innervated by:


1 - Lacrimal gland
2 - Muscles of the tongue
3 - Muscles of the larynx
A. Vagus n.
B. Greater petrosal n.
C. Hypoglossal n.
D. Glossopharyngeal n.
E. Lingual n.

29.95 In which of the following joints are the indicated below anatomical units:
1 - The temporomandibular joint
2 - The shoulder joint
3 - The knee joint
4 - The hip joint
A. The articular disk
B. The labrum glenoidalia
C. The cruciate ligaments
D. The ligament of the head of femur

29.96 During the descent of the testis from the abdominal cavity, the testicle had pulled layers of the anterior
abdominal wall and formed coverings of the testis:
1 - The tunica vaginalis testis
lsmusis.lsmuni.lt/Klausimai/Spausdinti?Length=0?Kalba=EN&KategorijaId=131&Kalbos_input=EN&Kalbos=EN&KategorijaEn_input=Anatomy&Kate… 11/23
3/27/2019 LSMUSIS
2 - The cremaster muscle and cremasteric fascia
3 - The internal spermatic fascia
4 - The external spermatic fascia
A. The transversalis fascia
B. The parietal peritoneum
C. The internal obligue muscle and its investing fascia
D. The external obligue aponeurosis

III type tasks. For each question there is one or more correct answers:
A – if correct answers are 1,2,3
B – if correct answers are 1 and 3
C – if correct answers are 2 and 4
D – if correct answer is 4
E – if correct are all answers above

29.97 What is significance of the bones?


1. Protection
2. Movement
3. Hematopoiesis
4. Support

29.98 Which disorders are characteristic for facial nerve injuries?


1. Paralysis of the expression muscles
2. Disorders of hearing
3. Disorders of taste sensation
4. Disorders of secretion of the parotid gland

29.99 Which organs are innervated by postganglionic sympathetic fibres of superior cervical sympathetic
ganglion?
1. M. dilatator pupillae
2. M. sphincter pupillae
3. Glandula parotis
4. M. platysma

29.100 Which statements about white rami communicantes (rr. communicantes albi) are correct?
1. They consist of preganglionic sympathetic fibres
2. They have visceral sensory fibres
3. They contain myelinated fibres
4. Their fibres directly innervate smooth muscles of skin

29.101 Which muscles are innervated by deep peroneal nerve?


1. M. tibialis anterior
2. M. extensor hallucis longus
3. M. extensor digitorum longus
4. M. peroneus tertius

29.102 Which nerves innervates the posterior muscles of the calf and sole?
1. N. peroneus superficialis
2. N. peroneus profundus
3. N. peroneus communis
4. N. tibialis

29.103 Which disorders are characteristic for radial nerve injury inside axillar fossa?
1. It's impossible to extend forearm
2. It's impossible to extend hand
3. Paralysis of triceps muscle of arm
4. Anaesthesia of anterior surface of forearm skin

29.104 Which nerves innervate the muscles of brachium?


1. N. musculocutaneus
2. N. medianus
3. N. radialis
lsmusis.lsmuni.lt/Klausimai/Spausdinti?Length=0?Kalba=EN&KategorijaId=131&Kalbos_input=EN&Kalbos=EN&KategorijaEn_input=Anatomy&Kate… 12/23
3/27/2019 LSMUSIS
4. N. ulnaris

29.105 Which injured nerves cause the disorder of the movements of elbow joint?
1. N. medianus
2. N. radialis
3. N. ulnaris
4. N. musculocutaneus

29.106 Which disorders are characteristic for facial nerve injury at stylomastoid foramen?
1. Paralysis of buccinator muscle
2. Paralysis of temporal muscle
3. Paralysis of platysma muscle
4. Paralysis of levator muscle of upper eyelid

29.107 What comes into orbit through superior orbital fissure?


1. N. oculomotorius
2. A. ophthalmica
3. N. abducens
4. N. opticus

29.108 Which statements about innervation of parotid gland are correct?


1. Secretory fibres of glossopharyngeal nerve innervate parotid gland
2. Facial nerve innervates parotid gland
3. Secretory fibres from otic ganglion innervate parotid gland
4. Sympathetic nerves from internal carotid artery plexus innervate parotid gland

29.109 Which injured nerve causes the disorders of swallowing?


1. N. vagus
2. N. maxillaris
3. N. mandibularis
4. Rr. buccales n. facialis

29.110 Which statements about hypophysis are correct?


1. It is in the same name fossa of the sphenoid bone
2. Cavernous sinus is around it
3. It is covered by sellar diaphragm
4. Decussation of optic nerve is above it and pineal gland is situated anteriorly

29.111 Which statements about cerebellum are correct?


1. It is in the posterior cranial fossa
2. The pons and medulla oblongata are ventrally from it
3. Tentorium cerebelli separates it from the occipital lobe of hemispheres
4. Their function – automatically regulate balance and coordinate movement

29.112 Which nerves form the pharyngeal plexus?


1. N. vagus
2. N. accessorius
3. Truncus sympathicus
4. N. glossopharyngeus

29.113 What goes through foramina of anterior cranial fossa?


1. N. opticus
2. N. maxillaris
3. A. ophthalmica
4. Nn. olfactorii

29.114 Which statements about transverse colon are correct?


1. It is supplied with blood by middle colic artery
2. Venous blood flows into superior mesenteric vein
3. Lymph flows into superior mesenteric lymph nodes
4. It is innervated by nerves from plexus of superior mesenteric artery

29.115 Which statements about ureter are correct?


lsmusis.lsmuni.lt/Klausimai/Spausdinti?Length=0?Kalba=EN&KategorijaId=131&Kalbos_input=EN&Kalbos=EN&KategorijaEn_input=Anatomy&Kate… 13/23
3/27/2019 LSMUSIS
1. It starts from renal pelvis
2. It is about 12 cm in length
3. It is in retroperitoneal space
4. It is a thin-walled, inelastic muscle duct

29.117 Which statements about the relations between liver and structures of peritoneum are correct?
1. Falciform ligament connects liver with anterior abdominal wall
2. Left trianglular ligament (lig. triangulare sin.) connects left lobe of liver with diaphragm
3. In free margin of falciform ligament is ligamentum teres
4. Lesser omentum stretches between the lesser curvature of the stomach and porta hepatis

29.118 Which organs form the spermatic cord?


1. Deferent duct
2. Genitofemoral nerve
3. Pampiniform plexus (plexus pampiniformis)
4. Inguinal ligament

29.119 What forms hepatic triad?


1. Proper hepatic artery
2. Portal vein
3. Common bile duct
4. Hepatic veins

29.120 Which anatomical structures are related to the walls of the omental foramen:
1. Hepatoduodenal ligament
2. Inferior vena cava
3. Superior part of duodenum
4. Pancreas

29.121 Which statements about inguinal canal are correct?


1. It is weak place in abdominal wall
2. In male canal is spermatic cord, in female - round ligament of uterus
3. The walls of canal are formed by muscles of abdominal wall and their aponeuroses
4. It ends in medial surface of hip

29.122 Which statements about sinuses of dura mater are correct?


1. They drain blood from brain
2. The arachnoid granulations of arachnoid protrude into the superior sagittal sinus
3. Sinuses makes anastomoses with veins of face, nape and calvaria
4. The blood from sinuses flows into external jugular vein

29.123 Which sinuses of dura mater anastomose with internal vertebral venous plexus?
1. Sinus occipitalis
2. Sinus petrosus inferior
3. Sinus basilaris
4. Sinus transversus

29.124 Which lymph nodes lymph from scrotum flows into?


1. Into deep inguinal
2. Into internal iliac
3. Into external iliac
4. Into superficial inguinal

29.125 Which lymph nodes lymph from prostate flows into?


1. Into external iliac and sacral
2. Into internal iliac
3. Into deep inguinal
4. Into internal iliac and sacral

29.126 Which lymph nodes lymph from vestibule of vagina flows into?
1. Into deep inguinal
2. Into external iliac
3. Into internal iliac
lsmusis.lsmuni.lt/Klausimai/Spausdinti?Length=0?Kalba=EN&KategorijaId=131&Kalbos_input=EN&Kalbos=EN&KategorijaEn_input=Anatomy&Kate… 14/23
3/27/2019 LSMUSIS
4. Into superficial inguinal

29.127 Which anatomical structures are related to the mediastinal surface of the right lung:
1. Right atrium
2. Thoracic aorta
3. Esophagus
4. Aortic arch

29.128 Which part of the heart is supplied with blood by right coronary artery?
1. Sinuatrial node
2. Right atrium
3. Atrioventricular node
4. Right branch of atrioventricular bundle (bundle of Hiss)

29.129 Where does lymph from gallbladder run to?


1. Into lymph nodes of the pancreas
2. Into pyloric nodes
3. Into pericardial nodes
4. Into lymph nodes of the liver

29.130 Which veins bring blood to inferior vena cava directly?


1. Right testicular vein
2. Right suprarenal vein
3. Hepatic veins
4. Left testicular vein

29.131 Which statements about hepatic veins are correct?


1. Formed by central veins of liver
2. Bring blood from liver
3. Run into inferior vena cava
4. Formed by branches of portal vein

29.132 What movements may be performed at the shoulder joint?


1. Flexion and extension
2. Adduction
3. Abduction
4. Rotation

29.133 In what groups are divided the muscles of the thigh?


1. Anterior
2. Medial
3. Posterior
4. Lateral

29.134 Through which axes movements may be performed at the knee joint?
1. Through all axes
2. Frontal and sagittal
3. Vertical and sagittal
4. Frontal and vertical

29.135 What cervical muscles bound the carotid triangle (trigonum caroticum) of the neck?
1. The anterior margin of the sternocleidomastoid muscle
2. The superior belly of the omohyoid muscle
3. The posterior belly of the digastric muscle
4. The stylohyoid muscle

29.136 Close to which anatomical structures pass female ureter:


1. Round ligament of the uterus
2. Ovarian artery at the pelvic brim
3. Uterine artery 2 cm superior to the ischial spine
4. Uterine tube

29.137 Which statements concerning the topography of the prostate gland are correct?
lsmusis.lsmuni.lt/Klausimai/Spausdinti?Length=0?Kalba=EN&KategorijaId=131&Kalbos_input=EN&Kalbos=EN&KategorijaEn_input=Anatomy&Kate… 15/23
3/27/2019 LSMUSIS
1. The prostate gland is below the neck of the urinary bladder
2. Urethra traverses prostate vertically
3. The apex of the gland rests upon the urogenital diaphragm
4. The prostate gland is located in the subperitonium part of the true pelvis

29.139 Which of the following gyri belong to the limbic system?


1. The superior frontal gyrus
2. The cingulate gyrus
3. The orbital gyri
4. The parahippocampal gyrus

29.140 Neurons of the somatosensory system are located in:


1. The dorsal root ganglions
2. The proper nucleus of the posterior horn of the spinal cord
3. The trigeminal ganglion
4. The mesencephalic nucleus of the trigeminal nerve

29.141 Which neural structures take part in transmission of visceral afferent signals?
1. The ganglions of the glossopharyngeal and vagus nerves
2. The nuclei of solitary tract of the brainstem
3. The dorsal root ganglions
4. The otic ganglion

29.142 Which statements about forming of the portal vein are correct?
1. The portal vein is formed between the oesophagus and pancreas
2. The portal vein is formed at the fusion of the superior mesenteric and splenic veins
3. The portal vein is formed at the site of the confluence of the superior mesenteric and the right renal vein
4. The portal vein is formed behind the head of the pancreas

29.143 The deep palmar arch is located:


1. Under the tendons of the flexor muscles of the forearm and hand
2. Under the palmar aponeurosis
3. Proximally to the superficial palmar arch
4. Beside the palmar cutaneous branch of the median nerve

29.144 The spleen lies:


1. On the level of IX-XI ribs
2. Behind the left kidney
3. Beneath the diaphragm – in the left hypochondrium
4. Beside the duodenum

29.145 Which statements concerning the internal carotid artery are correct?
1. The internal carotid artery ascends through the parapharyngeal space up to the base of the skull
2. In the neck the internal carotid artery makes anastomoses with branches of the vertebral artery
3. The ophthalmic artery branches of from the internal carotid artery
4. The maxillary artery branches of from the internal carotid artery

29.146 Which statements concerning the facial artery are correct?


1. The facial artery branches of from the external carotid artery in the carotid triangle
2. The facial artery passes beside the parotid gland
3. The facial artery pulse may be felt on the angle of mandible
4. The facial artery passes through the retromandibular fossair

29.147 The ureter has narrowings, where urinary calculi (stones) may occlude its lumen:
1. Near the site where the renal pelvis extends into the initial part of the ureter
2. Near the brim of the lesser pelvis
3. Near the wall of the urinary bladder
4. Near the place of crossing over the blood vessels of the kidney (aa. et vv. renales)

29.148 Behind the head of the pancreas is located:


1. The common bile duct (ductus choledochus)
2. The initial part of the portal vein
3. The coeliac trunk (truncus coeliacus)
lsmusis.lsmuni.lt/Klausimai/Spausdinti?Length=0?Kalba=EN&KategorijaId=131&Kalbos_input=EN&Kalbos=EN&KategorijaEn_input=Anatomy&Kate… 16/23
3/27/2019 LSMUSIS
4. The abdominal aorta

29.149 Anatomical constrictions of the oesophagus where foreign bodies can lodge in, are:
1. At the junction of the oesophagus with the pharynx in the neck
2. At the crossing with the left main bronchus
3. Where the oesophagus pierce the diaphragm
4. At the oesophagus passage to the stomach

29.150 Correct statements concerning the sensory nerves of the tongue include:
1. For general sensation, the mucosa of the anterior two-thirds of the tongue is supplied by the lingual nerve
2. The vagus nerve carries taste sensation from a small area of the tongue just anterior to the epiglottis
3. The mucosa of the posterior one-third of the tongue is supplied by the lingual branch of the
glossopharyngeal nerve
4. For special sensation, the anterior two-thirds of the tongue, except for the vallate papillae, are supplied by
the chorda tympani nerve

29.151 Which nerves originate form pterygopalatine ganglion?


1. Nasopalatine nerve
2. Lesser palatine nerve
3. Greater palatine nerve
4. Posterior superior alveolar nerve

29.152 Which of the following nervous structure(s) could be damaged if the patient moves the eyeball normally
and sees distant objects clearly, but cannot focus on near objects?
1. Ciliary ganglion
2. Oculomotor nerve
3. Short ciliary nerves
4. Superior cervical ganglion

29.153 Lesion of the greater petrosal nerve results in:


1. Decreased lacrimal gland secretion
2. Decreased diameter of the pupil
3. Dryness in the nose and palate
4. Decreased parotid gland secretion

29.154 The innervation of the cranial dura mater includes:


1. Vagus nerve
2. Trigeminal nerve
3. Upper cervical spinal nerves
4. Facial nerve

29.155 Which nerve(s) could be damaged if the corneal surface is dry due to decreased secretion of lacrimal
gland?
1. Terminal portion of lacrimal nerve
2. Zygomatic branch of maxillary nerve
3. Greater petrosal nerve
4. Lesser petrosal nerve

29.156 Which of the following structures are components of the auditory pathway?
1. Superior olivary nucleus
2. Inferior colliculus
3. Medial geniculate body
4. Lateral lemniscus

29.157 In which structures would chromatolysis develop, if the facial nerve was sectioned at its emergence from
the brain stem?
1. Solitary nucleus
2. Geniculate ganglion
3. Pterygopalatine ganglion
4. Superior salivatory nucleus

29.158 The integrity of which of the following nerves can be checked by testing the motions of the thumb:
1. Median nerve
lsmusis.lsmuni.lt/Klausimai/Spausdinti?Length=0?Kalba=EN&KategorijaId=131&Kalbos_input=EN&Kalbos=EN&KategorijaEn_input=Anatomy&Kate… 17/23
3/27/2019 LSMUSIS
2. Ulnar nerve
3. Radial nerve
4. Musculocutaneous nerve

29.159 Pelvic splanchnic nerves:


1. Contain preganglionic parasympathetic nerve fibres
2. Contain fibres that innervate the gastrointestinal tract from the splenic flexure distally
3. Contain fibres that arise from sacral spinal cord segments 2, 3 and 4
4. Contain the same type of efferent fibres as the lumbar splanchnic nerves

29.160 Which statements about spinal nerve roots and intervertebral discs are correct?
1. First spinal nerve pass out from the vertebral canal through fissure between atlas and occipital bone
2. Hernia of nucleus pulposus of C5 – C6 intervertebral disc can press the C5 nerve
3. Hernia of nucleus pulposus of L5 – S1 intervertebral disc can press the L5 nerve
4. All nerves roots of the lumbar and sacral segments are called cauda equine

29.161 Neurons located in the following structures indicated below, take part in transmission of the acute
superficial somatic pain (nociceptor) signals:
1. The gelatinous substance of the posterior horn of the spinal cord
2. The spinal nucleus of the trigeminus nerve
3. Tractus spinothalamicus
4. The medial lemniscus

IV type tasks. Choose only one best answer

29.162 A 45-year-old computer programmer with an unremarkable medical history has been experiencing sharp
pains in his hand over the last few months. This pain frequently radiates proximally to his elbow. His physician
observed weakness and incoordination of the thumb, with atrophy of the thenar eminence. Cutaneus sensation to
the thenar eminence was normal. Which of the following nerves is most likely affected?
A. ulnar
B. radial
C. lateral cutaneus nerve of the forearm
D. median
E. musculocutaneous

29.163 At his annual physical examination, Jonson mentioned to the doctor that he had been recently
experiencing difficulty in turning his head to the right. His physical examination was otherwise unremarkable.
Based solely on this information, damage to which of the following nerves might be expected?
A. right spinal accessory
B. right cranial accessory
C. left vagus
D. left spinal accessory
E. left cranial accessory

29.164 A 30-year-old lawyer was involved in a car accident in which he suffered a fracture of the humerus at the
spiral groove. Damage to the nerve that descends in this groove might be expected to cause all of the following
except:
A. weakened supination of the forearm
B. paralysis of anconeus
C. anesthesia over the dorsal ascect of the first three digits
D. wrist drop
E. paralysis of triceps

29.165 A 22-year-old college soccer player received a crushing blow to the lateral aspect of the head of her fibula.
She was brought immediately to the emergency room, where the attending physician observed weakened
dorsiflexion and eversion of her right foot. Which of the following nerves was most likely damaged?
A. tibial
B. common peroneal
C. sciatic
D. deep peroneal
E. superficial peroneal

lsmusis.lsmuni.lt/Klausimai/Spausdinti?Length=0?Kalba=EN&KategorijaId=131&Kalbos_input=EN&Kalbos=EN&KategorijaEn_input=Anatomy&Kate… 18/23
3/27/2019 LSMUSIS
29.166 A 25-year-old medical student had been experiencing numbness and tingling on the medial aspect of his
right hand. When questioned by his physician he mentioned that he had a tendency to study for long stretches of
time, with his elbows resting on the desk, supporting the weight of his head. Which of the following nerves is most
likely compromised in this patient?
A. radial
B. ulnar
C. median
D. musculocutaneous
E. axillary

29.167 A 61 – year old woman had been scheduled for a cholecystectomy. During the operation the scisors of the
surgical resident accidentally entered the tissues immediately posterior to the epiploic (omental) foramen (its
posterior boundary). The surgical field was filled immediately by profuse bleeding. Which of the following vessels
was the most likely source of bleeding?
A. aorta
B. inferior v.cava
C. portal vein
D. right renal artery
E. superior mesenteric artery

29.168 A 57-year-old man is admitted to the emergency department with the pain in his left flank and testicles.
Laboratory tests indicate hematuria and anemia. A CT evidence that blood flow in the left renal vein is being
occluded where it crosses anterior to aorta. Which of the following is the most likely cause of testicular pain?
A. compression of the testicular artery
B. occlusion of blood flow in the testicular vein
C. compression of the afferent fibers in the lumbar splanchnic nerves
D. compression of the sympathetic fibers in the paraortic plexus
E. compression of the posterior vagus nerve

29.169 A 45-year-old woman is admitted to the hospital after her automobile left the highway in the rainstorm and
hit a tree. She had been wearing a seat belt. On radiographic examination, it is observed that she has suffered
fractures of the ninth and tenth rib on her left side and that she has intraabdominal bleeding. Physical examination
reveals hypovolemic shock and progressive hypotension. Which of the following organs is most likely injured to
result in these clinical signs?
A. liver
B. pancreas
C. left kidney
D. spleen
E. ileum

29.170 A 52-year-old male presents to the emergency department complaining of persistent severe right upper
abdominal quadrant pain for the past 2 hours. During that period of time he felt nauseated, was sweating
profusely, and also experienced pain in the posterior aspect of his right shoulder. The pain began shortly after a
lunch consisting of “fast food”. Ultrasound examination reveals multiple stones in an inflamed gallbladder with a
normal bile duct. Which of the following spinal nerve segments are involved in the shoulder pain, associated with
cholecystitis?
A. C3 to C5
B. C5 to C8
C. T1 to T4
D. T5 to T9
E. T10 to T 11

29.171 A 54- year – old man is admitted to the hospital with vomiting and severe weight loss. Physical examination
reveals that the umbilical and epigastric regions are tender and painful. A CT scan examination reveals a massive
tumor originating from the third part of the duodenum. Which of the following structures is more likely to be
compressed or invaded by the tumor?
A. common bile duct
B. portal vein
C. superior mesenteric artery
D. gastroduodenal artery
E. posterior superior pancreaticoduodenal artery

lsmusis.lsmuni.lt/Klausimai/Spausdinti?Length=0?Kalba=EN&KategorijaId=131&Kalbos_input=EN&Kalbos=EN&KategorijaEn_input=Anatomy&Kate… 19/23
3/27/2019 LSMUSIS
29.172 A 65-year-old man is admitted to the emergency department with complaints of nonspecific abdominal
pain. Physical and radiographic examinations reveal mild intestinal ischemia due to atherosclerotic occlusion of the
midproximal part the superior mesenteric artery, but collateral blood supply has delayed the onset of necrosis.
What vessels provide collateral channels between the celiac trunk and superior mesenteric artery?
A. superior and inferior pancreaticoduodenal
B. left gastric and hepatic
C. cystic and gastroduodenal
D. right and felt colic
E. right and left gastroomental

29.173 A 34-year-old man admitted to the emergency department after a traumatic landing into a swimming pool
from a high diving platform. The patient has multiple traumas in the abdominal cavity. After the reconstructive
operation of his abdominal organs the patient develops a high fever. Radiographic examination reveals that the
lower portion of the descending colon and rectum has become septic and must be excised. Six months
postoperatively the patient complains of impotence. Which of the following structures was most likely injured
during the second operation?
A. pudendal nerve
B. sacral splanchnic nerves
C. pelvic splanchnic nerves
D. sympathetic chain (trunk)
E. vagus nerve

29.174 A 19-year-old college student is a member of the school's hockey team. During a recent game he had
been hit forcefully in the right eye by an opponent's stick. Fortunately, no bones were fractured. Soon after the
incident, he began to experience double vision (diplopia) when looking to his right. Which of the following nerves is
MOST likely damaged?
A. oculomotor
B. trigeminal
C. abducent
D. trochlear
E. optic

29.175 A 50-year-old woman is diagnosed with acute closed-angle glaucoma. Her visual acuity decreased
remarkably. Which of the following is the most likely mechanism for this condition?
A. Increased IOP caused by increased aqueous humor production
B. Nonreactive pupil leading to increased intraocular pressure
C. Decreased outflow of the aqueous humor
D. Separation of the retina leading to decreased visual acuity

29.176 A 7-year-old boy with a high fever is brought to the pediatrician. During physical examination the patient
complains of pain in his ear. His throat appears red and inflamed, confirming the diagnosis of pharyngitis. Which of
the following structures provided a pathway for the infection to spread to the tympanic cavity (middle ear)?
A. Choanae
B. Internal acoustic meatus
C. External acoustic meatus
D. Pharyngotympanic tube
E. Pharyngeal recess

29.177 Althought 72-year-old women was unconcerned herself, her daughters inspired mothers examination,
because her personality was changing drastically. For example, she now laughed inappropriately and seemed
utterly unconcerned with things that had previously interested her. After examination neurologist suspected tumor.
If the patient does have a tumor, which part of the brain is MOST likely being compressed by the tumor?
A. parietal lobe
B. occipital lobe
C. frontal lobe
D. temporal lobe
E. insula

29.178 Testing revealed blindness in the right eye; arteriograms of the circle of Willis and cerebral arterial supply
showed a complete occlusion of the right internal carotid artery where it lyies within the cavernous sinus. This is
due to decreased perfusion through wich branch of the internal carotid artery?
A. lenticulostriate
lsmusis.lsmuni.lt/Klausimai/Spausdinti?Length=0?Kalba=EN&KategorijaId=131&Kalbos_input=EN&Kalbos=EN&KategorijaEn_input=Anatomy&Kate… 20/23
3/27/2019 LSMUSIS
B. ophatalmic
C. middle cerebral
D. anterior cerebral
E. posterior communicating

29.179 Testing revealed loss of fine touch and balance sense in both lower extremities. Both pain and temperature
sensations appear to be normal. The loss of touch and balance sence is indicative of damage to the:
A. spinothalamic tracts
B. spinocerebellar tracts
C. posterior columns
D. corticospinal tracts
E. rubrospinal tracts

29.180 Following occlusion of an artery supplying the spinal cord, a patient has damage of spinal cord right side.
In such a patient, one might expect loss of pain and temperature sensation:
A. below and contralateral to the cord lesion
B. below and ipsilateral to the cord lesion
C. above and ipsilateral to the cord lesion
D. below and bilateraly
E. in both upper extremities

29.181 After accident patient was unable to detect painful stimuli applied to his right leg. Although pain and
temperature sensation from his left leg was intact, consciuos proprioceptive and fine touch sensations were lost.
There was observed no sensory loss to the upper limbs. Which of the following is most likely cause of this man's
syndrome?
A. transection of the lower cervical cord
B. left hemisection of lower thoracic spinal cord
C. right hemisection of upper thoracic spinal cord
D. cerebrovascular accident (CVA) of left internal capsule
E. right hemisection of the lower thoracic spinal cord

29.182 A 56-year-old woman presented to her physician with the chief complain of excruciating facial pain that
comes and goes. Triggered by eating, drinking, and face washing, the pain occurs on her nose, upper cheek,
upper lip, and upper theeth. What cranial nerve is involved in produsing the patient's symptoms?
A. facial
B. ophthalmic division of trigeminal
C. maxillary division of trigeminal
D. mandibular division of trigeminal
E. glossopharyngeal

29.183 A 24-year-old male had a third molar (wisdom tooth) extracted from his lower jaw. This resulted in the loss
of general sense and taste sensation from the anterior two thirds of the tongue. This loss was most likely due to
injury of which of the following nerves?
A. Auriculotemporal
B. Major petrosal
C. Lingual
D. Mental
E. Inferior alveolar

29.1 - C 29.2 - A 29.5 - B 29.8 - D 29.9 - B 29.10 - D 29.11 - C 29.12 - E


29.13 - A 29.14 - C 29.15 - B 29.16 - D 29.17 - B 29.18 - D 29.19 - D 29.20 - C
29.21 - B 29.22 - D 29.23 - A 29.24 - D 29.25 - B 29.26 - A 29.27 - D 29.28 - C
29.29 - D 29.30 - D 29.31 - C 29.32 - A 29.33 - D 29.34 - C 29.35 - C 29.36 - B
29.37 - B 29.38 - D 29.39 - C 29.40 - D 29.41 - D 29.42 - A 29.43 - D 29.44 - C
29.45 - D 29.46 - C 29.47 - C 29.49 - A 29.50 - B 29.51 - C 29.52 - C 29.53 - A
29.54 - D 29.55 - B 29.56 - C 29.57 - C 29.58 - A 29.59 - A 29.60 - A 29.61 - C
29.62 - C 29.63 - C 29.64 - C 29.65 - B 29.66 - A 29.67 - A 29.68 - A 29.69 - A
29.70 - A 29.71 - D 29.72 - C 29.73 - C 29.74 - D 29.75 - C 29.76 - C 29.77 - C
29.78 - C 29.3 - C 29.4 - D 29.6 - C 29.7 - A

lsmusis.lsmuni.lt/Klausimai/Spausdinti?Length=0?Kalba=EN&KategorijaId=131&Kalbos_input=EN&Kalbos=EN&KategorijaEn_input=Anatomy&Kate… 21/23
3/27/2019 LSMUSIS
29.79 29.80 29.81 29.82 29.83 29.84 29.85 29.86
1-C 1-B 1-C 1-C 1-B 1-B 1-B 1-D
2-D 2-A 2-D 2-B 2-D 2-C 2-A 2-A
3-A 3-C 3-E 3-D 3-C 3-D 3-C 3-C
4-B 4-D 4-A 4-A 4-A 4-D 4-B
29.87 29.88 29.89 29.90 29.91 29.92 29.93 29.94
1-D 1-C 1-C 1-E 1-D 1-B 1-A 1-B
2-A 2-D 2-D 2-C 2-C 2-A 2-C 2-C
3-C 3-B 3-B 3-D 3-B 3-C 3-D 3-A
4-B 4-A 4-B 4-A 4-D
5-E 5-A
29.95 29.96
1-A 1-B
2-B 2-C
3-C 3-A
4-D 4-D
29.97 29.98 29.99 29.100 29.101 29.102 29.103 29.104
1 1 1 1 1 4 1 1
2 2 3 2 2 2 3
3 3 3 3 3
4 4
29.105 29.106 29.107 29.108 29.109 29.110 29.111 29.112
2 1 1 1 1 1 1 1
4 3 3 3 3 2 2 2
3 3 3
4 4
29.113 29.114 29.115 29.117 29.118 29.119 29.120 29.121
4 1 1 1 1 1 1 1
2 3 2 2 2 2 2
3 3 3 3 3 3
4 4
29.122 29.123 29.124 29.125 29.126 29.127 29.128 29.129
1 1 4 4 4 1 1 4
2 3 3 2
3 3
29.130 29.131 29.132 29.133 29.134 29.135 29.136 29.137
1 1 1 1 4 1 1 1
2 2 2 2 2 2 2
3 3 3 3 3 3 3
4 4
29.139 29.140 29.141 29.142 29.143 29.144 29.145 29.146
2 1 1 2 1 1 1 1
4 2 2 4 3 3 3 2
3 3 3
4
29.147 29.148 29.149 29.150 29.151 29.152 29.153 29.154
1 2 1 1 1 1 1 1
2 4 2 2 2 3 3 2
3 3 3 3 3
4
29.155 29.156 29.157 29.158 29.159 29.160 29.161
1 1 2 1 1 1 1
2 2 4 2 2 3 2
3 3 3 3 3
4 4
29.162 - D 29.163 - D 29.164 - E 29.165 - B 29.166 - B 29.167 - B 29.168 - B 29.169 - D
29.170 - D 29.171 - C 29.172 - A 29.173 - C 29.174 - C 29.175 - C 29.176 - D 29.177 - C
29.178 - B 29.179 - C 29.180 - A 29.181 - B 29.182 - C 29.183 - C
lsmusis.lsmuni.lt/Klausimai/Spausdinti?Length=0?Kalba=EN&KategorijaId=131&Kalbos_input=EN&Kalbos=EN&KategorijaEn_input=Anatomy&Kate… 22/23
3/27/2019 LSMUSIS

lsmusis.lsmuni.lt/Klausimai/Spausdinti?Length=0?Kalba=EN&KategorijaId=131&Kalbos_input=EN&Kalbos=EN&KategorijaEn_input=Anatomy&Kate… 23/23
3/27/2019 LSMUSIS

Histology
I type tasks. Choose only one best answer

4.1 The blood-brain barrier is thought to occur because capillaries in the CNS have which of the following
characteristics:
A. Discontinuous basal lamina
B. Fenestrae with diaphragms
C. Fenestrae without diaphragms
D. Endothelial cells have only few pinocytic vesicles
E. No basement membrane

4.2 Which structure is the primary inductor of the nerve system development
A. Somites
B. Notochord
C. Lens

4.3 Which of the following substances can be absorbed directly by the surface lining cells of the stomach
A. Vitamin B₁₂
B. Polysacharides
C. Triglycerides
D. Alcohol

4.4 How early can the sex of the fetus be clearly determined from the appearance of his (her) external genitalia:
A. 7 weeks
B. 10-12 weeks

4.5 Which of the following statements concerning liver sinusoids is true:


A. They are continuous with bile canaliculi
B. They are surrounded by a well-developed basal lamina
C. They are lined by nonfenestrated endothelial cells
D. They deliver blood to the central vein
E. They deliver blood to the portal vein

4.6 Which statement about amniotic epithelium is correct:


A. Amniotic epithelium is a component of placental blood barrier
B. Amniotic epithelium is responsible for secretion of amnion fluid
C. Amniotic epithelium is derived from endoderm

4.7 Which one of the following features distinguishes the male urethra from the female urethra:
A. Pseudostratified columnar epithelium
B. Male urethra is appr. 5 times longer
C. Glands
D. External sphincter of skeletal muscle
E. Connective tissue layer underlying the epithelium

4.8 The striated heart muscle has following specialized cells:


A. Contractile cardiocytes
B. Myoendocrine cardiocytes
C. Nodal cardiocytes
D. All aforementioned

4.9 In which of the following sites do T lymphocytes become immunocompetent:


A. Germinal center of secondary lymphoid nodules
B. White pulp of the spleen
C. Thymus cortex
D. Red pulp of the spleen
E. Paracortex of lymph node

4.10 The most characteristic feature of the maternal surface of the placenta is:
A. Attachment of the umbilical cord
B. Amniotic epithelium
C. Cotyledons
lsmusis.lsmuni.lt/Klausimai/Spausdinti?Length=0?Kalba=EN&KategorijaId=106&Kalbos_input=EN&Kalbos=EN&KategorijaEn_input=Histology&Kate… 1/17
3/27/2019 LSMUSIS
4.11 An examination of the placenta and fetal membranes of male twins revealed two amnions, two chorions and
fused placentas. Most probably the twinning resulted from:
A. Dispermy
B. Fertilization of two ova
C. Fertilization of one ovum

4.12 Prolactin is synthesized and secreted by which of the following cells:


A. Acidophils in the pars distalis
B. Basophils in the pars tuberalis
C. Somatotrophs in the pars distalis
D. Basophils in the pars intermedia

4.13 By which of the following cells adrenocorticotropic hormone (ACTH) is produced:


A. Basophils in the pars distalis
B. Neurosecretory cells in the median eminence
C. Acidophils in the pars distalis
D. Neurons of the paraventricular nucleus in the hypothalamus
E. None of the above

4.14 Which of the following statements concerning the vaginal mucosa is true:
A. It is lined by stratified columnar epithelium
B. It is lined by stratified squamous keratinized epithelium
C. It is lubricated by glands located in the cervix
D. Its cells secrete lactic acid

4.15 Aqueous humor is drained from the eye's anterior chamber by passing:
A. Into the ciliary processes
B. From the anterior chamber into the posterior chamber
C. Through the canal of Schlemm (sinus venosus sclerae)
D. Into the vitreous body

4.16 Stratified squamous keratinized epithelium is always present in the:


A. Rectum
B. Esophagus
C. Pyloric part of stomach
D. Jejunum
E. Anus

4.17 Which of the following epidermal layers is a unique feature of thick skin:
A. Stratum basale
B. Stratum corneum
C. Stratum lucidum
D. Stratum spinosum

4.18 In the 5th month of the embryogenesis mitotic activity of the mesenchymal cells is blocked. Which structures
of the bronchial wall fail to develop?
A. Glands of the bronchi
B. Bronchial nerves
C. Epithelium of the bronchi
D. Muscles, connective tissue and cartilages

4.19 Which family is considered to be barren (sterile) i.e. as unable to have children:
A. That family which failed to conceive in spite of unprotected intimate sexual contacts in the course of 1
year
B. That family which failed to conceive in spite of unprotected intimate sexual contacts in the course of 5
years

4.20 An ovarian follicle, composed of a primary oocyte, a layer of cuboidal epithelial cells and a basement
membrane, is:
A. Primordial follicle
B. Tertiary follicle
C. Primary follicle

lsmusis.lsmuni.lt/Klausimai/Spausdinti?Length=0?Kalba=EN&KategorijaId=106&Kalbos_input=EN&Kalbos=EN&KategorijaEn_input=Histology&Kate… 2/17
3/27/2019 LSMUSIS
D. Attretic follicle

4.21 Smooth endoplasmic reticulum is involved in synthesis of:


A. ATP
B. Proteins
C. Steroid hormones

4.22 Spermatozoa with morphological abnormalities may cause:


A. Monosomy
B. No malformations
C. Trisomy
D. Klinefelter syndrome

4.23 What cells are alveolar epithelial cells in lungs:


A. Type I and type II respiratory cells
B. Endocrine cells
C. Macrophages
D. Goblet cells

4.24 Which organelles are abundant in cells with a high rate of energy metabolism:
A. Endoplasmic reticulum
B. Mitochondria
C. Centrioles
D. Lysosomes

4.25 What essential process takes place during fertilization:


A. Spermatozoa are ejaculated into the vagina
B. Spermatozoon gets fused with an oocyte
C. Spermatozoon is injected into oocyte

4.26 Which cells of the stomach mucosa secrete hydrochloric acid:


A. Argentaffin cells
B. Mucous cells
C. Parietal cells

4.27 Which cells produce cementum of the teeth:


A. Odontoblasts
B. Cementocytes
C. Cementoblasts

4.28 Dental surface is covered by the hardest organic substance:


A. Enamel
B. Dentin
C. Cementum

4.29 Which of the tongue structures are covered with white coating in some diseases of gastrointestinal tract:
A. Foliate papillae
B. Keratinized layer of the filiform papillae

4.30 Why the production of antibodies decreases after the removal of the thymus gland in a newborn animal:
A. T lymphocytes responsible for the cellular immunity are not produced
B. Production of T-killers is impaired
C. Production of T-helpers is impaired

4.31 Which cells are responsible for the destruction of transplanted tissue, and what is the source of their
development:
A. Macrophages of the spleen
B. T-lymphocytes-supressors of the thymus
C. T-lymphocytes-helpers of the spleen
D. T-lymphocytes-killers of the thymus
E. Macrophages in the place of transplantation

4.32 What processes take place during ovulation:


lsmusis.lsmuni.lt/Klausimai/Spausdinti?Length=0?Kalba=EN&KategorijaId=106&Kalbos_input=EN&Kalbos=EN&KategorijaEn_input=Histology&Kate… 3/17
3/27/2019 LSMUSIS
A. Mature ovum is discharged from the cortex of the ovary
B. Ovum enters the uterus
C. If sperm is present, to ovum meets with a spermatozoon

4.33 Which part of hemopoesis is disturbed because of diminished pH in the stomach and decreased iron
resorption:
A. Blood platetes formation
B. Monocytes formation
C. Erythrocytes formation

4.34 What is vein valves' structure:


A. Endothelial, connective tissue and smooth muscle cells
B. Endothelial cells, connective tissue fibers

4.35 Where in the spleen erythrocytes are destroyed:


A. In the white pulp
B. In the region of central artery
C. In the marginal zone
D. In red pulp

4.36 Erythrocytes, granulocytes, monocytes, blood platelets maturation and development is impaired. Which organ
is damaged?
A. Spleen
B. Liver
C. Bone marrow

4.37 Blood formed elements participating in blood cloting:


A. Blood platelets
B. Erythrocytes (red blood cells)
C. Leukocytes (white blood cells)
D. Monocytes

4.38 Why after the use of streptomycin the hearing of the patient may be impaired:
A. Acetylcholine synthesis is blocked in spiral ganglion
B. Endolymph circulation is impaired
C. Perilymph circulation is impaired
D. Sensory hair cells are damaged

4.39 Which of the following cells line more than 90 percent of alveolar septum:
A. Type II alveolar cells
B. Endothelial cells
C. Fibroblasts
D. Type I alveolar cells
E. Macrophages

4.40 Which of the following cell types is present in the pyloric glands of the stomach:
A. Goblet cells
B. Mucous cells
C. Paneth cells
D. Basal cells
E. Chief cells

4.41 Which structures are located in the space of Disse:


A. Hepatocytes
B. Kupffer cells
C. Sinusoidal capillaries
D. Plates of hepatocytes

4.42 Which cells produce bile:


A. Hepatocytes
B. Kupffer cells
C. Endothelial cells of sinusoidal capillaries
D. Epithelial cells of bile canaliculi
lsmusis.lsmuni.lt/Klausimai/Spausdinti?Length=0?Kalba=EN&KategorijaId=106&Kalbos_input=EN&Kalbos=EN&KategorijaEn_input=Histology&Kate… 4/17
3/27/2019 LSMUSIS
4.43 Acinar cells of the exocrine pancreas produce:
A. Glucagon
B. Insulin
C. Plasma proteins
D. Proteases

4.44 Which cells are not present in tubular glands of the stomach
A. Parietal cells
B. Chief cells
C. Endocrinocytes
D. Paneth cells
E. Mucous neck cells

4.45 Which of the following organelles lack a membranous component:


A. Vacuoles
B. Lysosomes
C. Mitochondria
D. Ribosomes
E. Smooth endoplasmic reticulum

4.46 Programmed cell death is also known as:


A. Hypertrophy
B. Hyperplasia
C. Necrosis
D. Apoptosis
E. Dysplasia

4.47 Cells containing abundant lysosomes are likely to be specialized for:


A. Contraction
B. Exitability
C. Secretion
D. Absorbtion
E. Phagocytosis

4.48 Which of the following is not a component of the extracellular matrix:


A. Tissue fluid
B. Collagen fibers
C. Glycosaminoglycans
D. Antibodies
E. Plasma cells

4.49 In which of the following covering epithelia do all cells touch the basement membrane but not all cells reach
or border the lumen:
A. Transitional
B. Stratified squamous non-keratinized
C. Stratified squamous keratinized
D. Pseudostratified columnar
E. Stratified cuboidal, as in the duct of a sweat gland

4.50 Dense regular connective tissue is present in:


A. Capsules of organs
B. Basement membrane
C. Tendons
D. Skin
E. None of the above

4.51 Which one of the following statements regarding hyaline cartilage is true:
A. It is vascular
B. It contains type IV collagen
C. It undergoes appositional growth only
D. It is located at the articular ends of long bones
E. None of the above

lsmusis.lsmuni.lt/Klausimai/Spausdinti?Length=0?Kalba=EN&KategorijaId=106&Kalbos_input=EN&Kalbos=EN&KategorijaEn_input=Histology&Kate… 5/17
3/27/2019 LSMUSIS
4.52 Which of the following cells arise from monocytes:
A. Plasma cells
B. Fibroblasts
C. Lymphocytes
D. Macrophages
E. None of the above

4.53 Circulating antibodies are synthesized by which of the following cells:


A. T memory cells
B. B memory cells
C. T helper cells
D. Plasma cells
E. T cytotoxic cells

4.54 Which of the following cells are under voluntary control:


A. Skeletal muscle cells
B. Smooth muscle cells
C. Visceral smooth muscle cells
D. Cardiac muscle cells
E. Myoepithelial cells

4.55 The endomysium is a connective tissue investment that surrounds:


A. Individual muscle fibers
B. Muscle fascicles
C. Individual myofibrils
D. An entire muscle

4.56 Which of the following statements concerning cardiac muscle cells is true:
A. They are spindle shaped
B. They require an external stimulus to undergo contraction
C. They are multinuclear cells
D. They are joined together end-to-end by intercalated disks
E. None of the above

4.57 The pulmonary trunk is a/an:


A. Elastic artery
B. Muscular artery
C. Arteriole
D. Metarteriole
E. Capillary

4.58 Which of the following cells is known to be phagocytic:


A. Oligodendrocyte
B. Microglia
C. Fibrous astrocyte
D. Protoplasmic astrocyte
E. Ependyma

4.59 Which of the following cell types is responsible for myelinating peripheral nerves:
A. Purkinje cells
B. Fibrous astrocytes
C. Schwann cells (neurolemmocytes)
D. Oligodendrocytes
E. Protoplasmic astrocytes

4.60 Which of the following lines the central canal of the spinal cord:
A. Oligodendrocytes
B. Fibrous astrocytes
C. Protoplasmic astrocyte
D. Microglia
E. Ependymal cells

lsmusis.lsmuni.lt/Klausimai/Spausdinti?Length=0?Kalba=EN&KategorijaId=106&Kalbos_input=EN&Kalbos=EN&KategorijaEn_input=Histology&Kate… 6/17
3/27/2019 LSMUSIS
4.61 Which of the following is found in the lumen of the auditory tube:
A. Endolymph
B. Perilymph
C. Air
D. Both endolymph and perilymph
E. Cerebrospinal fluid

4.62 The parafollicular cells (C-cells) of thyroid gland:


A. Possess an iodine pump in their plasma membrane
B. Secrete thyroglobulin into the thyroid follicles
C. Remove thyroglobulin from the thyroid follicles
D. Secrete tri- or tetraiodthyronine into adjacent perivascular structures
E. None of the above

4.63 The zona fasciculata of the adrenal cortex synthesizes and secretes:
A. Mineralocorticoids
B. Glucagon
C. Epinephrine
D. Aldosterone
E. None of the above

4.64 Which of the following statements concerning adrenal parenchymal cells is true:
A. Those of the zona glomerulosa produce androgens
B. Those of the adrenal medulla produce epinephrine and norepinephrine
C. Those of the zona glomerulosa produce glucocorticoids
D. Those of the cortex contain numerous secretory granules
E. All of the above

4.65 Pituitary basophils are believed to produce all of the following hormones EXCEPT:
A. Follicle stimulating hormone
B. Luteinizing hormone
C. Prolactin
D. Adrenocorticotropic hormone
E. Melanocytes stimulating hormone

4.66 Which of the following statements concerning the pancreas is NOT true:
A. It possesses islets of Langerhans
B. It possesses serous demilunes
C. Its acinar cells contain a round, basally located nucleus
D. It contains more beta cells then alpha cells
E. All of the above

4.67 The ileum includes which of the following structures:


A. Peyer’s patches
B. Brunner’s glands
C. Parietal cells
D. Chief cells
E. All of the above

4.68 Pepsinogen does not become a functional enzyme until exposed to the secretions of the:
A. Parietal cells
B. Chief cells
C. Mucous neck cells
D. Paneth cells
E. None of the above

4.69 Which of the following statements concerning the gallbladder is true:


A. It synthesizes bile
B. Mucosa is lined by a simple columnar epithelium
C. Bile leaves the gallbladder via the common bile duct
D. It has no muscle cells in the walls
E. It is affected by the hormone secretin

lsmusis.lsmuni.lt/Klausimai/Spausdinti?Length=0?Kalba=EN&KategorijaId=106&Kalbos_input=EN&Kalbos=EN&KategorijaEn_input=Histology&Kate… 7/17
3/27/2019 LSMUSIS
4.70 Alveoli are present in all of the following parts of the respiratory tract except:
A. Alveolar sacs
B. Respiratory bronchioles
C. Alveolar ducts
D. Terminal bronchioles
E. Respiratory bronchioles and alveolar sacs

4.71 Terminal bronchioles have which one of the following components:


A. Seromucous glands in their submucosa
B. An epithelium containing goblet cells
C. Clara cells within their lining epithelium
D. Cartilaginous plates
E. All of the above

4.72 Alveoli in alveolar sacs possess all of the following components except:
A. Elastic fibers in their walls
B. A simple squamous lining epithelium
C. Reticular fibers in their walls
D. Smooth muscle in their walls
E. Pneumocytes

4.73 The great alveolar cells (type II pneumocytes):


A. Are also know as pulmonary alveolar macrophages
B. Secrete surfactant
C. Are chemoreceptor cells (sensitive to O₂ levels of inspired air)
D. Are baroreceptor cells (sensitive to pulmonary blood pressure levels)
E. None of the above

4.74 Which one of the following statements concerning the mammary gland is not true:
A. It contains lactiferous ducts
B. It produces and secretes colostrum (pre-milk)
C. It is identical in males and females prior to puberty
D. It contains myoepithelial cells
E. It forms alveoli during proliferative phase of menstrual cycle

4.75 A nephron includes all of the following components except:


A. A renal corpuscle
B. A distal convoluted tubule
C. A thin limb of the loop of Henle
D. A collecting tubule
E. Pars recta of the proximal tubule

4.76 The visceral layer of Bowman’s capsule is formed by:


A. Kupffer cells
B. Capillary endothelium
C. Transitional epithelium
D. Podocytes
E. Cap cells

4.77 The enzyme renin is produced by the:


A. Juxtaglomerular cells
B. Cells of the macula densa
C. Mesangial cells
D. Glomerular endothelial cells
E. Podocytes

4.78 The principle storage site for spermatozoa is thought to be the:


A. Epididymis duct
B. Ductuli efferentes
C. Proximal segment of the vas deferens
D. Seminal vesicles
E. None of the above

lsmusis.lsmuni.lt/Klausimai/Spausdinti?Length=0?Kalba=EN&KategorijaId=106&Kalbos_input=EN&Kalbos=EN&KategorijaEn_input=Histology&Kate… 8/17
3/27/2019 LSMUSIS
4.79 Testosterone is produced by which of the following:
A. Interstitial cells of Leydig
B. Sertoli cells
C. Spermatogonia
D. Spermatids
E. Spermatocytes

4.80 Which one of the following changes occurs during the proliferative phase of the menstrual cycle in the
functional layer of the endometrium:
A. Blood vessels become ischemic
B. Elongation of glandular tubes
C. The stroma swells due to edema
D. Glands become coiled
E. All of the above

4.81 The endometrial layer(s) lost during menstruation and parturition is(are) the:
A. Stratum basale only
B. Stratum spongiosum only
C. Stratum compactum only
D. Stratum compactum and stratum spongiosum
E. Stratum spongiosum and stratum basale

4.82 Which of the following statements concerning the corpus luteum is true?
A. It produces luteinizing hormone
B. It produces follicle stimulating hormone
C. It derives its granulosa luteal cells from the theca externa
D. It becomes the corpus albicans
E. All of the above

4.83 The epicardium is one of the three layers of the heart. It is:
A. Continous with the endocardium
B. Also know as the visceral pericardium
C. Composed of modified cardiac muscle cells
D. Capable of increasing intraventricular pressure
E. Capable of decreasing intraventricular pressure

4.84 Which one of the following characteristics distinguishes somatic capillaries from visceral capillaries:
A. Presense or absense of fenestrae in endothelium
B. Size of the lumen
C. Thickness of the vessel wall
D. Presence or absence of pericytes
E. Thickness of the basal lamina

4.85 Vasa vasorum provide a functional analogue to that of:


A. Valves
B. Basal lamina
C. Coronary arteries
D. Endothelial diaphragms
E. Arterioles

4.86 The blood cell that is a precursor to microglia, Kupffer cells and Langerhan’s cells is:
A. A basophil
B. An eosinophil
C. A neutrophil
D. A monocyte
E. A lymphocyte

4.87 Which of the following cells possess specific granules, visible in light microscopy:
A. Monocytes
B. Lymphocytes
C. Myeloblasts
D. Promyelocytes

lsmusis.lsmuni.lt/Klausimai/Spausdinti?Length=0?Kalba=EN&KategorijaId=106&Kalbos_input=EN&Kalbos=EN&KategorijaEn_input=Histology&Kate… 9/17
3/27/2019 LSMUSIS
E. Basophilic granulocytes

4.88 The function of microvilli is:


A. Extensive movement of substances over cell surfaces
B. Increase in surface area for absorbtion
C. Cellular movement
D. Transport of intracellular organelles through the cytoplasm
E. Transport of intracellular macromolecules

4.89 A cell in exocrine glands uses several mechanisms of secretion. Which one of following terms does not refer
to a mechanism of secretion:
A. Apocrine
B. Paracrine
C. Merocrine
D. Holocrine

4.90 Which of the following is a function of the basement membrane:


A. Molecular filtering
B. Contractility
C. Exitability
D. Modification of secreted protein
E. Active transport

4.91 Which of the following contributes to epithelial cells' polarity:


A. Intercellular junctions
B. Targered delivery of proteins from the of Golgi complex
C. Membrane fluidity
D. Sorting of domain-specific proteins in the endoplasmic reticulum

4.92 Which one of following statements about epithelia is not true:


A. They are polarized
B. They are vascularized
C. They are separated from underlying connective tissue by a basal lamina
D. They contain only a small amount of intercellular substance
E. They line the lumen of blood vesels

4.93 A medical student who has chronic lower respiratory infections seeks the advice of an ear, nose and throat
specialist. A biopsy of the student’s respiratory epithelium reveals alterations in certain epithelial structures. This
patient is most likely to have abnormal:
A. Microvilli
B. Desmosomes
C. Cilia
D. Hemidesmosomes
E. Basal plasmalemma infoldings

4.94 Which one of the following statements about glands is true:


A. Exocrine glands lack ducts
B. Simple glands have ducts that branch
C. Endocrine glands secrete into ducts
D. Merocrine glands secrete by exocytosis

4.95 Which of the following cells in the inner ear are involved in detecting linear movements of the head:
A. Hair cells in the maculae
B. Outer pillar cells
C. Inner pillar cells
D. Cells of Hensen
E. Hair cells in the organ of Corti

4.96 Rods and cones form synapses with which of the following cells:
A. Amacrine
B. Bipolar
C. Ganglion cells

lsmusis.lsmuni.lt/Klausimai/Spausdinti?Length=0?Kalba=EN&KategorijaId=106&Kalbos_input=EN&Kalbos=EN&KategorijaEn_input=Histology&Kat… 10/17
3/27/2019 LSMUSIS
D. Muller cells

4.97 The retinal pigmented epithelium is characterized by:


A. Containment of the photoreceptor cells
B. Phagocytosis of worn-out components of the photoreceptor cells
C. Origin from the inner layer of the optic cup
D. Presence of rhodopsin
E. Synthesis of vitreous humor

4.98 Which set of nerve cells' layers is characteristic of cerebral cortex:


A. Molecular layer, external granular, Purkinje nerve cells' layer, internal granular layer
B. Molecular layer, Purkinje nerve cells' layer, bipolar and amacrine cells' layers
C. Molecular layer, external and internal granular, external and internal pyramidal, multiform nerve cells'
layer
D. Molecular layer, ganglionic nerve cells' layer, pseudounipolar and bipolar nerve cells' layers
E. Molecular layer, neuroepithelial, ependymal

4.99 The axon hillock contains:


A. Rough endoplasmic reticulum
B. Ribosomes
C. Microtubules
D. Golgi complex
E. Synaptic vesicles

4.100 Autonomic ganglia differ from dorsal root ganglia by:


A. Only parasympathetic ganglion possesses a connective tissue investment
B. Mitotic activity of nerve cells
C. Number of processes, that nerve cells possess
E. Only dorsal root ganglia possess neuroglia cells

4.101 Following their maturation in the thymus and release into circulation, T lymphocytes migrate preferentially to
which of the following sites:
A. Paracortex of lymph nodes
B. Cortical lymphoid nodules of lymph nodes
C. Hilum of lymph nodes
D. Lymphoid nodules of the tonsils
E. Lymphoid nodules in the spleen

4.102 Which one of the following does not act as a neurotransmitter:


A. Acetylcholine
B. Endorphins
C. Epinephrine
D. Tyrosine
E. Glutamic acid

4.103 Each smooth muscle cell


A. Has triads associated with its contraction
B. Has diads associated with its contraction
C. Possesses a single, centrally located nucleus
D. Is characterized by the absence of cytoplasmic vesicles

4.104 The nodes of Ranvier:


A. Occur only in the CNS
B. Contain Ca²⁺-gated channels
C. Represent the midpoints of myelination segments
D. Are completely covered by myelin
E. Lie between consecutive Schwann cells

4.105 Which of the following statements concerning transverse (T) tubules is true:
A. They are present in smooth muscle
B. They are extensions of sarcolemma
C. They are extensions of the sarcoplasmic reticulum

lsmusis.lsmuni.lt/Klausimai/Spausdinti?Length=0?Kalba=EN&KategorijaId=106&Kalbos_input=EN&Kalbos=EN&KategorijaEn_input=Histology&Kat… 11/17
3/27/2019 LSMUSIS
D. They form part of the diads present in skeletal muscle
E. They serve to sequester calcium ions

4.106 Triad of skeletal muscle is composed of:


A. Two T tubules, laterally surrounded by one terminal cistern of sarcoplasmic reticulum
B. One T tubule, laterally surrounded by two terminal cisterns of sarcoplasmic reticulum
C. One T tubule, surrounded by one terminal cistern of sarcoplasmic reticulum
D. Two T tubules, surrounded by two fenestrated cisterns of sarcoplasmic reticulum
E. There is no correct answer

4.107 Which of the following statements about muscle is true:


A. Adult cardiomyocytes retain the capacity for cell division
B. The mechanism of contraction of skeletal muscle closely resembles non-muscle cell motility
C. Smooth muscle contraction is independent of actin myosin interaction
D. Smooth muscle cells are multinuclear
E. Differentiated smooth muscle cells retain the ability to undergo cell division

4.108 Which of the following statements about cardiac muscle is true:


A. It is capable of extensive regeneration by proliferation
B. Atrial cardiomyocytes have endocrine function as well
C. Gap junctions are absent
D. Satellite cells are prominent
E. Intercalated disks contain zonulae occludentes (tight junctions)

4.109 After birth, growth in the length of long bones occurs through:
A. Appositional growth in the periosteum
B. The growth in the primary ossification center
C. The growth in the secondary ossification center
D. Interstitial growth in the diaphysis
E. Proliferation and hypertrophy of chondrocytes in the epiphyseal plate

4.110 Of the following cell types found in connective tissue, which one is the most often present along capillaries
and resembles fibroblasts:
A. Plasma cells
B. Lymphocytes
C. Macrophages
D. Pericytes
E. Monocytes

4.111 Precursors of collagen are secreted by:


A. Plasma cells
B. Stem cells
C. Fibroblasts
D. Lymphocytes
E. Monocytes

4.112 Giant phagocytic cells are formed by the coalescence of:


A. Macrophages
B. Lymphocytes
C. Fibroblasts
D. Adipose cells

4.113 Which of the following statements regarding osteoclasts is true:


A. They are non-nucleated cells
B. They produce collagen
C. They occupy Howships lacunae
D. They are derived from osteoprogenitor cells

4.114 Intima of muscular type artery contains:


A. Myelinated nerve fibers
B. Striated muscle fibers
C. Chondrocytes

lsmusis.lsmuni.lt/Klausimai/Spausdinti?Length=0?Kalba=EN&KategorijaId=106&Kalbos_input=EN&Kalbos=EN&KategorijaEn_input=Histology&Kat… 12/17
3/27/2019 LSMUSIS
D. Dense regularly arranged collagenous connective tissue
E. Endothelium

4.115 Which of the elements is vital for mineralization of bone tissue:


A. Potassium
B. Calcium
C. Iron
D. Sodium

4.116 Plasma cells are derived from:


A. Neural crest
B. B-lymphocytes
C. T lymphocytes
D. Monocytes
E. Neutrophils

4.117 The principal glycoprotein with which collagen type IV interacts is:
A. Fibronectin
B. Laminin
C. Entactin
D. Heparan sulfate
E. Dermatan sulfate

4.118 Vitamin C deficiency results in:


A. Excessive callus formation in healing fractures
B. Decreased breakdown of collagen
C. Formation of unstable collagen helices
D. Stimulation of prolyl hydroxylase
E. Establishment of multiple organs' fibrosis

4.119 Which of the following terms describes the developmental potential of gastrointestinal stem cells, that give
rise to secretory lineage - Paneth cells, goblet cells and enterocytes?
A. Embryonic
B. Metaplastic
C. Multipotent
D. Nullipotent

4.120 Cholesterol function in the plasmalemma is to:


A. Increase fluidity of the lipid bilayer
B. Decrease fluidity of the lipid bilayer
C. Facilitate the diffusion of ions through the lipid bilayer
D. Assist in the transport of hormones
E. Bind extracellular matrix molecules

II type tasks. For each numbered item,selct the one lettered heading that is most closely asssciated with it

4.121 At what age the bone marrow has the following characteristic distribution or appearance:
1 - Child
2 - Young age
3 - Old age
A. Bone marrow is located in epiphysis and diaphysis
B. In diaphysis there is – yellow, in epiphysis – red bone marrow
C. Bone marrow (red and yellow) is a gel like tissue
D. Only yellow bone marrow is present

4.122 For which part of the digestive system is characteristic the following:
1 - Brunner’s glands
2 - Peyer’s patches
3 - Stratified squamous nonkeratinized epithelium
4 - Tubular glands
5 - Taste buds
A. Tongue

lsmusis.lsmuni.lt/Klausimai/Spausdinti?Length=0?Kalba=EN&KategorijaId=106&Kalbos_input=EN&Kalbos=EN&KategorijaEn_input=Histology&Kat… 13/17
3/27/2019 LSMUSIS
B. Stomach
C. Duodenum
D. Ileum
E. Esophagus

4.123 What is the structure of different glial cells:


1 - Columnar cells with the cilia on apical surface
2 - Small cells with multiple radiating processes
A. Astrocytes
B. Ependymal cells
C. Oligodendrocytes

4.124 In which lymphoid and immune system organs you can find the following structures?
1 - Stratified squamous nonkeratinized epithelium
2 - Hassal's bodies
3 - Dense connective tissue trabecules with smooth muscle cells
A. Thymus
B. Spleen
C. Palatine tonsil
D. Lymph node
F. Bone marrow

4.125 In which heart wall layers you can find the following cells:
1 - Myocardium
2 - Endocardium
3 - Epicardium
A. Endothelial and smooth muscle cells
B. Striated muscle and endothelial cells
C. Striated cardiac myocytes and small blood vessels
D. Mesothelial cells, adipose cells and blood vessels, and nerves
E. Mesothelial and striated muscle cells

4.144 Provide period of gestation, during which a harmful agent caused corresponding developmental defects:
- 1-14 days
- 15-56 days
- 57-280 dienos
A. Fetopathies
B. Blastopathies
C. Embryopathies

III type tasks. For each question there is one or more correct answers:
A – if correct answers are 1,2,3
B – if correct answers are 1 and 3
C – if correct answers are 2 and 4
D – if correct answer is 4
E – if correct are all answers above

4.126 Which cells develop from mesenchyme:


1. Endothelial cells
2. Reticular cells
3. Pericytes
4. Fibrocytes

4.127 What is characteristic for the structure of the spleen in old age
1. Atrophy of white and red pulp
2. Proliferation of connective tissue
3. Increase in number of granulocytes and reticular cells
4. Decreased size and count of lymphatic follicles

4.128 Which cerebellar cells and nerve fibers make contacts with Purkinje cells:
1. Axons of stellate cells
2. Axons of the basket cells
lsmusis.lsmuni.lt/Klausimai/Spausdinti?Length=0?Kalba=EN&KategorijaId=106&Kalbos_input=EN&Kalbos=EN&KategorijaEn_input=Histology&Kat… 14/17
3/27/2019 LSMUSIS
3. Axons of the granule cells as parallel fibers
4. Axons of the interneurons in the brain and spinal cord, as climbing nerve fibers

4.129 Which of the following connective tissue cells can be identified easily and with a high degree of confidence
in routine hematoxylin and eosin preparations of loose connective tissue:
1. Eosinophils
2. Plasma cells.
3. Fat cells
4. Mast cells

4.130 The following is/are charasteristic of the bronchial wall:


1. Epithelium contain numerous goblet cells
2. Wall contains few striated muscle cells
3. There are hyaline cartilage plates
4. Epithelium is stratified squamous

4.131 Which cells may be involved in the formation of myelin?


1. Microglia
2. Schwann cells
3. Fibrous astrocytes
4. Oligondendroglia

4.132 A Haversian system or osteon consists of:


1. Osteocytes and concentric lamellae
2. Haversian canals and interstitial lamellae
3. Haversian canals
4. Outer and inner circumferential lamellae

4.133 The “T” or transverse tubules of muscle cell are:


1. Continuos with the sarcolemma
2. Part of the smooth endoplasmic reticulum
3. A characteristic feature of cardiac and skeletal myocytes
4. Part of the rough endoplasmic reticulum

4.134 A sarcomere is:


1. The smallest repeating unit of the myofibril in skeletal and cardiac muscle
2. Composed of one complete A band and one-half of two adjoining I bands
3. Delimited on both ends by a Z disc
4. Any smooth muscle cell

4.135 The Purkinje cells of cardiac muscle:


1. Are larger in average size than regular contractile cardiac muscle cells
2. Stain more lightly than regular cardiac muscle cells
3. Are modified to conduct the impulse for contraction
4. Contain more contractile myofilaments than regular cardiac muscle cells

4.136 Osteoclasts:
1. Resorb bone tissue
2. Synthesize osteoid
3. Secrete hydrolytic (lysosomal) enzymes
4. May differentiate into osteoblasts upon demand

4.137 The cell bodies (perikarya) of spinal, cranial and autonomic ganglia are surrounded and separated from
connective tissue by:
1. Oligodendroglial cells
2. Protoplasmic astrocytes
3. Reticular cells
4. Satellite cells

4.138 Eosinophilic granulocytes:


1. Become phagocytic in the presence of antigen – antibody complexes.
2. Exhibit ameboid movement
3. May be present in increased numbers during parasitic infection or in allergic states
lsmusis.lsmuni.lt/Klausimai/Spausdinti?Length=0?Kalba=EN&KategorijaId=106&Kalbos_input=EN&Kalbos=EN&KategorijaEn_input=Histology&Kat… 15/17
3/27/2019 LSMUSIS
4. Possess specific granules, which contain histamine

4.139 The secretory portion of exocrine glands may be:


1. Straight
2. Coiled
3. Branched tubular
4. Branched alveolar

4.140 Microglial cells are:


1. Developmentally related to blood monocytes
2. Mesodermal in origin
3. Phagocytic
4. Found both within the central nervous system and the periperal nervous system

4.141 Neurons in the central nervous system:


1. Are all similar in size
2. Regenerate by dividing mitotically
3. Possess mainly electrical synapses
4. In certain locations may contain neuromelanin pigment

4.142 Ependymal cells:


1. Line the brain ventricles and central canal of the spinal cord
2. May possess surface cillia at some time of their vital period
3. Are joined to each other with tight junctions
4. Secrete cerebrospinal fluid, if located in the choroid plexus

4.143 Enterochromaffin cells account for about 3% of cells in the respiratory epithelium. Which of the enumerated
cells also belong to this epithelium?
1. Columnar ciliated cells
2. Goblet cells
3. Basal cells
4. Squamous keratin-containing cells

4.1 - D 4.2 - B 4.3 - D 4.4 - B 4.5 - D 4.6 - B 4.7 - B 4.8 - D


4.9 - C 4.10 - C 4.11 - B 4.12 - A 4.13 - A 4.14 - C 4.15 - C 4.16 - E
4.17 - C 4.18 - D 4.19 - A 4.20 - C 4.21 - C 4.22 - B 4.23 - A 4.24 - B
4.25 - B 4.26 - C 4.27 - C 4.28 - A 4.29 - B 4.30 - C 4.31 - D 4.32 - A
4.33 - C 4.34 - B 4.35 - D 4.36 - C 4.37 - A 4.38 - D 4.39 - D 4.40 - B
4.41 - B 4.42 - A 4.43 - D 4.44 - D 4.45 - D 4.46 - D 4.47 - E 4.48 - E
4.49 - D 4.50 - C 4.51 - D 4.52 - D 4.53 - D 4.54 - A 4.55 - A 4.56 - D
4.57 - A 4.58 - B 4.59 - C 4.60 - E 4.61 - C 4.62 - E 4.63 - E 4.64 - B
4.65 - C 4.66 - B 4.67 - A 4.68 - A 4.69 - B 4.70 - D 4.71 - C 4.72 - D
4.73 - B 4.74 - E 4.75 - D 4.76 - D 4.77 - A 4.78 - A 4.79 - A 4.80 - B
4.81 - D 4.82 - D 4.83 - B 4.84 - A 4.85 - C 4.86 - D 4.87 - E 4.88 - B
4.89 - B 4.90 - A 4.91 - A 4.92 - B 4.93 - C 4.94 - D 4.95 - A 4.96 - B
4.97 - B 4.98 - C 4.99 - C 4.100 - C 4.101 - A 4.102 - D 4.103 - C 4.104 - E
4.105 - B 4.106 - B 4.107 - E 4.108 - B 4.109 - E 4.110 - D 4.111 - C 4.112 - A
4.113 - C 4.114 - E 4.115 - B 4.116 - B 4.117 - B 4.118 - C 4.119 - C 4.120 - B
4.121 4.122 4.123 4.124 4.125 4.144
1-A 1-C 1-B 1-C 1-C -B
2-B 2-D 2-A 2-A 2-A -C
3-C 3-E 3-B 3-D -A
4-B
5-A
4.126 4.127 4.128 4.129 4.130 4.131 4.132 4.133
1 1 1 1 1 2 1 1
2 2 2 2 3 4 3 3
3 3 3 3
lsmusis.lsmuni.lt/Klausimai/Spausdinti?Length=0?Kalba=EN&KategorijaId=106&Kalbos_input=EN&Kalbos=EN&KategorijaEn_input=Histology&Kat… 16/17
3/27/2019 LSMUSIS
4 4 4 4
4.134 4.135 4.136 4.137 4.138 4.139 4.140 4.141
1 1 1 4 1 1 1 4
2 2 3 2 2 2
3 3 3 3 3
4
4.142 4.143
1 1
2 2
3 3
4

lsmusis.lsmuni.lt/Klausimai/Spausdinti?Length=0?Kalba=EN&KategorijaId=106&Kalbos_input=EN&Kalbos=EN&KategorijaEn_input=Histology&Kat… 17/17
3/27/2019 LSMUSIS

Physics
I type tasks. Choose only one best answer

23.1 How does pitch of sound vary when measuring blood flow rate using Doppler method if directions of reflected
sound and blood flow coincide:
A. Increases
B. Decreases
C. Does not change

23.2 How does pitch of sound change if the stethoscope is pressed to the skin with bigger force:
A. Does not change
B. Decreases
C. Increases

23.3 What results in the concentration of blood cells in the central axial part of the vessel:
A. Pressure changes along the blood vessel
B. Due to velocity difference across the blood vessel
C. Due to the difference in diameters of blood cell and the vessel

23.4 Which of parameters is of greatest importance for the volume of flowing blood:
A. Gradient of pressure along the vessel
B. Viscosity of blood
C. Cross-section area of the vessel

23.5 What changes will appear in blood flow in the artery if it is affected with pressure a little smaller than systolic
pressure from outer side:
A. Blood flow will stop
B. Becomes turbulent
C. Remains laminar

23.6 What factors do not influence the rate erythrocyte sedimentation:


A. Viscosity of plasma
B. Diameter of erythrocyte
C. Force of gravity

23.7 Does the magnetic field appear around the heart with appearance of electric dipole:
A. Sometimes, not in all cases, additional conditions required
B. Only around the blood flow
C. Yes, in all cases

23.8 In what kind of body tissue nuclear magnetic resonance may appear and tomogram may be taken:
A. In bones
B. Tissues containing heavy metals
C. Tissues with fluid flow
D. In all kinds of tissues

23.9 Why in same parts of rheogram resistance increases with the time
A. Tissue is filled with the blood
B. Tissue looses the blood
C. Blood volume is constant, but increases saturation with carbon dioxide

23.10 How one can assess the peak voltage of ECG


A. According to the velocity of a recording tape
B. It is measured in millimeters. The number of millimeters is the actual answer
C. Calibration pulse is needed
D. Is necessary to compare to another ECG, in which peak value are known

23.11 Do meanings of the terms electrocardiogram, rhythmogram, rhytmohistogram differ?


A. They are synonyms
B. Only electrocardiogram and rhytmohistogram are synonyms
C. All of them have different meanings

lsmusis.lsmuni.lt/Klausimai/Spausdinti?Length=0?Kalba=EN&KategorijaId=125&Kalbos_input=EN&Kalbos=EN&KategorijaEn_input=Physics&Kategor… 1/8
3/27/2019 LSMUSIS
23.12 Why three standard leads of ECG I, II, III, but not two or four are commonly used:
A. For determination of the position of electric dipole vector in frontal plane at least three points are needed,
three potential differences between these points could be recorded
B. It is a matter of agreement

23.13 What is the minimal number of ECG leads to assess the electric dipole vector orientation of the heart in
frontal plane:
A. One
B. Two
C. Three

23.14 During the recording of ECG leads I and II the electrode is attached not to its standard place – wrist, but to
the right shoulder. Is it compatible with Einthoven theory:
A. No
B. Yes

23.15 What property of γ and x-rays is harmful for the living cell:
A. They pass the cell easily
B. They have a thermal effect
C. They excite atoms and molecules of surrounding objects
D. They evoke ionization

23.16 What characteristics of x-rays cannot be adjusted during x-ray procedure:


A. Wavelength
B. Energy of radiation
C. Velocity of waves
D. Exposition dose

23.17 Which radiation is of greatest effect human body, if a single absorbed dose is the same:
A. Alpha
B. Beta
C. Neutrons
D. Gamma
E. X-ray

23.18 What properties of electric circuit are characteristic for biological membrane:
A. Capacitance
B. Resistance
C. Resistance and capacitance
D. Inductance

23.19 What does the resting potential depend on:


A. Only on K⁺ distribution inside and outside of the cell
B. Only on Na⁺ distribution inside and outside of the cell
C. On membrane permeability for ions
D. On membrane permeability and distribution of K⁺, Na⁺, Cl⁻ ions across the membrane

23.20 What factors determine ionic current magnitude in the membrane:


A. Type of ions
B. Concentration difference of ions inside and outside of the cell
C. Membrane potential magnitude
D. Potential difference across the membrane and ion type
E. Thickness of the membrane and ion type

23.21 Which factor influences to the negativity of resting potential:


A. Excess of Cl⁻ ions inside the cell
B. Lack of Na⁺ inside the cell
C. Excess of unbalanced organic anions inside the cell
D. The inside of the cell lacks of K⁺ ions

23.22 Is energy input from the cell required for the ionic transport through open ionic channels of the biological
membrane?

lsmusis.lsmuni.lt/Klausimai/Spausdinti?Length=0?Kalba=EN&KategorijaId=125&Kalbos_input=EN&Kalbos=EN&KategorijaEn_input=Physics&Kategor… 2/8
3/27/2019 LSMUSIS
A. Energy input from the cell is necessary only for positive ions.
B. No energy from the cell is required for such a transport.
C. Energy input from the cell is necessary in all cases.
D. Energy input from the cell is necessary only for negative ions.

23.23 What factors result in action potential:


A. Action of Na⁺-K⁺ pump
B. Underthreshold depolarization
C. Na⁺, K⁺ concentration gradient across the membrane
D. Depolarization and Na⁺, K⁺ concentration gradient across the membrane

23.24 Why does grounding of a metal case of medical equipment prevents electric shock:
A. Fuse instantly cuts of the circuit
B. Metal case is of zero voltage
C. Apparatus brakes

23.25 Grounding is needed if electrocardiograph case:


A. Is made of metal and using outlet power
B. Is made of metal and using battery power
C. Is made of plastic and using outlet power

23.26 Why the functional grounding eliminates electromagnetic noise and improves quality of recording bioelectric
signals?
A. Earth has a constant electric potential, which is resistant to the influence of electromagnetic fields. The
same holds for biological object, which is connected to it
B. Increases the amplitude of biopotential
C. Increases sensitivity of ECG recorder

23.27 Why with the relocation of electrodes or body part we have to adjust anew oscillatory circuit of ultrahigh
frequency device to the resonance:
A. Capacitance of oscillatory circuit changes
B. Voltage of the power unit decreases

23.28 Why heat production is different in different tissues in high frequency electric field:
A. Due to different conductivity and permittivity constant
B. Due to different density of a tissue
C. Due to different innervation

23.29 Why during high frequency 30-50 MHz electric field physiotherapy patients are allowed to wear metal
chains, rings, watches:
A. High frequency electric field produces no heat neither in metals nor in dielectrics
B. Objects are small
C. High frequency affects only biological objects

23.30 Does sample mean (average) depends on sample size:


A. Yes
B. No

23.31 Which mean - population mean or sample mean - is constant, not random variable:
A. Population mean is constant variable
B. Sample mean is constant variable
C. Both means are random variables
D. Both means are constant variables

23.32 Is confidence interval calculated for population mean or for sample mean:
A. For population mean
B. For sample mean
C. For both of them
D. Neither for population mean, nor for sample mean

23.33 When difference between parameters of compared groups is statistically significant:


A. p-value < α

lsmusis.lsmuni.lt/Klausimai/Spausdinti?Length=0?Kalba=EN&KategorijaId=125&Kalbos_input=EN&Kalbos=EN&KategorijaEn_input=Physics&Kategor… 3/8
3/27/2019 LSMUSIS
B. p-value ≥ α

23.34 Where in our organism there is a largest amount of paramagnetics:


A. Where are most of water molecules.
B. In areas of largest oxygen concentration.
C. Where most free radicals are created.
D. Where faster cell fission takes place.

II type tasks. For each numbered item,selct the one lettered heading that is most closely asssciated with it

23.35 Which of two ECG recorders using outlet power need grounding:
1 - In metallic case
2 - In plastic case
A. Need both protective and functional grounding
B. Functional grounding is sufficient
C. Grounding is not needed

23.36 Which of groundings indicated by number serves a purpose indicated by letter:


1 - Protective
2 - Functional
A. In hazard situation decreases contact voltage
B. Eliminates electric noise during ECG registration

23.37 What units of ionizing radiation could be used for measurement of these radiation doses:
1 - Expositional dose
2 - Absorbed dose
3 - Integral dose
4 - Equivalent dose
A. Joule, gramrad
B. Sivert, rem
C. Coulomb per kilogram, roentgen
D. Gray, rad

23.38 What biological effect produce:


1 - Infrared radiaton
2 - Visible light
3 - Ultraviolet radiation
A. Light perception
B. Thermal
C. Bactericide

III type tasks. For each question there is one or more correct answers:
A – if correct answers are 1,2,3
B – if correct answers are 1 and 3
C – if correct answers are 2 and 4
D – if correct answer is 4
E – if correct are all answers above

23.39 What methods are most effective for tumor destruction:


1. Exposing tumor to proton emission
2. Exposing tumor to electron emission
3. Exposing tumor to nuclei of heavy metals
4. Exposing tumor to gamma radiation

23.40 During nuclear magnetic resonance tomography human body is exposed to:
1. Static magnetic field
2. Proton emission
3. Electromagnetic waves
4. High frequency currents

23.41 What does represent tomogram recorded by proton magnetic resonance method:
1. Net density of nuclei of a given layer
lsmusis.lsmuni.lt/Klausimai/Spausdinti?Length=0?Kalba=EN&KategorijaId=125&Kalbos_input=EN&Kalbos=EN&KategorijaEn_input=Physics&Kategor… 4/8
3/27/2019 LSMUSIS
2. Proton density in a given layer
3. The number of protons that passed the exposed surface
4. Proton relaxation time

23.43 What of these factors are important for electric shock or trauma:
1. Current magnitude
2. Voltage
3. Skin resistivity
4. Duration of current pulse
5. Frequency of a current
6. Current path through the human body
7. The phase of cardiac cycle

23.44 What characteristics of the preventive grounding of medical equipment are important:
1. Increased current passing through the fuse
2. Decreased hazardous current passing human body
3. Decreased contact voltage
4. Reduced melting time of the fuse
5. Decreased duration of current impact

23.45 What kind of primary processes induces electric field of 30-50 MHz frequency, when the electrode surfaces
are coated with insulator:
1. Alternated polarization of dielectric tissue
2. Vibration of free ions
3. Rotation of water molecules corresponding to the field frequency
4. Turbulent electric current of conductive tissue

23.46 Random variable is completely characterized by:


1. Mean (average)
2. Distribution function
3. Dispersion (variance)
4. Values of random variable and their probabilities
5. Values of random variable
6. Probabilities of values of random variable

23.47 Normal (Gaussian) distribution is completely characterized by:


1. Mean (average) and dispersion (variance)
2. Distribution function
3. Density function
4. Mean (average)
5. Dispersion (variance)

23.48 When does confidence interval get narrow:


1. When sample size is increased
2. When sample size is decreased
3. When confidence level is decreased
4. When confidence level is increased

23.49 Which statistical criterion is used to compare means of two populations:


1. McNemar’s
2. Fisher’s (F)
3. Chi-square (χ² )
4. Student’s (t)

23.50 Which statistical criterion is used to compare dispersions of two populations:


1. McNemar’s
2. Student’s (t)
3. Chi-square (χ² )
4. Fishers (F)

23.51 Which statistical criterion is used to compare probabilities (proportions) of two populations:
1. Student’s (t)

lsmusis.lsmuni.lt/Klausimai/Spausdinti?Length=0?Kalba=EN&KategorijaId=125&Kalbos_input=EN&Kalbos=EN&KategorijaEn_input=Physics&Kategor… 5/8
3/27/2019 LSMUSIS
2. Fisher’s (F)
3. Wilcoxon’s
4. Z test

23.52 Which statistical criteria are used testing effect of treatment:


1. McNemar’s
2. Paired Student’s (t)
3. Wilcoxon’s
4. Fisher’s (F)

23.53 Which statistical criterion is used to test dependence between two nominal variables:
1. Student’s (t)
2. Fisher’s (F)
3. Chi-square (χ²)
4. Wilcoxon’s

23.54 How the human body is magnetized during the Nuclear Magnetic Resonance tomography:
1. Positive ions are separated from negative ones.
2. Non coupled electrons are oriented.
3. Cells are stretched towards poles
4. Magnetic nuclei of atoms are oriented
5. Dipoles of water molecules are oriented
6. Cirlian’s effect takes place

23.55 What are differences of influence of electric scalpel compared to mechanical one:
1. Not only cuts, but also sterilizes
2. Not only cuts, but also proteins become coagulated
3. Not only cuts, but also leaves a track of burns
4. Not only cuts, but also anaesthetizes

23.56 What units could be used for measurements of current density:


1. A/m²
2. mM/cm
3. V/cm•s
4. mA/cm²
5. mA/s

23.57 When the amount of current density has to be taken into account:
1. When low frequency currents act.
2. During curative electrophoresis
3. During galvanization procedure
4. When patient is heated using infrared rays
5. When patient is heated using high frequency electric field

23.58 How does pulse magnetic field act:


1. Has to act for a long time because impulses do not have enough time to act
2. Turbulent currents are induced
3. Influences mostly the surface of an organism
4. Can excite excitable cells

23.59 Which are units of intensity of microwaves:


1. J/cm²
2. J/cm²s
3. V/s
4. W/m²
5. W/cm s
6. V/m³

23.60 How are electromagnetic field forces acting on ions called:


1. Hall
2. Coulomb
3. Cirlian

lsmusis.lsmuni.lt/Klausimai/Spausdinti?Length=0?Kalba=EN&KategorijaId=125&Kalbos_input=EN&Kalbos=EN&KategorijaEn_input=Physics&Kategor… 6/8
3/27/2019 LSMUSIS
4. Lorentz
5. Bolzmann

IV type tasks. Choose only one best answer

23.61 What is sound intensity in bels and decibels, if sound intensity exceeds the threshold by factor 1000:
A. 10 B, 100 dB
B. 10³ B, 10⁴dB
C. 3B, 30 dB
D. 30B, 300 dB

23.62 Loudness of a normal talk is 40 phones. What is the intensity of a talk in W/m2, if the threshold for 1 kHz
oscillations is 10⁻¹² W/m2:
A. 4*10⁻¹²
B. 40*10⁻¹²
C. 10⁻⁷
D. 10⁻⁸

23.63 Impaired patient is able to hear a sound of 40dB loudness. What should be the coefficient, of amplification of
hearing aid that patient could percept normal talk:
A. 10⁴
B. 10³
C. 40
D. 4*10⁴

23.64 Electromagnetic waves of 40 MHz for electric field therapy are modulated by 1 kHz frequency. Duration of a
pulse was 100 ms, strength of electric field varied from 1 to 3 V/m. What was approximate period of time between
pulses, if they were generated close to normal heart rate?
A. 0,3 s
B. 0,001 s
C. 100 ms
D. 0,9 s

23.65 The distance between the subject and the source of electromagnetic waves is reduced twice. What is the
change in energy affecting the subject over the same time:
A. More 1,4 times
B. More twice
C. Approximately 3 times more (exactly 2,8)
D. More 4 times
E. More 8 times

23.66 For a single heart contraction an electric pulse of 5mA and 1ms duration is needed. Electric capacity of
battery is 0,5Ah. What is the lifetime (in years) of such an electrocardiostimulator:
A. 3
B. 5
C. 9

23.67 The intensity of radiation is reduced by 50% after passing of concrete wall 5cm thickness. How much it
would be reduced, if the thickness is doubled:
A. 4 times
B. 10 times
C. 2,5 times

23.68 The distance between source of gamma radiation an pint of measurement increased from 3m to 12m. How
much the power of exposed dose have changed:
A. Increased by factor of 4
B. Decreased by factor of 4
C. Decreased by factor of 9
D. Decreased by factor of 16

23.1 - A 23.2 - C 23.3 - B 23.4 - C 23.5 - B 23.6 - C 23.7 - C 23.8 - D


23.9 - B 23.10 - C 23.11 - C 23.12 - A 23.13 - B 23.14 - B 23.15 - D 23.16 - C
lsmusis.lsmuni.lt/Klausimai/Spausdinti?Length=0?Kalba=EN&KategorijaId=125&Kalbos_input=EN&Kalbos=EN&KategorijaEn_input=Physics&Kategor… 7/8
3/27/2019 LSMUSIS

23.17 - A 23.18 - C 23.19 - D 23.20 - D 23.21 - C 23.22 - B 23.23 - D 23.24 - A


23.25 - A 23.26 - A 23.27 - A 23.28 - A 23.29 - A 23.30 - A 23.31 - A 23.32 - A
23.33 - A 23.34 - C
23.35 23.36 23.37 23.38
1-A 1-A 1-C 1-B
2-B 2-B 2-D 2-A
3-A 3-C
4-B
23.39 23.40 23.41 23.43 23.44 23.45 23.46 23.47
1 1 2 1 1 1 2 1
3 3 4 2 2 2 4 2
3 3 3 3
4 4
5 5
6
7
23.48 23.49 23.50 23.51 23.52 23.53 23.54 23.55
1 4 4 4 1 4 2 1
3 2 4 2
3 3
23.56 23.57 23.58 23.59 23.60
4 1 2 2 2
2 4 4 4
3
23.61 - C 23.62 - D 23.63 - A 23.64 - D 23.65 - D 23.66 - C 23.67 - A 23.68 - D

lsmusis.lsmuni.lt/Klausimai/Spausdinti?Length=0?Kalba=EN&KategorijaId=125&Kalbos_input=EN&Kalbos=EN&KategorijaEn_input=Physics&Kategor… 8/8
3/27/2019 LSMUSIS

biochemistry
I type tasks. Choose only one best answer

42.1 Which of the following carbohydrates is absorbed from the intestinal lumen into blood:
A. Starch
B. Glycogen
C. Sucrose
D. Monosaccharides
E. Polysaccharides

42.2 What is normal glucose concentration in blood:


A. 3,3–5,5 mmol/l
B. 5,5–8,5 mmol/l
C. 5,0–10,0 mg/100ml
D. 0,3–0,5 mmol/l

42.3 The major source of blood glucose during the early stages of starvation (i.e. from 8 to 16 hours) is:
A. Liver glycogen.
B. Muscle glycogen.
C. Liver gluconeogenesis from fatty acids.
D. Liver gluconeogenesis from proteins.

42.4 Which of the following is the final product of carbohydrate degradation:


A. CO₂
B. H₂O
C. CO₂, H₂O and energy
D. Urea

42.5 How many molecules of ATP are formed under complete degradation of glucose in aerobic conditions:
A. 2 ATP
B. 12 ATP
C. 3 ATP
D. 38 ATP

42.6 The largest amount of energy is released under breakdown of which of the following compounds:
A. Carbohydrates
B. Proteins
C. Nucleic acids
D. Fats

42.7 Which pathway of carbohydrate metabolism generates ribose and reduced NADP essential for biosyntheses
of nucleic acids, fatty acids and cholesterol:
A. Glycolysis
B. Krebs cycle
C. Pentose phosphate pathway
D. Gluconeogenesis

42.8 What is wrong in diabetes mellitus:


A. Carbohydrate digestion
B. Glucose absorption from intestine
C. Fructose absorption
D. Consumption of glucose in tissues

42.9 What are the energy-producing organelles in eukarotic cells?


A. Golgi
B. Nucleus
C. Endoplasmic reticulum
D. Mitochondria
E. Lysosomes

42.10 Which of the following is the main mode of glucose degradation under aerobic conditions:

lsmusis.lsmuni.lt/Klausimai/Spausdinti?Length=0?Kalba=EN&KategorijaId=144&Kalbos_input=EN&Kalbos=EN&KategorijaEn_input=biochemistry&K… 1/13
3/27/2019 LSMUSIS
A. Glucose → pyruvic acid → acetic acid (acetyl-CoA) → CO2 + H2O
B. Glucose →lactic acid → urea
C. Glucose → acetic acid → pyruvic acid → CO2 + H2O
D. Glucose → lactic acid → acetic acid→ CO2 + H2O

42.11 The universal energy currency for all cells is


A. ATP
B. NAD⁺
C. ADP
D. Enzymes
E. O₂

42.12 Cyanides produce hypoxia by:


A. Producing central hypoventilation
B. Interfering with O2 transport by blood
C. Slowing capillary circulation
D. Inhibition of tissue respiration

42.13 Branching enzyme is deficient in patients with a particular type of glycogen storage disease. Which of the
following is the most probable finding under this type of disorder?
A. Accumulation of glycogen with short outer chains
B. Accumulation of glycogen with long outer chains
C. Accumulation of high-amounts of glycogen with normal structure
D. None of the above

42.14 All the following are endogenous antioxidants EXCEPT


A. Vitamin E
B. Glutathione
C. Hydrogen peroxide
D. Superoxide dismutase
E. Vitamin C

42.15 Which of the following predetermines the nutritional quality of a protein:


A. Structure of a protein
B. Number of amino acids
C. Amount of essential amino acids
D. Presence of metal ion in a protein

42.16 Which one of the following compounds causes formation of methemoglobin in the blood:
A. α-Tocopherol
B. CO (carbon monoxide)
C. NaNO₂ (nitrites)
D. Selenium

42.17 Which of the following factors can not affect the oxygen-binding ability of hemoglobin?
A. Presence of water
B. Presence of CO2
C. Changes in pH
D. D. Presence of 2,3-bisphosphoglycerate

42.18 Which of the following statement describes the function of myoglobin:


A. It carries oxygen from the lungs to tissues
B. It carries CO₂ from tissues to the lungs
C. It accumulates oxygen in muscles
D. It carries oxygen within the lungs

42.19 The nitrogen balance can be characterised as:


A. Number of nitrogen atoms in a protein molecule
B. Amount of nitrogen in amino acids
C. The molecular mass of nitrogen
D. The difference between the total amount of nitrogen uptaken by human organism and the total amount of
nitrogen excreted from organism

lsmusis.lsmuni.lt/Klausimai/Spausdinti?Length=0?Kalba=EN&KategorijaId=144&Kalbos_input=EN&Kalbos=EN&KategorijaEn_input=biochemistry&K… 2/13
3/27/2019 LSMUSIS
42.20 Which reaction is catalyzed by transaminases (aminotransferases):
A. Transfer of amino group from alpha-amino acid to alpha-keto acid
B. Release of amino group from a protein molecule
C. Removal of amino groups (deamination) from amino acids
D. Catalysis of essential amino acid biosynthesis

42.21 The main pathway of ammonia detoxification in humans is:


A. Synthesis of pyrimidine nucleotides
B. Synthesis of purine nucleotides
C. Synthesis of biologically active amines
D. Urea synthesis

42.22 Which of the following amino acids is a precursor of histamine:


A. Tyrosine
B. Tryptophan
C. Histidine
D. Methionine

42.23 What product is formed during breakdown of non-protein part (heme) of hemoproteins:
A. Urea
B. Lipoproteins
C. Bile pigments
D. Bile acids

42.24 Cholesterol is required for the synthesis of:


A. Bile acids
B. Vitamin C.
C. Niacin.
D. Fats.

42.25 The activity of adipose tissue triglyceride lipase depends on:


A. Level of proteins in a cell
B. Coenzymes
C. The active center of the enzyme
D. Hormones

42.26 Which of the following is considered as essential fatty acids:


A. Short chain saturated fatty acids
B. Unsaturated fatty acids with more than one double bond
C. Long chain saturated fatty acids
D. Branched-chain fatty acids.

42.27 Ketonemia is characteristic in diabetes mellitus type I. Which of the following compounds is a ketone body?
A. Acetyl-CoA
B. Acetoacetate
C. Acetone
D. Propionyl-CoA

42.28 In the presence of excess acetyl- CoA in mammalian liver "ketone bodies" may build up in concentration in
the blood plasma. One of the three common "ketone bodies" is
A. Acetyl-Co
B. Pyruvate
C. Alpha-ketoglutarate
D. Alpha- ketobutyrate
E. Beta- hydroxybutyrate

42.29 Which of the following vitamin deficiency can cause anemia?


A. Thiamine
B. Pantothenate
C. Folic acid
D. Riboflavin

42.30 Which vitamin is the most powerful antioxidant:


lsmusis.lsmuni.lt/Klausimai/Spausdinti?Length=0?Kalba=EN&KategorijaId=144&Kalbos_input=EN&Kalbos=EN&KategorijaEn_input=biochemistry&K… 3/13
3/27/2019 LSMUSIS
A. Vitamin A
B. Vitamin D
C. Vitamin K
D. Vitamin E
E. Niacin

42.31 Which vitamin is a constituent part of a coenzyme common for reactions of amino acid transamination and
decarboxylation:
A. Vitamin B₆
B. Vitamin B₂
C. Vitamin PP (niacin)
D. Vitamin C

42.32 Vegetarians and vegans are at deficiency risk of:


A. Thiamine
B. Vitamin A
C. Cyancobalamin
D. Ascorbic acid

42.33 Which one of the following vitamins takes part in synthesis of collagen:
A. Vitamin A
B. Vitamin C
C. Vitamin B1
D. Vitamin B6

42.34 Light-sensitivity of retina depends on:


A. Vitamin K
B. Vitamin A
C. Vitamin B₂
D. Folic acid

42.35 Coenzyme A is essential in the metabolism of carbohydrates and lipids. Which one of the following vitamins
is a constituent part of coenzyme A?
A. Folic acid (vitamin B₉)
B. Ascorbic acid (vitamin C)
C. Pantothenic acid (vitamin B₅)
D. Riboflavin (vitamin B₂)

42.36 Gene expression can be induced by:


A. Vitamins
B. Amino acids
C. Steroid hormones
D. Prostaglandin

42.37 Synthesis of proteins in microorganisms is inhibited by:


A. Derivatives of barbituric acid
B. Hormones of the thyroid gland
C. ATP
D. Antibiotics

42.38 In DNA molecules, the point mutations can be induced by:


A. Changes in pH of body fluids
B. Hypoxia
C. Ionizing radiation
D. Deficiency of vitamins

42.39 Which of the following factors is the most powerful vasoconstrictor


A. Aldosterone
B. Atrial natriuretic peptide
C. Angiotensinogen
D. Angiotensin II

lsmusis.lsmuni.lt/Klausimai/Spausdinti?Length=0?Kalba=EN&KategorijaId=144&Kalbos_input=EN&Kalbos=EN&KategorijaEn_input=biochemistry&K… 4/13
3/27/2019 LSMUSIS
42.40 Which of the following hormones takes part in regulation of electrolyte balance:
A. Tyroxine
B. Insulin
C. Aldosterone
D. Corticosterone
E. Glucagon

42.41 Which of the following hormones takes part in regulation of H2O balance and osmotic pressure of blood
plasma:
A. Prolactin
B. Somatostatin
C. Insulin
D. Vasopressin
E. Thyroxine

42.42 Gout is characterized by accumulation of crystals of:


A. Urea
B. Sodium urate (sodium salt of uric acid)
C. Sodium carbonate
D. Calcium carbonate

42.43 Which of the following parameters is of the clinical importance under the first hours of myocardial infarction:
A. FABP
B. Phosphocreatine kinase
C. Myoglobin
D. Lactate dehydrogenase (LDH)
E. TnI/TnT

42.44 Isoenzymes of lactate dehydrogenase are specific for tissues: LDH1 and LDH2 isoenzymes are
accumulated in myocardial cells and red blood cells, whereas LDH4 and LDH5 – in cells of the liver and skeletal
muscles. Which of the following factors can give “positive fallse”results of LDH examination in blood serum of the
patient with non-typical myocardial infarction:
A. High amounts of LDL in blood serum
B. Blood hemolysis
C. High amounts of VLDL in blood serum
D. Liver pathology

42.45 In myocardial ischemia all the following are components of "lipid triad" except:
A. Increased LDL-cholesterol level
B. Decreased HDL-choelesterol level
C. Increased level of triglycerides in blood plasma
D. Incred level of free fatty acids in blood plasma

42.46 Which of the following factors does not contribute to dietary fat digestion?
A. Lipase
B. Carnitine
C. Phospholipase A2
D. Colipase
E. Bile salts

42.47 Which of the following is a direct source of energy in the first 5 seconds of muscle contraction?
A. Mg-ADP
B. Phosphoarginine
C. Carnitine
D. Phosphocreatine
E. Glucose

II type tasks. For each numbered item,selct the one lettered heading that is most closely asssciated with it

42.48 Match each numbered vitamin with coenzymatic function:


1 - Vitamin PP (niacin)
2 - Biotin (vitamin H)

lsmusis.lsmuni.lt/Klausimai/Spausdinti?Length=0?Kalba=EN&KategorijaId=144&Kalbos_input=EN&Kalbos=EN&KategorijaEn_input=biochemistry&K… 5/13
3/27/2019 LSMUSIS
3 - Vitamin B6 (pyridoxin)
4 - Vitamin C (ascorbic acid)
A. A coenzyme of oxidoreduction
B. A coenzyme of carboxylation
C. A coenzyme of transamination

42.49 Match each nutritional component with the energetic value:


1 - Carbohydrates 1g
2 - Proteins 1g
3 - Fats 1g
A. 9,3 kcal (38,9 kJ)
B. 4,1 kcal (17,2 kJ)

42.50 For each nutrient hydrolysed, selects the appropriate digestive enzyme:
1 - Carbohydrates
2 - Proteins
3 - Fats
A. Pepsin
B. Amylase
C. Pancreatic lipase

42.51 Match each type of medicine, with the mechanism of action:


1 - Sulfonamides
2 - Antibiotics
A. Inhibitors of folic acid synthesis
B. Inhibitors of protein biosynthesis

42.52 Match the type of diabetic comma with the typical condition:
1 - Ketoacidotic comma
2 - Hyperosmotic comma
3 - Lactacidemic comma
A. Hyperglycaemia
B. Lactic acidemia
C. Ketonemia

42.53 Match each hormone with its function:


1 - Vasopressin
2 - Estradiol
3 - Corticotropin
4 - Parathyroid hormone
5 - Insulin
A. The regulation of metabolism of fat, carbohydrate and amino acid
B. The regulation of adrenal cortex function
C. The regulation of fertility
D. The regulation of H2O homeostasis
E. The regulation of Ca+2 metabolism

42.54 For each enzyme below select the process catalysed:


1 - Transamination of amino acids
2 - Oxidative deamination of amino acids
3 - Decarboxylation of amino acids
A. Decarboxylases
B. Transaminases (aminotransferases)
C. Oxidases

42.55 Match each process of lipid metabolism with the most appropriate regulatory mechanism:
1 - Synthesis of fats in adipose tissue
2 - Decomposition of fats in adipose tissue
3 - Synthesis of cholesterol in the liver cells
A. Activation by insulin
B. Inhibition by insulin
C. Feet back inhibition

lsmusis.lsmuni.lt/Klausimai/Spausdinti?Length=0?Kalba=EN&KategorijaId=144&Kalbos_input=EN&Kalbos=EN&KategorijaEn_input=biochemistry&K… 6/13
3/27/2019 LSMUSIS
42.56 For synthesis of each compound below, select the appropriate enzyme:
1 - Glycogen
2 - Triglyceride
3 - Glucose - 6 - phosphate
A. Acyl-CoA transferase
B. Glucokinase
C. Glycogen synthase

42.57 For each type of blood lipoproteins listed below, select predominant lipid component:
1 - Chylomicron
2 - VLDL ( pre-b-lipoproteins)
3 - LDL (b-lipoproteins)
4 - HDL (a-lipoproteins)
A. TAG (triacylglycerol)
B. Cholesterol (free and esterified)
C. Cholesterol (free and esterified) and phospholipids

42.58 Match the numbered effects with the enzyme inhibited:


1 - Diminishing cholesterol amount in blood
2 - Relieve of inflammation
3 - Decreasing concentration of uric acid in blood (anti-uricemic effect)
4 - Diminishing of malignant tumour growth
A. Prostaglandin synthase
B. Xanthine oxidase
C. 3-hydroxyl, 3-methylglutaryl-CoA reductase (HMG-CoA reductase)
D. Thymidylate synthase

42.59 For each process of lipid metabolism below, select the place of location:
1 - Fatty acid oxidation
2 - Replication of genomic DNA
3 - Cholesterol biosynthesis
A. Endoplasmic reticulum
B. Mitochondria
C. Nucleus

42.60 Match each process with intracellular location:


1 - Glucose degradation to pyruvate
2 - Oxidative decarboxylation of pyruvate
3 - Acetyl-CoA oxidation (the citric acid cycle)
4 - Pentose phosphate pathway
A. Cytosol
B. Mitochondria

42.61 In which place of the cell do the following processes occur:


1 - Terminal steps of degradation of nutrient to water and CO2
2 - DNA replication
3 - Anaerobic glycolysis
A. Mitochondria
B. Cytosol
C. Mitochondria and cytosol
D. Nucleus

42.62 For each biologically active amine below select amino acids of which it is synthesised:
1 - Histamine
2 - Serotonin
3 - Gama-aminobutyric acid
A. Tryptophan
B. Histidine
C. Glutamic acid
D. Aspartic acid

III type tasks. For each question there is one or more correct answers:

lsmusis.lsmuni.lt/Klausimai/Spausdinti?Length=0?Kalba=EN&KategorijaId=144&Kalbos_input=EN&Kalbos=EN&KategorijaEn_input=biochemistry&K… 7/13
3/27/2019 LSMUSIS
A – if correct answers are 1,2,3
B – if correct answers are 1 and 3
C – if correct answers are 2 and 4
D – if correct answer is 4
E – if correct are all answers above

42.63 Which of the following statements describe catabolic processes?


1. Energy is produced under catabolic processes
2. Macromolecules undergo breakdown in catabolic processes
3. Catabolic processes occur in both mitochondria and cytoplasm
4. Energy is consumed in catabolic processes

42.64 Which one of the following statements about myoglobin is correct?


1. Heme iron in myoglobin is in reduced state (Fe2+)
2. Cooperative binding of oxygen
3. Myoglobin is intracellular protein
4. The protein contains alpha and beta chains

42.65 Which of the following compounds can be attached to hemoglobin:


1. CO₂
2. O₂
3. H⁺
4. 2,3-diphosphoglycerate

42.66 In cells, glucose can be used for:


1. Synthesis of glycogen in the liver
2. Production of energy under glucose degradation
3. Transformation of glucose to other carbohydrates
4. Synthesis of fats
5. Synthesis of glycoproteins and amino acids

42.67 In which metabolic pathway can glucose 6-phosphate be metabolized in cells:


1. Synthesis of glycogen
2. Anaerobic breakdown
3. Aerobic breakdown
4. Pentose phosphate pathway
5. Formation of free glucose

42.68 Which of the following compounds can be converted into acetyl-CoA:


1. Carbohydrate
2. Lipid
3. Protein
4. Purine nucleotides

42.69 In which of the following pathways acetyl-CoA is metabolized (is formed or used for synthesis):
1. Oxidation via citric acid (Krebs) cycle
2. Fatty acid biosynthesis
3. Cholesterol synthesis
4. Ketone body formation

42.70 Biological role of the citric acid cycle is:


1. Oxidation of acetyl-CoA formed in carbohydrate metabolism to the end products
2. Oxidation of acetyl-CoA formed in lipid metabolism
3. Oxidation of acetyl-CoA formed in protein metabolism
4. Synthesis of urea
5. Synthesis of uric acid

42.71 What happens with lactate formed under skeletal muscle contraction?
1. Phosphorus
2. Magnesium
3. Chrome
4. Iron
lsmusis.lsmuni.lt/Klausimai/Spausdinti?Length=0?Kalba=EN&KategorijaId=144&Kalbos_input=EN&Kalbos=EN&KategorijaEn_input=biochemistry&K… 8/13
3/27/2019 LSMUSIS
5. Molybdenum

42.72 What happens with lactate formed in glycolysis under skeletal muscle contraction:
1. It is transported via blood to the liver
2. The majority of lactate is converted to glucose in the muscle
3. The majority of lactate is converted to glucose and glycogen in the liver
4. Some bile acids are synthesized

42.73 Which of the following values of cholesterol concentration fit into the normal physiological range:
1. 6.5 mmol/l
2. 5 mmol/l
3. 7,8 mmol/l
4. 4 mmol/l

42.74 Gluconeogenesis can proceed from:


1. Lactate
2. Amino acids
3. Glycerol
4. Glucose
5. Urea

42.75 Which of the following enzymes participate in catalysis of ethanol metabolism in liver:
1. Catalase
2. Alcohol dehydrogenase
3. Microsomal ethanol-oxidising system
4. Hexokinase
5. Lactate dehydrogenase

42.76 Oxidative phosphorylation is:


1. A way of phosphate group uptake into cells
2. Removal of phosphate groups from proteins
3. Synthesis of ATP from other high-energy phosphates
4. Sunthesis of ATP using energy of electrochemical gradient of H+ of mitochondria

42.77 ATP hydrolysis provides energy for:


1. Contraction of muscles
2. Active transport of ions and metabolites across membranes
3. Protein biosynthesis
4. Hormone action

42.78 Mitochondrial ATP synthesis requires:


1. ATP synthase
2. An intact inner mitochondrial membrane
3. A [H+] gradient
4. Carnitine transporter

42.79 Methionine is used for:


1. Creatine biosynthesis
2. Adrenaline biosynthesis
3. Phosphatidylcholine biosynthesis
4. Methylation of DNA and RNA

42.80 The coenzyme form of vitamin B6 (Pyridoxal phosphate) is used in the following reactions of the amino acid
metabolism:
1. Transamination
2. Deamination
3. Decarboxylation
4. Decomposition of C-atom chain of amino acid

42.81 Which of the following pathways considered as a major in ammonia detoxification:


1. Creatine biosynthesis
2. Hem biosynthesis
3. Phosphatidylcholine biosynthesis
lsmusis.lsmuni.lt/Klausimai/Spausdinti?Length=0?Kalba=EN&KategorijaId=144&Kalbos_input=EN&Kalbos=EN&KategorijaEn_input=biochemistry&K… 9/13
3/27/2019 LSMUSIS
4. Urea biosynthesis

42.82 Urea is the end product of metabolism of the following compounds:


1. Carbohydrates
2. Lipids
3. Bile pigments
4. Proteins

42.83 The amount of urea is excreted in the urine within 24-hour period is:
1. 2 g
2. 1 mg/l
3. 10 mmol/l
4. 25-30 g

42.84 Major part of amino acids are catabolized by some common routes as:
1. Transamination
2. Purines and pyrimidines biosynthesis
3. Deamination and decarboxylation
4. Transmethylation

42.85 In humans ketonemia and ketonuria are the symptoms of:


1. Diabetes mellitus
2. Atherosclerosis
3. Starvation
4. The gout disease

42.86 Which of the following organic compounds are called ‘ketone bodies’:
1. Acetone
2. Acetoacetic acid
3. Beta hydroxybutyric acid
4. Lactic acid

42.87 Which of the following lipoproteins are involved in transfer of lipids in blood:
1. Chylomicrons
2. Low density lipoproteins (beta-lipoproteins)
3. Very low density lipoproteins (pre-beta-lipoproteins)
4. High-density lipoproteins (alpha-lipoproteins)

42.88 Which of the following vitamins are fat-soluble:


1. Vitamin A
2. Vitamin H
3. Vitamin E
4. Vitamin B₁₂

42.89 Which of the following reactions require vitamin B₁₂:


1. Transmethylation
2. Decarboxylation
3. Isomerisation
4. Transamination

42.90 Which condition is caused by deficiency of vitamin B1:


1. Scurvy
2. Beri-beri disease
3. Rickets
4. Vernicke-Korsakoff syndrome

42.91 Deficiency of vitamin K causes:


1. Night blindness
2. Fragile cell membranes
3. Rickets - bone problems
4. Delayed blood clotting

42.92 Which of the following vitamins take part in the process of blood cell formation:
lsmusis.lsmuni.lt/Klausimai/Spausdinti?Length=0?Kalba=EN&KategorijaId=144&Kalbos_input=EN&Kalbos=EN&KategorijaEn_input=biochemistry&… 10/13
3/27/2019 LSMUSIS
1. Vitamin C
2. Folic acid
3. Vitamin D
4. Vitamin B₁₂

42.93 Which of the following vitamins have antioxidant-properties:


1. Vitamin D
2. α-tocopherol (vitamin E)
3. Vitamin K
4. Vitamin C

42.94 Which of the following hormones act through cAMP-dependent pathway:


1. Epinephrine (adrenaline)
2. Estradiol
3. Glucagon
4. Cortisol

42.95 Which of the following processes is (are) stimulated by insulin:


1. Transfer of glucose across cell membrane
2. Lipogenesis
3. Glycogenesis
4. Protein biosynthesis

42.96 Which of the following hormones is (are) involved in the regulation of concentration of calcium and
phosphate in blood:
1. Progesterone
2. Parathyroid hormone
3. Cortisol
4. Calcitonin
5. Aldosterone

42.97 The second signal messengers in hormone action is (are):


1. cAMP
2. Ca²⁺
3. Inositol triphosphate
4. cGMP

42.98 Which of the following hormones take part in calcium homeostasis:


1. Parathyroid hormone
2. Calcitonin
3. Calcitriol
4. Calmodulin

42.99 Which of the following processes is (are) the major pathway of ammonia detoxification in humans:
1. Biosynthesis of fatty acids
2. Biosynthesis of uric acid
3. Biosynthesis of glycerol
4. Biosynthesis of urea

42.100 Which of the following genetic diseases are (is) related to disorders of amino acid metabolism:
1. Albinism
2. Down syndrome
3. Phenylketonuria
4. Gaucher’s disease

42.101 What is an “essential amino acid”?


1. Those amino acids that decompose during the process of digestion and must be resynthesized by the
body
2. Each of the twenty common amino acids found in proteins
3. Those amino acids which undergo transamination to form intermediates of the glycolytic and citric acid
cycles
4. An amino acid that must be furnished in the diet to support normal growth and development

lsmusis.lsmuni.lt/Klausimai/Spausdinti?Length=0?Kalba=EN&KategorijaId=144&Kalbos_input=EN&Kalbos=EN&KategorijaEn_input=biochemistry&… 11/13
3/27/2019 LSMUSIS
42.102 Ketone bodies are synthesized in:
1. Adipose tissue
2. Intestine
3. Kidney
4. Liver

42.103 Which of the following symptoms is (are) characteristics for diabetes mellitus:
1. Hyperglycemia
2. Glucosuria
3. Ketonemia
4. Acidosis
5. Ketonuria

42.104 Which of the following compounds is (are) derived from the cholesterol:
1. Glucocorticoids and mineralcorticoids
2. 7-Dehydrocholesterol
3. Bile acids
4. Androgens
5. Estrogen

42.105 Which of the following nucleic acids take part in gene expression:
1. DNA
2. mRNA
3. tRNA
4. rRNA

42.106 Which components are directly involved in protein synthesis:


1. tRNA
2. Ribosome
3. mRNA
4. Lysosome

42.1 - D 42.2 - A 42.3 - A 42.4 - C 42.5 - D 42.6 - D 42.7 - C 42.8 - D


42.9 - D 42.10 - A 42.11 - A 42.12 - D 42.13 - B 42.14 - C 42.15 - C 42.16 - C
42.17 - B 42.18 - C 42.19 - D 42.20 - A 42.21 - D 42.22 - C 42.23 - C 42.24 - A
42.25 - D 42.26 - B 42.27 - A 42.28 - E 42.29 - C 42.30 - D 42.31 - A 42.32 - D
42.33 - B 42.34 - B 42.35 - C 42.36 - C 42.37 - D 42.38 - C 42.39 - D 42.40 - C
42.41 - D 42.42 - B 42.43 - C 42.44 - B 42.45 - A 42.46 - B 42.47 - D
42.48 42.49 42.50 42.51 42.52 42.53 42.54 42.55
1-A 1-B 1-B 1-A 1-C 1-D 1-B 1-C
2-B 2-B 2-A 2-B 2-A 2-C 2-C 2-B
3-C 3-A 3-C 3-B 3-B 3-A 3-A
4-A 4-E
5-A
42.56 42.57 42.58 42.59 42.60 42.61 42.62
1-C 1-A 1-C 1-B 1-A 1-C 1-B
2-A 2-A 2-A 2-C 2-B 2-D 2-A
3-B 3-B 3-B 3-A 3-B 3-B 3-C
4-C 4-D 4-A
42.63 42.64 42.65 42.66 42.67 42.68 42.69 42.70
1 1 1 1 1 1 1 1
2 3 2 2 2 2 2 2
3 3 3 3 3 3 3
4 4 4 4
5 5
42.71 42.72 42.73 42.74 42.75 42.76 42.77 42.78
4 1 2 1 1 4 1 1
3 4 2 2 2 2
3 3 3 3

lsmusis.lsmuni.lt/Klausimai/Spausdinti?Length=0?Kalba=EN&KategorijaId=144&Kalbos_input=EN&Kalbos=EN&KategorijaEn_input=biochemistry&… 12/13
3/27/2019 LSMUSIS
42.79 42.80 42.81 42.82 42.83 42.84 42.85 42.86
1 1 4 4 4 1 1 1
2 3 3 3 2
3 3
4
42.87 42.88 42.89 42.90 42.91 42.92 42.93 42.94
1 1 1 2 4 2 2 1
2 3 3 4 4 4 3
3
4
42.95 42.96 42.97 42.98 42.99 42.100 42.101 42.102
1 2 1 1 4 1 4 4
2 4 2 2 3
3 3 3
4 4
42.103 42.104 42.105 42.106
1 1 1 1
2 2 2 2
3 3 3 3
4 4 4
5 5

lsmusis.lsmuni.lt/Klausimai/Spausdinti?Length=0?Kalba=EN&KategorijaId=144&Kalbos_input=EN&Kalbos=EN&KategorijaEn_input=biochemistry&… 13/13
3/27/2019 LSMUSIS

Biology
I type tasks. Choose only one best answer

10.1 Cell membrane contains two types of biological molecules:


A. Proteins and lipids
B. Proteins and carbohydrates
C. Carbohydrates and lipids
D. Proteins and nucleic acids

10.2 Of what chemical elements the tenths and the hundredths portions of percent are there in a cell:
A. O, N and C
B. Zn, Cu and J
C. N, C and H
D. Fe, H and O

10.3 Each amino acid is coded by:


A. One codone
B. Two codones
C. Different number of codones
D. One nucleotide

10.4 Number of proteins that are synthesized in ribosomes depends on:


A. Ribosome
B. DNA
C. mRNA
D. tRNA

10.5 Primary sex cells of animals divide by mitosis in:


A. Maturation zone
B. Division zone
C. Growth zone
D. They don't divide by mitosis

10.6 How does agent of amoebiasis enter the human organism:


A. In transmissible way (by mosquito bitting)
B. It is droplet infection
C. With contaminated food or water
D. During sexual relations

10.7 Which material can be examined in diagnosing of amoebiasis:


A. Blood
B. Content of the duodenum
C. Discharge of cutaneuos ulcer
D. Faeces

10.8 Who forms natural reservoir of visceral leishmaniasis:


A. Fishes
B. Dogs
C. Mosquitos
D. Frogs

10.9 Who is the vector of visceral leishmaniasis:


A. Human
B. Tsetse fly
C. Sandfly
D. Mosquito

10.10 What diseas can be suspected having found protosoan parasite in the cerebral fluid:
A. Lambliasis
B. Visceral leishmaniasis
C. Sleeping sickness
D. Malaria
lsmusis.lsmuni.lt/Klausimai/Spausdinti?Length=0?Kalba=EN&KategorijaId=112&Kalbos_input=EN&Kalbos=EN&KategorijaEn_input=Biology&Katego… 1/12
3/27/2019 LSMUSIS
10.11 Who is the intermediate host of liver fluke:
A. Crustacean
B. Mollusc
C. Fish
D. Ant

10.12 What developmental stage of Opistorchis felineus is infective for human:


A. Miracidium
B. Redia
C. Sporocyst
D. Metacercaria

10.13 Which helminths belong to class Nematoda (roundworms):


A. Taenia solium
B. Diphyllobothrium latum
C. Enterobius vermicularis
D. Hymenolepis nana
E. Dicrocoelium dendriticum

10.14 By which flagellata human can be infected during the sexual intercourse with the sick person:
A. Trypanosoma gambiense
B. Leishmania tropica
C. Giardia lamblia
D. Trichomonas vaginalis

10.15 What desease can be diagnosed by microscopic examination of faeces:


A. Cutaneous leishmaniasis
B. Toxaplasmosis
C. African trypanosomosis
D. Lambliasis

10.16 By which investigation microscopic diagnosis of toxoplasmosis is possible:


A. Serological investigation of blood
B. Examination of urine
C. Examination of stools
D. Examination of cerebral fluid

10.17 Which is the way of the prevention of diphylliobothriasis:


A. Keep to a personal hygiene
B. Avoid close contact with cats and dogs
C. Do not eat improperly cooked pork
D. Do not eat improperly cooked fish

10.18 In which case Taenia saginata is diagnosed:


A. In feces - solitary proglottids, the number of uterus side branches is 17 to 34
B. In feces - groups of 5-7 proglottids, the number of uterus side branches is 7 - 12
C. In feces - pieces of strobila, rosette - shaped uterus
D. In feces - 3-4 proglottids, rosette - shaped uterus

10.19 What is the way of infection of taeniasis:


A. Eating unproperly cooked pork
B. Acidentally swallowed egg of taenia solium
C. eating unwashed vegetables
D. eating fish

10.20 Which is the way to be infected with cysticercosis:


A. Having tasted raw beef with cysts
B. Having eaten food contaminated with beef tapeworm
C. Having eaten food contaminated with pork tapeworm
D. Having tasted raw pork with cysts

10.21 By which flukes can human become infected by eating a freshwater fish:
A. Fasciola hepatica
lsmusis.lsmuni.lt/Klausimai/Spausdinti?Length=0?Kalba=EN&KategorijaId=112&Kalbos_input=EN&Kalbos=EN&KategorijaEn_input=Biology&Katego… 2/12
3/27/2019 LSMUSIS
B. Dicrocoelium dendriticum
C. Opisthorchis felineus
D. Paragonium westermanii

10.22 Which is the way of prevention of trichinosis:


A. Do not eat uninspected mutton
B. Do not eat unwashed vegetables
C. Follow the rules of personal hygiene
D. Do not eat uninspected pork

10.23 What larvae of trichinas feed on:


A. Muscle glycogen
B. Content of intestine
C. Blood
D. Lymph

10.24 Which scientist proposed the name "Genetics" for the science about inheritance and variability:
A. G. Mendel
B. N. Vavilov
C. L.Johansen
D. W.Bateson

10.25 What is the name for the sum of specific chromosome characteristics including their number, size and
shape:
A. Genotype
B. Kariotype
C. Phenotype
D. Genome

10.26 What kind of mutation is given from generation to generation during the sexual reproduction of organisms:
A. Somatic mutations
B. Generative mutations
C. Extranuclear mutations
D. Somatic and generative mutations

10.27 What is the formula for the normal female kariotype:


A. 47,XXX
B. 46,XX
C. 45,X
D. 48,XXXX

10.28 Gene of color blindness is located on X chromosome. This anomaly more often occurs in males regardless
of they have only one X chromosome. Such traits are called:
A. Sex-linked
B. Sex - limited
C. Sex - influenced
D. Partly sex-linked

10.29 What is the set of sex chromatids in primary oocyte:


A. XX
B. X
C. XXXX
D. XXYY
E. YY

10.30 Which method in human genetics corresponds to hybridological method:


A. Method of twins
B. Method of population statistics
C. Genealogical method
D. Cytogenetical method

10.31 What is nucleoid:


A. Nucleus of eucaryotic cell
lsmusis.lsmuni.lt/Klausimai/Spausdinti?Length=0?Kalba=EN&KategorijaId=112&Kalbos_input=EN&Kalbos=EN&KategorijaEn_input=Biology&Katego… 3/12
3/27/2019 LSMUSIS
B. DNA molecule in procaryotic cell
C. The main part of DNA
D. Complex of nucleotids

10.32 Which of the cell organelles are semi-autonomic:


A. Nucleus and nucleoli
B. Golgi apparatus and lysosomes
C. Mitochondria and ribosomes
D. Mitochondria and chloroplasts
E. Ribosomes

10.33 Which type of asexual reproduction is usual for Phylum Apicomplexa (Class Sporosoea)
A. Budding
B. Schizogony
C. Amitosis
D. Mitosis

10.34 The definitive host of Plasmodium vivax is


A. Dog
B. Anopheles sp. mosquito
C. Human
D. Phlebotomus sp. sand fly

10.35 Who is the definitive host of Toxoplazma gondi:


A. Human
B. Human and rodents
C. Cats
D. Birds

10.36 Reservoir hosts of Trypanosoma gambiense are:


A. Phlebotomus sp. sand flies
B. Tsetse flies
C. Antelopes
D. Humans

10.37 Which tapeworm adsorbs vitamin B12 and folic acid from the intestine of the definitive host:
A. Pork tapeworm
B. Fish tapeworm
C. beef tapeworm
D. Dwarf tapeworm
E. Sheep tapeworm

10.38 Which of the parasites is geohelminth:


A. Ascaris
B. Pinworm
C. Fish tapeworm
D. Trichina

10.39 Which of the Arthropods is the transmitter (vector) of Lyme disease:


A. Triatoma sp. bug
B. Tick
C. Dog flea
D. Anopheles sp. mosquito

10.40 Which organism is pathogenic agent of tick-born encephalitis:


A. Bacteria borrelia
B. Virus
C. Tick
D. Triatoma sp. bug

10.41 What is the purpose of the test-cross:


A. To find out zygousity of twins
B. To find out the possible types of gametes
lsmusis.lsmuni.lt/Klausimai/Spausdinti?Length=0?Kalba=EN&KategorijaId=112&Kalbos_input=EN&Kalbos=EN&KategorijaEn_input=Biology&Katego… 4/12
3/27/2019 LSMUSIS
C. To find out the phenotype of one of the parents
D. To find out the genotype of one of the parents

10.42 In meiosis the homologous chromosomes disjoin during:


A. First anaphase
B. Second anaphase
C. First telophase
D. Second telophase
E. First prophase

10.43 In meiosis the crossing-over occurs during


A. First anaphase
B. Second anaphase
C. First telophase
D. Second telophase
E. First prophase

II type tasks. For each numbered item,selct the one lettered heading that is most closely asssciated with it

10.44 How the following diseases manifest themselves:


1 - Albinism
2 - Mioplegija
3 - Amaurotic idiocy
A. Weak - mindedness, blindness caused by damage of cerebral neurones
B. skin depigmentation, increased sensitiveness to light
C. periodical paralysis due to loss of K+ ions in muscle cells

10.45 Who are the intermediate hosts for the helminths:


1 - Opisthorchis felineus
2 - Taenia solium
3 - Dicrocoelium dendriticum
A. Cyclops
B. Pigs
C. Fishes
D. Ants
E. Sheeps, goats etc.

10.46 Who are definitive hosts for the helminths:


1 - Toxoplazma gondii
2 - Taenia solium
3 - Plasmodium vivax
4 - Balantidium coli
A. Humans
B. Cats
C. Mosquitoes
D. Pigs

10.47 Which gene interaction conditions the following traits:


1 - ABO blood types
2 - Eye colour
3 - Height
4 - "Spider's" fingers, cataract
A. Polyallelism
B. Polymery
C. Pleotropy
D. Dominance

10.48 What are the structural peculiarities of these tapeworms


1 - Pork tapeworm
2 - Beef tapeworm
3 - Fish tapeworm
A. Immature proglottids posses only uterus

lsmusis.lsmuni.lt/Klausimai/Spausdinti?Length=0?Kalba=EN&KategorijaId=112&Kalbos_input=EN&Kalbos=EN&KategorijaEn_input=Biology&Katego… 5/12
3/27/2019 LSMUSIS
B. Gravid proglottid contains rosette-shaped uterus
C. Gravid proglottid contains uterus with 17-34 side branches
D. Scolex has 8 suckers
E. Scolex has 4 suckers and crown of hooks

10.49 What are the functions of the following organelles:


1 - Cenrosome
2 - Nucleoli
3 - Golgi apparatus
A. Forms ribosomes and RNA
B. Takes part in cell division
C. Forms lysosomes

10.50 Make the link between the process and it's location in the cell
1 - Transcription
2 - Translation
3 - Transport of amino acids
A. Nucleus
B. Cytoplasm
C. Ribosomes
D. Lysosomes

10.51 By which methods these helminthoses can be diagnosed:


1 - Eye cysticercosis
2 - Taeniosis
3 - Echinococcosis
A. Examination of feaces
B. Oftalmoscopy
C. Alergical reactions

10.52 By which methods these helminthoses can be diagnosed:


1 - Ascariasis
2 - Muscle cysticercosis
3 - Enterobiasis
A. X-ray examination
B. Examination of perianal area swabs
C. Examination of feaces

10.53 What group of parasites do this protozoans belong to:


1 - Plazmodium vivax
2 - Toxoplazma gondii
3 - Trichomonas vaginalis
A. Luminal parasites
B. Intracellular parasites
C. Blood parasites

10.54 Which diseases are caused by following parasites:


1 - Dwarf tapeworm
2 - Pig tapeworm
3 - Fish tapewarm
4 - Cat liver fluke
A. Taeniasis
B. Dyphilliobothriasis
C. Hymenolepiasis
D. Opisthorchiasis

10.55 Which metabolism disorders are characteristics to following diseases:


1 - Alkaptonuria
2 - Albinism
3 - Galaktosemia
4 - Phenylketonuria
A. phenylalanine does not convert into tyrosine due to the blockage of enzyme phenylhydroxilase

lsmusis.lsmuni.lt/Klausimai/Spausdinti?Length=0?Kalba=EN&KategorijaId=112&Kalbos_input=EN&Kalbos=EN&KategorijaEn_input=Biology&Katego… 6/12
3/27/2019 LSMUSIS
B. tyrosine does not convert into melanin
C. Oxidation of homogentisine acid does not occure , patient's urine becomes dark in air
D. Incomplete disintegration of lactose due to the lack of enzyme galactose - 1-phosphate-
uridiltranspherase

10.56 Which means of public prevention protect against the following helminthoses:
1 - Enterobiasis
2 - Trichinosis
3 - Trichiuriasis
A. Suitable veterinary control of meat
B. Cleanness in children institutions
C. Protection of water reservoirs against contamination

10.57 In which field of genetic research the following terms are used
1 - Proband (propositus)
2 - Concordancy
3 - Sex chromatin
4 - Coefficient of inbreeding
A. Examination of interphase nucleus
B. Population genetics
C. Method of twins (twin data)
D. Pedigree analysis

10.58 Which means of personal prevention protect against the following helminthoses
1 - Enterobiasis
2 - Trichinosis
3 - Trichuriasis
A. Do not share bed linens and towels
B. Do not eat unwashed vegetables
C. Do not eat veterinary uncontroled pork

10.59 Which materialshuold be examined in diagnosing protozoan diseases:


1 - Quartan malaria
2 - Amoebiasis
3 - Giardiasis
4 - visceral leishmaniasis
A. Content of duodenal
B. Blood
C. Feaces
D. Culture of bone marrow

10.60 What mutations condition following hereditary diseases:


1 - Klinefelters syndrome
2 - Sickle-cell anemia
3 - Lejeune or "cat-cry" syndrome
A. Deleteon of the short arm of 5-th chromosome
B. Aneuploidy
C. Gene mutation

10.61 What are the functions of the following nucleic acids


1 - DNA
2 - mRNA
3 - tRNA
4 - rRNA
A. Transfer of amino acid
B. storage of the genetic information
C. Compound part of ribosome
D. Transfer of genetic information to ribosomes

10.62 Which period of gametogenesis corresponds to developmental stage of sex cells:


1 - Maturation
2 - Growth

lsmusis.lsmuni.lt/Klausimai/Spausdinti?Length=0?Kalba=EN&KategorijaId=112&Kalbos_input=EN&Kalbos=EN&KategorijaEn_input=Biology&Katego… 7/12
3/27/2019 LSMUSIS
3 - Proliferation
A. Primary spermatocyte
B. Ovotid
C. Spermatogonia

10.63 Which of the syndromes corresponds to caryotype formula:


1 - Down syndrome
2 - Lejeune or"cat-cry" syndrome
3 - Patau syndrome
4 - Edward syndrome
A. 47,XX,+18
B. 46,XX,(del 5p)
C. 46,XY,rob(15;21)(q10;q10),+21
D. 47,XY,+13

10.64 How it is possible to become infected with the following parasites


1 - Trypanosoma gambiense
2 - Leishmania tropica
3 - Trichomonas vaginalis
4 - Giardia lamblia
A. Through dirty hands or contaminated meal
B. By bitting of sand fly
C. during the sexual intercource with the sick partner
D. By bitting of tsetse fly

10.65 Which is the way of infection of these helminthoses:


1 - Diphyllobothriasis
2 - Opisthorhiasis
3 - Ascariasis
4 - Taeniasis
5 - Taeniarhinchosis
A. Having eaten improperly boiled meat
B. Having eaten insufficiently prepared fish
C. Having druns water contaminated with eggs
D. Having eaten unwashed vegetables

10.66 Which organs and system of organs the following protozoans parasites reside in:
1 - Giardia lamblia
2 - Trichomonas vaginalis
3 - Balantidium coli
A. Mouth cavity
B. Genital paths
C. Small intestine
D. Large intestine
E. Respiratory system

10.67 What is the mode of inheritance for the following diseases


1 - Vitamin d-resistant rickets
2 - Achondroplasia
3 - Phenylketonuria
4 - Haemophilia
A. Autosomal dominant
B. Autosomal recessyve
C. Sex-linked dominant
D. Sex-linked recessyve

III type tasks. For each question there is one or more correct answers:
A – if correct answers are 1,2,3
B – if correct answers are 1 and 3
C – if correct answers are 2 and 4
D – if correct answer is 4

lsmusis.lsmuni.lt/Klausimai/Spausdinti?Length=0?Kalba=EN&KategorijaId=112&Kalbos_input=EN&Kalbos=EN&KategorijaEn_input=Biology&Katego… 8/12
3/27/2019 LSMUSIS
E – if correct are all answers above

10.68 What are parts of outer structure of chromosome:


1. Sattelite
2. Centromere
3. Arms
4. Chromoneme
5. Nukleosome

10.69 Four pairs of parents make a claim to a stole baby with blood type B. Which of the pairs can pretend to be
the baby's parents, if their blood types are following:
1. B and B
2. B and O
3. A and AB
4. A and O

10.70 A woman with blood type B, N has a child with blood type O, MN. Which man with indicated blood types
could be a father of the child:
1. B, N
2. AB, MN
3. A, N
4. O, M

10.71 In which case does the complementary principle work:


1. Linking two DNR strands between themselves
2. Linking nukleotides in DNA strand
3. In the transcription process
4. In transport of amino acids

10.72 What cause mosaic cells with different chromosome sets:


1. Nondisjunction of chromosomes during mitosis
2. Amitosis
3. Endomitosis
4. Nondisjunction of homologous chromosomes during mejosis

10.73 Which factors belong to biological mutagenic agents of human environment:


1. Clorethylamine
2. Roseola virus
3. X - rays
4. Hepatitic virus
5. Thalidamide

10.74 In which cases it is recommended to examine the fetus for hereditary diseases:
1. For all pregnant women above 35 years old
2. or all pregnant women younger than 25 years old
3. In families where child ill with chromosomal disease was born
4. In families where one of the parents is sufferinfg from ishemic heart disease (IHD)
5. In families where one of the parents is sufferinfg from cancer

10.75 Which diseases depend upon genotype than upon external environment:
1. Epilepsy
2. Manic-depressive psycosis
3. Shizophrenia
4. Scarlet fever
5. Measles

10.76 Father and son are ill with haemophilia. Mother is healthy. The son inherited the disease from:
1. Father
2. Neither of parents
3. Both parents
4. Mother

lsmusis.lsmuni.lt/Klausimai/Spausdinti?Length=0?Kalba=EN&KategorijaId=112&Kalbos_input=EN&Kalbos=EN&KategorijaEn_input=Biology&Katego… 9/12
3/27/2019 LSMUSIS
10.77 Bimembrane organelles:
1. Vacuoles
2. Mitochondria
3. Ribosomes
4. Chloroplasts

10.78 Cytokinesis means:


1. The permanent movement of cytoplasm
2. The permanent movement of organelles
3. Cell movement
4. The division of cytoplasm at the end of telophase of mitosis

10.79 If maternal cell possesses twelve chromosomes, then after meiosis daughter cells posses
1. Six chromosomes
2. Twelve chromosomes
3. Six chromatids
4. Twelve chromatids

10.80 lac - operon consists of:


1. Promoter
2. Operator
3. Structural genes
4. Regulatory gene

10.81 In DNA molecule:


1. Nitrogen bases are linked together by hydrogen bonds
2. Desoxyribose is joined with phosphate by covalent bonds
3. Nitrogen bases are joined together by hydrogen bonds
4. Nitrogen bases are joined with phosphates by hydrogen bonds

10.82 The frequency of crossing-over depends on:


1. The distance between the gene and centromere
2. Number of progeny
3. Frequency of mutations
4. The distance between the genes

10.83 Phenylketonuria (PKU) is inherited as:


1. X-linked trait
2. Holandric trait
3. Autosomal- dominant trait
4. autosomal-recessyve trait
5. Sex-dependent trait

10.84 The distinctive features of Entamoeba histolytica cysts are:


1. Four nuclei
2. Eight nuclei
3. Two chromatoid bodies
4. Lack of chromatoid bodies

10.85 Which of the following helminths belong to flukes:


1. Hymenolepis nana
2. Opisthorchis felineus
3. Taenia solium
4. Fasciola hepatica
5. Echinococcus granulosus

10.86 To which of the following helminths the direct infetion from human to human is possible:
1. Ascaris lumbricoides
2. Hymenolepis nana
3. Taenia solium
4. Enterobius vermicularis
5. Trichinella spiralis

lsmusis.lsmuni.lt/Klausimai/Spausdinti?Length=0?Kalba=EN&KategorijaId=112&Kalbos_input=EN&Kalbos=EN&KategorijaEn_input=Biology&Kateg… 10/12
3/27/2019 LSMUSIS
10.87 Which of the helminths are biohelminths:
1. Fasciola hepatica
2. Trichinella spiralis
3. Opisthorchis felineus
4. Taenia solium
5. Diphyllobothrium latum

IV type tasks. Choose only one best answer

10.88 Both Don and his maternal grandfather are ill with hemophilia. Don's wife Diana is his cousin (mother's
sister's daughter). Don and Diana have one hemophiliac son Edward, two daughters Egle and Elvira who are also
ill with hemophilia, and daughter Aiste, which is healthy, but is carrier of hemophilia gene. What is the probability of
Egle's son to be ill with hemophilia:
A. 50%
B. 100%
C. 25%

10.89 Both Don and his maternal grandfather are ill with hemophilia. Don's wife Diana is his cousin (mother's
sister's daughter). Don and Diana have one hemophiliac son Edward, two daughters Egle and Elvira who are also
ill with hemophilia, and daughter Aiste, which is healthy, but is carrier of hemophilia gene. What is the mode of
inheritance:
A. Autosomal-dominant
B. Autosomal recessyve
C. X-linked recessyve

10.90 A pregnant woman appealed to genetic counseling.Pregnancy - 4 weeks. Her first child was born with Down
syndrome. balanced 21 chromosome translocation ((45,XX, rob(21q;21q))was disclosed in her leukocytes. What is
the probability that her child will be healthy:
A. 1
B. 0,5
C. 0
D. 0,25

10.91 A pregnant woman appealed to genetic counseling.Pregnancy - 4 weeks. Her first child was born with Down
syndrome. balanced 21 chromosome translocation (45,XX, rob(21q;21q))was disclosed in her leukocytes.What is
advisable in her case:
A. Examine fetal chromosomes
B. Terminate pregnancy before the term
C. To deliver the baby

10.92 A man appealed to genetic counseling due to sterility. the following facts were established: stature above
average in height, obvious gynecomastia, disordered spermatogenesis, low IQ. What is advisable:
A. Counselling by endocrinologist
B. To examine karyotype
C. Both ( counselling by endocrinologist and to examine karyotype)
D. Counselling by gynecologist

10.93 A man appealed to genetic counseling due to sterility. the following facts were established: stature above
average in height, obvious gynecomastia, disordered spermatogenesis, low IQ. Analyses have showh that amount
of gonadotropines in urine is significally increased. Karyotype formula 47,XXY (Klinefelter syndrome). How to treat
this patient:
A. Stimulate spermatogenesis with gonadotropic hormones
B. Stimulate sexuality with androgenic hormones
C. Cure gynecomastia with surgical methods
D. There is no treatment

10.94 During the routine biochemical screening in newborn's urine phenylpyruvic acid was found. What hereditary
disease could be suspected:
A. Alkaptonuria
B. Phenylketonuria
C. Galactosemija

lsmusis.lsmuni.lt/Klausimai/Spausdinti?Length=0?Kalba=EN&KategorijaId=112&Kalbos_input=EN&Kalbos=EN&KategorijaEn_input=Biology&Kateg… 11/12
3/27/2019 LSMUSIS
10.95 During the routine biochemical screening in newborn's urine phenylpyruvic acid was found. How is advisible
to cure the child:
A. Prescribe the diet with low amuont of phenylalanine
B. Prescribe fodd without phenylalanine
C. Nourish the child with mother's milk
D. No treatment or food limitation is needed

10.1 - A 10.2 - B 10.3 - A 10.4 - B 10.5 - B 10.6 - C 10.7 - D 10.8 - B


10.9 - C 10.10 - C 10.11 - B 10.12 - D 10.13 - C 10.14 - D 10.15 - D 10.16 - A
10.17 - D 10.18 - A 10.19 - A 10.20 - C 10.21 - C 10.22 - D 10.23 - A 10.24 - D
10.25 - B 10.26 - B 10.27 - B 10.28 - A 10.29 - C 10.30 - C 10.31 - B 10.32 - D
10.33 - B 10.34 - B 10.35 - C 10.36 - C 10.37 - B 10.38 - A 10.39 - B 10.40 - B
10.41 - D 10.42 - A 10.43 - E
10.44 10.45 10.46 10.47 10.48 10.49 10.50 10.51
1-B 1-C 1-B 1-A 1-E 1-B 1-A 1-B
2-C 2-B 2-A 2-D 2-C 2-A 2-C 2-A
3-A 3-D 3-C 3-B 3-B 3-C 3-B 3-C
4-D 4-C
10.52 10.53 10.54 10.55 10.56 10.57 10.58 10.59
1-C 1-C 1-C 1-C 1-B 1-D 1-A 1-B
2-A 2-B 2-A 2-B 2-A 2-C 2-C 2-C
3-B 3-A 3-B 3-D 3-C 3-A 3-B 3-A
4-D 4-A 4-B 4-D
10.60 10.61 10.62 10.63 10.64 10.65 10.66 10.67
1-B 1-B 1-B 1-C 1-D 1-B 1-C 1-C
2-C 2-D 2-A 2-B 2-B 2-B 2-B 2-A
3-A 3-A 3-C 3-D 3-C 3-D 3-D 3-B
4-C 4-A 4-A 4-A 4-D
5-A
10.68 10.69 10.70 10.71 10.72 10.73 10.74 10.75
1 1 4 1 1 2 1 1
2 2 3 2 4 3 2
3 3 3 3
10.76 10.77 10.78 10.79 10.80 10.81 10.82 10.83
4 2 4 1 1 1 4 4
4 3 2 2
3 3
10.84 10.85 10.86 10.87
1 2 2 1
3 4 4 2
3
4
5
10.88 - B 10.89 - C 10.90 - C 10.91 - B 10.92 - C 10.93 - D 10.94 - B 10.95 - A

lsmusis.lsmuni.lt/Klausimai/Spausdinti?Length=0?Kalba=EN&KategorijaId=112&Kalbos_input=EN&Kalbos=EN&KategorijaEn_input=Biology&Kateg… 12/12
3/27/2019 LSMUSIS

Physiology
I type tasks. Choose only one best answer

38.1 The repolarization of an action potential is associated with all of the following EXCEPT
A. decreased K⁺ permeability of the cell membrane
B. loss of positive charges from inside the cell
C. return the membrane potential toward its resting value
D. outward diffusion of K⁺
E. inactivation of Na⁺ channels in the cell membrane

38.2 The K+ concentration in extracellular fluid is progressively decreased. What happens to the resting potential
and excitability of the cell?
A. The resting potential increases (hyperpolarization) and excitability rises
B. The resting potential increases (depolarization) and excitability rises
C. Hyperpolarization of the cell membrane and excitability diminishes
D. Depolarization of the cell membrane and excitability diminishes
E. There is essentially no change

38.3 In the skeletal muscle, the


A. contraction precedes the action potential
B. action potential lasts as long as the contraction
C. action potential lasts longer than the contraction
D. contraction and action potential begin at the same time (simultaneously)
E. action potential precedes the contraction

38.4 What provides most of the energy used to maintain a normal resting potential?
A. Diffusion of Cl⁻
B. The calcium pump
C. The sodium-potassium pump
D. Diffusion of Na⁺
E. Diffusion of K⁺

38.5 Which of the following events takes place at the subsynaptic membrane during the action of excitatory
transmitter substance?
A. Increase in Na⁺ permeability
B. Decrease in K⁺ permeability
C. The permeability to cations remains unchanged
D. Large anions pass through the membrane
E. Local hyperpolarization

38.6 The most important role of the gamma motoneurons is to


A. stimulate skeletal muscle fibers to contract
B. maintain Ia afferent activity during contraction of muscle
C. generate activity in Ib afferent fibers
D. detect the length of resting skeletal muscle
E. prevent muscles from producing too much force

38.7 The electroencephalogram (EEG) usually exhibits rhythmic potential fluctuations. The underlying rhythmic
neuronal activity of the cortex has its pacemaker chiefly in
A. the cortex itself
B. the thalamus
C. the reticular formation
D. the cerebellum
E. the basal ganglia

38.8 Cutaneous receptive fields are smallest in


A. the forearms
B. fingertips
C. the back
D. the neck
E. the thighs
lsmusis.lsmuni.lt/Klausimai/Spausdinti?Length=0?Kalba=EN&KategorijaId=140&Kalbos_input=EN&Kalbos=EN&KategorijaEn_input=Physiology&Kat… 1/16
3/27/2019 LSMUSIS
38.9 Which statement is INCORRECT?
A. Enterokinase activates pancreatic enzymes
B. The active form of trypsinogen plays a major role in activating other digestive enzymes
C. The term "exopeptidase" refers to the origin of the peptidase from the exocrine pancreas
D. Digestion initiated in the stomach serves to increase the rate of gastric emptying
E. Trypsin is an endopeptidase

38.10 Glomerular filtration rate would be increased by


A. constriction of the afferent arteriole
B. a decrease in afferent arteriolar pressure
C. an increase in Bowman's capsule hydrostatic pressure
D. The decrease in plasma protein concentration
E. a decrease in renal blood flow

38.11 Which of the following statements about renin is correct?


A. It is secreted by cells of the proximal tubule
B. Its secretion leads to loss of Na⁺ and water from plasma to the urine
C. Its secretion is stimulated by increased renal arterial pressure
D. It converts angiotensinogen to angiotensin I
E. It converts angiotensin I to angiotensin II

38.12 Metabolic alkalosis can be caused by all following EXCEPT


A. hypersecretion of aldosterone
B. hyperventilation
C. hypokalemia
D. hyperadministration of diuretics
E. vomiting

38.13 Closure of the aortic valve occurs at the onset of which phase of the cardiac cycle?
A. Isovolumetric contraction
B. Isovolumetric relaxation
C. Ejection from the left ventricle
D. Rapid filling of the left ventricle
E. Slow filling of the left ventricle

38.14 The greatest percentage of blood volume is found normally in the


A. heart
B. aorta
C. arteries and arterioles
D. capillaries
E. veins

38.15 The greatest resistance to blood flow is found in the


A. aorta and large arteries
B. arterioles
C. capillaries
D. venules
E. large veins

38.16 Which of the following does NOT affect coronary blood flow?
A. Stimulation of beta adrenoreceptors
B. Blockade of beta adrenoreceptors
C. Decreased arterial oxygen partial pressure
D. Increased arterial carbon dioxide partial pressure
E. Blockade of alpha adrenoreceptors

38.17 Which of the following is a property of C fibers?


A. They have the slowest conduction velocity
B. They have the largest diameter
C. They are afferent nerves from muscle spindles
D. They are afferent nerves from Golgi tendon organs
E. They are preganglionic autonomic fibers

lsmusis.lsmuni.lt/Klausimai/Spausdinti?Length=0?Kalba=EN&KategorijaId=140&Kalbos_input=EN&Kalbos=EN&KategorijaEn_input=Physiology&Kat… 2/16
3/27/2019 LSMUSIS
38.18 Which of the following characteristics or components is shared by skeletal and smooth muscle?
A. Thick and thin filaments arranged in sarcomeres
B. Troponin
C. Elevation in intracellular Ca²⁺ for excitation-contraction coupling
D. Spontaneous depolarization of the membrane potential
E. High degree of electrical coupling between cells

38.19 When activated by β-adrenergic receptors, the G-protein


A. activates phospholipase C
B. activates adenylyl cyclase
C. activates protein kinase C
D. converts guanosine diphosphate (GDP) to guanosine triphosphate (GTP)
E. stimulates the release of Ca⁺ from the sarcoplasmic reticulum (SR)

38.20 Sweat glands are innervated by


A. cholinergic parasympathetic fibers
B. cholinergic sympathetic fibers
C. adrenergic sympathetic fibers
D. adrenergic parasympathetic fibers
E. none of the above

38.21 The smallest skeletal muscle element that can be activated by a single motoneuron is:
A. muscle fiber
B. muscle spindle
C. sarcomere
D. motor unit
E. myofibril

38.22 Each of the following is an excitatory neurotransmitter in the central nervous system EXCEPT:
A. Norepinephrine (noradrenaline)
B. Glutamate
C. GABA
D. Serotonin
E. Histamine

38.23 In certain pathologic condition a sudden stretch of a skeletal muscle will result in sustained rhythmic
contractions. This phenomenon is referred to as
A. clonus
B. hyperreflexia
C. hyporeflexia
D. spasticity
E. clasp knife reflex

38.24 Increased neural activity before a skilled voluntary movement is first seen in the:
A. spinal motor neurons
B. precentral motor cortex
C. basal ganglia
D. cerebellum
E. cortical association areas

38.25 In sensory receptors


A. stimulus energy is converted into local depolarization / hyperpolarization
B. the receptor (generator) potential is self-propagating in all-or-none fashion
C. a receptor potential can be produced by only one form of energy
D. the frequency of action potential doubles when the strength of the stimulus doubles
E. serving touch sensation, constant suprathreshold stimulation causes action potentials to be generated at
a constant rate

38.26 Which one of the following sensory systems uses unmyelinated fibers to convey information to the central
nervous system?
A. Proprioception
B. Vision

lsmusis.lsmuni.lt/Klausimai/Spausdinti?Length=0?Kalba=EN&KategorijaId=140&Kalbos_input=EN&Kalbos=EN&KategorijaEn_input=Physiology&Kat… 3/16
3/27/2019 LSMUSIS
C. Vibration
D. Temperature
E. Pressure

38.27 Which characteristic of a sensory stimulus is encoded better by phasic (dynamic) receptors than by tonic
(static) receptors?
A. How strong the stimulus is
B. The type of energy producing the stimulus
C. How rapidly the stimulus is applied
D. The duration of the stimulus
E. Where the stimulus is located

38.28 Which somatosensory receptors are most sensitive to vibration?


A. Merkel disks
B. Meissner corpuscles
C. Ruffini endings
D. Pacinian corpuscles
E. Free nerve endings

38.29 Visual accommodation involves


A. increased tension on the lens ligaments
B. a decrease in the curvature of the lens
C. relaxation of the sphincter muscle of the iris
D. contraction of the ciliary muscle
E. increased intraocular pressure

38.30 Motion of the endolymph in the semicircular canals when the head is held still will result in perception of:
A. Rapid ascent in an elevator
B. Being upside down
C. Moving in a straight line
D. Continued rotation
E. Being stationary and upright

38.31 A lesion of the chorda tympani nerve would most likely result in
A. impaired olfactory function
B. impaired vestibular function
C. impaired auditory function
D. impaired taste function
E. nerve deafness

38.32 The work performed by the left ventricle is substantially greater than that performed by the right ventricle,
because in the left ventricle
A. the contraction is slower
B. the wall is thicker
C. the stroke volume is greater
D. the preload is greater
E. the afterload is greater

38.33 Movement of fluid from a capillary into tissue is increased by


A. a fall in arterial pressure
B. a rise in venous pressure
C. a rise in plasma oncotic pressure
D. constriction of the arterioles
E. closure of precapillary sphincters

38.34 In which vascular bed does hypoxia cause vasoconstriction?


A. Coronary
B. Pulmonary
C. Cerebral
D. Muscle
E. Skin

lsmusis.lsmuni.lt/Klausimai/Spausdinti?Length=0?Kalba=EN&KategorijaId=140&Kalbos_input=EN&Kalbos=EN&KategorijaEn_input=Physiology&Kat… 4/16
3/27/2019 LSMUSIS
38.35 The greatest amount of hydrogen ion secreted by proximal tubule is associated with
A. excretion of potassium ion
B. excretion of hydrogen ion
C. reabsorption of calcium ion
D. reabsorption of bicarbonate ion
E. reabsorption of phosphate ion

38.36 In human males, testosterone is produced mainly by the


A. Sertoli cells
B. seminiferous tubules
C. Leydig cells
D. epididymis
E. vas deferens

38.105 Gases such as oxygen and carbon dioxide cross the plasma membrane by:
A. Primary active transport
B. Passive diffusion through the lipid bilayer
C. A specific carrier protein
D. Secondary active transport
E. Facilitated diffusion

38.106 Which of the following is the principal transmitter released by the axon terminals of the first-order pain and
temperature fibers?
A. Enkephalin
B. Acetylcholine (ACh)
C. Substance P
D. γ-aminobutyric acid (GABA)
E. Serotonin (5-HT)

38.107 Dietary fat, after being processed, is extruded from the mucosal cells of the gastrointestinal tract into
lymphatic ducts in the form of
A. Monoglycerides
B. Diglycerides
C. Triglycerides
D. Chylomicrons
E. Free fatty acids

II type tasks. For each numbered item,selct the one lettered heading that is most closely asssciated with it

38.37 Which hormones have which functions?


1 - Secretin
2 - Motilin
3 - Gastrin
4 - Cholecystokinin
A. Stimulation of pancreatic fluid and electrolyte secretion
B. Stimulation of gastric and duodenal motor activity
C. Stimulation of gastric HCl secretion
D. Stimulation of contractile activity of the gallbladder

38.38 Which autonomic fibers have which functions?


1 - Sympathetic preganglionic fibers
2 - Sympathetic postganglionic fibers
3 - Parasympathetic preganglionic fibers
4 - Parasympathetic postganglionic fibers
A. Release acetylcholine and activate postsynaptic N cholinoreceptors
B. Release acetylcholine and activate postsynaptic M cholinoreceptors
C. Release norepinephrine (noradrenaline) and activate postsynaptic alpha and beta adrenoreceprors
D. Release acetylcholine and activate postsynaptic alpha and beta adrenoreceprors

38.112 For each of the descriptions below choose the most appropriate option (Theme: Cardiovascular control)
1 - Explain(s) why cardiac output equals venous return
2 - Regulate(s) cutaneous blood flow in line with thermoregulatory needs

lsmusis.lsmuni.lt/Klausimai/Spausdinti?Length=0?Kalba=EN&KategorijaId=140&Kalbos_input=EN&Kalbos=EN&KategorijaEn_input=Physiology&Kat… 5/16
3/27/2019 LSMUSIS
3 - Expand(s) plasma volume and raise(s) BP by increasing renal Na⁺ absorption
4 - Adjust(s) vascular resistance to match blood flow to workload
5 - Stimulate(s) an increase in heart rate during exercise
6 - Pituitary hormone which causes vasoconstriction
A. Aldosterone 3- Expand plasma volume and raise PB by increasing renal Na+ absorption
B. Antidiuretic hormone6- Pituitary hormone which causes vasoconstriction
C. Sympathetic nerves 2- Regulate(s) cutaneous blood flow in line with thermoregulatory needs
5- Stimulate(s) an increase in heart rate during exercise
D. Parasympathetic nerves
E. Angiotensin II
F. Local metabolites 4- Adjust(s) vascular resistance to match blood flow to workload
G. Length-tension relationship 1- Explain(s) why cardiac output equals venous return
H. Atrial natriuretic peptide (hormone)

38.113 For each of the descriptions below choose the most appropriate option (Theme: Cell membrane transport)
1 - This is a passive transport process which can be saturated at high substrate concentrations
2 - This relies on a solute concentration gradient to drive the movement of solvent across the plasma
membrane
3 - This allows particles and large extracellular molecules (e.g. intact proteins) to be brought into the cell
4 - This uses ATP breakdown to move substrates from low to high concentration
5 - This process is passive and tends to be slow in the case of lipophobic substrates
6 - This uses a concentration gradient for one substance to drive the transport of another
A. Diffusion
B. Osmosis
C. Carrier-mediated transport
D. Facilitated diffusion
E. Primary active transport
F. Secondary active transport
G. Endocytosis
H. Exocytosis

38.114 For each of the descriptions below choose the most appropriate option (Theme: Blood - Leukocytes)
1 - These cells promote the rapid secondary immune response
2 - These are highly phagocytic agranular cells
3 - These cells secrete antibody into the circulation
4 - These cells contribute to nonspecific immunity by destroying abnormal body cells
5 - These cells mature within the thymus
6 - These cells promote inflammation by releasing vasodilator substances
A. Neutrophils
B. Eosinophils
C. Basophils
D. B lymphocytes
E. T lymphocytes
F. Monocytes
G. Plasma cells
H. Memory cells
I. Killer cells

38.115 For each of the descriptions below choose the most appropriate option (Theme: Renal transport
mechanisms)
1 - Can concentrate urine due to the high osmolality in the renal medulla
2 - Reabsorbs Na⁺ by an aldosterone sensitive mechanism
3 - Pumps NaCl but prevents water movement
4 - Is responsible for 70% of Na⁺ reabsorption
5 - Contains fluid with an osmolality similar to plasma throughout its length
6 - Has little active transport but is freely permeable to water and electrolytes
A. Glomerulus
B. Proximal convoluted tubule
C. Descending limb of the loop of Henle
D. Ascending limb of the loop of Henle
E. Distal convoluted tubule
F. Collecting duct

lsmusis.lsmuni.lt/Klausimai/Spausdinti?Length=0?Kalba=EN&KategorijaId=140&Kalbos_input=EN&Kalbos=EN&KategorijaEn_input=Physiology&Kat… 6/16
3/27/2019 LSMUSIS
38.116 For each of the descriptions below choose the most appropriate option (Theme: Autonomic control)
1 - Slow the heart when activated
2 - Increase cardiac contractility when activated
3 - Release catecholamines in response to activation of nicotinic receptors
4 - Cause vasoconstriction when activated
5 - Cause bronchodilatation when activated
6 - Lead to accommodation for near vision when activated
A. α-adrenoceptors
B. β₁-adrenoceptors
C. β₂-adrenoceptors
D. Muscarinic receptors
E. Adrenal medullary cells
F. Adrenal cortical cells

38.117 For each of the descriptions below choose the most appropriate option (Theme: Neuromuscular
transmission)
1 - Propagates down T-tubules
2 - Ligand-gated ion channels
3 - Occurs in response to Ca²⁺ influx
4 - Open during axonal action potential
5 - Nonpropagating electrical event
6 - Releases transmitter into the neuromuscular cleft
A. Presynaptic depolarization
B. End plate potential
C. Muscle action potential
D. Vesicular exocytosis
E. Nicotinic receptors
F. Fast Na⁺ channels

38.118 For each of the descriptions below choose the most appropriate option (Theme: Hormones and
reproduction)
1 - A peptide which stimulates Sertoli cell function
2 - The dominant steroid hormone during the luteal phase of the ovarian cycle
3 - Released from the posterior pituitary
4 - Stimulates ovulation
5 - Secreted by granulosa cells
6 - Secreted by interstitial cells of Leydig
A. Estradiol
B. Testosterone
C. Progesterone
D. Luteinizing hormone
E. Follicle-stimulating hormone
F. Oxytocin

38.119 For each of the descriptions below choose the most appropriate option (Theme: Gas transport)
1 - Increases the H⁺ buffering power of hemoglobin
2 - Is decreased by a fall in Po₂ but not by anaemia
3 - Decreases the O₂ affinity of hemoglobin
4 - Results from increased HCO₃⁻ production in erythrocytes
5 - Accounts for about 25% of CO₂ transport in blood
6 - Is increased at any given Po₂ in fetal, as compared with adult hemoglobin
A. Deoxygenation
B. Haldane effect
C. O₂ saturation
D. Carbonic anhydrase
E. Cl⁻ shift
F. Carbamino group
G. Bicarbonate group
H. pH fall

III type tasks. For each question there is one or more correct answers:

lsmusis.lsmuni.lt/Klausimai/Spausdinti?Length=0?Kalba=EN&KategorijaId=140&Kalbos_input=EN&Kalbos=EN&KategorijaEn_input=Physiology&Kat… 7/16
3/27/2019 LSMUSIS
A – if correct answers are 1,2,3
B – if correct answers are 1 and 3
C – if correct answers are 2 and 4
D – if correct answer is 4
E – if correct are all answers above

38.39 In a normal man breathing quietly at rest the partial pressure of:
1. CO₂ in alveolar air is about twice that in room air
2. CO₂ in pulmonary arteries is greater than in alveolar air
3. water vapor in alveolar air is less than in room air
4. O₂ in expired air is greater than in alveolar air

38.40 Carbon dioxide is carried in the blood:


1. In combination with the hemoglobin molecule.
2. In combination with plasma proteins.
3. In physical solution in plasma.
4. By the red blood cells to a greater extent than by the plasma.

38.41 A shift of the oxygen dissociation curve of blood to the right:


1. Occurs in the pulmonary capillaries
2. Is favoured by a rise in temperature
3. Favors the passage of oxygen from alveoli to blood.
4. Favors the passage of oxygen from blood to tissues

38.42 Increasing the resistance of the afferent arteriole in the glomerulus of the kidney will decrease
1. the renal plasma flow
2. the filtration fraction
3. the oncotic pressure of the peritubular capillary blood
4. the glomerular filtration rate

38.43 The renal clearance of a substance:


1. Is inversely related to its urinary concentration U.
2. Is directly related to the rate of urine formation V.
3. Is directly related to its plasma concentration P'
4. Is expressed in units of volume per unit time.

38.44 The collecting ducts in the kidney:


1. Are responsible for most of the renal reabsorption of water.
2. Pass through a region of exceptional hypertonicity
3. Are rendered (made) impermeable to water by antidiuretic hormone (ADH),
4. Determine to a large extent the final osmolality of urine.

38.45 Aldosterone
1. Is a steroid hormone
2. Is produced mainly in the juxtaglomerular apparatus of the kidney
3. Increases sodium reabsorption by the nephron.
4. Increases potassium reabsorption by the nephron.

38.46 Gastric acid secretion is stimulated by the presence of


1. gastrin
2. acetylcholine
3. histamine
4. norepinephrine (noradrenaline)

38.47 Absorption of fat-soluble vitamins requires


1. bile
2. chymotripsin
3. pancreatic lipase
4. pancreatic amylase

38.48 The following agents increase blood calcium:


1. calcitonin
lsmusis.lsmuni.lt/Klausimai/Spausdinti?Length=0?Kalba=EN&KategorijaId=140&Kalbos_input=EN&Kalbos=EN&KategorijaEn_input=Physiology&Kat… 8/16
3/27/2019 LSMUSIS
2. vitamin D
3. phosphate
4. parathyroid hormone

38.49 Adrenocorticotropic hormone (ACTH):


1. Is produced mainly by hypothalamic neurons.
2. Output increases as the circulating cortisol level falls.
3. Has a greater effect on aldosterone than on cortisol secretion.
4. Production is increased by a hypothalamic releasing hormone.

38.50 The free Ca²⁺ concentration in the relaxed muscle is very low because
1. the outer cell membrane is impermeable to Ca²⁺
2. the contractile proteins (actin and myosin) bind Ca⁺ to themselves and thus reduce the intracellular
concentration of free Ca²⁺
3. Ca²⁺ are consumed during contraction so that at the end of the contraction the free Ca²⁺ concentration in
the cytoplasm drops to a very low value
4. a calcium pump transports Ca²⁺ into the sarcoplasmic reticulum

38.51 What is the dependence between the extracellular Ca²⁺ concentration and excitability of the cells?
1. When the extracellular Ca²⁺ concentration increases, excitability decreases
2. When the extracellular Ca²⁺ concentration increases, excitability increases
3. When the extracellular Ca²⁺ concentration decreases, excitability increases
4. When the extracellular Ca²⁺ concentration decreases, excitability decreases

38.52 Which of the following statements about rigor mortis is (are) correct?
1. Myosin heads are attached to actin
2. Myosin ATPase is inhibited by low concentration of Ca²⁺
3. ATP is absent
4. Myosin heads are detached from actin

38.53 Which of the following statements about myelinated and unmyelinated nerve fibers is (are) correct?
1. Myelinated fibers conduct an action potential faster
2. The myelin sheath has higher resistance than the membrane in unmyelinated fibers
3. Myelinated fibers conduct an action potential saltatory and unmyelinated fibers - continuously
4. The propagation of action potentials in myelinated fibers needs more energy than that in unmyelinated
fibers

38.54 Correct comparisons between skeletal and smooth muscle function include which of the following?
1. the rate of sliding of myofilaments in smooth muscle is slower
2. the propagation of excitation in smooth muscle is slower
3. the rate of ATP splitting in smooth muscle is slower
4. the latent period of contraction in smooth muscle is shorter

38.55 Which statement(s) about acetylcholine (ACh) is (are) true?


1. Its action is invariably excitatory on the postsynaptic structure
2. The transmitter released by recurrent collaterals of alpha motoneurons at their junction with inhibitory
interneurons (Renshaw cells) is ACh
3. The receptors at the neuromuscular junction are muscarinic (i.e. they can be blocked by atropine)
4. Some postganglionic sympathetic neurons release ACh

38.56 Tapping the patella tendon elicits a reflex contraction of the quadriceps muscle. During the contraction of the
quadriceps muscle,
1. the Ib afferents from the Golgi tendon organ increase their rate of firing
2. the Ia afferents from the muscle spindle increase their rate of firing
3. the alpha motoneurons innervating the muscle increase their rate of firing
4. the gamma motoneurons innervating the muscle spindles increase their rate of firing

38.57 Which of the following symptom(s) is (are) characteristic of the absence of cerebellar function?
1. Tremor at rest
2. Intention tremor
3. Hemiplegia
4. Hypotonia
lsmusis.lsmuni.lt/Klausimai/Spausdinti?Length=0?Kalba=EN&KategorijaId=140&Kalbos_input=EN&Kalbos=EN&KategorijaEn_input=Physiology&Kat… 9/16
3/27/2019 LSMUSIS
38.58 What is characteristic of the basal ganglia disorders?
1. Parkinson's disease, a syndrome caused by degeneration of nigrostriatal neurons
2. Tremor which is not present at rest but appears on commencing a movement
3. Hypokinesis
4. A marked sensory loss

38.59 Correct statements regarding rapid eye movement (REM) sleep include which of the following?
1. It is the first state of sleep entered when a person falls asleep
2. It is accompanied by loss of skeletal muscle tone
3. It is characterized by slow-wave EEG
4. It is associated with dreaming

38.60 A single receptor unit (sensory neuron) might signal a change in stimulus intensity by
1. an increase in rate (frequency) of discharge
2. a change in the latency of discharge onset
3. a decrease in rate of discharge
4. a change in the amplitude of the discharge

38.61 Which statement(s) about pain is (are) true?


1. Intense stimulation of any peripheral nerve fibers or receptor types results in pain
2. In the Brown-Séquard syndrome, which results from unilateral damage to the spinal cord, patients report
a loss of pain and thermal sensation ipsilaterally to the lesion and below the level of section
3. Pain receptors are rapidly adapting
4. Referred pain is the term given to the painful sensations that are felt in a peripheral structure, but which
are associated with trauma to the deep viscera

38.62 Light shining on the retina will result in which of the following events?
1. Synthesis of rhodopsin from vitamin A
2. Hydrolysis of cyclic GMP
3. Generation of action potentials by rod photoreceptors
4. Hyperpolarization of cone photoreceptors

38.63 Which statement(s) about hearing is (are) true?


1. Displacement of the basilar membrane leads to bending of the cilia of the hair cells and the production of
a receptor potential
2. The normal range of audilble frequencies for humans is from 50 Hz to 50 kHz (50,000 Hz)
3. For normal speech to be intelligible, we need to hear frequencies in the range approximately 250 Hz to 6
kHz (6,000 Hz)
4. There is a linear relationship between increase in sound pressure and the perceived increase in loudness

38.64 Which of the following functions of the gastrointestinal tract is (are) correctly paired with the structure which
performs that function?
1. Duodenum - active absorption of divalent ions
2. Small intestine - active absorption of bile acids and vitamin B₁₂
3. Colon - active absorption of fluids and electrolytes
4. Small intestine - absorption of chlorides and secretion of bicarbonates

38.65 Functions of pancreas include


1. neutralization of hydrochloric acid
2. secretion of enzymes that comediate with bile the digestion of fats
3. secretion of insulin and other hormones
4. secretion of intrinsic (Castle) factor

38.66 Which of the following statements about gastrin secretion is (are) correct?
1. Gastrin secretion is stimulated by acetylcholine
2. Somatostatin inhibits hydrochloric acid secretion via a direct effect on parietal cells or indirectly by
inhibiting gastrin secretion
3. Intraluminal amino acids and peptides exert a direct excitatory effect on gastrin release from G cells
4. Bombesin decreases hydrochloric acid and gastrin secretion

38.67 Which of the following statements about thrombin is (are) correct?


1. It converts fibrinogen to spontaneously polymerizing molecules

lsmusis.lsmuni.lt/Klausimai/Spausdinti?Length=0?Kalba=EN&KategorijaId=140&Kalbos_input=EN&Kalbos=EN&KategorijaEn_input=Physiology&K… 10/16
3/27/2019 LSMUSIS
2. The activation of thrombin needs an interaction with phospholipids
3. It activates a factor that catalyzes the formation of covalent cross-linkages between fibrin molecules
4. The activation of thrombin does not require Ca²⁺

38.68 Which of the following buffer systems is (are) principal in the body?
1. Hemoglobin
2. Phosphate
3. Bicarbonate
4. Sulfate

38.69 Which of the following organs has an almost absolute requirement for blood glucose as its energy?
1. Brain
2. Heart muscle
3. Erithrocytes
4. Skeletal muscle

38.70 Which of the following statements about gas exchange in peripheral tissue is (are) correct?
1. Gas diffusion paths are longer in other organs than that in the lung
2. Cyanide is poisonous because it binds to hemoglobin
3. The oxygen partial pressure in some tissues is less than 5 mmHg
4. If the oxygen supply is transiently interrupted, metabolism ceases

38.71 The medulla oblongata is important for respiration control, because it contains
1. neurons that generate the breathing rhythm
2. chemoreceptors that are directly sensitive to small changes in arterial pH
3. chemoreceptors that are sensitive to changes in cerebrospinal fluid pH
4. chemoreceptors that are sensitive to changes in arterial oxygen partial pressure

38.72 Which pressure falls during the initial phase of inspiration?


1. Alveolar pressure
2. Intra-abdominal pressure
3. Pleural pressure
4. Oxygen partial pressure in the dead space

38.73 In which part(s) of nephron urine normally can be hypotonic?


1. Collecting duct
2. Distal convoluted tubule
3. Thick ascending limb of the loop of Henle
4. Proximal convoluted tubule

38.74 Which of the following substances will be more concentrated at the end of the proximal tubule than at the
beginning of the proximal tubule?
1. Glucose
2. Creatinine
3. Na⁺
4. para-aminohippurate

38.75 Renin is released from the juxtaglomerular cells in response to


1. increased activity of the sympathetic nerves innervating the kidney
2. When afferent arterioles reduces blood pressure
3. lower-than-normal concentrations of plasma Na⁺
4. higher-than-normal concentrations of plasma K⁺

38.76 Antidiuretic hormone promotes the retention of fluids in the body by stimulating the
1. active transport of water from the renal tubules
2. active transport of Cl⁻ from the renal tubules
3. active transport of K⁺ from the renal tubules
4. permeability of the collecting duct to water

38.77 Which of the following statements about ventricular repolarization in the heart is (are) correct?
1. It begins in the same location and travels in the same direction as the depolarization wave
2. It is represented by the T wave on the ECG

lsmusis.lsmuni.lt/Klausimai/Spausdinti?Length=0?Kalba=EN&KategorijaId=140&Kalbos_input=EN&Kalbos=EN&KategorijaEn_input=Physiology&Ka… 11/16
3/27/2019 LSMUSIS
3. It is unaltered if the ventricular activation sequence is changed (i.e., His bundle branch block)
4. It normally occurs so that the integrative T wave vector has nearly the same direction as the integrative
QRS vector

38.78 During the middle of diastole


1. the mitral valve is closed
2. the second heart sound is heard
3. ventricular pressure is falling
4. aortic pressure is falling

38.79 Lymph flow is increased by


1. elevated interstitial fluid protein concentration
2. exercise
3. elevated capillary pressure
4. elevated plasma protein concentration

38.80 Which of the following hormonal changes is (are) required for ovulation?
1. A decrease in plasma progesterone concentration
2. A rising estradiol concentration in plasma
3. A fall in plasma follicle-stimulating hormone concentration
4. An increase in plasma luteinizing hormone concentration

38.81 Proteins that play an important role in the contractile activity of smooth muscle include:
1. myosin
2. calmodulin
3. actin
4. troponin

38.82 Voltage-gated membrane channels are required for the generation of:
1. an excitatory postsynaptic potential (EPSP)
2. end-plate potential
3. an inhibitory postsynaptic potential (IPSP)
4. an action potential

38.83 In the neuromuscular junction region of skeletal muscle:


1. The motor end plate is the end of the motor nerve
2. The nerve ending contains vesicles of acetylcholine
3. Spontaneous (miniature) potentials are recorded from the motor nerve
4. The neuromuscular cleft is rich in acetylcholinesterase

38.84 Beta-adrenergic receptors mediate:


1. increased myocardial contractility
2. vasodilatation
3. bronchial muscle relaxation
4. ciliary muscle contraction

38.85 Which of these characteristics describe postsynaptic potentials?


1. They are graded in amplitude
2. They decrease in size with distance
3. They can summate
4. They are produced by chemically regulated gates

38.86 Inhibitory postsynaptic responses are produced by increasing the membrane conductance to:
1. Na⁺
2. Cl⁻
3. Ca²⁺
4. K⁺

38.87 Purkinje cells of the cerebellar cortex are excited by:


1. granule cells
2. basket cells
3. climbing fibers

lsmusis.lsmuni.lt/Klausimai/Spausdinti?Length=0?Kalba=EN&KategorijaId=140&Kalbos_input=EN&Kalbos=EN&KategorijaEn_input=Physiology&K… 12/16
3/27/2019 LSMUSIS
4. Golgi cells

38.88 Primary neurons serving muscle proprioception:


1. have their cell bodies in the posterior horn of the spinal cord
2. conduct impulses at a similar rate as neurons which innervate extrafusal muscle fibers
3. synapse with secondary neurons whose axons pass up the spinal cord in the dorsal column
4. synapse with neurons which cross the midline in the brain stem

38.89 Cutaneous pain:


1. is due to excitation of receptors by chemical substances in the injured tissue
2. is due to overstimulation of receptors serving other sensory modalities
3. can be elicited more readily if the tissue has recently been injured
4. shows marked adaptation

38.90 Constriction of the pupil


1. is mediated via sympathetic nerves
2. improves visual acuity
3. increases the refractive power of the eye
4. cannot be elicited by light directed into one eye the optic nerve of which is cut

38.91 In the refracting system of the eye:


1. The lens can double the refractive power of the eye during accommodation in a young adult
2. The back surface of the lens contributes more to accommodation than the front
3. More refraction occurs at the inner surface of the cornea than at the outer surface
4. The cornea causes more refraction than the lens

38.92 Lateral inhibition in the retina


1. is mediated by horizontal and amacrine cells
2. promotes sensory adaptation
3. increases contrast in the visual field
4. increases the sensitivity of receptors

38.93 The basilar membrane of the cochlea


1. is broader at the apex of the cochlea than at the base
2. vibrates more strongly the louder the sound
3. vibrates with the same frequency as the applied sound
4. vibrations result in the generation of impulses in the auditory nerve at the same frequency as the applied
sound

38.94 Olfactory sensory cells:


1. are epithelial cells which synapse with the underlying olfactory nerves
2. can detect only substances which are volatile and fat- and water-soluble
3. relay with secondary neurons which pass to the thalamus
4. show adaptation

38.95 Which statements about the cerebral cortex are true?


1. The association areas of the cerebral cortex consist of all except the sensory receiving zones
2. The left hemisphere is dominant in most right-handed people
3. The right hemisphere is dominant in most left-handed people
4. The lips and tongue can be moved voluntarily (e.g. in eating) in spite of a lesion in Broca’s area

38.96 The ascending reticular formation:


1. is activated by collateral branches of sensory neurons transmitting impulses to the thalamus
2. sends impulses to most parts of the cerebral cortex
3. when stimulated electrically tends to increase alertness
4. transmits impulses to higher centers via multisynaptic pathway

38.97 Ventricular filling:


1. Depends mainly on contraction of the atria at rest.
2. Occurs mainly in the first half of diastole.
3. Begins during isometric ventricular relaxation.
4. Gives rise to a third heart sound in some healthy people,

lsmusis.lsmuni.lt/Klausimai/Spausdinti?Length=0?Kalba=EN&KategorijaId=140&Kalbos_input=EN&Kalbos=EN&KategorijaEn_input=Physiology&K… 13/16
3/27/2019 LSMUSIS
38.98 Increased muscle blood flow during exercise involves:
1. Vasoconstriction in other vascular beds
2. Metabolic changes produced by the contracting muscles.
3. An increase in cardiac output.
4. Active dilatation of capillaries.

38.99 Increased activity in the sympathetic nerves to heart increases the:


1. Rate of diastolic depolarization in the sinus node
2. Coronary blood flow
3. Slope of the Frank-Starling (work versus stretch) curve of the heart
4. Stroke volume of the left ventricle.

38.100 The velocity of blood flow:


1. In the veins is greater than in the venules.
2. In capillaries is low because they offer high resistance.
3. Can fall to zero in the ascending aorta during diastole.
4. In an artery falls with the haematocrit.

38.101 The Purkinje cells in the heart:


1. Conduct impulses faster than some neurons.
2. Are larger than ventricular myocardial cells.
3. Are responsible for the short duration of the QRS complex.
4. Lead to contraction of the base before the apex of the heart.

38.102 The breakdown of erythrocytes in the body yields:


1. iron, most of which is excreted in the urine
2. iron, some of which is included into the pigment bilirubin
3. erythropoietin, thereby regulating the further production of erythrocytes
4. amino acids which are available for general use

38.103 Blood platelets are important in stopping bleeding in that:


1. they can release lipids which have a role in the clotting process
2. the bleedings time tends to increase when the platelet count is low
3. on exposure to collagen they become sticky and adhere together
4. a fall in the platelet count prolongs the clotting time before the bleeding time is affected

38.104 Monocytes:
1. are phagocytic
2. originate from precursor cells in the lymph nodes
3. may migrate from the blood into tissues
4. manufacture immunoglobulins

38.108 The otolith organs are responsible for


1. Producing the vestibular-ocular reflex
2. Producing rotatory nystagmus
3. Detecting angular acceleration
4. Detecting the position of the head in space

38.109 Starling's law of the heart:


1. States that the strength of myocardial contraction is a function of the initial length of the muscle fibers
2. Could explain how stroke volume may be maintained when peripheral resistance rises
3. Could explain the fall in cardiac output when a person changes from the lying to the standing position
4. Could explain why the outputs of the left and right ventricles are equal in the long term

38.110 The respiratory center


1. Is situated in the hypothalamus
2. Sends out regular bursts of impulses to the inspiratory muscles during quiet respiration
3. Sends out regular bursts of impulses to the expiratory muscles during quiet respiration
4. Is inhibited during swallowing

38.111 Insulin
1. Stimulates the release of free fatty acids from adipose tissue
2. Facilitates entry of glucose into skeletal muscle
lsmusis.lsmuni.lt/Klausimai/Spausdinti?Length=0?Kalba=EN&KategorijaId=140&Kalbos_input=EN&Kalbos=EN&KategorijaEn_input=Physiology&K… 14/16
3/27/2019 LSMUSIS
3. Secretion tends to raise the plasma potassium level
4. Facilitates entry of amino acids into skeletal muscle

38.1 - A 38.2 - C 38.3 - E 38.4 - C 38.5 - A 38.6 - B 38.7 - B 38.8 - B


38.9 - C 38.10 - D 38.11 - D 38.12 - B 38.13 - B 38.14 - E 38.15 - B 38.16 - B
38.17 - A 38.18 - C 38.19 - B 38.20 - B 38.21 - D 38.22 - C 38.23 - A 38.24 - E
38.25 - A 38.26 - D 38.27 - C 38.28 - D 38.29 - D 38.30 - D 38.31 - D 38.32 - E
38.33 - B 38.34 - B 38.35 - D 38.36 - C 38.105 - B 38.106 - C 38.107 - D
38.37 38.38 38.112 38.113 38.114 38.115 38.116 38.117
1-A 1-A 1-G 1-D 1-H 1-F 1-D 1-C
2-B 2-C 2-C 2-B 2-F 2-E 2-B 2-E
3-C 3-A 3-A 3-G 3-G 3-D 3-E 3-D
4-D 4-B 4-F 4-E 4-I 4-B 4-A 4-F
5-C 5-A 5-E 5-B 5-C 5-B
6-B 6-F 6-C 6-C 6-D 6-D
38.118 38.119
1-E 1-A
2-C 2-C
3-F 3-H
4-D 4-E
5-A 5-F
6-B 6-C
38.39 38.40 38.41 38.42 38.43 38.44 38.45 38.46
2 1 2 1 2 2 1 1
4 2 4 2 4 4 3 2
3 3 3
4
38.47 38.48 38.49 38.50 38.51 38.52 38.53 38.54
1 2 2 4 1 1 1 1
3 4 4 3 3 2 2
3 3
38.55 38.56 38.57 38.58 38.59 38.60 38.61 38.62
2 1 2 1 2 1 4 2
4 3 4 3 4 2 4
3
38.63 38.64 38.65 38.66 38.67 38.68 38.69 38.70
1 1 1 1 1 1 1 1
3 2 2 2 2 2 3 3
3 3 3 3 3
4
38.71 38.72 38.73 38.74 38.75 38.76 38.77 38.78
1 1 1 2 1 4 2 4
3 3 2 4 2 4
3 3
38.79 38.80 38.81 38.82 38.83 38.84 38.85 38.86
1 2 1 4 2 1 1 2
2 4 2 4 2 2 4
3 3 3 3
4
38.87 38.88 38.89 38.90 38.91 38.92 38.93 38.94
1 2 1 2 4 1 1 2
3 4 3 4 3 2 4
3
38.95 38.96 38.97 38.98 38.99 38.100 38.101 38.102
2 1 2 1 1 1 1 4
4 2 4 2 2 3 2
3 3 3 3
4 4
lsmusis.lsmuni.lt/Klausimai/Spausdinti?Length=0?Kalba=EN&KategorijaId=140&Kalbos_input=EN&Kalbos=EN&KategorijaEn_input=Physiology&K… 15/16
3/27/2019 LSMUSIS
38.103 38.104 38.108 38.109 38.110 38.111
1 1 4 1 2 2
2 3 2 4 4
3 3
4

lsmusis.lsmuni.lt/Klausimai/Spausdinti?Length=0?Kalba=EN&KategorijaId=140&Kalbos_input=EN&Kalbos=EN&KategorijaEn_input=Physiology&K… 16/16
3/27/2019 LSMUSIS

Pathophysiology
I type tasks. Choose only one best answer

9.1 Clinical manifestations of extracellular fluid volume deficit are:


A. Weight loss, postural blood pressure drop, flat neck veins, dizziness and oliguria.
B. Weight loss, hypertension, bradycardia, distended neck veins.
C. Weight loss, palpitations, increased urinary output.
D. Weight loss, hypotension, distended neck veins, nausea and vomiting.

9.2 Which of the following is not an action of histamine?


A. Increases vascular permeability
B. Relaxes bronchial smooth muscle
C. Causes edema
D. Increases blood flow into affected area

9.3 What is the cause of neurogenic shock?


A. Decreased body fluid volume
B. Sympathetic overstimulation
C. Increased systemic vascular resistance
D. Massive vasodilation

9.4 Increased secretion of epinephrine during the stress response would cause:
A. Increased heart rate and arterial blood pressure
B. Bronchodilatation, dilation of blood vessels for muscles
C. Vasoconstriction in many part of the body
D. All of the above

9.5 Where usually will stick embolus from leg veins?


A. In the brain
B. In the lungs
C. In the coronary arteries
D. In each of the visceral organs

9.6 Which of the following types of shock is not related to low systemic vascular resistance?
A. Septic
B. Neurogenic
C. Anaphylactic
D. Cardiogenic

9.7 What is the unit of microcirculation:


A. Arteriole
B. Venule
C. Capillaron
D. Capillary

9.8 Which electrolyte imbalance may contribute to edema formation:


A. Hypokalemia
B. Hypercalcemia
C. Hypophosphatemia
D. Hypernatremia

9.9 Which type of hypersensitivity response is mediated by T cells?


A. Hypersensitivity type I
B. Hypersensitivity type II
C. Hypersensitivity type IIII
D. Hypersensitivity type IV

9.10 Which of the following viruses is not linked to the development of cancer:
A. Epstein- Barr virus
B. Hepatitis B virus
C. Hepatitis A virus
D. Human papillomavirus
lsmusis.lsmuni.lt/Klausimai/Spausdinti?Length=0?Kalba=EN&KategorijaId=111&Kalbos_input=EN&Kalbos=EN&KategorijaEn_input=Pathophysiology… 1/7
3/27/2019 LSMUSIS
9.11 Which of the following genes in their normal state negatively regulate cell proliferation:
A. Oncogenes
B. Tumor-suppressor genes
C. Proto-oncogenes
D. Telomeres

9.12 Which type of hypersensitivity response is mediated by IgE:


A. Hypersensitivity type I
B. Hypersensitivity type II
C. Hypersensitivity type III
D. Hypersensitivity type IV

9.13 What is the cellular mediators of inflammation:


A. Histamin and prostaglandins
B. Complement system
C. Hageman factor
D. Blood kinins system

9.14 Whish are humoral (blood plasma) mediators of inflammation:


A. Leukotrienes
B. Eosinophil chemotactic factor
C. Kinins, complement and cloting systems
D. Histamin

9.15 What happens during the sudden decrease in atmosphere pressure:


A. Gas embolism
B. Thrombosis
C. Gas saturation
D. Fat embolism

9.16 Which of the following is NOT one of the four most common causes of edema formation:
A. Decreased hydrostatic pressure in the capillaries
B. Decreased plasma oncotic pressure
C. Increased capillary membrane permeability.
D. Lymphatic obstruction.

9.17 Which of the following statements apply to metastases:


A. They are histologically different from the primary tumor
B. They spread from the primary site to only one other site
C. They may occur with either malignant or benign neoplasms.
D. They indicate a poorer prognosis than when they have not developed

9.18 Which of the following statements regarding aldosterone is TRUE?


A. It is a hormone that is secreted when plasma sodium level is decreased
B. It is a hormone that is secreted when plasma potassium level is decreased
C. Its action is to decrease the retention of sodium
D. Its action is to decrease the secretion of potassium

9.19 Which cause does not produce homeometric intracardial compensatory mechanism:
A. Arterial hypertension
B. Stenosis of heart valves
C. Atherosclerosis of aorta
D. Insufficiency of heart valves
E. Increased resistance of the arteries

9.20 Which cause does not produce heterometric intracardial compensatory mechanism:
A. Isufficiency of heart valves
B. Erythremia
C. Increase of circulatory blood volume
D. Stenosis of heart valves

9.21 A patient experiences the atrophic gastritis. Which of the following anemia associated with it?
A. Fe deficiency anemia
lsmusis.lsmuni.lt/Klausimai/Spausdinti?Length=0?Kalba=EN&KategorijaId=111&Kalbos_input=EN&Kalbos=EN&KategorijaEn_input=Pathophysiology… 2/7
3/27/2019 LSMUSIS
B. Aplastic anemia
C. Vit B12 deficiency anemia
D. Hemolytic anemia

9.22 Which of the statements is TRUE regarding water balance:


A. Isotonic fluids cause increased cellular swelling
B. Hypertonic fluid causes increased cellular swelling
C. Hypotonic fluid causes cellular swelling
D. Hypernatremia causes cellular swelling

9.23 All of the following are manifestations of end-stage renal failure EXCEPT:
A. Congestive heart failure
B. Hypokalemia
C. Pulmonary edema
D. Anemia

9.24 What type of hypoxia will develop during the excessive diarrhea:
A. Anemic
B. Histotoxic
C. Circulatory
D. Respiratory

9.25 What type of the shock is NOT associated with low blood volume:
A. Shock after bleeding
B. Cardiogenic shock
C. Shock after vomiting
D. Hypovolemic shock

9.26 Excessive amount of glucocorticoids is detected during:


A. Grave’s disease
B. Adison’s disease
C. Cushing’s syndrome
D. Goiter

9.27 Which of the following terms describes this condition of an occlusion in a blood vessel from a bolus of
circulating matter in the bloodstream:
A. Thrombosis
B. Embolism
C. Thrombophlebitis
D. Accumulation of foam cells

9.28 Diabetes insipidus results from:


A. Hyposecretion of insulin
B. Hypersecretion of insulin
C. Hyposecretion of aldosterone
D. Hypersecretion of ADH

9.29 Which pathogenetical factor is essential for the development of pernicious anemia (Addison-Birmer disease):
A. Vit. B12 deficiency in the diet
B. Problem in conjugation with gastromucoprotein (Intrinsic factor)
C. Problem in activation of vit. B12 by folic acid
D. Problem in vit. B12 transport (Transcobalamin I and II)

9.30 An autoimmune disease in which an antibody mimics the action of TSH is:
A. Myxedema
B. Cretinism
C. Acromegaly
D. Cusing's disease/syndrome
E. Grave's disease
F. Cusing's disease/syndrome

9.31 What stimulates water and sodium reabsorption in the kidney tubules:
A. Aldosteron hyposecretion
lsmusis.lsmuni.lt/Klausimai/Spausdinti?Length=0?Kalba=EN&KategorijaId=111&Kalbos_input=EN&Kalbos=EN&KategorijaEn_input=Pathophysiology… 3/7
3/27/2019 LSMUSIS
B. Aldosteron hypersecretion
C. Osmotic diuresis (in case of glucosuria)
D. Vasopresin hiposecretion

9.32 What phenomenon is caused by heavy glucoseuria:


A. Oliguria
B. Polyuria
C. Anuria
D. Hematuria

9.33 Which cause is essential for the development of Cushing’s syndrome:


A. Vasopresin hyposecretion
B. Glucocorticoids hypersecretion
C. Somatothropin hypersecretion
D. Mineralcorticoids hypersecretion

9.34 Which of the following is TRUE regarding the effects of the epinephrine circulating in the body during the
alarm stage of general adaptation syndrome:
A. Heart rate is slowing down
B. Heart’s contractility will be decreased
C. Blood vessels to skeletal muscle will constrict
D. There will be transient hyperglycemia.

9.35 What type of hypoxia will develop in case of decreased atmosphere pressure:
A. Anemic
B. Hypoxic
C. Circulatory
D. Respiratory

9.36 Which of the following is NOT one of the stages in the development of General Adaptation Syndrome:
A. Stage of alarm
B. Stage of resistance or adaptation
C. Stage of termination
D. Stage of exhaustion

II type tasks. For each numbered item,selct the one lettered heading that is most closely asssciated with it

9.37 How these factors influence development of gastric ulcer


1 - Increaced acidity of gastric juise
2 - Disorders in blood circulation of gastric mucosa
3 - Prostaglandin E1 and E2
4 - Bile acids
5 - Salicilic acid
A. Suppress
B. Enhance
C. Do not influence

9.39 Which causes are essential for coronarogenic and noncoronarogenic injury of myocardium
1 - Anemia
2 - Immune injury of myocardium
3 - Hypernatremia and hyperaldosteronism
4 - Hypoglycemia
5 - Hypoxic hypoxia
6 - Arterial hypotension
7 - Hypersecretion of catecholamines
8 - Atherosclerosis of coronary arteries
9 - Activation of sympathetic nervous system
A. Coronarogenic
B. Noncoronarogenic
C. Mixed

9.40 Which essential pathogenetical factors induce appropriate systemic changes during inflammation

lsmusis.lsmuni.lt/Klausimai/Spausdinti?Length=0?Kalba=EN&KategorijaId=111&Kalbos_input=EN&Kalbos=EN&KategorijaEn_input=Pathophysiology… 4/7
3/27/2019 LSMUSIS
1 - Fever
2 - Leucocytosis and phagocytosis
3 - Increased ESR
4 - Hyperglobulinemia
A. Decreased electrostatic charge of erythrocytes
B. Activation of leucopoesis
C. Action of pyrogens
D. Adaptive immune response

9.42 What causes indicated type of dyspnea


1 - Bronchial asthma
2 - Pulmonary emphysema
3 - Upper airways obstruction
4 - Pneumonia
5 - Anemia
A. Expiratory dyspnea
B. Inspiratory dyspnea
C. Frequent superficial respiration

III type tasks. For each question there is one or more correct answers:
A – if correct answers are 1,2,3
B – if correct answers are 1 and 3
C – if correct answers are 2 and 4
D – if correct answer is 4
E – if correct are all answers above

9.43 Complications of chronic renal failure include which of the following:


1. Anemia
2. Hyperkalemia
3. Osteomalacia
4. Pericarditis

9.44 Why does osteodystrophy develop in chronic renal failure?


1. There is decreased intestinal absorption of calcium
2. Osteoclasts are stimulated by increased parathyroid hormone secretion
3. There is decreased renal activation of vitamin D
4. There is increased renal excretion of calcium
5. Decreased serum levels of phosphate inhibit osteoblastic activity

9.45 What is responsible for the development of ascites in liver cirrhosis:


1. Kidney dysfunction
2. Decreased serum albumin levels
3. Increased blood volume
4. Portal hypertension

9.46 How changes exchange of O₂ and CO₂ between the blood and tissues in hyperbaric conditions (over 3 atm.):
1. Tissues use O₂ from oxihemoglobin
2. Tissues use pasma O₂
3. CO₂ transport from the tissues doesn't change
4. CO₂ accumulates in the tissues

9.47 What compensatory mechanisms are initiated in the initial stage of hypovolemia:
1. Increased secretion of antidiuretic hormone
2. Increased secretion by both the adrenal cortex and medulla
3. Stimulation of the sympathetic nervous system
4. Decreased secretion of renin
5. Generalized vasodilatation

9.48 Blood glucose level increases during a stress response due to the action of:
1. Epinephrine
2. Thyroid hormone

lsmusis.lsmuni.lt/Klausimai/Spausdinti?Length=0?Kalba=EN&KategorijaId=111&Kalbos_input=EN&Kalbos=EN&KategorijaEn_input=Pathophysiology… 5/7
3/27/2019 LSMUSIS
3. Cortisol
4. Growth hormone

9.49 Which of the following is positive effect of arterial hyperemia?


1. Better blood supply
2. Isolation of microorganisms
3. More oxygen
4. Hypoxia develops
5. Decrease local metabolism

9.50 What is the cause of neurogenic shock:


1. Decreased blood volume
2. Increased systemic vascular resistance
3. Sympathetic overstimulation
4. Massive vasodilation

9.51 Which of the following are NOT an actions of histamine:


1. Causes edema
2. Decreases vascular permeability
3. Increases blood flow into affected area
4. Relaxes bronchial smooth muscle

9.52 In which cases glomerular filtration rate decreases:


1. When excessive burns are present
2. During edema development
3. During hiperproteinemia
4. During the resorption of transudates

9.53 In which cases glomerulal filtration rate increases:


1. When glomerular capillary hidrostatic pressure increases
2. During resorption of transudates
3. When plasma oncotic pressure decreases
4. When hidrostatic pressure in glomerular capsule increases

9.54 Which of the following statements applies to diabetes mellitus Type I:


1. It is an autoimmune disorder
2. Onset is usually in childhood and adolescence
3. There is absolute insulin deficiency
4. Individuals with the disease are usually obese
5. The onset of manifestations is gradual

9.55 Which of the following is positive effect of venous hyperemia:


1. Isolation of microorganisms
2. Hypoxia develops
3. Migration of leukocytes
4. Better blood supply

9.56 Which of the following stimulate Renin-angiotensin-aldosteron system:


1. Decrease blood flow to kidney
2. Excess of extracellular fluid volume in the body
3. Decrease of extracellular fluid volume in the body
4. Increase blood flow to kidney

9.57 Venous hyperemia can be cause by:


1. Pressing on the vein
2. Stick of the embolus in the artery
3. Thrombosis of the vein
4. Constriction of smooth muscles of the artery

9.58 What is responsible for the development of edema in kidney falure when proteinuria is present:
1. Sodium retension
2. Low serum albumins
3. Decreased fluid filtration
lsmusis.lsmuni.lt/Klausimai/Spausdinti?Length=0?Kalba=EN&KategorijaId=111&Kalbos_input=EN&Kalbos=EN&KategorijaEn_input=Pathophysiology… 6/7
3/27/2019 LSMUSIS
4. Hypovolemia
5. Portal hypertension

9.59 Which phenomenon is seen during polycitemic hypovolemia:


1. Relative polycitemia
2. Decreased hematocrit
3. Increased hematocrit
4. Absolute polycitemia

9.60 Which of the following types of shock is related to low systemic vascular resistance:
1. Septic
2. Neurogenic
3. Anaphylactic
4. Cardiogenic

9.61 Which of the following statements regarding leukemic blast cells are true?
1. The presence of blast cells is used to confirm the diagnosis of leukemia
2. They are much more abundant in acute leukemia than in chronic leukemia
3. They may infiltrate lymphoid and other body tissues
4. Blast cells are functionally normal

9.62 Which features are typical for primary aldosteronism (Conn's syndrome):
1. Arterial hypertension
2. Hypokalemis
3. Hypernatremia
4. Hypersecretion of renin

9.1 - A 9.2 - B 9.3 - D 9.4 - D 9.5 - B 9.6 - D 9.7 - C 9.8 - D


9.9 - D 9.10 - C 9.11 - B 9.12 - A 9.13 - A 9.14 - C 9.15 - A 9.16 - A
9.17 - D 9.18 - A 9.19 - D 9.20 - D 9.21 - C 9.22 - C 9.23 - B 9.24 - C
9.25 - B 9.26 - C 9.27 - B 9.28 - D 9.29 - B 9.30 - B 9.31 - B 9.32 - B
9.33 - B 9.34 - D 9.35 - B 9.36 - C
9.37 9.39 9.40 9.42
1-B 1-B 1-C 1-A
2-B 2-B 2-B 2-A
3-A 3-B 3-A 3-B
4-B 4-B 4-D 4-C
5-B 5-B 5-C
6-B
7-B
8-A
9-B
9.43 9.44 9.45 9.46 9.47 9.48 9.49 9.50
1 1 2 2 1 1 1 4
2 2 4 4 2 3 3
3 3 3
4
9.51 9.52 9.53 9.54 9.55 9.56 9.57 9.58
2 1 1 1 1 1 1 1
4 2 2 2 3 3 3 2
3 3 3 3
9.59 9.60 9.61 9.62
1 1 1 1
3 2 2 2
3 3 3

lsmusis.lsmuni.lt/Klausimai/Spausdinti?Length=0?Kalba=EN&KategorijaId=111&Kalbos_input=EN&Kalbos=EN&KategorijaEn_input=Pathophysiology… 7/7
3/27/2019 LSMUSIS

Microbiology
I type tasks. Choose only one best answer

35.1 What statement concerning the human papillomaviruses (HPV) is not correct:
A. Size 40—60 nanometers
B. Cause warts
C. Can be easy grown in cell cultures
D. They are DNA viruses
E. Associated with cervical cancer

35.2 What the substance of microbes can join the Fc fragment of immunoglobulin molecules:
A. Gram positive bacteria's endotoxins
B. Coagulase of staphylococci
C. Lipoteichoic acid of gram-positive bacteria's
D. M protein of streptococci
E. A protein of staphylococci

35.3 What the statement concerning cytomegalovirus (CMV) is correct:


A. CMV cause rabies
B. CMV are RNA viruses
C. CMV cause only local respiratory tract diseases
D. CMV are herpesviruses
E. CMV are paramyxoviruses.

35.4 What are the enzymes which help RNA oncogenic viruses to integrate into cells genome?
A. RNA dependant RNA polymerase
B. RNA dependant DNA polymerase
C. DNA dependant RNA polymerase
D. DNA dependant DNA polymerase.

35.5 Why influenza vaccine is constantly renewed:


A. According to technology
B. According to changes in antigenic structure of circulating virus in nature
C. According to increase of virulence
D. According to allergic reactions
E. Because the vaccine losses antigenicity.

35.6 What the statement concerning Mycoplasma pneumoniae is correct:


A. They do not have a cell wall
B. They are intracellular parasites
C. They do not grow in normal nutrient medium
D. They are protozoa
E. They are Mycobacteria

35.7 What the statement on pneumococci is correct:


A. Observing in smears; they form chains
B. Produce exotoxins
C. They are gram-positive microorganisms
D. They are obligate anaerobes
E. No multiply in the nutrient media

35.8 What the statement concerning meningococci is correct:


A. They are stained by Ziehl-Neelsen method
B. Can be isolated from blood in septic status
C. They are transmitted by rodents
D. They are gram-positive diplococci
E. They are representatives of normal microflora

35.9 What the statement concerning Proteus spp. is not correct:


A. They cause infections of urinary system
B. They are opportunistic pathogens of purulent infections

lsmusis.lsmuni.lt/Klausimai/Spausdinti?Length=0?Kalba=EN&KategorijaId=137&Kalbos_input=EN&Kalbos=EN&KategorijaEn_input=Microbiology&K… 1/11
3/27/2019 LSMUSIS
C. They grow by "creeping" in solid nutrient medium
D. They are normal microflora of intestines
E. They are gram-positive non-spore forming bacteria

35.10 How do you understand that immunity against A group streptococci are type specific:
A. They have the same immunity against all streptococci which produce the same haemolysins
B. The immunity are active only against A group streptococci which produce erythrogenic toxin
C. The immunity is effective against streptococci with the same C antigen
D. The immunity is effective against all A group streptococci serotypes
E. The immunity is effective only against A group streptococci with the same M protein serotype

35.11 According to which property Staphylococcus aureus can be identified:


A. The synthesis of plasma coagulase
B. Phagotype
C. Production of catalase
D. Morphology
E. Sensitivity to antibiotics

35.12 For which infectious disease is the cellular immunity the most important:
A. Tuberculosis
B. Diphtheria
C. Pneumococcal pneumonia
D. Meningococcal meningitis
E. Tonsillitis caused by A group streptococci.

35.13 How to assess the finding of α haemolytic streptococci in pharynx:


A. That are microbes of nosocomial infection
B. That are microbes which cause tonsillitis
C. That are microbes of normal microflora of pharynx
D. They are saprophytes because in pharynx they indicate very rarely
E. That's microbes which cause scarlet fever

35.14 What of these microorganisms are strict anaerobes:


A. Neisseria meningitidis
B. Escherichia coli
C. Salmonella typhi
D. Corynebacterium diphtheriae
E. Clostridium perfringens

35.15 What properties of staphylococci are the most important for their pathogenicity?
A. The presence of microcapsule
B. Sensitivity to some antibiotics
C. Synthesis of pigments
D. They disperse in smears according to grapelike clusters
E. Synthesis of plasma coagulase

35.16 What's the essence of rush in scarlet fever:


A. Accumulation of bacteria in skin
B. High temperature
C. Reaction to erythrogenic toxin
D. Reaction to enterotoxin
E. None of the above

35.17 What the disease is caused by Mycoplasma:


A. Pneumonia
B. Actinomycosis
C. Candidiasis
D. Tuberculosis
E. Hepatitis

35.18 For which disease prevention the attenuated vaccines are used:
A. Botulism

lsmusis.lsmuni.lt/Klausimai/Spausdinti?Length=0?Kalba=EN&KategorijaId=137&Kalbos_input=EN&Kalbos=EN&KategorijaEn_input=Microbiology&K… 2/11
3/27/2019 LSMUSIS
B. Whooping cough
C. Tuberculosis
D. Tetanus
E. Meningococcal infection

35.19 Which of these statements concerning the mechanism of antibiotics produced effect is incorrect:
A. Vancomycin inhibits the synthesis of peptidoglycan
B. Quinolones (e.g. ciprofloxacin) inhibit the DNA gyrase of bacteria
C. Erythromycin, as well as detergents, injures the cytoplasmic membrane of bacteria
D. Aminoglycosides inhibit the synthesis of prokaryotic proteins

35.20 What prophylaxis toxoid is used for:


A. Cytomegalic infections
B. Enteric fever
C. Whooping cough
D. Diphtheria
E. Poliomyelitis

35.21 What microorganisms don't produce spores?


A. Clostridia of gaseous gangrene
B. Clostridium tetani
C. Bacilli of anthrax
D. Corynebacterium diphtheriae
E. Clostridium botulinum

35.22 What are gram-negative diplococci indicated in liquor?


A. Streptococcus pneumoniae
B. Neisseria meningitidis
C. Escherichia coli
D. Hemophilus influenzae
E. None of them

35.23 What are the methods for identification of hospital strains?


A. Microscopy of Gram stained smears of isolated strains
B. According to antigenic structure of staphylococci strains
C. By phagotyping of strains
D. According to sensitivity of staphylococci to polyvalent bacteriophage
E. According to toxigenicity

35.24 Which of the mechanisms is of greatest importance to the virulence of Streptococcus pneumoniae capsule:
A. Capsule protects streptococcus from dehydration
B. Capsule blocks phagocytosis
C. Capsule blocks chemotaxis
D. Capsule has a collagenase activity and it is the factor of invasion into the tissues

35.25 Which of the defence mechanisms against pneumococcus polysaccharides is effective:


A. Sensibilisation of T- lymphocytes
B. Enzymes, which destroy polysaccharides of a capsule
C. Anti-capsular antibodies
D. Activation of macrophages

35.26 Which one of the pathogens of intestine infections causes bacteraemia:


A. Shigella flexneri
B. Campylobacter jejuni
C. Vibrio cholerae
D. Salmonella typhi
E. Shigella sonnei

35.27 Which of the statements concerning clostridia is incorrect:


A. Pathogenic clostidria are found in the soil and they are the normal microflora of the intestine
B. When antibiotics are used not rational, pseudomembranous colitis is caused by the exotoxins, produced
by Clostridium difficile.

lsmusis.lsmuni.lt/Klausimai/Spausdinti?Length=0?Kalba=EN&KategorijaId=137&Kalbos_input=EN&Kalbos=EN&KategorijaEn_input=Microbiology&K… 3/11
3/27/2019 LSMUSIS
C. The anaerobic conditions in the wound for the development of tetanus are not necessary, because
Clostridium tetani spores are formed in the environment of atmospheric oxygen.
D. Wound contamination with Clostridium botulinum spores can cause the wound disease

35.28 Which of the statements concerning gram-negative bacteria is incorrect:


A. Escherichia coli is a part of autochtonous microflora of large intestines
B. Escherichia coli does not cause diarrhoea
C. Klebsiella pneumoniae is a part of the normal intestinal microflora and it may cause an infection of
respiratory tract.
D. Bacteria of Proteus genus are very motile and they may cause an infection of urinary tract.

35.29 Which of the phenomenon is not related to B group Streptococcus agalactiae:


A. It causes impetigo
B. It is possible to detect this streptococcus in the vagina of 5 – 25% of young healthy women
C. It causes sceptical meningitis to new-borns
D. It causes β hemolysis of erythrocytes in nutrient media

35.30 Which of the indicated bacteria do not cause diarrhoea:


A. Clostridium perfringens
B. Enterococcus faecalis
C. Escherichia coli
D. Vibrio cholerae

35.31 Which of the indicated bacteria do not cause infections of urinary tract:
A. Escherichia coli
B. Proteus mirabilis
C. Klebsiella pneumoniae
D. Bacteroides fragilis

35.32 Which of the statements concerning the mechanism of endotoxin effect is incorrect:
A. Endotoxin is opsonine
B. Endotoxin is pyrogenic
C. Endotoxin makes the cascades of coagulation more active
D. Endotoxin is the reason of hypotension

35.33 Which of the statements concerning the effect of antibodies activity on bacteria is incorrect:
A. Antibodies activate complement, that cause lysis of gram-negative bacteria
B. Antibodies facilitate phagocytosis
C. Antibodies increase the frequency of lysogenesis
D. Antibodies block bacteria adhesion on mucous membranes

35.34 Which of the statements concerning the mechanisms of bacteria destruction is incorrect:
A. Lysozyme hydrolyses cell wall of bacteria
B. Silver nitrates inactivate bacterial enzymes
C. Detergents destroy bacterial membranes
D. Ultraviolet rays destroy bacterial capsule

35.35 Which of the following statements describe passive acquired immunity:


A. Induces long-term immunity against some infectious diseases.
B. May be used to prevent disease after a known exposure.
C. Increases the amount of memory cells and protect against re-infection.
D. Causes desensitisation of organism by suppressing of mast cell degranulation.
E. It formed after injection of the protective antigens of the microbes.

35.36 Which of the statements concerning MCH II class proteins is incorrect:


A. They are on the surface of T lymphocytes
B. They are very polymorphic molecules
C. They participate in the antigen’s presentation by macrophages
D. They participate in the interaction with CD4 cell receptors

35.37 Which of the statements concerning the mechanism of viral infections pathogenesis is incorrect:
A. Viruses cause a cytopatic effect in the infected cells
B. Viruses may cause a transformation of infected cells
lsmusis.lsmuni.lt/Klausimai/Spausdinti?Length=0?Kalba=EN&KategorijaId=137&Kalbos_input=EN&Kalbos=EN&KategorijaEn_input=Microbiology&K… 4/11
3/27/2019 LSMUSIS
C. Viruses cause immunopathological reactions
D. Viruses produce exotoxin, which actyvates the adenilatcyclase

35.38 Which of the statements concerning the antibodies effect is correct:


A. Antibodies activate lysozyme, which destroys the cell wall of bacteria
B. Antibodies increase proteolysis of exotoxins
C. Antibodies facilitate phagocytosis
D. Antibodies inhibit the synthesis of bacterial proteins

35.39 Which of the statement concerning the interferons is incorrect:


A. Interferons are proteins, which act against viruses
B. Interferons are produced only by viral infected cells
C. Interferons affect all the viruses
D. Interferons initiate an “antiviral state” in the non-infected cells
E. Interferons are not specific for viruses

35.40 Which of the statements concerning Treponema is incorrect:


A. Treponema pallidum produces exotoxin, which increase adenilate cyclase activity
B. Treponema pallidum does not grow in artificial media
C. Some species of Treponema are part of the normal microflora of nasopharynx
D. Antibodies, which react with cardiolipins from beef heart, are detected in the organisms of infected
people with Treponema pallidum

35.41 Which of the statements concerning viral vaccines is incorrect:


A. Viruses of live attenuated vaccines do not cause the disease, but they may induce the synthesis of
neutralising antibodies
B. Live attenuated vaccines may be harmful for immunocompromise people
C. Inactivated vaccines induce the IgA synthesis of mucous immunity
D. Inactivated vaccines induce the synthesis of protective IgG

II type tasks. For each numbered item,selct the one lettered heading that is most closely asssciated with it

35.42 What are the main properties of diagnostic methods?


1 - The method of serology
2 - . The method of bacteriological diagnosis
3 - The method of molecular diagnosis
A. Identification of microorganisms by determination of their antigenic structure
B. Isolation and identification of pure microbe culture
C. Detection of antibodies in blood sera
D. Detection of specific microbial polymerase in the infected cells
E. Detection of microbial nucleic acids in test material by polymerase chain reaction

35.43 Which kind of the immune response has the indicated features?
1 - Immune response develops faster; the duration of immunity is longer
2 - More of IgG are synthesised during the immune response
3 - At the beginning of immune response IgM are synthesised the first
4 - A greater number of antibodies are in blood only after 7 – 10 days
A. Primary immune response
B. Secondary immune response

35.44 What the diseases are caused by these microorganisms:


1 - Epstein-Barr virus
2 - Chlamydia trachomatis
3 - Bacteroides spp.
A. Infectious mononucleosis
B. Whooping cough
C. Abscess formation
D. Urogenital tract infections
E. Meningitis

35.45 Select the factors of invasion for the next microorganisms:


1 - Streptococcus pyogenes

lsmusis.lsmuni.lt/Klausimai/Spausdinti?Length=0?Kalba=EN&KategorijaId=137&Kalbos_input=EN&Kalbos=EN&KategorijaEn_input=Microbiology&K… 5/11
3/27/2019 LSMUSIS
2 - Neisseria gonorrhoeae
A. Endotoxin
B. Streptokinase
C. Antigens of capsule
D. Flagella
E. Por protein of outer membrane

35.46 What of the next microorganisms are common in the normal human flora of:
1 - Skin
2 - Intestines
3 - Respiratory tract
A. Streptococcus pneumoniae
B. Propionibacterium acne
C. Clostridium spp.
D. Gardnerella spp.
E. Pseudomonas spp.

35.47 Which of the microorganisms cause indicated infections:


1 - Nosocomial infections and pneumonias for adults
2 - It is the most frequent reason of the urinary tract infections
3 - It may cause infections of urinary tract as well as it produces urease
A. Escherichia coli
B. Klebsiella pneumoniae
C. Salmonella enteritidis
D. Proteus mirabilis

35.48 Which of the microorganisms have the indicated features?


1 - Gram-negative coccobacilli, cause meningitis of children
2 - Gram-negative oxidase-positive bacteria, cause infections of burn wounds
3 - Bacteria cause atypical pneumonia in immunocompromised people
4 - Anaerobic gram-negative bacteria, cause abscess and peritonitis
A. Bacteroides fragilis
B. Haemophilus influenzae
C. Pseudomonas aeruginosa
D. Chlamydia pneumoniae

35.49 Which kind of viruses have the indicated features?


1 - DNA enveloped virus, which spreads via blood
2 - RNA enveloped virus, in which there are surface antigens of other virus
3 - RNA virus with own envelope and spreads via blood
4 - RNA virus without envelope and spreads by a faecal-oral way
5 - Surface protein of the envelope is a protective antigen for vaccination
A. Hepatitis A virus
B. Hepatitis B virus
C. Hepatitis C virus
D. Hepatitis D virus

35.50 What cells have the next functions?


1 - CD4 lymphocytes
2 - CD8 lymphocytes
3 - Macrophages
A. Delayed type hypersensitivity reactions
B. Cytotoxic effect
C. Immediate hypersensitivity or allergic reactions
D. The presentation of the antigen
E. The degranulation of basophils

35.51 Which of the cells have the indicated features?


1 - It synthesises interleukin–1
2 - Interleukin–1 it’s activates
3 - It synthesises interleukin-2
4 - It expresses class I MHC molecules
lsmusis.lsmuni.lt/Klausimai/Spausdinti?Length=0?Kalba=EN&KategorijaId=137&Kalbos_input=EN&Kalbos=EN&KategorijaEn_input=Microbiology&K… 6/11
3/27/2019 LSMUSIS
5 - It expresses class II MHC molecules
6 - In the lymphocyte’s membrane it expresses immunoglobulin’s receptors for antigens
A. T lymphocytes
B. B lymphocytes
C. Macrophages
D. B lymphocytes and macrophages
E. T and B lymphocytes, macrophages

35.52 What are the main properties of these vaccines?


1 - Vaccine against diphtheria
2 - Vaccine against tuberculosis
3 - Vaccine against Hepatitis B virus
A. Live vaccine from viruses not virulent for human
B. Inactivated toxoid vaccine
C. Live vaccine from attenuated mutants
D. Inactivated vaccine from killed microbes
E. Subunit vaccine from protective antigen

35.53 What of the next properties appopriate to the microorganism:


1 - Helicobacter pylori
2 - Mycoplasma hominis
3 - Gardnerella vaginalis
A. They are Protozoa
B. They are microaerophils
C. They do not have a cell wall
D. Cause pneumonia
E. They are polymorphic, gram-positive or gram-variable rods

35.54 Which class of the MHC molecules participate in the indicated processes:
1 - In the process of antigen presentation to CD4 lymphocytes
2 - In the process of antigen presentation to CD8 lymphocytes
3 - In the Thymus-dependent immune response to the antigens
4 - In the process of cell target detection by the cytotoxic T lymphocytes
A. I class MHC molecules
B. II class MHC molecules

III type tasks. For each question there is one or more correct answers:
A – if correct answers are 1,2,3
B – if correct answers are 1 and 3
C – if correct answers are 2 and 4
D – if correct answer is 4
E – if correct are all answers above

35.55 What the main properties of Rhabdoviridae:


1. They are RNA viruses
2. They are rabies viruses
3. . Exist effective vaccine for prophylaxis
4. They are DNA viruses

35.56 What is the coagulase of bacteria?


1. The factor of pathogenicity
2. Blood sera coagulase, released by stimulus of staphylococci
3. Enzyme synthesized by bacteria, cause coagulation of animal plasma
4. Protease synthesized by bacteria coagulating milk proteins

35.57 What properties have exotoxins:


1. Potent toxigenicity
2. Antigenicity
3. Immunogenicity
4. Termostability

35.58 What properties are important for S.typhi:


lsmusis.lsmuni.lt/Klausimai/Spausdinti?Length=0?Kalba=EN&KategorijaId=137&Kalbos_input=EN&Kalbos=EN&KategorijaEn_input=Microbiology&K… 7/11
3/27/2019 LSMUSIS
1. They are gram-negative bacteria
2. They are monopathogenic
3. During bacteremia they can enter into bone morrow
4. They are strict aerobes

35.59 What properties have pathogenic staphylococci:


1. Produce coagulase
2. Synthesize penicillinase and become resistant to penicillin
3. The carriers of them may be healthy human
4. The sick man cannot be the source of infection

35.60 What is haemoculture:


1. The culture of microorganisms isolated from blood of patient with any infectious disease
2. The culture of microorganisms isolated in blood agar nutrient medium
3. The culture of microorganisms isolated from blood in case of sepsis
4. The culture of microorganisms producing haemolysins in blood agar

35.61 What are the properties of erythrogenic (pyrogenic) toxin of streptococci:


1. It is produced by A group β haemolytic strains of streptococci
2. It is haemolysin of streptococci
3. It's associated with skin rush in scarlet fever case
4. It produces cross reaction with antibodies to all streptococci

35.62 In what cases of disease there are no antitoxins in blood sera:


1. Salmonellosis
2. Pneumonia, caused by Streptococcus pneumoniae
3. Botulism
4. Diphtheria

35.63 What microorganisms are not cultivated in nutrient media?


1. Mycobacterium spp.
2. Treponema pallidum
3. Mycoplasma spp.
4. Chlamydia spp.

35.64 What viruses have only one serotype?


1. Measles
2. Rubella
3. Parotitis
4. Adenovirus

35.65 What is used for rabies prophylaxis?


1. The inactivated viruses from diploid cell culture
2. Attenuated vaccine
3. Vaccine used by inoculating into infected organism
4. Pasteur's vaccine

35.66 What are prions:


1. Defected, without nucleic acid “viruses”
2. Defected, without capsid viruses
3. The latent "virus" infection
4. The latent protozoa infection agents of central nervous system

35.67 What statements concerning Chlamydia trachomatis are correct?


1. They are intracellular parasites
2. They cause urogenital tract infections
3. They cause the diseases of eyes
4. The humans can be infected from sick animals

35.68 For what disease diagnosis the allergic skin test can be used:
1. Tuberculosis
2. Brucellosis
3. Leprosy
lsmusis.lsmuni.lt/Klausimai/Spausdinti?Length=0?Kalba=EN&KategorijaId=137&Kalbos_input=EN&Kalbos=EN&KategorijaEn_input=Microbiology&K… 8/11
3/27/2019 LSMUSIS
4. Diphtheria

35.69 What are the properties of ECHO viruses?


1. Genome — single stranded RNA
2. They are enteroviruses
3. The human may be carrier of ECHO viruses
4. They are Hepatitis A viruses

35.70 What are the Reoviridae properties?


1. Genome — double stranded fragmented RNA
2. They are rotaviruses
3. They cause respiratory virus infections
4. Genome — double stranded DNA

35.71 What are the components of blood sera which produce immunity to B hepatitis:
1. HBsAg
2. HBcAg
3. Antibodies against HBcAg
4. Antibodies against HBsAg

35.72 What statements concerning Hepatitis are A viruses are correct:


1. They are Picornaviridae
2. They are RNA viruses
3. They are enteroviruses
4. They are DNA viruses

35.73 What is bifidumbacterin:


1. That — lyophilized, live, anaerobes, gram-positive bacteria of normal human microflora
2. That — lyophilized, killed, anaerobes, gram-positive bacteria of normal human microflora
3. That — the medicine for disbacteriosis treatment
4. That — the allergen for disbacteriosis diagnosis

35.74 What are characteristics of BCG vaccine:


1. After inoculation produce antibodies against Mycobacteria
2. Acid fast aerobic, rod shaped non sporeforming bacteria
3. Tuberculosis bacteria inactivated by formalin
4. After inoculation produce cellular immunity

35.75 What pathological material for Hepatitis B virus is investigated?


1. Blood
2. Sperm
3. The punctate of the liver
4. Feaces (stool)

35.76 What pathological material for Hepatitis A virus is investigated?


1. Feaces (stool)
2. Blood
3. The punctate of the liver
4. Sperm

35.77 What statements concerning Clostridium spp. are correct?


1. Produce exotoxins
2. They are strict anaerobes
3. Produce spores
4. Acid fast rods

35.78 What statements concerning Hepatitis B viruses are correct:


1. They are DNA viruses
2. They spread by parenteral way
3. They have HBsAg
4. They spread by fecal-oral way

35.79 What the components of bacteria are adhesive factors:


lsmusis.lsmuni.lt/Klausimai/Spausdinti?Length=0?Kalba=EN&KategorijaId=137&Kalbos_input=EN&Kalbos=EN&KategorijaEn_input=Microbiology&K… 9/11
3/27/2019 LSMUSIS
1. The lipoteichoic acids of streptococci
2. Streptococcus pyogenes M protein
3. Neisseria gonorrhoeae pili
4. Escherichia coli A lipid

35.80 What the procedures for Corynebacterium diphtheriae identification are necessary:
1. The differentiation from diphteroids by biochemical activity
2. Detection toxigenicity in vitro
3. Way of growth in diagnostic media determination
4. Staining by Neisser or Gram methods and morphology determination

35.81 What types of vaccines are dangerous for persons with immunodeficiency
1. Attenuated vaccine of influenza virus
2. Attenuated vaccine of Rubella virus
3. Per oral vaccine of poliomyelitis virus
4. Killed vaccine of influenza virus

35.82 What statements for Mycobacterium tuberculosis are correct?


1. They are bacteria
2. They have long generation time; pure culture can be isolated only after 2- – 3 weeks
3. Stained by special method of Ziel-Neelsen
4. Belong to the fungi species

35.83 What statement(s) concerning disbacteriosis is(are) correct?


1. That's the status of sterile organism
2. The presence of bacteroids in the enteral tract
3. . The treatment of disbacteriosis — by wide spectrum antibiotics
4. There is disbalance of normal organism microflora

35.84 What is specific feature for diphtheria?


1. Immunity depends from antitoxic antibodies in blood
2. Only human is the host
3. Toxin is produced only by lysogenic strains with tox+ gene
4. Exotoxin is identical for all biotypes of bacteria

35.85 What statements concerning HIV (Human immunodeficiency virus) are correct?
1. HIV belong to Retroviridae
2. HIV is DNA virus
3. HIV is integrative
4. HIV is oncogenic

35.86 What are the express and effective methods for virus diseases diagnosis?
1. Electron microscopy of test material
2. Detecting of virus antigens by sensitive immunological assays
3. Cultivation of viruses in tissue cultures
4. The methods of molecular hybridization

35.87 What statements of acquired immunity are correct?


1. Phagocytosis and natural killers
2. Immunoglobulins
3. Complement system
4. T lymphocytes-effectors

35.88 Which statements concerning B group streptococci are correct?


1. They are in the normal microflora of vagina
2. They are resistant to bacitracine
3. Positive agglutination with antibodies to Streptococcus agalactiae
4. They cause sepsis and meningitis for new-borns

35.89 The virulence factors of Streptococcus pneumoniae are:


1. C-reactive protein (CRP)
2. Flagella
3. Endotoxin
lsmusis.lsmuni.lt/Klausimai/Spausdinti?Length=0?Kalba=EN&KategorijaId=137&Kalbos_input=EN&Kalbos=EN&KategorijaEn_input=Microbiology&… 10/11
3/27/2019 LSMUSIS
4. Capsule polysaccharides

35.1 - C 35.2 - E 35.3 - D 35.4 - B 35.5 - B 35.6 - A 35.7 - C 35.8 - B


35.9 - E 35.10 - E 35.11 - A 35.12 - A 35.13 - C 35.14 - E 35.15 - E 35.16 - C
35.17 - A 35.18 - C 35.19 - C 35.20 - D 35.21 - D 35.22 - B 35.23 - C 35.24 - B
35.25 - C 35.26 - D 35.27 - C 35.28 - B 35.29 - A 35.30 - B 35.31 - D 35.32 - A
35.33 - C 35.34 - D 35.35 - B 35.36 - A 35.37 - D 35.38 - C 35.39 - B 35.40 - A
35.41 - C
35.42 35.43 35.44 35.45 35.46 35.47 35.48 35.49
1-C 1-B 1-A 1-B 1-B 1-B 1-B 1-B
2-B 2-B 2-D 2-E 2-C 2-A 2-C 2-D
3-E 3-A 3-C 3-A 3-D 3-D 3-C
4-A 4-A 4-A
5-B
35.50 35.51 35.52 35.53 35.54
1-A 1-C 1-B 1-B 1-B
2-B 2-A 2-C 2-C 2-A
3-D 3-A 3-E 3-E 3-B
4-E 4-A
5-D
6-B
35.55 35.56 35.57 35.58 35.59 35.60 35.61 35.62
1 1 1 1 1 1 1 1
2 3 2 2 2 3 3 2
3 3 3 3 3
35.63 35.64 35.65 35.66 35.67 35.68 35.69 35.70
2 1 1 1 1 1 1 1
4 2 3 3 2 2 2 2
3 3 3 3 3
35.71 35.72 35.73 35.74 35.75 35.76 35.77 35.78
4 1 1 2 1 1 1 1
2 3 4 2 3 2 2
3 3 3 3
35.79 35.80 35.81 35.82 35.83 35.84 35.85 35.86
1 1 1 1 4 1 1 2
3 3 2 2 2 3 4
3 3 3
4
35.87 35.88 35.89
2 1 4
4 2
3
4

lsmusis.lsmuni.lt/Klausimai/Spausdinti?Length=0?Kalba=EN&KategorijaId=137&Kalbos_input=EN&Kalbos=EN&KategorijaEn_input=Microbiology&… 11/11
3/27/2019 LSMUSIS

Pathological anatomy
I type tasks. Choose only one best answer

15.1 Which function isn’ t characteristic for lymphocytes


A. Cytolysis
B. Activation of the B lymphocytes
C. Suppression of the T lymphocytes
D. Apprehension and adaptation of the antigen
E. Production of a lymphokines

15.2 Human immunodeficiency virus (HIV) destroys indicated cells


A. T killers
B. B lymphocytes
C. T helpers
D. Plasmocytes
E. Neutrophils

15.3 AIDS patients most frequently die from complications, related, to indicated processes, except
A. Cytomegalovirus infection
B. Sarcoma Kaposi’s
C. Pneumococcal pneumonia
D. Tuberculosis
E. Lymphoma

15.4 Arterial hypertension is not characteristic to indicated pathological situation


A. Cushing syndrome
B. Pheochromocytoma
C. Hyperplasia of the adrenal medullae
D. Neoplasia of the pancreas

15.5 Which cause of death is most frequent in patients with fibrocavernous tuberculosis:
A. Chronic insufficiency of the heart et lungs
B. Bleeding from lungs
C. Amyloidosis
D. Progressing of the tuberculous processus

15.6 In which organ may develop “ cold abscessus” in case of tuberculosis:


A. Lung
B. Kidney
C. Ovary or testis
D. Vertebral column
E. Knee joint

15.7 In which cellular structure may be found virus inclusions:


A. Cytoplasm
B. Nucleus
C. Cytoplasm and nucleus

15.8 Which substance is produced by the cell after virus penetration:


A. Serotonine
B. Interferon
C. Bradikinin
D. Lecithin

15.9 Which cause of the infarction in organs is most frequent:


A. Compression of the artery
B. Spasms of the artery
C. Thrombus of the artery
D. Inflammation of the arterial wall
E. Aneurysm of the artery

15.10 Which pathology is characteristic to acute ischaemic heart disease status:


lsmusis.lsmuni.lt/Klausimai/Spausdinti?Length=0?Kalba=EN&KategorijaId=117&Kalbos_input=EN&Kalbos=EN&KategorijaEn_input=Pathological+an… 1/12
3/27/2019 LSMUSIS
A. Postinfarctional myocardial scar
B. Focal myocardial fibrosis
C. Myocardial infarction
D. Postinfarctional chronic aneurysm of the heart ventricle

15.11 Which pathological processes more frequently disorders blood circulation in the coronary arteries and
determine developing of the myocardial infarction:
A. Stenotic atherosclerosis of the coronary arteries
B. Obturative thrombosis of the coronary arteries
C. Spasm of the coronary arteries
D. Thromboembolism of the coronary arteries

15.12 In which organ develops red infarction:


A. Heart
B. Spleen
C. Lung
D. Kidney

15.13 Which case may be suspected in presence of the glomerullar capillaries thrombosis, necrosis and many “
half - moons” in kidney biopsy material:
A. Acute diffuse proliferative glomerulonephritis
B. Rapidly progressive (crescentic) glomerulonephritis
C. Membranous glomerulonephritis
D. Membranoproliferative glomerulonephritis
E. Minimal changes glomerulonephritis

15.14 Which disease may be suspected in presence of macrogranular, irregular, scary surface of the kidney:
A. Polycystic kidney disease
B. Kidney carcinoma
C. Chronic glomerulonephritis
D. Syndrome of the acute renal injury
E. Chronic pyelonephritis

15.15 Which disease may be suspected in presence of proteinuria, hypalbuminemia, spread edema and
hyperlipidemia:
A. Urolithiasis
B. Polycystic kidney disease
C. Nephrotical syndrome
D. Kidney carcinoma

15.16 Which pathological processus may be suspected in presence of diminished organs ant it’s structures:
A. Hypertrophy
B. Metaplasia
C. Atrophy
D. Hyperplasia

15.17 Which cause determine atrophy of the extremity fixating muscles:


A. Gradually diminished arterial blood supply
B. Immobility
C. Compression of the bandage
D. Disorder of the innervation
E. Deficiency of the hormonal regulation

15.18 In which tissue happens intracellular regeneration only:


A. Blood
B. Cardiomyocytes
C. Cartilagineas tissue
D. Osseous tissue

15.19 Which methods of investigation more earlier reveals adaptive changes:


A. Electron microscopy
B. Autoradiography

lsmusis.lsmuni.lt/Klausimai/Spausdinti?Length=0?Kalba=EN&KategorijaId=117&Kalbos_input=EN&Kalbos=EN&KategorijaEn_input=Pathological+an… 2/12
3/27/2019 LSMUSIS
C. Immunomorphology
D. Cytochemistry

15.20 What means phrase “ Carcinoma in situ”


A. Non spread intraepithelial carcinoma
B. Noninfiltratively growing intraepithelial carcinoma
C. Intraepithelial carcinoma formed from comparatively differentiated cells
D. Intraepithelial carcinoma still not grown through basic membrane

15.21 How and where metastase malignant stomach tumors of epithelial origin:
A. At first by blood into liver and lung
B. At first by blood into bone marrow and liver
C. At first by lymph into liver, lungs, cerebrum and bone marrow
D. At first by lymph into regional lymph nodes

15.22 Which carcinomatous metastases are called Krukenberg tumor:


A. Lymphogenic retrogradic carcinomas metastases from breast or stomach into ovaries
B. Hematogenous ortogradic metastases from breast or stomach into ovaries
C. Hematogenous retrogradic metastases from breast into ovaries
D. Lymphogenic ortogradic metastases from stomach into ovaries

15.23 Which quality isn’t characteristic for semimalignant tumors:


A. Infiltrative growth
B. Postoperative recurrence
C. Early metastasizing
D. Expansive growth
E. Slow growth
F. Late metastasizing

15.24 Fluid that collects during acute inflammation and that has a protein content exceeding 3g/dl and a specific
gravity exceeding 1.015 is referred to as:
A. Edema
B. Effusion
C. Transudate
D. Serum
E. Exudate

15.25 Which of the following cell types differentiates into morphologically distinct cells capable of immunoglobulin
production:
A. Neutrophils
B. Basophils
C. B cells
D. T cells
E. Plasma cells

15.26 An exuberant, hypertrophic, collagenous reaction that can occur following an injury is referred to as:
A. Callus
B. Cicatrix
C. Contracture
D. Keloid
E. Adhesion

15.27 The most important prognostic factor for human cancer is:
A. Tumor grade
B. Tumor stage
C. Metastasis in lymphonodes
D. Vascular invasion
E. The mitotic index

15.28 Thromboangitis obliterans occurs predominantly in people with:


A. Congenital cardiac atrial defects
B. Ischemic heart disease

lsmusis.lsmuni.lt/Klausimai/Spausdinti?Length=0?Kalba=EN&KategorijaId=117&Kalbos_input=EN&Kalbos=EN&KategorijaEn_input=Pathological+an… 3/12
3/27/2019 LSMUSIS
C. Diets high in saturated fats
D. Heavy cigarette - smoking habits
E. Low exercise tolerance

15.29 Which of the following tumors is associated mainly with occupational exposure to asbestos:
A. Bronchoalveolar carcinoma
B. Small cell carcinoma
C. Mesothelioma
D. Squamous cell carcinoma
E. Adenocarcinoma

II type tasks. For each numbered item,selct the one lettered heading that is most closely asssciated with it

15.30 How treat the changes of vessels in diabetes mellitus:


1 - Diabetic macroangiopathy
2 - Diabetic microangiopathy
A. Nephroangiopathy (nephroangiosclerosis)
B. Atherosclerosis of the coronary arteries

15.31 For each type of sarcoma that follows, select the tissue that it recapitulates
1 - Rhabdomyosarcoma
2 - Neurofibrosarcoma
3 - Angiosarcoma
4 - Liposarcoma
5 - Fibrosarcoma
A. Endothelium
B. Adipose tissue
C. Connective tissue
D. Skeletal muscle
E. Schwann cell

15.32 Which is prognostic characteristic of indicated tumours:


1 - Neoplasma benignum
2 - Neoplasma melignum
3 - Neoplasma semimalignum
A. Osteoblastoclastoma, basaloma and carcinoid
B. Cavernous hemangioma
C. Melanoma and seminoma

15.33 To which pathological processes are characteristic indicated pathogenetic mechanisms of the heart failure:
1 - Overloading of the atrium and ventricle caused by hypertension in it’s cavities
2 - Overloading of the atrium and ventricle caused by increased amount of blood in it’s cavities
3 - Changes of optimal frequency of the heart contractions
4 - Disorder of the heart contractive function
A. Stenosis of aortic orifice
B. Insufficiency of the aortic and mitral valvae and defectus of interventricular septi
C. Tachycardia and bradycardia
D. Endocardial fibrosis, necrosis of the myocardium and scar

15.34 Which is more frequent outcome of tuberculous primary focus:


1 - In cases of tuberculosis progression
2 - In cases of tubercles stabilization
A. Cavernisation and dissemination
B. Petrification and incapsulation
C. Caseous pneumonia

15.35 Which mechanism is characteristic for every hyperergic reaction:


1 - Atopic (anaphylactic) reaction
2 - Immune complex - mediated reaction
3 - Cell - mediated reaction
4 - Isoimmune(cytotoxic) reaction
A. Fixation of the IgM and IgG on surface of the cells

lsmusis.lsmuni.lt/Klausimai/Spausdinti?Length=0?Kalba=EN&KategorijaId=117&Kalbos_input=EN&Kalbos=EN&KategorijaEn_input=Pathological+an… 4/12
3/27/2019 LSMUSIS
B. Formation of antigen and IgG, IgM complexes with fixated complement
C. Action of mediators from sensitized (stimulated) lymphocytes
D. Fixation of Ag and IgE at mastocytes (labrocytes)

15.36 Where may be discovered first lymphogenic metastases of the indicated localization tumors:
1 - Carcinoma of the lung
2 - Carcinoma of the stomach
3 - Carcinoma of the breast
A. Supraclavicular and axillary lymph nodes
B. Lymph nodes along minor and major curvature of the stomach
C. Peribifurcative and paratracheal lymph nodes
D. Vertebral column
E. Cerebrum
F. Liver

15.37 Which elements forming exudate are characteristic to indicated pneumonias:


1 - Bronchopneumonia
2 - Lobar pneumonia
A. Fibrin
B. Neutrophils
C. Lymphocytes
D. Plasmocytes

15.38 Which histological picture is characteristic to indicated diseases of the thyroid gland:
1 - Colloid goiter
2 - Parenchymal (thyreotoxic) goiter
3 - Autoimmune thyroiditis (Hashimoto’s goiter)
4 - Thyroid carcinoma
A. Many various size, filled with thick colloid follicles. Epithelium of the follicles flattened.
B. Many various size and form follicles with epithelial papillae. Abundant mitoses. Some groups of
epithelial cells don’t form follicles. The cells penetrate into lymphoid clefts and capsule
C. The follicles are small, “starry” with high, cubical alike columnar epithelium. Scant amount of colloid in
the follicles with watery colloid and many resorbtive vacuoles. Some lymphoid infiltration in the stroma
D. Abundant amount of the connective tissue in stroma, large infiltrates of lymphocytic elements, here
and there forming lymphoid follicles with bright centers. Follicular epithelium mostly high.

15.39 Which signs are characteristic to indicated diseases:


1 - Viral respiratory infections
2 - Microbic infections
A. Punctative hemorrhages into mucoses of the bronchi and reddish mucinous exudate
B. Fibrinous or purulent inflammation of the respiratory tract

15.40 To which cells characteristic indicates orphophysiological qualities:


1 - Effectively act against intracellular parasites and are capable to present information about antigen to
immunocompetent cells
2 - Have many acidophylic granules and can top hyperergic reaction
3 - Have many lysosomes and first appear in the area of inflammation
4 - Have many metachromatic granules
5 - Release of lymphokines
6 - Have expressed granular endoplasmatic reticulum and eccentric nucleus
A. Macrophages
B. Neutrophils
C. Mastocytes (labrocytes)
D. Eosinophils
E. Plasmocytes
F. T lymphocytes

15.41 Which complications are characteristic to indicated disease:


1 - Central bronchogenic carcinoma of the lung
2 - Peripheral carcinoma of the lung
A. Obstruction of the bronchus and atelectasis of the lung
B. Emphysema of the lung
lsmusis.lsmuni.lt/Klausimai/Spausdinti?Length=0?Kalba=EN&KategorijaId=117&Kalbos_input=EN&Kalbos=EN&KategorijaEn_input=Pathological+an… 5/12
3/27/2019 LSMUSIS
C. Pleuritis

15.42 Which morphological qualities are characteristic to indicated structures:


1 - Thrombus
2 - Postmortal clot
A. Elastic and freely flowing in the lumen of the blood vessel
B. Fragile, dry, connected with wall of the blood vessel

15.43 By which way stomach carcinoma spreads into indicated organs:


1 - Lungs
2 - Ovary
A. Lymphogenic way
B. Hematogenic way

15.44 In which organs are more expressed morphological changes in cases of indicated disease:
1 - Grippe
2 - Paragrippe
3 - Adenovirosis
4 - Respiratory syncytiovirosis
5 - Rhinovirosis
A. Conjunctiva and nasopharynx
B. Larynx
C. Bronchus, bronchioles and lungs
D. Trachea
E. Mucosa of the nasus

15.45 Match each description of an inflammatory cell to the most appropriate cell type:
1 - These cells are the first arriving at a site of injury
2 - These cells can abort hypersensitivity reactions
3 - These cells have metachromatic granules and are found mainly in the tissues
4 - These cells are involved in both type I and type IV hypersensitivity reactions
5 - These cells are particulary effective against intracellular or encapsulated microbes
A. Macrophage
B. Mast cell
C. Neutrophil
D. Eosinophil
E. Basophil

III type tasks. For each question there is one or more correct answers:
A – if correct answers are 1,2,3
B – if correct answers are 1 and 3
C – if correct answers are 2 and 4
D – if correct answer is 4
E – if correct are all answers above

15.46 The prognosis for a patient with sarcoma is related to the tumor’s:
1. Size
2. Stage
3. Grade
4. Depth in the tissues

15.47 In most cases, acute (hemorrhagic) pancreatitis is associated with:


1. Hyperparathyroidism or drug toxicily
2. Autodigestion of the pancreatic parenchyma by activation of disaccharidases and pepsingen
3. Pancreatic tissue autolysis activated pepsinogen and disacharidoze
4. Ductal obstruction or direct toxicity to pancreatic acinar cells

15.48 Which of the following morphologic signs are typical for adult respiratory distress syndrome (ARDS):
1. Pulmonary edema
2. Hyaline membrane formation
3. Hemorrhage
4. Lung fibrosis
lsmusis.lsmuni.lt/Klausimai/Spausdinti?Length=0?Kalba=EN&KategorijaId=117&Kalbos_input=EN&Kalbos=EN&KategorijaEn_input=Pathological+an… 6/12
3/27/2019 LSMUSIS
15.49 Thrombocytopenia is commonly caused by
1. Folate deficiency
2. Myelofibrosis
3. Acute leukemia
4. Marked splenomegaly

15.50 What are functions of the inflammatory response:


1. Isolation of the infected tissue
2. Inactivation of the harmful factor
3. Neutralization of the toxins
4. Removal of the dead tissue

15.51 For which immune granulomas are characteristic caseous (or other) necrosis:
1. Tuberculosis
2. Syphilis
3. Cat - scratch disease
4. Sarcoidosis
5. Yersiniosis

15.52 What are characteristic signs for the hypertrophy of prostate:


1. Urocistitis
2. Retention of urine
3. Compression of the urethra
4. Paraproctitis
5. Neoplasmic infiltration of the urinary bladder wall

15.53 What is the most common pathology of the cervix uteri caused by dysfunction of the hormonal processes:
1. Erosio vera
2. Pseudoerosio papillaris
3. Ectropion
4. Pseudoerosio glandularis
5. All of the above

15.54 What is an insulin action on the metabolism of carbohydrates:


1. Amelioration of transmembraneous transport of glucose and its oxidation
2. Stimulation of glycogen syntheses
3. Blockade of gluconeogenesis
4. Stimulation of gluconeogenesis
5. Blockade of transmembraneous transport of glucose

15.55 Amount of following lipoproteins increases due to diabetes mellitus:


1. Pre beta lipoproteins
2. Alfa lipoproteins
3. Beta lipoproteins
4. Chilomicrons
5. Lipoproteins in flotation

15.56 Which of following renal changes are caused by diabetes mellitus:


1. Proliferation of the capsular epithelium of renal corpuscle and of the podocytes
2. Accumulation of PAS - positive deposits in the basement membranes of glomerular capillaries and in the
mesangium
3. Accumulation of amyloid in the basement membranes of glomerular capillaries
4. Accumulation of glycogen in the nephrocytes of proximal tubules
5. All of the above

15.57 What are characteristic symptoms of the parathyroid glands adenoma (hyperfunction):
1. Urolithiasis
2. Metastatic calcinosis of the lungs, stomach, myocardium
3. Thrombosis of the veins
4. Ulcer of the stomach

15.58 What are significant pathological processes for the genesis of bronchopneumonia:

lsmusis.lsmuni.lt/Klausimai/Spausdinti?Length=0?Kalba=EN&KategorijaId=117&Kalbos_input=EN&Kalbos=EN&KategorijaEn_input=Pathological+an… 7/12
3/27/2019 LSMUSIS
1. Derangement of lung ventilation
2. Viral inflammation of the respiratory tree
3. Chronic pulmonary congestion
4. Pulmonary emphysema
5. Atherosclerosis of the pulmonary arteries

15.59 What pathological processes can cause the development of pulmonary emphysema:
1. Bronchitis chronica
2. Increased release of proteolytic enzymes
3. Decreased amount of the inhibitors of proteolytic enzymes
4. Acute fibrinous pneumonia
5. Chronic pulmonary congestion

15.60 What are predisposing conditions for the deep venous thrombosis:
1. Prolonged bed rest after operation
2. Chronic right ventricle failure
3. Neoplasms
4. Prolonged bed rest after the trauma

15.61 What pathological changes are characteristic for the chronic right ventricle failure:
1. Edema of lower extremities
2. Ascites
3. Acute gastric ulcers
4. Chronic congestion of the liver

15.62 What pathological changes are characteristic for the chronic left ventricle failure:
1. Induration and hemosiderosis of pulmonary tissue
2. Chronic bronchitis with “ brown” macrophages in the sputum
3. Atherosclerosis of pulmonary arteries
4. Accumulation of fluid in the pleural cavity
5. Thoracic subcutaneous edema

15.63 Which pathological processes can cause mechanical jaundice:


1. Carcinoma in the head of the pancreas
2. Carcinoma in the common bile duct
3. Gallstone in the common bile duct
4. Gallstone in the cystic duct
5. Gallstone in the gallbladder

15.64 What type of exudate is characteristic for the uremic inflammation:


1. Serosal
2. Purulent
3. Hemorrhagic
4. Fibrinous

15.65 Which are precancerous lesions:


1. Dyskeratosis of the skin, leukoplakia, focal hyperpigmentation
2. Chronic ulcers of the stomach, oral mucous membranes and skin
3. Intestinal polyps
4. Pseudoerosion of the cervix of uterus
5. Glandular hyperplasia of the endometrium, breast and prostate

15.66 What are morphological signs of pernicious anemia (Adison - Birmer)


1. Atrophy of mucous linings of the tongue and stomach
2. Porphyric spleen
3. Pathological forms of erythrocytes
4. Hypochromic anemia
5. Agranulocytosis

15.67 What are signs of Hodgkin’s lymphoma:


1. Packages of lymph nodes adhering with each othe
2. Hodgkin’s cells

lsmusis.lsmuni.lt/Klausimai/Spausdinti?Length=0?Kalba=EN&KategorijaId=117&Kalbos_input=EN&Kalbos=EN&KategorijaEn_input=Pathological+an… 8/12
3/27/2019 LSMUSIS
3. Reed - Sternberg cells
4. Enlarged and non adhered with each other lymph nodes
5. Langerhan’s giant cells

15.68 What can be complications of syphilitic aneurysms:


1. Rupture of aorta
2. Aortic valvae disease
3. Myocardial infarction
4. Pulmonary embolism
5. Pulmonary atelectasis

15.69 Which factors of the nonspecific immune response are significant against viral infection:
1. Nucleases
2. Interferon
3. Proteases
4. Mechanical barryer of epithelium
5. Granulocytes
6. Lysozyme

IV type tasks. Choose only one best answer

15.70 The patient was ill with chronic lymphocytic leukemia (hemoblastomatosis lymphocytica). He died of wide
spread tuberculosis. What type of the immunodeficiency had the patient:
A. Primary humoral
B. Secondary cellular
C. Primary cellular
D. Secondary humoral

15.71 30-years-old woman complained of irregular bleeding from genitals. Patomorphological investigation of
curettage specimens showed endometrial glands in proliferation, numerous lymphocytes and plasma cells in the
stroma. What is a preliminary diagnosis:
A. Precancerous lesion of endometrium
B. Stein - Leventhal syndrome
C. Ectopic pregnancy
D. Excess of estrogens

15.72 30-years-old woman felt not severe pains in the loins and in the lower part of abdomen, she began to bleed
slightly from genitals. It occurred after 1,5 months menopause. During patomorphological investigation of curettage
specimens were detected tortuous endomitrial glands in secretion phase and hypercellular stroma with obvious
decidual response. What is a preliminary diagnosis:
A. Myoma uteri
B. Ovarian folliculoma
C. Pregnancy in the uterus
D. Endometrial inflammation
E. Ectopic pregnancy

15.73 The patient rubbed the ointment of penicillin on the evelid. Soon after a skin erythema appeared, patient felt
shortness of breath, unconsciously defecated and died of collapse. Which type of hypersensitivity reaction had the
patient:
A. Delayed type of cell - mediated hypersensitivity reaction
B. Systemic immune complex mediated hypersensitivity reaction
C. Isoimmunic hypersensitivity reaction
D. Local Arthus hypersensitivity reaction
E. Atopic (anaphylactic type) hypersensitivity reaction

15.74 The patient rubbed the ointment of penicillin on the evelid. Soon after a skin erythema appeared, patient felt
shortness of breath, unconsciously defecated and died of collapse. What are the mechanisms of the reaction:
A. Fixation of antigen and IgE complexes on the mast cells
B. Action of the mediators of the stimulated lymphocytes (lymphokines)
C. Fixation of the complexes of antigen with IgM and IgG on the cells
D. Formation of the complexes of antigen with IgM and IgG and activation of the complement system

lsmusis.lsmuni.lt/Klausimai/Spausdinti?Length=0?Kalba=EN&KategorijaId=117&Kalbos_input=EN&Kalbos=EN&KategorijaEn_input=Pathological+an… 9/12
3/27/2019 LSMUSIS
15.75 The patient was ill with rheumatic fever. He died of intoxication and heart failure. During autopsy was
detected: an enlarged hyperplastic spleen, enlarged soft kidneys with hemorrhagies, hypertrophied left ventricle of
the heart, the aortic valves thickened, deformed with some calcium deposits and ulcerations. The ulcers are
covered by polypus-like thrombotic massses. What was the complication of the rheumatic fever which caused the
death:
A. Hemorrhagic glomerulonephritis
B. Systemic vasculitis
C. Sepsis lenta

15.76 Elderly woman for a long time had aches in the vertebral column, pelvic and other bones. Because of the
increasing ache in the loins she was admitted to the hospital. Laboratory investigations showed increased amount
of blood urea and proteinuria. Glomerulonephritis was diagnosed. The other investigations were not completed as
the patient died of pneumonia soon. During autopsy were found enlarged, firm, whitish kidneys of a waxy
appearance. There were focal lesions in the bone marrow of the, skull, vertebral column and ribs. Macroscopically
the bony defects were soft and red, histologically - composed of the oval cells with eccentrically located nuclei. In
the lower lobes of the lungs were foci of more compact tissue. What is the diagnosis of the main disease:
A. Pyelonephritis
B. Glomerulonephritis
C. Plasmocytoma
D. Leucaemia myeloblastica

15.77 25- years-old woman died of bleeding from gastrointestinal tract. During autopsy it were detected whitish
skin and patchy hemorrhagies in the mucous membranes of the stomach and intestinum. Though ulcers were not
detected, there were bloody contents in the gastrointestinal. A spleen was dark - violet, firm with numerous
infarctions, its weight - 3000g. The weight of pale liver 3000g also. The bone marrow grayishly reddish. There are
40x10, leucocytes in the blood smear, which was taken before the death. Among leucocytes prevailed myelocytes
and promyelocytes. What is the diagnosis of the main disease:
A. Hemoblastomatosis myelocytica
B. Hemoblastomatosis lymphocytica
C. Lymphosarcoma
D. Aplastic anemia

15.78 58 years-old man unexpectedly felt a severe pain in the heart, weakness. He died after 15 minutes from the
beginning of the symptoms. It were detected during autopsy: atherosclerotic plaques and obturative fresh
thrombus in the left circumflex coronary artery. There were diversity of colors in the myocardium of the anterior -
lateral wall of left ventricle. The weight of the right and left lungs was 850 and 1150g, respectively. During pressure
of the lungs reddish fluids flowed out of the slice of pulmonary tissue. What is the diagnosis:
A. Rupture of the wall of heart
B. Ventriculae fibrillation
C. Thromboembolic complications
D. Acute left ventricle failure

15.79 The patient complained of shortness of breath, palpitation and edema of legs. The complaints appeared
after the slightest plysical load. After the death autopsy showed hypertrophy and dilatation of the left ventricle,
chronic pulmonary congestion. To which phase of compensatory process can be attributed the changes of the
heart:
A. Emergency
B. Consolidation
C. Decompensation

15.80 What type of viral hepatitis is characterized by shrinked and decreased in size liver (weight 1200g) and by
its massive panlobular confluent necrosis (pathomorphologically):
A. Acute classical lobular hepatitis
B. Acute lobular with confluent necrosis hepatitis
C. Fulminant hepatitis
D. Chronic active hepatitis
E. Chronic persistent hepatitis

15.81 46-years-old female continually used alcoholic drinks. She complained of liver dysfunction. Twenty four
hours before the death she felt a pain in the abdomen, suddenly decreased blood pressure. During the operation
the site of bleeding was not detected. Anemia progressed and the patient died. During autopsy were detected: the
veins varices in the abdominal wall, enlarged liver (1700g), its surface yellowish, firm and rough. The slices of the
lsmusis.lsmuni.lt/Klausimai/Spausdinti?Length=0?Kalba=EN&KategorijaId=117&Kalbos_input=EN&Kalbos=EN&KategorijaEn_input=Pathological+a… 10/12
3/27/2019 LSMUSIS
liver with yellowish small nodules. Weight of the pancreas-40 g. In the stomach and intestine coagulated blood.
There were esophageal veins varices, one of them was rupted. 4.133. What is the diagnosis of the main disease:
A. Acute viral hepatitis
B. Chronic viral hepatitis
C. Alcoholismus
D. Cirrhosis of liver

15.82 46-years-old female continually used alcoholic drinks. She complained of liver dysfunction. Twenty four
hours before the death she felt a pain in the abdomen, suddenly decreased blood pressure. During the operation
the site of bleeding was not detected. Anemia progressed and the patient died. During autopsy were detected: the
veins varices in the abdominal wall, enlarged liver (1700g), its surface yellowish, firm and rough.The slices of the
liver with yellowish small nodules. Weight of the pancreas-40 g. In the stomach and intestine coagulated blood.
There were esophageal veins varices, one of them was rupted. 4.134. What is a direct cause of death:
A. Esophageal veins varices
B. Bleeding from rupted varices
C. Veins varices in the abdominal wall
D. Hepatic insufficiency

15.83 During autopsy were detected beginning of incapsulation of caseotic necrosis focus in the 3-rd segment of
right lung and enlarged caseotic peribroncheal and bifurcation lymph nodes. What is an exact diagnosis of such
form of tuberculosis:
A. Tuberculosis focalis fibrocaseosa
B. Tuberculosis focalis caseosa acuta
C. Complexus primarius tuberculosus
D. Pneumonia caseosa tuberculosa

15.1 - D 15.2 - C 15.3 - C 15.4 - D 15.5 - A 15.6 - D 15.7 - C 15.8 - B


15.9 - C 15.10 - C 15.11 - B 15.12 - C 15.13 - B 15.14 - E 15.15 - C 15.16 - C
15.17 - B 15.18 - B 15.19 - D 15.20 - D 15.21 - D 15.22 - A 15.23 - C 15.24 - E
15.25 - E 15.26 - D 15.27 - B 15.28 - D 15.29 - C
15.30 15.31 15.32 15.33 15.34 15.35 15.36 15.37
1-B 1-D 1-B 1-A 1-A 1-D 1-C 1-B
2-A 2-E 2-C 2-B 2-B 2-B 2-B 2-A
3-A 3-A 3-C 3-C 3-A
4-B 4-D 4-A
5-C
15.38 15.39 15.40 15.41 15.42 15.43 15.44 15.45
1-A 1-A 1-A 1-A 1-B 1-B 1-D 1-C
2-C 2-B 2-D 2-C 2-A 2-A 2-B 2-D
3-D 3-B 3-A 3-B
4-B 4-C 4-C 4-E
5-F 5-E 5-A
6-E
15.46 15.47 15.48 15.49 15.50 15.51 15.52 15.53
1 4 1 1 1 1 1 2
2 2 2 2 2 2 4
3 3 3 3 3 3
4 4 4
15.54 15.55 15.56 15.57 15.58 15.59 15.60 15.61
1 1 2 1 1 1 1 1
2 3 4 2 2 2 2 2
3 3 3 3 3 3
4 4 4
15.62 15.63 15.64 15.65 15.66 15.67 15.68 15.69
1 1 4 1 1 1 1 1
2 2 2 3 2 2 2
3 3 3 3 3 3
4
5

lsmusis.lsmuni.lt/Klausimai/Spausdinti?Length=0?Kalba=EN&KategorijaId=117&Kalbos_input=EN&Kalbos=EN&KategorijaEn_input=Pathological+a… 11/12
3/27/2019 LSMUSIS
15.70 - B 15.71 - D 15.72 - E 15.73 - E 15.74 - A 15.75 - C 15.76 - C 15.77 - A
15.78 - D 15.79 - C 15.80 - C 15.81 - C 15.82 - B 15.83 - C

lsmusis.lsmuni.lt/Klausimai/Spausdinti?Length=0?Kalba=EN&KategorijaId=117&Kalbos_input=EN&Kalbos=EN&KategorijaEn_input=Pathological+a… 12/12
3/27/2019 LSMUSIS

Pharmacology
I type tasks. Choose only one best answer

25.1 The most common side effect of ACE inhibitors is:


A. Taste disturbances
B. Dry cough
C. Somnolence
D. Hypokalemia

25.2 Which drug is drug of choice for “pulse steroid“ therapy?


A. Methylprednisolonum
B. Hydrocortisonum
C. Prednisolonum
D. Prednisonum

25.3 Which proposition describes Good Clinical Practice standard best?


A. Each invidual involved in conducting a trial should be qualified by education, training, and experience to
perform his or her respective task(s)
B. Before a trial is initiated, foreseeable risks and inconveniences should be weighed against the anticipated
benefit for the individual trial subject and society
C. The rights, safety, and well-being of the trial subjects are the most important Considerations of
Investigator
D. Good clinical practice (GCP) is an international ethical and scientific quality standard for designing,
conducting, recording, and reporting trials that involve the participation of human subjects. Compliance with
this standard provides public assurance that the rights, safety, and wellbeing of trial subjects are protected,
consistent with the principles that have their origin in the Declaration of Helsinki, and that the clinical trial
data are credible

25.4 Allergic reactions are most common in the use of:


A. Streptokinasum
B. Urokinasum
C. Alteplasum

25.5 Name drugs, which are used because they act positively on development of early and late asthmatic reaction.
A. Theophyllinum and beta adrenergic agonists
B. Theophyllinum and beta adrenergic antagonists
C. Anticholinergics
D. Cromolinum and nedocromilum
E. Cromolinum, nedocromilum and theophyllinum

25.6 Which daily dose of aspirin should be used for the prevention of myocardial infarction?
A. 75-150 mg
B. 330-500 mg
C. 500- 1000 mg
D. 1,0-3,0 g

25.7 The strongest action on cholesterol and LDL reduction in plasma has:
A. Bile acid sequestrans
B. Statins
C. Nicotinic acid derivates
D. Fibrates

25.8 Name the drugs that are used extensively currently due to effective inhibition of inflammatory component of
asthma.
A. Theophyllinum
B. Beta-2 adrenergic agonists
C. CIpratropium
D. Corticosteroides
E. No positive answer

25.9 The biggest risk of overdose is in the use of:

lsmusis.lsmuni.lt/Klausimai/Spausdinti?Length=0?Kalba=EN&KategorijaId=127&Kalbos_input=EN&Kalbos=EN&KategorijaEn_input=Pharmacology&… 1/18
3/27/2019 LSMUSIS
A. Barbiturates and tricycle antidepressants (TCAs)
B. Tranquillises
C. Neuroleptics
D. Selective Serotonin Reabsoption Inhibitors (SSRI)

25.10 Name the drug - Dopamine receptor antagonist for the prevention of nausea and vomiting:
A. Aspirinum
B. Cimetidinum
C. Metoclopramidum
D. Erythromycinum
E. Bisacodylum

25.12 Most rapid onset of action of nitroglycerine’s pharmaceutical form has:


A. Sublingual tablet
B. Slow release capsule
C. Film coated tablet
D. Transdermal disc

25.13 The longest duration of action of nitroglycerine’s pharmaceutical form has:


A. Sublingual tablet
B. Transdermal disc
C. Film coated tablet
D. Nitroglycerin ointment

25.14 What is the most useful method of administration of statins:


A. Three times per day (without relation to meal)
B. Once daily on evening ours
C. Three times per day (30 min. before meal)

25.15 Contraindication of β-adrenergic blockers:


A. 2nd or 3rd degree of heart block
B. Supraventricular tachycardia
C. Mitral stenosis

25.16 The shortest half life of antiarrythmic medicine is for:


A. Lidocainum
B. Amiodaronum
C. Quinidinum
D. Procainamidum
E. Adenosinum

25.17 The highest risk of heart block is in the case of:


A. Felodipinum
B. Nifedipinum
C. Nicardipinum
D. Verapamilum

25.18 Effectiveness of oral contraceptives can be reduced and abnormal menstrual cycle can arise if the oral
contraceptives are used together with:
A. Acetaminofenum
B. Aspirinum
C. Digoxinum
D. Ampicillinum
E. Cimetidinum

25.19 The Drug of choice” for the short term treatment of sleep disturbances is:
A. Phenobarbitalum
B. Diazepamum
C. Medazepamum ar oxazepamum
D. Clonazepamum
E. Nitrazepamum ar triazolamum

25.20 Which drug is the drug of choice for the treatment of ulcerative colitis?
lsmusis.lsmuni.lt/Klausimai/Spausdinti?Length=0?Kalba=EN&KategorijaId=127&Kalbos_input=EN&Kalbos=EN&KategorijaEn_input=Pharmacology&… 2/18
3/27/2019 LSMUSIS
A. Sulfasalazinum
B. Acidum 5-aminosalicylicum
C. Ibuprofenum
D. Metoclopramidum

25.21 Omeprazole is:


A. H2 receptor antagonist
B. Cytoprotector
C. Inhibitor of proton pump
D. Anticholinergic medicine

25.22 Many patients with hepatic insufficiency and hepatic coma are treated with nonabsorbsable antimicrobials
such as neomycin. Name the basis for this tactic:
A. A. Antibiotic, inhibiting gut microorganisms, decreases the quantity of ammonia and other bacterial
products, which can be absorbable
B. Antibiotic destroys microorganisms which are highly pathogenic
C. C. Neomycin acts not only as antibiotic but also as a diuretic, causing transfer of fluids from peritoneum
into intestinal tract
D. D. Neomycin is indicated in case of ascending cholangit only, which could be the consequence of hepatic
insufficiency

25.23 Name the point when penicillin has weak antibacterial activity or no antibacterial activity at all:
A. Treponema pallidum
B. Actively dividing microorganisms
C. Meningococci
D. Actively non-dividing microorganisms

25.24 Which calcium antagonist has highest selectivity on vascular smooth muscles?
A. Verapamilum
B. Diltiazemum
C. Nifedipinum
D. Nicardipinum

25.25 ACE Inhibitors should been prescribed for the treatment of heart failure:
A. In all cases in the absence of contraindications
B. In the case of renal failure
C. If heart failure is severe or moderate degree
D. In the case when potasium excretion diuretics are prescribed
E. When digitalis treatment is not effective

25.26 The risk of arrhythmias in healthy people on antiarrhythmics medicine treatment is:
A. More common
B. Less common
C. Without changing

25.27 The strongest anti-inflammatory action has:


A. Hydrocortizonum (3mg)
B. Dexamethazonum (3mg)
C. Prednisolonum (3mg)
D. Metylprednizolonum (3mg)

25.28 Oral contraceptives can decrease the effect of:


A. Insulin
B. Aminophyllinum
C. Prednizolonum
D. Diazepamum
E. Dexamethazonum

25.29 Which dose of heparin is most effective in the prevention of thromboembolic disorders after elective
gynecological or abdominal-thoracic surgery?
A. Low-dose 10000 units/daily
B. Medium-dose 20000 units/daily

lsmusis.lsmuni.lt/Klausimai/Spausdinti?Length=0?Kalba=EN&KategorijaId=127&Kalbos_input=EN&Kalbos=EN&KategorijaEn_input=Pharmacology&… 3/18
3/27/2019 LSMUSIS
C. Very high-dose 40000 units/daily

25.30 Which medicine could worse the condition of closed angle glaucoma?
A. Amitriptyllinum
B. Fluoxetinum
C. Diazepamum
D. Sumatriptanum
E. Chloramphenicolum

25.31 Which answer is correct?


A. Geriatric patients are more sensitive to beta adrenoblockers effect
B. Geriatric patients are less sensitive to beta adrenoblockers effect
C. Geriatric patients are less sensitive to benzodiazepines effect
D. Geriatric patients are less sensitive to calcium channels blocks effect

25.32 Which medicines are with the increased risk in geriatric age?
A. There are no such medicines
B. Medicines that have short T1/2, psychotropic and anticholinergic medicines
C. Medicines that have long T1/2, psychotropic and anticholinergic medicines
D. Inject able medicines

25.33 Witch proposition describes “tolerance to nitrates” best?


A. Tolerance to nitrates becomes after long term treatment and we have to increase dose of nitrates
B. BTolerance to nitrates becomes in case when nitroglycerine in combination with the aspirin
C. D. When nitroglycerine medications are used in 24 hours regime and there is not intermittent regime (free
nitrate concentration period)
D. When patient use sublingual capsules together with nitroglycerine transdermal disc and effect is
summarizing

25.34 What is the maximum daily dose of paracetamol for the adult?
A. 2 g
B. 4 g
C. 6 g
D. 10 g

25.35 What effect can be expected in pregnant women if she is using antiepileptics?
A. Abortion
B. Teratogenic effect
C. Mutagenic effect
D. Cancerogenic effect
E. All listed

25.88 Which anticonvulsant has half-life longer then 24 hr?


A. Carbamazepin
B. Valproic acid
C. Phenytoin
D. Oxcarbazepin

25.89 Which proposition describes drug development time and costs best?
A. 100.000 EUR/0.2% sales/1 month
B. 1.000.000 EUR/0.6% sales/6 month
C. 10.000.000 EUR/6% sales/6 years
D. 900.000.000 EUR/14-16% sales/14 years

25.90 What type of testing is used in the preclinical phase of drug discovery?
A. Computers based disease models
B. Cellular methods
C. Cellular tissues methods
D. Laboratory animals’ method
E. All listed

25.91 What characteristics of a new drug discovered goes for testing during preclinical phase:
A. Pharmacokinetics in healthy volunteers
lsmusis.lsmuni.lt/Klausimai/Spausdinti?Length=0?Kalba=EN&KategorijaId=127&Kalbos_input=EN&Kalbos=EN&KategorijaEn_input=Pharmacology&… 4/18
3/27/2019 LSMUSIS
B. New indications and pharmacodynamics in patients
C. Side effects in patients with renal failure
D. Bioequivalence and bioavailability
E. Pharmacokinetics and pharmacodynamics of the new drug in computers based disease models, cellular
tissues methods and laboratory animals’ methods

25.92 What type of clinical trials are going in phase I clinical trials:
A. Pharmacokinetics trials in healthy volunteers
B. Clinical trials for new indications
C. Safety trials after drug authorization
D. Trials on drug interactions
E. Pharmacodynamics trials in elderly patients

25.93 What type of methods of clinical trials is going in phase II clinical trials?
A. "Duble-blind" method
B. “Blind” method
C. Randomized method
D. Controlled method
E. All listed

25.94 What type of clinical trials is going in phase III clinical trials?
A. Preclinical safety trials
B. Pharmacokinetics trials in healthy volunteers
C. Bioequivalence and bioavailability trials
D. Detailed trials on pharmacokinetics and pharmacodynamics in large group of 3000-5000 patients
E. Pharmacodynamics trials in elderly patients

25.95 What type of clinical trials is going in phase IV clinical trials?


A. Preclinical safety trials
B. Pharmacokinetics trials in healthy volunteers
C. Bioequivalence and bioavailability trials
D. Detailed trials on pharmacokinetics and pharmacodynamics in large group of 3000-5000 patients
E. New indications and safety trials after authorization

25.96 Pharmacoeconomics’ analysis:


A. It’s analytical methods that compare the value of one pharmaceutical drug to another or drug therapy to
another health service in correlation to outcomes and it serves optimal healthcare resource allocation
B. It’s settlement of margin of retail price for drugs
C. It’s settlement of margin of whole price for drugs
D. It’s treatment with the cheapest drug

25.97 Cost minimization analysis is:


A. When investigational and comparative drugs have equal efficacy and equal tolerability and comparison is
only for costs
B. When two or more drugs or alternative health service have different costs and outcomes, that are
measured in physical units
C. When cost utility is evaluated in social aspect and results are measured in quality-adjusted life years
(QALYs)
D. When incremental cost-effectiveness ratio (ICER) is under evaluation

25.98 Cost effectiveness analysis is:


A. When investigational and comparative drugs have equal efficacy and equal tolerability and comparison is
only for costs
B. When two or more drugs or alternative health service have different costs and outcomes and evaluation
of incremental cost-effectiveness ratio (ICER) shows additional costs for additional benefit. The incremental
cost-effectiveness ratio (ICER) is the ratio between the difference in costs and the difference in benefits of
two interventions
C. When cost utility is evaluated in social aspect and results are measured in quality-adjusted life years
(QALYs)
D. Cost analysis is performed in social aspect with calculation of costs that are not related to health service
costs and benefits and these results are coming from outside health service budget

lsmusis.lsmuni.lt/Klausimai/Spausdinti?Length=0?Kalba=EN&KategorijaId=127&Kalbos_input=EN&Kalbos=EN&KategorijaEn_input=Pharmacology&… 5/18
3/27/2019 LSMUSIS
25.99 Cost benefits analysis is in case:
A. When results are measured in quality-adjusted life years (QALYs) and additional costs are measured in
terms of additional years of full health lived, using a measure such as quality-adjusted life years
B. When investigational and comparative drugs have equal efficacy and equal tolerability and comparison is
only for costs
C. When two or more drugs or alternative health service have different costs and outcomes and evaluation
of incremental cost-effectiveness ratio (ICER) shows additional costs for additional benefit
D. The incremental cost-effectiveness ratio (ICER) is calculation for one clinical outcome

25.100 Pharmacoeconomics analysis is doing evaluation of:


A. Reduction in death
B. Reduction in treatment complications
C. Reduction in adverse drug reactions
D. Reduction in disease symptoms
E. Reduction in hospitalization
F. All listed

25.101 Direct medical costs are


A. Costs for medicines
B. Out patients costs
C. Costs for hospitalization
D. Costs for diagnostic procedures and laboratory tests
E. Costs for adverse drug reactions treatment
F. All listed

25.102 Nimesulide has more expressed adverse drug reaction than other NSAID’s:
A. Hepatotoxic action
B. Allergic reaction
C. Renal failure
D. Stomach ulcer
E. Bleeding from stomach
F. Agranulocitosis

25.103 Name the true sentence. The cardio selectivity of beta-adrenergic blockers is:
A. Therapeutic dose has action only on beta 1-adrenergic receptors
B. Les risk of bronchospasm
C. Les risk reduction in the peripheral circulation
D. Relative selectivity. High dose has action on beta 2-adrenergic receptors two
E. All listed

25.104 If the virus infection suspected, the use of aspirin is restricted due to:
A. Reye syndrome
B. Increase risk of hypothermic seizure
C. Increase risk of hyperergic reactions
D. Increase risk of stomach bleeding
E. All listed events

25.105 Which answer is correct?


A. Geriatric patients are more sensitive to H2 blockers effect
B. Geriatric patients are more sensitive to diuretic effect
C. Geriatric patients are more sensitive to antibiotic effect
D. Geriatric patients are more sensitive to benzodiazepines effect

25.106 Which answer is correct?


A. In geriatric age, content of body water and lipids increases
B. In geriatric age, content of body water and lipids does not change
C. In geriatric age, content of body water increases while lipids - increases
D. In geriatric age, content of body water decreases while lipids - increases

25.107 Which answer is correct?


A. In geriatric age, both phases of metabolism are disturbed
B. In geriatric age, first phase of metabolism is disturbed

lsmusis.lsmuni.lt/Klausimai/Spausdinti?Length=0?Kalba=EN&KategorijaId=127&Kalbos_input=EN&Kalbos=EN&KategorijaEn_input=Pharmacology&… 6/18
3/27/2019 LSMUSIS
C. In geriatric age, second phase of metabolism is disturbed
D. In geriatric age, metabolism is not disturbed

25.108 Which answer is correct?


A. In geriatric age, T1/2 of majority of medicines is increased
B. In geriatric age, T1/2 of majority of medicines is decreased
C. In geriatric age, T1/2 of majority of medicines is unchanged
D. In geriatric age, T1/2 of majority of medicines disappears

25.109 The teratogenic effect is?


A. All congenital abnormalities
B. Congenital abnormalities when medicine was used during pregnancy
C. Congenital abnormalities when medicine was used before fertilisation
D. Death of fetus

25.110 Which antibiotics are safe for pregnant women?


A. Penicillins
B. Tetracyclins
C. Fluoroquinolons
D. Aminoglycosids

25.111 Which medicines are with increased risk for faint?


A. Penicillins
B. Glucocorticoids
C. Neuroleptics
D. Paracetamol

25.112 Rash on skin was observed after the start of using penicillin. After stopping of medicine, rash disappears.
No other medicine is used. This type of adverse effect is described in package leaflet. Patient had rash in the past
from the amoxicillin use. What is causal relationship?
A. Direct
B. Probable
C. Possible
D. Remote

25.113 After 2 days of using levodopa, nausea appeared. After stopping of medicine, nausea diminished but not
disappeared. Patient was using diclofenac at the same time. This type of adverse effect is described in package
leaflet. What is causal relationship to levodopa?
A. Direct
B. Probable
C. Possible
D. Remote

25.114 Adverse effect is observed in 3% of patients. What category of frequency is relevant to this effect?
A. Very frequent
B. Frequent
C. Not frequent
D. Rare
E. Very rare

25.115 Which medicines most frequently cause allergy?


A. Neuroleptics
B. Antihypertensives
C. Antibiotics
D. Anticoagulants

25.116 Which medicines most frequently cause thrombocytopenia?


A. Heparins
B. Benzodiazepines
C. Antidepressants
D. Antiulcer medicines

25.117 Which medicines can most frequently damage kidneys?


lsmusis.lsmuni.lt/Klausimai/Spausdinti?Length=0?Kalba=EN&KategorijaId=127&Kalbos_input=EN&Kalbos=EN&KategorijaEn_input=Pharmacology&… 7/18
3/27/2019 LSMUSIS
A. Heparin
B. Benzodiazepines
C. NSAIDs
D. Antiulcer medicines

25.118 Which medicines can most frequently damage liver?


A. Benzodiazepines
B. Paracetamol
C. Penicilines
D. Cardiac glycosides

25.119 Which medicines disturb ability to drive?


A. Benzodiazepines and antidepressants
B. Paracetamol and neuroleptics
C. Amiodarone and antiepileptics
D. Cardiac glycosides and antiparkinsonics

25.120 Which adverse effect is most probably serious?


A. If patient is not able to go working 20 days
B. If patient experienced rash in 2/3 of body
C. If patient was hospitalised for 3 days due to adverse effect
D. If patient experienced shortness of breath

25.121 Which of adverse effects is most probably serious?


A. If patient is not able to go working 15 days
B. If patient experienced fatigue for 3 hours
C. If patient was vomiting
D. If patient has prolonged stay in hospital

25.122 The most rationale way for monitoring of therapeutic drugs effect:
A. Character of patient’s complaints
B. Establishment of objective symptoms (physical findings)
C. Measurement of drug concentration
D. Interpretation of clinical laboratory tests

25.123 What proposition is correct?


A. All methods, using for monitoring of therapeutic drugs effect, are expensive
B. Drug concentration is measured for all patients, which are treated with combination of drugs
C. Drug concentration should never replace clinical judgment
D. Instrumental procedures are basal for monitoring of therapeutic drugs effect

25.124 Common clinical methods for monitoring of therapeutic effect of antimicrobial drugs:
A. Laboratory tests (blood and urine)
B. Clinical assessment (pain, fever) and laboratory blood tests (white cells count, C reactive protein)
C. Dynamic of X-ray data
D. Dynamic of palpation and auscultation

25.125 What character of chest pain should be estimate during monitoring of therapeutic effect of antianginal
drugs?
A. Stabbing pain in left side, associated with breathing
B. Long-lasting chest pain, beginning at rest
C. Burning pain in the pit of the stomach at hunger
D. Substernal pain at physical exertion, in the cold, in the wind, relieving after stopping exertion

25.126 Which findings can indicate effect of drugs, using for treatment of hypertension?
A. Sodium concentration in blood
B. Heart X-ray
C. Arterial blood pressure
D. ECG

25.127 Method for monitoring of antithrombotic effect of oral anticoagulants:


A. Laboratory tests (International normalized ratio and Prothrombin index)
B. Patient’s complaints
lsmusis.lsmuni.lt/Klausimai/Spausdinti?Length=0?Kalba=EN&KategorijaId=127&Kalbos_input=EN&Kalbos=EN&KategorijaEn_input=Pharmacology&… 8/18
3/27/2019 LSMUSIS
C. Chest X-ray
D. Echoscopy

25.128 Which findings can indicate effect of antihyperlipidemic drugs?


A. Patient’s complaints
B. Cholesterol plasma concentration
C. Patient’s weighing
D. ECG

25.129 Pediatric pharmacology is the mostly influenced by:


A. Different aspects of pharmacokinetics in children
B. Different aspects of pharmacodynamics in children
C. Both

25.130 Absorption of drugs from injection site in severe sick hypotrophic children is:
A. Increased
B. Decreased
C. Steady

25.131 Absorption of drugs in children from gastrointestinal tract in case of diarrhoea is:
A. Increased
B. Decreased
C. Steady

25.132 Concentration of water-soluble drugs (e.g. aminoglicoside antibiotics) when dosing g/kg of body weight will
be:
A. Greater in younger children than in older
B. Greater in older children than in younger
C. The same

25.133 Concentration of fat-soluble drugs when dosing g/kg of body weight will be:
A. Greater in younger children than in older
B. Greater in older children than in younger
C. The same

25.134 Drug efficacy in hypoalbuminemia in children is:


A. Elevated
B. Decreased
C. The same

25.135 Liver metabolism of drugs in younger children comparing to adult patients is:
A. Decreased
B. Increased
C. The same

25.136 Renal excretion of drugs in younger children comparing to adult patients is:
A. Decreased
B. Increased
C. The same

25.137 Pharmacodynamics in children and adults. Please select correct statement:


A. Pharmacodynamics of most drugs does not differ in children and adults
B. Drug sensibility is more common in children than adults
C. Drug resistance is more common in children than adults
D. Allergic reactions more common in children than adults

25.138 Why is important to shake suspension for children before using?


A. Drug distributes evenly in the bottle
B. Because expected efficacy of the drug may decrease at the beginning of therapy
C. Because of the increased risk of intoxication at the end of therapy
D. All correct

25.139 Bioavailability is a term that defines amount of medicine that appears into:
lsmusis.lsmuni.lt/Klausimai/Spausdinti?Length=0?Kalba=EN&KategorijaId=127&Kalbos_input=EN&Kalbos=EN&KategorijaEn_input=Pharmacology&… 9/18
3/27/2019 LSMUSIS
A. Stomach
B. Liver
C. Systemic circulation
D. All mentioned places
E. None of mentioned

25.140 The most precise definition of AUC is:


A. This is theoretical meaning of absorption of medicine
B. This is description of amount of medicine in cell
C. This is description of changes of concentration in time
D. This is description of concentration in mg/ml
E. All definitions are correct

25.141 Which of drug absorption speed limiting process could be the first after taking tablet orally?
A. Ionization of medicine
B. Diffusion of medicine through epithelium of gastrointestinal tract
C. Solubility of medicine in juice of gastrointestinal tract
D. Solubility of medicine in blood
E. Disintegration of tablet

25.142 Which of processes could be drug absorption speed limiting process after taking oral medicine form?
Please choose most appropriate answer
A. Disintegration of tablet
B. Dissolution of active substance
C. Diffusion of active substance through epithelium of gastrointestinal tract
D. All processes
E. None of processes are speed limiting

25.143 Where drug metabolism could take place? Please choose most appropriate answer
A. Skin
B. Lungs
C. Liver
D. Kidneys
E. In all mentioned organs

25.144 Highest variability of bioavailability is possible in cases when medicine is


A. Injected into skin
B. Injected into vein
C. Taken orally
D. Taken sublingually
E. Injected into muscles

25.145 54 years old men, 90 kg, creatinine clearance (according to Cocroft-Gault) is 30 ml/min needs to be given
gentamycin for treatment of Gram negative infection. What dose of Gentamycin would be most reasonable for this
patient if the goal is to achieve 8 mg/ml maximal and 1 mg/ml minimal concentration? (Vd of gentamycin is 0.25
l/kg).
A. 80 mg every 24 hrs
B. 80 mg every 8 hrs
C. 160 mg every 24 hrs
D. 240 mg every 24 hrs

25.146 85 years old men, 60 kg, creatinine clearance (according to Cocroft-Gault) is 20 ml/min is using 0.25 mg of
digoxin tablets once daily for the treatment of atrial fibrillation. After one month of treatment sudden emesis and
vomiting is experienced. After measurement of serum concentration it was found that digoxin concentration is 2.6
mcg/l. Is this a concentration that could be predictable? (Absorption of digoxin is 60%, clearance of digoxin (in
ml/min/kg) is equal to creatinine clearance + 0.33).
A. Yes
B. No. Predicable concentration is 1.5 mcg/ml
C. No. Predicable concentration is 2.0 mcg/ml
D. No. Predicable concentration is 3.5 mcg/ml

lsmusis.lsmuni.lt/Klausimai/Spausdinti?Length=0?Kalba=EN&KategorijaId=127&Kalbos_input=EN&Kalbos=EN&KategorijaEn_input=Pharmacology… 10/18
3/27/2019 LSMUSIS
25.147 85 years old men, 85 kg, creatinine clearance (according to Cocroft-Gault) is 20 ml/min is using 0.25 mg of
digoxin tablets once daily for the treatment of atrial fibrillation. After one month of serum concentration was
measured and serum concentration was found to be 3.0 mcg/l. What dose should be given in order to achieve
usual therapeutic concentration of 1.5 mcg/l? (Absorption of digoxin is 60%, clearance of digoxin (in ml/min/kg) is
equal to creatinine clearance + 0.33).
A. 125 mcg
B. 150 mcg
C. 175 mcg
D. 200 mcg

25.148 During use of levodopa preparations, antiparkinsonic effect may be decreased in case of concomitant use
of:
A. Acetylsalicylic acid
B. Ascorbic acid
C. Pyridoxine hydrochloride
D. All mentioned medicines effects levodopa
E. None of mentioned medicines effects levodopa

25.149 Which medicine can not be missed with gentamycin sulfate in one syringe for intravenous use?
A. Phenytoin
B. Ticarcillin sodium
C. Acetazolamide
D. All mentioned medicines are allowed
E. None of mentioned medicines is allowed

25.150 Cholestiramine disturbs absorption process in gastrointestinal tract. Therefore, it is not rational to use
cholestiramine together with:
A. Warfarin sodium
B. Pyridoxine hydrochloride
C. Combination with both mentioned medicines is allowed
D. None combination is allowed

25.151 In case when medicine A is pushed from binding to plasma proteins by medicine B, this interaction most
probably will be clinically significant when:
A. Association constant with protein is high for A
B. Association constant with protein is high for B
C. Association constant with protein is high for B and B is used in high dosage
D. All situations are relevant
E. None of situations is relevant

25.152 In case hemorrhagic disturbance is caused by warfarin, this could be inhibited by:
A. Dihydrotachisterol products
B. Egocalciferol products
C. Phytomenadion products
D. All situations are possible
E. None of situations is suitable

25.153 In case of treatment with high dosages of gentamycin, most dangerous combination is with:
A. Doxicycline
B. Torasemid
C. Enalapril
D. All are dangerous
E. None is dangerous

25.154 In case of treatment with warfarin, most dangerous combination is with:


A. Cimetidin
B. Acetaminofen
C. Enalapril
D. All are dengeraous
E. None is dengeraous

III type tasks. For each question there is one or more correct answers:

lsmusis.lsmuni.lt/Klausimai/Spausdinti?Length=0?Kalba=EN&KategorijaId=127&Kalbos_input=EN&Kalbos=EN&KategorijaEn_input=Pharmacology… 11/18
3/27/2019 LSMUSIS
A – if correct answers are 1,2,3
B – if correct answers are 1 and 3
C – if correct answers are 2 and 4
D – if correct answer is 4
E – if correct are all answers above

25.37 Verapamil doesn‘t cause:


1. Bradycardia
2. Constipation
3. Heart block
4. Reflex tachycardia

25.38 Name medicines for the treatment of paroxysm tachycardia:


1. Nifedipinum
2. Adenosinum
3. Lidocainum
4. Verapamilum
5. Labetololum

25.39 Name contraindications for the use of -adrenergic receptors blockers:


1. Arterial hypertension
2. Sinus bradycardia
3. Sinus tachycardia
4. 2nd degree of heart block
5. Supraventricular extra systoles

25.40 Contraindications for the use of captopril include:


1. Hyperkalemia
2. Hypersensitivity to the drug components
3. 3. Both a. renalis stenosis
4. Previously therapy of diuretics
5. Renal failure

25.41 Mechanism of action of nonsteroidal anti-inflammatory drugs includes:


1. Inhibition synthesis of COX-I
2. Inhibition synthesis of COX-II
3. Inhibition synthesis of leucotriens
4. Inhibition of CNS endorphins system
5. Direct inhibition of thermoregulation

25.42 Hypersensitivity to heparin includes:


1. Thrombocytopenia
2. Necrosis in injection sites
3. Formation of white thrombi
4. Formation of red thrombi
5. Lymphocytosis
6. Anaemia

25.43 Name medicines with ototoxic side effects:


1. Furosemidum
2. Acidum aethacrynicum
3. Streptomycinum
4. Gentamycinum

25.44 Name penicillins mainly resistant to penicillinase:


1. Ampicillinum
2. Oxacillinum
3. Carbenicillinum
4. Nafcillinum

25.45 The resorption of medicines in case of acute migraine episode can be increased by concomitant use of:
1. Metoclopramidum
lsmusis.lsmuni.lt/Klausimai/Spausdinti?Length=0?Kalba=EN&KategorijaId=127&Kalbos_input=EN&Kalbos=EN&KategorijaEn_input=Pharmacology… 12/18
3/27/2019 LSMUSIS
2. Chlorpromazinum
3. Coffeinum
4. Morfinum
5. Aspirinum

25.47 Chose right propositions of antiepileptic therapy:


1. Symptoms can improve after one month treatment
2. Treatment with several medicines is choice
3. Treatment can be discontinued if there are not episodes of seizures after 6 month therapy
4. Antiepileptic medicines can have interaction for the metabolism of other medicines in liver

25.48 What differences are between antidepressants of SSRI group and tricycle antidepressants (TCAs)?
1. Less common urinary retentions in patients with adenoma of prostate
2. Less risk of overdose
3. Less risk of arrhythmias
4. Do not select (error)
5. Greater efficacy

25.49 What medicines could be used for the prevention of ischemic stroke?
1. Dipyridamolum
2. Aspirinum
3. Aminophyllinum
4. Ticlopidinum
5. Acidum nicotinicum (i/v)

25.50 Side effects of haloperidol are most common that for chlorpromazine:
1. Constipation and dry mouth
2. Sedation
3. 3. Disturbances of menstrual cycle
4. Extra pyramidal symptoms
5. Galactorrhea

25.51 Side effects of ticlopidine are


1. Neutropenia
2. Diarrhea
3. Skin rash
4. Elevation of cholesterol
5. Bleeding

25.52 Name the advantages of ticlopidine over aspirin:


1. Reduces adhesion and aggregation of platelet
2. Less costly
3. Equal in efficacy for man and women
4. No risk of neutropenia

25.53 What kind of actions should be started in case of acute episode of ischemic stroke?
1. Phenobarbitalum
2. Aspirinum
3. Prednisolonum
4. Maintenance of optimal blood pressure
5. Phenitoinum

25.54 Medicines that should be avoided for patients that like herring, matured cheese, red vine:
1. Tricycle antidepressants (TCAs)
2. Strong neuroleptics
3. Atypical antidepressants
4. MAO inhibitors
5. Benzodiazepines

25.55 Medicines that should be useful for the prevention of migraine attacks:
1. Sumatriptanum
2. Propranololum

lsmusis.lsmuni.lt/Klausimai/Spausdinti?Length=0?Kalba=EN&KategorijaId=127&Kalbos_input=EN&Kalbos=EN&KategorijaEn_input=Pharmacology… 13/18
3/27/2019 LSMUSIS
3. Aminophyllinum
4. Amitriptyllinum
5. Clonidinum

25.56 Side effects of oral hormonal contraceptives are:


1. Ischemic stroke
2. Gall-bladder disturbances
3. Liver failure
4. Myocardial infarction
5. Thromboembolic disorders

25.57 Medicines for the treatment of acute attacks of migraine are:


1. Sumatriptanum
2. Propranololum
3. Ergotaminum
4. Amitriptillinum
5. Furosemidum

25.58 Side effects of chlorpromazine are more common than that of haloperidol:
1. Neurolepsia
2. Impotence
3. Pharmacogenic parkinsonism
4. Dry mouth
5. Akathisia

25.59 As high neuroleptic potency as common side effects:


1. Allergic reactions
2. Constipation
3. Urinary retention
4. Underdose for the neurolepsia.
5. Galactorrhea

25.60 Diseases or conditions when oral contraceptives should be avoided:


1. Complicated diabetes
2. Surgery after 1 week with long immobilization after
3. Age over 35 year and smoking
4. Asthma bronchiale with acute episodes at night ours
5. Nonspecific ulcerative colitis

25.61 Atrial fibrillation can be caused by:


1. Alcohol
2. Aminophyllinum
3. Amfetaminum
4. Enalaprilum
5. ISDN

25.62 Medicines, which can be useful for the treatment of supraventricular tachycardia in patient with bronchial
asthma:
1. Nifedipinum
2. Adenosinum
3. Lidocainum
4. Verapamilum
5. Labetololum

25.63 Before examination student had excitation, diarrhea thus took 4 tablets of diazepam (5 mg). What results
can he expect?
1. Less excitation
2. More mistakes
3. Improve symptoms of diarrhoea
4. Somnolence
5. Less sadness in case of negative mark.

lsmusis.lsmuni.lt/Klausimai/Spausdinti?Length=0?Kalba=EN&KategorijaId=127&Kalbos_input=EN&Kalbos=EN&KategorijaEn_input=Pharmacology… 14/18
3/27/2019 LSMUSIS
25.64 During treatment of severe heart failure with hydrochlorotiazide several signs should be monitored:
1. Body weight and diuresis
2. Uric acid, glucose
3. Serum potassium
4. QRS voltage
5. PQ interval

25.65 Name the advantages of COX-II inhibitors over the traditional NSAIDs:
1. Les risk of ulcer
2. Les risk of stomach bleeding
3. Les risk of perforation
4. Les risk of thromboembolic complications

25.66 Name the advantages of traditional NSAIDs over the COX-II inhibitors:
1. More pronounced pain relief
2. Les risk of hyperergic reactions
3. Les risk of gastrointestinal system
4. Les risk of thromboembolic and stroke complications
5. Les risk of hepatotoxic action

25.67 Name the advantages of angiotensin receptors antagonists over the ACI inhibitors
1. More pronounced suppression of rennin-angiotensin system
2. Les risk of hypotension
3. Les risk of dry cough
4. Potassium level is not increased
5. All listed

25.68 The anticoagulant effect of warfarin increase in combination with:


1. Amiodarone
2. Estrogene
3. Statines
4. Vit K
5. Barbiturates

25.69 The effect of amiodarone on thyroid gland:


1. Does not exist
2. Can course hyperthyroidism
3. Normalizes thyroid function after thyroidytis
4. Can course hypothyroidism
5. Can improve thyroid function in districts with increased amount of iodine in food.

25.70 Differences of low molecular weight heparin in comparison with unfractionated heparin:
1. Better bioavailability after subcutaneous usage
2. Uncommon trombocytopenia
3. Longer half-life
4. Require more intensive laboratory monitoring

25.71 Name the drugs most often implicated as cause of proarrhythmia:


1. Beta-blockers
2. Sotalol
3. Amiodarone
4. Quinidine
5. Lidocaine

25.72 Metformin should be discontinued immediately in the presence of:


1. Lactatacidosis
2. Recent myocardial infarction
3. Severe peripheral blood vessel disease
4. Hypertension
5. Hypotension

25.73 What are the difference between meglitinides and sulfonylcarbamides?

lsmusis.lsmuni.lt/Klausimai/Spausdinti?Length=0?Kalba=EN&KategorijaId=127&Kalbos_input=EN&Kalbos=EN&KategorijaEn_input=Pharmacology… 15/18
3/27/2019 LSMUSIS
1. Meglitinides do not induce hypoglycemia
2. Metiglinides decrease postprandial glycemia
3. Increase dyslipidemia
4. Meglitinides are taken 30 min. before meal
5. All listed

25.74 What are criteria for treatment change in DM?


1. HbA1C >7,5%
2. Postprandial glucose is more than > 9 mmol/l
3. Fasting glucose > 7 mmol/l
4. Fasting glucose < 7 mmol/l
5. HbA1C < 7,5%

25.75 Advantages of rapid action insulin analogs:


1. Produce more physiological insulin patterns
2. Reduce risk of postprandial hypoglycemia
3. May be taken with meal
4. More freedom in meal scheduling

25.76 Advantages of insulin glargine to human insulin of intermediate action:


1. Significant decreased risk of nocturnal hypoglycemia
2. Better fasting glucose control
3. Breakfast and lunch may be missed
4. Easy for use
5. Cheaper

25.77 What is characteristic for cephalosporins:


1. Cephalosporins act as bactericidal agents in multiplying microorganisms
2. Cephalosporins and penicillins have similar spectrum of activity but different structure
3. Cephalosporins are resistant to inhibitory effect of betalactamase
4. There is partial cross-resistancy among cephalosporins and penicillins

25.78 What is an intensive insulin therapy?


1. Rapid-acting insulin analog injection with meal and 1 or 2 injection of long-action insulin analog
2. One injection of biphasic insulin mixtures in the morning
3. Three injections of short-action human insulin before meal and 1 or 2 injections of intermediate-action
human insulin
4. Two injections of biphasic insulin mixtures in the morning and in the evening
5. All listed

25.79 Severe side effects of thiamazole are:


1. Agranulocytosis, thrombopenia, aplastic anemia
2. Hypoprothrombinemia, hypoproteinemia, hepatitis
3. Vaskulitis
4. Glomerulonephritis

25.80 Name direct medical costs:


1. Costs for patient travelling to hospital
2. Hospitalization costs
3. Nursing costs
4. Lose of money due to illness
5. Out patient costs

25.81 Criteria for estimation of efficacy of oral amiodarone:


1. The blood pressure
2. The heart rate
3. The function of thyroid gland
4. The frequency of arrhythmia

25.82 Possible combinations of oral antidiabetic agents:


1. Metformin and sufonylcarbamide preparation
2. Metformin with thiazolidinedione preparation

lsmusis.lsmuni.lt/Klausimai/Spausdinti?Length=0?Kalba=EN&KategorijaId=127&Kalbos_input=EN&Kalbos=EN&KategorijaEn_input=Pharmacology… 16/18
3/27/2019 LSMUSIS
3. Sulfonylcarbamide and thiazolidinedione
4. Insulin and thiazolidinedione

25.83 In case of H. pylori resistance to Clarithromycin and Metronidazole, plus Amoxicillin hypersensitivity,
antibiotics used for eradication are:
1. Tetracycline
2. Fluorochinolon
3. Rifampicin
4. Cephalosporins
5. Aminoglycosids

25.84 Side-effects of thiazolidindiones:


1. Hepatotoxicity
2. Heart failure
3. Weight increase
4. Edema

25.85 Monitoring of blood concentration is necessary when:


1. Medicine with narrow therapeutic range
2. Medicine with high risk of side effects
3. High dose in life threatening situation
4. Long term therapy with medium size dose

IV type tasks. Choose only one best answer

25.86 52 yr. old man was admitted to hospital's reception centre. He has swelled foot with an open wound. There
is green colour, bad odour pus that is getting out of the wound. He trod upon nail 2 weeks ago. He has I type
diabetes mellitus for 15 years, which complicated into neuropathy, smokes 1 pack per day; is allergic to penicillines
(has a history of mild rash after injection). Upon admission, his T - 38 C, diminished sensitivity and deep tendon
reflexes in legs, hot, swollen right foot, with purulent fluid from wound. Lab evaluation: Le 17 x 10/l, glucose - 15,9
mmol/l (at 6 hrs.); in pus speciment: growth of Pseudomonas aeruginosa, foot rentgenography - destruction of
bone tissue. Name the right answer?
A. The is a complication of bad-controlled blood sugar level's diabetes and it is enough to correct the
treatment with insulin only.
B. There is a right foot osteomyelitis, caused by Pseudomonas aeruginosa and needs treatment with
ceftazidim and amikacin.
C. It is suspected ostemyelitis in right foot, caused by Pseudomonas aeruginosa and needs treatment with
antipseudomonal carbenicilline and cefazoline.

25.87 40- year old man has a dilated cardiomyopathy, phlebitis of the profound veins of the left leg and failure of
both parts af the heart. pulmonary embolism repeated for several times. after seven days therapy with
thrombolysis and heparin 40 000 IU/day administration clinical status improved shortly but episodes of pulmonary
embolism repeats again. Lab of activated partial thromboplastin time (aPTT) started to decrease from 79 seconds
to 22 seconds despite increased dose of Heparin. how can we improve treatment?
A. Increase the dose of heparin till 60-80.000IU per day
B. Add warfarin therapy
C. Add antithrombine III and fresh frozen plasma
D. Repeat thrombolysis and add aspirin 150 mg per day

25.1 - B 25.2 - A 25.3 - D 25.4 - A 25.5 - D 25.6 - A 25.7 - B 25.8 - D


25.9 - A 25.10 - C 25.12 - A 25.13 - B 25.14 - B 25.15 - A 25.16 - E 25.17 - D
25.18 - D 25.19 - E 25.20 - B 25.21 - C 25.22 - A 25.23 - D 25.24 - D 25.25 - A
25.26 - A 25.27 - B 25.28 - A 25.29 - A 25.30 - A 25.31 - B 25.32 - C 25.33 - C
25.34 - B 25.35 - B 25.88 - C 25.89 - D 25.90 - E 25.91 - E 25.92 - A 25.93 - E
25.94 - D 25.95 - E 25.96 - A 25.97 - A 25.98 - B 25.99 - A 25.100 - F 25.101 - F
25.102 - A 25.103 - E 25.104 - A 25.105 - D 25.106 - D 25.107 - B 25.108 - A 25.109 - B
25.110 - A 25.111 - C 25.112 - A 25.113 - C 25.114 - B 25.115 - C 25.116 - A 25.117 - C
25.118 - B 25.119 - A 25.120 - C 25.121 - D 25.122 - C 25.123 - C 25.124 - B 25.125 - D
25.126 - C 25.127 - A 25.128 - B 25.129 - A 25.130 - B 25.131 - B 25.132 - B 25.133 - C

lsmusis.lsmuni.lt/Klausimai/Spausdinti?Length=0?Kalba=EN&KategorijaId=127&Kalbos_input=EN&Kalbos=EN&KategorijaEn_input=Pharmacology… 17/18
3/27/2019 LSMUSIS
25.134 - A 25.135 - A 25.136 - A 25.137 - A 25.138 - D 25.139 - C 25.140 - C 25.141 - E
25.142 - B 25.143 - E 25.144 - C 25.145 - C 25.146 - A 25.147 - A 25.148 - C 25.149 - E
25.150 - A 25.151 - C 25.152 - C 25.153 - B 25.154 - A
25.37 25.38 25.39 25.40 25.41 25.42 25.43 25.44
4 2 2 1 1 1 1 2
4 4 2 2 2 2 4
3 3 3 3
4
25.45 25.47 25.48 25.49 25.50 25.51 25.52 25.53
1 4 1 2 4 1 1 2
3 2 4 2 3 4
3 3
4
5
25.54 25.55 25.56 25.57 25.58 25.59 25.60 25.61
4 2 1 1 2 4 1 1
4 2 3 4 2 2
3 3 3
4
5
25.62 25.63 25.64 25.65 25.66 25.67 25.68 25.69
4 1 1 1 2 1 1 2
2 2 2 4 3 3 4
3 3 3
4
5
25.70 25.71 25.72 25.73 25.74 25.75 25.76 25.77
1 2 1 2 1 1 1 1
2 4 2 4 2 2 2 2
3 3 3 3 3 3
4
25.78 25.79 25.80 25.81 25.82 25.83 25.84 25.85
1 1 1 4 1 1 1 1
3 2 3 2 2 2 3
3 3 3 3
4 4
25.86 - B 25.87 - C

lsmusis.lsmuni.lt/Klausimai/Spausdinti?Length=0?Kalba=EN&KategorijaId=127&Kalbos_input=EN&Kalbos=EN&KategorijaEn_input=Pharmacology… 18/18
3/27/2019 LSMUSIS

Medical diagnostics
I type tasks. Choose only one best answer

49.1 Which breathing is characterized by alternating periods of normal ventilation and apnea:
A. Cheyne-Stokes
B. Kussmaul’s
C. Biot’s
D. Grocco’s

49.2 Which sign is characteristic for ketoacidotic coma:


A. Normal breathing
B. Deranged consciousness
C. Moist skin
D. Acetone odor

49.3 Ratio of inspiration and expiration 1:2 or larger during auscultation of lung is:
A. The lung substance itself is solid (consolidated)
B. In case of increasment airness in the lung
C. In case of pleural effusion in the projection of effusion
D. In case of air in pleural cavity
E. In case of cavity in the lung

49.4 Which breath sound is audible during auscultation of healthy person:


A. Amphorophony audible in special site
B. Only bronchial breath sound
C. Bronchovesicular breath sound
D. Vesicular and bronchial breath sounds
E. Vesicular breath sound

49.5 Which sign helps to differentiate crepitation and fine rales:


A. Crepitation diminishes after cough
B. Crepitation is better audible at the beginning of inspiration
C. Crepitation is audible only at the end of inspiration
D. Crepitation is audible only during expiration
E. Rales can not be altered by cough

49.6 What is the cause of cardiac beat:


A. Left ventricular hypertrophy
B. Right atrial hypertrophy
C. Right ventricular hypertrophy
D. Right ventricular failure
E. Heart denudation

49.7 What is the semiotic meaning of palpable thrill:


A. Only regurgitation
B. Stenosis and high degree of regurgitation
C. Myocardial pathology
D. Fibrinous pericarditis

49.8 What is the semiotic meaning of carotids pulsation:


A. The sign of right ventricular failure
B. The sign of left ventricular failure
C. The sign of aortic regurgitation
D. The sign of tricuspid regurgitation

49.9 What is the semiotic meaning of jugular pulsation:


A. Aortic regurgitation
B. Mitral regurgitation
C. Tricuspid regurgitation
D. Right ventricular failure

49.10 Indicate the cause of extracardiac murmurs:


lsmusis.lsmuni.lt/Klausimai/Spausdinti?Length=0?Kalba=EN&KategorijaId=151&Kalbos_input=EN&Kalbos=EN&KategorijaEn_input=Medical+diagno… 1/13
3/27/2019 LSMUSIS
A. Increased velocity of blood flow
B. Due to pathological changes in pericardium
C. Reduced blood viscosity
D. Relative valve incompetency

49.11 Indicate the cause of inward movement of the apex beat:


A. Exudative pericarditis
B. Obesity
C. Massive chest muscles
D. Pleuropericardial adhesions (mediastinopericarditis)

49.12 Indicate the cause of pulse deficit:


A. Third degree atrioventricular block
B. Atrial fibrillation
C. Sinoatrial block
D. Regular atrial flutter

49.13 In which case is the apex beat in the fifth intercostal space at the midclavicular line strong and lifted?
A. Mitral stenosis
B. Significant tachycardia
C. The first stage of arterial hypertension
D. The second stage of arterial hypertension

49.14 What is the cause of increased venous pressure?


A. Tricuspid regurgitation
B. Left ventricular failure
C. Right ventricular failure
D. Shock

49.15 Chronic progressive dysphagia is characteristic for:


A. Corrosive esophagitis
B. Perforation of esophagus
C. Tumor of esophagus
D. Achalasia of stomach

49.16 Which blood biochemical parameter does not mean hepatic cytolysis?
A. Increased activity of glutamatpiruvattransaminase
B. Increased activity of glutamatoxaloacetattransaminase
C. Increased activity of ɣ-glutamyltransferase
D. Increased activity of glutamatdehydrogenase

49.17 What is characteristic for prehepatic jaundice?


A. Unconjugated hyperbilirubinemia
B. Increased activity of aminotransferases
C. Conjugated hyperbilirubinemia
D. Absence of stercobilin in feces

49.18 What are the causes of telangiectasis?


A. Impaired destruction of estrogens in the liver
B. Conjugated hyperbilirubinemia
C. Hemorrhages in the skin
D. Skin itching and excoriations

49.19 What is characteristic for portal hypertension?


A. Hepatocellular jaundice
B. Hyperazotemia
C. Splenomegaly
D. Skin itching

49.20 Which biochemical parameter does not mean cholestasis?


A. Decreased activity of serum cholinesterase
B. Increased activity of serum ɣ-glutamyltransferase
C. Hypercholesterolemia
lsmusis.lsmuni.lt/Klausimai/Spausdinti?Length=0?Kalba=EN&KategorijaId=151&Kalbos_input=EN&Kalbos=EN&KategorijaEn_input=Medical+diagno… 2/13
3/27/2019 LSMUSIS
D. Conjugated hyperbilirubinemia

49.21 What symptom is to be called dysuria?


A. Painful urination
B. A compelling need to urinate
C. Frequent and painful urination
D. Urination is frequent and small in volume

49.22 What does specific gravity of normal urine depend on?


A. Concentration of protein in urine
B. Concentration of glucose in urine
C. Concentration of urea in urine
D. Concentration of creatinine in urine

49.23 Which symptom is characteristic for hemorrhagic syndrome?


A. Leukocytosis
B. Eosinophilia
C. Thrombocytopenia
D. Hypoproteinemia

49.24 Which is the earliest sign of chronic renal failure?


A. Increased concentration of creatinine in the urine
B. Reduced creatinine clearance
C. Increased concentration of urea in the urine
D. Pathological changes in renal concentrating and dilution tests

49.25 What symptom is to be called “ polydipsia”?


A. Increased appetite
B. Increased urination
C. Oral dryness and thirst
D. Increased sweating

49.26 Which symptom is characteristic for hypoglycemic coma?


A. Deranged consciousness
B. Deep and rare pathological breathing
C. Increased sweating
D. Soft on pressure eye balls

49.27 Which symptom is characteristic for Raynaud’s syndrome?


A. Joint pain
B. Muscle pain
C. Morning stiffness
D. Tingling and numbness of the affected digits

49.28 When is oral glucose tolerance test performed?


A. In age over 30 years
B. In case of hypoglycemia of unknown etiology
C. Increased blood pressure
D. In case of increased attraction to sweets

II type tasks. For each numbered item,selct the one lettered heading that is most closely asssciated with it

49.29 What is the distribution of subcutaneous fats in the following syndromes?


1 - Cushing's syndrome
2 - Hypothyroidism
3 - Alimentary obesity
4 - Hypogonadism
A. Proportional distribution of subcutaneous fats, rounded body shape, and increased body mass index
B. Fat deposits in abdominal region, buttocks, and the chest
C. Truncal obesity with prominent supraclavicular and dorsal cervical fat pads (“ Buffalo”) hump, the
rounded face, but quite slender extremities.
D. Increased subcutaneous layer over wrists, ankles, supraclavicular region.

lsmusis.lsmuni.lt/Klausimai/Spausdinti?Length=0?Kalba=EN&KategorijaId=151&Kalbos_input=EN&Kalbos=EN&KategorijaEn_input=Medical+diagno… 3/13
3/27/2019 LSMUSIS
49.30 Which joints are typically affected by the following diseases?
1 - Rheumatoid arthritis
2 - Degenerative joint disease
3 - Gout
4 - Rheumatic polyarthritis
A. Symmetric affection of small joints
B. Asymmetric affection of big joints
C. Short-term affection of all joints
D. Asymmetric affection of small joints

49.31 Which pathological changes of sounds are characteristic for conditions listed below?
1 - Ventricular gallop
2 - Atrial gallop
3 - Splitted S1
A. First degree atrioventricular block
B. Bundle branch block
C. Complete atrioventricular block
D. Q wave myocardial infarction

49.32 Which radiation of heart murmurs is characteristic for the following conditions?
1 - Carotids
2 - Left armpit
3 - Heart apex
4 - Without radiation
A. Tricuspid regurgitation
B. Mitral regurgitation
C. Aortic regurgitation
D. Aortic stenosis

49.33 What ECG changes are characteristic for conditions listed below?
1 - Acute myocardial infarction
2 - Adams-Stokes attacks
3 - Acute rheumatic myocarditis
A. Pathologic Q wave and ST dislocation
B. P-Q prolongation
C. Complete atrioventricular block

49.34 Which type of hemorrhagic diatheses is characteristic for the following syndromes and diseases?
1 - Hemorrhagic vasculitis
2 - Hemophilia
3 - Chronic hepatic failure
4 - Idiopathic thrombocytopenic purpura
A. Coagulopathy
B. Vasculopathy
C. Thrombocytopathy

49.35 What stool specimen is characteristic for the following syndromes?


1 - Acid dyspepsia
2 - Intestinal putrid dyspepsia
3 - Gastrogenic failure
4 - Insufficient exocrine function of the pancreas
A. The consistency is soft (so-called formed stools), alkaline reaction,kre atorrhea, the presence of
connective tissue (+) and vegetable cellular tissue (++), the mucus is absent.
B. The stool is of porridge consistency, alkaline reaction,kreatorrhea, the presence of connective tissue
(+), neutral fats (+++),vegetable cellular tissue (+++), the mucus is absent.
C. The stool is of porridge consistency, foamy, acid reaction, the connective tissue is absent, muscle
fibers present (+), the presence of starch grains (+++), iodophilic flora (+)
D. The stool is liquid, alkaline reaction, the presence of muscle fibers (+) and iodophilic flora (+), the
absence of connective tissue and starch grains

49.36 Which mechanisms are responsible for changes of sound transmission in the lungs in the following
pathologic conditions?
lsmusis.lsmuni.lt/Klausimai/Spausdinti?Length=0?Kalba=EN&KategorijaId=151&Kalbos_input=EN&Kalbos=EN&KategorijaEn_input=Medical+diagno… 4/13
3/27/2019 LSMUSIS
1 - Pulmonary consolidation
2 - Increased airiness of the lungs
3 - Cavity in the lung
4 - Compressive atelectasis
A. Shortening of a way for the sound transmission
B. Increased density of affected area of the lung
C. Overfilling of the lungs with air and reduced elasticity

49.37 Which symptoms and signs are characteristic for the following syndromes?
1 - Chronic hepatic failure
2 - Posthepatic jaundice
3 - Portal hypertension
A. Telangiectasis
B. Enlargment of abdomen
C. Intense skin itching

49.38 Which symptoms and signs are characteristic for the following syndromes?
1 - Acute adrenocortical insufficiency
2 - Hyperosmotic coma
3 - Hypoglycemia
4 - Ketoacidotic coma
A. Very moist and sweating skin
B. Increased serum concentration of urea and creatinine
C. Nausea, vomiting, diarrhea
D. Kusmaul’s breathing

49.39 Which signs are characteristic for the following conditions listed below?
1 - Permanent overfilling of the jugular veins
2 - Increased pulsation of carotids
3 - Paradoxical pulsation of the jugular veins
A. Aortic regurgitation
B. Adhesive pericarditis
C. Left ventricular failure
D. Right ventricular failure

49.40 What pathological changes may be found in sputum in the following conditions?
1 - Asthma
2 - Pneumonia
3 - Bronchiectases
4 - Acute bronchitis
A. Mucous and small in volume
B. Three layers
C. Curschmann’s spirals

49.41 Which symptoms and signs are most typical for the endocrine disorders listed below?
1 - Dry and swelling skin, eyelids' droop, dull facial expression.
2 - The rounded “moon” face, hypertrichosis, purple striae on the abdomen and the inner surface of the
thighs.
3 - Protrusion of eyeballs, mimic (active,” frightened “) face expression, the bad nutrition, the moist skin,
tremor of hands and eyelids.
4 - Coarse facial features and skin wrinkles, increased hand and feet size.
A. Hyperthyroidism
B. Hyperproduction of glucocorticosteroids
C. Hypothyroidism
D. Hyperproduction of somatotropin

49.42 Which type of pleural fluid is characterized by the following findings in the pleural fluid?
1 - Pleural fluid protein 33 g/ l
2 - Pleural fluid amount 1800 ml
3 - 2X10 9 leukocytes in pleural fluid
4 - Pleural fluid to serum lactate dehydrogenase (LDH) ratio is 0.5
A. Exudate
lsmusis.lsmuni.lt/Klausimai/Spausdinti?Length=0?Kalba=EN&KategorijaId=151&Kalbos_input=EN&Kalbos=EN&KategorijaEn_input=Medical+diagno… 5/13
3/27/2019 LSMUSIS
B. Transudate
C. The finding has no diagnostic significance

49.43 Which hyperenzymopathy is most specific for each type of jaundice?


1 - Prehepatic jaundice
2 - Hepatocellular jaundice
3 - Cholestatic jaundice
A. Increased serum activity of aminotransferases
B. Increased serum activity of gamma-glutamyl tranferase
C. Normal serum activity of aminotransferases and gamma-glutamyl tranferase

49.44 Which findings are characteristic for the following syndromes?


1 - Renal eclampsia
2 - Nephrotic syndrome
3 - Nephritic syndrome
4 - Renal failure
A. Hematuria
B. Hypoproteinemia
C. Convulsive seizures (tonic and clonic contractions)
D. Hyperazotemia

49.45 Which symptoms and signs are most typical for the following syndromes?
1 - Portal hypertension
2 - Acute hepatic failure
3 - Chronic hepatic failure
A. Significant hepatocellular jaundice
B. Skin telangiectasis and palmar erythema
C. Splenomegaly

III type tasks. For each question there is one or more correct answers:
A – if correct answers are 1,2,3
B – if correct answers are 1 and 3
C – if correct answers are 2 and 4
D – if correct answer is 4
E – if correct are all answers above

49.46 Which symptoms and signs are characteristic for pulmonary consolidation syndrome?
1. Locally increased translucency on X-rays
2. Bronchial breath sounds
3. Sibilant wheezes
4. Dullness on percussion
5. Amphoric breath sounds

49.47 Which symptom is characteristic for the cavity in the lung ?


1. Dullness on percussion
2. Reduced vesicular breath sounds
3. Increased translucency of the whole lung on X-rays
4. Local tympany on percussion
5. Sputum production (20 ml daily)

49.48 Which symptoms and signs occur in increased airiness of the lungs?
1. Reduced vesicular breath sounds over the whole chest
2. Translucent both lungs on X-rays
3. Hyperresonance on percussion
4. Sibilant wheezes
5. Tympany note on percussion

49.49 Which symptoms and signs occur in air accumulation in the pleural cavity?
1. Reduced vesicular breath sounds
2. An area of locally increased density on X-rays
3. Tympany note on percussion
4. Hyperresonance on percussion
lsmusis.lsmuni.lt/Klausimai/Spausdinti?Length=0?Kalba=EN&KategorijaId=151&Kalbos_input=EN&Kalbos=EN&KategorijaEn_input=Medical+diagno… 6/13
3/27/2019 LSMUSIS
5. Soft rales

49.50 Which symptoms and signs occur in fluid accumulation in the pleural cavity?
1. Diminished vesicular breath sounds
2. Homogeneous increased density at the bottom of the chest on X-rays
3. Dullness on percussion
4. Reduced voice sounds
5. Mediastinum shifted toward the opposite side on X-rays

49.51 Functional residual capacity of the lungs consists of:


1. Residual volume
2. Tidal volume
3. Expiratory reserve volume
4. Inspiratory reserve volume
5. Forced expiratory volume in 1 second

49.52 Systolic regurgitation murmur is characteristic for:


1. The prolapsed mitral valve
2. Tricuspid regurgitation
3. Mitral regurgitation
4. Aortic regurgitation

49.53 What is characteristic for the aortic configuration of the heart?


1. Isolated hypertrophy of left ventricle
2. Straightening of the left cardiac border
3. Left cardiac border becomes more apparent
4. Left atrial and left ventricular enlargement

49.54 When is the distension of the neck veins observed?


1. Decompensated cor pulmonale
2. Exudative pericarditis
3. Prolonged paroxysmal tachycardia
4. Adhesive pericarditis

49.55 Which heart sounds are better heard with the bell?
1. The physiologic third heart sound
2. Diastolic aortic regurgitation murmur
3. The fourth heart sound
4. Pericardial friction rub

49.56 Which auscultatory phenomena are better heard by forced held inspiration?
1. Murmurs due to mitral valve affection
2. Murmurs due to tricuspid valve affection
3. Splitting of the second heart sound due to left bundle branch block
4. Splitting of the second heart sound due to right bundle branch block, physiologic splitting of the second
heart sound

49.57 Which murmurs are well audible over Erb’s area?


1. Diastolic murmur due to mitral stenosis
2. Diastolic murmur due to aortic regurgitation
3. Systolic murmur due to aortic stenosis
4. Systolic murmur due to mitral regurgitation

49.58 What are characteristics of mid-diastolic crescendo murmur audible in mitral stenosis?
1. Occurs following a short pause after the second heart sound.
2. Occurs in early diastole immediately after the second heart sound.
3. Best heard at the apex
4. Best heard over Erb’s area

49.59 What are the types of triple rhythm?


1. Gallop rhythm
2. The physiologic third sound
3. Opening sound of the mitral valve
lsmusis.lsmuni.lt/Klausimai/Spausdinti?Length=0?Kalba=EN&KategorijaId=151&Kalbos_input=EN&Kalbos=EN&KategorijaEn_input=Medical+diagno… 7/13
3/27/2019 LSMUSIS
4. Prosthetic valve sound

49.60 What is the semiotic meaning of clubbing fingers?


1. Congenital heart disease with central cyanosis
2. Chronic purulent lung infections
3. Subacute infective endocarditis
4. Liver cirrhosis

49.61 What are the causes of increased pulse pressure?


1. Aortic regurgitation
2. Thyrotoxicosis
3. Atherosclerosis
4. Mitral regurgitation

49.62 What are causes of malabsorbtion?


1. Intestinal resection
2. Radial enteritis
3. Insufficiency of digestive enzymes
4. Starving

49.63 What are the causes of melena?


1. The use of iron containing medications
2. Duodenum bleeding
3. Achalasia of oesophagus
4. Bleeding into stomach

49.64 What symptoms occur in malabsorbtion syndrome?


1. Edema
2. Diarrhea
3. Anemia
4. Melena

49.65 Which biochemical changes indicate hepatocytolysis?


1. Hypercholesterolemia
2. Increased activity of glutamatdehidrogenase
3. Increased activity of alkaline phosphatase
4. Increased activity of aminotransferases

49.66 What symptoms and signs occur in acute hepatic failure?


1. Dyspepsia syndrome
2. Hepatocellular jaundice
3. Hepatomegaly
4. Asthenia syndrome

49.67 What symptoms and signs DO NOT occur in acute hepatic failure?
1. Dyspepsia syndrome
2. Hemorrhagic syndrome
3. Hepatomegaly
4. Skin telangiectasy

49.68 Which symptoms and signs DO NOT occur in prehepatic jaundice?


1. Conjugated hyperbilirubinemia
2. Increased serum aminotransferases activity
3. Normal urinary urobilinogen activity
4. Hypoprothrombinemia

49.69 Which symptoms and signs are characteristic for portal hypertension?
1. Splenomegaly
2. Cholestatic jaundice
3. Ascites
4. Edema

49.70 Which symptoms are characteristic for hepatorenal syndrome?


lsmusis.lsmuni.lt/Klausimai/Spausdinti?Length=0?Kalba=EN&KategorijaId=151&Kalbos_input=EN&Kalbos=EN&KategorijaEn_input=Medical+diagno… 8/13
3/27/2019 LSMUSIS
1. Hyposthenuria
2. Normal urine specific gravity
3. Conjugated hyperbilirubinemia
4. Hyperazotemia

49.71 What symptoms and signs DO NOT occur in hepatic cholestatic jaundice?
1. Increased serum gamma-glutamyltransferase activity
2. Mixed hyperbilirubinemia
3. Conjugated hyperbilirubinemia
4. Obstruction of ductus choledochus

49.72 What are the causes of hemorrhagic syndrome in hepatobiliary diseases?


1. Impaired synthesis of prothrombin
2. Impaired synthesis of VIII clotting factor
3. Impaired synthesis of vitamin K depending clotting factors
4. Thrombocytopenia

49.73 What symptoms and signs are characteristic for nephritic syndrome?
1. Arterial hypertension
2. Heavy proteinuria
3. Hematuria
4. Hypoproteinemia

49.74 What are the causes of functional proteinuria?


1. Fever
2. After work
3. Ortostasis
4. Systemic congestion

49.75 What are symptoms and signs of renal eclampsia syndrome?


1. Tonic and clonic convulsive seizures
2. The bitten tongue
3. Coma
4. Retrograde amnesia

49.76 What are clinical symptoms of acute leukemia?


1. Generalized enlargement of lymphnodes
2. Subcutaneous hemorrhages
3. Stomach achilia
4. Necrotizing angina

49.77 What symptoms and signs occur in hyperglycemic ketoacidotic coma?


1. Kusmaul’s respiration
2. Urine without acetone
3. Soft eyeballs
4. Moist skin

49.78 Which symptoms and signs are characteristic for hyperthyroidism?


1. Purple and moist skin
2. Protrusion of eyeballs
3. Expressed face mimic
4. Weight gain

49.79 What skin is characteristic for hypothyroidism?


1. Dry
2. Coarse and puffy
3. Scurfy
4. Pale

49.80 In which cases a normoglycemic patient should be administered oral glucose tolerance test?
1. Frequent skin furunculosis
2. Unjustified thirst
3. Skin itching (particularly in genitals)
lsmusis.lsmuni.lt/Klausimai/Spausdinti?Length=0?Kalba=EN&KategorijaId=151&Kalbos_input=EN&Kalbos=EN&KategorijaEn_input=Medical+diagno… 9/13
3/27/2019 LSMUSIS
4. Diabetes mellitus in relatives

49.81 What symptoms and signs are characteristic for arthropathy syndrome?
1. Increased tendon reflexes
2. Joint stiffness in the morning
3. Numbness of the hands
4. Reduced mobility of joints

IV type tasks. Choose only one best answer

49.82 50-year-old patient developed fever up to 37,5 ₀C. He suffered from right-sided pleuritic pain, productive
cough with expectoration of yellow sputum. Later permanent dyspnea occurred. On percussion of the chest right
side dullness was revealed, on auscultation – breath sounds were absent.Which syndrome do you suspect?
A. Pulmonary consolidation
B. Increased airiness of the lungs
C. Fluid in pleural cavity

49.83 A 50-year-old patient developed fever up to 37,5 ₀C. He suffered from right-sided pleuritic pain, productive
cough with expectoration of yellow sputum. Later permanent dyspnea occurred. On percussion of the chest right
side dullness was revealed, on auscultation – breath sounds were absent.What changes of Tiffeneau index are to
be expected?
A. 60–70%
B. Over 70%
C. Less 60%

49.84 A patient with a long history of chronic bronchitis reported that one month ago subicterus in sclera occurred,
later the skin became yellow, swelling of the legs developed, daily urinary output decreased.Which type of heart
failure may be present in this patient?
A. Left heart failure
B. Right heart failure
C. Total heart failure
D. Hypodiastolic heart failure

49.85 A patient with a long history of chronic bronchitis reported that one month ago subicterus in sclera occurred,
later the skin became yellow, swelling of the legs developed, daily urinary output decreased.Which symptom
during palpation may be found in this case?
A. Strong, lifted, dilated apex beat
B. Soft, small, frequent radial artery pulse
C. Cardiac beat
D. Inward apex beat

49.86 A patient complains of severe exertional dyspnea. Inspection revealed significant generalized peripheral
cyanosis, distended jugular veins (without systolic pulsation), enlargement of abdomen, swelling of legs. The heart
borders were normal on percussion. Auscultation of the heart revealed tachycardia and diminished heart sounds
(murmurs were absent). What is the diagnosis?
A. Tricuspid regurgitation
B. Acute rheumatic myocarditis
C. Adhesive pericarditis

49.87 A patient complains of severe exertional dyspnea. Inspection revealed significant generalized peripheral
cyanosis, distended jugular veins (without systolic pulsation), enlargement of abdomen, swelling of legs. The heart
borders were normal on percussion. Auscultation of the heart revealed tachycardia and diminished heart sounds
(murmurs were absent). Which type of heart failure may be present in this case?
A. Total heart failure
B. Left heart failure
C. Hypodiastolic heart failure

49.88 The results of prophylactic examination of 42-year-old male: blood pressure 150/70 mmHg, diminished S1 at
the apex, mild middiastolic murmur without radiation at the apex, diminished S2 over the aorta, prolonged IV₀
protodiastolic murmur in Erb’s area , radiating to the apex. What is the diagnosis?
A. Dilated cardiomyopathy
B. Hypertrophic cardiomyopathy
lsmusis.lsmuni.lt/Klausimai/Spausdinti?Length=0?Kalba=EN&KategorijaId=151&Kalbos_input=EN&Kalbos=EN&KategorijaEn_input=Medical+diagn… 10/13
3/27/2019 LSMUSIS
C. Mitral regurgitation
D. Mitral stenosis and mitral regurgitation
E. Aortic regurgitation

49.89 Fifteen years ago a 48-year-old male suffered from acute arthritis. After that he was well for a long time, he
had an easy job. Not far ago he referred to a doctor due to dizziness. Physical examination revealed pallor in the
face, capillary pulsation, blood pressure 150/40 mmHg, pulse rate 80 /min, radial pulse of normal volume, the apex
beat palpable in the 6th intercostal space 1,5 cm to left from midclavicular line. What is the diagnosis?
A. Mitral regurgitation
B. Tricuspid regurgitation
C. Postmyocarditic myocardiosclerosis
D. Aortic stenosis
E. Aortic regurgitation

49.90 A 36-year-old female is an intellectual worker. She suffered from acute arthritis some years ago. Now she is
well. The prophylactic examination revealed: accentuated S1 at the apex, IIIº presystolic murmur at the apex.
What is the diagnosis?
A. Mitral regurgitation
B. Mitral stenosis
C. Mitral stenosis and mitral regurgitation (with predominant stenosis)
D. Postmyocarditic myocardiosclerosis
E. Mitral regurgitation and mitral stenosis (with predominant regurgitation)

49.91 A 32-year-old female suffered from acute arthritis 11 years ago, she was hospitalized for four weeks. She
complains of atypical pain in precordial area, and mild dyspnea during moderate physical exercise. Findings of
physical examination: mild cyanosis around the lips, normal apex beat, mild cardiac beat, diastolic palpable thrill.
What is the diagnosis?
A. Aortic stenosis
B. Mitral regurgitation
C. Mitral stenosis
D. Tricuspid regurgitation
E. Mitral and tricuspid valvular disease

49.92 A patient with arterial hypertension took a large dose of diuretics. Some hours later during his getting up
suddenly he felt severe weakness, dizziness, tachycardia. He became pale, but consciousness was not lost. Due
to disability to stand he stayed in a bed and some minutes later he was better. Which syndrome may be diagnosed
in this case?
A. Syncope
B. Collapse
C. Shock
D. Eclampsia

49.93 A patient with arterial hypertension took a large dose of diuretics. Some hours later during his getting up
suddenly he felt severe weakness, dizziness, tachycardia. He became pale, but consciousness was not lost. Due
to disability to stand he stayed in a bed and some minutes later he was better. Which testing could confirm the
diagnosis?
A. The measurement of basal arterial pressure
B. The measurement of central venous pressure
C. Orthostatic test
D. Biochemistry of blood

49.94 Profuse bleeding from the nose and genitals started in a 32-year-age woman. Findings of physical
examination: bruises in the skin, enlargement of the spleen, blood platelet count 30x109/l. What is the diagnosis?
A. Acute leukemia
B. Posthemorrhagic anemia
C. Idiopathic thrombocytopenic purpura
D. Hemophilia

49.95 An unconscious man was found in the street. A diabetic passport was detected in the pocket of his jacket.
Findings of physical examination: pupillary and patellar reflexes were absent. The skin is moist. What is the
diagnosis?
A. Hypoglycemic coma

lsmusis.lsmuni.lt/Klausimai/Spausdinti?Length=0?Kalba=EN&KategorijaId=151&Kalbos_input=EN&Kalbos=EN&KategorijaEn_input=Medical+diagn… 11/13
3/27/2019 LSMUSIS
B. Hyperglycemic ketoacidotic coma
C. Syncope
D. Sopor

49.96 An unconscious man was found in the street. A diabetic passport was detected in the pocket of his jacket.
Findings of physical examination: pupillary and patellar reflexes were absent. The skin is moist. What would be the
results of laboratory examination?
A. Serum glucose concentration more than 8 mmol/l
B. Acetone in urine
C. Serum glucose concentration less than 3,33 mmol/l
D. Proteinuria

49.97 An unconscious man was found in the street. A diabetic passport was detected in the pocket of his jacket.
Findings of physical examination: pupillary and patellar reflexes were absent. The skin is moist. What should be
the first aid?
A. Insulin (intramuscular)
B. 40℅ glucose solution (intravenous)
C. Sugar per os
D. Infusion of fluids (intravenous)

49.98 A 30-year-age female suffers from paroxysmal dyspnea. She is ill for more than 10 years, she attends
outpatient clinic, the effect of treatment is moderate, temporary. An exacerbation developed a few days ago: the
attacks of dyspnea became more frequent and more severe, and the patient was hospitalized. The findings of lung
function examination: FVC – 3,2 l (norma 3,8 l; 84% of predicted value), FEV1 – 1,6 l/s (norma – 3,1 l; 51% of
predicted value), Gensler index – 50% (norma – 85,9%), MEF50 – 0,9 l/s (norma – 5 l/s; 18% of predicted value).
Which pattern of impaired ventilation is present?
A. Obstructive
B. Restrictive
C. Mixed

49.99 A 30-year-age female suffers from paroxysmal dyspnea. She is ill for more than 10 years, she attends
outpatient clinic, the effect of treatment is moderate, temporary. An exacerbation developed a few days ago: the
attacks of dyspnea became more frequent and more severe, and the patient was hospitalized. The findings of lung
function examination: FVC – 3,2 l (norma 3,8 l; 84% of predicted value), FEV1 – 1,6 l/s (norma – 3,1 l; 51% of
predicted value), Gensler index – 50% (norma – 85,9%), MEF50 – 0,9 l/s (norma – 5 l/s; 18% of predicted value).
What condition could be responsible for this type of impaired ventilation?
A. Pneumonia
B. Asthma
C. Tracheitis
D. Exudative pleuritis

49.100 A 50 year-age male suffers from permanent dyspnea, cough with moderate sputum production. He is ill
since juvenility. He attends the outpatient clinic irregular. The findings of lung function examination: VC – 1,3 l
(norma 2,5 l; 52% of predicted value), FVC – 1,5 l (norma 2,5 l; 60% of predicted value), FEV1 – 0,8 l (norma – 2 l;
40% of predicted value), Gensler index – 60% (norma – 82,2%; 72% of predictive value), MEF50 – 0,5 l/s (norma
– 4,1 l/s; 12% of predicted value). Which pattern of impaired ventilation is present?
A. Obstructive
B. Restrictive
C. Mixed

49.101 A 50 year-age male suffers from permanent dyspnea, cough with moderate sputum production. He is ill
since juvenility. He attends the outpatient clinic irregular. The findings of lung function examination: VC – 1,3 l
(norma 2,5 l; 52% of predicted value), FVC – 1,5 l (norma 2,5 l; 60% of predicted value), FEV1 – 0,8 l (norma – 2 l;
40% of predicted value), Gensler index – 60% (norma – 82,2%; 72% of predictive value), MEF50 – 0,5 l/s (norma
– 4,1 l/s; 12% of predicted value). What condition could be responsible for this type of impaired ventilation?
A. Pneumonia
B. Exudative pleuritis
C. Kyphoscoliosis with concomitant bronchitis
D. Tracheitis

49.1 - C 49.2 - D 49.3 - A 49.4 - D 49.5 - C 49.6 - C 49.7 - B 49.8 - C


49.9 - C 49.10 - B 49.11 - D 49.12 - B 49.13 - D 49.14 - C 49.15 - C 49.16 - C
lsmusis.lsmuni.lt/Klausimai/Spausdinti?Length=0?Kalba=EN&KategorijaId=151&Kalbos_input=EN&Kalbos=EN&KategorijaEn_input=Medical+diagn… 12/13
3/27/2019 LSMUSIS
49.17 - A 49.18 - A 49.19 - C 49.20 - A 49.21 - C 49.22 - C 49.23 - C 49.24 - D
49.25 - C 49.26 - C 49.27 - D 49.28 - C
49.29 49.30 49.31 49.32 49.33 49.34 49.35 49.36
1-C 1-A 1-D 1-D 1-A 1-B 1-C 1-B
2-D 2-B 2-A 2-B 2-C 2-A 2-D 2-C
3-A 3-D 3-B 3-C 3-B 3-A 3-A 3-A
4-B 4-C 4-A 4-C 4-B 4-B
49.37 49.38 49.39 49.40 49.41 49.42 49.43 49.44
1-A 1-C 1-D 1-C 1-C 1-A 1-C 1-C
2-C 2-B 2-A 2-A 2-B 2-C 2-A 2-B
3-B 3-A 3-B 3-B 3-A 3-A 3-B 3-A
4-D 4-A 4-D 4-B 4-D
49.45
1-C
2-A
3-B
49.46 49.47 49.48 49.49 49.50 49.51 49.52 49.53
2 4 1 1 1 1 1 1
4 2 3 2 3 2 3
3 3 3
4
5
49.54 49.55 49.56 49.57 49.58 49.59 49.60 49.61
1 1 2 2 1 1 1 1
2 3 4 4 3 2 2 2
3 3 3 3
4 4
49.62 49.63 49.64 49.65 49.66 49.67 49.68 49.69
1 2 1 2 1 4 1 1
2 4 2 4 2 2 3
3 3 3 3
4 4
49.70 49.71 49.72 49.73 49.74 49.75 49.76 49.77
2 2 1 1 1 1 2 1
4 4 3 3 2 2 4 3
3 3
4 4
49.78 49.79 49.80 49.81
1 1 1 2
2 2 2 4
3 3 3
4 4
49.82 - C 49.83 - B 49.84 - B 49.85 - C 49.86 - C 49.87 - C 49.88 - E 49.89 - E
49.90 - B 49.91 - C 49.92 - B 49.93 - C 49.94 - C 49.95 - A 49.96 - C 49.97 - B
49.98 - A 49.99 - B 49.100 - C 49.101 - C

lsmusis.lsmuni.lt/Klausimai/Spausdinti?Length=0?Kalba=EN&KategorijaId=151&Kalbos_input=EN&Kalbos=EN&KategorijaEn_input=Medical+diagn… 13/13
3/27/2019 LSMUSIS

Cardiology
I type tasks. Choose only one best answer

50.1 The increase of which marker is the most specific sign of myocardium necrosis?
A. Creatine Kinase–MB
B. Troponin T or I
C. Myoglobin
D. Copeptin

50.2 "In acute myocardial infarction, when is cardiogenic shock likely to develop?
A. When 15% of the left ventricle myocardium is injured
B. When 25% of the left ventricle myocardium is injured
C. When more than 40 % of the left ventricle myocardium is injured

50.3 Which medicine (drugs) shoudnt be used for treatment of the acute anterior ST segment elevation myocardial
infarction?
A. Beta-Adrenergic Blockers
B. Calcium channel blockers
C. Thrombolytics
D. Ticagrelor
E. ACE inhibitors

50.4 Which statement concerning mitral valve stenosis is incorrect?


A. Rheumatic dissease is the main cause of mitral valve stenosis.
B. Pulmonary hypertension is frequent complication of mitral stenosis
C. Mitral valve replacement is not common treatment method for severe mitral stenosis
D. Severe mitral stenosis is diagnosed when mitral valve area is less than 1 cm2

50.5 When is the earliest time for the operation in case of an aortic stenosis:
A. In case of the 1st functional NYHA class
B. In the presence of symptoms of heart failure
C. When starts paroxysmal atrial fibrilation
D. After the symptoms of angina pectoris become clear and the mean pressure gradient exceeds 40 mmHg

50.6 Which change of heart sounds may allow to suspect ventricular diastolic dysfunction:
A. Decresed intensity of first heart sound (S1)
B. Splitting of first heart sound (S1)
C. Systolic click
D. Presystolic gallop (arrise of fourth (S4) heart sound)

50.7 2D echocardiography revealed 4 mm interatrial septal defect with shunting from left to right atrium. What
clinical complication can occur?
A. Pulmonary hypertension
B. Arrhythmias
C. Stroke
D. Pulmonary embolism

50.8 Which test most exactly indicates the degree of the mitral stenosis?
A. Two-dimensional Doppler echocardiography
B. Chest X-ray
C. M- mode echocardiography
D. Transoesophageal three-dimensional echocardiography

50.9 What is the most appropriate management for the patient with proximal left anterior descending artery 75
percent stenosis when SYNTAX score is more than 33?
A. Optimal medical treatment only
B. Percutaneous coronary intervention with stenting
C. Follow-up and correction of risk factors
D. Coronary artery bypass graft operation, risk factor correction and optimal medical treatment

50.10 Which cardiomyopathy manifests with left ventricular systolic dysfunction?


A. Hypertrophic
lsmusis.lsmuni.lt/Klausimai/Spausdinti?Length=0?Kalba=EN&KategorijaId=152&Kalbos_input=EN&Kalbos=EN&KategorijaEn_input=Cardiology&Kate… 1/9
3/27/2019 LSMUSIS
B. Dilated
C. Infiltrative

50.11 Which sign is not characteristic for hypertrophic cardiomyopathy?


A. Diastolic dysfunction
B. Genetic origin of the disease
C. Left ventricular outflow trackt obstruction
D. Thinning of the interventricular septum

50.12 Which statement about the treatment of myocarditis is incorrect?


A. Heart failure treatment is important part of myocarditis treatment
B. Non-steroidal anti-inflammatory drugs (NSAIDs) are important for myocarditis treatment
C. Limitation of physical activity is important part of myocarditis treatment
D. Left and (or right) ventricular assisted devices could be used for severe myocarditis treatment

50.13 In which case a pericardiocentesis should be performed?


A. In case of constrictive pericarditis
B. In case of pericardial effussion, when effussion is less than 1 cm
C. In case of cardiac tamponade
D. In case of acute pericarditis

50.14 When prophylaxis of infective endocarditis is recommended?


A. Mitral valve prolapse and I* mitral regurgitation
B. Aortic valve prosthesis
C. Hypertrophic obstructive cardiomyopathy

50.15 Which symptom is attributed to the “major” or main criteria of rheumatic fever:
A. Arthralgia
B. Fever
C. Disproteinemia
D. Polyarthritis

50.16 In which case adenosine can be administered for treatment of cardiac rhythm disturbance?
A. In case of atrioventricular nodal paroxysmal tachycardia
B. In case of ventricular paroxysmal tachycardia
C. In case of paroxysmal atrial fibrillation.

50.17 Which sign is not typical of the isolated right ventricular failure:
A. Neck veins distension
B. Hepatic enlargement
C. Pulmonary rales
D. Low blood pressure

50.18 Which medications are not used for the treatment of sinus tachycardia?
A. Metoprolol succinate
B. Verapamil hydrochloride
C. Ivabradine
D. Amlodipine

50.19 Which signe isn’t typical of rheumatic fever:


A. Symptoms of valves damage
B. Splenomegaly
C. Swelling warm and tender joints
D. Fever

50.20 What treatment would you suggest for the patient with bradyarrhythmic permanent atrial fibrillation,
cardiomegaly and chronic congestive heart failure III-IV NYHA functional class?
A. Electric cardioversion
B. AAI type pacemaker
C. Intravenous amiodarone
D. VVI type pacemaker

lsmusis.lsmuni.lt/Klausimai/Spausdinti?Length=0?Kalba=EN&KategorijaId=152&Kalbos_input=EN&Kalbos=EN&KategorijaEn_input=Cardiology&Kate… 2/9
3/27/2019 LSMUSIS
50.21 A patient has be suffering from repeated syncope as aresult of paroxysmal ventriculat tachycardia. Which
treatment is teh best for the prevention of sudden death in case of the ischaemic heart disease and an old
myocardial infarction:
A. Continuous treatment with metaprolol
B. Continuous treatment with amiodaron
C. Treatment with sotalol
D. Implantation of cardioverter-defibrillator

50.22 Which patients belong to the high risk group for infective endocarditis?
A. Having atrial septal defect
B. Having mitral valve prolapse
C. Having artificial heart valves
D. Having secondary mitral valve regurgitation

50.23 Which medication is the most effective for treatment of the idiopathic acute pericarditis?
A. Tramadol
B. Aspirin 1 g/day
C. Ibuprofen 1200-1400 mg/day
D. Glucocorticoids

50.24 What symptoms do not reflect reperfusion in acute myocardial infarction?


A. ST segment return to isoelectric line
B. Ventricular arrhythmias
C. Decrease or disappearance of anginal pain
D. Worsening of left ventricular systolic function

50.25 Which medication is not indicated in case of non-ST-segment elevation acute coronary syndrome?
A. Enoxaparine
B. Clopidogrel
C. Metoprolol
D. Thrombolytics

II type tasks. For each numbered item,selct the one lettered heading that is most closely asssciated with it

50.26 Which treatment is most optimal in acute myocardial infarction?


1 - Primary percutaneous coronary angioplasty (PTCA), heparin, dual antiplatelet theraphy (DAPT), beta-
blockers and statins
2 - Intravenous thrombolysis
3 - Heparin, aspirin, ACE inhibitors and beta-blockers and statins
4 - Heparin, dual antiplatelet therapy (DAPT), beta-blockers and statins
A. Anterior uncomplicated acute myocardial infarction within 3 hours from the beginning of pain
B. Anterior myocardial infarction complicated by the cardiogenic shock
C. Inferior myocardial infarction, 14 hours from the beginning of pain
D. Inferior-posterior-lateral myocardial infarction, the second day from the beginning of pain

50.27 Which class of stable angina pectoris according to the Canadian Cardiovascular Society classification (CCS)
is determined in each case of the indicated situation:
1 - Angina during strenuous or rapid or prolonged exercise
2 - Angina walking in cold and windy weather or walking for more than 500m
3 - Angina on carrying any physical activity or at rest
4 - Angina in walking for less than 200m and even in a standstill position or climbing one flight of stairs
A. I
B. II
C. IV
D. III

50.28 Which pericardial disease are indicated clinical symptoms typical of:
1 - A small, “silent” heart, distensed neck veins and ascites
2 - Dilatation of heart on percussion, the left ventricular apical beat absent, distensed neck veins and
paradoxical pulse
3 - The chest pain increases while breathing, coughing and moving forwards and pericardial friction rub
sound

lsmusis.lsmuni.lt/Klausimai/Spausdinti?Length=0?Kalba=EN&KategorijaId=152&Kalbos_input=EN&Kalbos=EN&KategorijaEn_input=Cardiology&Kate… 3/9
3/27/2019 LSMUSIS
A. Acute fibrinous pericarditis
B. Exsudative pericarditis
C. Constrictive pericarditis

50.29 In which situation and what medications are recommended for the prevention of cardiac disease?
1 - Atrial fibrillation
2 - Secondary prevention of rheumatic fever
3 - Infective endocarditis
A. Amiodarone
B. Benzathin-Penicillin
C. Amoxicillin

50.30 Which echocardiographic changes are typical to each of the below mentioned disease?
1 - Left ventricular end-diastolic diameter is 4.5 cm, thickness of the walls 1.2 cm, ejection fraction > 50%
2 - Left ventricular end-diastolic diameter is 6.6 cm, thickness of the walls 0.9 cm, ejection fraction 25%
3 - Left ventricular end-diastolic diameter is 5.5 cm. An echo-free space around a heart, ejection fraction
50%
4 - Left ventricular end-diastolic diameter is 5.2 cm, akinesis in the anterior wall mid- and apical segments,
ejection fraction 45%
A. Dilative cardiomyopathy
B. Exudative pericarditis
C. Ischemic heart disease
D. Arterial hypertension

50.31 What are common two-dimensional echocardiographic findings in each indicated disease?
1 - The left atrium is enlarged, mitral valve leaflets are thickened and there is impaired opening of the
leaflets
2 - Asymmetric left ventricular hypertrophy, LV outflow tract obstruction
3 - Leaflets of mitral valve thickened, late in systole displaced into the left atrium, regurgitation jet into left
atrium
4 - Symmetric concentric left ventricular hypertrophy, aortic transvalvular pressure gradient
A. Hypertrophic obstructive cardiomyopathy
B. Mitral stenosis
C. Aortic stenosis
D. Mitral valve prolapse

50.32 Which ECG changes are typical of each of below listed heart rhythm disturbance?
1 - FF waves
2 - RR intervals are absolutely different, ff waves
3 - RR inervals are equal, P wave absent/invisible
A. Atrial fibrillation
B. Atrial flutter
C. Atrioventricular nodal paroxysmal tachycardia

50.33 For what etiology of infective endocarditis the below mentioned antibiotics are most effective:
1 - Caused by Str.viridans
2 - Caused by Staph.aureus
3 - Caused by Enterococcus
4 - Caused by Pseudomona
A. Imipenem
B. Ampicillin
C. Penicillin
D. Oxacyllin

50.34 Match diseases and their classifications:


1 - Acute myocardial infarction
2 - Unstable angina pectoris
3 - Stable angina pectoris
4 - Chronic heart failure
A. New York Heart Association (NYHA) functional classes
B. Grace risk groups
C. Killip’s classes
lsmusis.lsmuni.lt/Klausimai/Spausdinti?Length=0?Kalba=EN&KategorijaId=152&Kalbos_input=EN&Kalbos=EN&KategorijaEn_input=Cardiology&Kate… 4/9
3/27/2019 LSMUSIS
D. Canadian Cardiovascular Society (CCS) classes

50.35 Which hypertrophy are determined by the following ECG changes:


1 - Sv₁+Rv₆ > 35 mm
2 - R : S V₁ > 1,0
3 - P duration > 0,12”, it consists of two humps
4 - P amplitude in the II lead > 2,5 mm, duration = 0,12”
A. Left ventricle
B. Left atrium
C. Right ventricle
D. Right atrium

50.36 Match class of medications and their mechanism of action:


1 - Diuretics
2 - Beta-adrenoblockers
3 - Calcium channels blockers
4 - ACE inhibitors
A. Decreases the volume of circulating blood
B. Dilates the peripheral blood vessels
C. Inhibits renin - angiotensin - aldosterone system
D. Inhibits sympathetic nervous system activity and renin discharge

III type tasks. For each question there is one or more correct answers:
A – if correct answers are 1,2,3
B – if correct answers are 1 and 3
C – if correct answers are 2 and 4
D – if correct answer is 4
E – if correct are all answers above

50.37 When the statins should be prescribed for the patient?


1. Patient after myocardial infarction, LDL cholesterol 2.0 mmol/l
2. Patient 25 years old with diabetes mellitus without end-organ damage, LDL cholesterol 2.2mmol/l
3. Asymptomatic 60 years old smoker, systolic BP 160 mmHg, total cholesterol 8.2mmol/l, LDL cholesterol
4.9mmol/l
4. Asymptomatic 55 years old woman, BP 120/80 mmHg, total cholesterol 7.0 mmol/l, LDL cholesterol 2.9
mmol/l

50.38 A 50 year old man has suffered an uncomplicated acute inferior myocardial infarction. A 1,5 mm depression
of ST segment in leads V3-6 was determined by ECG during the tredmil test after 8 days. What test should be
chosen by the doctor:
1. Radionuclear myocardial perfusion imaging
2. Dipiridamol test
3. Small doses dobutamin’s test
4. Coronaroangiography

50.39 Angina pectoris (II CCS class) has been diagnosed to a 35 year old non-smoker woman. HR-68 bpm, BP -
120/80mmHg, LDL cholesterol 4.6mmol/l, a 2mm ST segment depressio in leads II, III and V5-6 during the
Physical stress test. Coronary angiography showed two vessel disease with stenoses <30%. Which medicine
shoul be prescribed?
1. Propranolol (Metoprolol)
2. Simvastatin (Atorvastatin)
3. Nifedipin (Amlodipin)
4. Diltiazem

50.40 What is the significance of ACE inhibitors:


1. They prolong life span for patients with chronic heart failure (HF)
2. They prevent LV remodeling after acute MI
3. They decrease the HF progress and diminish the number of rehospitalisations
4. They increase the quality of life and prolong life span for patients with HF

50.41 Which symptoms are characteristic of the critical aortic stenosis:

lsmusis.lsmuni.lt/Klausimai/Spausdinti?Length=0?Kalba=EN&KategorijaId=152&Kalbos_input=EN&Kalbos=EN&KategorijaEn_input=Cardiology&Kate… 5/9
3/27/2019 LSMUSIS
1. Aorta valve area - 0.6 cm²
2. The presistolic gallop rhythm
3. Mean pressure gradient between left ventricle and aorta > 40 mmHg
4. Protodiastolic gallop rhythm

50.42 Which statements concerning treatment of rheumatic fever are correct?


1. Inflammatory process is most effectively inhibited by diclofenac
2. For prolonged active rheumatic fever treatment with hydroxyehloroqninum is very effective
3. The latent rheumatic fever is not indication for prescribtion of prednisolone
4. Secondary prophylaxis is targeted for prevention of recurrent rheumatic fever attack

50.43 Which diagnostic criteria of infective endocarditis are considered as “major” according to Duke criteria?
1. Positive blood culture
2. Increasing intensity of murmur
3. The presence of vegetations on the valves on echocardiography
4. CRP elevation

50.44 Which statements concerning heart failure are correct?


1. The most common symptoms of HF: dyspnea, fatigue, edema
2. Cardiac magnetic resonance imaging is the first-choice technique to diagnose HF
3. Specific features of heart failure are elevated jugular venous pressure, hepatojugular reflux, gallop rhythm
4. Plasma natriuretic peptide concentration is normal in HF

50.45 What symptoms are characteristic for pericardial effusion?


1. Heart borders are enlarged on heart percussion
2. Low voltage of QRS on ECG
3. Elevated venous pressure
4. ST segment elevation in all ECG leads

50.46 Which medicines (drugs) shouldn’t be used for treatment of hypertrophic obstructive cardiomyopathy:
1. Beta-blockers
2. Cardiac glycosides
3. Ca channel blockers
4. Nitrates

50.47 Which medicines (drugs) usually prolong QT interval:


1. IA class antiarrhythmics (quinidine etc.)
2. Beta-blockers (atenolol, metoprolol, etc.)
3. Phenothiazines
4. IB class antiarrhythmics (lidocain, etc.)

50.48 Which are the most common complications of permanent atrial fibrillation:
1. Chronic heart failure
2. Angina pectoris
3. Embolism
4. Arterial hypertension

50.49 Which of the following are the side effect of ACE inhibitors?
1. Cough
2. Arterial hypotension
3. Hyperkalemia
4. Hypokalemia

50.50 20-year-old man has had a virus infection two weeks ago. Now his complaints are: shortness of breath on
physical exertion, weakness, palpitations, legs swelling. ECG – sinus tachycardia, flattened T in all leads,
ventricular premature beats (type trigeminy). On echocardiogrphy – left ventricular end-diastolic diameter 52 mm,
LV walls - 10 mm, hypokinetic all segments. LV ejection fraction 30%, CRP – 36mg/l. Which disease do you
suspect?
1. Dilated cardiomyopathy
2. Acute myocardial infarction
3. Infective endocarditis
4. Myocarditis

lsmusis.lsmuni.lt/Klausimai/Spausdinti?Length=0?Kalba=EN&KategorijaId=152&Kalbos_input=EN&Kalbos=EN&KategorijaEn_input=Cardiology&Kate… 6/9
3/27/2019 LSMUSIS
50.51 Which diagnostic criteria are characteristic to severe tricuspid regurgitation?
1. Pulsation of distensed neck veins
2. Epigastric pulsation
3. Systolic murmur at the lower left sternal border, increasing during inspiration
4. Systolic liver pulsation

50.52 What diseases should be suspected for the patient with exertional dyspnea?
1. Stable angina pectoris
2. Hypertorphic obstructive cardiomyopathy
3. Aotic valve stenosis
4. Acute pericarditis

50.53 Which analysis should be done when arterial hypertension is suspected:


1. Urine analysis
2. Blood potassium
3. Total cholesterol and triglyceride levels
4. Serum glucose

IV type tasks. Choose only one best answer

50.54 "For a 50 year old man with rheumatic aortic stenosis mechanical valve prothesis was implanted 2 months
ago. Now he complains of weakness and fever. Objective data: T-37.60C, pale skin,HR-90 bpm, BP-120/80
mmHg, at the aorta and Erb-Botkin’s zone 3/6 grade systolic murmur and 2/6 grade protodiastolic murmur, clear
prosthesis opening and closing sounds are heard. Laboratory: Hb-116 g/l, CRP 80mg/l, leucocytes – 10,2 x10⁹/l,
INR 2,8. What is the preliminary diagnosis of the disease?"
A. Thrombosis of the prosthesis
B. Relapse of rheumatic fever
C. Infective endocarditis

50.55 "For a 50 year old man with rheumatic aortic stenosis mechanical valve prothesis was implanted 2 months
ago. Now he complains of weakness and fever. Objective data: T-37.60C, pale skin,HR-90 bpm, BP-120/80
mmHg, at the aorta and Erb-Botkin’s zone 3/6 grade systolic murmur and 2/6 grade protodiastolic murmur, clear
prosthesis opening and closing sounds are heard. Laboratory: Hb-116 g/l, CRP 80mg/l, leucocytes – 10,2 x10⁹/l,
INR 2,8. Wihich diagnostic test may be most informative for this patient? "
A. ECG
B. Chest computed tomography (CT)
C. Two-dimensional tranthoracic echocardiography
D. Transesophageal echocardiography

50.56 72 year old women several days ago started to complaint on dyspnea, dizziness, have had several episodes
of loss of consciousness during this period. Objective data: HR – 36 bpm, BP – 160/80 mmHg, on ECG – number
of P waves significantly exceeds number of QRS complexes. There is no relationship between P and QRS, QRS
complexes are narrow. When measuring, frequency between P waves corresponds to 80 bpm, and measured
frequency of QRS complexes is 36 bpm. What cardiac rhythm disturbance is most probable?
A. First degree AV block
B. Atrial fibrillation
C. Third degree (complete) AV block
D. Ventricular tachycardia

50.57 "84 year old man, who was diagnosed with permanent atrial fibrillation 16 years ago, during last 3 months
complains on fatigue, dyspnea during moderate physical activity. When measuring BP he has noticed that the
heart rate usually is below 50 bpm. On 24 hour (Holter) ECG monitoring: minimum heart rate (HR) – 28 bpm,
average HR – 46 bpm, maximum HR – 58 bpm, longest R-R interval – 4.8 sec. On echocardiography: both atria
significantly dilated, mild hypertrophy of the left ventricle, no signs of significant valvular heart disease. What
method of treatment should be recommended for this patient? "
A. Treatment with amiodarone
B. Treatment with dihydropyridine calcium channel blockers
C. Implantation of cardioverter-defibrillator
D. Implantation of VVIR type cardiac pacemaker

50.58 During the invasive hemodynamics examination in a young woman the following data were obtained:
systolic left ventricular and aortic pressure is 110 mmHg, right ventricular pressure – 35 mmHg, left atrial – 9

lsmusis.lsmuni.lt/Klausimai/Spausdinti?Length=0?Kalba=EN&KategorijaId=152&Kalbos_input=EN&Kalbos=EN&KategorijaEn_input=Cardiology&Kate… 7/9
3/27/2019 LSMUSIS
mmHg, right atrial – 8 mmHg, pulmonary artery - 35 mmHg. Diastolic blood pressure in the aorta – 75 mmHg.
Venous blood oxygen saturation from pulmonary artery was 85 percent. What is the most probable diagnosis?
A. The heart is normal
B. Heart failure
C. Atrial or ventricular septal defect
D. Pulmonary disease suspected

50.59 A 17 years old woman suffers from episodes of syncope 3 - 4 times per year, lasting 10 - 20 sec, provoked
by physical activity or emotional stress. On ECG - non-sustained episodes of wide QRS tachycardia (Torsades de
pointes) with high heart rate have been registered. Physical examination normal, BP – 120/70 mmHg. ECG on
admission showed a sinus rhythm 70 bpm, PR – 0,18 s, QTc – 0,52 s. What disease should be suspected?
A. Myocarditis
B. Reflex syncope
C. Congenital long QT interval syndrome
D. WPW syndrome

50.60 A 17 years old woman suffers from episodes of syncope 3 - 4 times per year, lasting 10 - 20 sec, provoked
by physical activity or emotional stress. On ECG - non-sustained episodes of wide QRS tachycardia (Torsades de
pointes) with high heart rate have been registered. Physical examination normal, ABP – 120/70 mmHg. ECG on
admission a sinus rhythm 70 bpm, PR – 0,18 s, QTc – 0,52 s. Which method of treatment is not appropriate for the
patient?
A. Amiodaron
B. Beta-blockers
C. Avoidance of competitive sports
D. Implantation of cardioverter-defibrillator

50.61 "An 18 year old student (female) complains of permanent stitching pain in the precordial area. She denies
rheumatic history. Her height – 176cm, weight – 56 kg, scoliosis, pectus excavatus. On auscultation there is a
midsystolic click at Ist point, II0 telesystolic murmur at the apex with more intensity in the upright posture. Which
disease will you suspect? "
A. Congenital ventricular septum defect
B. Aortic valve stenosis
C. Mitral valve prolapse
D. Functional murmur and 3rd heart sound

50.62 "An 18 year old student (female) complains of permanent stitching pain in the precordial area. She denies
rheumatic history. Her height – 176cm, weight – 56 kg, scoliosis, pectus excavatus. On auscultation there is a
midsystolic click and 2/6 telesystolic murmur at the apex, both occurring earlier in upright posture. Heart rate – 92
bpm, BP – 120/76 mmHg. Which medicine (drug) group would you choose for treatment?"
A. Calcium channel blockers
B. .Sedatives
C. Food supplements
D. Beta-blockers

50.63 "A 52 year old obese man, smoker and having history of arterial hypertension, complains of retrosternal pain
which is provoked by physical exertion. The pain occurres with walking distance of less than 100 meters or
climbing stairs to the first floor. Nitroglycerine taken sublingually usually relieves or decreases the pain. His
symptoms continue for more than one year. ECG at rest is normal. What syndrome or disease can you suspect? "
A. Unstable angina
B. Stable functional class 2 angina
C. Gastric ulcer
D. Stable functional class 3 angina

50.64 "A 52 year old obese man, smoker and having history of arterial hypertension, complains of retrosternal pain
which is provoked by physical exertion. The pain occurres with walking distance of less than 100 meters or
climbing stairs to the first floor. Nitroglycerine taken sublingually usually relieves or decreases the pain. His
symptoms continue for more than one year. ECG at rest is normal. What is the most appropriate management of
such condition "
A. Risk factors correction and glyceryltrinitrate when needed
B. Risk factors correction, myocardial ischemia verification, coronary angiography and appropriate
revascularisation with optimal medical treatment
C. Risk factors correction and optimal medical treatment
lsmusis.lsmuni.lt/Klausimai/Spausdinti?Length=0?Kalba=EN&KategorijaId=152&Kalbos_input=EN&Kalbos=EN&KategorijaEn_input=Cardiology&Kate… 8/9
3/27/2019 LSMUSIS
D. Coronary angiography and appropriate revascularisation without any medical treatment

50.1 - B 50.2 - C 50.3 - B 50.4 - C 50.5 - D 50.6 - D 50.7 - C 50.8 - D


50.9 - D 50.10 - B 50.11 - D 50.12 - B 50.13 - C 50.14 - B 50.15 - D 50.16 - A
50.17 - C 50.18 - D 50.19 - B 50.20 - D 50.21 - D 50.22 - C 50.23 - C 50.24 - D
50.25 - D
50.26 50.27 50.28 50.29 50.30 50.31 50.32 50.33
1-B 1-A 1-C 1-A 1-B 1-B 1-B 1-C
2-A 2-B 2-B 2-B 2-C 2-A 2-A 2-D
3-D 3-C 3-A 3-C 3-D 3-D 3-C 3-B
4-C 4-D 4-A 4-C 4-A
50.34 50.35 50.36
1-C 1-A 1-A
2-B 2-C 2-D
3-D 3-B 3-B
4-A 4-D 4-C
50.37 50.38 50.39 50.40 50.41 50.42 50.43 50.44
1 4 2 1 1 1 1 1
3 4 2 2 2 3 3
3 3 3
4 4
50.45 50.46 50.47 50.48 50.49 50.50 50.51 50.52
1 2 1 1 1 4 1 1
2 4 3 3 2 2 2
3 3 3 3
4
50.53
1
2
3
4
50.54 - C 50.55 - D 50.56 - C 50.57 - D 50.58 - C 50.59 - C 50.60 - A 50.61 - C
50.62 - D 50.63 - D 50.64 - B

lsmusis.lsmuni.lt/Klausimai/Spausdinti?Length=0?Kalba=EN&KategorijaId=152&Kalbos_input=EN&Kalbos=EN&KategorijaEn_input=Cardiology&Kate… 9/9
3/27/2019 LSMUSIS

Pulmonology
I type tasks. Choose only one best answer

31.1 Acute pneumonia is:


A. An acute inflammation of whole lung tissue
B. An acute imflammation of interstitial lung tissue
C. An acute imflammation of alveolar membranes
D. An acute imflammation of bronchial and alveolar mucosa

31.2 The most common ethiologic agent of community acquired pneumonia:


A. Streptococcus
B. Pneumococcus
C. Staphylococcus
D. Pseudomonas aeruginosa
E. Mycoplasma

31.3 Which pathogens are the most common etiologic factor in Hospital-acquired pneumonia:
A. Staphylococci, Pseudomonas aeruginosa, Klebsiellae
B. Pneumococci, Streptococci, Gram-negative bacilli
C. Fungi, Viruses
D. Rickettsiae, Mycoplasmae

31.4 What is the classification of clinical course of pneumonia:


A. Staphylococcic and Streptococcic
B. Lobar and lobular
C. Primary and Secondary
D. Viral and Bacterial
E. Massive and Athypical

31.5 Which complications of pneumonia are the most common:


A. Shock, Myocarditis
B. Lung Abscess, Exudative Pleural Effusion
C. Collapse, Pleural Empyema
D. Bronchial obstruction, Hepatitis

31.6 Which of the following investigations should be taken to prove the diagnosis of pneumonia:
A. Sputum culture
B. Blood laboratory findings
C. Chest X-ray examination
D. We don’t need any investigations (anamnestical data and objective findings are enough for diagnosis)
E. Chest CT

31.7 What Phenomenons are common for mycoplasmal Pneumonia:


A. High Leucocytosis, Tendency to form Abscess
B. Ineffective antibiotic Treatment
C. Rusty or purulent Sputum, Fever with Chill
D. Patients are young People from closed Groups, there is no Correlation between Auscultation and X-ray
Findings

31.8 Which Antibiotic is effective for mycoplasmal Pneumonia Treatment:


A. Penicillin
B. Ceftazidime
C. Augmentin
D. Tetracyclin

31.9 Which Antibiotic is Drug of Choice to treat streptococcal Pneumonia:


A. Levomycetin
B. Tetracyclin
C. Penicillin
D. Oxacyllin

31.10 Which antibiotic is preferred for staphylococcal pneumonia treatment:


lsmusis.lsmuni.lt/Klausimai/Spausdinti?Length=0?Kalba=EN&KategorijaId=133&Kalbos_input=EN&Kalbos=EN&KategorijaEn_input=Pulmonology&K… 1/13
3/27/2019 LSMUSIS
A. Oxacyllin
B. Tetracyclin
C. Penicillin
D. Levomycetin

31.11 Which acute pneumonia treatment strategy is optimal:


A. Start immediately Penicillin and Gentamycin
B. Do not start the Treatment before the Results of Sputum Culture are obtained
C. Treat with Cephalosporins
D. Immediately investigate sputum microscopically by Gram stain method and prescribe antibiotics
according to the results of investigation

31.12 What is the classification of asthma according to clinical course:


A. Occupational and aspirinic
B. Infectional and allergic
C. Atopical and allergic
D. Extrinsic and intrinsic

31.13 What is the most important part in pathogenesis of asthma:


A. Mast Cells Degranulation and liberation of biologically active substances
B. Arachidonic Acid and Thromboxan Impact to the bronchial Mucosa
C. Impact of oxidants and polutants to mast cells
D. Decrease of cAMP (cyclic adenosin monophosphate) concentration

31.14 What is common for the Status asthmaticus:


A. Dyspnea attack lasts more than 24 hours
B. Dyspnea attack lasts more than 12 hours
C. 3 - 4 dyspnea episodes per day
D. Dyspnea Atack does not respond to Drugs, which were previously effective (especially β2 Agonists) and
Symptoms include dry nonoproductive Cough as well

31.15 Which of these medicines belongs to inhaled steroids:


A. Fenoterol
B. Ipratropium Bromide
C. Salbutamol
D. Budesonide

31.16 What medicines can be used for severe asthma treatment:


A. Sympathomymetic and cholinolytic agents only
B. Usual asthma treatment and oral Steroids
C. Methylxanthines and beta-2 agonists only
D. Disodium cromoglycate, second-generation H1-antihistamine and beta-2 agonists

31.17 What is the Treatment of Status Asthmaticus:


A. β2 agonists, methylxanthines and oxygen
B. Rehydration, methylxanthines and β2 agonists
C. Rehydration, oxygen, methylxanthines and inhaled steroids
D. Rehydration, steroids intravenously, methylxanthines and oxygen

31.18 Which of the following medicines belongs to β2 agonists:


A. Salbutamol
B. Beclomethasone
C. Flunisolide
D. Ketotifen
E. Montelukast

31.19 Which administration way of steroids for asthmatic patients has the minor systemic effect?
A. Oral
B. Intramuscular
C. Intravenously
D. Inhalation

31.20 What is the most important in Treatment of persistent Asthma in Out-Patient Department:
lsmusis.lsmuni.lt/Klausimai/Spausdinti?Length=0?Kalba=EN&KategorijaId=133&Kalbos_input=EN&Kalbos=EN&KategorijaEn_input=Pulmonology&K… 2/13
3/27/2019 LSMUSIS
A. Patient must visit Out-Patient Department at least once a week
B. Patient must be send to the sanatorium for rehabilitation at least once a year
C. Recommendation to use continuously β2 Agonists and inhaled Steroids
D. Patients must receive consistent inhaler technique training
E. Chest X-ray should be performed at least once a year

31.21 The cause of pulmonary emphysema could be:


A. α1 Antitrypsin Deficiency
B. Increase of cAMP Concentration
C. Progressive Increase of Collagen Fibers
D. Lack of Surfactant in the Lungs

31.22 What are two clinical forms of chronic bronchitis:


A. Catarrhal and purulent
B. Obstructive and catarrhal
C. Purulent and obstructive
D. Obstructive and nonobstructive

31.23 What X-ray findings are common in chronic bronchitis:


A. Peribronchial infiltration
B. Signs of pulmonary emphysema
C. Dilated pulmonary artery
D. Nonspecific findings - increased lung markings throughout both lungs due to peribronchial and
perivascular fibrosis

31.24 The main feature of idiopathic pulmonary fibrosis (Hamman - Rich syndrome) is:
A. Dry Cough
B. Febrile Fever
C. Nocturnal Dyspnea Attacks
D. Progressing Dyspnea
E. Weakness

31.25 What is the treatment of idiopathic pulmonary fibrosis (Hamman - Rich disease):
A. Nonsteroidal antiinflamatory drugs (NSAID) and corticosteroids
B. Imunosuppressants and antibiotics
C. Antibiotics and corticosteroids
D. Corticosteroids and imunosuppressants

31.26 Which morphological changes are common to sarcoidosis:


A. Non-caseating Granulomas
B. Granuloma with caseous necrosis
C. Granuloma with infiltration of eosinophils and basophils
D. Granuloma with infiltration of erythrocytes

31.27 Which drugs are used for sarcoidosis treatment:


A. Immunosuppressants
B. Non-steroidal anti-inflammatory Drugs
C. Antibiotics
D. Corticosteroids

31.28 What morphological changes are commmon for pulmonary Wegener’s granulomatosis:
A. Granulomatous inflammation with palisading epithelioid cells, Langhans' giant cells and necrotizing
vasculitis of small to medium-size vessels
B. Non-caseating Granulomas
C. Granuloma with caseous necrosis
D. Granulomatous inflammation with palisading epithelioid cells, Langhans' giant cells

31.29 Which organs are usually involved in Goodpasture syndrome:


A. Lung and liver
B. Lung and heart
C. Lung and sex organs
D. Lung and kidneys

lsmusis.lsmuni.lt/Klausimai/Spausdinti?Length=0?Kalba=EN&KategorijaId=133&Kalbos_input=EN&Kalbos=EN&KategorijaEn_input=Pulmonology&K… 3/13
3/27/2019 LSMUSIS
31.30 What is the treatment of Goodpasture syndrome:
A. Antibiotics, immunosuppressants, plasmapheresis
B. Immunosuppressants, nonsteroidal anti-inflamatory drugs, haemosorption
C. Corticosteroids, antibiotics, plasmapheresis
D. Corticosteroids, immunosuppressants, plasmapheresis

31.31 Which combination of symptoms is common for bronchiectasis:


A. Purulent Sputum, Loss of Weight, Rales, Nail clubbing
B. Purulent Blood-streaked Sputum, febrile Fever and amphoric Breath Sounds
C. Mucoid sputum, dyspnea and wheezes
D. Nail clubbing, purulent sputum and wheezes

31.32 Which investigation proves the diagnosis of bronchiectasis:


A. Chest CT
B. Posteroanterior View Chest X-ray
C. Bronchoscopy
D. Ventilation/perfusion Lung Scan

31.33 Which complication is common for bronchiectasis:


A. Infective (septic) endocarditis
B. Iron-deficiency anemia
C. Amyloidosis
D. Cushing's syndrome

31.34 Which combination of symptoms is common for lung abscess:


A. Purulent Sputum, febrile Fever, extreme Leukocytosis
B. Blood-streaked Sputum, subfebrile Fever, Nail clubbing
C. Abundant mucoid Sputum, febrile Fever, Leukocytopenia
D. Rusty sputum, subfebrile Fever, Leukocytosis

31.35 Which source of pulmonary embolism is the most common:


A. Left atrial Thrombus
B. Superior vena cava Thrombosis
C. Inferior vena cava Thrombosis
D. Right atrial Thrombus

31.36 Which combination of symptoms is common for massive pulmonary thromboembolism:


A. Shock, Cyanosis, Dyspnea, Tachycardia
B. Pulmonary oedema, Dyspnea, Cyanosis
C. Acute Psychosis, Bradycardia, Collapse
D. Massive Bleeding from the Lungs, Shock, Tachycardia, Cyanosis

31.37 Which combination of symptoms is common for fat embolism:


A. Dyspnea, tachycardia, cyanosis, nonpalpable petechial skin rash in the chest, axilla, conjunctiva and
neck
B. Pulmonary oedema, Dyspnea, Cyanosis
C. Massive Bleeding from the Lungs, Shock, Tachycardia
D. Cyanosis, Dyspnea, Shock, Anuria

31.38 Which dangerous syndromes developes in amniotic fluid embolism:


A. Respiratory Distress Syndrome and Goodpasture Syndrome
B. Disseminated intravascular Coagulation and Goodpasture Syndrome
C. Goodpasture Syndrome and Kaplan Syndrome
D. Respiratory Distress Syndrome and Disseminated intravascular Coagulation

31.39 Which treatment of massive pulmonary thromboembolism is optional:


A. Thrombolysis, if Thrombolytics are not effective - Embolectomy
B. Heparin, if Heparin is not effective - Embolectomy
C. Embolectomy
D. Warfarin, if Treatment with Wis not effective - Embolectomy

31.40 Venous thromboembolic disease clinically usually manifests with:


A. Deep Vein Thrombosis
lsmusis.lsmuni.lt/Klausimai/Spausdinti?Length=0?Kalba=EN&KategorijaId=133&Kalbos_input=EN&Kalbos=EN&KategorijaEn_input=Pulmonology&K… 4/13
3/27/2019 LSMUSIS
B. Foot Gangrene
C. Ischemic stroke
D. Central Retinal Artery Thrombosis

31.41 Nail clubbing is not compatible with:


A. Infective (septic) endocarditis
B. Liver Cirrhosis
C. Primary Lung Emphysema
D. Chronic obstructive Bronchitis

31.42 In case of massive bleeding from the lungs patients usually die from:
A. Blood Aspiration
B. Hemorrhagic Shock
C. Posthemorrhagic anemia
D. Acute heart failure

II type tasks. For each numbered item,selct the one lettered heading that is most closely asssciated with it

31.43 What is the treatment of these purulent lung diseases:


1 - 12.76. Local bronchiectasis involving two segments
2 - 12.77. Peripheral lung abscess
3 - 12.78. Central lung abscess
4 - 12.79. Purulent pneumonia caused by Pseudomonas aeruginosa
A. Percutaneus transthoracic Abscess Drainage
B. Surgical removal of Segments
C. Therapeutic Bronchoscopies
D. Antibiotical Treatment with ceftazidime

31.44 Which microbial agent could cause these pneumonias:


1 - 12.64. Pneumonia with Abscesses
2 - 12.65. Gangrenous pneumonia
3 - 12.66. Lobar Pneumonia
4 - 12.77. Interstitial Pneumonia
A. Anaerobic flora
B. Staphylococci
C. Streptococcus pneumoniae
D. Mycoplasma pneumoniae

31.45 Which antibiotic should be given to treat pneumonia caused by:


1 - 12.72. Streptococcus pneumoniae
2 - 12.73. Staphylococcus aureus
3 - 12.74. Pseudomonas aeruginosa caused pneumonia with abscesses
4 - 12.75. Mycoplasma pneumoniae
A. Erythromycin
B. Ceftazidime
C. Oxacillin
D. Penicillin

31.95 Which treatment is the best to cure these different forms of pulmonary embolism:
1 - Massive emblism
2 - Microthromboembolism
3 - Fat embolism
4 - Recurrent pulmonary thromboembolism despite the adequate treatment
A. Thrombolytic therapy or embolectomy
B. Placement of vena cava Filters
C. Lipostabil
D. Heparin

III type tasks. For each question there is one or more correct answers:
A – if correct answers are 1,2,3
B – if correct answers are 1 and 3
C – if correct answers are 2 and 4
lsmusis.lsmuni.lt/Klausimai/Spausdinti?Length=0?Kalba=EN&KategorijaId=133&Kalbos_input=EN&Kalbos=EN&KategorijaEn_input=Pulmonology&K… 5/13
3/27/2019 LSMUSIS
D – if correct answer is 4
E – if correct are all answers above

31.46 Which statements about bronchial asthma are true:


1. Chronic recurrent disease
2. Bronchial hyperresponsiveness is a hallmark of asthma
3. Clinical presentation - narrowing of the airway and the classic symptoms of wheezin
4. Treatment with inhaled steroids is not effective

31.47 What statements about sarcoidosis are correct:


1. Multisystemic granulomatous disease of unknown ethiology and pathogenesis
2. Mantoux test is always positive
3. Positive Kveim - Siltzbach test
4. Hypocalcemia is common

31.48 What statements about status asthmaticus are correct:


1. Severe persisting, life-threatening paroxysm of dyspnea
2. Dyspnea paroxysm disappears after inhalation of β2 agonists
3. Dyspnea paroxysm does not respond to standard treatments of bronchodilators (inhalers)
4. Patient recovers after atropine intravenous injection

31.49 Complications of acute pneumonia are:


1. Exsudative pleuritis
2. Atelectasis
3. Bacterial (septic) shock
4. Pulmonary thromboembolism

31.50 What treatment should receive the patient who presents with haemoptoe:
1. Calmness, cold on chest wall
2. Blood transfusion
3. Drugs improving coagulation
4. Surgical treatment

31.51 Complications of chronic cor pulmonale are:


1. Secondary erythrocytosis
2. Anemia
3. Gastric bleeding
4. Hypercalcemia

31.52 Clinical presentation of central lung cancer could be:


1. Haemopthysis
2. Reccurrent lung infections
3. Atelectasis
4. Horner’s syndrome
5. Pleuritic chest pain

31.53 Right Middle Lobe Syndrome presents with:


1. Right middle lobe inflamation
2. Bronchial breath sounds in the area of right middle lobe
3. Right middle lobe hypoventilation
4. Right side pleural effusion

IV type tasks. Choose only one best answer

31.54 28 years old student complains of fever, chills, pain in the right side of the chest which increases when
coughing. Onset of the disease was acute after catching a cold. Objective status: T - 38*C. Breathing rate -
28/min. Percussion and auscultation of the lower right part of the chest wall reveal dullness and bronchial breath
sounds. What preliminary diagnosis would you suggest:
A. Excudative pleuritis
B. Alveolitis
C. Lobular pneumonia
D. Lobar pneumonia

lsmusis.lsmuni.lt/Klausimai/Spausdinti?Length=0?Kalba=EN&KategorijaId=133&Kalbos_input=EN&Kalbos=EN&KategorijaEn_input=Pulmonology&K… 6/13
3/27/2019 LSMUSIS
31.55 28 years old student complains of fever, chills, pain in the right side of the chest which increases when
coughing. Onset of the disease was acute after catching a cold. Objective status: T - 38*C. Breathing rate -
28/min. Percussion and auscultation of the lower right part of the chest wall reveal dullness and bronchial breath
sounds. Which etiologic agent could be the cause of the disease:
A. Streptococcus pneumoniae
B. Pseudomonas aeruginosa
C. Anaerobic flora
D. Fungi

31.56 28 years old student complains of fever, chills, pain in the right side of the chest which increases when
coughing. Onset of the disease was acute after catching a cold. Objective status: T - 38*C. Breathing rate -
28/min. Percussion and auscultation of the lower right part of the chest wall reveal dullness and bronchial breath
sounds. Which test would confirm the diagnosis:
A. ECG
B. Urine test
C. Spirometry
D. Chest X-ray

31.57 28 years old student complains of fever, chills, pain in the right side of the chest which increases when
coughing. Onset of the disease was acute after catching a cold. Objective status: T - 38*C. Breathing rate -
28/min. Percussion and auscultation of the lower right part of the chest wall reveal dullness and bronchial breath
sounds. Which antibiotic should be administered:
A. Oxacillin
B. Cefotaxime
C. Penicillin
D. Erythromycin

31.58 48 years old male has had a daily consumption of a pint of whiskey for at least 10 years. He complains of
fever, productive cough with large amount of rusty watery sputum. Objective status: T - 39*C. Breathing rate -
28/min. Auscultation findings: rales are presented in the lower part of the right lung. What prelimenary diagnosis
would you suggest:
A. Acute bronchitis
B. Bronchiectases
C. Pulmonary embolism
D. Pneumonia

31.59 48 years old male has had a daily consumption of a pint of whiskey for at least 10 years. He complains of
fever, productive cough with large amount of rusty watery sputum. Objective status: T - 39*C. Breathing rate -
28/min. Auscultation findings: rales are presented in the lower part of the right lung. Which infectious agent could
cause this disease:
A. Streptococcus pneumoniae
B. Mycoplasma pneumoniae
C. Klebsiella pneumoniae
D. Anaerobic flora

31.60 48 years old male has had a daily consumption of a pint of whiskey for at least 10 years. He complains of
fever, productive cough with large amount of rusty watery sputum. Objective status: T - 39*C. Breathing rate -
28/min. Auscultation findings: rales are presented in the lower part of the right lung. Which antibiotics is the best
option for the treatment in this case:
A. Penicillin
B. Oxacillin
C. Ceftazidime
D. Erythromycine

31.61 65 years old male suffers from rheumatoid arthritis. Intramuscular injections of diclophenac and novocaine
electrophoreses were prescribed in the department of rheumatology. The patient started complaining of fever,
cough and chest pain after fifth injection of diclophenac. Objective status. T - 39*C. Rales in the base of the left
lung. Chest x-ray showed poorly defined homogenous nodular infiltrates of various sizes in the left lung. Complete
blood cell count analysis: leukocyte count 10.0 x 10(9)/l, neutrophils: band - 4%, segmented - 26%; eosinophils -
40%, lymphocytes - 20%, monocytes - 10%. What is preliminary diagnosis:
A. Infarct-pneumonia
B. Lobular pneumonia
lsmusis.lsmuni.lt/Klausimai/Spausdinti?Length=0?Kalba=EN&KategorijaId=133&Kalbos_input=EN&Kalbos=EN&KategorijaEn_input=Pulmonology&K… 7/13
3/27/2019 LSMUSIS
C. Lobar pneumonia
D. Acute eosinophylic, pulmonary infiltrate

31.62 65 years old male suffers from rheumatoid arthritis. Intramuscular injections of diclophenac and novocaine
electrophoreses were prescribed in the department of rheumatology. The patient started complaining of fever,
cough and chest pain after fifth injection of diclophenac. Objective status. T - 39*C. Rales in the base of the left
lung. Chest x-ray showed poorly defined homogenous nodular infiltrates of various sizes in the left lung. Complete
blood cell count analysis: leukocyte count 10.0 x 10(9)/l, neutrophils: band - 4%, segmented - 26%; eosinophils -
40%, lymphocytes - 20%, monocytes - 10%. What treatment would you prescribe for the patient:
A. Antibiotics
B. Sulfonamides
C. Antihistamines
D. Glucocorticoids

31.63 25 years old patient was ill with not complicated lobular pneumonia. On the 10th day of treatment with
penicillin allergic itchy maculopapular rash appeared. The best further treatment tactics is:
A. Change penicillin to another antibiotic
B. Continue penicillin and start antiallergic drugs
C. Continue penicillin and start glucocorticoids
D. Stop antibiotics and start antiallergic drugs

31.64 50 years old female complains of dyspnea atack. Dyspnea attacks often occur in the morning or after
inhalation of dust or vapours. Objective status: Breathing rate - 24/min., wheezing on lung auscultation. Which
disease would you suspect:
A. Preasthma
B. Obstructive bronchitis
C. Cardiac asthma
D. Asthma

31.65 50 years old female complains of dyspnea atack. Dyspnea attacks often occur in the morning or after
inhalation of dust or vapours. Objective status: Breathing rate - 24/min., wheezing on lung auscultation. Which
drugs would you prescribe:
A. Methylxanthines and β-adrenergic blockers
B. Inhaled steroids and atropine
C. β-2 adrenergic agonists and nitrates
D. β-2 adrenergic agonists, inhaled steroids and cromolyn sodium

31.66 50 years old male is ill with asthma. He complains of persisting severe paroxysm of dyspnea, which doesn’t
respond to treatment with beta-2 adrenergic agonists. Objective status: Breathing rate - 30/min. Heart rate - 100
beats/min. Blood pressure - 120/80 mmHg. There is prolonged expiration and wheezing. What complication would
you suspect:
A. Pulmonary edema
B. Severe asthma attack
C. Pulmonary thromboembolism
D. Status asthmaticus

31.67 50 years old male is ill with asthma. He complains of persisting severe paroxysm of dyspnea, which doesn’t
respond to treatment with beta-2 adrenergic agonists. Objective status: Breathing rate - 30/min. Heart rate - 100
beats/min. Blood pressure - 120/80 mmHg. There is prolonged expiration and wheezing. Which department would
you choose for further patient's follow-up:
A. Pulmonology
B. Internal diseases
C. Intensive care unit
D. This patient could be terated at home

31.68 50 years old male is ill with asthma. He complains of persisting severe paroxysm of dyspnea, which doesn’t
respond to treatment with beta-2 adrenergic agonists. Objective status: Breathing rate - 30/min. Heart rate - 100
beats/min. Blood pressure - 120/80 mmHg. There is prolonged expiration and wheezing.How would you treat this
patient:
A. Oxygen, rehydration, glucocorticoids and methylxanthines
B. Oxygen, β2 agonists and methylxanthines
C. Glucocorticoids

lsmusis.lsmuni.lt/Klausimai/Spausdinti?Length=0?Kalba=EN&KategorijaId=133&Kalbos_input=EN&Kalbos=EN&KategorijaEn_input=Pulmonology&K… 8/13
3/27/2019 LSMUSIS
D. Rehydration, oxygen, β-2 agonists and methylxanthines

31.69 56 years old female 8 days after cholecystectomy got up and went to therestroom. Suddenly patient’s face
became cyanotic with prominent pulse of jugular veine and woman fell down unconscious. Objective status:
Tachypnoea - 40/min. Tachycardia - 146 beats/min. Blood pressure - 50/30 mmHg. What is your preliminary
diagnosis:
A. Ischaemic stroke
B. Collapse
C. Massive pulmonary thromboembolism
D. Myocardial infarction

31.70 56 years old female 8 days after cholecystectomy got up and went to therestroom. Suddenly patient’s face
became cyanotic with prominent pulse of jugular veine and woman fell down unconscious. Objective status:
Tachypnoea - 40/min. Tachycardia - 146 beats/min. Blood pressure - 50/30 mmHg. The best tactics in this situation
would be:
A. ECG registration
B. arterial blood gas analysis
C. Ventilation/perfusion lung scan
D. Administration of vasopressor drugs, heparin 5000 UA intravenously and admition the patient to intensive
care unit

31.71 56 years old female 8 days after cholecystectomy got up and went to therestroom. Suddenly patient’s face
became cyanotic with prominent pulse of jugular veine and woman fell down unconscious. Objective status:
Tachypnoea - 40/min. Tachycardia - 146 beats/min. Blood pressure - 50/30 mmHg. What treatment would you
prescribe:
A. Heparin and methylxanthins
B. Heparin
C. Fibrinolytic drugs and then after - treatment with heparine
D. Warfarin

31.72 70 years old female complains of dyspnea, pain in the right calf, expectoration of blood. She is suffering
from ischemic heart disease about 10 years. She is using nitrates, digoxin. During last two months dyspnea is
progressing. Objective status: cyanosis of the face, prominent pulse of jugular veins. Breathing rate - 28/min.
Pulse rate - 146/min. Blood pressure - 80/40 mmHg. Auscultation findings: rales are presented bilateraly in the
lower parts of lungs. Right calf is swollen, possitive Homand’s sign. What is preliminary diagnosis:
A. Atrial fibrillation
B. Myocardial infarction
C. Ischaemic stroke
D. Pulmonary thromboembolism

31.73 70 years old female complains of dyspnea, pain in the right calf, expectoration of blood. She is suffering
from ischemic heart disease about 10 years. She is using nitrates, digoxin. During last two months dyspnea is
progressing. Objective status: cyanosis of the face, prominent pulse of jugular veins. Breathing rate - 28/min.
Pulse rate - 146/min. Blood pressure - 80/40 mmHg. Auscultation findings: rales are presented bilateraly in the
lower parts of lungs. Right calf is swollen, possitive Homand’s sign. What ECG changes you should find:
A. S1, Q3, T3, P pulmonale in leads II, III, aVF
B. Deep Q wave in leads III, aVL, aVF
C. High R wave in leads V5, V6, P pulmonale in leads V4, V5
D. Left bundle branch block, high R wave in leads V5, V6

31.74 70 years old female complains of dyspnea, pain in the right calf, expectoration of blood. She is suffering
from ischemic heart disease about 10 years. She is using nitrates, digoxin. During last two months dyspnea is
progressing. Objective status: cyanosis of the face, prominent pulse of jugular veins. Breathing rate - 28/min.
Pulse rate - 146/min. Blood pressure - 80/40 mmHg. Auscultation findings: rales are presented bilateraly in the
lower parts of lungs. Right calf is swollen, possitive Homand’s sign. What X-ray changes you will find:
A. Dilatation of heart left atrium and ventricle
B. Extended shadow of pulmonary artery
C. Triangle form infiltrate
D. Chest X-ray will be normal

31.75 70 years old female complains of dyspnea, pain in the right calf, expectoration of blood. She is suffering
from ischemic heart disease about 10 years. She is using nitrates, digoxin. During last two months dyspnea is

lsmusis.lsmuni.lt/Klausimai/Spausdinti?Length=0?Kalba=EN&KategorijaId=133&Kalbos_input=EN&Kalbos=EN&KategorijaEn_input=Pulmonology&K… 9/13
3/27/2019 LSMUSIS
progressing. Objective status: cyanosis of the face, prominent pulse of jugular veins. Breathing rate - 28/min.
Pulse rate - 146/min. Blood pressure - 80/40 mmHg. Auscultation findings: rales are presented bilateraly in the
lower parts of lungs. Right calf is swollen, possitive Homand’s sign. What treatment would you prescribe:
A. Analgetics
B. Large dosage of nitrates
C. Anticoagulants (Warfarin) and vasopressor drugs
D. Vasopressor drugs and fibrinolytics

31.76 62 years male complains of fever 38*C, dyspnea, chest pain, which increases during deep breathing,
sputum with traces of blood. Dyspnea and tachycardia started suddenly 4 days ago, later fever (38*C) appeared
and dyspnea increased. Objective status: T - 38*C. Facial cyanosis is noticed. Breathing rate - 28/min. Pulse rate -
120/min. Blood pressure - 120/80 mmHg. Auscultation findings: rales are present in the lower part of the right lung.
Swelling in the right calf is noticed. Possitive Homand’s sigh in the same calf is determined. X-ray - triangle form
infiltrate in the lower part of the right lung. What is preliminary diagnosis:
A. Lobular pneumonia
B. Lobar pneumonia
C. Infarct-pneumonia

31.77 62 years male complains of fever 38*C, dyspnea, chest pain, which increases during deep breathing,
sputum with traces of blood. Dyspnea and tachycardia started suddenly 4 days ago, later fever (38*C) appeared
and dyspnea increased. Objective status: T - 38*C. Facial cyanosis is noticed. Breathing rate - 28/min. Pulse rate -
120/min. Blood pressure - 120/80 mmHg. Auscultation findings: rales are present in the lower part of the right lung.
Swelling in the right calf is noticed. Possitive Homand’s sigh in the same calf is determined. X-ray - triangle form
infiltrate in the lower part of the right lung.Which drugs would you prescribe:
A. Expectorants (acetylcysteine) and analgetics
B. Antibiotics and heparin
C. Spazmolytics and anticoagulants

31.78 36 years old male complains of weakness, fever 37 - 38*C, dry cough. These symptoms started two months
ago. Objective status: Pulse - 80/min. Blood pressure - 130/80 mmHg. Erythema nodosum is noticed on the
anterior surface of the right calf. Chest film shows bilateral hilar enlargement. What disease would you suspect:
A. Sarcoidosis
B. Lung cancer
C. Pneumonia
D. Lymphoma (Hodgkin’s disease)

31.79 36 years old male complains of weakness, fever 37 - 38*C, dry cough. These symptoms started two months
ago. Objective status: Pulse - 80/min. Blood pressure - 130/80 mmHg. Erythema nodosum is noticed on the
anterior surface of the right calf. Chest film shows bilateral hilar enlargement. Which test you need to performe to
prove diagnosis:
A. Lung tissue or lymph node biopsy
B. Chest tomograpgy
C. Ventilation/perfusion lung scan
D. Chest CT

31.80 36 years old male complains of weakness, fever 37 - 38*C, dry cough. These symptoms started two months
ago. Objective status: Pulse - 80/min. Blood pressure - 130/80 mmHg. Erythema nodosum is noticed on the
anterior surface of the right calf. Chest film shows bilateral hilar enlargement. Which treatment would you
prescribe:
A. Antibiotics
B. Corticosteroids
C. Cytostatic drugs
D. Non specific anti-inflamatory drugs

31.81 45 years old female complains of dyspnea, fever up to 38*C, sputum with traces of blood. Symptoms started
two months ago. The first signs were fever and breathlessness. She was admitted to the hospital where
haemoptysis started. Objective status: Breathing rate - 28/min. Rales bilateraly in the lower parts of the lungs are
presented. Heart rate - 110 beats/min. Blood pressure - 120/80 mmHg. Blood test: Hb - 80g/l, erythrocyte count -
1.9 x 1012/l, leucocyte count - 5.0 x 109/l, eosinophils - 1%, neutrophils:, segmented - 70%; lymphocytes - 19%,
monocytes - 10%, ESR - 45mm/h, potassium - 6.5mmol/l, urea - 40.1mmol/l. Urine analysis revealed hematuria,
urine gravity - 1.010. Conclusion of the chest X-ray - interstitial pulmonary edema. Preliminary diagnosis is:
A. Acute glomerulonephitis
lsmusis.lsmuni.lt/Klausimai/Spausdinti?Length=0?Kalba=EN&KategorijaId=133&Kalbos_input=EN&Kalbos=EN&KategorijaEn_input=Pulmonology&… 10/13
3/27/2019 LSMUSIS
B. Leptospirosis
C. Pulmopnary embolism
D. Goodpasture syndrome

31.82 45 years old female complains of dyspnea, fever up to 38*C, sputum with traces of blood. Symptoms started
two months ago. The first signs were fever and breathlessness. She was admitted to the hospital where
haemoptysis started. Objective status: Breathing rate - 28/min. Rales bilateraly in the lower parts of the lungs are
presented. Heart rate - 110 beats/min. Blood pressure - 120/80 mmHg. Blood test: Hb - 80g/l, erythrocyte count -
1.9 x 1012/l, leucocyte count - 5.0 x 109/l, eosinophils - 1%, neutrophils:, segmented - 70%; lymphocytes - 19%,
monocytes - 10%, ESR - 45mm/h, potassium - 6.5mmol/l, urea - 40.1mmol/l. Urine analysis revealed hematuria,
urine gravity - 1.010. Conclusion of the chest X-ray - interstitial pulmonary edema. What test should be made to
prove diagnosis:
A. Seeking leptospiras (direct examination of blood or urine by dark - field methods)
B. Ventilation/perfusion lung scan
C. Chest CT
D. Detection of Anti-Glomerular Basement Membrane Antibodies

31.83 45 years old woman complains of dyspnea. At the beginning dyspnea was while working and now patient
feels it even at rest. Symptoms started after viral infection. Objective status: Cyanosis of lips and face is noticed.
Heart rate - 100 beats/min. Blood pressure - 110/60 mmHg. Auscultation findings: bibasilar, inspiratory crackles
(dry or “Velcro” type in quality). Blood analysis: Hb - 120g/l, leukocyte count - 2.8 x 109/l, eosinophils - 1%,
neutrophils: band - 1%, segmented - 73%; lymphocytes - 18%, monocytes - 7%, ESR - 32mm/h. Chest X-ray
reveals a pattern of diffuse reticular markings prominent in the lower lung zones. What is the prelimenary
diagnosis:
A. Idiopatic pulmonary fibrosis (Hemann - Rich disease)
B. Residual changes after viral infection
C. Pneumonia
D. Pulmonary embolsim

31.84 45 years old woman complains of dyspnea. At the beginning dyspnea was while working and now patient
feels it even at rest. Symptoms started after viral infection. Objective status: Cyanosis of lips and face is noticed.
Heart rate - 100 beats/min. Blood pressure - 110/60 mmHg. Auscultation findings: bibasilar, inspiratory crackles
(dry or “Velcro” type in quality). Blood analysis: Hb - 120g/l, leukocyte count - 2.8 x 109/l, eosinophils - 1%,
neutrophils: band - 1%, segmented - 73%; lymphocytes - 18%, monocytes - 7%, ESR - 32mm/h. Chest X-ray
reveals a pattern of diffuse reticular markings prominent in the lower lung zones. What test should be done to
prove diagnosis:
A. Analysis of arterial blood gases
B. Spirogram
C. Pulmonary angiograpgy
D. Lung tissue biopsy

31.85 45 years old woman complains of dyspnea. At the beginning dyspnea was while working and now patient
feels it even at rest. Symptoms started after viral infection. Objective status: Cyanosis of lips and face is noticed.
Heart rate - 100 beats/min. Blood pressure - 110/60 mmHg. Auscultation findings: bibasilar, inspiratory crackles
(dry or “Velcro” type in quality). Blood analysis: Hb - 120g/l, leukocyte count - 2.8 x 109/l, eosinophils - 1%,
neutrophils: band - 1%, segmented - 73%; lymphocytes - 18%, monocytes - 7%, ESR - 32mm/h. Chest X-ray
reveals a pattern of diffuse reticular markings prominent in the lower lung zones. The patient should be treated
with:
A. Antibiotics and glucocorticoids
B. Antibiotics and cytostatic drugs
C. Antibiotics and non-steroidal anti-inflamatory drugs
D. Corticosteroids and cytostatic drugs

31.86 62 years old woman complains of fever up to 38*C, long lasting rhinitis and maxillary sinusitis. She had
rhinitis et sinusitis many times during last year. Patient is admitted to department of ORL. Blood analysis: Hb -
110g/l, leucocytes count - 5.0 x 109/l, monocytes - 10%, neutrophils: band - 1%, segmented - 68%; eosinophils -
1%, lymphocytes - 20%, ESR - 66mm/h. Urine analysis: gravity - 1.015, leuc. 6 - 10, erythrocytes 12 - 24 in vision
field. Chest X-ray reveals poorly defined nodular infiltrate in the right hilum. The biopsy of bronchi and maxillary
sinus was performed. Which disease would you suspect:
A. Lung cancer
B. Sarcoidosis
C. Goodpasture syndrome
lsmusis.lsmuni.lt/Klausimai/Spausdinti?Length=0?Kalba=EN&KategorijaId=133&Kalbos_input=EN&Kalbos=EN&KategorijaEn_input=Pulmonology&… 11/13
3/27/2019 LSMUSIS
D. Wegener's granulomatosis

31.87 62 years old woman complains of fever up to 38*C, long lasting rhinitis and maxillary sinusitis. She had
rhinitis et sinusitis many times during last year. Patient is admitted to department of ORL. Blood analysis: Hb -
110g/l, leucocytes count - 5.0 x 109/l, monocytes - 10%, neutrophils: band - 1%, segmented - 68%; eosinophils -
1%, lymphocytes - 20%, ESR - 66mm/h. Urine analysis: gravity - 1.015, leuc. 6 - 10, erythrocytes 12 - 24 in vision
field. Chest X-ray reveals poorly defined nodular infiltrate in the right hilum. The biopsy of bronchi and maxillary
sinus was performed. In the biopsy you expect to find:
A. Granuloma with caseous necrosis
B. Granulomas with Langhan’s and giant cells
C. Granulomas infiltrated by lymphocytes and macrophages
D. Granulomas which consist of epitheloid Langhan’s and giant cells, fibrin and necrotic masses of small
vessels

31.88 56 years old male with primary arterial hypertension from youth. he complains of fever 38 - 39*C,
progressive severe dyspnea. These complains have been lasting for two weeks. He was treated with variuos
antibiotics (cefotaxime, genthamicine, penicillin). There was no effect. Objective status. Cyanosis of the face.
Heart rate - 110 beats/min. Blood pressure - 180/110 mmHg. Auscultation findings: crakles on inspiration (as
polyethylen bag) at the lung bases bilateraly. Chest X-ray shows increased bronchovascular markings and venous
congestion. Blood analysis: Hb - 138g/l, leucocyte count - 4.0 x 109/l, eosinophils - 1%, eutrophils: band - 0%,
segmented - 70%; lymphocytes - 19%, monocytes - 10%, ESR - 45mm/h. What is the prelimenary diagnosis:
A. Pulmonary embolsim
B. Wegener’s granulomatosis
C. pneumonia
D. Idiopatic interstitial pulmonary fibrosis

31.89 56 years old male with primary arterial hypertension from youth. he complains of fever 38 - 39*C,
progressive severe dyspnea. These complains have been lasting for two weeks. He was treated with variuos
antibiotics (cefotaxime, genthamicine, penicillin). There was no effect. Objective status. Cyanosis of the face.
Heart rate - 110 beats/min. Blood pressure - 180/110 mmHg. Auscultation findings: crakles on inspiration (as
polyethylen bag) at the lung bases bilateraly. Chest X-ray shows increased bronchovascular markings and venous
congestion. Blood analysis: Hb - 138g/l, leucocyte count - 4.0 x 109/l, eosinophils - 1%, eutrophils: band - 0%,
segmented - 70%; lymphocytes - 19%, monocytes - 10%, ESR - 45mm/h. Which test should be performwd to
prove diagnosis:
A. Pulmonary angiography
B. Bronchoscopy
C. Tomography through the lung hylum
D. Lung tissue biopsy

31.90 45 years old male regular consumer of alcohol, complains of cough with purulent sputum and sometimes of
blood. Mild dyspnea is presented while working. Fever up to 38*C is often presented. Symptoms have been lasting
for five years. Objective status. Pale. Auscultation findings: coarse breathing, few rales are present at the lung
bases. Chest radiograph reveals increased picture of the lungs. Blood analysis: leucocyte count - 7.2 x 109/l, Hb -
90g/l. Urine analysis: protein 1g/l. What disease would you suspect:
A. Idiopatic interstitial pulmonary fibrosis
B. Disease of Randeux - Osler
C. Pulmonary tuberculosis
D. Bronchiectasies

31.91 45 years old male regular consumer of alcohol, complains of cough with purulent sputum and sometimes of
blood. Mild dyspnea is presented while working. Fever up to 38*C is often presented. Symptoms have been lasting
for five years. Objective status. Pale. Auscultation findings: coarse breathing, few rales are present at the lung
bases. Chest radiograph reveals increased picture of the lungs. Blood analysis: leucocyte count - 7.2 x 109/l, Hb -
90g/l. Urine analysis: protein 1g/l. Which test should be performed to prove the diagnosis:
A. Transbronchial lung biopsy
B. Bronchoscopy
C. Sputum culture
D. Chest CT

31.92 45 years old male regular consumer of alcohol, complains of cough with purulent sputum and sometimes of
blood. Mild dyspnea is presented while working. Fever up to 38*C is often presented. Symptoms have been lasting
for five years. Objective status. Pale. Auscultation findings: coarse breathing, few rales are present at the lung
lsmusis.lsmuni.lt/Klausimai/Spausdinti?Length=0?Kalba=EN&KategorijaId=133&Kalbos_input=EN&Kalbos=EN&KategorijaEn_input=Pulmonology&… 12/13
3/27/2019 LSMUSIS
bases. Chest radiograph reveals increased picture of the lungs. Blood analysis: leucocyte count - 7.2 x 109/l, Hb -
90g/l. Urine analysis: protein 1g/l. What is the cause of anemia:
A. Bleeding from the lungs
B. Amyloidosis
C. Hemolysis
D. This hemoglobin rate is normal

31.93 After examination squamous cell carcinoma of the right medium lobe was diagnosed and metastatic nodules
were also found in the apical segment of the right inferior lobe, increased lymphnodes were confirmed in the right
hillium. What is the stadium of the disease:
A. I A or I B
B. II A or II B
C. III A or III B
D. IV

31.94 After examination squamous cell carcinoma of the right medium lobe was diagnosed and metastatic nodules
were also found in the apical segment of the right inferior lobe, increased lymphnodes were confirmed in the right
hillium. How this patient should be treated:
A. Right pulmonectomy must be performed, then symptomatic treatment
B. Radiotherapy and/or chemotherapy
C. Symptomatic treatment
D. Chemotherapy only

31.1 - A 31.2 - B 31.3 - A 31.4 - B 31.5 - B 31.6 - C 31.7 - D 31.8 - D


31.9 - C 31.10 - A 31.11 - D 31.12 - D 31.13 - A 31.14 - D 31.15 - D 31.16 - B
31.17 - D 31.18 - A 31.19 - D 31.20 - D 31.21 - A 31.22 - D 31.23 - D 31.24 - D
31.25 - D 31.26 - A 31.27 - D 31.28 - A 31.29 - D 31.30 - D 31.31 - A 31.32 - A
31.33 - C 31.34 - A 31.35 - C 31.36 - A 31.37 - A 31.38 - D 31.39 - A 31.40 - A
31.41 - C 31.42 - A
31.43 31.44 31.45 31.95
1-B 1-B 1-D 1-A
2-A 2-A 2-C 2-D
3-C 3-C 3-B 3-C
4-D 4-D 4-A 4-B
31.46 31.47 31.48 31.49 31.50 31.51 31.52 31.53
1 1 1 1 1 1 1 1
2 3 3 2 3 3 2 3
3 3 3
31.54 - D 31.55 - A 31.56 - D 31.57 - C 31.58 - D 31.59 - C 31.60 - C 31.61 - D
31.62 - D 31.63 - D 31.64 - D 31.65 - D 31.66 - D 31.67 - C 31.68 - A 31.69 - C
31.70 - D 31.71 - C 31.72 - D 31.73 - A 31.74 - B 31.75 - D 31.76 - C 31.77 - B
31.78 - A 31.79 - A 31.80 - B 31.81 - D 31.82 - D 31.83 - A 31.84 - D 31.85 - D
31.86 - D 31.87 - D 31.88 - D 31.89 - D 31.90 - D 31.91 - D 31.92 - B 31.93 - D
31.94 - B

lsmusis.lsmuni.lt/Klausimai/Spausdinti?Length=0?Kalba=EN&KategorijaId=133&Kalbos_input=EN&Kalbos=EN&KategorijaEn_input=Pulmonology&… 13/13
3/27/2019 LSMUSIS

Tyberculosis
I type tasks. Choose only one best answer

28.1 What is the main natural reservoir of tuberculosis infection:


A. Human
B. Cows
C. Goats
D. Pigs
E. Birds

28.2 What is the most frequent way tuberculosis infection can be spread:
A. aerogenic way
B. Alimentary way
C. Contact way
D. Through the placenta

28.3 Which M. tuberculosis diagnostics method is mandatory in all Primary Health care institutions:
A. Simple bacterioscopy
B. Luminescence microscopy
C. Sputum cultures
D. Radiometric (BAC-TEC) culture

28.4 What is the most effective method to investigate M. tuberculosis and its resistance to antimicrobial therapy:
A. Levenstain – Jensen culture
B. Liquid culture
C. Radiometric (BAC-TEC) culture
D. Biological test

28.5 What cells are pathognomic for tuberculous granuloma:


A. Lymphocytes
B. Macrophages
C. Epitheloid cells
D. Pirogov – Langhans giant cells
E. Alien giant bodies

28.6 Which tuberculin test should be used to identify M. tuberculosis infection:


A. Pirke (scarification)
B. Mantoux (intradermal)
C. Koch (subdermal).
D. Moore (epidermal)

28.7 Why Koch test is performed:


A. To identify the “virage” of tuberculin
B. To identify the infection of M. tuberculosis
C. To diagnose the process of tuberculosis
D. To identify the specific immunity
E. To select the people which need BSG revaccination

28.8 Which kinds of Mycobacteria cause tuberculosis for human:


A. M. tuberculosis
B. M. bovis
C. M. africanum
D. M. fortuitum

28.9 What is the most outstanding feature of tuberculous granuloma:


A. Caseous necrosis
B. Epitheloid cells
C. Pirogov’s – Langhan’s cells
D. Lymphocytes

28.10 Primary complex of tuberculosis consist of:


A. Primary foccus, TB lymphadenitis, TB lymphangoitis
lsmusis.lsmuni.lt/Klausimai/Spausdinti?Length=0?Kalba=EN&KategorijaId=130&Kalbos_input=EN&Kalbos=EN&KategorijaEn_input=Tyberculosis&Kat… 1/5
3/27/2019 LSMUSIS
B. Primary foccus, TB lymphangoitis, TB bronchadenitis
C. Primary foccus, TB vasculitis
D. TB lymphadenitis, TB vasculitis

28.11 The Gohn foccus is:


A. Calcified lymph node
B. Calcified infiltration
C. Calcified primary foccus
D. Calcified pleura

28.12 Tuberculin virage is:


A. Positive Montoux test
B. Positive Koch test
C. Positive Pirke test
D. First time found tuberculin positive reaction, which means that organism has already been infected (when
previous tuberculin tests is known being negative)

28.13 In cases of disseminated tuberculosis bacilli are spread almost exclusively from:
A. Mediastinal lymph nodes TB
B. Bronchial TB
C. Infiltrative TB
D. Asman infiltration

28.14 What is tuberculous sepsis:


A. Focal TB
B. Acute disseminated miliary tuberculosis
C. Chronic miliary dissemination
D. Asman infiltration

28.15 What is caseotic pneumonia:


A. Infiltrative TB form
B. Staphylococcal pneumonia
C. Pneumonia caused by P. aeruginosa
D. Disseminated TB

28.16 Which form of tuberculosis is known as the most dangerous epidemiologicaly:


A. Fibro-cavernous TB
B. Cirrhotic TB
C. Primary TB
D. Focal TB

28.17 Common changes in cirrhotic pulmonary tuberculosis are:


A. Prominent connective tissue hyperplasia with active inflammation
B. Prominent connective tissue hyperplasia
C. Massive fibroblast infiltration
D. Massive lymphocyte infiltration

28.18 Which of these following combinations of antituberculosis medications are the most effective ones:
A. Ethambutol and pyrazinamide
B. Rifampicin and isoniazid
C. Ethambutol and rifampicin
D. Rifampin and ethambutol

28.19 Where tuberculosis is the leading cause of death in AIDS patients:


A. Africa
B. USA
C. Russia
D. Western Europe

28.20 First TB management category are linked all cases which are:
A. Newly discovered TM (+) or (and) has very prominent pathological changes in lugs or other organs
B. Newly discovered TM (-)
C. Chronic TB patients
lsmusis.lsmuni.lt/Klausimai/Spausdinti?Length=0?Kalba=EN&KategorijaId=130&Kalbos_input=EN&Kalbos=EN&KategorijaEn_input=Tyberculosis&Kat… 2/5
3/27/2019 LSMUSIS
D. Extrapulmonary TB patients

28.21 Patients who belongs to the first TB management category are given:
A. INH-RIF-PZA-EMB-(SM)
B. INH-RIF-SM
C. INH-RIF-SM-PZA
D. INH-RIF

II type tasks. For each numbered item,selct the one lettered heading that is most closely asssciated with it

28.22 Which anti-tuberculosis medication combination WHO recommends to use for TB treatment according the
TB treatment line (category):
1 - first tuberculosis therapy category
2 - Second tuberculosis therapy category
3 - Third tuberculosis therapy category
4 - Fourth tuberculosis therapy category
A. HRZ
B. HRZE(S)
C. HRZES
D. WHO does not indicates any drug combination

III type tasks. For each question there is one or more correct answers:
A – if correct answers are 1,2,3
B – if correct answers are 1 and 3
C – if correct answers are 2 and 4
D – if correct answer is 4
E – if correct are all answers above

28.23 Positive Mantoux test means:


1. TM infection
2. The reaction after vaccination
3. Active TB disease
4. TB in the past

28.24 How TB bacili are disseminated through the body tissues:


1. Hematogenic way
2. Lymphogenic way
3. Direct contact
4. Bronchogenic way

28.25 Primary TB complications:


1. Local TB forms
2. Dissemination
3. Pleural effusion
4. Extrapulmonary TB

28.26 The primary complex has the following radiographical stages:


1. Pneumonic
2. Organization and consolidation
3. Calcification
4. Dissemination

28.27 Primary tuberculosis joins the following entities:


1. Mediastinal lymph node TB
2. Primary complex
3. TB pleuritis
4. Disseminated pulmonary TB

28.28 Infiltrative TB forms:


1. "Clouds” form
2. Broncholobular form
3. Lobitis and perisisuritis
lsmusis.lsmuni.lt/Klausimai/Spausdinti?Length=0?Kalba=EN&KategorijaId=130&Kalbos_input=EN&Kalbos=EN&KategorijaEn_input=Tyberculosis&Kat… 3/5
3/27/2019 LSMUSIS
4. Asman infiltration

28.29 What are complications of cirrhotic TB:


1. Respiratory failure
2. Chronic cor pulmonale
3. Bleeding from the lungs
4. Amyloidosis

28.30 What are tuberculosis management principles:


1. Early as possible and right timed
2. Monotherapy is not allowed
3. Treatment must be continued at least 6 months.
4. The continuation of treatment is obligatory

28.31 What specific clinical manifestation appears in AIDS patients, when tey get ill with TB:
1. Generalized lymphadenopathy, milliary TB form
2. Mantoux test is negative and TM are not found
3. Diarrhea for more than 1 month, Herpes zoster infection, Kaposhi sarcoma
4. Fast progressive course

IV type tasks. Choose only one best answer

28.32 Since 1989-1990 more and more TB cases has been registered in Lithuania. According to WHO experts,
when TB morbidity excess 100 for 100000 inhabitants, the epidemic TBsituation could be expected. Which
epidemiological value shows very well the possibility of new TB cases, especially among children and young
people:
A. Infectivity with mycobacteria (the risks of TM infection each year)
B. The rates of active disease
C. Morbidity
D. Mortality

28.33 Since 1989-1990 more and more TB cases has been registered in Lithuania. According to WHO experts,
when TB morbidity excess 100 for 100000 inhabitants, the epidemic TB situation could be expected. What TB
prophylactics are recommended by WHO experts as the most effective:
A. Tuberculin test use
B. BCG vaccination
C. DOTS in short standard regimen
D. Fluorographic X-ray screening

28.34 60 years old man, sick with RA and taking oral glucocorticoids, started to cough, haemoptysis and
intoxication syndrome appeared. 5 years ago he had infiltrative form of pulmonary TB and has been treated with
anti-tuberculosis medications for 4 months in the hospital. What is your preliminary diagnosis:
A. Lung cancer
B. Pulmonary TB
C. Pneumonia
D. Chronic bronchitis

28.35 60 years old man, sick with RA and taking oral glucocorticoids, started to cough, haemoptysis and
intoxication syndrome appeared. 5 years ago he had infiltrative form of pulmonary TB and has been treated with
anti-tuberculosis medications for 4 months in the hospital. Which tests should be performed to prove the diagnosis:
A. Sputum culture and bronchoscopy
B. Blood smear and chest X-ray
C. Bronchoscopy and chest X-ray
D. Sputum smear by Ziehl – Neelsen stain and chest X-ray

28.1 - A 28.2 - A 28.3 - A 28.4 - C 28.5 - D 28.6 - B 28.7 - C 28.8 - A


28.9 - A 28.10 - A 28.11 - C 28.12 - D 28.13 - A 28.14 - B 28.15 - A 28.16 - A
28.17 - A 28.18 - B 28.19 - A 28.20 - A 28.21 - A
28.22
1-B
2-C

lsmusis.lsmuni.lt/Klausimai/Spausdinti?Length=0?Kalba=EN&KategorijaId=130&Kalbos_input=EN&Kalbos=EN&KategorijaEn_input=Tyberculosis&Kat… 4/5
3/27/2019 LSMUSIS
3-A
4-D
28.23 28.24 28.25 28.26 28.27 28.28 28.29 28.30
1 1 1 1 1 1 1 1
2 2 2 2 2 2 2 2
3 3 3 3 3 3 3 3
4 4 4 4 4 4
28.31
1
2
3
4
28.32 - A 28.33 - C 28.34 - B 28.35 - D

lsmusis.lsmuni.lt/Klausimai/Spausdinti?Length=0?Kalba=EN&KategorijaId=130&Kalbos_input=EN&Kalbos=EN&KategorijaEn_input=Tyberculosis&Kat… 5/5
3/27/2019 LSMUSIS

Gastroenterology
I type tasks. Choose only one best answer

37.1 Which type of acute gastritis is characterized by these histological changes: hyperemia, swelling, pronounced
infiltration of polimorphonucleares in all layers of gastric wall
A. Catharic
B. Erosive
C. Hemorrhagic
D. Phlegmonic

37.2 There are the following types (variants) of functional dyspepsia, except
A. Ulcer-like dyspepsia
B. Gaastroesophageal reflux disease-like dyspepsia
C. Dismotility-like dyspepsia
D. Non-specific dyspepsia

37.3 All represented features of H.pylori diagnostic tests are correct except one:
A. Sensitivity of CLO rest is reduced if stomach contains blood
B. All invasive tests can be false-negative if biopsy is taken from atrophic gastric mucosa
C. Serological tests are best for evaluation of treatment efficacy
D. Rapid urease test can be false negative if biopsy specimens are taken just after stopping antibiotics

37.4 Following features are characteristic for diffuse type of gastric cancer except one:
A. Males and females are affected at similar rates
B. Incidence is similar in different countries
C. Malignant cells infiltrate gastric wall without destroying neoplasm
D. Malignant cells are well differentiated, pronounced reaction of stroma

37.5 What type of abdominal pain is mostly characteristic for irritable bowel syndrome:
A. Colic type of pain
B. Pain after fatty meal
C. Pain relieved by defecation or gas outlet
D. Pain localised in the middle of abdomen, around umbilicus

37.6 14.6. Which of following clinical features suggests rectal cancer:


A. Absence of urge to defecate
B. Altered stool form (lumpy or watery stool)
C. Necessity of finger-help for complete defecation
D. Bloody stool, urgency

37.7 Which feature of stool is mostly characteristic for active ulcerative colitis:
A. White colour of stool
B. Consistency of liquid mush
C. Undigested fibers in stool
D. Bloody diarrhoea

37.8 Which endoscopic feature of large bowel ulcers is mostly relevant for ulcerative colitis:
A. Segmental pattern
B. Ulcers in diffusely inflamed colonic mucosa
C. Aphthous ulcers
D. Ulcers with irregular margins

37.9 Which of the following considerations about large bowel diverticulosis is incorrect:
A. Diverticulosis most often is symptomatic condition
B. Most frequent localisation of diverticula’s is sigmoid colon
C. Diverticulitis increases risk of complications during colonoscopy or double contrast X ray examination of
large bowel
D. Diverticulitis most often is managed by surgery

37.10 Which of these features is most relevant for initial clinical presentation of right colon cancer:
A. Ferrum deficiency anaemia without obvious blood in stool
B. Urge to defecate with mucus and blood
lsmusis.lsmuni.lt/Klausimai/Spausdinti?Length=0?Kalba=EN&KategorijaId=139&Kalbos_input=EN&Kalbos=EN&KategorijaEn_input=Gastroenterology… 1/7
3/27/2019 LSMUSIS
C. Constipation changed by short periods of diarrhoea
D. Symptoms of bowel obstruction

37.11 Which therapeutic option is the best for managing chronic pancreatitis induced malabsorption:
A. Pancreatic enzymes in tablets containing big amount of lipase and covered with acid resistant capsule
B. Pancreatic enzymes in minimicro-granules containing big amount of lipase and covered with acid
resistant capsule
C. Pancreatic enzymes in tablets containing little amount of lipase and covered with acid resistant capsule,
used before every meal (6-8 at once)
D. Pancreatic enzymes in simple tablets containing little amount of lipase and used with acid suppressive
agents

37.12 Which one of following criteria is not used in Child–Pugh classification of liver function in cirrhosis:
A. Prothrombin time
B. Elevation of transaminases
C. Amount of ascites
D. Serum albumin level

37.13 Which one of following biochemical syndromes is most relevant for alcoholic hepatitis:
A. AST – 80 U/L, ALT- 120 U/L, AP – 150 U/L, γGT- 45 U/L
B. AST – 240 U/L, ALT- 120 U/L, AP – 120 U/L, γGT- 250 U/L
C. AST – 40 U/L, ALT- 60 U/L, AP – 600 U/L, γGT- 200 U/L
D. AST – 500 U/L, ALT- 700 U/L, AP – 210 U/L, γGT- 40 U/L

37.14 Which drug combination can be used in the management of chronic active hepatitis B:
A. Prednisolone and Azathioprine
B. Interferon and Ribavirin
C. Interferon and Entecavir
D. Prednisolone and Ribavirin

37.15 Which combination of biochemical parameters is most relevant for cholestatic jaundice:
A. Elevation of conjugated bilirubin, decreased level of serum AP, elevation of γGT
B. Elevation of unconjugated bilirubin, increased level of serum AP, elevation of γGT
C. Elevation of conjugated bilirubin, increased level of serum AP, elevation of γGT
D. Normal value of bilirubin, increased level of serum AP, normal level of γGT

II type tasks. For each numbered item,selct the one lettered heading that is most closely asssciated with it

37.16 Choose correct answer for each combination of biochemical parameters and viral markers:
1 - HBsAg (+), HBeAg (+)
2 - antiHCV (+), HCV-RNR (+)
3 - Decreased level of serum albumin, prothrombin, increased concentration of bilirubin
4 - HBsAg(-); Anti-HBs (+); AntiHBc IgG (-)
A. Infection with hepatitis C virus
B. Active replication of hepatitis B virus
C. Vaccine induced immunity against hepatitis B virus
D. Hepatocellular insufficiency

37.17 Which type of peptic ulcer can be characterized by following pathophysiological changes:
1 - Increased food stimulated HCl secretion
2 - Decreased mass of gastric parietal cells
3 - Increased nocturnal secretion of HCl
4 - Delayed gastric emptying
A. Gastric ulcer
B. Duodenal ulcer
C. Gastric and duodenal ulcer
D. These changes are not characteristic for peptic ulcer

37.18 Which grade of Los-Angeles erosive esophagitis classification is represented it the following description:
1 - Several longitudinal erosions less than 5 mm length, non confluent
2 - Several longitudinal erosions more than 10 mm length, confluent in about 50 percents of circumference
3 - One longitudinal erosion 3 mm length

lsmusis.lsmuni.lt/Klausimai/Spausdinti?Length=0?Kalba=EN&KategorijaId=139&Kalbos_input=EN&Kalbos=EN&KategorijaEn_input=Gastroenterology… 2/7
3/27/2019 LSMUSIS
4 - Many longitudinal erosions, confluent in more than 75 percents of circumference
A. Grade A
B. Grade B
C. Grade C
D. Grade D

37.44 Which class of drugs do these medicaments, used for treatment of GERD, belong to:
1 - Ranitidine
2 - Calcium carbonate
3 - Metoclopramide
4 - Pantoprazole
A. Antacid
B. H2 receptor blockers
C. H+ K+ ATPase inhibitors
D. Procinetic agent

37.45 Choose correct medical therapy for liver disease:


1 - Primary biliary cholangitis
2 - Liver cirrhosis
3 - Viral hepatis B
4 - Autoimmune hepatitis
A. Ursodeoxycholic acid
B. Prednisolone, azathioprine
C. Lamivudin, entecavir
D. Direct antivirals

37.46 What is the pathophysiological mechanism for each type of jaundice:


1 - Parenchymal
2 - Obstructive
3 - Hemolytic
A. Increased degradation of erythrocytes
B. Bile flow disturbance
C. Liver cell damage

37.47 Choose the appropriate set of laboratory test values for each of the diseases:
1 - Viral hepatitis B
2 - Viral hepatitis C
3 - Alcoholic hepatitis
4 - Autoimmune hepatitis
A. ANA(+); antiHCV(-); HBsAg(-)
B. HbsAg(+); antiHBsAg(-); antiHBc (+); HCV-RNR (-)
C. AntiHCV (+); HCV-RNR(+); antiHBc (-)
D. HBsAg(-); anti-HCV(-); ANA (-), increased MCV

III type tasks. For each question there is one or more correct answers:
A – if correct answers are 1,2,3
B – if correct answers are 1 and 3
C – if correct answers are 2 and 4
D – if correct answer is 4
E – if correct are all answers above

37.19 Which statements concerning GERD are correct:


1. Presence of typical symptoms – heartburn and regurgitation is enough for establishing diagnosis
2. Mucosal break (erosion) is the only significant criteria of reflux-esophagitis
3. GERD can present without endoscopic lesions (erosions)
4. There is no correlation of clinical symptoms and morphologic changes of oesophagus in GERD patients

37.20 What statements about Helicobacter pylori induced gastritis are correct:
1. Acute H.pylori gastritis is usually asymptomatic condition
2. H.pylori is a life-long infection if not eradicated, inducing chronic gastritis
3. H.pylori gastritis can progress to peptic ulcer disease

lsmusis.lsmuni.lt/Klausimai/Spausdinti?Length=0?Kalba=EN&KategorijaId=139&Kalbos_input=EN&Kalbos=EN&KategorijaEn_input=Gastroenterology… 3/7
3/27/2019 LSMUSIS
4. H.pylori gastritis progresses from superficial to atrophic with intestinal metaplasia

37.21 What type of vomiting suggests obstruction of pylorus:


1. Big volume of vomiting masses
2. Residuals of “old” food
3. Vomiting predominating in afternoon and at night
4. Bile in vomiting mass

37.22 Management of peptic ulcer disease is based on these rules:


1. H.pylori eradication is the first step therapy of H.pylori associated ulcers.
2. H.pylori eradication is the main tool preventing relapses of H.pylori associated ulcers
3. Following drug combinations are used for eradication of H.pylori: Proton pump inhibitor + two of these
antibiotics (amoxicillin, metronidazole, clarithromycin)
4. Control endoscopy is not necessary for evaluation of healing of non-complicated duodenal ulcer

37.23 What endoscopic signs suggests ulcerative gastric carcinoma:


1. Solid and indurated margins of the ulcer
2. Inflammatory oedema around ulcer base
3. Abruption of mucosal folds near ulcer base
4. Smooth and regular ulcer margins

37.24 What conditions increase risk for gastric carcinoma:


1. Gastric adenomas
2. Chronic atrophic autoimmune gastritis
3. Atrophic H.pylori gastritis
4. Hyperplastic gastric polyps

37.25 What peculiarities of diarrhoea are relevant for irritable bowel syndrome: 1. 2. 3. 4.
1. Diarrhoea in the morning or just after meal
2. Diarrhoea at night
3. Presence of liquid mucus in stool
4. Blood in stool

37.26 What statements concerning management of irritable bowel syndrome are correct:
1. Medical therapy is only symptomatic
2. Good therapeutic placebo effect
3. Education and reassurance are very important management options
4. Neuroleptic drugs are necessary

37.27 What diseases or conditions can cause symptomatic constipation:


1. Hyperthyreosis
2. Parkinson disease
3. Ellison-Zollinger syndrome
4. Diabetes mellitus

37.28 What types of diarrhoea resolve when fasting:


1. Secretory
2. Osmotic
3. Inflammatory (exudative)
4. Malabsorption-induced

37.29 When liver biopsy is contraindicated:


1. Thrombocytes <50 x 10⁹ l
2. INR < 1.4 - 1.5
3. Įtemptas ascitas
4. ALT >1000 u/l

37.30 Most important factors for development of portal ascites are:


1. Hypoalbuminemia
2. Portal hypertension
3. Increased serum aldosterone
4. Increased activity of transaminases

lsmusis.lsmuni.lt/Klausimai/Spausdinti?Length=0?Kalba=EN&KategorijaId=139&Kalbos_input=EN&Kalbos=EN&KategorijaEn_input=Gastroenterology… 4/7
3/27/2019 LSMUSIS
37.31 What of following cases present with unconjugated hyperbilirubinemia:
1. Pancreatic cancer
2. Haemolytic anaemia
3. Choledocholithiasis
4. Congenital deficiency of glucuronosyltransferase (Gilbert’s syndrome)

37.32 What statements about primary biliary cirrhosis are correct:


1. Itching is very common clinical symptom
2. Disease mostly affects elder males
3. Positive antimitochondrial antibodies (AMA) are common
4. Strictures of common bile duct on ERCP

IV type tasks. Choose only one best answer

37.33 48 years old male for several days complains of general weakness, dizziness. Since yesterday he defecates
3-4 times per day and his stool are soft and black. For two months he uses oral diclofenac for back pain,
occasionally adds injections of same drug. Patients is pale, his skin is whitish. Pulse 92 b/min. BP 116/78; No
other changes on physical examination. Digital examination of rectum showed residuals of black faeces. Hb 104
g/l; RBC 3.52x10¹²/l. What suggestion for diagnosis you will make:
A. Signs of bleeding from the stomach
B. Signs of bleeding from upper alimentary tract
C. Signs of bleeding from large bowel
D. Signs of rectal bleeding

37.34 48 years old male for several days complains of general weakness, dizziness. Since yesterday he defecates
3-4 times per day and his stool are soft and black. For two months he uses oral diclofenac for back pain,
occasionally adds injections of same drug. Patients is pale, his skin is whitish. Pulse 92 b/min. BP 116/78; No
other changes on physical examination. Digital examination of rectum showed residuals of black faeces. Hb 104
g/l; Er. 3.52x10¹²/l. What will be your first management option:
A. Sigmoidoscopy without bowel cleaning
B. Bowel cleaning and preparation for colonoscopy
C. Stomach cleaning with the help of gastric tube, in case of finding fresh blood, additional washing with
cold water and upper endoscopy
D. Upper endoscopy without stomach washing

37.35 63 years old male applied his doctor due to altered stool frequency: constipation is changed by 1-2 day’s
periods of loose stool. He did not notice any blood or mucus in faeces. He has lost appetite and his working
abilities clearly decreased. He hasn’t been sick of any diseases previously. Physical and rectal examination did not
show any abnormalities. Test for occult blood in stool is positive. What disease your will suspect first of all?
A. Irritable bowel syndrome
B. Ischemic colitis
C. Microscopic colitis
D. Distal large bowel cancer

37.36 32 years old male has functional constipation. Dietary and life-style changes did not result in resolution of
symptoms. Prescription of laxatives has been considered. Which group of laxatives should be administered first:
A. Bulking laxatives
B. Osmotic salts
C. Stimulating- antraglykozides
D. Castor oil

37.37 38 years old female was admitted to the hospital due to diarrhoea with bloody mucus for 8-10 times per day.
The patient is sick for 6 years, two times due to exacerbations of diarrhoea was treated in the hospital ward. After
prolonged medical therapy symptoms usually resolved. This time disease relapsed before two weeks. T° - 37.8° C;
Slight tenderness of left abdominal part on palpation. Pulse 86 b/min. BP 112/62; Hb. 95 g/l; Er. 3.7 x 10¹². CRP
35.3 mg/l; sigmoidoscopy without bowel preparation: 15 cm was investigated. Mucosa is diffusely oedematous,
hyperaemic, contact bleeding, erosions. Colonoscopy has been done one year ago – diffuse inflammatory
changes were seen in rectum, sigmoid colon, descending colon and left flexure. Which is the right diagnosis:
A. Mb. Crohn: colitis
B. Colitis ulcerosa: proctosigmoiditis, forma levis
C. Colitis ulcerosa: colitis sinistra, forma moderatus
D. Colitis ulcerosa: colitis sinistra, forma gravis
lsmusis.lsmuni.lt/Klausimai/Spausdinti?Length=0?Kalba=EN&KategorijaId=139&Kalbos_input=EN&Kalbos=EN&KategorijaEn_input=Gastroenterology… 5/7
3/27/2019 LSMUSIS
37.38 38 years old female was admitted to the hospital due to diarrhoea with bloody mucus for 8-10 times per day.
The patient is sick for 6 years, two times due to exacerbations of diarrhoea was treated in the hospital ward. After
prolonged medical therapy symptoms usually resolved. This time disease relapsed before two weeks. T° - 37.8° C;
Slight tenderness of left abdominal part on palpation. Pulse 86 b/min. BP 112/62; Hb. 95 g/l; Er. 3.7 x 10¹². CRP
35.3 mg/l; sigmoidoscopy without bowel preparation: 15 cm was investigated. Mucosa is diffusely oedematous,
hyperaemic, contact bleeding, erosions. Colonoscopy has been done one year ago – diffuse inflammatory
changes were seen in rectum, sigmoid colon, descending colon and left flexureWhat therapy you will administer:
A. Oral mesalazine
B. Oral steroids and mesalazine
C. Intravenous steroids
D. Azathioprine and sulfasalazine

37.39 42 years old female has Crohn’s disease. She is subfebrile, has bowel obstruction symptoms, Hb 101 g/l;
CRP 43 mg/l; Colonoscopy did not show any lesions in the colon. MR enterography was done. Terminal ileum has
inflammatory segment: thickened mucosal folds, large lymphoid folliculus, aphthous ulcers, 10 cm length stenosis
of the terminal ileum. Treatment with steroids was started, but no resolution of symptoms was achieved. What
management option you will choose:
A. Sulfasalazine and Budesonide
B. Mesalazine and azathioprine
C. Surgical resection of terminal ileum
D. Infliximab

37.40 44 years old female complains of general weakness. Ten years ago she received blood transfusion. Last 7
month slight elevation of liver transaminases was observed. ALT – 86 U/l, AST- 74 U/l, AP – 120 U/l, gamma GT –
50 U/l, bilirubin – 17 mikromol/l, prothrombin (SPA) – 88%, antiHCV – positive, HCV-RNR – positive, HBsAg –
negative, ANA – negative, AMA – negative, ANCA – negative. Histology of the liver: activity index – 4 points,
fibrosis – 0. What is the right clinical diagnosis:
A. Chronic, mild hepatitis B
B. Chronic, mild hepatitis C
C. Alcoholic hepatitis
D. Autoimmune hepatitis

37.41 44 years old female complains of general weakness. Ten years ago she received blood transfusion. Last 7
month slight elevation of liver transaminases was observed. ALT – 86 U/l, AST- 74 U/l, AP – 120 U/l, gamma GT –
50 U/l, bilirubin – 17 mikromol/l, prothrombin (SPA) – 88%, antiHCV – positive, HCV-RNR – positive, HBsAg –
negative, ANA – negative, AMA – negative, ANCA – negative. Histology of the liver: activity index – 4 points,
fibrosis – 1. What treatment you will prescribe: A. B. C. D.
A. Alfa-interferon 6 mln U/l three times per week for 3 month
B. Prednisolone
C. Pegilated interferon and Ribavirin
D. No specific treatment is needed

37.42 Patient for 15 years has hepatitis B, was observed and treated on the out patient basis. He drinks little
alcohol. Three months ago he noticed the enlargement of his abdomen, swelling of his legs. Three weeks ago he
became jaundiced and started to bleed from the gums. General weakness also appeared. Physical examination
showed spider angiomas of the chest, ascites, oedema of lower extremities, jaundice. Abdominal ultrasound: liver
enlarged and irregular, spleen – enlarged, v.portae – 1.4 cm; ascites. ALT – 126 U/l, AST- 134 U/l, AP – 180 U/l,
gamma GT – 120 U/l, bilirubin – 64 mikromol/l, conjugated – 49 mikromol/l, prothrombin SPA – 46%, albumin – 26
g/l, antiHCV – negative, HCV-RNR – negative, HBsAg – positive, ANA – – negative, AMA –– negative, ANCA – –
negative, Hb – 102 g/l, Er – 3,0 x 10¹² l, leukocytes – 2,8 x 10⁹ l, thrombocytes - 71 x 10⁹ l. Upper endoscopy:
oesophageal varices, III grade. What is the preliminary diagnosis:
A. Idiopathic liver cirrhosis
B. Chronic viral hepatitis
C. Viral B liver cirrhosis
D. Alcoholic liver cirrhosis

37.43 Patient for 15 years has hepatitis B, was observed and treated on the out patient basis. He drinks little
alcohol. Three months ago he noticed the enlargement of his abdomen, swelling of his legs. Three weeks ago he
became jaundiced and started to bleed from the gums. General weakness also appeared. Physical examination
showed spider angiomas of the chest, ascites, oedema of lower extremities, jaundice. Abdominal ultrasound: liver
enlarged and irregular, spleen – enlarged, v.portae – 1.4 cm; ascites. ALT – 126 U/l, AST- 134 U/l, AP – 180 U/l,
gamma GT – 120 U/l, bilirubin – 64 mikromol/l, conjugated – 49 mikromol/l, prothrombin SPA – 46%, albumin – 26
lsmusis.lsmuni.lt/Klausimai/Spausdinti?Length=0?Kalba=EN&KategorijaId=139&Kalbos_input=EN&Kalbos=EN&KategorijaEn_input=Gastroenterology… 6/7
3/27/2019 LSMUSIS
g/l, antiHCV – negative, HCV-RNR – negative, HBsAg – positive, ANA – – negative, AMA –– negative, ANCA – –
negative, Hb – 102 g/l, Er – 3,0 x 10¹² l, leukocytes – 2,8 x 10⁹ l, thrombocytes - 71 x 10⁹ l. Upper endoscopy:
oesophageal varices, III grade. What diagnostic option should be the first:
A. Liver biopsy
B. Diagnostic paracentesis of ascitic fluid
C. Plane X-ray of the abdomen
D. ERCP

37.1 - D 37.2 - B 37.3 - C 37.4 - D 37.5 - C 37.6 - D 37.7 - D 37.8 - B


37.9 - D 37.10 - A 37.11 - B 37.12 - B 37.13 - B 37.14 - C 37.15 - C
37.16 37.17 37.18 37.44 37.45 37.46 37.47
1-B 1-B 1-A 1-B 1-A 1-C 1-B
2-A 2-A 2-C 2-A 2-D 2-B 2-C
3-D 3-B 3-A 3-D 3-C 3-A 3-D
4-C 4-A 4-D 4-C 4-B 4-A
37.19 37.20 37.21 37.22 37.23 37.24 37.25 37.26
1 1 1 1 1 1 1 1
2 2 2 2 3 2 3 2
3 3 3 3 3 3
4 4
37.27 37.28 37.29 37.30 37.31 37.32
2 2 1 1 2 1
4 4 2 2 4 3
3 3
37.33 - B 37.34 - C 37.35 - D 37.36 - A 37.37 - D 37.38 - B 37.39 - D 37.40 - B
37.41 - C 37.42 - C 37.43 - B

lsmusis.lsmuni.lt/Klausimai/Spausdinti?Length=0?Kalba=EN&KategorijaId=139&Kalbos_input=EN&Kalbos=EN&KategorijaEn_input=Gastroenterology… 7/7
3/27/2019 LSMUSIS

Onkology
I type tasks. Choose only one best answer

6.1 In most European countries, breast cancer mortality is reduced by:


A. Less women have breast cancer
B. Ongoing screening programs, early diagnosis and improvement of treatment options
C. Palliative care for patients
D. Eliminating exposure to carcinogenic agents

6.2 Most common causes of death and place of oncological diseases in the mortality structure:
A. First place
B. Second place after cardiovascular diseases
C. Third place after cardiovascular diseases and trauma
D. Third place after cardiovascular and gastrointestinal diseases

6.3 Most common cancer among females:


A. Kidney cancer
B. Breast cancer
C. Endometrial cancer
D. Cervical cancer
E. Stomach cancer

6.4 Primary cancer prevention means:


A. Minimizing or eliminating exposure to carcinogenic agents
B. Early detection of precancerous diseases
C. Early detection of cancer
D. Rehabilitation of cancer patients.

6.5 What is the purpose of secondary cancer prevention (screening:


A. Reducing mortality from oncological diseases
B. High quality treatment for oncological diseases
C. Rehabilitation of Oncological Patients
D. Avoiding clinical complications

6.6 Palliative treatment for the cancer patient means:


A. Pain relief
B. Symptomatic treatment
C. Care
D. Improvement of the quality of life and prolongation of survival due to specific treatment methods and
active advanced cancer symptom control

6.7 How we can achieve cancer control?


A. By implementation of new anticancer drugs to the clinical practice
B. By evaluation of cancer causes and development
C. By cancer prevention, early diagnosis, evidence based treatment and palliative care
D. By development of new radiation therapy techniques

6.8 How we can prognose the radiosensitivity of the tumour:


A. According to the depth of the tumour infiltration
B. According to the morphology
C. According to the localization of the tumour
D. According to the dissemination of the tumour

6.9 In which phase of cell cycle cells are most sensitive to irradiation:
A. G1
B. S
C. G2
D. M
E. M and G2

6.10 What is Radiation Therapy?


A. Use of Ionizing Radiation in Diagnostics of Diseases;
lsmusis.lsmuni.lt/Klausimai/Spausdinti?Length=0?Kalba=EN&KategorijaId=108&Kalbos_input=EN&Kalbos=EN&KategorijaEn_input=Onkology&Kate… 1/13
3/27/2019 LSMUSIS
B. Use of Ionizing Radiation to treat Oncological Diseases;
C. Treatment of tumors with cytotoxic drugs;
D. Laser or ultrasound use for tumor destruction.

6.11 When we can expect response to the breast cancer hormonotherapy:


A. Early stage of breast cancer
B. Positive estrogen and/or progesterone receptors
C. Negative estrogen and/or progesterone receptors
D. No axillary lymphnodes metastases

6.12 The highest risk to have breast cancer is if the first degree relatives had a breast cancer:
A. In premenopausal status
B. In premenopausal status and bilateral breast cancer was diagnosed
C. In postmenopausal status
D. Bilateral breast cancer was diagnosed

6.13 How much of all breast cancer cases constitute hereditary forms of breast cancer?
A. From 5 till 10 percent
B. Aproximatelly 50 percent
C. From 80 till 90 percent
D. Unknown

6.14 The main treatment methods after breast - conserving surgery for patients with positive lymph nodes and
negative hormonal receptor status are:
A. Chemotherapy and radiation therapy
B. Chemotherapy and endocrine treatment
C. Endocrine treatment and targeted therapies
D. Radiotherapy and follow-up

6.15 Which infection is associated with cervical cancer:


A. Hepatitis B virus
B. Human papiloma virus
C. Herpes virus
D. Helicobacter pylori infection

6.16 Which drugs are not used in cancer chemotherapy:


A. Antimetabolites
B. Estrogens
C. Antitumor antibiotics
D. Alkylating agents
E. Vinca alkaloids

6.17 What is Complex Cancer management:


A. Operation + Radiation Therapy + Chemotherapy
B. Pain relieve
C. Targeted Therapy
D. Hyperthermia

6.18 The frequency of PAP smears in case of cervical cancer screening depends on:
A. Grade of cell atypy in PAP smear
B. Woman’s age
C. Number of childbearing
D. Cervical cancer screening finance

6.19 The mammographic screening of breast cancer starts at the age of:
A. 20 years
B. 50 years
C. 60 years
D. 70 years

6.20 Increasing breast cancer incidence is associated with:


A. Tobacco Smoking
B. High physical activity
lsmusis.lsmuni.lt/Klausimai/Spausdinti?Length=0?Kalba=EN&KategorijaId=108&Kalbos_input=EN&Kalbos=EN&KategorijaEn_input=Onkology&Kate… 2/13
3/27/2019 LSMUSIS
C. Decreasing number of deliveries, late birth, no breastfeeding
D. Vegetarian nutrition.

6.21 The mammographic screening of colorectal cancer starts at the age of:
A. 50 years
B. 30 years
C. 60 years
D. 70 years

6.22 The most common complication of radical prostatectomy:


A. Weak urine flow
B. Erectile dysfunction and urinary
C. Wound infection and deep leg vein thrombosis
D. Rectal damage

6.23 The indications for surgical treatment in case of spinal cord compression:
A. Pathologic fracture of the vertebra
B. Patient age over 70 year old
C. High grade differentiation of the tumour
D. Weakness, lasting over 1 month

6.24 Etiology of hypercalcemia in case of malignant tumours:


A. Increased calcium supply with food
B. Renal dysfunction
C. Osteolytic factors produced by the tumour cells
D. Diseases of the thyroid gland

6.25 Most common cause of vena cava superior syndrome:


A. Atherosclerosis
B. Small cell lung cancer
C. Prostate cancer
D. Bazalioma

6.26 Symptoms of vena cava superior syndrome:


A. Shortness of breath due to pulmonary edema
B. Paraplegija
C. Face and neck edema
D. Hand tremor

6.27 The most informative method to diagnose of vena cava superior syndrome:
A. Colposcopy
B. Ultrasonoscopy
C. Chest CT
D. Bone scan

6.28 The leading role in the development of breast cancer plays:


A. Growth hormone
B. Estrogens
C. Testosterone
D. Prostaglandins

6.29 Fourth stage of cancer disease is diagnosed when:


A. Regional lymph nodes metastases are obtained
B. Tumour is more than 5 cm size
C. At least one distant metastases is found
D. Patient is not suitable for specific treatment

6.30 Main principles of cancer pain relief are:


A. By WHO ladder, by clock, if possible - start with oral analgesics
B. By WHO ladder
C. Drug management, when pain increased
D. Start with morphine

lsmusis.lsmuni.lt/Klausimai/Spausdinti?Length=0?Kalba=EN&KategorijaId=108&Kalbos_input=EN&Kalbos=EN&KategorijaEn_input=Onkology&Kate… 3/13
3/27/2019 LSMUSIS
6.31 R1 criteria according malignant tumours TNM classification means:
A. Confirmed local tumour recurrence
B. Confirmed distant tumour recurrence (metastasis)
C. Microscopic residual tumour, surgical treatment is not radical
D. Macroscopic residual tumour, surgical treatment is not radical

6.32 The pathological TNM:


A. Replaces the clinical TNM
B. Does not alow to estimate prognosis of cancer
C. Both classifications (pathological and clinical TNM) are retained unaltered in the patient's record.
D. Is not used for the choice of treatment

6.33 Which drugs are not used for the treatment of emesis and vomiting?
A. Antagonists of 5-HT3 receptors
B. Aprepitants, a potent and selective antagonists of the neurokinin (NK) neurotransmitter receptor
C. Corticosteroids
D. Antagonists of Dopamine receptors
E. Topoisomerase inhibitors

6.34 Which type of therapy is not biological therapy:


A. Vaccines
B. Monoclonal antibodies
C. Granulocyte colony stimulating factors
D. Tyrosine kinase inhibitors
E. Checkpoint inhibitors

6.35 Which type of therapy is not targeted therapy:


A. Gene therapy
B. Monoclonal antibodies
C. Granulocyte colony stimulating factors
D. Tyrosine kinase inhibitors
E. Checkpoint inhibitors

6.36 Low-risk prostate cancer is, when:


A. PSA <10 ng/ml, T3, Gleason 7
B. PSA >20 ng/ml, T2a, Gleason ≤6
C. PSA <10 ng/ml, T1-T2a, Gleason ≤6
D. PSA 10-20 ng/ml, T3, Gleason 8-10

6.37 High-risk prostate cancer is, when:


A. PSA >20 ng/ml, T3, Gleason 8-10
B. PSA 10-20 ng/ml, T2b, Gleason 7
C. PSA <10 ng/ml, T1-T2a, Gleason ≤6
D. PSA 10-20 ng/ml, T2a, Gleason ≤6

6.38 Specific cancer treatment might be applied for patients with performance status according ECOG (Eastern
Cooperative Oncology Group):
A. Grade ≥2
B. Grade≤ 2
C. Grade <2
D. Grade >2

6.39 Metastases are considered synchronous if they are diagnosed within the period since the primary tumor was
found:
A. within 2 weeks
B. within 1 month
C. within 2 months
D. within 6 months

6.40 The main tumor marker of colorectal cancer:


A. CEA
B. GI 19-9

lsmusis.lsmuni.lt/Klausimai/Spausdinti?Length=0?Kalba=EN&KategorijaId=108&Kalbos_input=EN&Kalbos=EN&KategorijaEn_input=Onkology&Kate… 4/13
3/27/2019 LSMUSIS
C. CA 125
D. CA 15-3

6.41 Cervical cancer screening reduces:


A. Cervical cancer incidence rate
B. Cervical cancer mortality rate
C. Cervical cancer incidence and mortality rates
D. Does not reduce morbidity or mortality rates

6.42 The most important aetiological factor of cervical cancer is:


A. Herpes 2 virus
B. Epstein barr virus
C. Human papillomavirus
D. AIDS

6.43 The most effective treatment for early prognosis for prostate cancer:
A. External-beam radiotherapy
B. Radical prostatectomy
C. Brachiterapy with permanent implants (I-125 seeds)
D. All these treatments are equally effective

6.44 Treatment of metastatic prostate cancer:


A. Radical prostatectomy and postoperative radiation therapy
B. External-beam rediotherapy
C. Hormonal treatment or chemotherapy
D. Brachytherapy

6.45 Prostate brachytherapy is:


A. Treatment using external-beam radiotherapy
B. Methos of kryotherapy
C. Radiation therapy whem radioactive sources are placed in the prostat
D. Method of hormonal treatment

6.46 The main method of breast cancer screening program is:


A. Breast self-examination
B. Clinical breast examination
C. Mamography
D. Ultrasonography

6.47 Basic method of cervical cancer screening program is:


A. Colposcopy
B. Cytological smear
C. Cervical canal currettage
D. Hysteroscopy

6.48 The following metabolites are released from the tumor into the blood during tumor lysis syndrome:
A. Mg, Co, I
B. Na, Mg
C. C. K, P, uric acid

6.49 Precancerous condition of colon and rectum is:


A. Hemaroid node
B. Fibroadenomatous polyp
C. Inflamation
D. Krono's Disease

6.50 Early symptoms of prostate cancer:


A. Weakness
B. Erectile dysfunction
C. Urinary disorders
D. Early prostate cancer usually does not cause any symptoms

6.51 The most important test for diagnosing prostate cancer is:
lsmusis.lsmuni.lt/Klausimai/Spausdinti?Length=0?Kalba=EN&KategorijaId=108&Kalbos_input=EN&Kalbos=EN&KategorijaEn_input=Onkology&Kate… 5/13
3/27/2019 LSMUSIS
A. Pelvic CT
B. Prostate MRT
C. Digital rectal examination and prostate biopsy
D. Bone scan

6.52 What malignancies are most commonly metastasizing into skeleton:


A. Lymphomas and hematological malignancies
B. Seminoma and ovarian cancer
C. Gastrointestinal malignancies and soft tissue sarcomas
D. Lung, breast and prostate cancer
E. Head and neck cancer and melanoma

6.53 Performing radical surgery for large bowel cancer it is mandatory to remove the following number of regional
lymphnodes:
A. 10-12
B. 8-9
C. 15-16
D. More than 20

6.54 The main methods of treatment of locally advanced colon cancer (ascending, transversal, descending,
sigmoid colon) used in daily clinical practice:
A. Surgery + radiotherapy
B. Surgery + chemotherapy
C. Surgery + chemotherapy + radiotherapy
D. Surgery + biological therapy

6.55 The most common methods used for treatment of locally advanced rectal cancer (stage II and III) are:
A. Surgery alone
B. Surgery + chemotherapy
C. Surgery + chemotherapy + radiotherapy
D. Surgery + biological therapy

6.56 Population groups at greatest risk for colorectal cancer:


A. Men and women at age 25 - 35
B. Men and women at age over 40
C. Young male
D. Female

6.57 The most common histological type of colorectal cancer:


A. Non - small cell cancer
B. Adenocarcinoma
C. Small cell cancer
D. Carcinoid

6.58 The most common adverse event of biological therapy:


A. Myelosuppression
B. Allergic reaction
C. Stomatitis
D. Suppression of the liver function
E. Diarrhea

6.59 Most sensitive tumour to the hormonotherapy:


A. Thyroid cancer
B. Prostate cancer
C. Endometrial cancer
D. Testicular cancer
E. Cancer of the adrenal glands

6.60 The most common adverse event of chemotherapy:


A. Myelodepression
B. Dermatitis allergica
C. Dyspepsia

lsmusis.lsmuni.lt/Klausimai/Spausdinti?Length=0?Kalba=EN&KategorijaId=108&Kalbos_input=EN&Kalbos=EN&KategorijaEn_input=Onkology&Kate… 6/13
3/27/2019 LSMUSIS
D. Suppression of the liver function
E. Suppression of the kidney function

6.61 The type of personalized cancer therapy:


A. Chemotherapy
B. Targeted therapy
C. Biological therapy
D. Hormonotherapy

6.62 Acute emesis and vomiting occurs after initiation of chemotherapy:


A. 1 hour
B. 12 hour
C. 24 hour
D. 72 hour

6.63 Mechanism of action of monoclonal antibodies:


A. Recognising and interaction with extracellular part of one particular protein
B. Recognising and interaction with intracellular part of one particular protein
C. Recognising and interaction with transmembranal part of multiple proteins
D. Recognising and interaction with extracellular part of multiple proteins

6.64 Histologically confirmed metastasis in the neck lymphnode. What primary tumor You will suspect?
A. Breast cancer
B. Lung cancer
C. Gastric cancer
D. Head and neck cancer

6.65 Metastasis to lumbar vertebrae and pelvis are common in case of:
A. Pancreatic cancer
B. Osteosarcoma
C. Prostate cancer
D. Kidney cancer
E. Seminoma

6.66 Prostate cancer usually metastasizes to:


A. Lungs
B. Bones
C. Liver
D. Brain

III type tasks. For each question there is one or more correct answers:
A – if correct answers are 1,2,3
B – if correct answers are 1 and 3
C – if correct answers are 2 and 4
D – if correct answer is 4
E – if correct are all answers above

6.67 Which viruses are associated with the development of cancer:


1. Human immunodeficiency virus (HIV)
2. Human papilloma virus
3. Hepatitis B and C virus
4. Epstein-Barr virus

6.68 Screening applies to the prevention of these malignancies:


1. Gastric cancer
2. Cervical cancer
3. Pancreatic cancer
4. Breast cancer

6.69 Prognostic factors of breast cancer:


1. Stage of the disease
2. Age of the patient
lsmusis.lsmuni.lt/Klausimai/Spausdinti?Length=0?Kalba=EN&KategorijaId=108&Kalbos_input=EN&Kalbos=EN&KategorijaEn_input=Onkology&Kate… 7/13
3/27/2019 LSMUSIS
3. Estrogen and progesteron receptor status
4. Tumor histology

6.70 Which gene mutations increases the risk to have breast cancer from 50 till 80 percent?
1. p53
2. BRCA1
3. HER2
4. BRCA2

6.71 Pharmacological-iatrogenic risk factors for cancer are:


1. Chemotherapeutic drugs
2. Tamoxifen
3. Ionizing radiation
4. Surgical intervention

6.72 Causes of hypercalcemia caused by cancer:


1. Tumor lysis and necrosis
2. Renal insufficiency
3. Osteolytic substances secreted by the tumor
4. Digestive disorders

6.73 The main diagnostic procedures for breast cancer:


1. Mamography
2. Ultrasound
3. Fine- needle aspiration
4. MRI

6.74 High risk breast cancer patients are:


1. With regional lymph node metastases
2. Negative hormonal receptor status
3. Overexpressed HER-2
4. Postmenopausal

6.75 Classification in oncology is needed for:


1. For complex cancer management
2. Cancer prognosis
3. Treatment planning (radical or palliative)
4. Development of oncology science

6.76 Melanoma risk factors:


1. Young age
2. Fair complexion/skin
3. Smoking
4. UV light
5. Lack of physical activity

6.77 Main drugs for cancer pain relief are:


1. Strong opioids and adjuvant drugs
2. Anabolic hormones and corticosteroids
3. NSAID’s and codeine, tramadol
4. Local anesthetics

6.78 Causes of cancer pain can be:


1. Related to tumor
2. Due to specific anticancer treatment
3. Due to debility, caused by cancer and complications
4. Due to concurrent disease

6.79 Surgery for cancer patients is used:


1. Radical operations
2. Palliative operations
3. Surgical treatment of cancer metastases
4. Reconstructive surgery
lsmusis.lsmuni.lt/Klausimai/Spausdinti?Length=0?Kalba=EN&KategorijaId=108&Kalbos_input=EN&Kalbos=EN&KategorijaEn_input=Onkology&Kate… 8/13
3/27/2019 LSMUSIS
6.80 Symptoms of hypercalcemia:
1. Weakness
2. Apathy and mental disorders
3. Atrial or ventricular arrythmia
4. Uremia
5. Icterus

6.81 Diagnostic methods suitable for detection of skeletal metastases:


1. Skeletal scintigraphy
2. Plain x-ray
3. CT scan
4. Ultrasonography

6.82 Melanoma risk increase:


1. Sunbathing in solarium under 35 years of age
2. Excessive sunburns in history
3. Immunosuppression
4. Chronic damaging of existing nevus

6.83 Which histopathological tumour types are associated with the worse prognosis of the patient?
1. Sarcoma
2. Lymphoma
3. Glioblastoma
4. Seminoma

6.84 Symptoms of spinal cord compression:


1. Headache
2. Urinary retention and constipation
3. Dysphagy
4. Sensory loss

6.85 Characteristics of a malignant tumor:


1. Uncontrolled cell division
2. Invasion
3. Potential to spread/metastasize
4. Inhibition of angiogenesis

6.87 Factors for choosing chemotherapy regimen include:


1. Histollogy of the cancer
2. Stage of the cancer
3. Patient’s age
4. Patient’s general state of health
5. Other types of anticancer treatment given in the past

6.88 One year ago patient was operated because of rectal cancer. Rectal adenocarcinoma was diagnosed. Now
during the abdomen CT three metastatic lesions in the right lobe of the liver were found. Which treatment methods
might be used for the treatment of the patient?
1. Chemotherapy
2. Liver operation
3. Molecular-targeted agents
4. Radiation therapy

6.89 Tumor markers:


1. Their value in establishing initial diagnosis has not been clearly established
2. The concentration might be elevated in healthy people (without malignant tumor)
3. Suitable to assess disease progression and treatment effectiveness
4. Are the essential and sufficient criteria for diagnosis (the disease is not necessary to verify
morphologically)

6.90 Which clinical situations should be considered as emergency:


1. Pathologic compressive fracture of vertebra with minimal symptoms of neurological deficit
2. Vertebral metastasis growing into epidural space with minimal symptoms of neurological deficit

lsmusis.lsmuni.lt/Klausimai/Spausdinti?Length=0?Kalba=EN&KategorijaId=108&Kalbos_input=EN&Kalbos=EN&KategorijaEn_input=Onkology&Kate… 9/13
3/27/2019 LSMUSIS
3. Paraplegia and small pelvis organs dysfunction due to metastatic spinal cord compression with duration
of 2 days
4. Paraplegia and small pelvis organs dysfunction due to metastatic spinal cord compression with duration
of 2 weeks
5. Asymptomatic vertebral metastasis when there are multiple skeletal metastases

6.91 What are characteristic features of pain due to spine cord metastases:
1. Pain decreases in laying position
2. Pain increases in laying position
3. Severity of pain not influenced by coughing
4. Pain is more severe at night

6.92 In which clinical situation bone scan is indicated:


1. After diagnosis of early breast cancer
2. After diagnosis of prostate cancer with PSA level 100 ng/ml
3. After diagnosis of lung cancer with brain metastases when patient has no skeleton pain
4. After diagnosis of any malignancy with potency to metastasize when patient has back pain
5. After diagnosis of brain tumor with hemiparesis

6.93 cTNM is:


1. Based solely on evidence gathered before initial treatment of the primary tumor – i.e. clinical examination,
imaging studies, and pathologic examination of tumour tissue obtained by biopsy
2. Used to make local/regional treatment recommendations
3. Clinical classification
4. Established by pathologist

6.94 pTNM is:


1. Used to assess prognosis and make recommendations for adjuvant treatment;
2. Incorporates the results of clinical staging with evidence obtained from surgery and from detailed
pathological examination of the primary tumor
3. Pathological classification
4. Established by pathologist

6.95 Symptoms of intracranial pressure:


1. Arterial hypertension
2. Headache
3. Nystagmus
4. Nausea, vomiting
5. Paresis

6.96 Risk factors for colorectal carcinoma:


1. Age over 40
2. Sedementary lifestyle
3. Wrong nutrition habits
4. Obesity

IV type tasks. Choose only one best answer

6.98 68 years old man complains about weak urine stream, frequent urination during the night. The result of
prostate-specific antigen (PSA) was – 12 ng/ml, during digital rectal examination in the right lobe of the prostate
was found a rigid and rough tumor. What will be your further examination:
A. Diagnosis is already established.
B. he computer tomography or magnetic resonance tomography
C. Tumor biopsy
D. Tumor ultrasound

6.99 76 years man complains of bone pain, especialli in the lumbar area and in the right site of pelvis. Before 3
years he was treated for the prostate cancer by radical radiotherapy. What is the most informative method to
detect the bone metastases:
A. Bone scintigraphy
B. Prostate-specific antigen (PSA)
C. Sceletal survey

lsmusis.lsmuni.lt/Klausimai/Spausdinti?Length=0?Kalba=EN&KategorijaId=108&Kalbos_input=EN&Kalbos=EN&KategorijaEn_input=Onkology&Kat… 10/13
3/27/2019 LSMUSIS
D. Osteodensitometry

6.100 67 years old man underwent examination in the ambulatory. The tests showed prostate-specific antigen
(PSA)- 7 ug/ml, during digital rectal examination the left lobe of the prostate was more rigid after biopsy the
carcinoma of prostate was established. The clinical diagnosis is Adenocarcinoma of prostate cT1cNxM0, Gleason
6. What will be the treatment:
A. Follow up or radical prostectomy or radiotherapy.
B. Radiotherapy or hormone therapy or radical prostectomy.
C. Chemotherapy or radiotherapy or radical prostectomy.
D. Hormone therapy or radical prostectomy.

6.101 63 years man was treated for small cell lung carcinoma by radiotherapy and chemotherapy. After 6 months
of this treatment the patient was admitted to the hospital because dyspnoea and cyanosis. Patient examination
showed the oedema of the neck and face, dilatation of the thoracic veins. The preliminary diagnosis is:
A. Pulmonary embolism
B. Superior vena cava compression syndrome
C. Metastasis of the lung cancer to the neck and thoracic lymph nodes.
D. Tumor lysis syndrome.

6.102 Patient of 70 years old was treated for the right kidney cancer 2 years ago. The patient mentioned that
suddenly appear leg numbness, weakness, sensory disorder. Actions for suspected spinal cord compression:
A. Computer tomography, neurological examination and steroids.
B. Neurological examination, steroids and magnetic resonance tomography
C. Analgesics, magnetic resonance tomography and steroids.
D. Neurological examination, computer tomography and analgesics.

6.103 52 years woman who was treated with chemotherapy for advanced breast carcinoma cT3N2M1 with the
multiple metastasis in the bone. After two weeks patient begin to complain about the nausea, diarrhea, severe
muscles weakness, fatigue, bone pain. What will be the preliminary diagnosis:
A. Hyponatremia
B. Hyperuricemia
C. Hypercalcaemia
D. Progress of the breast carcinoma

6.104 58 years old, smoking gentlemen complains in cough, lasting about 6 months. Chest x-ray and CT scan
showed peripheral lung tumor, 4 cm in diameter, and enlarged mediastinal lymphnodes.What further investigations
would you perform:
A. Diagnosis is clear
B. MRI
C. Tumor biopsy

6.105 A 62 year-old man was treated with chemotherapy for prostate carcinoma spread to bones using docetaxel.
In 6 days after the 2nd chemotherapy course, the patient had an elevated temperature up to 38,5 ⁰C. He arrived to
emergency care department, where after performing BKT Neu-0,3X109/l CRB-100 mg/l. What was the patient‘s
diagnosis?
A. Febrile neutropenia after chemotherapy
B. Viral infection,
C. Prostatitis

6.106 A 58 year-old man was tested for dyspepsia events. Gastroscopy: in the great stomach curve, a visible
ulceration is present, about 1.5 inch in size. Biopsy was taken: poorly differentiated, circular cell carcinoma G3. To
assess the spread of cancer, the thoracic, abdominal and pelvic organ CT was performed: the stomach wall at a
large curve thickened up to 0.9 cm, the tumor infiltration of the musculiaris propria layer is present, lymphnodes
are not involved, distant metastases are not visible (radiologically cT2N0M0). What is the further treatment tactics?
A. Surgical treatment
B. Neoadjuvant chemotherapy, after that surgical treatment and adjuvant chemotherapy
C. Chemotherapy

6.107 A 64 year-old woman with good performance status was diagnosed with a colon ascendentis carcinoma. A
right haemicolectomy was performed. Histological evaluation: adenocarcinoma pT3N1. MTS not determined in
radiological study, adjuvant chemotherapy was performed. At the present time, after an ultrasound procedure, a

lsmusis.lsmuni.lt/Klausimai/Spausdinti?Length=0?Kalba=EN&KategorijaId=108&Kalbos_input=EN&Kalbos=EN&KategorijaEn_input=Onkology&Kat… 11/13
3/27/2019 LSMUSIS
formation was observed in the liver S3 segment; chest, abdomen and pelvic CT was performed: in the liver S3
segment, 3 cm MTS was found. Colonoscopy: no pathology was determined. What is the further treatment tactics?
A. Neoadjuvant treatment, after that, surgical treatment and adjuvant chemotherapy
B. Symptomatic treatment
C. Chemotherapy+- targeted therapy

6.108 52 years old postmenopausal woman noticed a lump in the left breast. On palpation there was a 25 x 25 mm
large tumor with fixation to the skin. No ”peau-d’orange” symptom. In the left axilla - 1 enlarged firm node. What
investigations you will perform primarily:
A. Biopsy of the lymphnode
B. Mammography and breast tumor core needle biopsy
C. Breast ultrasonoscopy
D. Chest x - ray

6.109 52 years old postmenopausal woman noticed a lump in the left breast. On palpation there was a 25 x 25 mm
large tumor with fixation to the skin. No ”peau-d’orange” symptom. In the left axilla - 1 enlarged firm node. Clinical
TNM:
A. T1 N1 M0
B. T2 N1 MX
C. T3 N1 MX
D. T4 N1 MX

6.110 56 years old lady 3 weeks ago noticed lump in her right breast. On exam: 2,5 x 2,0 cm lump in the upper
external quadrant of the right breast, no detectable axillary lymph nodes. Mammography: 20 x 15 mm tumor with
microcalcifications. Breast ultrasound: 18x13 mm tumor, normal axillary lymph nodes. CNB confirmed invasive
ductal carcinoma with positive estrogen and progesterone receptor, HER2(0). Pro-posed type of surgery:
A. Mastectomy
B. Breast conserving surgery
C. Breast conserving surgery and sentinel node biopsy

6.111 50 years old lady with 2 month history of severe pain between shoulder blades. Last week dyspnoea, while
coming upstairs, occurred. On exam: 3 x 3,5 lump in the left breast, enlarged axillar lymph nodes. Examination
plan:
A. Mammography and core needle biopsy
B. Mammography, core needle biopsy, bone scintigraphy and x - ray or CT, chest x – ray or CT
C. . Chest x-ray and x-ray of shoulder blades or CT
D. ECG, cardioechoscopy and other investigations of cardiovascular system

6.112 67 years old gentlemen was complains on rectal bleeding and mucos discharge. He lost about 10kg of
weight during last few months. He also has pain in perineal area.Your preliminary diagnosis:
A. Haemorroids
B. Colorectal cancer
C. Prostate cancer
D. Crohn disease

6.113 67 years old gentlemen was complains on rectal bleeding and mucos discharge. He lost about 10kg of
weight during last few months. He also has pain in perineal area.The most informative examination method:
A. Digital rectal examination
B. Blood test in the stool
C. Colonoscopy
D. PSA
E. Abdominal ultrasonography

6.114 60 year old man was diagnosed having stage T3N1M1 small-cell lung cancer. Patient was treated with
chemotherapy. Six months later head and neck edema and dilatation of neck veins occurred. Preliminary
diagnosis:
A. Brain metastases
B. Neck lymphnodes metastases
C. Superior vena cava syndrome
D. Hypercalcemia

lsmusis.lsmuni.lt/Klausimai/Spausdinti?Length=0?Kalba=EN&KategorijaId=108&Kalbos_input=EN&Kalbos=EN&KategorijaEn_input=Onkology&Kat… 12/13
3/27/2019 LSMUSIS
6.115 A 64 year-old male was diagnosed with hepatitis C. In abdominal CT: in the liver S6 segment a single 2 cm
node was identified with a typical contrast accumulation for hepatocellular carcinoma. No systemic tumour spread
was observed in radiological studies. Which of the following criteria will determine the choice of the further
treatment tactics:
A. Liver insufficiency according Child-Pugh criteria
B. Alpha-fetoprotein level
C. ALT, AST level

6.116 57 years old lady was diagnosed having left breast tumor, 4.5x3.8 cm in diameter, with positive axillary
lymph nodes (T2 N1 M0), estrogen and progesterone receptor negative, HER2(3+). You propose start complex
treatment with:
A. Breast conserving surgery
B. Mactectomy
C. Neoadjuvant chemotherapy, biological therapy
D. Preoperative hypofractionated radiotherapy

6.1 - B 6.2 - B 6.3 - B 6.4 - A 6.5 - A 6.6 - D 6.7 - C 6.8 - B


6.9 - E 6.10 - B 6.11 - B 6.12 - B 6.13 - A 6.14 - A 6.15 - B 6.16 - B
6.17 - A 6.18 - D 6.19 - B 6.20 - C 6.21 - A 6.22 - B 6.23 - A 6.24 - C
6.25 - B 6.26 - C 6.27 - C 6.28 - B 6.29 - C 6.30 - A 6.31 - C 6.32 - C
6.33 - E 6.34 - D 6.35 - C 6.36 - C 6.37 - A 6.38 - B 6.39 - C 6.40 - A
6.41 - C 6.42 - C 6.43 - D 6.44 - C 6.45 - C 6.46 - C 6.47 - B 6.48 - C
6.49 - B 6.50 - D 6.51 - C 6.52 - D 6.53 - A 6.54 - B 6.55 - C 6.56 - B
6.57 - B 6.58 - B 6.59 - B 6.60 - A 6.61 - B 6.62 - C 6.63 - A 6.64 - D
6.65 - C 6.66 - B
6.67 6.68 6.69 6.70 6.71 6.72 6.73 6.74
1 2 1 2 1 1 1 1
2 4 2 4 2 3 2 2
3 3 3 3 3
4 4 4
6.75 6.76 6.77 6.78 6.79 6.80 6.81 6.82
1 2 1 1 1 1 1 1
2 4 3 2 2 2 2 2
3 3 3 3 3 3
4 4 4 4
6.83 6.84 6.85 6.87 6.88 6.89 6.90 6.91
1 2 1 1 1 1 1 2
3 4 2 2 2 2 2 4
3 3 3 3 3
4
5
6.92 6.93 6.94 6.95 6.96
2 1 1 2 1
4 2 2 4 2
3 3 3
4 4
6.98 - C 6.99 - A 6.100 - A 6.101 - B 6.102 - B 6.103 - C 6.104 - C 6.105 - A
6.106 - B 6.107 - A 6.108 - B 6.109 - B 6.110 - C 6.111 - B 6.112 - B 6.113 - C
6.114 - C 6.115 - A 6.116 - C

lsmusis.lsmuni.lt/Klausimai/Spausdinti?Length=0?Kalba=EN&KategorijaId=108&Kalbos_input=EN&Kalbos=EN&KategorijaEn_input=Onkology&Kat… 13/13
3/27/2019 LSMUSIS

Endocrinology
I type tasks. Choose only one best answer

30.1 Which sign of Graves' disease is typical :


A. Heart rate reduction
B. Galactorrhea
C. Hirsutism
D. Pretibial myxedema
E. Abdominal obesity

30.2 Which sign of endocrine ophthalmopathy is typical:


A. Chvostek's
B. Trousseau's
C. Graefe's
D. Courvoisier's
E. Kernig's

30.3 What type of cardiac arrhythmia is characteristic for toxic goiter:


A. I degree atrioventricular block
B. Left bundle branch block
C. Wolff-Parkinson-White syndrome
D. Atrioventricular rhythm
E. Atrial fibrillation

30.4 What maintenance daily dose of thiamazole (methizole) should be administered for treatment of
thyrotoxicosis:
A. 30-40 mg/d
B. 25 mg/d
C. 20 mg/d
D. 15 mg/d
E. 5-10 mg/d

30.5 What complications may be caused by treatment of beta-blockers:


A. Tachycardia
B. Vomiting
C. Diarrhoea
D. Dyspnoea
E. Thrombophlebitis

30.6 Which adverse event is not characteristic for thiamazole (methizole) treatment:
A. Leucopenia
B. Leucocytosis
C. Nausea
D. Diarrhoea
E. Right side pain

30.7 Which medication should be used before surgical treatment of toxic goiter:
A. Metformin
B. Thiamazole
C. Fludrocortisone
D. Hydrocortisone
E. Gliclazide

30.8 Which sign is not characteristic for phaeochromocytoma:


A. Tachycardia
B. Hypertension
C. Atrial fibrillation
D. Exophthalmos
E. Dizziness

30.9 Which sign is characteristic for phaeochromocytoma:

lsmusis.lsmuni.lt/Klausimai/Spausdinti?Length=0?Kalba=EN&KategorijaId=132&Kalbos_input=EN&Kalbos=EN&KategorijaEn_input=Endocrinology&… 1/19
3/27/2019 LSMUSIS
A. Arterial hypertension
B. Dry skin
C. Weight gain
D. Thrombophlebitis
E. Hypoglycaemia

30.10 Which test is not characteristic for phaeochromocytoma:


A. Hyperglycaemia
B. Increased urinary epinephrine excretion rate
C. Increased urinary norepinephrine excretion rate
D. Hypoglycaemia
E. Increased urinary vanillylmandelic acid excretion rate

30.11 Which symptom is not characteristic for Addison’s disease:


A. Weakness
B. Hyperpigmentation
C. Nausea
D. Tachycardia
E. Galactorrhoea

30.12 Which symptom is characteristic for Addison’s disease:


A. Obesity
B. Muscle cramps
C. Hypertension
D. Hypoglycaemia
E. Hypercholesterolemia

30.13 Which symptom is characteristic for hypoglycaemia:


A. Thirst
B. Dry skin
C. Perspiration
D. Muscle pain
E. Arterial hypertension

30.14 Which symptom is not characteristic for ketoacidosis:


A. Nausea
B. Abdominal pain
C. Vomiting
D. Clonic muscle cramps
E. Dry mucosa and skin

30.15 Which clinical state is an indication for insulin therapy:


A. Impaired carbohydrate metabolism
B. Type 2 diabetes mellitus with initial signs of complications
C. Type 2 diabetes without complications
D. Pregnant patient with type 2 DM

30.16 How fast do the clinical symptoms of hypoglycaemic coma progress:


A. Months
B. Weeks
C. Days
D. Hours
E. Minutes

30.17 What symptoms are characteristic for hypoglycaemic state:


A. Dehydration, drowsiness
B. Hunger, weakness, sweating, hand tremor
C. Increased muscle tone, dry skin
D. Kussmaul’s breathing, acetone odor to the breath
E. Drowsiness, abdominal pain

30.18 What symptoms are characteristic for diabetic ketoacidotic precoma:

lsmusis.lsmuni.lt/Klausimai/Spausdinti?Length=0?Kalba=EN&KategorijaId=132&Kalbos_input=EN&Kalbos=EN&KategorijaEn_input=Endocrinology&… 2/19
3/27/2019 LSMUSIS
A. Adynamia, skin hyperpigmentation, nausea, impaired consciousness
B. Arterial hypertension, hypokalaemia, hypernatraemia
C. Deep, noisy, rhythmic respiration (breathing) and dehydration
D. Sweating, hand tremor
E. Slowness, hypothermia,. apathy, hypodynamia

30.19 Which symptom is not characteristic for Addison’s disease:


A. Weakness
B. Hypotension
C. Diarrhoea
D. Hypertension
E. Skin hyperpigmentation

30.20 Which test is not characterstic for Addison’s disease:


A. Hypoglycaemia
B. Hyperkalaemia
C. Hypochloraemia
D. Decreased 24h urinary free cortisol excretion
E. Decreased 24h urinary catecholamines

30.21 Which symptom is characteristic for Cushing’s syndrome:


A. Hypotension
B. Diarrhoea
C. Exophthalmos
D. Hypoglycaemia
E. Weight gain

30.22 Which symptom is not characteristic for hypopituitarism:


A. Cachexia
B. Normal axillary and pubic hair
C. Adrenal insufficiency
D. Hypothyroidism
E. Amenorrhoea

30.23 Which test result is not characteristic for Cushing’s syndrome:


A. Hyperglycaemia
B. Hypernatraemia
C. Hypokalaemia
D. Polycitaemia
E. Decreased 24h urinary free cortisol excretion

30.24 Which drug is the first line drug in treatment of hypopituitarism:


A. Hydrocortisone
B. Levothyroxine
C. Testosterone
D. Norepinephrine
E. Mesatone

30.25 Which statement is not characteristic for hypoglycaemic coma:


A. Rapid onset
B. Hunger, tachycardia, dwelling in eyes before onset
C. Deep and loud respiration
D. Cold sweating and increased salivation
E. Seizures

30.26 Which factor plays the main role in the development of diabetic ketoacidotic coma:
A. Impaired glucose utilization in body tissues
B. Increased activity of glyconeogenesis
C. Increased secretion of STH, catecholamines and glucocorticoids
D. Lack of insulin
E. Increased lipolysis

lsmusis.lsmuni.lt/Klausimai/Spausdinti?Length=0?Kalba=EN&KategorijaId=132&Kalbos_input=EN&Kalbos=EN&KategorijaEn_input=Endocrinology&… 3/19
3/27/2019 LSMUSIS
30.27 Which sign is characteristic for hyperosmolar coma:
A. Kussmaul’s breathing
B. Ketonuria
C. Hypernatraemia
D. Normal blood glocose level
E. Oedema

30.28 Which symptom is suspectable for hypothyroidism:


A. Paroxysmally elevated arterial blood pressure
B. Fever
C. Bradycardia
D. Paroxysmal tachycardia

30.29 Which symptom is characteristic for hypothyroidism:


A. Paroxysmal hypertension
B. Weight loss
C. Nausea
D. Constipation

30.30 Which blood test is the most useful for diagnosis of hypothyroidism:
A. Blood prolactin concentration
B. ACTH concentration
C. TSH concentration
D. Growth hormone level

30.31 What disease hypothyroidism should be differentiated with:


A. Primary hypertension
B. Ectopic Cushing syndrome
C. Chronic nephritis
D. Vegetodystonia

30.32 Which drug would you prefer for treatment of hypothyroidism:


A. Amiodarone
B. Metoprolol
C. Levothyroxine
D. Thiamazole

30.33 Inadequate treatment of hypothyroidism can result in:


A. Cushing syndrome
B. Acute adrenal insufficiency
C. Myxedema coma
D. Pneumonia

30.34 Which hormone secretion is decreased in hypothyroidism:


A. Cortisol
B. Adrenaline
C. Thyroxine and tri-iodothyronine
D. Noradrenaline

30.35 Who was (were) the first to extract insulin from dog’s pancreas:
A. Eli Lilly
B. Leonard Thompson
C. Frederic Banting and Charles Best
D. Margerie
E. F. Sanger

30.36 Which type of insulin therapy should not be recommended:


A. One injection of long-acting insulin (24 hrs)
B. One injection of long-acting insulin and 3-5 injections of rapid-acting insulin
C. 2 injections of premixed insulin [before breakfast and supper (dinner)]
D. 2 injections of premixed insulin [before breakfast and supper (dinner)] and 1 injection of rapid-acting
insulin before lunch

lsmusis.lsmuni.lt/Klausimai/Spausdinti?Length=0?Kalba=EN&KategorijaId=132&Kalbos_input=EN&Kalbos=EN&KategorijaEn_input=Endocrinology&… 4/19
3/27/2019 LSMUSIS
30.37 What is the initial symptom of diabetic polyneuropathy:
A. Walking leg pain
B. Cold feet
C. Numbness of legs
D. Trophic ulcers in feet
E. Leg muscle atrophy

30.38 What are the initial complaints in developing of diabetic retinopathy:


A. Eye twinkling
B. No complaints
C. Decreased visual acquity
D. Dwelling in eyes
E. Occurrence of “Flying dots”

30.39 Which symptom is not common for hypoglycaemic state:


A. Hunger
B. Tremulousness
C. Dizziness
D. Sweating
E. “Red” face

30.40 Which symptom is not common for diabetic ketoacidosis:


A. Weakness
B. Polydypsia
C. Polyuria
D. Acetone odor
E. Hunger

30.41 What is the first line medication in treatment of hypoglycaemia:


A. 60 ml 40% glucose (dextrose) i/v
B. 500 ml 5% glucose infusion
C. Insulin s/c
D. 10 ml 25% Mg SO4 i/v
E. Hydrocortisone 150 mg i/m

30.42 What is characteristic for the skin of patient in hypoglycaemic coma:


A. Pale and dry
B. Reddish
C. Pale and sweating
D. Dry with the marks of scratching and haemorrhages
E. Cyanotic and dry

30.43 Which symptom is not characteristic for myxedema coma:


A. Sleepiness
B. Hypodynamia
C. Hypothermia
D. Tachycardia
E. Hypotension

30.44 Which symptom is characteristic for hypoparathyroidism:


A. Trousseau’s sign
B. Graefe’s sign
C. Kocher’s sign
D. Dalrimple’s sign
E. Stelvag’s sign

II type tasks. For each numbered item,selct the one lettered heading that is most closely asssciated with it

30.45 What blood test results are characteristic for the following diseases:
1 - Hyperparathyroidism
2 - Hypoparathyroidism
3 - Hypercorticism

lsmusis.lsmuni.lt/Klausimai/Spausdinti?Length=0?Kalba=EN&KategorijaId=132&Kalbos_input=EN&Kalbos=EN&KategorijaEn_input=Endocrinology&… 5/19
3/27/2019 LSMUSIS
4 - Chronic adrenal insufficiency
A. Hypocalcaemia and hyperphosphataemia
B. Hyperglycaemia
C. Hypoglycaemia
D. Hypercalcaemia and hypophosphataemia

30.46 What blood test changes are characteristic for the following conditions:
B 1 - Addisonic coma
A 2 - Uraemic coma
D 3 - Myxoedema coma
C 4 - Hypoparathyroidism
A. Azotaemia, hyperkalaemia, anaemia
B. Hyponatraemia, hyperkalaemia, hypoglycaemia
C. Hypocalcaemia, hyperphosphataemia
D. Hypercholesterolaemia, anaemia

30.47 Which test results are characteristic for clinical conditions:


D 1 - Diabetic ketoacidosis
C 2 - Chronic renal failure
B 3 - Chronic adrenal insufficiency
A 4 - Phaeochromocytoma
A. Increased urine vanilylmandelic acid excretion
B. Hypoglycaemia
C. Hyperkalaemia
D. Hypokalaemia

30.48 Which is the most characteristic etiological factor of these diseases:


1 - Addison’s disease
2 - Phaeochromocytoma
3 - Diffuse toxic goiter (Grave’s disease)
4 - Sheehan’s syndrome
A. Autoimmune process
B. Tumor growth
C. Postpartum haemorrhage

30.49 What blood tests result is characteristic for the following diseases:
1 - Hypothyroidism
2 - Toxic goiter
3 - Diabetes mellitus
4 - Ectopic Cushing’s syndrome
A. Hypercholesterolaemia
B. Hypocholesterolaemia
C. Hyperglycaemia
D. Increased blood cortisol level

30.50 Which blood tests result is characteristic for the following diseases:
1 - Phaeohromocytoma
2 - Toxic goiter
3 - Diabetes mellitus
4 - Addison’s disease
A. Increased urinary catecholamine excretion
B. Hyperglycaemia
C. Increased blood FT4 concentration
D. Hypoglycaemia

30.51 What laboratory tests result is characteristic for the following diseases:
1 - Diabetes mellitus
2 - Toxic goiter
3 - Addison’s disease
4 - Phaeochromocytoma
A. Ketonuria
B. Hypocholesterolaemia
lsmusis.lsmuni.lt/Klausimai/Spausdinti?Length=0?Kalba=EN&KategorijaId=132&Kalbos_input=EN&Kalbos=EN&KategorijaEn_input=Endocrinology&… 6/19
3/27/2019 LSMUSIS
C. Hypoglycaemia
D. Increased urine catecholamine excretion

30.52 Which complaint is characteristic for the following diseases:


1 - Hypoparathyroidism
2 - Hypothyroidism
3 - Toxic goiter
4 - Addison’s disease
A. Body oedema
B. Weight loss in good appetite
C. Marked general weakness
D. Hand and leg cramps

30.53 Which pathogenetic mechanism is characteristic for these diseases:


1 - Toxic goiter
2 - Phaeochromocytoma
3 - Addison’s disease
4 - Acute adrenal insufficiency
A. Hypoglycaemia due to decreased glucocorticoid secretion
B. Hyperglycaemia due to increased epinephrine and norepinephrine secretion
C. Tachycardia due to increased sensitivity of myocardium to catecholamines

30.54 Which symptoms are characteristic for the following diabetic complications:
1 - Polyneuropathy
2 - Nephropathy (IV stage)
3 - Damage of leg's large blood vessels
4 - Ketoacidosis
A. Nausea and vomiting
B. Oedema and hypertension
C. Leg numbness and coldness
D. Claudicatio intermittens

30.55 Which symptom is characteristic for each disease:


1 - Toxic goiter
2 - Phaeochromocytoma
3 - Addison’s disease
4 - Acute adrenal insufficiency
A. Hypotension
B. Vitiligo
C. Systolic and diastolic hypertension
D. Positive Graefe’s sign

30.56 Which symptom is characteristic for each indicated disease or condition:


D 1 - Hypoglycaemic coma
A 2 - Diabetic ketoacidotic coma
C 3 - Addison’s disease
B 4 - Primary hyperaldosteronism
A. Kussmaul’s breathing
B. Hypernatraemia and hypokalaemia
C. Hyponatraemia and hyperkalaemia
D. Sweating and hand tremor

30.57 Which is the most characteristic symptom for each indicated disease:
1 - Hypothyroidism
2 - Toxic goiter
3 - Diabetes mellitus
4 - Phaeochromocytoma
A. Bradycardia
B. Thirst
C. Tachycardia
D. Paroxysmal hypertension

lsmusis.lsmuni.lt/Klausimai/Spausdinti?Length=0?Kalba=EN&KategorijaId=132&Kalbos_input=EN&Kalbos=EN&KategorijaEn_input=Endocrinology&… 7/19
3/27/2019 LSMUSIS
30.58 Which test result is characteristic for each indicated disease:
1 - Toxic goiter
2 - Phaeochromocytoma
3 - Addison’s disease
4 - Acute adrenal insufficiency
A. Hyponatraemia and hyperkalaemia
B. Increased thyroid peroxidase antibodies concentration
C. Leucocytosis

30.59 Which medication should be administered for treatment of the following diseases:
1 - Toxic goiter
2 - Phaeochromocytoma
3 - Hypopituitarism
4 - Addison’s disease
A. Doxasosin
B. Thiamazole
C. Hydrocortisone

30.60 What medication should be administered for treatment of each indicated disease:
1 - Hypothyroidism
2 - Toxic goiter
3 - Diabetes mellitus
4 - Addison’s disease
A. Levothyroxine
B. Thiamazole
C. Metformin
D. Hydrocortisone

30.61 What medication should be used for treatment of each indicated disease:
1 - Toxic goiter
2 - Phaeochromocytoma
3 - Addison’s disease
4 - Acute adrenal insufficiency
A. Thiamazole
B. Prazosin
C. Hydrocortisone

30.112 For what diseases are specific these hormone changes:


- Increased FT4, FT3; decreased TSH
- Decreased FT4, FT3; increased TSH
- Normal FT4, FT3; decreased TSH
- Normal FT4, FT3; increased TSH
A. Thyrotoxicosis
B. Subclinical thyrotoxicosis
C. Hypothyrosis
D. Subclinical hypothyrosis

30.113 For what disease are specific scintigraphic images of thyroid gland:
- Diffusely accumulate radiolabelled
- Radiolabelled accumulate in one node, no accumulation in the rest of the thyroid tissue
- Bo radiolabelled accumulation in the thyroid gland
- Radiolabelled accumulate more in multiple nodes, the rest of the tissue accumulate less
A. Multinodular toxic goitre
B. Toxic adenoma
C. Grave's disease
D. Subacute thyroiditis

30.124 Classify described thyroid examination data according to WHO:


- Goitre is palpable and visible in supine head position or with nodes, even if thyroid is not enlarged
- Goitre is visible from distance
- Goitre is palpable and visible in normal head position
- Goitre is palpable but invisible
lsmusis.lsmuni.lt/Klausimai/Spausdinti?Length=0?Kalba=EN&KategorijaId=132&Kalbos_input=EN&Kalbos=EN&KategorijaEn_input=Endocrinology&… 8/19
3/27/2019 LSMUSIS
A. Thyroid enlargement I A
B. Thyroid enlargement I B
C. Thyroid enlargement II
D. Thyroid enlargement III

30.125 Classify described changes according to clincal conditions:


- Insufficient mineralisation of mature bone, decreased calciferol concentration in blood
- Insufficient mineralisation of growing bone, decreased calciferol concentration in blood
- Hypocalcaemia, decreased PTH
- Hypercalcaemia, increased PTH, decreased phosphorus concentration in blood
A. Primary hyperparathyroidism
B. Hypoparathyroidism
C. Rachitis
D. Osteomalacia

30.127 Which hormones produce each of listed active pituitary adenomas:


B - Prolactinoma
D - Somatotrophinoma
A - Thyrotrophinoma
C - Gonadotrophinoma
A. TSH
B. PRL
C. LH, FSH
D. Growth hormone

30.130 For what diseases are specific these clinical expressions:


- Paroxysmal hypertension
- Hypokalaemia and hypertension
- Hyperkalaemia and hypotension
- Tachycardia and hypertension
A. Conn's syndrome
B. Addison's disease
C. Grave's disease
D. Phaeochromocytoma

30.131 Please, indicate possible localisation of lesions that cause these diseases:
- Phaeochromocytoma
- Conn's syndrome
- Cushing's syndrome
- Hyperandrogenism
A. Adrenal's medulla
B. Zona reticularis
C. Zona glomerulosa
D. Zona fasciculata

30.132 Please, indicate treatment sequence of diabetic ketoacidosis:


- Correction of glycaemia by rapid action insulin
- Correction of dehidratation
- Correction of the potassium in the blood
- Correction of acid alkaline balance
A. In the first place
B. Second place
C. Third place
D. Fourth place

30.133 What are the tests (by sequence)performed to assess kidney lesion and function in patient with diabetes
mellitus:
- Creatinine
- Albumin level in the blood
- Albumin level in urine
- Glomerular filtration rate
A. First
lsmusis.lsmuni.lt/Klausimai/Spausdinti?Length=0?Kalba=EN&KategorijaId=132&Kalbos_input=EN&Kalbos=EN&KategorijaEn_input=Endocrinology&… 9/19
3/27/2019 LSMUSIS
B. Second
C. Third
D. Fourth

30.138 For which neuroendocrine obesity type are specific listed descriptions of appearance:
- Equal obesity of all body with fatty "bracelets" above wrist and tarsus
- Truncal obesity with supraclavicular fat pads, round face, buffalo hump, thin limbs
- Fat accumulation in breast, pelvic, nates area, eunuchoidic proportions
- Obesity of all body, fat accumulations in few wrinkles in submental, breast, stomach, limbs area
A. Hypothalamic obesity
B. Hypothyrotic obesity
C. Cushing's obesity
D. Hypogonadal obesity

III type tasks. For each question there is one or more correct answers:
A – if correct answers are 1,2,3
B – if correct answers are 1 and 3
C – if correct answers are 2 and 4
D – if correct answer is 4
E – if correct are all answers above

30.62 Etiogical factors of hypothyroidism are:


1. Autoimmune thyroiditis
2. Thyroidectomy
3. Iodine deficiency
4. Antithyroid treatment

30.63 Symptoms characteristic for toxic goitre:


1. Graefe’s sign
2. Stelvag’s sign
3. Tachycardia
4. Purple striae

30.64 Acute adrenal insufficiency is caused by:


1. Autoimmune damage of adrenal cortex
2. Idiopathic atrophy of adrenal cortex
3. Adrenal amyloidosis
4. Haemorrhage into adrenals during meningococcal sepsis

30.65 Indications for insulin treatment in 2 type diabetes mellitus


1. Ineffective treatment with max doses of sulfonylurea and biguanides
2. Pregnancy
3. Progressive late diabetes complications
4. Joint pain

30.66 Symptoms and signs characteristic for diabetic ketoacidotic coma:


1. Kusssmaul’s breathing
2. Dehydration, dry skin
3. Acetone odor to the breath
4. Sweating, hand tremor

30.67 First line treatment of the acute adrenal insufficiency:


1. Hydrocortisone
2. Fludrocortisone
3. Glucose and electrolyte solutions
4. Levothyroxine

30.68 Treatment of diabetic ketoacidotic coma (pH=7,0) should be started with:


1. Long-acting insulin 16-20 IU i/m
2. Rapid-acting insulin – i/v infusion
3. 2,5% sodium bicarbonate 100ml infusion i/v
4. Isotonic saline 0,9% 1000 ml/hr infusion i/v
lsmusis.lsmuni.lt/Klausimai/Spausdinti?Length=0?Kalba=EN&KategorijaId=132&Kalbos_input=EN&Kalbos=EN&KategorijaEn_input=Endocrinology… 10/19
3/27/2019 LSMUSIS
30.69 Hypothyroidism is caused by:
1. Radioactive iodine treatment
2. Levothyroxine treatment
3. Thyroid surgery
4. Prednisolone treatment

30.70 Symptoms and signs characteristic for hypothyroidism:


1. Tachycardia
2. Exophthalmos
3. Subfebrile temperature
4. Bradycardia

30.71 Symptoms and signs characteristic for diabetes mellitus:


1. Polydypsia
2. Polyuria
3. Hyperglycaemia
4. Hyperkalaemia

30.72 Diabetes mellitus directly can be caused by:


1. Pneumonia
2. Adrenal haemorrhage
3. Hyperplasia of thyroid cells
4. Lymphocytic infiltration of islets of Langerhans

30.73 Symptoms and signs of hypoglycaemia are:


1. Disorientation
2. Dwelling in eyes
3. Hand tremor
4. Sweating

30.111 What hormone changes are specific in thyrotoxicosis:


1. Decreased FT4, FT3, TSH
2. Decreased FT4, FT3; increased TSH
3. Increased FT4, FT3, TSH
4. Increased FT4, FT3; decreased TSH

30.114 When is recommended radioiodine treatment for thyrotoxicosis:


1. In case of retrosternal goitre with pressure into throat organs
2. In case of toxic adenoma
3. In case of endocrine ophthalmopathy and big goitre
4. Reccurence thyrotoxicosis after treatment till 2 years with antithyroid drugs when thyroid gland is small
and without nodes

30.115 What are the main mechanisms of biguanides (metformin)action:


1. Reduces hepatic glucose production by inhibiting gluconeogenesis and glycogenolysis
2. Increases insulin sensitivity, improves glucose uptake and utilization in peripheral tissues
3. Slows down glucose absorption from the gut
4. Increases insulin secretion

30.116 What are the contraindications to sulfonylureas treatment:


1. Pregnancy and lactation
2. Acute conditions (infection, trauma, surgery)
3. Severe renal and hepatic failure
4. Poor glycemic control

30.118 Indications for temporary insulin therapy in type 2 diabetes:


1. Pregnancy and lactation
2. Ketoacidosis
3. Perioperative treatment
4. Diabetic foot syndrome

30.117 What are mechanisms of GLP-1 action:


1. Stimulates insulin secretion in glucose-dependent manner
lsmusis.lsmuni.lt/Klausimai/Spausdinti?Length=0?Kalba=EN&KategorijaId=132&Kalbos_input=EN&Kalbos=EN&KategorijaEn_input=Endocrinology… 11/19
3/27/2019 LSMUSIS
2. Through apetite and hunger centers in the hypothalamus increases satiety, reduces hunger
3. Slows down food evacuation from stomach
4. Stimulates insulin secretion regardless of the glucose concentration

30.119 What are components of metabolic syndrome:


1. Central obesity
2. Arterial hypertension
3. Dyslipidaemia
4. Hyperglycaemia

30.120 Which diagnostic tests and analyses are used to confirm acromegaly diagnosis:
1. Glucose tolerance test
2. Low dose dexamethason suppression test
3. Growth hormone and IGF-1 blood serum concentration measurement
4. Saline infusion test

30.121 Which features are specific for "Kussmaul" breathing:


1. Deep
2. Rhythmic
3. Noisy
4. Superficial

30.122 Criteria of adequate preparation before phaeochromocytoma surgery:


1. Blood pressure less than 165/90 mmHg
2. No changes of ST in ECG during 2 weeks
3. Orthostatic hypotension no less than 80/45 mmHg
4. Heart beats rate less than 70 beats/min

30.123 Which laboratory tests changes are specific for Conn's syndrome:
1. Hypokalaemia
2. Decreased renin concentration in blood
3. Increased aldosterone concentration in blood
4. Hyperkalaemia

30.126 What are causes of hypercalcaemia:


1. Excess of calciferol and it's active metabolites
2. Hyperparathyroidism
3. Malignant tumours
4. Deficiency of calciferol and it's active metabolites

30.128 What is specific for Conn's syndrome:


1. Hypertension
2. Metabolic acidosis
3. Hypokalaemia
4. Increased renin concentration

30.129 What is specific for Conn's syndrome:


1. Hypertension
2. Hyperkalaemia
3. Metabolic alcalosis
4. Increased renin concentration

30.134 Acute complications of diabetes mellitus:


1. Hyperglycaemic ketoacidosis
2. Hypoglycaemic coma
3. Hyperosmolar coma
4. Myocardial infarction

30.135 Chronic microvascular complications of diabetes mellitus:


1. Neuropathy
2. Hepatic steatosis
3. Retinopathy
4. Nephrotic syndrome
lsmusis.lsmuni.lt/Klausimai/Spausdinti?Length=0?Kalba=EN&KategorijaId=132&Kalbos_input=EN&Kalbos=EN&KategorijaEn_input=Endocrinology… 12/19
3/27/2019 LSMUSIS
30.136 Chronic macrovascular complications of diabetes mellitus:
1. Peripherial artery disease of legs
2. Varicose veins of legs
3. Myocardial infarction
4. Embolisation of pulmonary artery

30.137 Diagnostic tests which are required for diabetic nephropathy diagnosis:
1. Daily amount of protein in the urine
2. Creatinine level in the blood
3. Albumin/creatinine ratio in the urine
4. Urea level in the blood

IV type tasks. Choose only one best answer

30.74 32 yrs old patient was admitted to the hospital in critical situation: cachectic, nervous, seldom unconscious, t
– 37.3 C, complaining of nausea and occasional vomiting. Examination results: absolute cardiac arrhythmia, heart
rate – 100 times/min, arterial blood pressure – 220/120 mm Hg. Diffuse (IA) goiter on palpation. Several years ago
treatment resistant paroxysmal arterial hypertension was diagnosed, didn’t take antihypertensive medication every
day. Which drug should be administered:
A. Hydrocortisone
B. Thiamazole
C. Prazosin
D. Triiodthyronine

30.75 25 yrs old patient complaints of weakness, fatigue, anorexia, weight loss (6 kg during 2 months), chest pain,
diarrhoea, dizziness (changing body position). She is sick for about 1 year (after viral infection). Examination
results: height = 170 cm, weight = 59 kg. Skin hyperpigmentation., especially in neck area, waist, breasts nipples.
Hyperpigmentation of dental mucosa is present. Heart rate = 84/min, SBP in supine position – 85/60, standing –
70/40 mmHg, painful epigastric area on palpation. Hb 110 g/l, WBC – 5.0 x10/1, ESR – 28 mm/h. Blood glucose
level – 3,3 mmol/l, blood sodium concentration – 110 mmol/l. potassium – 6,0 mmol/l, decreased urine 24 hrs 17-
OH-corticosteroid and 17-corticosteroid excretion rate. What is the diagnosis:
A. Chronic gastritis
B. Hypothalamic dysfunction
C. Addison’s disease
D. Encephalitis

30.76 25 yrs old patient complaints of weakness, fatigue, anorexia, weight loss (6 kg during 2 months), chest pain,
diarrhoea, dizziness (changing body position). She is sick for about 1 year (after viral infection). Examination
results: height = 170 cm, weight = 59 kg. Skin hyperpigmentation., especially in neck area, waist, breasts nipples.
Hyperpigmentation of dental mucosa is present. Heart rate = 84/min, SBP in supine position – 85/60, standing –
70/40 mmHg, painful epigastric area on palpation. Hb 110 g/l, WBC – 5.0 x10/1, ESR – 28 mm/h. Blood glucose
level – 3,3 mmol/l, blood sodium concentration – 110 mmol/l. potassium – 6,0 mmol/l, decreased urine 24 hrs 17-
OH-corticosteroid and 17-corticosteroid excretion rate. What is the cause of hyperpigmentation:
A. Increased ACTH (adrenocorticotrophic hormone) concentration
B. Increased growth hormone concentration
C. Decreased TSH concentration
D. Elevated prolactin concentration

30.77 32 years old patient has Addison’s disease for 5 years. She was fine on replacement therapy (prednisolone
– 10mg/d., fludrocortisone 0.1mg every second day). 4 days ago she became sick with influenza. The patient
continued replacement therapy, but a few days later her condition markedly impaired: nausea, vomiting, general
weakness, hypodynamia, hyperthermia occurred (t = 41°C). BP decreased untill 50/30 mmHg. What kind of
treatment would you prefer:
A. Stop glucocorticoids and fludrocortisone
B. Stop glucocorticoids and treat only with fludrocortisone
C. Increase glucocorticoid doses and stop fludrocortisone
D. Stop fludrocortisone, leaving glucocorticoids

30.78 32 years old patient has Addison’s disease for 5 years. She was fine on replacement therapy (prednisolone
– 10mg/d., fludrocortisone 0.1mg every second day). 4 days ago she became sick with influenza. The patient
continued replacement therapy, but a few days later her condition markedly impaired: nausea, vomiting, general

lsmusis.lsmuni.lt/Klausimai/Spausdinti?Length=0?Kalba=EN&KategorijaId=132&Kalbos_input=EN&Kalbos=EN&KategorijaEn_input=Endocrinology… 13/19
3/27/2019 LSMUSIS
weakness, hypodynamia, hyperthermia occurred (t = 41°C). BP decreased untill 50/30 mmHg. What initial doses
should be:
A. Hydrocortisone – 7.5-12.5 mg and fludrocortisone 0.1 mg
B. Hydrocortisone – 20-25 mg i/v every 6 hrs and fludrocortisone 0.3 mg
C. Hydrocortisone – 50-75 mg and fludrocortisone 0.1 mg
D. Hydrocortisone – 100 mg i/v every 6 hrs

30.79 24 yr old patient was admitted to the hospital in critical situation, complaining of nausea, vomitting,
drowsiness, pain in epigastrium. She has diabetes for 5 years. Examination results: T 38 C, decreased weight, dry
mucosa and skin, pulse 98/min, BP – 100/60 mmHg. Blood glucose level 22 mmol/l, WBC – 9,1x10/l. Urine test:
glucose – 110 mmol/l, ketones (++++). The surgeon didn’t find surgical disease. What is the diagnosis:
A. Severe hypoglycaemia
B. Diabetic hyperosmolar precoma
C. Diabetic ketoacidosis
D. Acute adrenal insufficiency

30.80 24 yr old patient was admitted to the hospital in critical situation, complaining of nausea, vomitting,
drowsiness, pain in epigastrium. She has diabetes for 5 years. Examination results: T 38 C, decreased weight, dry
mucosa and skin, pulse 98/min, BP – 100/60 mmHg. Blood glucose level 22 mmol/l, WBC – 9,1x10/l. Urine test:
glucose – 110 mmol/l, ketones (++++). The surgeon didn’t find surgical disease. Which drug should be
administered:
A. Sulphonylurea
B. Metformin
C. Insulin
D. Glucagon

30.81 40 yr old man was admitted to the department of Internal Diseases complaining of a pain in epigastium
when touching, weight loss, thirst, anorexia, increased abdomen in size. The same omplaints occurred 1,5 yr
before. In the hospital pancreatic cysta was diagnosed and removed. After surgery impaired metabolism of
carbohytrates was found: fasting blood glucose 7,5-9 mmol/l, postprandial – 10-14 mmol/l. Glucosuria 5,55 mmol/l,
ketones - absent. What is the diagnosis:
A. Impaired glucose tolerance
B. Type 1 DM
C. Type 2 DM
D. Other specific types: due to diseases of the exocrine pancreas

30.82 40 yr old man was admitted to the department of Internal Diseases complaining of a pain in epigastium
when touching, weight loss, thirst, anorexia, increased abdomen in size. The same omplaints occurred 1,5 yr
before. In the hospital pancreatic cysta was diagnosed and removed. After surgery impaired metabolism of
carbohytrates was found: fasting blood glucose 7,5-9 mmol/l, postprandial – 10-14 mmol/l. Glucosuria 5,55 mmol/l,
ketones - absent. What is the treatment:
A. Diet
B. Sulphonylurea
C. Biguanides
D. Insulin

30.83 Diffuse-toxic goiter was diagnosed for 60 yr old woman and treated with radioactive iodine 131 5 years
before. The present complaints are sleepinesss, cold intolerance, constipation. Examination results; dry skin,
hardly palpable thyroid gland. Bradycardia – 56 beats/min, blood pressure 140/80 mmHg. What is the diagnosis:
A. Adrenal insufficiency
B. Leucosis
C. Hypothyroidism
D. Vegetodystonia

30.84 Diffuse-toxic goiter was diagnosed for 60 yr old woman and treated with radioactive iodine 131 5 years
before. The present complaints are sleepinesss, cold intolerance, constipation. Examination results; dry skin,
hardly palpable thyroid gland. Bradycardia – 56 beats/min, blood pressure 140/80 mmHg. What is the cause of
bradycardia:
A. Increased potassium concentration in blood
B. Hypercholesterolemia
C. Decreased thyroxine and triiodthyronine concentration in blood
D. Leucopenia
lsmusis.lsmuni.lt/Klausimai/Spausdinti?Length=0?Kalba=EN&KategorijaId=132&Kalbos_input=EN&Kalbos=EN&KategorijaEn_input=Endocrinology… 14/19
3/27/2019 LSMUSIS
30.85 30 yr old woman came to take medical advice because of turning for the worse Addison’s disease (she has
Addison’s disease for 5 years). She was treated with hydrocortisone 10 mg on waking and 5 mg at midday, and
fludrocortisone 0,05 mg daily. 3 days before she had influenza. Examination results: marked hypotension,
dyspeptic state. Which test is the most specific for this disease:
A. Ultrasonography of suprarenal glands
B. Determination of cortisol in blood
C. Electrolytes and glucose concentration in blood
D. 24 hrs urine catecholamines

30.86 30 yr old woman came to take medical advice because of turning for the worse Addison’s disease (she has
Addison’s disease for 5 years). She was treated with hydrocortisone 10 mg on waking and 5 mg at midday, and
fludrocortisone 0,05 mg daily. 3 days before she had influenza. Examination results: marked hypotension,
dyspeptic state. What is the first line treatment:
A. Glucocorticoids orally
B. 0,9 % NaCl infusion
C. Glucocorticoids i/v
D. Fludrocortisone orally
E. Glucose infusion

30.87 Thyroidectomy for thyroid carcinoma was performed for 34 yr old woman. What medication should be
prescribed for replacement therapy:
A. Hydrocortisone
B. Prednisolone
C. Levothyroxine
D. Octreotide

30.88 Thyroidectomy for thyroid carcinoma was performed for 34 yr old woman. The next day after surgery the
hand cramps occurred, Trousseau’s and Chvostek’s signs were present. What drug should be administered:
A. Panangin
B. Potassium chloride
C. Calcium carbonate
D. Potassium orotate

30.89 6 months before resection of toxic goiter was performed for 38 yrs old woman. A few months after surgery
she started complaining of weight gain, cold intolerance, dryness of skin, hair loss, anorexia. What is the
preliminary diagnosis:
A. Gastritis
B. Skin ichthiosis
C. Hypothyroidism
D. Vegetodystonia

30.90 6 months before resection of toxic goiter was performed for 38 yrs old woman. A few months after surgery
she started complaining of weight gain, cold intolerance, dryness of skin, hair loss, anorexia. What test can be
used for the confirmation of diagnosis:
A. Gastric X-ray
B. Urine test
C. Thyroxine and triiodthyronine level in blood
D. Cortisol concentration in blood

30.91 24 yr old patient had a marked haemorrhage during delivery. 6 months later mental disorder, hypotonia,
constipation, cold intolerance occurred. Amenorrhoea, decreased axillary and pubic hair were present. What
diagnosis should be established:
A. Anorexia nervosa
B. Primary hypothyroidism
C. Primary adrenal insufficiency
D. Sheehan’s syndrome

30.92 28 yr old woman was admitted to the hospital. Hypoglycaemic coma was diagnosed. What is the treatment:
A. 5% glucose infusion
B. 40% glucose 40-100 ml i/v
C. Prednisolone 30mg i/v
D. All earlier mentioned

lsmusis.lsmuni.lt/Klausimai/Spausdinti?Length=0?Kalba=EN&KategorijaId=132&Kalbos_input=EN&Kalbos=EN&KategorijaEn_input=Endocrinology… 15/19
3/27/2019 LSMUSIS
30.93 28 yr old woman was admitted to the hospital. Hypoglycaemic coma was diagnosed. What are the
complications of hypoglycaemic coma:
A. Cerebral oedema
B. Nephrotic syndrome
C. Hepatic failure
D. Possible all complications

30.94 45 yr old woman is complaining of an itching in the region of genitalia, dryness of the mouth, increased
urination. Examination results: Obese, pulse rate 80 beats/min, blood pressure 130/80 mmHg. What is the
preliminary diagnosis:
A. Toxic goiter
B. Diabetes mellitus
C. Diabetes insipidus
D. Fungal skin infection

30.95 45 yr old woman is complaining of an itching in the region of genitalia, dryness of the mouth, increased
urination. Examination results: Obese, pulse rate 80 beats/min, blood pressure 130/80 mmHg. Which test is the
best for the confirmation of diagnosis:
A. Vaginal smear
B. Glycaemia
C. Serum thyrotropin
D. Urine swab

30.96 62 yr old woman was complaining of a paroxysmal tachycardia episodes predominately at 5-6 h AM,
accompanied by sweating and insomnia. She has diabetes mellitus for 5 years, is treated with insulin NovoMix
28U at 8 AM (before breakfast) and 12U at 8 PM. Morning fasting glycaemia 10-11 mmol/l. Hb A1c 6,4 %. What do
you suspect:
A. Hyperglycaemia due to low dose of evening insulin.
B. Hypoglycaemia and later occurrence of hyperglycaemia
C. Cardiac arrhythmia
D. Complains are not serious

30.97 62 yr old woman was complaining of a paroxysmal tachycardia episodes predominately at 5-6 h AM,
accompanied by sweating and insomnia. She has diabetes mellitus for 5 years, is treated with insulin NovoMix
28U at 8 AM (before breakfast) and 12U at 8 PM. Fasting glycaemia 10-11 mmol/l. Hb A1c 6,4 %. What is the
treatment:
A. Treatment has not to be changed
B. Increase morning insulin dose
C. Decrease evening insulin dose
D. Neurotropic drugs in the evening

30.98 16 yr old teenager was admitted to the hospital. 2 day before he had an acute virusal infection, T 39 C. Later
he became thirsty, the skin and mouth dry, abdominal pain, nausea and vomiting were present. Examination
results: hypodynamia, impaired consciousness, the skin and mucous membranes dry, the breath smells of
acetone. Blood glucose level 23 mmol/l, glucosuria >55,5 mmol/l, ketonuria > 7,8 mmol/l (+++). What is the
diagnosis:
A. Hypoglycaemic coma
B. Ketoacidotic coma
C. Hyperosmolar coma
D. Lactic acidosis

30.99 16 yr old teenager was admitted to the hospital. 2 day before he had an acute virusal infection, T 39 C. Later
he became thirsty, the skin and mouth dry, abdominal pain, nausea and vomiting were present. Examination
results: hypodynamia, impaired consciousness, the skin and mucous membranes dry, the breath smells of
acetone. Blood glucose level 23 mmol/l, glucosuria >55,5 mmol/l, ketonuria > 7,8 mmol/l (+++). What is the
treatment:
A. 40% glucose 60 ml i/v
B. Sodium bicarbonate infusion
C. Glucagon 1 mg i/m
D. Isotonic Saline – 2000-3000 ml and insulin i/v

lsmusis.lsmuni.lt/Klausimai/Spausdinti?Length=0?Kalba=EN&KategorijaId=132&Kalbos_input=EN&Kalbos=EN&KategorijaEn_input=Endocrinology… 16/19
3/27/2019 LSMUSIS
30.100 56 yr old patient has diabetes mellitus for 8 years. He is treated by 2 premixed insulin HumalogMix 25
injections: 40U in the morning and 32U in the evening). After intense physical activity the patient became
unconscious. In examination: sweating, smell of acetone in breath is absent, normal skin turgor. Glycaemia 2,5
mmol/l. What is the diagnosis:
A. Ketoacidotic coma
B. Hypoglycaemic coma
C. Cerebrovascular insufficiency
D. Uraemic coma

30.101 56 yr old patient has diabetes mellitus for 8 years. He is treated by 2 premixed insulin HumalogMix 25
injections: 40U in the morning and 32U in the evening). After intense physical activity the patient became
unconscious. In examination: sweating, smell of acetone in breath is absent, normal skin turgor. Glycaemia 2,5
mmol/l. What kind of treatment should be administered:
A. Isotonic saline infusion 2000-3000 ml with insulin infusion
B. 40% - 60 ml glucose i/v
C. 4 mg of dexamethasone i/m
D. 24% Euphyllin 10 ml i/v

30.102 30 yr old patient was complaining of a dry skin, impaired memory and facial intumescence (oedema). 4
months before she had acute purulent thyroiditis. Examination results: coarse dry skin. Pulse rate 56 beats/min,
small volume; Heart sounds are toneless; Normal breathing; Stresses constipation. Urination is normal. I-II goiter,
painless. Eyelid oedema. What is the preliminary diagnosis?
A. Recurrent acute purulent thyroiditis
B. Hypothyroidism
C. Hypoparathyroidism
D. Vegetodystonia

30.103 30 yr old patient was complaining of a dry skin, impaired memory and facial intumescence (oedema). 4
months before she had acute purulent thyroiditis. Examination results: coarse dry skin. Pulse rate 56 beats/min,
small volume; Heart sounds are toneless; Normal breathing; Stresses constipation. Urination is normal. I-II goiter,
painless. Eyelid oedema. What tests are useful for confirmation of diagnosis:
A. Concentration of parathyroid hormone in blood
B. Blood protein and protein fractions
C. Concentration of Ca and P in blood
D. Thyroid stimulating hormone, triiodthyronine and thyroxine blood concentration

30.104 30 yr old patient was complaining of a dry skin, impaired memory and facial intumescence (oedema). 4
months before she had acute purulent thyroiditis. Examination results: coarse dry skin. Pulse rate 56 beats/min,
small volume; Heart sounds are toneless; Normal breathing; Stresses constipation. Urination is normal. I-II goiter,
painless. Eyelid oedema. Which drug should be administered for treatment:
A. Levothyroxine
B. Penicillin
C. Prednisolone
D. Calcium gluconate

30.105 29 yr old patient complaints of sleep disturbance, increased nervousness, sweating, palpitation. After
thyroid surgery she is treated with L-thyroxin = 100 mcg twice a day (in the morning and in the evening) and
triiodthyronine 20 µg before dinner. Examination results: sweating skin, rhythmic pulse 90/min, BP 145/70 mmHg,
vesicular breathing. Scar on the neck after thyroid surgery, the thyroid tissue is not found. Pathological eye signs
are absent. What is the diagnosis:
A. Hypoparathyroidism
B. Vegetodystonia
C. Postoperative goiter with light thyrotoxicosis
D. Overdosage of thyroid hormones

30.106 29 yr old patient complaints of sleep disturbance, increased nervousness, sweating, palpitation. After
thyroid surgery she is treated with L-thyroxin = 100 mcg twice a day (in the morning and in the evening) and
triiodthyronine 20 µg before dinner. Examination results: sweating skin, rhythmic pulse 90/min, BP 145/70 mmHg,
vesicular breathing. Scar on the neck after thyroid surgery, the thyroid tissue is not found. Pathological eye signs
are absent. What is the treatment:
A. Stop thyroid medication
B. Stop thyroid medication and begin thiamazole treatment
lsmusis.lsmuni.lt/Klausimai/Spausdinti?Length=0?Kalba=EN&KategorijaId=132&Kalbos_input=EN&Kalbos=EN&KategorijaEn_input=Endocrinology… 17/19
3/27/2019 LSMUSIS
C. Decrease doses of thyroid medication and prescribe them in the morning
D. Leave the same treatment and administer B-blockers

30.107 Thyrotoxic storm was diagnosed for 34 yr old patient after thyroid surgery. Pulse – 156 times/min, arterial
blood pressure – 50/0 mmHg. Which glucocorticoid should be administered:
A. Hydrocortisone
B. Triamcinolone
C. Dexamethasone
D. Prednisolone

30.108 36 years old patient complaints of increased nervousness, sensitivity, palpitation, weakness. He fell ill 3
months before after psychological stress. During these 3 months the weight loss exceeded 4 kg, hand tremor
occurred. Examination results: diffuse II degree goiter, frightened sight, positive Kocher’s and Graefe’s signs.
Pulse rate 112 times/min, rhythmic. Cardiac tones well heard. No other pathological changes. What is the
diagnosis:
A. Myocarditis
B. Vegetodysfunction
C. Toxic goiter
D. Hipopituitarism

30.109 36 years old patient complaints of increased nervousness, sensitivity, palpitation, weakness. He fell ill 3
months before after psychological stress. During these 3 months the weight loss exceeded 4 kg, hand tremor
occurred. Examination results: diffuse II degree goiter, frightened sight, positive Kocher’s and Graefe’s signs.
Pulse rate 112 times/min, rhythmic. Cardiac tones well heard. No other pathological changes. Which medication
should be administered:
A. Levothyroxine
B. Thiamazole
C. Bromocriptine
D. Octreotide

30.110 32 yrs old patient was admitted to the hospital in critical situation: cachectic, nervous, seldom unconscious,
t – 37.3 C, complaining of nausea and occasional vomiting. Examination results: absolute cardiac arrhythmia,
heart rate – 100 times/min, arterial blood pressure – 220/120 mm Hg. Diffuse (IA) goiter on palpation. Several
years ago treatment resistant paroxysmal arterial hypertension was diagnosed, didn’t take antihypertensive
medication every day. What is the diagnosis:
A. Acute adrenal insufficiency
B. Thyrotoxic storm
C. Pheochromocytoma
D. Acute pancreatitis

30.1 - D 30.2 - C 30.3 - E 30.4 - E 30.5 - D 30.6 - B 30.7 - B 30.8 - D


30.9 - A 30.10 - D 30.11 - E 30.12 - D 30.13 - C 30.14 - D 30.15 - D 30.16 - E
30.17 - B 30.18 - C 30.19 - D 30.20 - E 30.21 - E 30.22 - B 30.23 - E 30.24 - A
30.25 - C 30.26 - D 30.27 - C 30.28 - C 30.29 - D 30.30 - C 30.31 - C 30.32 - C
30.33 - C 30.34 - C 30.35 - C 30.36 - A 30.37 - C 30.38 - B 30.39 - E 30.40 - E
30.41 - A 30.42 - C 30.43 - D 30.44 - A
30.45 30.46 30.47 30.48 30.49 30.50 30.51 30.52
1-D 1-B 1-D 1-A 1-A 1-A 1-A 1-D
2-A 2-A 2-C 2-B 2-B 2-C 2-B 2-A
3-B 3-D 3-B 3-A 3-C 3-B 3-C 3-B
4-C 4-C 4-A 4-C 4-D 4-D 4-D 4-C
30.53 30.54 30.55 30.56 30.57 30.58 30.59 30.60
1-C 1-C 1-D 1-D 1-A 1-B 1-B 1-A
2-B 2-B 2-C 2-A 2-C 2-C 2-A 2-B
3-A 3-D 3-B 3-C 3-B 3-A 3-C 3-C
4-A 4-A 4-A 4-B 4-D 4-A 4-C 4-D
30.61 30.112 30.113 30.124 30.125 30.127 30.130 30.131
1-A -A -C -B -D -B -D -A
2-B -C -B -D -C -D -A -C
3-C -B -D -C -B -A -B -D
lsmusis.lsmuni.lt/Klausimai/Spausdinti?Length=0?Kalba=EN&KategorijaId=132&Kalbos_input=EN&Kalbos=EN&KategorijaEn_input=Endocrinology… 18/19
3/27/2019 LSMUSIS
4-C -D -A -A -A -C -C -B
30.132 30.133 30.138
-C -B -B
-A -D -C
-B -A -D
-D -C -A
30.62 30.63 30.64 30.65 30.66 30.67 30.68 30.69
1 1 4 1 1 1 2 1
2 2 2 2 3 4 3
3 3 3 3
4
30.70 30.71 30.72 30.73 30.111 30.114 30.115 30.116
4 1 4 1 4 2 1 1
2 2 4 2 2
3 3 3 3
4
30.118 30.117 30.119 30.120 30.121 30.122 30.123 30.126
1 1 1 1 1 1 1 1
2 2 2 3 2 2 2 2
3 3 3 3 3 3 3
4
30.128 30.129 30.134 30.135 30.136 30.137
1 1 1 1 1 1
3 3 2 3 3 3
3
30.74 - C 30.75 - C 30.76 - A 30.77 - C 30.78 - D 30.79 - C 30.80 - C 30.81 - D
30.82 - D 30.83 - C 30.84 - C 30.85 - B 30.86 - C 30.87 - C 30.88 - C 30.89 - C
30.90 - C 30.91 - D 30.92 - B 30.93 - A 30.94 - B 30.95 - B 30.96 - B 30.97 - C
30.98 - B 30.99 - D 30.100 - B 30.101 - B 30.102 - B 30.103 - D 30.104 - A 30.105 - D
30.106 - C 30.107 - A 30.108 - C 30.109 - B 30.110 - C

lsmusis.lsmuni.lt/Klausimai/Spausdinti?Length=0?Kalba=EN&KategorijaId=132&Kalbos_input=EN&Kalbos=EN&KategorijaEn_input=Endocrinology… 19/19
3/27/2019 LSMUSIS

Otorhinolaryngology
I type tasks. Choose only one best answer

45.1 For what disease the otoscopic signs of retracted tympanic membrane and air - fluid level line with air
bubbles are typical:
A. Otitis media purulenta chronica
B. Otitis media exudatyva (serosa)
C. Otitis media acuta
D. Fractura pyramidalis

45.2 What is the most frequent cause of nasal polyps:


A. Infection
B. Allergy
C. Tumours
D. Idiopathic

45.3 In what case of acute otitis media the paracentesis is not indicated?
A. Bulging eardrum and non-controlled earache
B. Hyperemic, non-bulging eardrum
C. Signs of otogenic meningitis are present
D. Hyperemic, infiltrated eardrum and fever

45.4 Detection of the stage of laryngeal stenosis is based on:


A. Stridor (dyspnea) intensity
B. Glottal gap (mm)
C. Clinical features (stridor, cianosis, patient position, general status)
D. Pulsoxymetry

45.5 What kind of treatment should be recommended for the snoring patient with AHI – 10t/h?
A. CPAP
B. Uvulopalatopharyngoplasty (UVPP)
C. Radiofrequency thermoablation
D. No treatment recommended

45.6 What urgently should be done in case of nasal bleeding?


A. Anterior nasal packing
B. Intranasal haemostatic solution drops
C. To press nasal tip with fingers and raise the head up
D. To press nasal tip with fingers and bend the head down

45.7 Which disease is the absolute indication for tonsillectomy:


A. Tonsillitis chronica
B. Pyelonephritis
C. Angina lacunaris
D. Tonsillar hypertrophy and OSAS

45.8 Which of the following symptoms is the mostly typical for malignant tumor the nasal cavity :
A. Common nasal bleeding
B. Headache
C. Permanent hemorrhagic - purulent nasal secretion
D. Olfactory disorders

45.9 Increasing hoarseness with dyspnea in childhood is typical for:


A. Vocal fold nodules
B. Laryngeal papillomatosis
C. Chronic tonsillitis
D. Adenoids

45.10 The patient suffering from chronic purulent otitis media with cholesteatoma felt severe vertigo while deeply
cleaning the external ear canal. What disease can you suspect:
A. Fistula canalis semicircularis horizontalis
B. Absessus cerebri
lsmusis.lsmuni.lt/Klausimai/Spausdinti?Length=0?Kalba=EN&KategorijaId=147&Kalbos_input=EN&Kalbos=EN&KategorijaEn_input=Otorhinolaryngol… 1/5
3/27/2019 LSMUSIS
C. Perforatio membranae tympani
D. Deffectus ossicularum

45.11 What should be the treatment strategy for acute hearing loss :
A. Out-patient medical treatment
B. Referral for the specialist’s (ophthalmologist, neurologist, internist) consultations for clearing up the
causes of deafness
C. Should be considered as medical emergency following the urgent treatment of neuropathy and searching
for possible cause of sudden deafness
D. Patient doesn't need the specific treatment because of spontaneous remission

III type tasks. For each question there is one or more correct answers:
A – if correct answers are 1,2,3
B – if correct answers are 1 and 3
C – if correct answers are 2 and 4
D – if correct answer is 4
E – if correct are all answers above

45.12 Which of the following diseases can be suspected in case of earache and hearing loss:
1. Acute otitis media with effusion (serootitis)
2. Acute otitis media
3. Acute sensorineural hearing loss
4. Exacerbation of chronic otitis media

45.13 Which are the most typical symptoms of laryngeal cancer:


1. Permanent hoarseness
2. Sore throat
3. Progressive dyspnea
4. Attacks of dry cough

45.14 What are the indications for adenoidectomy:


1. Recurrent otitis media complicated with hearing impairment
2. Child is in preparation for school
3. "Adenoid face" and nasal obstruction, dental occlusion problems
4. Frequent nasal bleeding

45.15 Which of the following laryngeal diseases are precancerous?


1. Adult onset laryngeal papillomas
2. Polyp of the vocal fold
3. Chronic hyperplastic laryngitis with keratosis
4. Syphilis of the larynx

45.16 What are the most typical symptoms of acute mastoiditis:


1. „Pus reservoir“ sign
2. Sensorineural hearing loss
3. Hyperemia and swelling of retroauricular region
4. Vertigo

IV type tasks. Choose only one best answer

45.19 A 60 year old farmer machine - operator complains of on pain in the right side of the throat during
swallowing, there is a sensation of a foreign body in the throat and feeling of itching. Two weeks ago he has
noticed a hard tumor on the right side of the neck. Treatment with antibiotics and gargle were not effective.
Objective examination: the right tonsil is enlarged and rigid; there is ulceration with irregular borders on the upper
part of the tonsil covered with grey plaque. The hard and fixed, 3x5 cm diameter „mass“ is felt by the palpation on
the right side of the neck. What is the preliminary diagnosis?
A. Paratonsillar abscess
B. Parapharyngeal abscess
C. Malignant tumor of the tonsil
D. Necrotic tonsillitis

lsmusis.lsmuni.lt/Klausimai/Spausdinti?Length=0?Kalba=EN&KategorijaId=147&Kalbos_input=EN&Kalbos=EN&KategorijaEn_input=Otorhinolaryngol… 2/5
3/27/2019 LSMUSIS
45.20 A 44 years old female complains of stridor during physical activities, sensation of a foreign body in the
through and hoarseness. She had periodical several treatment courses for reflux and heartburn. The patient had
complicated surgery of stomach 2 months ago, after which she was under the endotracheal intubation for 10 days.
Laryngoscopy: bilateral rough mass on the posterior part of the vocal folds, continuing to the infraglottic space;
vocal fold mobility not impaired. What is the preliminary diagnosis?
A. Bilateral laryngeal palsy
B. Vocal fold granulomas
C. Vocal fold cysts
D. Vocal fold polyps

45.21 A female patient complains of severe pain in the left ear and headache, purulent discharge from the left ear
and hearing loss. Her left ear is unhealthy since childhood after she has had scarlet fever. The surgical treatment
was suggested earlier, but she refused. The purulent discharge recurred 3 days ago after she had got a cold.
Objective examination: yellowish, fetid pus in the left external ear canal; marginal perforation of the tympanic
membrane and whitish mass are seen after cleaning the external external ear canal. Mastoid process is intact.
What supplementary examination is necessary to make a diagnosis?
A. Blood test and audiometry
B. Blood test, audiometry and electroencephalography
C. Blood test, audiometry and computed tomography of the temporal

45.22 A 8 months old infant suffers for running nose during 4 days. Since the last two days he is fussy, crying,
turning the head to the sides, trying to touch the right ear, refuses to take mothers breast for feeding, vomits and
has diarrhea. He was awake frequently during the last night, short seizures were observed. Objective data: body
temperature 39,2oC, decreased activity, pale, dry tongue, hyperemic and swollen nasal mucosa. Pharyngeal
mucosa is reddish and without plagues. The otoscopy of the right ear: severe redness and bulging of the tympanic
membrane, no malleus handle neither the light reflex are visible. The left tympanic membrane is intact. What is the
diagnosis?
A. Serootitis
B. Acute rhinitis
C. Acute otitis media

45.23 A 10 years old child complains of recurrent discharge from the left ear and hearing loss. After the common
cold 3 days ago the discharge from the left ear reappeared, the headache started, the left ear became bulging.
Objective data: T- 38,8oC, purulent yellowish discharge in the left ear canal, after cleaning it up with the suction
the marginal perforation of the tympanic membrane as well as the infiltrated upper and posterior walls of the ear
canal are seen. The retroauricular region is swollen and painful. CT of the temporal bone revealed destruction of
the bony mastoid walls and the cavity filed with material consistent with thickened substance or pus. Formation of
abscess can be noticed behind the left ear. What should be the treatment tactics?
A. Cleaning of external ear canal, decongestants, oral antibiotics
B. Mastoidectomy and antibiotics
C. Tympanoplasty and oral antibiotics

45.24 The 28 years old builder complains for the fever 37,6C0 and pain in the right side of the pharynx occurred
after drinking of cold drink for. As the same situation he had experienced some time ago, the man treated himself
with the tea infusions and antiseptic pastilles. Two days ago the pharyngeal pain recurred; the neck swelling and
hoarseness appeared. The GP prescribed Amoxicilline. On the following day the patient was not able to swallow
even the saliva, swelling of the neck increased, diyspnea and retrosternal pain appeared. The emergency car took
the patient to the emergency room. On admittance: body temperature 38,6C0, P – 84 times/min, ABP 120/80
mm/Hg, respiratory rate – 30 times/min. Examination: hyperemic pharyngeal mucosa, pharyngeal tonsils and
palate are intact, however swelling of the right posterior pharyngeal arc and lateral pharyngeal wall is evident . No
pus obtained during puncture of that area. Swelling of epiglottis and right arytenoid, intact and mobile vocal folds,
glottis is free. Swelling of the right side of the neck, very painful palpation of that region. What is the preliminary
diagnosis?
A. Paratonsillar abscess
B. Parapharyngeal abscess
C. Acute tonsillitis

45.25 The parents of 5 years old boy visited the ENT specialist because of the hoarseness, dyspnoe and apnea
episodes during the night sleep. Conventional ENT examination revealed the III0 hypertrophy of pharyngeal tonsils
and the IV0 hypertrophy of palate tonsils. The vocal fold nodules in the middle third area were observed during the
videolaryngoscopy. What of treatment tactics should be recommended?
A. Endolaryngeal microsurgery on vocal fold nodules
lsmusis.lsmuni.lt/Klausimai/Spausdinti?Length=0?Kalba=EN&KategorijaId=147&Kalbos_input=EN&Kalbos=EN&KategorijaEn_input=Otorhinolaryngol… 3/5
3/27/2019 LSMUSIS
B. Observation
C. Adenotonsillectomy

45.26 A 46 years old male visited the ENT specialist for recurrent nasal bleedings from the left side of the nose,
blockage of the left side of the nose, headache, facial asymmetry and double vision, changed shape of the nose.
The symptoms lasted for 3 years. Objective examination: left eye is dislocated forwards and downwards.
Rhinoscopy and endoscopy: the left nasal cavity obstructed with the dark red, rough, rigid mass, which bleeds
during palpation. Right side of the nose is normal. What disease do you suspect?
A. Chronic rhinosinusitis with nasal polyposis
B. Acute complicated rhinosinusitis
C. Nasal foreign body
D. Cancer of nasal and paranasal sinuses

45.27 A 46 years old male visited the ENT specialist for recurrent nosebleeds from the left side, blockage of the
left side of the nose, headache, facial asymmetry and double vision, changed form of the nose. The symptoms
lasted for 3 years. Objective examination: left eye is dislocated forwards and downwards. Rhinoscopy and
endoscopy: the left nose cavity obstructed with the dark red, rough, rigid masses, which bleed during palpation.
Right side of the nose is normal. What should be the first test for proving the diagnosis?
A. X-ray of paranasal sinuses
B. MRI
C. Biopsy and CT

45.28 A female patient complains of chills, high fever to 39 C, and pain on the right head side. She suffers from
periodical aural discharge since childhood and was suggested for surgery some time ago. The purulent discharge
repeated 3 days ago after she had gotten a cold. Objective examination: the fetid pus in the right external ear
canal, pulsating reflex of the pus, the marginal defect of the tympanic membrane, with whitish mass seen in the
defect. The palpation of the mastoid apex is painful. CT scan: sclerotic changes of the mastoid process with
destruction in the right side. What is the diagnosis?
A. Recurrent acute otitis
B. Exacerbation of tubotympanic disease with mastoiditis
C. Exacerbation of atticoantral disease with cholesteatoma

45.29 What would be the preliminary diagnosis of a patient, complaining on permanent discharge from the nose
and posterior nose drip, nose congestion, forehead pain and fever episodes, hyposmia for 5 weeks?
A. Acute rhinitis
B. Chronic rhinitis
C. Acute rhinosinusitis
D. Chronic rhinosinusitis

45.30 A 60 years old patient complains of persistent hoarseness lasting for 5 months. He works in the chemistry
and smokes more that 20 years. Objective examination: the right vocal fold is thickened, rough, and immobile
during phonation. Lymphnodes on the right side of the neck are enlarged. What is the preliminary diagnosis?
A. Chronic laryngitis
B. Laryngeal palsy
C. Laryngeal carcinoma
D. Polyp of the larynx

45.31 The student complains for forehead pain, nose congestion, posterior nasal drip, hyposmia. During the 2
weeks he has a common cold, but no fever. The hot tea infusions and nose lavage with the sea water were used
for the treatment, however with no relief of the symptoms. What would you add to the treatment?
A. Secretolitics
B. Decongestants and/or intranasal steroids
C. Antibiotics
D. Systemic steroids

45.32 26 years old patient was tracheotomized due to laryngeal stenosis after the neck trauma. The tracheotomy
tube N.5 was inserted. Three hours later, after attack of deep cough, airway distress occurred again. Tracheotomy
tube is free. What is the possible cause of the dyspnea:
A. Rupture of the trachea
B. Cordial insufficiency
C. Tracheotomy tube displaced out of trachea
D. Laryngeal edema

lsmusis.lsmuni.lt/Klausimai/Spausdinti?Length=0?Kalba=EN&KategorijaId=147&Kalbos_input=EN&Kalbos=EN&KategorijaEn_input=Otorhinolaryngol… 4/5
3/27/2019 LSMUSIS
45.33 A 25 years old male underwent the tonsillectomy 5 days ago. There was some fever after the surgery; the
patient consumed a small amount of liquids due to the pain. Today right after the meal a fresh blood appeared in
the mouth. The bleeding became more intensive after coughing. The patient spat out around a half glass of blood
clots. He vomited about a half liter of dark gastric content. The patient was sweating, he felt general weakness.
What treatment tactics should be?
A. No treatment needed
B. Advice to take more liquids, do not take any food during one day
C. Antihaemorrhagic treatment at home
D. Hospitalization: revision of the post tonsillectomy areas, intravenous infusion, observation

45.34 A 33 years old patient complains of dyspnea increasing during physical activity. She underwent
thyroidectomy 2 months ago. Dyspnea and hoarseness occurred immediately after the surgery. Dyspnea has
increased during the time and the patient became disable. Difficulty to walk occurred; the patient sleeps in
semisitting position. Objective examination: cyanosis of the lips, slightly hoarse voice, and intensive inspiratory
stridor, indrawing of the suprasternal and jugular notches on inspiration, some dry rales in the lungs. Blood
pressure -130/70 mmHg, pulse rate – 110 t/min. Which of the following examinations will help to make a correct
diagnosis?
A. X-ray of the chest
B. Indirect laryngoscopy, videolaryngoscopy
C. Bronchoscopy
D. Spirometry

45.35 A 40 years old female complains of hoarseness, and severe dyspnea which appears after talking and
walking. She sleeps in semisitting position, noisy breathing is heard. Two days ago the patient underwent
thyroidectomy. Objective examination: cyanosis of the lips, slightly hoarse voice, inspiratory stridor, indrawing of
the suprasternal and jugular notches on inspiration. Arterial blood pressure -130/70 mmHg, pulse rate – 110 t/min.
Laryngoscopy revealed that focal folds are whitish immobilized in paramedial position, width of the glottis 1-2 mm.
What are the diagnosis and the treatment tactics?
A. Chronic laryngitis. Anti-inflammatory treatment and inhalations should be prescribed
B. Bronchial asthma. Anti-asthmatic treatment should be started
C. Bilateral paralysis of the vocal folds. Laterofixation of the vocal fold or tracheotomy is necessary
D. Edema of the larynx. Anti edematous treatment should be prescribed

45.1 - B 45.2 - B 45.3 - B 45.4 - C 45.5 - C 45.6 - D 45.7 - D 45.8 - C


45.9 - B 45.10 - A 45.11 - C
45.12 45.13 45.14 45.15 45.16
2 1 1 1 1
4 3 3 3 3
45.19 - C 45.20 - B 45.21 - C 45.22 - C 45.23 - B 45.24 - B 45.25 - C 45.26 - D
45.27 - C 45.28 - C 45.29 - C 45.30 - C 45.31 - B 45.32 - C 45.33 - D 45.34 - B
45.35 - C

lsmusis.lsmuni.lt/Klausimai/Spausdinti?Length=0?Kalba=EN&KategorijaId=147&Kalbos_input=EN&Kalbos=EN&KategorijaEn_input=Otorhinolaryngol… 5/5
3/27/2019 LSMUSIS

Forensic medicine
I type tasks. Choose only one best answer

27.1 Who must do the Forensic medical expertise in a criminal case, when the medical personnel is accused by
professional crimes:
A. Highly qualified forensic pathologist
B. Commission of competent doctors which is lead by highly qualified forensic pathologist
C. Any forensic pathologist
D. Chief forensic pathologist of the Republic

27.2 How must the case be evaluated, when the patient died during narcosis, though narcosis was done due all
rules:
A. Accident
B. Doctor's mistake
C. Careless act
D. Crime

27.3 How must the case be estimated, when without checking the label on flacon and incorrect drugs are injected
which leads to hard consequences:
A. Doctor's mistake
B. Accident
C. Careless act
D. Murder caused by carelessness

27.4 What the paramedic not allowed do:


A. Give the prescription of strong acting drugs
B. Do intravenous injections
C. Write the case histories
D. To give sick - sheet

27.5 When does the "crush" syndrome develop:


A. After long pressing of chest and abdomen
B. After long pressing of muscles of extremities
C. When traumatic amputation of extremities is present
D. When multiple long bones fractures are present

27.6 How is the skull fracture named, when a bone fragment less than 1cm2 in size is absent:
A. Depressed
B. Perforated
C. Terracing (stair)
D. Fragmented

27.7 A wound is in the penetrating abdominal wall, its length is 2cm, with smooth margins, one end is sharp, other
blunt, a canal ended in abdominal cavity. How is the wound called:
A. Vulnus incizum (incised wound)
B. Vulnus punctum (puncture)
C. Vulnus puncto-incizum (stab wound)
D. Vulnus contusum (laceration)

27.8 Which of the following wounds is characteristic to suicide:


A. One superficial wound
B. One deep wound
C. One deep wound with some superficial cuttings at the ends or margins of the wound
D. Superficial wounds in different places of the body

27.9 How must the crime be qualified, when mother killed her child, but not new-born (neonates):
A. Murder
B. Neonaticide, mother’s purposely murder ad her new-born.
C. Murder in aggravating circumstances
D. Murder after excitement
E. Careless murder

lsmusis.lsmuni.lt/Klausimai/Spausdinti?Length=0?Kalba=EN&KategorijaId=129&Kalbos_input=EN&Kalbos=EN&KategorijaEn_input=Forensic+medici… 1/5
3/27/2019 LSMUSIS
27.10 The microscopical view of separating place of the umbilical cord:
A. Edema of tissues
B. Leukocytic infiltration, edema and hemorrhage of ring of the umbilical cord
C. Capillary hyperemia
D. Erythrocytes are deformed, with little extravasation

27.11 When is the meconium excreted:


A. 24 hours after birth (at the end of the first day)
B. 48 hours after birth (at the end of the second day)
C. 48-96 hours after birth (in third or fourth day)
D. More than 96 hours after birth (after 4 day)

27.12 When do the remnants of the umbilical cord of infant completely separate:
A. 48 hours after birth
B. 48-72 hours after birth
C. 5-7 days after birth
D. In the beginning of the second week after birth

27.13 At what time after birth does the demarcation ring of the umbilical cord of the neonaticide (new-born)
become seen:
A. 3-4 hours after birth
B. 6-8 hours after birth
C. 24 hours after birth
D. 48 hours after birth
E. 72 hours after birth

27.14 The sign of biological death is:


A. Absence of pulse
B. Absence of corneal reflex
C. Post-modem hypostasis
D. Absence of breathing

27.15 When do terracing (stair) like fractures usually appear:


A. During falling on a hard surface
B. During tangential hit on the skull by a hard blunt object
C. During falling from a height on the head
D. During perpendicular hit on the skull by a hard blunt object with a limited surface

II type tasks. For each numbered item,selct the one lettered heading that is most closely asssciated with it

27.16 Select the signs characteristic to the following mechanical asphyxia:


1 - Manual strangulation
2 - Drowning
3 - Hanging
4 - Pressing of chest and abdomen (traumatic asphyxia )
5 - Ligature strangulation
A. Horizontal ligature mark
B. Oblique ligature mark
C. Abrasions and bruises on the neck
D. Numerous petechial hemorrhages in the skin of face
E. Froth around mouth and nostrils

27.17 Select the signs characteristic to the following kinds of trauma:


1 - Hitting by pads of moving vehicle
2 - Railway injury
3 - 'Running over' injury
4 - Vehicle occupant's trauma
A. Transaction of the body
B. 'Bumper' fractures
C. Pattern of wheel's tread
D. Fractures of cervical part of spine

lsmusis.lsmuni.lt/Klausimai/Spausdinti?Length=0?Kalba=EN&KategorijaId=129&Kalbos_input=EN&Kalbos=EN&KategorijaEn_input=Forensic+medici… 2/5
3/27/2019 LSMUSIS
27.18 Indicate signs characteristic to the following wounds:
1 - Vulnus contusum (laceration split)
2 - Vulnus caesum (chop wound)
3 - Vulnus incisum (incise wound)
4 - Vulnus lacerotum (loceration - stretches)
A. Bone injury
B. Tissue bridges in the bottom wound
C. Smooth margins, sharp ends
D. Variable forms wounds without abrasions on margins

27.19 Select the signs characteristic to the following poisonings:


1 - Ethylalcohol intoxication
2 - Poisoning by dichlorethane
3 - Poisoning by acetic acid
4 - Poisoning by strychnine
5 - Poisoning by Cowbane (Cicuta virosa)
A. Smell of dry mushrooms from contents of stomach and intestines
B. Yellow color of the skin
C. Subendocardial petechia
D. Fast and hard rigor mortis
E. Urinary bladder full of urine

27.20 What does the following hair belong to:


1 - Hair's pulp is thin
2 - Hair's pulp is thick
3 - Margins of cuticular cells are dentated
4 - Pigment is brightly brown in color
A. Human's hair
B. Animal's hair
C. May be human's and animal's
D. Neither human's or animal's

27.21 Indicate the signs characteristic to different mechanisms of drowning and 'sudden death in water' ( due to
immersion):
1 - Asphyxial (dry) drowning
2 - Real drowning (moist)
3 - 'Sudden death in water' (due to immersion)
A. Inflated dry lungs
B. Heavy watery lungs
C. Negative results of “diatom test”

27.22 Indicate the signs of each shooting range:


1 - Close range
2 - Long range
3 - Contact wound
A. Additional shooting factors
B. Tissues defect, abrasion and dirty collar without additional shooting factors
C. Tears at the margins of entrance wound, additional shooting factors are in the wound's canal

27.23 Which injuries are characteristic to indicated circumstances:


1 - Laceration on the head
2 - Multiple fractures of skelet bones
3 - Signs of internal organs commotion
4 - Contusion of brain
5 - Implantation fractures of femoral bones with
A. Injury caused by falling on a smooth hard surfac
B. Injury caused by falling from a height
C. Injury caused either by falling on a smooth hard surface or by falling from a height

III type tasks. For each question there is one or more correct answers:
A – if correct answers are 1,2,3
B – if correct answers are 1 and 3
lsmusis.lsmuni.lt/Klausimai/Spausdinti?Length=0?Kalba=EN&KategorijaId=129&Kalbos_input=EN&Kalbos=EN&KategorijaEn_input=Forensic+medici… 3/5
3/27/2019 LSMUSIS
C – if correct answers are 2 and 4
D – if correct answer is 4
E – if correct are all answers above

27.24 What conditions have influence on poisons action:


1. Characteristic of poison (dose, solubility, admixtures)
2. Ways of entrance of poison into organism
3. Characteristic of the organism
4. External conditions

27.25 Due to what causes in some poisoning cases the results of chemical examination can be negative:
1. It's impossible to estimate the poison by chemical methods
2. Poison was excreted when the person was still alive
3. Poison disintegrates in cadaver
4. Poison was destroyed by enzymes of alive organism

27.26 In what cases poisons can be found during chemical examination even when there is no poisoning:
1. Poison gets from contaminated instruments
2. Poison gets due to mistakes during chemical examination
3. Poison gets into organism as a drug
4. Contaminated reagents were used in reactions

27.27 What must be estimated during examination of a spot similar to blood:


1. Is it a blood spot
2. The kind of blood
3. Blood group
4. Which place of the organism bleeding was from

27.28 What methods must be used to estimate a sperm spot:


1. Microluminescence examination
2. Florans reaction
3. Microscopical examination of stained smear
4. Using the ultraviolet beam

27.29 Which signs are characteristic to manual strangulation:


1. Dark liquid blood
2. Hyperemia of internal organs
3. Semilunar abrasions on the sides of neck
4. Full urinary bladder

27.30 Signs of drowning:


1. Froth around mouth and nostrils
2. Ruptures in carotid arteries intima
3. Paltauf's spots on the surface of lungs
4. Hemorrhage in m. sternocleidomastoideus

27.31 Signs of ligature strangulation:


1. Petechia in eyelids
2. Petechia under epicardium
3. Ligature mark on the neck
4. Fractures of as hyoideum and laryngeal cartilages

27.32 Signs of manual strangulation:


1. Not distinct ligature mark on the neck
2. Fluid in sinus sphenoidalis
3. Spot-like hemorrhages under visceral pleura
4. Round or oval bruises on the neck

27.33 Indicate signs of external and internal injuries caused by falling from height:
1. External injuries are on one side of the body
2. External injuries are symmetrical
3. Not correspond external and internal injuries

lsmusis.lsmuni.lt/Klausimai/Spausdinti?Length=0?Kalba=EN&KategorijaId=129&Kalbos_input=EN&Kalbos=EN&KategorijaEn_input=Forensic+medici… 4/5
3/27/2019 LSMUSIS
4. Brain contusion

27.34 What injuries are characteristic to railway trauma:


1. Implantation fractures of femoral bone
2. Amputations of parts of the body
3. Comminuted fracture of the skull
4. Abraded and crushed zones

27.35 In which cases death will be violent:


1. Cause of death - sepsis caused by spine and spinal cord injury
2. Cause of death - chronic alcoholism
3. Cause of death - alcohol intoxication
4. Cause of death - typhoid fever

27.36 In which cases the forensic autopsy must be done:


1. In violent death
2. In sudden death, when acute disease isn't suspected
3. In suspected violent death
4. In sudden death cams when chronic disease is confirmed

27.37 In which cases appointed medics – legal examination olive persons:


1. When person is injured, and needed to determine the degree of injury and answer other question related
with injury
2. In the case of sexual offenses
3. In the sexual cases
4. To determine psychical state of person

27.1 - B 27.2 - A 27.3 - C 27.4 - A 27.5 - B 27.6 - B 27.7 - C 27.8 - C


27.9 - A 27.10 - B 27.11 - B 27.12 - C 27.13 - C 27.14 - C 27.15 - D
27.16 27.17 27.18 27.19 27.20 27.21 27.22 27.23
1-C 1-B 1-B 1-E 1-A 1-A 1-A 1-C
2-E 2-A 2-A 2-A 2-B 2-B 2-B 2-B
3-B 3-C 3-C 3-B 3-B 3-C 3-C 3-B
4-D 4-D 4-D 4-D 4-C 4-C
5-A 5-D 5-B
27.24 27.25 27.26 27.27 27.28 27.29 27.30 27.31
1 1 1 1 1 1 1 1
2 2 2 2 3 2 3 2
3 3 3 3 3 3
4 4 4 4 4
27.32 27.33 27.34 27.35 27.36 27.37
4 1 2 1 1 1
3 4 3 3 2
3

lsmusis.lsmuni.lt/Klausimai/Spausdinti?Length=0?Kalba=EN&KategorijaId=129&Kalbos_input=EN&Kalbos=EN&KategorijaEn_input=Forensic+medici… 5/5
3/27/2019 LSMUSIS

Geriatrics
I type tasks. Choose only one best answer

39.1 According to demographers old age begins from:


A. 50 years
B. 60 years
C. 70 years
D. 80 years

39.2 The leading causes of death in people over age 65 are:


A. Heart disease, stroke, accidents
B. Heart disease, Alzheimer,s disease, cancer
C. Heart disease, fractures, pneumonia
D. Heart disease, cancer, stroke

39.3 The most important principe of geriatric care is:


A. To prolong life as much as possible
B. To treat all acute and chronic diseases
C. To improve patient's autonomy and quality of life
D. To create safe environment

39.4 What is particularly characteristic to the diseases of the elderly:


A. Twice rarer adverse drug reactions
B. One main disease
C. Disease course and convalescence are shorter
D. Impaired biopsychosocial functions

39.5 A 66-years-old woman is worried about the tremor in her left hand. Your best recommendation to her is:
A. She needs consultation of a neurologist
B. This is normal aging
C. Prescribe antidepressants
D. To follow-up a patient for half a year

39.6 What could improve communication with elderly person who has presbycusis:
A. To shout into affected ear
B. To speak in a loud, low – pitched voice
C. To whisper
D. To use sign language

39.7 What tools are used for malnutrition screening in the elderly:
A. MNA, NRS 2002, MUST, SGA, MST
B. MMSE, GDS, IADL
C. MNA, GDS, NRS 2002, CAM

39.8 A true statement regarding fever in the elderly is:


A. Older adults do not experience a fever with infections
B. Body temperature below 38 C is normal
C. The aging processes increase body temperature in 1-2 C
D. Hypothermia in old age is more frequent than hyperthermia

39.9 Explain why infections more easily develop in the elderly:


A. Impairment of cell mediated and humoral immunity
B. Sensitivity to different antigens is increased
C. Usual infection control measures are not effective
D. A majority of aged persons are in a debilitated state

39.10 Which medication is the first choice for older adults in treatment of delirium:
A. Sertraline
B. Haloperidol
C. Amitriptyline
D. Alprazolam

lsmusis.lsmuni.lt/Klausimai/Spausdinti?Length=0?Kalba=EN&KategorijaId=141&Kalbos_input=EN&Kalbos=EN&KategorijaEn_input=Geriatrics&Kate… 1/10
3/27/2019 LSMUSIS
39.11 Which finding in respiratory system is the most characteristic in old age:
A. Increased mobility of chest
B. Increased vital capacity of lungs
C. Decreased resistance for hypoxemia and hypercapnia
D. Increased assimilation of O2

39.12 Risk factors of hypoglycemia in the elderly are:


A. Alcohol, malnutrition, renal failure, insulin therapy, strict glycemic control
B. Delirium, surgery, depression, obesity, antibiotics, smoking
C. Pneumonia, renal failure, dementia, stroke
D. Antidepressants, antihypertensive drugs, thiazide diuretics, glucocorticosteroids

39.13 Which statement is true regarding syncope in the elderly:


A. Loss of consciousness present
B. Consciousness is not impaired
C. Syncope is not related to cardiovascular diseases
D. Convulsions are present

39.14 The target arterial blood pressure in the elderly is:


A. <140/90
B. <160/90
C. >140/90

39.15 A 82-year-old man with history of coronary artery disease, diabetes mellitus, osteoporosis and constipation
was found on the floor in semi-conscious state after straining to have a bowel movement. You could suspect this
episode to be a result of:
A. Spontaneous hip fracture
B. Hyperglycemic reaction
C. Vasovagal (situational) syncope
D. Bowel obstruction

39.16 What is the most frequent side effect of the ACE inhibitors (when treating the increased blood pressure):
A. Edemas of lower extremities
B. Cough
C. Allergy
D. Pain in stomach

39.17 Which statement about the recommended elderly diet is true:


A. It is recommended to eat very fatty and calorific food in a quiet environment
B. The food must be warm, flavorful, calorific enough; it is recommended to eat frequently, regularly, not
alone
C. Double vitamin and iron doses must be in the food
D. It is recommended to eat food with a lesser protein and carbohydrate amount

39.18 Which diet is proper for elderly patient with constipation:


A. Diet with increased protein amount
B. Low calory diet
C. Nutrition through gastric tube
D. 6-8 glasses of fluids, increased fiber amount

39.19 Which symptom is characteristic to aging of the gastrointestinal tract:


A. Reduction in taste sensation
B. Increased blood flow in stomach
C. Increase in gastric secretion
D. Decreased absorption of fat-soluble vitamins

39.20 A factor that promotes the functional capacity of persons with Alzheimer’s disease is:
A. Unrestricted travelling
B. A lot of positive impressions
C. New exciting experiences each day
D. Consistency of caregiving approaches

39.21 Which statement about urinary incontinence in the elderly is the most true:
lsmusis.lsmuni.lt/Klausimai/Spausdinti?Length=0?Kalba=EN&KategorijaId=141&Kalbos_input=EN&Kalbos=EN&KategorijaEn_input=Geriatrics&Kate… 2/10
3/27/2019 LSMUSIS
A. Condition, related to social and hygiene problems
B. Normal part of aging
C. Urinary incontinence does not related to comorbid conditions and medications.
D. Multifactorial syndrome, the intersection of age-related factors, comorbid conditions, medications etc.

39.22 Which are the outcomes of menopause:


A. Ischaemic heart disease, osteoporosis, skin and mucosal disorders, memory and cognitive changes
B. Ischaemic heart disease, uterus cancer, intestinal cancer
C. Thyroid diseases, thrombosis, anorgasmia
D. Falls, syncope, dementia

39.23 Which antidepressants cause the most side effects in older adults:
A. Selective serotinine reabsorption inhibitors
B. Selective serotonine and noradreanaline reabsorption inhibitors
C. St John’s wort preparations
D. Tricyclic antidepressants

39.24 Which sign is most common in dementia syndrome:


A. Urinary incontinence
B. Headache
C. Various somatic complaints
D. Progressively weakening of memory and other cognitive functions

II type tasks. For each numbered item,selct the one lettered heading that is most closely asssciated with it

39.25 How do laboratory tests values change in the elderly:


1 - Creatinine clearance
2 - Fasting plasma glucose level
3 - Serum level of follicle-stimulating hormone
4 - Partial arterial oxygen pressure
A. Increases
B. Decreases
C. Does not change

39.26 Dynamics of the main functions of stomach in aging:


1 - Acid production
2 - Activity in mucus production
3 - Motoric activity
4 - Fermentative activity
5 - Gastrin production
A. Decreases
B. Increases

39.27 Which symptoms and signs are common in the elderly with hypothyrosis and hyperthyrosis?
1 - Constipation
2 - Tachycardia, rhythm disorder
3 - Anorexia, weight loss
4 - Low temperature, cognitive impairment
A. In hypothyrosis
B. In hyperthyrosis

39.28 To which groups belong these used in psychogeriatrics drugs:


1 - Donepezil
2 - Doxepin
3 - Nicergoline
4 - Sertraline
A. Alpha-blocker, increases circulation and metabolism of brain, especially in ischemia
B. Selective serotonine reuptake inhibitor
C. Tricyclic antidepressant
D. Acetylcholinesterase inhibitor

39.29 In older age:

lsmusis.lsmuni.lt/Klausimai/Spausdinti?Length=0?Kalba=EN&KategorijaId=141&Kalbos_input=EN&Kalbos=EN&KategorijaEn_input=Geriatrics&Kate… 3/10
3/27/2019 LSMUSIS
1 - Volume of distribution of lipid-soluble drugs
2 - .Elimination half-time of lipid-soluble drugs
3 - Volume of distribution of water-soluble drugs
4 - Risk of drug intoxication
5 - Liver blood circulation
6 - Body mass
A. Increases
B. Decreases

39.30 Which scales is useful in diagnostics of:


1 - Dementia
2 - Stroke
3 - Depression
4 - Falls
A. GDS, Zung Self-Rating Scale
B. MMSE, Blessed Scale, Clock Drawing Test, IADL
C. MMSE, Barthel Index, Functional Independence Measure
D. Morse Scale

39.31 List which morphological changes in aging respiratory system cause corresponding functional impairment:
1 - Increase of rigidity and decrease of elasticity of chest wall
2 - Diminished mucociliary clearance
3 - Fibrosis and obliteration of capillaries in the lungs
4 - Peribronchial sclerosis, deformation and deviation of bronchi
A. Worsening of rheological characteristics of mucus in bronchi
B. Reduced vital capacity (VC) of lungs
C. Reduced respiration volume, uneven ventilation
D. Hypoxia and hypercapnia

39.32 Which physiological changes in the elderly cause these physical findings:
1 - Pseudohypertension
2 - Orthostatic hypotension
3 - Systolic hypertension
4 - Prominent jugular venous pulsation on the left
A. Reduced baroreflex response
B. Rigid aorta
C. Rigid brachial artery
D. Tortuous ascending aorta compresses the innominate vein

39.33 Which symptoms are characteristic for the elderly with anemia
1 - Microcytic
2 - Macrocytic
3 - Normocytic
A. Ataxia
B. Chronic disease in history
C. Changed taste

39.34 Which drugs can result in these symptoms in elderly people:


1 - Urinary incontinence
2 - Hyperglycemic reaction
3 - Disorientation
4 - Deterioration of peripheral circulation
A. Sedatives
B. Thiazide diuretics
C. Beta-blockers

39.35 With which nonspecific signs may these diseases manifest in the elderly
1 - Parkinson's disease
2 - Heart failure
3 - Thyrotoxicosis
4 - Depression
A. Atrial fibrillation, weight loss, anorexia
lsmusis.lsmuni.lt/Klausimai/Spausdinti?Length=0?Kalba=EN&KategorijaId=141&Kalbos_input=EN&Kalbos=EN&KategorijaEn_input=Geriatrics&Kate… 4/10
3/27/2019 LSMUSIS
B. Apathy, pseudodementia
C. Swallowing impairment, slow speech
D. Dementia, weakness, oedemic syndrome

III type tasks. For each question there is one or more correct answers:
A – if correct answers are 1,2,3
B – if correct answers are 1 and 3
C – if correct answers are 2 and 4
D – if correct answer is 4
E – if correct are all answers above

39.36 Which symptoms describe hypoglycemia of the elderly


1. Weakness
2. Paresthesias
3. Falls
4. Cognitive impairment

39.37 What dysfunctions could you observe in patients with dementia:


1. Memory impairment
2. Agnosia
3. Language disturbancies
4. Apraxia

39.38 The geropharmacology is characterized by a sentence:


1. Except of digoxin the doses of most medicines are greater or does not change.
2. The action of medicines on the aging body is the same as on the young one
3. Drugs make more harm than are useful for the old organism
4. The doses of many medicines are smaller

39.39 What could you recommend for the healthy 66-years-old man who complains of constipation:
1. To increase the amount of fibers in the food
2. To increase the physical activity
3. To increase the amount of liquids in the meals
4. Everyday usage of laxatives

39.40 What symptoms of diabetes may manifest in the elderly?


1. Cognitive impairment
2. Nocturia
3. Weight loss
4. Urinary infection

39.41 Name the factors, which are related to development of pneumonia in the elderly
1. Decreased cough reflex, abnormal alveolar macrophages , mucosal atrophy.
2. Severe comorbid conditions
3. Dysphagis
4. Recurrent infections in the urinary tract

39.42 Which recommendations for the elderly patients with malnutrition are right:
1. To avoid long periods without food during the night (>12 hours)
2. To eat products with high calories and protein amount
3. To have snacks in a social environment, prepare food and eat it with company
4. To increase the eating frequency (snacks)

39.43 Name the cerebral complications of stroke in the elderly


1. Cognitive impairment
2. Bed sores
3. Delirium
4. Urinary tract infection

39.44 What are characteristic features of respiratory system of the elderly:


1. Decreased drainage of bronchi
2. Decreased cough reflex
lsmusis.lsmuni.lt/Klausimai/Spausdinti?Length=0?Kalba=EN&KategorijaId=141&Kalbos_input=EN&Kalbos=EN&KategorijaEn_input=Geriatrics&Kate… 5/10
3/27/2019 LSMUSIS
3. Decreased local and general immune reactivity
4. Increased microcirculation and perfusion of lung tissue

39.45 Which disorders could predispose the falls in the elderly:


1. Musculoskeletal
2. Neurological
3. Cardiovascular
4. Disorders of senses

39.46 Which conditions of the elderly are complicated by acute urinary tract infections more often
1. Influenza
2. Diabetes
3. Hypothyrosis
4. Urinary incontinence

39.47 Which complications can be often caused by diverticulosis in the elderly:


1. Bleeding
2. Peptic ulcer
3. Peritonitis
4. Cholecystitis

IV type tasks. Choose only one best answer

39.48 A 78 years old woman second day after the surgery due to hip fracture became irritated, disoriented, poorly
concentrating attention, developed visual hallucinations, and inadequately behaving. What is diagnosis:
A. Lewy body dementia
B. Mild cognitive impairment
C. Delirium
D. Panic attack

39.49 A 78 years old woman second day after the surgery due to hip fracture became irritated, disoriented, poorly
concentrating attention, developed visual hallucinations, and inadequately behaving. Which scale could be the
most useful evaluating the status of this patient:
A. Barthel Index
B. Geriatric depression scale
C. Confusion assessment method
D. Morse Falls Scale

39.50 A 78 years old woman has kyphosis due to compressive fractures of vertebrae of spinal column. She has
had thyrotoxicosis, and has undergone thyroid gland resection 10 years ago. Menopause since the age of 40. Her
mother died at the age of 96 and at the end of her life she had considerable osteoporosis and multiple bone
fractures.What osteoporosis risk factor can you name for that woman:
A. Heredity
B. Thyrotoxicosis, early menopause
C. Early menopause
D. Heredity, early menopause, thyrotoxicosis

39.51 A 78 years old woman has kyphosis due to compressive fractures of vertebrae of spinal column. She has
had thyrotoxicosis, and has undergone thyroid gland resection 10 years ago. Menopause since the age of 40. Her
mother died at the age of 96 and at the end of her life she had considerable osteoporosis and multiple bone
fractures.What treatment would you choose in this case:
A. Bisphosphonates
B. 1 g of calcium and 1000 UA vit. D daily
C. Anabolic therapy drugs (parathormone, teriparatide)
D. All the measures mentioned above

39.52 A 82-years-old woman with very neglected oral cavity must undergo extraction of the dental roots. She has
diabetes, osteoarthritis and artificial hip joint.Is the prevention with antibiotics needed:
A. No
B. Yes
C. Only if she has prosthetic heart valves

lsmusis.lsmuni.lt/Klausimai/Spausdinti?Length=0?Kalba=EN&KategorijaId=141&Kalbos_input=EN&Kalbos=EN&KategorijaEn_input=Geriatrics&Kate… 6/10
3/27/2019 LSMUSIS
39.53 A 82-years-old woman with very neglected oral cavity must undergo extraction of the dental roots. She has
diabetes, osteoarthritis and artificial hip joint.If you decided to prescribe antibiotics, what antibiotics suit the best in
this situation:
A. Macrolides
B. Aminoglycosides
C. Cephalosporines
D. Fluorochinolons

39.54 The operation of 72-years-old woman is planned. The surgeon thinks, that the women will get to much
excited if the operation and its risks will be discussed with the women herself. He asks for the consent for the
operation by her husband. The husband proposes to get the consent from their son “who is the doctor and knows
better what is about”. Whose consent must obtain the surgeon for the operation:
A. The consent must be obtained from the woman
B. The consent must be obtained from the husband
C. Because the son of the women is a doctor be can give consent for the operation
D. The surgeon must obtain the consent for the operation from the women without mentioning risks of
operation

39.55 A 78-years-old man because of recurrent and persistent stomach ulcer underwent gastric resection
operation. Eventually patient began to feel weakness, appeared paresthesia, ataxia, and mental disorder. Fe
concentration in serum was normal, MCV – 105 fl. What could be the cause of these ailments?
A. Dumping syndrome
B. Megaloblastic anemia
C. Diabetes
D. Iron deficiency anemia

39.56 A 78-years-old man because of recurrent and persistent stomach ulcer underwent gastric resection
operation. Eventually patient began to feel weakness, appeared paresthesia, ataxia, and mental disorder. Fe
concentration in serum was normal, MCV – 105 fl. What tactics of treatment do you choose?
A. Folic acid
B. Parenteral iron preparations
C. Cianocobalamin
D. No treatment is needed, but the man must be followed-up

39.57 A 65-years-old obese woman has hypertension, coronary heart disease and congestive heart failure.
Because of that she received metoprolol, ramipril, nitroglycerin, furosemide. Sudden getting out from her bed she
fall down without loss of consciousness.Preliminary diagnosis is:
A. Pulmonary thromboembolism
B. Orthostatic hypotension
C. Syncope
D. Stroke

39.58 A 65-years-old obese woman has hypertension, coronary heart disease and congestive heart failure.
Because of that she received metoprolol, ramipril, nitroglycerin, furosemide. Sudden getting out from her bed she
fall down without loss of consciousness.What could most likely have caused this situation?
A. Stroke
B. Increased blood pressure
C. Medicaments
D. Joint desease

39.59 A 79 years man with ischemic heart disease, presenting with angina pectoris and atrial fibrillation, diabetes
mellitus with complications and prescribed insulin, constipation; benign prostate hyperplasia, after 2 months after
wife’s death started complaining with insomnia, anxiety, impaired senses, and decreased appetite. His daughter,
who lives together, noticed apathy, unwillingness to go out, to take care about home and himself. What is the most
probable diagnosis:
A. Dementia
B. Depression
C. Delirium
D. Bipolar disorder

39.60 A 79 years man with ischemic heart disease, presenting with angina pectoris and atrial fibrillation, diabetes
mellitus with complications and prescribed insulin, constipation; benign prostate hyperplasia, after 2 months after

lsmusis.lsmuni.lt/Klausimai/Spausdinti?Length=0?Kalba=EN&KategorijaId=141&Kalbos_input=EN&Kalbos=EN&KategorijaEn_input=Geriatrics&Kate… 7/10
3/27/2019 LSMUSIS
wife’s death started complaining with insomnia, anxiety, impaired senses, and decreased appetite. His daughter,
who lives together, noticed apathy, unwillingness to go out, to take care about home and himself.What treatment is
needed:
A. Amitriptyline
B. Duloxetin
C. Metformin
D. Diazepam

39.61 A 83-year-old inhabitant of home of the elderly who has smoked for 30 years and worked as welder, began
to feel weakness, dyspnea, heaviness in the chest, to cough without sputum. Patient is sluggish, lethargic,
temperature is normal, breath rate 26 times/min., SaO2 90 percent. What diagnosis is most probable:
A. Acute bronchitis
B. Exacerbation of chronic bronchitis
C. Catarrh of superior respiratory ways
D. Acute pneumonia

39.62 A 83-year-old inhabitant of home of the elderly who has smoked for 30 years and worked as welder, began
to feel weakness, dyspnea, heaviness in the chest, to cough without sputum. Patient is sluggish, lethargic,
temperature is normal, breath rate 26 times/min., SaO2 90 percent.Complex treatment for 2 week was not
effective. The non-productive cough, dyspnea and weakness had progressed, the fever dragged out, arthralgias
and symmetrical skin rash developed, the patient has lost 11 kg in weight. What is the most probable diagnosis:
A. Lung abscess
B. Lung carcinoma
C. Allergy
D. Fibrosive alveolitis

39.63 A 83-year-old inhabitant of home of the elderly who has smoked for 30 years and worked as welder, began
to feel weakness, dyspnea, heaviness in the chest, to cough without sputum. Complex treatment for 2 week was
not effective. The non-productive cough, dyspnea and weakness had progressed, the fever dragged out,
arthralgias and symmetrical skin rash developed, the patient has lost 11 kg in weight. What tests must be
performed?
A. Serum immunoglobulins
B. CT
C. CT, bronchoscopy, biopsy
D. Bronchoscopy

39.64 When examining a 68-year-woman you detect on sacrum epidermal and dermal ulceration with surrounding
erythema. You could suspect this to be associated most probably with:
A. Herpes zoster
B. Dermatitis
C. Decubitus ulcer
D. Scabies

39.65 When examining a 68-year-woman you detect on sacrum epidermal and dermal ulceration with surrounding
erythema.What treatment of this lesion will be?
A. Wound revision, disinfection, dressing
B. Systemic antibiotics
C. Dressing with alcohol containing disinfectants
D. Ointment with corticsteroids

39.66 A 69-years-old woman has complains on changed urination: she has increased frequency (8-10 times per
day) and nocturia (2-3 times per night), urgency, sometimes incontinence.What reason of changed urination:
A. Stress urinary incontinence
B. Functional urinary incontinence
C. Overactive bladder
D. Cystitis

39.67 A 69-years-old woman has complaints on changed urination: she has increased frequency (8-10 times per
day) and nocturia (2-3 times per night), urgency, sometimes incontinence.What treatment you’ll recommend?
A. Anticholinergics
B. Antidepressants
C. Surgery

lsmusis.lsmuni.lt/Klausimai/Spausdinti?Length=0?Kalba=EN&KategorijaId=141&Kalbos_input=EN&Kalbos=EN&KategorijaEn_input=Geriatrics&Kate… 8/10
3/27/2019 LSMUSIS
D. Antibiotics

39.68 A 68 –years-old female patient has complained that several days ago after catching cold her temperature
has raised to 37.5-37.8 C, frequent, painful urination and back pain began. She has 15 yrs history of diabetes.
Similar symptoms occur during 5 last years. After taking ampicillin for two days the health did not improve. Protein
in urine 0.33 g/l, leuc. 30-35, er. 3-5 in vision field, nit (+), bact. (+), CRP 76 mg/l. What is the most probable
diagnosis:
A. Acute glomerulonephritis
B. Exacerbation of chronic glomerulonephritis
C. Acute pyelonephritis
D. Exacerbation of chronic pyelonephritis

39.69 A 68 –years-old female patient has complained that several days ago after catching cold her temperature
has raised to 37.5-37.8 C, frequent, painful urination and back pain began. She has 15 yrs history of diabetes.
Similar symptoms occur during 5 last years. After taking ampicillin for two days the health did not improve. Protein
in urine 0.33 g/l, leuc. 30-35, er. 3-5 in vision field, nit (+), bact. (+), CRP 76 mg/l. What should be the tactics of the
doctor?
A. To repeat the urine analysis
B. To take urine for bacterial culture and wait for reply, then according to results to begin antibacterial
treatment
C. To take urine for bacterial culture growth and to prescribe ciprofloxacin or cefuroxime.
D. To prescribe gentamycin immediately

39.70 A 71-years-old woman who is suffering from third functional class heart failure feels pain behind sternum,
increased shortness of breath during exercise; faeces every four days, recurrent abdominal pain, bloating. Patient
is diagnosed with constipation.What should the physician recommend:
A. Use of fibrous foods
B. Use more than 2 litres fluid a day
C. Increasing physical activity
D. Reduce the dose of diuretics

39.71 A 71-years-old woman who is suffering from third functional class heart failure feels pain behind sternum,
increased shortness of breath during exercise; faeces every four days, recurrent abdominal pain, bloating. Patient
is diagnosed with constipation.What complications can cause constipation?
A. Appendicitis
B. Fecal incontinence
C. Cholecystitis
D. Pancreatitis
E. Gastritis

39.72 A 84 years old woman was hospitalised because of pneumonia and bed sores. BMI – 26.5 kg/m2, albumin
conc. – 33 g/l, abs. lymphocyte count – 0.8x109 c/l, MNA score – 17, NRS 2002 score – 3. What is the nutritional
status of this patient:
A. Overweight
B. Sarcopenia
C. Kvashiorcor
D. Malnutrition

39.73 A 84 years old woman was hospitalised because of pneumonia and bed sores. BMI – 26.5 kg/m2, albumin
conc. – 33 g/l, abs. lymphocyte count – 0.8x109 c/l, MNA score – 17, NRS 2002 score – 3.What should be the
consequent tactics for this nutritional condition?
A. Fortified food, oral nutritional supplements
B. Thick pureed food and thickened liquids
C. Parenteral nutrition
D. Weight correction, immunonutrition

39.1 - B 39.2 - D 39.3 - C 39.4 - D 39.5 - A 39.6 - B 39.7 - A 39.8 - D


39.9 - A 39.10 - B 39.11 - C 39.12 - A 39.13 - A 39.14 - A 39.15 - C 39.16 - B
39.17 - B 39.18 - D 39.19 - A 39.20 - D 39.21 - D 39.22 - A 39.23 - D 39.24 - D
39.25 39.26 39.27 39.28 39.29 39.30 39.31 39.32
1-B 1-A 1-A 1-D 1-A 1-B 1-B 1-C

lsmusis.lsmuni.lt/Klausimai/Spausdinti?Length=0?Kalba=EN&KategorijaId=141&Kalbos_input=EN&Kalbos=EN&KategorijaEn_input=Geriatrics&Kate… 9/10
3/27/2019 LSMUSIS
2-C 2-B 2-B 2-C 2-A 2-C 2-A 2-A
3-A 3-A 3-B 3-A 3-B 3-A 3-D 3-B
4-B 4-A 4-A 4-B 4-A 4-D 4-C 4-D
5-B 5-B
6-B
39.33 39.34 39.35
1-C 1-B 1-C
2-A 2-B 2-D
3-B 3-A 3-A
4-C 4-B
39.36 39.37 39.38 39.39 39.40 39.41 39.42 39.43
1 1 4 1 1 1 1 1
2 2 2 2 2 2 3
3 3 3 3 3 3
4 4 4 4
39.44 39.45 39.46 39.47
1 1 2 1
2 2 4 3
3 3
4
39.48 - C 39.49 - C 39.50 - D 39.51 - D 39.52 - B 39.53 - C 39.54 - A 39.55 - B
39.56 - C 39.57 - B 39.58 - C 39.59 - B 39.60 - B 39.61 - D 39.62 - B 39.63 - C
39.64 - C 39.65 - A 39.66 - C 39.67 - A 39.68 - D 39.69 - C 39.70 - A 39.71 - B
39.72 - D 39.73 - A

lsmusis.lsmuni.lt/Klausimai/Spausdinti?Length=0?Kalba=EN&KategorijaId=141&Kalbos_input=EN&Kalbos=EN&KategorijaEn_input=Geriatrics&Kat… 10/10
3/27/2019 LSMUSIS

Hematology
I type tasks. Choose only one best answer

3.1 Which laboratory test is of greatest importance to evaluate total body iron stores:
A. Serum ferritin
B. Serum iron
C. Serum transferin
D. Free red blood cell protoporphyrin
E. Marrow iron stores

3.2 What is the earliest sign of the effective treatment of iron deficiency anemia
A. Decrease of fatigue and dyspnea
B. Nausea, constipation, looseness of stools
C. Serum iron normalization
D. Increase in hemoglobin concentration
E. Reticulocytosis on the 5 - 7-t.h day from begining of the treatment

3.3 What are the most common causes of iron deficiency anemia for women:
A. Small bowel inflammation
B. Polymenorrhea, metrorrhagia
C. Gastric surgery
D. Gastrointestinal bleeding (peptic ulcers, neoplasmas etc.)

3.4 Which statement about the treatment of iron deficiency anemia is incorrect:
A. Iron ions are "bound" by antacids and their absorbtion is reduced
B. The administration of iron parenterally is more effective than oral iron preparations for the treatment of
iron deficiency anemia
C. Tetracyclines reduce iron ions absorbtion
D. Serum ferritin level bocomes normal only after bone marrow iron stores are replenished.

3.5 Which statement about megaloblastic anemia caused by folate deficiency is incorrect:
A. May develop after 3 months without complete use of folic acid
B. The dominant symptom is caused by nerves demyelination
C. Diagnosis is supported by plasma folate concentration evaluation
D. Treatment: with per oral folic acid

3.6 The feature of severe aplastic anaemia ais:


A. > 50% of normal cellularity in the bone marrow
B. The platelets count > 50000/ µl
C. Hemoglobin level > 80 g/l
D. Absolut neutrophils count < 500 µl

3.7 Which medication does inhibit platelet aggregation:


A. Acetylsalicylic acid
B. Warfarin
C. Prednisolone
D. Rituximab

3.8 How long does blood (plasma) coagulation takes in intrinsic coagulation pathway:
A. 15-20 min
B. 7-10 min
C. 1,5-2 min
D. 12-15 min.

3.9 Which statement is incorrect? Prolonged bleeding means:


A. Thrombocytopenia
B. Thrombocytopathia
C. Disorders in coagulation system
D. Iron deficiency

3.10 Which statement is incorrect of prothrombin time:


A. Provides a measure of the extrinsic coagulation pathway
lsmusis.lsmuni.lt/Klausimai/Spausdinti?Length=0?Kalba=EN&KategorijaId=105&Kalbos_input=EN&Kalbos=EN&KategorijaEn_input=Hematology&Ka… 1/25
3/27/2019 LSMUSIS
B. Determinate the activity of factors II, V. VII, X
C. Can be used by monitoring of coumarins administration
D. Measures factor XIII activity

3.11 Which statement is incorrect? Deep thrombocytopenia may be caused by:


A. Acute leukemia
B. Agranulocytosis
C. Aplastic anemia
D. Megaloblastic anemia

3.12 Which statement is incorrect? Autoimmune thrombocytopenia may be caused by treated with:
A. Prednisolone
B. Rituximab
C. Eltrombopag
D. Hydroxicarbamide

3.13 Which statement incorrect? Coagulopathia are characterised by:


A. Delayed bleeding after trauma (hours or days after)
B. Deep muscle and subcutaneus hemorrhage
C. Visceral hemmorrhage
D. Good response to local pressure

3.14 Which complex of tests proves hemophilia:


A. Normal activated partial thromboplastin time, prolonged prothrombin tine, normal bleeding time
B. Prolonged activated partial thromboplastin time, normal prothrombin, and bleeding times
C. Prolong activated partial time, normal prothrombin and prolonged bleeding times
D. Prolonged activated partial thromboplastin time, prolonged prothrombin time.

3.15 Which syndrome is not common for aplastic anaemia:


A. Thrombocytopenic syndrome
B. Anaemic syndrome
C. Neutropenic syndrome
D. Haepatosplenomegaly

3.16 Which statement is incorrect? In deficiency of factor VII is found:


A. Normal bleeding time
B. Normal activated partical thromboplastin time
C. Normal prothrombin time
D. Prolonged coagulation time

3.17 What is the cause of immune thrombocytopenic purpura:


A. Platelet destruction caused by antiplatelet antibodies
B. Platelet phagocytosis in bone marrow
C. Folate deficiency
D. Bone marrow failure

3.18 Which statement is incorrect? Bleeding time is prolonged in:


A. Haemophilia.
B. Thrombocytopenia
C. Thrombocytopathia
D. Aspirin ingestion

3.19 Which statement is incorrect? Activated partial thromboplastin time:


A. Measures intrinsic coagulation pathway
B. Is more sensitive than blood coagulation time test
C. Measures activity of factors VII and XIII
D. Measures activity of factors XII, XI, IX and VIII

3.20 How much one percent of VIII coagulation factor for one kilogram patient‘s weight elevate of VIII coagulation
factor in serum plasma:
A. 1%
B. 2%
C. 4%
lsmusis.lsmuni.lt/Klausimai/Spausdinti?Length=0?Kalba=EN&KategorijaId=105&Kalbos_input=EN&Kalbos=EN&KategorijaEn_input=Hematology&Ka… 2/25
3/27/2019 LSMUSIS
D. 5%

3.21 Which statement is incorrect? Idiopathic thrombocytopenic purpura is characterised by:


A. Severe thrombocytopenia
B. Normal platelet function
C. Prolonged bleeding time
D. Prolonged activated partial tromboplastin time

3.22 Which statment is incorrect? Idiopathic thrombocytopenic purpura is characterised by:


A. Thrombocytopenia
B. Low protrombin index
C. Prolonged bleeding time
D. Normal or increased number of megakariocytes

3.23 Which statment is incorrect? Coagulopathies are characterised by:


A. Muscle hematomas
B. Bleeding into joints
C. Petechias
D. Bleeding into cavities

3.24 Which coagulation factor is lacking in hemophilia A;


A. IX faktor
B. VII faktor
C. XI faktor
D. VIII faktor

3.25 What signs of bleeding are not characteristic of vitamin K coagulopathy:


A. Bleeding from the mucous membrane
B. Skin hematomas
C. Hematuria
D. Hemarthrosis

3.26 Which factor does not take part in extrinsic coagulation pathway:
A. X
B. IX
C. VII
D. II

3.27 Which statement is incorrect? Primary hemostasis is characterised by:


A. Platelet number
B. Platelet function
C. Bleeding time
D. Blood coagulation time

3.28 Which statement is incorrect? If platelet number is lower than 50x109/l the next clinical manifestations
appear:
A. Petechias
B. Ecchymoses (bruises)
C. Bleeding after dental extration
D. Telangiectasia

3.29 Which statement is incorrect? Prothrombin time shows activity of these factors:
A. II
B. VII
C. VIII
D. X

3.30 Which statement is incorrect? Clinical manifestations in the case of acute idiopathic thrombocytopenic
purpura:
A. Petechias, ecchymoses
B. Muccous membranes bleeding
C. Splenomegaly
D. Thrombocytopenia lower than 20x109/1
lsmusis.lsmuni.lt/Klausimai/Spausdinti?Length=0?Kalba=EN&KategorijaId=105&Kalbos_input=EN&Kalbos=EN&KategorijaEn_input=Hematology&Ka… 3/25
3/27/2019 LSMUSIS
3.31 What is the first line treatment of acute idiopathic thrombocytopenic purpura:
A. Splenectomy
B. Glucocorticoids
C. Immunosupressive therapy
D. Platelet transfusions

3.32 What disorder causes difficulties in differential diagnosis of idiopathic trombocytopenic purpura:
A. Aplastic anemia
B. Acute leukemias
C. Megaloblastic anemias
D. Drug induced autoimune thrombocytopenias

3.33 Idiopathic thrombocytopenic purpura should be differentiated with:


A. Hemophilia
B. Haemorrhagic vasculitis
C. Secondary thrombocytopenias
D. Scurvy

3.34 Which statement is incorrect? Severe thrombocytopenia is caused by:


A. Idiopathic thrombocytopenic purpura
B. Aplastic anemia
C. Megaloblastic anemia
D. Severe iron deficiency anemia

3.35 Which statement incorrect? Hemophilia is characterised


A. Large skin and subcutaneous hematomas
B. Petechias, ecchymoses
C. Muscle hematomas
D. Hemarthroses

3.36 Which statement is incorrect? Hemophilia is treated by:


A. Clotting factors replacement
B. Analgesics
C. Platelet transfusions
D. Prevention therapy

3.37 The most common causes of vit. K deficiency


A. Burns
B. Sepsis
C. Heparin administration
D. Coumarin administration

3.38 How hemophilia is inherited:


A. Inherited in an Y-linked pattern
B. Autosomal recessive pattern
C. Autosomal dominant pattern
D. Inherited in an X-linked pattern

3.39 What is not characteristic of anemic syndrome:


A. Tachycardia
B. Mucosal pallor
C. Petechiae
D. Headache

3.40 The principles of treatment of chronic idiopathic thrombocytopenic purpura are:


A. Gliucocorticoids
B. Thrombopoietin receptor agonists
C. Splenectomy
D. All above mentioned means

3.41 Which statement is incorrect? Severe thrombocytopenia is caused by:


A. Bone marrow carcinomatosis
B. Hypersplenism
lsmusis.lsmuni.lt/Klausimai/Spausdinti?Length=0?Kalba=EN&KategorijaId=105&Kalbos_input=EN&Kalbos=EN&KategorijaEn_input=Hematology&Ka… 4/25
3/27/2019 LSMUSIS
C. Megaloblastic anemia
D. Polycythemia vera

3.42 Who is a candidate for autologous stem cell transplantation in multiple myeloma disease:
A. Patient older then 70 years
B. Patient under 65 years
C. Patient under 18 years
D. Paient in any age old

3.43 What is indication for radiotherapy in multiple myeloma:


A. When patient intolerates chemotherapy
B. Patient with osteoporosis
C. Hypercalcaemia syndrome
D. Patient with pain in the bone destruction sites

3.44 Which factor does not belong to intrinsic coagulation pathway (without “common trunk“):
A. XI
B. X
C. IX
D. VIII

3.45 The morphologic substrate of Hodgkin's lymphoma:


A. Fibroblasts
B. Reticulum cells
C. Lymphocytes
D. Reed-Stenberg cells

3.46 Which test is the most important in confirming Hodgkin's lymphoma:


A. Peripheral blood examination
B. Bone marrow aspiration
C. Lymphnode biopsy
D. Lymphnode punction

3.47 The diagnosis of multiple myeloma is confirmed with:


A. Serum M component
B. Peripheral blood examination
C. Lymphnode biopsy
D. Bone marrow examination

3.48 What disorder is characterised by Ph+ chromosome:


A. Myelofibrosis
B. Chronic myelogenous leukemia
C. Polycythaemia vera
D. Hodgkin's lymphoma

3.49 The main difference between leukemias and lymphomas


A. Different characteristics of malignant cells
B. Different number of blast cells
C. The begining of malignancy is in the different organs
D. Prognosis and outcome

3.50 The characteristics of lymphnodes in Hodgkin's lymphoma


A. Elastic, tender, no connection with derma
B. Tender, soft, no conglomerate
C. Elastic, hard, painless, mobile
D. Hard, painless, with necrosis and fistulles

3.51 When will you start the treatment of chronic lymphocytic leukemia:
A. At once after diagnosis was made
B. When leucocyte count is above 10xl09/1
C. When hematologic decompensation is present
D. When fever and features of bronchitis appear

lsmusis.lsmuni.lt/Klausimai/Spausdinti?Length=0?Kalba=EN&KategorijaId=105&Kalbos_input=EN&Kalbos=EN&KategorijaEn_input=Hematology&Ka… 5/25
3/27/2019 LSMUSIS
3.52 Differences between ABO red cell antigen system and other systems?
A. ABO has the highest number of antigens
B. ABO has not only antigens, but also antibodies
C. BO antigens are the most immunogenic

3.53 How will you start treatment of chronic myelogenous leukemia:


A. Hydroxyurea
B. Cyclophosphamide
C. Interferon
D. Tyrosine-kinase inhibitor

3.54 How many will you find in bone marrow of a patient in complete remission of acute myeloid leukaemia:
A. 1-2%
B. Not more than 5%
C. Not more than 10%
D. Not more than 15%

3.55 The characteristics of multiple myeloma:


A. Benign lymphoproliferative disorder with serum M component
B. Malignant myeloproliferative disorder with anemia and increased plasma cells number in bone marrow
C. Malignant plasma cells tumor with anemia, osteoporosis and lytic bone lesions
D. B lympocyte benign tumor with 3 - 5% of plasma cells in bone marrow

3.56 What line of cells is increased in bone marrow of patients with polycythaemia vera:
A. Myeloblasts
B. Erythroblasts
C. Megakariocytes
D. Megaloblasts

3.57 What is detected in serum protein electrophoresis asssay during multiple myeloma disease:
A. Hemoglobin
B. Monoclonal protein
C. Plasma cells
D. Amino acids

3.58 Which disorder can be completely cured:


A. Hodgkin's lymphoma
B. Chronic lymphocytic laukemia
C. Chronic myeloid leukemia
D. Polycythaemia vera

3.59 Which disorder can be treated with „wait and watch“ approach in the early disease stages:
A. Chronic myeloid leukemia
B. Myelofibrosis
C. Acute myeloid leukemia
D. Acute lymphocytic leukemia

3.60 Which statement about myelofibrosis is wrong:


A. Anaemia at the begining of the disease is rare event
B. Anaemia is due to inefective erythropoesis, shorthening of erythrocytes life, increased erythrocytes
storage in the spleen
C. Frequently leukocythocis is found
D. Leukopenia is less frequently found

3.61 Which statement about myelofibrosis is wrong:


A. Circulating blasts in peripheral blood can be found
B. Can be found left shift in peripheral blood formula
C. Blast count in peripheral blood doesn‘t afect prognosis
D. CD34 cells are increased in peripheral blood

3.62 Which statement about myelofibrosis is wrong:


A. . Platelets number in early stages are normal or increased
B. Trombocytopenia is characteristic for late stages of disease
lsmusis.lsmuni.lt/Klausimai/Spausdinti?Length=0?Kalba=EN&KategorijaId=105&Kalbos_input=EN&Kalbos=EN&KategorijaEn_input=Hematology&Ka… 6/25
3/27/2019 LSMUSIS
C. The bone marrow fibrosis doesn‘t depend on fibroblasts
D. Extramedullary hematopoiesis sometimes can be found in different organs

3.63 Which statement about polycythemia vera is wrong:


A. Decreased capacity to bind vitamin B12
B. Thrombocythosis can be found
C. Frequent splenomegaly
D. During final stages of disease bone marrow fibrosis and anaemia developes

3.64 Which statement about polycythemia vera is wrong:


A. Main cause of death – vascular complications
B. Median survival while treating with phlebotomies only is 10-12 years
C. Untreated patients experience death in average during first two years
D. In case of marked thrombocythosis treatment is started from phlebotomies

3.65 Which statement about polycythemia vera is wrong:


A. With the help of phlebotomies hematocrit and hemoglobin values should be reduced till lower limits of
normal
B. Mortality and morbidity of untreated patients postoperatively is increased 4-5 times comparing with
treated
C. Thrombocythosis is indication for myelosupresive treatment
D. Disease is treated with cytosar, rubomycin

3.66 Which diagnostic sign of chronic myeloid leukemia is wrong:


A. Hepatomegaly
B. Granulocytic leukocythosis
C. Low activity of alkaline phosphatase in granulocytes
D. Splenomegaly

3.67 Which statement about chronic myeloid leukemia is wrong:


A. Chronic phase of disease is effectively treated with drugs
B. Diagnosis is confirmed when BCR/ABL transcript is found
C. Blastic phase of disease is effectively treated with polychemotherapy schemes
D. Diagnosis is confirmed when Philadelphia chromosome is found

3.68 Which statement about chronic lymphocytic leukemia is wrong:


A. Often developes hypogamaglobulinaemia
B. Patients can experience disturbances of gastrointestinal and urogenital systems
C. Isolated splenomegaly can be easily controlled with radiation therapy
D. Drug of choice for treatment is tyrosine kinase inhibitors.

3.69 Which statement about chronic lymphocytic leukemia is wrong:


A. There is granulopoesis hyperplasia in the bone marrow
B. Age mediana of patients is 60 years
C. Glucocorticoids express strong lympholytic effect and they do not suppress normal hematopoesis
D. Signs of disease can be determined by cancerous infiltration of lymph nodes, bone marrow, liver, spleen

3.70 Which statement about non Hodgkin lymphoma is wrong:


A. There are about 30 different types of it
B. Necessary to perform all investigations in order to establish disease stage
C. Can be completely cured
D. During 2 – 3 months of disease 30-50% of cases develope acute lymphocytic leukemia

3.71 Which statement about myelofibrosis is wrong:


A. Glucocorticoids are administered if there are signs of marked ineffective erythropoesis
B. Glucocorticoids can potentiate effectiveness of anabolics
C. If hepatomegaly persists radiation is the choice of treatment
D. Imunomodulatory medications are used to reduced anemia

3.72 Which statement about myelofibrosis is wrong:


A. Spleen is increased in almost all patients
B. Frequent hepatomegaly
C. Extramedullary hematopoiesis occur only in the spleen
lsmusis.lsmuni.lt/Klausimai/Spausdinti?Length=0?Kalba=EN&KategorijaId=105&Kalbos_input=EN&Kalbos=EN&KategorijaEn_input=Hematology&Ka… 7/25
3/27/2019 LSMUSIS
D. In rare cases lymphadenopatia, icterus, ascites, bone pain can develope

3.73 Which statement about polycythemia vera is wrong:


A. Flebotomies usually are the first line treatment
B. During phlebotomies iron substitution therapy must be administered
C. P32 is only recommended for patients over 80 years of age without effective treatment
D. If treatment with hydroxyurea is ineffective, JAK2 inhibitors are the second line treatment

3.74 Which statement about polycythemia vera is wrong:


A. All hemotopoetic lines hyperplasia is found in bone marrow
B. Fibrotic focuses can occur in central parts of bone marrow
C. Splenomegaly can occur
D. Epistaxis and gastrointestinal bleeding never occur

3.75 Which statement about chronic myeloleukemia is wrong:


A. It is difficult to differenciate from viral type of leukemoidic reaction
B. In rare cases it is difficult to differentiate from mielofibrosis.
C. Chronic myeloleukemia is treated with tyrosine kinase inhibitors
D. It is impossible to induce cytogenetic remission

3.76 Mieloid tissue does not produce:


A. Erytrocytes
B. Platelates
C. Lymphocytes
D. Granulocytes

3.77 Which statement about chronic myeloleukemia is wrong:


A. Bone marrow is hipercellular, a count of young granulocytes forms is increased
B. Thrombocytosis is found in half of all CML cases at disease onset
C. A count of myeloblasts in bone marrow is more than 30 %
D. Initial symptoms are anemia, splenomegaly, hypercatabolism

3.78 Which statement about chronic lymphocytic leukemia is wrong:


A. Treatment begins when clinical hematological decompensation appears
B. Frequent blastic transformation
C. Disease diagnosis is confirmed by flow cytometry
D. Bone marrow is hypercellular. Lymphocytes > 30%

3.79 Which statement about chronic lymphocytic leukemia is wrong:


A. Most frequent cause of death - infection
B. Most frequent cause of death - infection
C. Autoimmune hemolysis may develop
D. During progression of the disease anemia, thrombocytopenia, granulocytopenia can develop

3.80 Which statement about chronic lymphocytic leukemia is wrong:


A. It is often diagnosed during accidental examination of peripheral blood
B. Disease diagnosis is confirmed by flow cytometry
C. Disease diagnosis is confirmed by neutophilic alkaline phosphatase in the peripheral blood
D. Gumprecht shadows are found in the peripheral blood

3.81 What can’t be diagnosed in case of lymphogranulomatosis:


A. Lympadenopathy
B. Fever
C. Jaundice
D. Absolute lymphocytosis in blood

3.82 Which statement about lymphogranulomatosis is wrong:


A. The disease is diagnosed after biopsy of the enlarged lymph nodes
B. Frequent infections
C. There are three stages of the disease
D. About in 10% of cases the disease begins in extralymphic organs

3.83 Which statement about the possibility of splenomegaly in case of following diseases is wrong:
lsmusis.lsmuni.lt/Klausimai/Spausdinti?Length=0?Kalba=EN&KategorijaId=105&Kalbos_input=EN&Kalbos=EN&KategorijaEn_input=Hematology&Ka… 8/25
3/27/2019 LSMUSIS
A. Hodgkin lymphoma
B. Agranulocytosis
C. Chronic lymphocytic leukemia
D. Acute lymphocytic leukemia

3.84 Blasts in peripheral blood are found in case of:


A. Acute leukemia
B. Chronic leukemia
C. Myelofibrosis
D. In all mentioned cases

3.85 Pancytopenia is diagnosed in case of:


A. Acute leukemia
B. Polycythemia
C. Myelofibrosis
D. In all mentioned cases

3.86 Which statement about myelofibrosis is wrong:


A. Osteosclerosis is presented also trough the enlarged bone tissue density, which can be estimated by X-
ray exposure
B. Median survival is 2 years
C. A part of the patients lives up to 15 years
D. Degree of splenomegalia is very important for the prognosis of the disease

3.87 Which statement about myelofibrosis is wrong:


A. Possible complications are gout and port hypertension
B. In case of anemia chemotherapy with „7+3“ protocol is administered
C. If the patients is young treatment with HSCT could be the choice
D. The patients survival is worse if he is in high risk group

3.88 Which statement about myelofibrosis is wrong:


A. Anemia is treated by erythrocyte mass transfusion
B. It is always useful to administer radiation therapy in case of splenomegaly
C. Androgens are effective only for 20% of the patients
D. To reduce splenomegaly JAK2 inhibitors are effective

3.89 Which statement about polycythemia is wrong:


A. Splenomegalia, enlarged production of erythrocytes, thrompbocytes and granulocytes
B. High amount of erythropoietin
C. The symptoms are caused by high plasma capacity and blood viscosity
D. Thrombocytosis and/or hemorrhages can appear

3.90 Which statement about polycythemia is wrong:


A. Haemorrhages can be caused by thrombocytes defects
B. The disease can end in pancytopenia
C. Itching especially after hot bath is often
D. Hipouricemia is diagnosed in 25–30% of the patients

3.91 Which statement about polycythemia is wrong:


A. ESR is often enlarged
B. Leucocytosis is often diagnosed
C. Absolute basophilia can occur
D. JAK2 mutation is found

3.92 Which statement about chronic myeloid leukemia is wrong:


A. Accelerated phase of the disease lasts 3-6 months before blastic phase of the disease
B. Absolute lymphocytosis is found in peripheral blood
C. Disease ends in acute leucosis
D. Switching to blastic phase can be fast and sudden (a few weeks)

3.93 What can’t be diagnosed in bone marrow in case of chronic myeloid leukemia:
A. Disordered maturation of erythrocytes
B. Increased amount of adipose tissue
lsmusis.lsmuni.lt/Klausimai/Spausdinti?Length=0?Kalba=EN&KategorijaId=105&Kalbos_input=EN&Kalbos=EN&KategorijaEn_input=Hematology&Ka… 9/25
3/27/2019 LSMUSIS
C. Normal or slightly lower amount of lymphocytes
D. Suppressed erythropoesis

3.94 What can help acute leukemia to differentiate from chronic myeloid leukemia blastic crisis?
A. Anamnesis and level of splenomegaly
B. Bone marrow aspiration
C. Hiatus leucaemicus
D. Count of blasts

3.95 Which test is the most significant to diagnose chronic lymphocytic leukemia?
A. Flow cytometry
B. Lymph node biopsy
C. Lymph node aspiration
D. Blood count test

3.96 Which statement about malignant lymphoma is wrong?


A. Fever, sweating, loss of weight are common symptoms
B. Peripheral lymph nodes are enlarged (Hodgkin‘s lymphoma) in many cases
C. Sometimes only abdominal lymph nodes are enlarged
D. In 60 % of cases Hodgkin‘s lymphoma is diagnosed.

3.97 Which statement about malignant lymphoma is wrong?


A. Fever is frequent symptom
B. Pruritus is poor prognostic sign
C. Alcohol can provoke pain in lymph node area
D. V. Cava compression can be manifestation of disease

3.98 Which statement about malignant lymphoma is wrong?


A. Sometimes diagnostic laparatomy is required
B. Disease can not affect only spleen
C. Any lymph node, which does not decrease during 6 months period and are bigger than 1 cm, must be
bioptated.
D. Adrenal function is normal for most patients (until treatment).

3.99 The importance of the Rh antigen system?


A. Has not only antigens, but also antibodies.
B. Rh D antigen is the most immunogenic.
C. Has the highest number of antigens.

3.100 Erythroblastosis can occur in cases of:


A. Acute leukemia
B. Myeloproliferative diseases
C. Acute massive bleeding
D. In all mentioned cases.

3.101 What is the meaning of a positive indirect Coombs test for antibodie screening?
A. IgG antibodies against clinically important antigens.
B. Anti-D antibodies.
C. Anti-A and/or anti-B antibodies.

3.102 Will you transfuse red blood cells in the situations below?
A. Hb 78 g/l, no complaints.
B. Hb 110 g/l, slight general weakness.
C. Hb 82 g/l, heart rate 110/min., weakness.

3.103 In a patient with chronic blood loss and increased erythropoietin stimulation, the bone marrow will probably
show:
A. Increased myeloid :erythroid precursors ratio
B. Erythroblasts hyperplasia
C. Erythroblasts hypoplasia
D. Increased number of fat cells

3.104 Urgent actions in a case of acute hemolytic transfusion reaction:


lsmusis.lsmuni.lt/Klausimai/Spausdinti?Length=0?Kalba=EN&KategorijaId=105&Kalbos_input=EN&Kalbos=EN&KategorijaEn_input=Hematology&K… 10/25
3/27/2019 LSMUSIS
A. 0,9% NaCl solution and diuretics.
B. Corticosteroids and 0,9% NaCl solution.
C. 0,9% NaCl solution and hemodialysis.

3.105 Myeloid : erythroid precursors ratio 9:1 in the bone marrow is:
A. Normal for adults
B. Decreased
C. Increased
D. Normal for infants

3.106 Which statement about iron deficiency anemia is wrong:


A. Feritin concentration in the plasma is decreased
B. Anemia is hiperchromic macrocytic
C. The cause of it can be chronic long lasting bleeding
D. While treating You must restore body iron reserves

3.107 In hypersplenism one would expect to find:


A. Thrombocytosis
B. Bone marrow hypoplasia
C. Bone marrow hyperplasia
D. Leukocytosis

3.108 Anemia due to vitamin B12 deficiency is:


A. Normochromic normocytic
B. Hiperchromic makrocytic
C. Hipochromic mikrocytic
D. Hipochromic makrocytic

3.109 Which statement about reticulocytes is wrong:


A. This cell belongs to erythroid cells lineage
B. In normal blood they are till 2%
C. Number increased in aplastic anemia
D. Number increased in autoimmune hemolytic anemia

3.110 The most common symptom of delayed haemolytic transfusion:


A. Renal insufficiency.
B. Anaemia.
C. Jaundice and dark urine.

3.111 What causes relative erythrocytosis:


A. Chronic lung diseases
B. High-altitude
C. High oxygen-affinity hemoglobin
D. Dehidratation

3.112 Haptoglobin in plasma may become depleted in:


A. Inflammatory conditions
B. Acute hemolytic anemia
C. Infectious diseases
D. Kidney diseases

3.113 A leukocyte count and differential in a 35-year old white man was 5.2x109/l. There were 5% neutrophils,
73% lymphocytes, 20% monocytes and 2% eosinophils. This represents:
A. Absolute lymphocytosis
B. Relative neutrophilia
C. Agranulocytosis
D. Leukopenia

3.114 Which of the following is the most frequent cause of neutrophilia:


A. Viral infection
B. Allergic reactions
C. Acute bacterial infection
D. Myeloperoxidase deficiency
lsmusis.lsmuni.lt/Klausimai/Spausdinti?Length=0?Kalba=EN&KategorijaId=105&Kalbos_input=EN&Kalbos=EN&KategorijaEn_input=Hematology&K… 11/25
3/27/2019 LSMUSIS
3.115 Which of the following laboratory parameters is the most specific for deficiency of vitamin B12 or folic acid:
A. Decreased ferritin
B. Fragmented erythrocytes in the blood smear
C. Positive Coombs test
D. Increased MCV (mean corpuscular volume)

3.116 Which statement about folic acid deficiency anemia is wrong:


A. Anemia is makrocytic
B. Folic acid body reserves are sufficient till 10 years
C. In bone marrow smear You find megaloblasts
D. In blood smear You find hypersegmented neutrophils

3.117 Which findings in the bone marrow are the most specific for vitamin B12 or folic acid deficiency anemia:
A. Normoblastic hypocellular
B. Megaloblastic hypercellular
C. Megaloblastic hypocellular
D. Normoblastic hypercellular

3.118 Complete blood count was done and results presented: leukocytes 0.8x109/l, hemoglobin 50g/l, erythrocytes
1.5x1012/l, platelets 20x109/l. What term best describes this peripheral blood picture:
A. Granulocytopenia
B. Pancytopenia
C. Bicytopenia
D. Pure red cell aplasia

3.119 Method of treatment of first choice of autoimmune warm antibodies hemolytic anemia:
A. Splenectomy
B. Red blood cells transfusion
C. Steroids
D. Infution therapy

3.120 In warm autoimmune hemolytic anemia the offending antibody is usually:


A. IgG
B. IgM
C. IgA
D. A mixture of IgG ir IgM

3.121 What is the most likely mechanism of hemolysis in warm autoimmune haemolytic anemia:
A. Phagocytosis of IgG-coated erythrocytes
B. Increased snsitivity to complement
C. Complement fixation to IgM- coated erythrocytes
D. Increased membrane instability

3.122 A leukemoid neutrophilic reaction may be best distinquished from chronic myelocytic leukemia by:
A. The total leukocyte count
B. The presence or absence of immature neutrophils
C. Chromosome and molecular studies
D. The presence or absence of anemia

3.123 Which statement about chronic lymphocytic leukemia is wrong:


A. Most frequent among leukemias in Western countries
B. The most frequent cause of disease is long living T lymphocytes
C. III and IV stages of CLL according Rai is determined by hemglobin concentration and platelets number in
blood
D. Disease can be treated with fludarabine and cyclophosphamide combination

3.219 Which statement about lymphogranulomatosis is wrong:


A. It is rarely diagnosed, if Sternberg giants are not found
B. ESR in most cases is increased; it is the evidence of the disease activity
C. In 80–60% of the patients, if disease is diffuse, full remission is impossible to achieve
D. BEACOPP or ABVD scheme treatment is applied

lsmusis.lsmuni.lt/Klausimai/Spausdinti?Length=0?Kalba=EN&KategorijaId=105&Kalbos_input=EN&Kalbos=EN&KategorijaEn_input=Hematology&K… 12/25
3/27/2019 LSMUSIS
II type tasks. For each numbered item,selct the one lettered heading that is most closely asssciated with it

3.124 For which reasons in situations mentioned below folic acid deficiency developes:
1 - Chronic alkoholism
2 - Hemodialysis
3 - Small bowel disorders
4 - Hemolytic anaemia
5 - Pregnancy
A. Malabsorbtion of folic acid
B. Disturbed resorbtion of folic acid
C. Increased requirements of folic acid
D. Increased loss of folic acid

3.125 Which stage of Hodgkin‘s disease will be diagnosed according following findings:
1 - Palpable 2.0 cm. size lymphnodes in neck and inquinal areas, no other symptoms
2 - Palpable 3.0 cm. size lymphnodes in axillar area, in bone marrow biopsy Reed-Stenberg cells are found,
patient has fever without obvious course till 39oC lasting two months
3 - Palpable 2.0 cm. size lymphnodes in neck and axillar areas, patient lost more than 20% of body weight
during last 3 months
4 - Palpable 3.0 cm. size lymphnodes in inquinal area, no other symptoms
A. Stage IVB
B. Stage IA
C. Stage IIB
D. Stage IIIA

3.126 Which tests do you need to diagnose each of these disorders:


1 - Chronic myelocytic leukemia
2 - Polycythemia vera
3 - Chronic lymphocytic leukemia
4 - Multiple myeloma
A. Bone marrow aspiration, flow cytometry of blood or bone marrow lymphocytes
B. Determination of JAK2 mutation and erythrocytes mass, bone marrow biopsy
C. Bone marrow aspiration, serum or urine protein electropheresis or immunofixation, bone x-ray survey
D. Bone marrow aspiration, cytogenetic test for Philadelphia chromosome and molecular test for
BCR/ABL transcript

3.127 What will you chose for the treatment of mentioned diseases:
1 - Hemophilia
2 - Idiopathic thrombocitopenic purpura
3 - Prothrombin complex coagulopathy
4 - Scurvy
A. Glucocorticoids
B. Clotting factors replacement
C. Platelets transfusions
D. Frosen plasma
E. Vit. C

3.128 Which laboratory data correlate the best with clinical situation:
1 - lntravascular hemolysis
2 - Estravascular hemolysis
3 - Acute hemolytic crisis
4 - Chronic hemolysis
A. Reticulocytosis, hemoglobinemia and hemoglobinuria
B. Progressive normochromic anaemia, increase slightly indirect bilirubin level
C. Combination of both laboratory findings
D. Any variant can be chosen

3.129 Choose right equivalent for statements marked in numbers from statements marked in letters:
1 - Deficiency can be related to glositis
2 - Organism get needed substances from bowel microflora
3 - Absorbtion takes place in distal part of small bowel
4 - Megaloblastic anemia appear after 4 month period of “alimentary hunger“
lsmusis.lsmuni.lt/Klausimai/Spausdinti?Length=0?Kalba=EN&KategorijaId=105&Kalbos_input=EN&Kalbos=EN&KategorijaEn_input=Hematology&K… 13/25
3/27/2019 LSMUSIS
5 - The deficiency is associated with autoantibodies
6 - The deficiency can appear because of the small bowel diseases
7 - Can be found in vegetables
8 - Special protein is needed for transport
9 - The deficiency can cause neurological symptoms
10 - Megaloblastic anemia appears after 4 years period of alimentary "hunger"
A. Folic acid
B. Vit. B₁₂
C. Both mentioned
D. Anyone mentioned

3.130 Which laboratory data gives most information for diagnosis of mentioned diseases and pathological states:
1 - Idiopathic thrombocytopenic purpura
2 - Hemophilia
3 - Progressive hepotocellulare insufficiency
4 - Overdosage of coumarin and its analogues
A. Bleeding time
B. Prothrombin time
C. Activated partial thromboplastin time
D. Platelet count
E. Number of megakaryocytes in the bone marrow

3.131 Which complex of tests is characteristic for each disorder:


1 - Acute myeloblastic leukemia
2 - Acute lymphoblastic leukemia
3 - Agranulocytosis
4 - Primary myelofibrosis
A. Blood granulocytes < 0.5x109/l, myelokaryocytes number in bone marrow is normal, but granulocytes
precursors are absent in bone marrow
B. Myelokaryocytes number in bone marrow is increased, blasts >20% , peroxidase reaction in blast is
positive
C. Increased number of reticulin-fibrotic tissue in bone marrow, in blood smear erythroblasts and early
granulocytes precursors are found
D. Myelokaryocytes number in bone marrow is increased, blasts >20% , peroxidase reaction in blast is
negative, periodic acid-schiff reaction in blats is positive, blats are found in cerebrospinal fluid

3.132 What complex of tests is characteristic for each mentioned disorder:


1 - Hemolytic anemia
2 - Aplastic anemia
3 - Iron deficiency anemia
4 - Megaloblastic anemia
A. Pancytopenia in peripheral blood, "'dry tap" bone marrow aspiration
B. Pancytopenia in peripheral blood, magaloblastic bone marrow
C. Reticulocytosis, increased indirect bilirubin. erythroid hyperplasia of bone marrow
D. Reticulocytosis, decreased serum iron, erythroid hyperplasia of bone marrow

3.133 What clinical features are characteristic for each disorder:


1 - Aplastic anemia
2 - Megaloblastic anemia
3 - Hemolitic anemia
4 - Iron deficiency anemia
A. Skin and mucosa pallor, dyspnea on exertion, weakness, hair loss, brittle nails
B. Mucosa pallor, tachycardia on exertion, numbness in the hands or feet, reddened tongue.
C. General weakness, jaundice in the skin and mucosa, splenomegaly, darl urine.
D. General weakness, mucosa pallor, petechiae rash in the skin, common infections.

3.134 What test do you need to diagnose each of these disorders:


1 - Iron deficiency anemia
2 - Megaloblastic anemia
3 - Hemolytic anemia
5 - Aplastic anemia
A. Complete blood count, reticulocytes count, trepanobiopsy
lsmusis.lsmuni.lt/Klausimai/Spausdinti?Length=0?Kalba=EN&KategorijaId=105&Kalbos_input=EN&Kalbos=EN&KategorijaEn_input=Hematology&K… 14/25
3/27/2019 LSMUSIS
B. Plasma cobalamin level, bone marrow aspiration
C. Serum bilirubin and it’s fractions level, erythrocytes osmotic fragility, autohemolysis test, Coomb's tests
D. Ferritin blood test, TIBC, transferrin saturation.

III type tasks. For each question there is one or more correct answers:
A – if correct answers are 1,2,3
B – if correct answers are 1 and 3
C – if correct answers are 2 and 4
D – if correct answer is 4
E – if correct are all answers above

3.135 Which findings are typical for vitamin B12 anemia:


1. Pancytopenia
2. Megaloblasts in bone marrow
3. Neurological damage
4. Increased MCV

3.136 In which diseases hypohromic microcytic anemia can be found:


1. Autoimmune hemolytic anemia
2. Iron deficiency anemia
3. Vitamin B12 deficiency anemia
4. Anemia of chronic disease

3.137 In which diseases can be found Philadelphia chromosome:


1. Neutrophil leukemoid reaction
2. Polycythemia vera
3. Chronic lymphocytic leukemia
4. Chronic myelocytic leukemia

3.138 In which diseases allogeneic bone marrow transplantation can be performed during treatment period:
1. Chronic myelocytic leukemia
2. Acute lymphoblastic leukemia
3. Primary myelofibrosis
4. Acute myeloblastic leukemia

3.139 In which diseases autologous bone marrow transplantation can be performed during treatment period:
1. Multiple myeloma
2. Aplastic anemia
3. Hodgkin‘s disease
4. Iron deficiency anemia

3.140 Which of the following are used for the treatment of hemophilia B:
1. Desmopresin
2. Prothrombin complex
3. IX clotting factor concentrate
4. Platelets transfusion

3.141 When prothrombin time is prolonged:


1. Congenital factor VII deficiency
2. Severe acute liver insufficiency
3. Administration of terapeutic dose of varfarin
4. Congenital factor VIII deficiency

3.142 When activated partial thromboplastin time is within normal range:


1. Severe hemophilia A
2. Severe hemophilia B
3. During administration of terapeutic heparin dose
4. Congenital factor VII deficiency

3.143 During course of chronic lymphocytic leukemia following complications can occur:
1. Autoimmune hemolytic anemia
2. Recurrent infections due to deficiency of IgG
lsmusis.lsmuni.lt/Klausimai/Spausdinti?Length=0?Kalba=EN&KategorijaId=105&Kalbos_input=EN&Kalbos=EN&KategorijaEn_input=Hematology&K… 15/25
3/27/2019 LSMUSIS
3. Autoimmune trombocytopenia
4. Transformation to Richter syndrome

3.144 Which drugs can be used for the treatment of autoimmune idiopathic thrombocytopenia:
1. Prednisolone
2. Intravenous immunoglobulin
3. Rituksimab
4. Vincristin

3.145 Which of the following is characteristic for severe aplastic anemia:


1. Agranulocytosis
2. Severe trombocytopenia
3. Few myelokaryocytes in bone marrow biopsy
4. Significantly prolonged activated partial thromboplastin time

3.146 In which of the following conditions trombocytopenia can develope:


1. Systemic lupus erythemathosus
2. Hepatitis C
3. During administration of heparin
4. HIV infection

3.147 Laboratory data provide the findings in the patients with chronic lymphocytic leukemia:
1. Anemia
2. Thrombocytopenia
3. Reticulocytosis
4. Reticulocytopenia

3.148 The genesis of anemia in the case of chronic lymphocytoc leukemia is:
1. Impairment of erythropoesis
2. Hemolysis
3. Hemorrhage
4. Megalablastic hematopoiesis

3.149 Blast cells can be detectable in the peripheral blood in the case of:
1. Acute leukemia
2. Myeloma
3. Chronic myelogenous leukemia
4. Hodgkin‘s lymphoma

3.150 Myelofibrosis is often seen in the course of:


1. Chronic myelogenous leukemia
2. Aplastic anemia
3. Polycythemia vera
4. Iron deficiency anemia

3.151 Pruritus after hot shower is specific for:


1. Myeloma
2. Hodgkin‘s lymphoma
3. Acute leukemia
4. Polycythemia vera

3.152 Laboratory data provide the findings in the patient with chronic lymphocytic leukemia:
1. Absolute lymphocytosis
2. Anemia
3. Thrombocytopenia
4. Leukocytosis

3.153 Laboratory data provide the findings in the patient with chronic lymphocytic leukemia:
1. The shift to the left in the leukogram
2. Monocytosis
3. Thrombocytosis
4. Leukocytosis

lsmusis.lsmuni.lt/Klausimai/Spausdinti?Length=0?Kalba=EN&KategorijaId=105&Kalbos_input=EN&Kalbos=EN&KategorijaEn_input=Hematology&K… 16/25
3/27/2019 LSMUSIS
3.154 The main complications of acute hemolytic transfusion reaction:
1. Sepsis
2. DIC syndrome.
3. Hepatic insufficiency
4. Renal insufficiency

3.155 Delayed haemolytic transfusion is caused by:


1. Antibodies of ABO system
2. HLA antibodies
3. All the reasons mentioned above
4. Antibodies against antigens of Rh and other clinically important systems.

3.156 Neutrophilic leukemoid reactions are associated with:


1. Tissue necrosis
2. Carcinoma
3. Inflammatory diseases
4. Acute bacterial infections

3.157 Characteristic complications of polycythemia vera:


1. Intestinal hemorrhage
2. Peripheral venous thromboses
3. Venous occlusions of retinae
4. A.pulmonalis thromboses

3.158 Bone marrow aspirations results in a "dry-tap" in the case of:


1. Aplastic anemia
2. Agranulocytosis
3. Myelofibrosis
4. Lymphogranulomatosis

3.159 Pancytopenia is associated with


1. Acute leukemia
2. Megaloblastic anemia
3. Aplastic anemia
4. The III stage of polycythaemia vera

3.160 Increased amount of blasts is detectable in the bone marrow aspirate:


1. Metastatic disease of the bone marrow
2. Bone marrow aplasia (the phase of recovery)
3. In Hodgkin‘s lymphoma
4. Agranulocytosis (the phase of recovery)

3.161 Blasts cells shows acute leukemia when:


1. There are ≤ 5% blasts in the bone marrow
2. There are ≥ 20% blasts in the bone marrow
3. There are 5-15% blasts in peripheral blood or bone marrow
4. here are ≥ 20% blasts in peripheral blood

3.162 Clinico-laboratory syndromes of acute leukemia:


1. Reduced normal hemotopoiesis.
2. Palpable findings
3. Leukemic infiltration of tissues
4. Hypercatabolism

3.163 Relative lymphocytosis in peripheral blood is detectable in:


1. Chronic myelofibrosis
2. Chronic lymphocytic leukemia
3. Myelofibrosis
4. Agranulocytosis

3.164 Massive splenomegaly is present in:


1. Myelofibrosis
2. Chronic lymphocytic leukemia
lsmusis.lsmuni.lt/Klausimai/Spausdinti?Length=0?Kalba=EN&KategorijaId=105&Kalbos_input=EN&Kalbos=EN&KategorijaEn_input=Hematology&K… 17/25
3/27/2019 LSMUSIS
3. Chronic myelogenous leukemia
4. Myeloma

3.165 When erythrocytosis is stimulated by low arterial oxygen pressure:


1. Chronic obstructive bronchitis with emphysema
2. High altitude
3. Congenital "blue babies" heart disease
4. Hypernephroma

3.166 The causes of iron deficiency in polycythaemia vera are:


1. Decrease secretion of eritropoetin
2. Iron stores depletion
3. Hemolysis
4. Phlebotomies

3.167 Lymphoproliferative disorders:


1. Myeloma
2. Lymphoma malignum
3. Chronic lymphocytic leukemia
4. Lymphogranulomatosis

3.168 Myeloproliferative disorders:


1. Chronic myelogenous leukemia
2. Polycythemia vera
3. Myelofibrosis
4. Acute lymphoblastic leukemia

3.169 Myeloma staging system according Durie Salmon classification system include the following criteria:
1. Degree of anemia
2. Serum calcium value
3. Production rates of immunoglobulins
4. Thrombocytes value
5. Findings of the mielogram

3.170 M-component in the serum is detectable in:


1. Multiple myeloma
2. Lymphoma malignum
3. Chronic lymphocytic leukemia
4. Chronic myelogenous leukemia
5. Acute leukemia

3.171 Symptoms of intoxication (B) in Ann Arbor staging system of lymphogranulomatosis:


1. Fever
2. Profused sweating
3. Loss of body weight
4. Pruritus

3.172 Secondary erythrocytosis can found in the case of:


1. Renal disorders
2. Pulmonary diseases
3. Endocrine diseases
4. Cardiovascular diseases

3.173 Laboratory findings in the blood in myelofibrosis (agnogenic myeloid metaplasia):


1. Thrombocytosis
2. Leukocytosis
3. Anemia
4. Monocytosis, plasmacytosis

3.174 Laboratory findings in myelofibrosis (agnogenic myeloid metaplasia)


1. Anemia
2. Poikylocytosis
3. Anisocytosis
lsmusis.lsmuni.lt/Klausimai/Spausdinti?Length=0?Kalba=EN&KategorijaId=105&Kalbos_input=EN&Kalbos=EN&KategorijaEn_input=Hematology&K… 18/25
3/27/2019 LSMUSIS
4. Leukocytosis
5. "Shift to the letf“ of leukogram

3.175 Laboratory findings of peripheral blood in myelofibrosis:


1. Erythrocytosis
2. Blasts cells
3. Lymphocytosis
4. Megaloblasts

3.176 Laboratory findings in polycytaemia vera (stage III):


1. Anemia
2. Erythrocytosis
3. Thrombocytopenia
4. Thrombocytosis
5. Leukocytosis

3.177 Clinical and laboratory manifestation in myelofibrosis (agnogenic myeloid metaplasia):


1. Massive splenomegaly
2. Frequent morphologic abnormalities of erythrocytes
3. "Dry" aspiration of the bone marrow
4. lncreased amount of fat tissue in trephine biopsy material

3.178 Acute myelogenous leukemia is treated with:


1. Vincristine
2. Citozinarabinoside
3. Prednisolone
4. Rubomicine

3.179 In the terminai phase of chronic myelogenous leukaemia you can find
1. Blastic crisis
2. Myelofibrosis
3. Resistancy to myelosuppressive treatment
4. Bone marrow aplasia

3.180 Syndrome of "bone marrow insufficienty" is characterized by:


1. Fatigue, paleness
2. Infections
3. Echymoses, haemorrhage
4. Splenomegaly
5. Lymphadenopathia

3.181 Common clinical syndromes in multiple myeloma:


1. Anemic syndrome
2. Bone disease
3. Renal insufficiency
4. Hepatosplenomegaly

3.182 The causes of the jaundice in the case of Hodgkin's disease:


1. Obstruction of biliary tract by enlarged lymphnodes
2. Hemolysis
3. Infiltration of liver by neoplastic cells
4. Gallstones in the ductus choledochus

3.183 Clinical manifestations of multiple myeloma:


1. Back pain
2. Anemia
3. Hyperviscosity
4. Erythromelalgia

3.184 Coagulopathy is characterized by:


1. Hematoma formation in muscles
2. Hematoma formation in soft tissues
3. Hemorrhage into cavities
lsmusis.lsmuni.lt/Klausimai/Spausdinti?Length=0?Kalba=EN&KategorijaId=105&Kalbos_input=EN&Kalbos=EN&KategorijaEn_input=Hematology&K… 19/25
3/27/2019 LSMUSIS
4. Purpura and ecchimoses

3.185 Laboratory data characteristics of severe hemophilia:


1. Prolongation of coagulation time
2. Prolongation of prothrombin time
3. Prolongation of activated partial thromboplastin time
4. Prolongation of bleeding time

3.186 Profound thrombocytopenia is characterised by:


1. Reduction of prothrombin time
2. Decreased fibrinogen concentration
3. Prolongation of activated thromboplastin time
4. Prolongation of bleeding time

3.187 Profound thrombocytopenia is found in the case of:


1. Idiopathic thrombocytopenic purpura
2. Acute leukemia
3. Aplastic anemia
4. Megaloblastic anemia
5. Myelodysplastic syndrome

3.188 Laboratory data confirming ferrodeficitic anemia:


1. Reduced plasma iron concentration, transferrin saturation, increased plasma iron-binding capacity.
2. Microcytosis, hypochroma, reduced mean corpuscular hemoglobin concentration
3. Reduced amount of bone marrow sideroblasts
4. Reduced tissue enzyme iron

3.189 The deficiency of vit. B12 causes:


1. Hypersegmentation of neutrophils
2. Appearance of megaloblasts in bone marrow
3. Appearance of autoantibodes if it‘s pernicious anemia
4. Appearance of normocytic anemia

3.190 What syndromes are prevalent in B12 deficiency anemia?


1. Anemic
2. Neurological
3. Gastroenterological
4. Intoxication

3.191 Whish answer about vit B12 absorbtion in correct:


1. Intrinsic factor produced by gastric parietal cells is needed
2. Vit B12 is connected with intrinsic factorin the stomach
3. Complex of vit B12 and intrinsic factor absorbed in the distal part of jejunum
4. Complex of vit. B12 intrinsic factor is absorbed in intestinum crassii

3.192 Hemolytic anemia is characterized by:


1. Normochromic anaemia
2. Reticulocytosis in the peripheral blood
3. Rise of iron iones and indirect bilirubin in plasma
4. Decreased amount of haptoglobine
5. Erythoblastic hyperplasia in bone marrow

3.193 When does pancytopenia of central origin occur:


1. Myelofibrosis with myeloid metaplasia in the spleen
2. Marrow infiltration by malignant cells
3. Acute leukemia
4. In the case of hypersplenism

3.194 Causes of bone marrow failure are:


1. Acute blood loss
2. Toxic aplasia because of treatment with neuroleptics
3. Poisoning with carbon monoxide
4. Acute viral hepatitis
lsmusis.lsmuni.lt/Klausimai/Spausdinti?Length=0?Kalba=EN&KategorijaId=105&Kalbos_input=EN&Kalbos=EN&KategorijaEn_input=Hematology&K… 20/25
3/27/2019 LSMUSIS
5. Chronic renal failure
6. Cirrhosis of the liver

3.195 Measures directed at reversal of aplasia are:


1. Bone marrow transplantation
2. Anti-lymphocyte globulin
3. Erythropoetin
4. Glucocorticoids
5. Packed red cells and platelets concentrates transfusions

3.196 In which diseases Coombs test can be positive:


1. Autoimmune hemolytic anemia
2. Aplastic anemia
3. Chronic lymphocytic leukemia
4. Hereditary spherocytosis

IV type tasks. Choose only one best answer

3.197 25 years old women complain of fast fatigue, drowsiness, palpitations, burning tongue, excessive
mensturations. She has 2 children. During general examination pallor, scalling of the skin, fragility of nails was
noticed. Pulse 90/min. II0 systolic murmur. BP- 110/70 mm/Hg, Hb – 80g/l, eryth. 3,2.1012/l, ret 10%, plat.
300.109/1 leuk. 4,5.10911, segmen.53%, limf.35%, eos. 1%, band 6%, mon 5%, ESR -12mm/hour. Iron
concentration in the plasma is 7,4 mmol/l. What kind of treatment will you administer:
A. Estrogen injections
B. Transfusions of packed red cells
C. Oral iron preparations until remision, after remision is reached, treatment finished
D. Oral iron preparations until remision, later continue for 4-6 months.

3.198 67 years old man complain of shortness of breath on mild exertion. Laboratory tests data are following: Hb
88 g/l, erytr. 3,17x1012/l, MCV 80 fl, MCH 28 pg, leuk.4,5x109/l, platelets 320x109/l, feritin 10 mg/ l. Wich
proposition is correct:
A. There is no of sideroblasts in bone marrow
B. Decreased cancentration of iron in plasma
C. Quick reaction to iron preparations
D. All propositions are correct

3.199 67 years old man complain of shortness of breath on mild exertion. Laboratory tests data are following: Hb
88 g/l, erytr. 3,17x1012/l, MCV 80 fl, MCH 28 pg, leuk.4,5x109/l, platelets 320x109/l, feritin 10 mg/ l. What
pre1iminary diagnosis:
A. Sideroblastic anaemia
B. Lead intoxication
C. Sepsis
D. Iron deficiency anemia

3.200 67 years old man complain of shortness of breath on mild exertion. Laboratory tests data are following: Hb
88 g/l, erytr. 3,17x1012/l, MCV 80 fl, MCH 28 pg, leuk.4,5x109/l, platelets 320x109/l, feritin 10 mg/ l. What is
common cause of this disorder:
A. Atherosclerotic lesions of vessels
B. Chronic bleeding from gastrointestinal tract
C. Inadequate diet
D. Diarrhoea

3.201 25 year old man was admitted to the hospital because of the bleeding from the gastrointestinal tract. Is
known that the patient was diagnosed hemophilia A in the childhood. In the blood tests: Hb 95 g/l, aPPT 108 sec,
SPA 98%, INR 0.8. What will be the treatment of the first choice:
A. Red blood cells transfusion
B. Infusion therapy
C. Desmopresin
D. VIII coagulation factor substitution

3.202 18 years old man noticed weakness and bleeding gums. Seven days ago he painted the house. On general
examination pallor of the skin, petechiae in the distal portions of the extremities and in the chest skin was noticed.

lsmusis.lsmuni.lt/Klausimai/Spausdinti?Length=0?Kalba=EN&KategorijaId=105&Kalbos_input=EN&Kalbos=EN&KategorijaEn_input=Hematology&K… 21/25
3/27/2019 LSMUSIS
Laboratory tests data are following: Hb 100 gll, eryth. 2,8x1012l/l, ret. 5%, platelets 20x109/l, leuk. 3,8x109/1,
eosin.1%, segmen. 62%, Ht 30%, mon. 7%, ESR 6 mm/hour. Bone marrow aspiration - myelokariocytes 7,5x109/t,
only few erythrocytes and granulocytes, fat compromises 75% of the marrow, only few megarocytes and platelets.
What preliminary diagnosis:
A. Iron deficiency anaemia
B. Myelofibrosis
C. Idiopatic trombocytopenic purpura
D. Anemia aplastica

3.203 18 years old man noticed weakness and bleeding gums. Seven days ago he painted the house. On general
examination pallor of the skin, petechiae in the distal portions of the extremities and in the chest skin was noticed.
Laboratory tests data are following: Hb 100 gll, eryth. 2,8x1012l/l, ret. 5%, platelets 20x109/l, leuk. 3,8x109/1,
eosin.1%, segmen. 62%, Ht 30%, mon. 7%, ESR 6 mm/hour. Bone marrow aspiration - myelokariocytes 7,5x109/t,
only few erythrocytes and granulocytes, fat compromises 75% of the marrow, only few megarocytes and platelets.
What is first line treatment in this situation:
A. Packed red cells transfusions
B. Fresh platelets concentrate transfusions
C. Leukocytes concentrate transfusion

3.204 18 years old man noticed weakness and bleeding gums. Seven days ago he painted the house. On general
examination pallor of the skin, petechiae in the distal portions of the extremities and in the chest skin was noticed.
Laboratory tests data are following: Hb 100 gll, eryth. 2,8x1012l/l, ret. 5%, platelets 20x109/l, leuk. 3,8x109/1,
eosin.1%, segmen. 62%, Ht 30%, mon. 7%, ESR 6 mm/hour. Bone marrow aspiration - myelokariocytes 7,5x109/t,
only few erythrocytes and granulocytes, fat compromises 75% of the marrow, only few megarocytes and platelets.
What addition treatment you can administer:
A. Broad spectrum antibiotics
B. Prednisolone
C. Iron preparations
D. Vitamin B12 injections

3.205 56 old man without obvious reason got fever over 38° C; chills, pain in the lower back, later he noticed
jaundice and dark urine. One week ago he suffered viral infection of the upper respiratory tract, he was taking
aspirin 1,0 g daily. During general examination pallor, mild jaundice was noticed. Pulse 90/min, BP-130/70 mmHg,
Hb 80 g/l, eryt 2,4x1012/1, microspherocytes , ret 56%, platelets 180x109 /1, leukocytes 5,8x109/1, eozin 1 %,
band 10%, segmen 58%, limfoc 21 %, mon 10%, ESR 10 mm/hour. Bilirubin 35 mmol/l, direct antiglobulin test (+ +
), urobilin in the urine ( + ). What preliminary diagnosis:
A. Viral hepatitis
B. Viral respiratory tract infection
C. Autoimmune haemolytic anaemia
D. Persistent hepatitis

3.206 56 old man without obvious reason got fever over 38° C; chills, pain in the lower back, later he noticed
jaundice and dark urine. One week ago he suffered viral infection of the upper respiratory tract, he was taking
aspirin 1,0 g daily. During general examination pallor, mild jaundice was noticed. Pulse 90/min, BP-130/70 mmHg,
Hb 80 g/l, eryt 2,4x1012/1, microspherocytes , ret 56%, platelets 180x109 /1, leukocytes 5,8x109/1, eozin 1 %,
band 10%, segmen 58%, limfoc 21 %, mon 10%, ESR 10 mm/hour. Bilirubin 35 mmol/l, direct antiglobulin test (+ +
), urobilin in the urine ( + ). What treatment you will administer:
A. Washed erythrocytes transfusions
B. Oral prednisolone 1mg/l kg body weight daily
C. Vincristine 1,5 mg i/v once weekly
D. Splenectomy

3.207 19 years old women complain of general weakness. Pancytopenia in the peripherial blood film. Palpable
spleen on the examination. What preliminary diagnosis:
A. Aplastic anaemia
B. Agranulocitosis
C. Acute leukemia
D. Polycythaemia vera
E. Megaloblastic anaemia

3.208 19 years old women complain of general weakness. Pancytopenia in the peripherial blood film. Palpable
spleen on the examination. What data are necessary for correct diagnosis?
lsmusis.lsmuni.lt/Klausimai/Spausdinti?Length=0?Kalba=EN&KategorijaId=105&Kalbos_input=EN&Kalbos=EN&KategorijaEn_input=Hematology&K… 22/25
3/27/2019 LSMUSIS
A. Spleen aspiration (punction)
B. Bone marrow aspiration
C. Determination of Philadelphia chromosoma
D. Reticulocytes count

3.209 60 year old man was admited to the hospital because of weakness. On physical examination enlarged firm,
discrete, painless, rubbery cervical and axillary lymph nodes were found. Hb 75 g/l, leuk 50x109/1, 85%
lymphocytes. What is preliminary diagnosis:
A. Hemolytic anemia
B. Acute leukemia
C. Lymphoma malignum
D. Chronis lymphocytic leukemia
E. Chronic myelocytic leukemia

3.210 60 year old man was admited to the hospital because of weakness. On physical examination enlarged firm,
discrete, painless, rubbery cervical and axillary lymph nodes were found. Hb 75 g/l, leuk 50x109/1, 85%
lymphocytes. What additional data do you need for correct diagnosis:
A. Spleen aspiration (punction)
B. Lymph node aspiration (punction)
C. Bone marrow aspiration
D. Lymph node biopsy

3.211 60 year old man was admited to the hospital because of weakness. On physical examination enlarged firm,
discrete, painless, rubbery cervical and axillary lymph nodes were found. Hb 75 g/l, leuk 50x109/1, 85%
lymphocytes. Which drug in this situation is the best:
A. Vincristine
B. Chlorambucin (laukeran)
C. Prednisolone
D. Cyclophosphamide
E. Busulfan (Myleran)

3.212 26 year old man was admited to the hospital because of weakness. On physicial examination enlarged firm,
discrete, painless, rubbery cervical and axillory lymph nodes were found. Hb 75 g/l, leuk 50x109/1, 85% blasts.
What is preliminary diagnosis:
A. Haemolytic anaemia
B. Acute leukemia
C. Lymphoma malignum
D. Chronic myeloleucosis

3.213 26 year old man was admited to the hospital because of weakness. On physicial examination enlarged firm,
discrete, painless, rubbery cervical and axillory lymph nodes were found. Hb 75 g/l, leuk 50x109/1, 85%
lymphoblasts. What additional data do you need for correct diagnosis:
A. Spleen punction
B. Lymph node punction
C. Bone marrow aspiration
D. Lymph node biopsy

3.214 26 year old man was admited to the hospital because of weakness. On physicial examination enlarged firm,
discrete, painless, rubbery cervical and axillary lymph nodes were found. Hb 75 g/l, leuk 50x109/1, 85% blasts.
What treatment you will administer:
A. Chlorambucil, prednisolone
B. Hydroxyurea
C. CHOP protocol
D. NOPHO protocol

3.215 36 years old women noticed petechies and ecchymoses in lower extremities. She emphasized exessive last
mensturation. General condition is good. Lymph nodes and spleen not palpable. Hb 110 g/l, erythr 3,6x1012/1,
leuk: 7,8x109/1, retik 1,8%, platelets 30x109/l, ESR 20 mm/hour. Leukocytes differencial normal. What data in this
situation is more useful:
A. Bleeding time
B. Protrombine index
C. Activated partial thromboplastine time

lsmusis.lsmuni.lt/Klausimai/Spausdinti?Length=0?Kalba=EN&KategorijaId=105&Kalbos_input=EN&Kalbos=EN&KategorijaEn_input=Hematology&K… 23/25
3/27/2019 LSMUSIS
D. Fibrinogen concentration
E. Bone marrow examination

3.216 36 years old women noticed petechies and ecchymoses in lower extremities. She emphasized exessive last
mensturation. General condition is good. Lymph nodes and spleen not palpable. Hb 110 g/l, erythr 3,6x1012/1,
leuk: 7,8x109/1, retik 1,8%, platelets 30x109/l, ESR 20 mm/hour. Leukocytes differencial normal. Such results
were obtained: bleeding time 12 min, prothrombin index 100%, APTT 30 sec, fibrinogen 3500 mg/l, bone marrow
examination: increased number of megakariocytes, there are no platelets. What is first line treatment:
A. Splenectomy
B. Fresh frosen plasma infusions
C. Cryoprecipitate and platelets transfusions
D. Vit K injections
E. Glucocorticoids

3.217 25 years old women complain of fast fatigue, drowsiness, palpitations, burning tongue, excessive
mensturations. She has 2 children. During general examination pallor, scalling of the skin, fragility of nails was
noticed. Pulse 90/min. II0 systolic murmur. BP- 110/70 mm/Hg, Hb – 80g/l, eryth. 3,2.1012/l, ret 10%, plat.
300.109/1 leuk. 4,5.10911, segmen.53%, limf.35%, eos. 1%, band 6%, mon 5%, ESR -12mm/hour. Iron
concentration in the plasma is 7,4 mmol/l. Which diagnostics procedures must be administered:
A. Esophagogastroduodenoscopy, gastric acid determination in gastric juice
B. Bone marrow aspiration
C. Gynaecologist consultation
D. Echocardiography, fonocardiography

3.218 25 years old women complain of fast fatigue, drowsiness, palpitations, burning tongue, excessive
mensturations. She has 2 children. During general examination pallor, scalling of the skin, fragility of nails was
noticed. Pulse 90/min. II0 systolic murmur. BP- 110/70 mm/Hg, Hb – 80g/l, eryth. 3,2.1012/l, ret 10%, plat.
300.109/1 leuk. 4,5.10911, segmen.53%, limf.35%, eos. 1%, band 6%, mon 5%, ESR -12mm/hour. Iron
concentration in the plasma is 7,4 mmol/l. What diagnosis:
A. Anaemia Addisoni-Birmeri
B. Iron deficiency anemia
C. Anemia aplastica
D. Hemolytic anemia

3.1 - A 3.2 - E 3.3 - B 3.4 - B 3.5 - B 3.6 - D 3.7 - A 3.8 - B


3.9 - D 3.10 - D 3.11 - B 3.12 - D 3.13 - D 3.14 - B 3.15 - D 3.16 - C
3.17 - A 3.18 - A 3.19 - C 3.20 - B 3.21 - D 3.22 - B 3.23 - C 3.24 - D
3.25 - D 3.26 - B 3.27 - D 3.28 - D 3.29 - C 3.30 - C 3.31 - B 3.32 - D
3.33 - C 3.34 - D 3.35 - B 3.36 - C 3.37 - D 3.38 - D 3.39 - C 3.40 - D
3.41 - D 3.42 - B 3.43 - D 3.44 - B 3.45 - D 3.46 - C 3.47 - D 3.48 - B
3.49 - C 3.50 - C 3.51 - C 3.52 - B 3.53 - D 3.54 - B 3.55 - C 3.56 - C
3.57 - B 3.58 - A 3.59 - D 3.60 - A 3.61 - C 3.62 - C 3.63 - A 3.64 - D
3.65 - D 3.66 - A 3.67 - C 3.68 - D 3.69 - A 3.70 - D 3.71 - C 3.72 - C
3.73 - B 3.74 - D 3.75 - A 3.76 - C 3.77 - C 3.78 - B 3.79 - B 3.80 - C
3.81 - D 3.82 - C 3.83 - B 3.84 - D 3.85 - D 3.86 - D 3.87 - B 3.88 - B
3.89 - B 3.90 - D 3.91 - A 3.92 - B 3.93 - B 3.94 - A 3.95 - A 3.96 - D
3.97 - B 3.98 - B 3.99 - B 3.100 - D 3.101 - A 3.102 - C 3.103 - B 3.104 - A
3.105 - C 3.106 - B 3.107 - C 3.108 - B 3.109 - C 3.110 - B 3.111 - D 3.112 - B
3.113 - C 3.114 - C 3.115 - D 3.116 - B 3.117 - B 3.118 - B 3.119 - C 3.120 - A
3.121 - A 3.122 - C 3.123 - B 3.219 - C
3.124 3.125 3.126 3.127 3.128 3.129 3.130 3.131
1-B 1-D 1-D 1-B 1-A 1-C 1-D 1-B
2-D 2-A 2-B 2-A 2-B 2-D 2-C 2-D
3-A 3-C 3-A 3-D 3-A 3-B 3-B 3-A
4-C 4-B 4-C 4-E 4-B 4-A 4-B 4-C
5-C 5-B
6-C
7-A
lsmusis.lsmuni.lt/Klausimai/Spausdinti?Length=0?Kalba=EN&KategorijaId=105&Kalbos_input=EN&Kalbos=EN&KategorijaEn_input=Hematology&K… 24/25
3/27/2019 LSMUSIS
8-B
9-B
10 - B
3.132 3.133 3.134
1-C 1-D 1-D
2-A 2-B 2-B
3-D 3-C 3-C
4-B 4-A 5-A
3.135 3.136 3.137 3.138 3.139 3.140 3.141 3.142
1 2 4 1 1 1 1 4
2 4 2 3 3 2
3 3 3
4 4
3.143 3.144 3.145 3.146 3.147 3.148 3.149 3.150
1 1 1 1 1 1 1 1
2 2 2 2 2 2 3 3
3 3 3 3 3 3
4 4 4 4
3.151 3.152 3.153 3.154 3.155 3.156 3.157 3.158
4 1 4 2 4 1 1 1
2 4 2 2 3
3 3 3
4 4 4
3.159 3.160 3.161 3.162 3.163 3.164 3.165 3.166
1 2 2 1 4 1 1 2
2 4 4 2 2 2 4
3 3 3 3
4 4
3.167 3.168 3.169 3.170 3.171 3.172 3.173 3.174
1 1 1 1 1 1 1 1
2 2 2 2 2 2 2 2
3 3 3 3 3 3 3 3
4 4 4
5
3.175 3.176 3.177 3.178 3.179 3.180 3.181 3.182
2 1 1 2 1 1 1 1
4 3 2 4 2 2 2 2
3 3 3 3 3
3.183 3.184 3.185 3.186 3.187 3.188 3.189 3.190
1 1 1 4 1 1 1 1
2 2 3 2 2 2 2
3 3 3 3 3 3
4 4
5
3.191 3.192 3.193 3.194 3.195 3.196
1 1 1 2 1 1
2 2 2 4 2 3
3 3 3 3
4 4
5 5
3.197 - D 3.198 - D 3.199 - D 3.200 - B 3.201 - D 3.202 - D 3.203 - B 3.204 - B
3.205 - C 3.206 - B 3.207 - C 3.208 - B 3.209 - D 3.210 - C 3.211 - C 3.212 - B
3.213 - C 3.214 - D 3.215 - E 3.216 - E 3.217 - C 3.218 - B

lsmusis.lsmuni.lt/Klausimai/Spausdinti?Length=0?Kalba=EN&KategorijaId=105&Kalbos_input=EN&Kalbos=EN&KategorijaEn_input=Hematology&K… 25/25
3/27/2019 LSMUSIS

Dermatovenereology
I type tasks. Choose only one best answer

20.1 The primary lesion of urticaria:


A. Macule
B. Wheal
C. Vesicle
D. Papule
E. Plaque

20.2 Which proposition is wrong about psoriasis:


A. The lesion is a wheal
B. Erythroderma can occur
C. Auspitz sign is positive
E. Psoriasis can affect scalp

20.3 A positive Nikolsky’s sign is characteristic for:


A. Herpes simplex virus infection
B. Pemphigus
C. Psoriasis
D. Herpes zoster virus infection

20.4 A positive patch test confirms the diagnosis of:


A. Urticaria
B. Contact allergic dermatitis (contact allergy)
C. Contact irritant (toxic) dermatitis
D. Seborrheic dermatitis

20.5 A gluten–free diet is recommended for the treatment of:


A. Lichen planus
B. Erythema nodosum
C. Atopic dermatitis
D. Dermatitis herpetiformis

20.6 The most effective agent to treat acne conglobata is:


A. Benzoyl peroxide
B. Isotretinoin
C. Terbinafin
D. Vitamin B complex

20.7 Dark–field microscopic examination is used to detect:


A. Chlamydia trachomatis
B. Trichomona vaginalis
C. Treponema pallidum
D. Mycoplasma hominis
E. Escherichia coli

20.8 Which laboratory examination is not informative diagnostic approach for urogenital chlamydial infection:
A. Cell culture
B. Direct fluorescent antibody assays (DFA)
C. Serological tests
D. DNA amplification

20.9 Wood’s light examination is used for diagnostics of:


A. Psoriasis
B. Tinea capitis caused by Epidermophyton infection
C. Tinea capitis caused by Microsporon infection
D. Scleroderma circumscripta

20.10 The confirmative laboratory test for neurosyphilis:


A. RPR
B. TPHA
lsmusis.lsmuni.lt/Klausimai/Spausdinti?Length=0?Kalba=EN&KategorijaId=122&Kalbos_input=EN&Kalbos=EN&KategorijaEn_input=Dermatovenere… 1/10
3/27/2019 LSMUSIS
C. Dark–field microscopy
D. VDRL–CSF

20.11 Which proposition is wrong about scabies:


A. Typical locations of skin lesions are interdigital webs of hands and feet, wrists and buttocks
B. Typical locations of skin lesions are face and back
C. Pruritus usually worst at night
D. Papules or vesicles in pairs are characteristic

20.12 Which tumour of the skin can be treated using topical immunomodulator imiquimod:
A. Superficial basal cell carcinoma
B. Squamous cell carcinoma
C. Melanoma
D. Dermatofibrosarcoma

20.13 Malignant skin tumour, which very rarely gives metastasis:


A. Squamous cell carcinoma
B. Melanoma
C. Basal cell carcinoma
D. Cutaneous lymphoma

20.14 Which method is used to confirm the diagnosis of basal cell carcinoma:
A. Dermatoscopy
B. Siascopy
C. Cytological investigation
D. Histological investigation

20.15 Non–invasive method to detect the depth of melanomas:


A. Dermatoscopy
B. High frequency ultrasound
C. Cytological investigation
D. Skin biopsy

20.16 Non–invasive an optical instrument with a light source for diagnostics of melanoma:
A. Loupe
B. Microscopical examination
C. Dermatoscope
D. Ultrasound

20.67 What is the primary lesion of lichen ruber planus:


A. Wheal
B. Macule
C. Lichenification
D. Papule

20.68 What is the average incubation period of syphilis:


A. 2-5 days
B. 5-10 days
C. More than 1 month
D. 3 weeks (21 day)

20.69 Intraepidermal bulla is characteristic for:


A. Herpes simplex virus infection
B. Pemphigus vulgaris
C. Bullous pemphigoid
D. Dermatitis herpetiformis (Duhring's disease)

II type tasks. For each numbered item,selct the one lettered heading that is most closely asssciated with it

20.17 What diseases have these characteristic features:


1 - Wickham’s striae
2 - Auspitz sign
3 - Nikolsky’s sign
lsmusis.lsmuni.lt/Klausimai/Spausdinti?Length=0?Kalba=EN&KategorijaId=122&Kalbos_input=EN&Kalbos=EN&KategorijaEn_input=Dermatovenere… 2/10
3/27/2019 LSMUSIS
4 - Vesicles or papules in pairs and burrows of mites
5 - Ulcus durum
A. Psoriasis
B. Pemphigus vulgaris
C. Lichen planus
D. Syphilis
E. Scabies

20.18 What microorganisms can cause these diseases:


1 - Candidiasis
2 - Condyloma accuminatum
3 - Nongonococcal urethritis
4 - Pityriasis versicolor
A. Papilloma virus hominis
B. Candida albicans
C. Pityrosporum orbiculare
D. Chlamydia trachomatis

20.19 What microorganisms can cause these diseases:


C 1 - Scabies
D 2 - Condyloma accuminatum
A 3 - Syphilis
E 4 - Tinea capitis
B 5 - Furuncle
A. Treponema pallidum
B. Staphylococcus aureus
C. Sarcoptes scabiei
D. Papilloma virus hominis
E. Microsporum canis

20.20 What diseases do we treat with these medications:


1 - Itraconazole
2 - Acitretin
3 - Isotretinoin
4 - Acyclovir and valacyclovir
5 - Dapsone (diamindiphenilsulfon)
A. Acne conglobata
B. Onychomycosis
C. Psoriasis
D. Herpes genitalis
E. Dermatitis herpetiformis

20.21 What laboratory examinations confirm these sexually transmitted diseases:


2 - Neurosyphilis
3 - Secondary syphilis
4 - Chlamydial infection
A. VDRL – CSF
B. PCR or LCR of nucleic acids
C. TPHA, RPR

20.66 What skin lesions are characteristic for these diseases:


1 - Herpes virus infection
2 - Lichen planus
3 - Bullous pempfigoid
4 - Erythema multiforme
A. Grouped vesicles
B. Reddish violaceous pruritic papules
C. Tense blisters
D. Target lesions

III type tasks. For each question there is one or more correct answers:
A – if correct answers are 1,2,3
lsmusis.lsmuni.lt/Klausimai/Spausdinti?Length=0?Kalba=EN&KategorijaId=122&Kalbos_input=EN&Kalbos=EN&KategorijaEn_input=Dermatovenere… 3/10
3/27/2019 LSMUSIS
B – if correct answers are 1 and 3
C – if correct answers are 2 and 4
D – if correct answer is 4
E – if correct are all answers above

20.22 Reiter’s syndrome presents with:


1. Urethritis
2. Arthritis
3. Conjunctivitis
4. White dermographism
5. Alopecia diffusa

20.23 AIDS associated dermatoses are:


1. Oropharyngeal candidiasis
2. Herpes zoster
3. Oral hairy leukoplakia
4. Bacillary angiomatosis

20.24 Which diseases can be caused, provoked or aggravated by sun (sun radiation):
1. Condyloma accuminatum
2. Melanoma
3. Onychomycosis
4. Basal cell carcinoma

20.27 Which medications are the most effective for the treatment of herpes genitalis:
1. Prednisolone
2. Calcipotriol
3. Isotretinoin
4. Acyclovir or valacyclovir

20.28 Sites of predilection for psoriasis:


1. Knees
2. Elbows
3. Scalp
4. Oral mucous membrane

20.29 Characteristic clinical features of atopic dermatitis in adults:


1. Skin lesions located in flexural sides of arms and legs
2. Skin lesions located in extensoral sides of arms and legs
3. Lichenification
4. Skin lesions located in scalp area

20.30 Characteristic clinical features of scabies:


1. Itching
2. Nodules in pairs
3. Burrows
4. Auspitz’s sign

20.31 Complications of scabies:


1. Alopecia
2. Blisters
3. Arthritis
4. Pyoderma

20.32 Correct propositions describing psoriasis:


1. Primary lesion – papule or plaque
2. Skin lesions only rarely can appear in scalp area
3. Sometimes pustules can appear
4. Primary lesion – wheal

20.33 Phototherapy is used in treatment of:


1. Psoriasis
lsmusis.lsmuni.lt/Klausimai/Spausdinti?Length=0?Kalba=EN&KategorijaId=122&Kalbos_input=EN&Kalbos=EN&KategorijaEn_input=Dermatovenere… 4/10
3/27/2019 LSMUSIS
2. Localized scleroderma
3. Atopic dermatitis
4. Cutaneous chronic lupus erythematoides

20.34 Clinical presentation of secondary syphilis:


1. Nonpruritic skin lesions
2. Leukoderma
3. Alopecia areata
4. Palmar and plantar papules

20.35 Clinical presentation of primary syphilis:


1. Ulcus durum
2. Palmar and plantar papules
3. Lymphadenitis
4. Syphilitic roseola on the trunk

20.36 Distinguishing features of ulcus durum:


1. Always painful
2. Mostly located in genital area
3. Adjacent tissues are blue–red and painful
4. Ulcus is firm on palpation

20.37 Distinguishing features of regional lymphadenitis in primary syphilis:


1. Mostly located in inguinal area
2. Adhesions with adjacent tissues
3. Painless
4. Painful

20.38 Distinguishing features of herpes zoster:


1. Intense pain
2. Eruption of grouped vesicles
3. Distribution of lesions shows dermatomal limitation
4. Disease is predominant among children

20.39 Kobner phenomenon is characteristic for:


1. Irritant contact dermatitis
2. Psoriasis
3. Impetigo
4. Lichen planus

20.40 Diseases, which often have lesions on the scalp:


1. Dermatitis seborrhoica
2. Microsporum canis infection
3. Psoriasis
4. Pityriasis versicolor

20.41 Precancerous skin lesions, which can develop into squamous cell carcinoma:
1. Actinic keratosis
2. Basal cell carcinoma
3. Bowen’s disease
4. Lipoma

20.42 Significant prognostic features and parameters in melanoma:


1. Thickness and ulceration of the tumour
2. Asymmetry
3. Areas of regression of the tumour
4. Dark brown colour of the tumour

20.43 The most common and very fast growing types of melanoma:
1. Superficial spreading melanoma
2. Lentigo maligna melanoma
3. Nodular melanoma
4. Amelanotic non–pigmented melanoma
lsmusis.lsmuni.lt/Klausimai/Spausdinti?Length=0?Kalba=EN&KategorijaId=122&Kalbos_input=EN&Kalbos=EN&KategorijaEn_input=Dermatovenere… 5/10
3/27/2019 LSMUSIS
20.70 What drugs induce lichen planus:
1. Diuretics
2. Antibiotics
3. Antituberculosis agents
4. Local steroids

IV type tasks. Choose only one best answer

20.44 Twenty nine years old patient was admitted to Dermatovenereological Clinic for the evaluation of enlarged
inguinal lymphatic nodes and ulceration in preputium area. Six and eight weeks ago patient had casual sex. On
physical examination there were chestnut and haricot bean size lymphatic nodes in the left and right inguinal
areas, respectively. They were painless and freely movable. The ulceration was firm and painless on palpation.
Preliminary diagnosis:
A. Primary syphilis
B. Secondary syphilis
C. Herpes simplex virus infection
D. Furunculus

20.45 Twenty nine years old patient was admitted to Dermatovenereological Clinic for the evaluation of enlarged
inguinal lymphatic nodes and ulceration in preputium area. Six and eight weeks ago patient had casual sex. On
physical examination there were chestnut and haricot bean size lymphatic nodes in the left and right inguinal
areas, respectively. They were painless and freely movable. The ulceration was firm and painless on palpation.
Laboratory tests conforming diagnosis:
A. RPR and TPHA serological tests
B. Acantholytic cells in smear form ulceration using cytological test
C. Culture of microorganisms from ulceration
D. Blood cell count

20.46 After the influenza infection forty five years old woman developed isolated and grouped tightly distended
painful vesicles with clear content. The lesions were located in the perioral region, especially the lips. Later on the
vesicles burst and left behind painful crusting erosions. Treatment with topical antiviral preparations was
ineffective, and lesions disseminated to other sides of the face and neck area. Similar lesions were observed
repeatedly following the catching a cold. Since childhood patient has had atopic dermatitis. Preliminary diagnosis:
A. Pemphigus vulgaris
B. Bullous pemphigoid
C. Disseminated herpes simplex infection
D. Dermatitis herpetiformis

20.47 After the influenza infection forty five years old woman developed isolated and grouped tightly distended
painful vesicles with clear content. The lesions were located in the perioral region, especially the lips. Later on the
vesicles burst and left behind painful crusting erosions. Treatment with topical antiviral preparations was
ineffective, and lesions disseminated to other sides of the face and neck area. Similar lesions were observed
repeatedly following the catching a cold. Since childhood patient has had atopic dermatitis. The most effective
etiological treatment:
A. Amoxicyllin per os
B. Prednisolone per os
C. Acyclovir or valacyclovir per os
D. Hydrocortisone ointment

20.48 Eight months ago 40 years old male patient developed intense headache and weakness in arms.
Consequently he was consulted by neurologist. Laboratory investigations revealed positive TPHA test. Patient was
admitted to Dermatovenereological Clinic. Six years ago patient has had ulceration in genital area and enlarged
lymphatic nodes in inguinal area. Ulceration healed after 5 days course of doxycycline 100 mg 2 times per day,
and patient was not consulted for this reason by any physician. Physical examination did not reveal any lesions on
the skin and mucous membranes.Preliminary diagnosis:
A. Primary syphilis
B. Secondary syphilis
C. Neurosyphilis
D. Late latent syphilis

20.49 Eight months ago 40 years old male patient developed intense headache and weakness in arms.
Consequently he was consulted by neurologist. Laboratory investigations revealed positive TPHA test. Patient was
lsmusis.lsmuni.lt/Klausimai/Spausdinti?Length=0?Kalba=EN&KategorijaId=122&Kalbos_input=EN&Kalbos=EN&KategorijaEn_input=Dermatovenere… 6/10
3/27/2019 LSMUSIS
admitted to Dermatovenereological Clinic. Six years ago patient has had ulceration in genital area and enlarged
lymphatic nodes in inguinal area. Ulceration healed after 5 days course of doxycycline 100 mg 2 times per day,
and patient was not consulted for this reason by any physician. Physical examination did not reveal any lesions on
the skin and mucous membranes. Laboratory test confirming diagnosis:
A. TPHA
B. RPR
C. VDRL–CSF
D. Darkfield microscopy

20.50 Eight months ago 40 years old male patient developed intense headache and weakness in arms.
Consequently he was consulted by neurologist. Laboratory investigations revealed positive TPHA test. Patient was
admitted to Dermatovenereological Clinic. Six years ago patient has had ulceration in genital area and enlarged
lymphatic nodes in inguinal area. Ulceration healed after 5 days course of doxycycline 100 mg 2 times per day,
and patient was not consulted for this reason by any physician. Physical examination did not reveal any lesions on
the skin and mucous membranes. The most effective etiological treatment:
A. Erythromycin 100 mg per day for 10 days
B. Doxycycline 100 mg per day for 5 days
C. Benzathine penicillin in intramuscular injections 2.4 million IU one time per week (3 weeks)
D. Penicillin G in intravenous injections 5 million IU 6 times per day for 14–21 days

20.51 Twenty eight years old woman was admitted to Dermatovenereological Clinic for the evaluation of pruritic
lesions in the periumbilical area, where was direct contact with metallic button of trousers. Sometimes similar
lesions appeared in areas of contact with watch or earrings. Physical examination revealed erythema and
coalescent reddish scaling small papules in the periumbilical area. Preliminary diagnosis:
A. Allergic contact dermatitis (nickel allergy)
B. Irritant contact dermatitis
C. Seborrhoic dermatitis
D. Lichen planus

20.52 Twenty eight years old woman was admitted to Dermatovenereological Clinic for the evaluation of pruritic
lesions in the periumbilical area, where was direct contact with metallic button of trousers. Sometimes similar
lesions appeared in areas of contact with watch or earrings. Physical examination revealed erythema and
coalescent reddish scaling small papules in the periumbilical area. Test confirming diagnosis:
A. Patch testing
B. Microscopical examination
C. Prick testing
D. Diascopy

20.53 Twenty eight years old woman was admitted to Dermatovenereological Clinic for the evaluation of pruritic
lesions in the periumbilical area, where was direct contact with metallic button of trousers. Sometimes similar
lesions appeared in areas of contact with watch or earrings. Physical examination revealed erythema and
coalescent reddish scaling small papules in the periumbilical area. Management:
A. Hydrocortisone ointment and avoidance of contact allergen
B. Calcipotriol ointment
C. Hydrogene peroxide solution
D. Clotrimasole cream
E. Acidum fucidicum ointment

20.54 Thirty eight years old male patient developed multiple, sharply defined, yellowish, with pityriasiform scaling
macular lesions in the central parts of the back and chest. On the exposure to the sun the lesions exfoliated and
became reddish and later on whitish. There was observed positive iodine test in the skin of lesion (higher
absorption of iodine solution). Preliminary diagnosis:
A. Leukoderma syphiliticum
B. Vitiligo
C. Pityriasis versicolor
D. Erytrasma
E. Pityriasis rosea

20.55 Thirty eight years old male patient developed multiple, sharply defined, yellowish, with pityriasiform scaling
macular lesions in the central parts of the back and chest. On the exposure to the sun the lesions exfoliated and
became reddish and later on whitish. There was observed positive iodine test in the skin of lesion (higher
absorption of iodine solution). Laboratory test confirming diagnosis:
lsmusis.lsmuni.lt/Klausimai/Spausdinti?Length=0?Kalba=EN&KategorijaId=122&Kalbos_input=EN&Kalbos=EN&KategorijaEn_input=Dermatovenere… 7/10
3/27/2019 LSMUSIS
A. Wood's light examination
B. Dermatoscopy
C. Microscopical examination of scales
D. Diascopy

20.56 Thirty eight years old male patient developed multiple, sharply defined, yellowish, with pityriasiform scaling
macular lesions in the central parts of the back and chest. On the exposure to the sun the lesions exfoliated and
became reddish and later on whitish. There was observed positive iodine test in the skin of lesion (higher
absorption of iodine solution). Treatment:
A. Clotrimasole cream
B. Hydrocortisone ointment
C. Erythromycin gel
D. Metronidasole cream

20.57 Thirty years old male patient since childhood had recurrent head cold and pruritic eruptions in the face, neck
and flexural elbow areas. The disease mostly exacerbated in spring time. The skin in the flexural areas of elbow
was reddish, slightly infiltrated, with moderate scaling and crusted excoriations. There were observed thickened
infraorbital folds and scaling erythema in the periorbital areas. Laboratory tests have revealed increased IgE in the
serum. Preliminary diagnosis:
A. Irritant contact dermatitis
B. Allergic contact dermatitis
C. Atopic dermatitis
D. Infectious dermatitis

20.58 Thirty years old male patient since childhood had recurrent head cold and pruritic eruptions in the face, neck
and flexural elbow areas. The disease mostly exacerbated in spring time. The skin in the flexural areas of elbow
was reddish, slightly infiltrated, with moderate scaling and crusted excoriations. There were observed thickened
infraorbital folds and scaling erythema in the periorbital areas. Laboratory tests have revealed increased IgE in the
serum. Distinguishing clinical feature for this disease:
A. White dermographism
B. Red dermographism
C. Urticarial dermographism
D. Nikolsky's sign

20.59 Thirty years old male patient since childhood had recurrent head cold and pruritic eruptions in the face, neck
and flexural elbow areas. The disease mostly exacerbated in spring time. The skin in the flexural areas of elbow
was reddish, slightly infiltrated, with moderate scaling and crusted excoriations. There were observed thickened
infraorbital folds and scaling erythema in the periorbital areas. Laboratory tests have revealed increased IgE in the
serum. Test confirming diagnosis:
A. Patch testing with contact allergens
B. Prick testing with food and inhalant allergens
C. Microscopic test
D. Blood cell count

20.60 One year ago fifty five years old male patient on his left arm developed slowly enlarging 0,5 cm in diameter
dark brown plaque with irregular borders. Peripheral lymphatic nodes were not enlarged on palpation. There were
irregular streaks at periphery of the lesion on dermatoscopic examination. Using high frequency ultrasound the
depth of tumour was unmeasured. Preliminary diagnosis:
A. Bowen's disease
B. Pigmented nevus
C. Pigmented basal cell carcinoma
D. Superficial spreading melanoma

20.61 One year ago fifty five years old male patient on his left arm developed slowly enlarging 0,5 cm in diameter
dark brown plaque with irregular borders. Peripheral lymphatic nodes were not enlarged on palpation. There were
irregular streaks at periphery of the lesion on dermatoscopic examination. Using high frequency ultrasound the
depth of tumour was unmeasured. Examination confirming diagnosis:
A. Cytological examination
B. Ultrasound of internal organs
C. Biopsy of the tumour and histological examination
D. Excision of the tumour and histological examination

lsmusis.lsmuni.lt/Klausimai/Spausdinti?Length=0?Kalba=EN&KategorijaId=122&Kalbos_input=EN&Kalbos=EN&KategorijaEn_input=Dermatovenere… 8/10
3/27/2019 LSMUSIS
20.62 Thirty three years old white women with bright hears and abundant freckles was admitted to
Dermatovenereological Clinic for the evaluation of slowly enlarging pigmented lesion on the extensor surface of
her left thin. The lesion developed ten tears ago. There was asymmetric, sharply limited, dark brown patch the size
of a small coin, with irregular polycyclic borders and irregular uneven surface. Central part of the lesion had whitish
non–pigmented. There were no observed palpable regional lymphatic nodes. Dermatoscopy showed
distinguishing features of melanoma. Radiological examination did not show metastases in the lymphatic nodes
and internal organs. Which type of melanoma can be preliminary diagnosed:
A. Nodular melanoma
B. Superficial spreading melanoma
C. Amelanotic non–pigmented melanoma
D. Acrolentiginous melanoma

20.63 Thirty three years old white women with bright hears and abundant freckles was admitted to
Dermatovenereological Clinic for the evaluation of slowly enlarging pigmented lesion on the extensor surface of
her left thin. The lesion developed ten tears ago. There was asymmetric, sharply limited, dark brown patch the size
of a small coin, with irregular polycyclic borders and irregular uneven surface. Central part of the lesion had whitish
non–pigmented. There were no observed palpable regional lymphatic nodes. Dermatoscopy showed
distinguishing features of melanoma. Radiological examination did not show metastases in the lymphatic nodes
and internal organs. Non–invasive examination for the measuring of thickness of melanoma:
A. Dermatoscopy
B. High frequency ultrasound
C. Video–dermatoscopy
D. Biopsy of the skin with histological examination

20.64 Thirty three years old white women with bright hears and abundant freckles was admitted to
Dermatovenereological Clinic for the evaluation of slowly enlarging pigmented lesion on the extensor surface of
her left thin. The lesion developed ten tears ago. There was asymmetric, sharply limited, dark brown patch the size
of a small coin, with irregular polycyclic borders and irregular uneven surface. Central part of the lesion had whitish
non–pigmented. There were no observed palpable regional lymphatic nodes. Dermatoscopy showed
distinguishing features of melanoma. Radiological examination did not show metastases in the lymphatic nodes
and internal organs. Thickness of the tumour as indication for surgical treatment of melanoma in combination with
sentinel lymphatic node biopsy:
A. Less than 1 mm
B. More than 1 mm
C. More than 2 mm
D. More than 3 mm

20.65 Thirty three years old white women with bright hears and abundant freckles was admitted to
Dermatovenereological Clinic for the evaluation of slowly enlarging pigmented lesion on the extensor surface of
her left thin. The lesion developed ten tears ago. There was asymmetric, sharply limited, dark brown patch the size
of a small coin, with irregular polycyclic borders and irregular uneven surface. Central part of the lesion had whitish
non–pigmented. There were no observed palpable regional lymphatic nodes. Dermatoscopy showed
distinguishing features of melanoma. Radiological examination did not show metastases in the lymphatic nodes
and internal organs. Histological examination showed the thickness of the primary tumour according Breslow being
2.5 mm and micrometastases in the excised sentinel lymphatic node. Further examination and corresponding
treatment:
A. Ultrasound examination of regional lymphatic nodes and adjuvant therapy with interferon
B. Follow–up of the patient
C. Radical extirpation of the regional lymphatic nodes and adjuvant therapy with interferon
D. Adjuvant therapy and chemotherapy

20.71 Thirty years old female noticed a reddish patch on the trunk. When several weeks passed the lesion
became indurated and ivory white in color with a lilac edge. After a year the patient referred to a
dermatovenereologist. There was an oval patch 7cm in diametre on the front trunk. The surface of the lesion was
smooth, shiny and hairless. Preliminary diagnosis:
A. Vitiligo
B. Circumscribed scleroderma (morphea)
C. Pityriasis versicolor
D. Dermatomyositis

20.72 Thirty years old female noticed a reddish patch on the trunk. When several weeks passed the lesion
became indurated and ivory white in color with a lilac edge. After a year the patient referred to a
lsmusis.lsmuni.lt/Klausimai/Spausdinti?Length=0?Kalba=EN&KategorijaId=122&Kalbos_input=EN&Kalbos=EN&KategorijaEn_input=Dermatovenere… 9/10
3/27/2019 LSMUSIS
dermatovenereologist. There was an oval patch 7cm in diametre on the front trunk. The surface of the lesion was
smooth, shiny and hairless. Test confirming diagnosis:
A. Video-dermoscopy
B. Wood's lamp examination
C. Biopsy of the skin with histological examination
D. Blood cell count, biochemical tests

20.1 - B 20.2 - A 20.3 - B 20.4 - B 20.5 - D 20.6 - B 20.7 - C 20.8 - C


20.9 - C 20.10 - D 20.11 - B 20.12 - A 20.13 - C 20.14 - D 20.15 - B 20.16 - C
20.67 - D 20.68 - D 20.69 - B
20.17 20.18 20.19 20.20 20.21 20.66
1-C 1-B 1-C 1-B 2-A 1-A
2-A 2-A 2-D 2-C 3-C 2-B
3-B 3-D 3-A 3-A 4-B 3-C
4-E 4-C 4-E 4-D 4-D
5-D 5-B 5-E
20.22 20.23 20.24 20.27 20.28 20.29 20.30 20.31
1 1 2 4 1 1 1 4
2 2 4 2 3 2
3 3 3 3
4
20.32 20.33 20.34 20.35 20.36 20.37 20.38 20.39
1 1 1 1 2 1 1 2
3 2 2 3 4 3 2 4
3 3 3
4
20.40 20.41 20.42 20.43 20.70
1 1 1 1 1
2 3 3 3 2
3 3
20.44 - A 20.45 - A 20.46 - C 20.47 - C 20.48 - C 20.49 - C 20.50 - D 20.51 - A
20.52 - A 20.53 - A 20.54 - C 20.55 - C 20.56 - A 20.57 - C 20.58 - A 20.59 - B
20.60 - D 20.61 - D 20.62 - B 20.63 - B 20.64 - B 20.65 - C 20.71 - B 20.72 - C

lsmusis.lsmuni.lt/Klausimai/Spausdinti?Length=0?Kalba=EN&KategorijaId=122&Kalbos_input=EN&Kalbos=EN&KategorijaEn_input=Dermatovener… 10/10
3/27/2019 LSMUSIS

Infectious diseases
I type tasks. Choose only one best answer

18.1 Which pathogen is the most common cause of travelers diarrhea:


A. Leptospires
B. Staphylococci
C. Escherichia
D. Clostridia
E. Pneumococci

18.2 Which pathogen is the most common cause of food poisoning infection:
A. Rotaviruses
B. Clostridium difficile
C. Leptospira
D. Streptococcus
E. Salmonella

18.3 Which microorganism is the most common cause of sepsis after splenectomy:
A. Staphylococcus aureus
B. Listeria monocytogenes
C. Pneumococcus
D. Salmonella

18.4 Which disease is characterized by normal or subfebrile temperature, dyspeptic and bulbar symptoms,
meteorism, and visual impairment:
A. Poisoning, caused by mushrooms
B. Botulism
C. Lyme borreliosis
D. Tick-borne encephalitis

18.5 Which disease is characterized by fever, arthralgias, red and swollen palms and soles (“gloves” and “socks”),
symmetric rash around the joints and sometimes jaundice:
A. Hepatitis A
B. Trichinellosis
C. Leptospirosis
D. Toxoplasmosis
E. Pseudotuberculosis (Yersiniosis)

18.6 Which disease is characterized by the same time occurring gastroenteritis, catarrh of the respiratory tract,
and conjunctivitis:
A. Yersiniosis
B. Viral gastroenteritis
C. Shigellosis
D. Salmonellosis
E. Trichinellosis

18.7 What is the most common outcome of viral Hepatitis A:


A. Chronic hepatitis
B. Gilbert syndrome
C. Liver cirrhosis
D. Recovery

18.8 What clinical symptoms characterize trichinosis:


A. Headache, nausea, vomiting
B. Weakness, arthralgia, dark urine
C. Swelling, redden and pustulary erupted eyelids
D. Fever, muscle pain, periorbital edema, subconjunctival splinter haemorrhages
E. Fever, diarrhea, increased lymphic nodes of the neck

18.9 What clinical symptoms are common for taeniasis:


A. Epileptic cramps

lsmusis.lsmuni.lt/Klausimai/Spausdinti?Length=0?Kalba=EN&KategorijaId=120&Kalbos_input=EN&Kalbos=EN&KategorijaEn_input=Infectious+dise… 1/12
3/27/2019 LSMUSIS
B. Patients find the peaces of tapeworms in their faeces
C. Abdominal pain, tenesmes
D. Nausea, ache under the right arch of ribs
E. Increased liver

18.10 Which laboratory test confirms HIV infection:


A. Widal agglutination test
B. Western blot or PCR
C. Blood culture
D. Immunogram

18.11 What symptoms characterize primary HIV infection:


A. Fever, bloody diarrhea
B. Fever, lymphadenopathy, maculopapular rash
C. Fever, arthralgies, redden and swelling palms and soles
D. Dyspeptic symptoms, jaundice, hepatomegaly

18.12 What laboratory test specify the diagnosis of tick borne encephalitis::
A. Enzyme-linked immunosorbent assay (ELISA) for detection of specific antibodies in the blood
B. Bacterioscopic test of the blood
C. Blood culture
D. Bacteriological test of the cerebrospinal fluid

18.13 What features characterise the meningococcal infection:


A. Acute onset, high temperature, vomiting, meningeal symptoms
B. Acute onset, normal temperature, meningeal symptoms
C. Silent onset, high temperature, vomiting, diarrhea
D. Acute onset, normal temperature, vomiting

18.14 What disease could be suspected if diarrhea and weight loss last for one month and longer:
A. Norovirus infection
B. Food borne infection
C. HIV infection
D. Adenoviral infection
E. Rotaviral infection

18.15 What tests are recommended for diagnosis of prenatal toxoplasmosis:


A. Blood culture
B. Detection of specific IgM and IgG antibodies and ultrasound diagnosis of the foetus
C. Bacteriological test of the blood and ultrasound diagnosis of the foetus
D. General blood test

18.16 What test confirms the diagnosis of pneumocystosis:


A. Bacteriological test of the urine
B. Coprogram
C. Histopathologic or cytopathologic demonstration of pathogen in bronchoalveolar lavage
D. Serological test of the blood

18.17 What disease is characterised as annular erythema with clear margins, spreading to the periphery:
A. Malaria
B. Tick borne encephalitis
C. Lyme borreliosis
D. Varicella zoster
E. Pseudotuberculosis

18.18 What test confirms the diagnosis of faucial diphtheria:


A. Serological tests in pair serums
B. Bacteriological test of patch, taken from tonsils
C. Blood culture
D. Allergical test of the skin

18.19 What disease could be suspected if episodes of fever with chills occur every third day and last for 5-8 hours:
A. Influenza
lsmusis.lsmuni.lt/Klausimai/Spausdinti?Length=0?Kalba=EN&KategorijaId=120&Kalbos_input=EN&Kalbos=EN&KategorijaEn_input=Infectious+dise… 2/12
3/27/2019 LSMUSIS
B. Sepsis
C. Tropical malaria
D. P. vivax caused malaria

18.20 What are the indications of antiretroviral treatment of HIV infection:


A. Laboratory confirmed HIV infection
B. HIV-RNA >10 000 copies/ml
C. CD₄ T cell count< 500 per mm³
D. Laboratory confirmed primary immunodeficiency

18.21 Listeria monocytogenes strains are genetically resistant to:


A. Cephalosporins
B. Ampicillin
C. Penicillin
D. Trimethoprim-sulfamethoxazole

18.22 Haemophilus influenza type b (Hib) vaccine is given:


A. For all infants, starting at 2 months of age
B. Before solid organ transplantation
C. For people having regular contact with animals (hunters, veterinarians, etc.)
D. For all people above 65 years of age

II type tasks. For each numbered item,selct the one lettered heading that is most closely asssciated with it

18.23 Which circumstances indicate which CNS infection:


1 - Meningococcal meningitis
2 - Listeria monocytogenes meningitis
3 - Pneumococcal meningitis
4 - Neuroborreliosis
A. If otitis or sinusitis is a concomitant infection
B. If haemorrhagic rash is present
C. If the patient is over the age of 50 or a pregnant woman
D. If paralysis of the facial nerve is present

18.24 What disorders of the skin are typical for these diseases:
1 - Pseudotuberculosis
2 - Meningococical sepsis
3 - Lyme borreliosis
4 - Typhoid fever
A. Rose rash, starting in the 8-10 day of disease
B. Swelling and redness of the face, palms and soles (“hood”, “gloves” and “socks”)
C. Petechial rash
D. Circular, outwardly expanding rash

18.25 Which symptoms are typical for these diseases:


1 - Influenza
2 - Parainfluenza
3 - Adenoviral infection
4 - Infectious mononucleosis
A. Conjunctivitis, tracheitis, bronchitis
B. Increased lymph nodes, conjunctivitis, tonsillopharyngitis
C. Hepatomegaly, splenomegaly, increased lymph nodes, tonsillitis, fever
D. Laryngitis

18.26 What syndromes are typical for these diseases:


1 - Infectious mononucleosis
2 - Hepatitis B
3 - Salmonellosis
A. Jaundice syndrome
B. Lymphadenitis syndrome
C. Gastroenteral syndrome

lsmusis.lsmuni.lt/Klausimai/Spausdinti?Length=0?Kalba=EN&KategorijaId=120&Kalbos_input=EN&Kalbos=EN&KategorijaEn_input=Infectious+dise… 3/12
3/27/2019 LSMUSIS
18.27 What symptom or complex of symptoms is typical for each of these diseases:
1 - Toxocariasis
2 - Enterobiosis
3 - Echinococcosis
4 - Trichinosis
A. Periorbital and facial edema, fever, muscle pains
B. Bronchitis with bronchospastic component
C. Itching of the anus
D. Cysts detected by ultrasound.

18.28 Which symptoms are typical for these diseases:


1 - Meningococcal infection
2 - Rotavirus infection
3 - Infectious mononucleosis
A. Enteritis
B. Polyadenitis, hepatolienal syndrome
C. Meningitis

18.29 What symptoms characterize these diseases:


1 - Parainfluenza
2 - Viral hepatitis A
3 - Epidemic myalgia
A. Muscle pain
B. Laryngitis
C. Jaundice

18.30 What symptoms characterize these diseases:


1 - Botulism
2 - Rabies
3 - Tick borne encephalitis
4 - Tetanus
A. Trismus, risus sardonicus, opisthotonos, the episodes of tonic and clonic seizure
B. Fever, headache, slow paralyses of upper limb and neck muscles
C. Agitation, hydrophobia, dysphagia, hoarseness, dilated pupils
D. Hoarseness, dysarthria, double vision, strabismus, drooping of both eyelids, shortness of breath

18.31 Select specific risk factor for each of these infections:


1 - Pneumococcal infection
2 - Meningococcal infection
3 - Listeria monocytogenes infection
4 - Pseudomonas aeruginosae infection
A. Impaired cell mediated immunity
B. Visiting some African regions
C. Asplenia
D. Neutropenia

18.32 Which test is the most significant for the diagnosis of each of these diseases:
1 - Neuroborreliosis
2 - Infectious mononucleosis
3 - Acute viral hepatitis B
4 - Leptospirosis
A. ELISA test for detection of specific antibodies against viral capsid antigen
B. Anti HBc IgM in the serum
C. Microagglutination test
D. The detection of specific antibodies in CSF

18.33 Which antibiotic should be prescribed for the treatment of these diseases:
1 - Staphylococcal infection
2 - Lyme borreliosis
3 - Meningococcal infection
A. Doxycycline
B. Oxacillin
lsmusis.lsmuni.lt/Klausimai/Spausdinti?Length=0?Kalba=EN&KategorijaId=120&Kalbos_input=EN&Kalbos=EN&KategorijaEn_input=Infectious+dise… 4/12
3/27/2019 LSMUSIS
C. Penicillin

18.34 What method confirms the diagnosis:


1 - Diphtheria
2 - Typhoid fever
3 - Pneumocystosis
A. Blood culture
B. Bacterioscopical and bacteriological examination of a patch, taken from the throat
C. Special staining methods of respiratory secretion for the detection of the pathogen

18.35 Select the first choice antibiotic for each of these infections treatment:
1 - Mycoplasma pneumoniae
2 - Pneumocystis jiroveci pneumoniae
3 - Listeria monocytogenes meningitis
A. Ampicillin
B. Trimethoprim - sulfamethoxasole
C. Clarithromycin

18.36 Which drug should be prescribed to treat each of these diseases:


1 - Influenza
2 - Lyme disease
3 - Diphtheria
4 - Shigellosis
A. Ciprofloxacin
B. Oseltamivir
C. Doxycycline
D. Specific antitoxin

18.37 What pathogen causes each of these diseases:


- Erysipelas
- Tick borne encephalitis
- Malaria
A. Protozoa
B. Bacteria
C. Viruses

18.38 What immunological markers confirm the diagnosis:


- Hepatitis A
- Hepatitis B
- Hepatitis D
A. HBsAg, anti-HBc IgM
B. HBsAg, anti—HD IgM
C. Anti—HAV IgM

18.39 What test confirms the diagnosis:


- Toxocariasis
- Enterobiasis
- Ascariasis
A. The ova in the patch, taken from anus
B. The detection of specific antibodies in the blood
C. The ova of a.lumbricoides in the faeces

III type tasks. For each question there is one or more correct answers:
A – if correct answers are 1,2,3
B – if correct answers are 1 and 3
C – if correct answers are 2 and 4
D – if correct answer is 4
E – if correct are all answers above

18.40 What is typical for pneumococcal infection:


1. It can cause sepsis and meningitis
2. It causes a great risk of rapidly progressive infection after splenectomy
lsmusis.lsmuni.lt/Klausimai/Spausdinti?Length=0?Kalba=EN&KategorijaId=120&Kalbos_input=EN&Kalbos=EN&KategorijaEn_input=Infectious+dise… 5/12
3/27/2019 LSMUSIS
3. It can cause otitis, sinusitis, pneumoniae
4. It can cause urinary tract infections
5. First choice treatment is metronidazole

18.41 Which serological markers can be detected in chronic Hepatitis B infection:


1. HBsAg
2. Anti-HBs
3. HBeAg
4. Anti-HAV IgM

18.42 Which serological markers are common in acute viral Hepatitis B infection:
1. Anti-HBc IgM
2. HBs Ag
3. HBe Ag
4. Anti-HBs

18.43 The diagnosis of trichinosis could be confirmed by:


1. Demonstration of trichinae in meat
2. Positive serological tests
3. Eosinophilic leukocytosis
4. Detection of trichinae larvae in the stool

18.44 What are clinical features of salmonella infection:


1. Sudden onset, fever
2. Nausea, vomiting
3. Watery diarrhea
4. Meningeal symptoms
5. Herpetic rash

18.45 Diphtheria is treated with:


1. Erythromycin
2. Chloramphenicol
3. Diphtheria antitoxin
4. Diphtheria toxoid

18.46 What are the possible outcomes of acute Hepatitis B:


1. Recovery
2. Chronic hepatitis
3. Carriers of HBs Ag
4. Cirrhosis

18.47 The following infections may be caused by enteroviruses:


1. Herpangina
2. Herpes zoster
3. Epidemic myalgia
4. Infectious mononucleosis

18.48 The diagnosis of malaria could be confirmed by:


1. Detection of parasites in leukocytes of cerebrospinal fluid
2. Stool test
3. Blood culture
4. Microscopic examination of thick and thin blood smears

18.49 Clinical symptoms of botulism are:


1. Dyspepsia
2. Visual disturbance
3. Dysphagia, dyspnea
4. Hallucination
5. Jaundice

18.50 Symptoms of HIV infection pre AIDS stage (cat. B) are:


1. Lymphadenopathy
2. Weight loss
lsmusis.lsmuni.lt/Klausimai/Spausdinti?Length=0?Kalba=EN&KategorijaId=120&Kalbos_input=EN&Kalbos=EN&KategorijaEn_input=Infectious+dise… 6/12
3/27/2019 LSMUSIS
3. Over 1 month lasting diarrhea with no apparent cause
4. Over 1 month lasting intermittent fever

18.51 Which microorganisms are the most common cause of skin and soft tissue infections:
1. Listeria monocytogenes
2. S.pyogenes
3. Proteus
4. S.aureus
5. S.pneumoniae

18.52 What is typical for legionellosis:


1. It can cause severe pneumoniae or Pontiac fever
2. It is resistant to beta-lactamic antibiotics
3. It is treated with macrolides or quinolones
4. It can cause intestinal bleeding

18.53 What are clinical features of shigellosis:


1. Jaundice
2. Frequent, bloody, mucoid stools
3. Liver enlargement
4. Tenesmus

18.54 Tick borne encephalitis is transmitted to humans by:


1. Ingestion of raw milk of cow or goat
2. Ingestion of food contaminated with oocysts
3. Tick bite
4. Animal products (brushes, woolen articles)
5. Direct contact with infected secretions, usually respiratory droplets

18.55 What are clinical symptoms of meningococcal infection:


1. Sudden onset and fever
2. Meningeal symptoms
3. Large number of polymorphonuclear leukocytes in cerebrospinal fluid
4. Liver enlargement and splenomegaly

18.56 Clinical symptoms of erysipelas are:


1. Chills
2. Seizures
3. Limited skin redness
4. Conjunctivitis

18.57 Post exposure prophylaxis of rabies is:


1. Local desinfection of the wound
2. Antiviral therapy
3. Vaccination
4. Glucocorticoids

18.58 Lyme borreliosis is transmitted by:


1. Drinking unboiled cow‘s milk
2. Drinking unboiled goat‘s milk
3. Blood transfusions
4. Tick bite

18.59 Which prophylactic measures are used for the prevention of meningococcal infection:
1. Patient's isolation for 24 h from the beginning of antibacterial treatment
2. Chemoprophylaxis for those who had a contact with respiratory secretions of the person with
meningococcal infection
3. Vaccination of the contact persons in case of an outbreak
4. Specific immunoglobulin during 48 h after a contact with the person with meningococcal infection

18.60 Select markers of effective HIV antiretroviral treatment:


1. Clinical improvement
2. Viral load is undetectable or <50 copies per ml
lsmusis.lsmuni.lt/Klausimai/Spausdinti?Length=0?Kalba=EN&KategorijaId=120&Kalbos_input=EN&Kalbos=EN&KategorijaEn_input=Infectious+dise… 7/12
3/27/2019 LSMUSIS
3. ELISA and WB become negative
4. Increased CD4 cell count

IV type tasks. Choose only one best answer

18.61 A 52 - year - old woman noticed an erythematous macule on the right calf at the site of a tick bite three days
ago. Redness is an annular form centrifugally expanding with a bright margins. Body temperature is normal,
regional lymph nodes are not increased. The initial diagnosis is:
A. Erysipelas
B. Lyme borreliosis
C. Herpes zoster
D. Allergy

18.62 A 52 - year - old woman noticed an erythematous macule on the right calf at the site of the tick bite three
days ago. Redness is an annular form centrifugally expanding with a bright margins. Body temperature is normal,
regional lymph nodes are not increased.The medicine of choice is:
A. Doxycycline
B. Trimethoprim - sulfamethoxazole
C. Furazolidone
D. Chloramphenicol

18.63 A 35 - year - old man, pilot, who returned from Guinea, complained of chills, fever up to 39.5°C, that
dropped shortly with profuse sweating after 5 hours. The next attack of fever followed after 3 days. The initial
diagnosis is:
A. Typhoid fever
B. Influenza
C. Malaria
D. Leptospirosis

18.64 A 35 - year - old man, pilot, who returned from Guinea, complained of chills, fever up to 39.5°C, that
dropped shortly with profuse sweating after 5 hours. The next attack of fever followed after 3 days. Laboratory
tests helpful for the diagnosis are:
A. Demonstration of parasites in blood (microscopy examination of a thin smear)
B. Demonstration of parasites in blood (microscopy examination of a thick smear)
C. Demonstration of parasites in blood (microscopy examination of a thin and thick smears)
D. Serological tests

18.65 A 25 - year - old woman was admitted to the department of infectious diseases for examination of three
months lasting sub-febrile fever, asthmatic bronchitis, leukocytosis with eosinophilia. The initial diagnosis is:
A. Trichinosis
B. Septicemia
C. Toxocariasis
D. Acute HIV infection

18.66 A 25 - year - old woman was admitted to the department of infectious diseases for examination of three
months lasting sub-febrile fever, asthmatic bronchitis, leukocytosis with eosinophilia. Laboratory test helpful for the
diagnosis is:
A. Widal test
B. Specific serological test for Toxocariasis
C. Detection of antibodies against HIV by immunoassay
D. Blood culture

18.67 The 18-year-old student suddenly became ill: fever till 38.5°C, sore throat. Physical examination: red,
enlarged tonsils with white coating, swollen neck, submandibular, axillary and inguinal lymph nodes, enlarged liver.
Laboratory data: Hb 130 g/l, leukocytes (total) 14.0x10⁹/l, unseg.neutro. 4, seg.neutro. 39, lymphocytes 39,
monocytes 18, ALT 250 U/l. The initial diagnosis is:
A. Infectious mononucleosis
B. Adenoviral infection
C. Hepatitis B
D. Pseudotuberculosis

lsmusis.lsmuni.lt/Klausimai/Spausdinti?Length=0?Kalba=EN&KategorijaId=120&Kalbos_input=EN&Kalbos=EN&KategorijaEn_input=Infectious+dise… 8/12
3/27/2019 LSMUSIS
18.68 The 18-year-old student suddenly became ill: fever till 38.5°C, sore throat. Physical examination: red,
enlarged tonsils with white coating, swollen neck, submandibular, axillary and inguinal lymph nodes, enlarged liver.
Laboratory data: Hb 130 g/l, leukocytes (total) 14.0x10⁹/l, unseg.neutro. 4, seg.neutro. 39, lymphocytes 39,
monocytes 18, ALT 250 U/l. The diagnosis can be confirmed by:
A. HBs antigen in blood
B. Blood culture
C. PCR for the detection of adenoviral DNA from nasopharingeal secreations
D. Detection of heterophilic antibodies and EBV IgM antibodies

18.69 A 45 - year - old businessman was admitted to the hospital with a one-month history of fever, diarrhoea,
weight loss (approximately 5 kg). He reported that he had used a variety of injectable drugs during two last years.
Physical examination: temperature 38°C, swollen neck, axillary, and inguinal lymph nods. The initial diagnosis is:
A. Shigellosis
B. Tuberculosis
C. Typhoid fever
D. HIV infection

18.70 A 45 - year - old businessman was admitted to the hospital with a one-month history of fever, diarrhoea,
weight loss (approximately 5 kg). He reported that he had used a variety of injectable drugs during two last years.
Physical examination: temperature 38°C, swollen neck, axillary, and inguinal lymph nods. Laboratory tests helpful
for diagnosis are:
A. Nasopharyngeal culture
B. Urine culture
C. ELISA, Western blot for detection of specific antibodies
D. Microscopy of cerebro spinal fluid

18.71 A 38 - year - old woman suddenly became ill: temperature increased, crampy abdominal pain more severe
in the left lower region, and bloody, mucoid diarrhoea appeared. The initial diagnosis is:
A. Shigellosis
B. Rota viral infection
C. Cholera
D. Staphylococcal food poisoning

18.72 A 38 - year - old woman suddenly became ill: temperature increased, crampy abdominal pain more severe
in the left lower region, and bloody, mucoid diarrhoea appeared. Laboratory test helpful for the diagnosis is:
A. Stool test
B. Stool culture
C. Stool test and stool culture
D. Serological tests

18.73 A 35-year old hunter complains of fever, headache, nausea and vomiting at the end of July. Two weeks ago
he was ill with influenza-like illness. The preliminary diagnosis is:
A. Rabies
B. Trichinosis
C. Tick-borne encephalitis
D. Lyme disease

18.76 A 40 - year - old farmer suddenly became ill in summer. The main symptoms were severe muscle pain in the
thighs and lumbar areas, chills, fever (39°C), cutaneous haemorrhages and skin rashes, dark urine, oliguria,
jaundice, splenomegaly, hepatomegaly. The early diagnosis could be confirmed by:
A. Blood culture
B. Widal agglutination test
C. Positive microagliutination reaction (MAR)
D. Identification of HBsAg

18.77 A 62-year old woman complains of high fever, abdominal pain, and watery diarrhoea 6-8 times per day. A
few days ago she was taking care of her 2-year-old grandchild with similar symptoms. The preliminary diagnosis
is:
A. Salmonellosis
B. Rota viral infection
C. Campylobacteriosis
D. Shigellosis

lsmusis.lsmuni.lt/Klausimai/Spausdinti?Length=0?Kalba=EN&KategorijaId=120&Kalbos_input=EN&Kalbos=EN&KategorijaEn_input=Infectious+dise… 9/12
3/27/2019 LSMUSIS
18.78 A 62-year old woman complains of high fever, abdominal pain, and watery diarrhoea 6-8 times per day. A
few days ago she was taking care of her 2-year-old grandchild with similar symptoms. Which lab test is needed for
confirmation of the diagnosis:
A. The detection of the viral antigen in stool sample
B. Stool culture
C. Serology tests
D. Stool microscopy

18.81 A 66 - year - old man was admitted to the hospital for examination of acute jaundice. Despite the treatment
patient’s state did not improve: jaundice increased, consciousness disorders appeared, cutaneous haemorrhages
were noticed. This patients state is estimated as:
A. Normal course of viral hepatitis B
B. Fulminant hepatitis
C. Chronic active hepatitis
D. Cholestatic form of hepatitis

18.82 A 66 - year - old man was admitted to the hospital for examination of acute jaundice. Despite the treatment
patient’s state did not improve: jaundice increased, consciousness disorders appeared, cutaneous haemorrhages
were noticed. The severity of the patient’s state can be estimated by:
A. Prothrombin time (SPA)
B. Gamma glutamyltranspeptidase in serum
C. Alkaline phosphatase in serum
D. Glucose in blood

18.83 A 60 - year - old housewife suddenly became ill. The main complaints were fever and pain in the right side
of a chest. After 2 days her skin on the right side of the chest became red. Vesicular rash within a dermatome
associated with severe pain appeared. The initial diagnosis is:
A. Erysipelas
B. Chickenpox
C. Lyme disease
D. Herpes zoster

18.85 A 32 - year - old woman was gone with the family to gather mushrooms. She noticed erythematous oval skin
lesion developed on the right thigh 8 days later. Physical examination: oval 8x6 erythematous skin lesion with
distinct limits and bright center is visible on the right thigh. The initial diagnosis is:
A. Erysipelas
B. Yersiniosis
C. Varicella zoster
D. Lyme borreliosis

18.87 A 25 - year - old man fell ill 2 days ago. The main symptoms were fever (39°C), headache, sore throat,
myalgia, cough. The initial diagnosis was influenza. The test must be done is:
A. Blood culture
B. Detection of influenza virus RNA in nasopharyngeal swab
C. X-ray examination of the chest
D. All above - mentioned tests

18.88 A 25 - year - old man fell ill 2 days ago. The main symptoms were fever (39°C), headache, sore throat,
myalgia, cough. The diagnosis of influenza was confirmed by tests. Patient’s condition became worse despite the
treatment: fever, cough, sputum, chest pain during breathing. Complication of disease is:
A. Sepsis
B. Pneumonia
C. Meningitis
D. Bronchitis

18.84 A 60 - year - old housewife suddenly became ill. The main complaints were fever and pain in the right side
of a chest. After 2 days her skin on the right side of the chest became red. Vesicular rash within a dermatome
associated with severe pain appeared. The medicine of a first choice is:
A. Penicillin
B. Doxycycline
C. Acyclovir
D. Ganciclovir

lsmusis.lsmuni.lt/Klausimai/Spausdinti?Length=0?Kalba=EN&KategorijaId=120&Kalbos_input=EN&Kalbos=EN&KategorijaEn_input=Infectious+dis… 10/12
3/27/2019 LSMUSIS
18.86 A 32 - year - old woman was gone with the family to mushroom. She noticed erythematous oval skin lesion
developed on the right thigh 8 days later. Physical examination: oval 8x6 erythematous skin lesion with distinct
limits and bright center is visible on the right thigh. The medicine of the first choice is:
A. Cyprofloxacin
B. Acyclovir
C. Cyprofloxacin and acyclovir
D. Doxycyclin

18.79 A 42 - year - old woman complains of fever with chills, redness with bright borders, and painful swelling of
the right leg appeared after injury of the right foot. The initial diagnosis is:
A. Herpes zoster
B. Lyme diseases
C. Allergic dermatitis
D. Erysipelas

18.80 A 42 - year - old woman complains of fever with chills, redness with bright borders, and painful swelling of
the right leg appeared after injury of the right foot. The medicine of first choice is:
A. Acyclovir
B. Penicillin
C. Norfloxacin
D. Metronidazol

18.74 A 35-year old hunter complains of fever, headache, nausea and vomiting at the end of July. Two weeks ago
he was ill with influenza-like illness. Tests for confirmation of the diagnosis are:
A. CSF examination and the detection of tick-borne encephalitis IgM antibodies in blood
B. Blood eosinophil count and trichinosis serology
C. CSF examination and the detection of Borrelia burgdorferii antibodies in CSF
D. The detection of rabies antigen in saliva by PCR method

18.75 A 40 - year - old farmer suddenly became ill in summer. The main symptoms were severe muscle aching in
the thighs and lumbar areas, chills, fever (39°C), cutaneous haemorrhages and skin rashes, dark urine, oliguria,
jaundice, splenomegaly, hepatomegaly. The initial diagnosis is:
A. Typhoid fever
B. Leptospirosis
C. Trichinosis
D. Viral hepatitis

18.1 - C 18.2 - E 18.3 - C 18.4 - B 18.5 - E 18.6 - B 18.7 - D 18.8 - D


18.9 - B 18.10 - B 18.11 - B 18.12 - A 18.13 - A 18.14 - C 18.15 - B 18.16 - C
18.17 - C 18.18 - B 18.19 - D 18.20 - A 18.21 - A 18.22 - A
18.23 18.24 18.25 18.26 18.27 18.28 18.29 18.30
1-B 1-B 1-A 1-B 1-B 1-C 1-B 1-D
2-C 2-C 2-D 2-A 2-C 2-A 2-C 2-C
3-A 3-D 3-B 3-C 3-D 3-B 3-A 3-B
4-D 4-A 4-C 4-A 4-A
18.31 18.32 18.33 18.34 18.35 18.36 18.37 18.38
1-C 1-D 1-B 1-B 1-C 1-B -B -C
2-B 2-A 2-A 2-A 2-B 2-C -C -A
3-A 3-B 3-C 3-C 3-A 3-D -A -B
4-D 4-C 4-A
18.39
-B
-A
-C
18.40 18.41 18.42 18.43 18.44 18.45 18.46 18.47
1 1 1 1 1 1 1 1
2 3 2 2 2 3 2 3
3 3 3 3 3
4
18.48 18.49 18.50 18.51 18.52 18.53 18.54 18.55
lsmusis.lsmuni.lt/Klausimai/Spausdinti?Length=0?Kalba=EN&KategorijaId=120&Kalbos_input=EN&Kalbos=EN&KategorijaEn_input=Infectious+dise… 11/12
3/27/2019 LSMUSIS
4 1 1 2 1 2 1 1
2 2 4 2 4 3 2
3 3 3 3
4
18.56 18.57 18.58 18.59 18.60
1 1 4 1 2
3 3 2 4
3
18.61 - B 18.62 - A 18.63 - C 18.64 - C 18.65 - C 18.66 - B 18.67 - A 18.68 - D
18.69 - D 18.70 - C 18.71 - A 18.72 - C 18.73 - C 18.76 - C 18.77 - B 18.78 - A
18.81 - B 18.82 - A 18.83 - D 18.85 - D 18.87 - B 18.88 - B 18.84 - C 18.86 - D
18.79 - D 18.80 - B 18.74 - A 18.75 - B

lsmusis.lsmuni.lt/Klausimai/Spausdinti?Length=0?Kalba=EN&KategorijaId=120&Kalbos_input=EN&Kalbos=EN&KategorijaEn_input=Infectious+dis… 12/12
3/27/2019 LSMUSIS

Nephrology
I type tasks. Choose only one best answer

40.1 Which statement describes tubular proteinuria:


A. Proteinuria is less than 2 g/day
B. Nephrotic syndrome usually develops in tubular proteinuria
C. The level of proteinuria is often higher than 3,5 g/day

40.2 Are diuretics allowed in treatment of nephrotic patients:


A. They are not allowed because low oncotic pressure causes edema and diuretics appear ineffective
B. They are allowed, however should be used carefully because nephrotic patients usually are hypovolemic
and can be worsened by diuretics
C. They are allowed; however small doses of diuretic should be used

40.3 Which one of the listed below is the most precise in diagnosing renal amyloidosis:
A. Increase of Alfa 2 globulin level in serum protein measured by electrophoresis
B. The test of methylen blue
C. Biopsy of the rectal mucosa
D. Congo red accumulation test
E. Renal biopsy

40.4 Which clinical picture is the most characteristic for rapidly progressive glomerulonephritis:
A. Severe arterial hypertension, failure of the left ventricle of the heart, hematuria and casts in urinalysis
B. Severe arterial hypertension and rapidly progressing renal insufficiency
C. Severe arterial hypertension, high fever, disuria and chronic renal insufficiency
D. Nephrotic syndrome, serosits, elevation of gamma globuline and myocarditis

40.5 Which type of chronic glomerulonephritis is the most rapidly progressing:


A. Membranous nephropathy
B. Minimal change glomerulopathy
C. Mesangioproliferative glomerulonephritis
D. Focal-segmental glomerulosclerosis

40.6 What is the kidney size for patients with analgetic nephropathy:
A. Enlarged kidney
B. Reduced kidney
C. No change in kidney size

40.7 What is characteritic for patients with diabetic nephropathy:


A. Reduction of renal size and decreased glomerular filtration rate is found from the start of the disease
B. Enlargement of renal size and increase of glomerular filtration rate is found from the beginning of the
disease
C. Glomerular basement membrane gets thinner and mesangial atrophy is observed from the beginning of
the disease
D. There are no morphological changes in diabetic nephropathy

40.8 What is the future prognosis for the patient with bacterical endocarditis, complicated by diffuse
glomerulonephritis with renal failure:
A. Prognosis is bad because renal insufficiency is continuously progressing
B. Successful treatment of the basic illness can lead to full restitution of renal function
C. Prognosis is bad because we must interrupt antibacterial treatment and treat rapidly progressive
glomerulonephritis with cytostatic drugs and prednisolone
D. Prognosis is not so bad. We have to stop using antibiotics which are responsible for the development of
acute interstitial nephritis

40.9 How to treat patient with severe nephrotic syndrome:


A. Start hemodialysis
B. Increase protein and salt intake, intravenous albumin infusion
C. Increase protein intake, administer intravenous albumin, diuretics
D. Decrease protein intake, administer intravenous albumin, diuretics.

40.10 What should you recommend for patients with chronic renal failure:
lsmusis.lsmuni.lt/Klausimai/Spausdinti?Length=0?Kalba=EN&KategorijaId=142&Kalbos_input=EN&Kalbos=EN&KategorijaEn_input=Nephrology&Ka… 1/10
3/27/2019 LSMUSIS
A. Food with high content of calcium and phosphate and calcitriol (vit. D₃)
B. Food with low content of phosphates, phosphate binders (calcium carbonate, calcium acetate) and
calcitriol (vit. D₃)
C. Continuously examine levels of calcium, phosphates and parathyroid hormone (PTH) in blood, use food
with low phosphates. Recommend phosphate binders and calcitriol (vit. D₃) (according to test's results)

40.11 What is the level of parathyroid hormone in the blood of patient with chronic renal insufficiency:
A. Elevated
B. Reduced
C. Unchanged
D. Possible all cases mentioned above

40.12 What kind of food will increase hyperkalemia in patients with chronic renal failure:
A. Green stuff (fruits and vegetables)
B. Milk products
C. Meat products
D. Fat food

40.13 In which disease of the listed bellow spontaneous remission is characteristic:


A. Membranous nephropathy
B. Minimal change glomerulopathy
C. Focal-segmental glomerulosclerosis
D. Rapidly progressive glomerulonephritis

40.14 What tactics should be used in treatment of acute glomerulonephritis:


A. Bed rest, low sodium diet, diuretics and antihypertensive drugs, antibacterial treatment (if infection still
active)
B. Treatment has to be intensive in order to avoid progression of disease (glucocorticoids,
immunosuppressants, antiagregants)
C. Treatment has to be directed against disorders of blood coagulation (anticoaguliants, antiagregants)
D. The aim is to eliminate locus of infection in advance (tonsillectomy, treatment of bad tooth)

II type tasks. For each numbered item,selct the one lettered heading that is most closely asssciated with it

40.15 What combination of symptoms is characteristic for diseases listed below:


1 - Acute pyelonephritis
2 - Hypertonic form of chronic glomerulonephritis
3 - Renovascular hypertension
4 - Cohn's syndrome (primary hyperaldosteronism)
5 - Essential hypertension
A. Disease lasts 10-15 years . At first elevations of blood pressure are short. Arterial hypertension
becomes stable. No changes in urinalysis. Radionuclide renograms are symmetrical.
B. Stable arterial hypertension develops during 1-2 years. No changes in urinalysis. Radionuclide
renograms are assymmetrical.
C. Stable and severe arterial hypertension develops during few years. Proteinuria 3 g/l, hematuria (RBC
10-12 µl), leukocyturia (WBC 10-40 µl). Radionuclide renograms are symmetrical.
D. Stable and severe arterial hypertension develops during 10-15 years. Proteinuria (0,3 g/l), hematuria
(RBC 0-1 µl), leukocyturia (WBC 10-14 µl). Radionuclide renograms are assymmetrical.
E. Stable and severe arterial hypertension develops during 1-2 years. No changes in urinalysis. Severe
hypokalemia. Radionuclide renograms are symmetrical.

40.16 What changes in urinalysis are characteristic for diseases listed below:
1 - Acute pyelonephritis
2 - Minimal change nephropathy
3 - Acute glomerulonephritis
4 - Primary arterial hypertension
A. Specific gravity of urine 1014, in urinalysis – no protein, RBC 1-2 µl , WBC 2-4 µl
B. Specific gravity of urine 1020, in urinalysis – traces of protein, no RBC, WBC 897 µl
C. Specific gravity of urine 1025, in urinalysis – protein 1g/l , RBC 20-30 µl, WBC 4-6 µl
D. Specific gravity of urine 1020, in urinanalysis protein 3g/l , RBC 1-2 µl, WBC 2-4 µl

40.17 What combination of symptoms is characteristic for diseases listed below:


lsmusis.lsmuni.lt/Klausimai/Spausdinti?Length=0?Kalba=EN&KategorijaId=142&Kalbos_input=EN&Kalbos=EN&KategorijaEn_input=Nephrology&Ka… 2/10
3/27/2019 LSMUSIS
1 - Acute pyelonephritis
2 - Chronic pyelonephritis
3 - Acute glomerulonephritis
4 - Minimal change nephropathy
A. Back pain , oliguria , normal or elevated specific gravity of urine , proteinuria , hematuria and elevated
blood pressure
B. Back pain, fever, dysuria, proteinuria, hematuria, pyuria and bacteriuria
C. Oedema, high proteinuria, hypoproteinemia, normal or elevated specific gravity of urine, oliguria
D. Back pain, elevated blood pressure, reduced specific gravity of urine, proteinuria, pyuria and
bacteriuria

40.18 What changes in urinalysis are characteristic for diseases listed below:
1 - Acute pyelonephritis
2 - Minimal change nephropathy
3 - Acute glomerulonephritis
4 - Hypertensive emergency in essential hypertension
A. Specific gravity of urine - 1,014, in urinalysis - no protein, RBC 1-2 µl, WBC 2-4 µl
B. Specific gravity of urine - 1,020, in urinalysis - traces of protein, no RBC, WBC – 897 µl
C. Specific gravity of urine - 1,025, in urinalysis - protein 1g/l, RBC 20-30 µl, WBC 4-6 µl
D. Specific gravity of urine - 1,020, in urinalysis - protein 3g/l, RBC 1-2 µl, WBC 2-4 µl

III type tasks. For each question there is one or more correct answers:
A – if correct answers are 1,2,3
B – if correct answers are 1 and 3
C – if correct answers are 2 and 4
D – if correct answer is 4
E – if correct are all answers above

40.18 (1) Which of analysis reflects function of kidneys:


1. Glomerular filtration rate (GFR)
2. Level of creatinine in blood
3. Radionuclide renogrames
4. Urinalysis (24h proteinuria)
5. Urinalysis (microscopy)

40.19 Which of the symptoms listed below are characteristic for nephrotic syndome:
1. Massive proteinuria
2. Hypoproteinemia
3. Oedema
4. Arterial hypertension
5. Bacteriuria
6. Hematuria

40.20 Which of diseases listed below are associated with secondary (AA type) amyloidosis:
1. Stomach ulcers
2. Rheumatoid arthritis
3. Myeloma
4. Bronchiectasis

40.21 Complications of acute glomerulonephritis:


1. Acute insufficiency of left ventricle of the heart with pulmonary oedema
2. Sudden death
3. Brain oedema (Eclampsia)
4. Poliuria and hypokalemia

40.22 Which kind of renal involvement is characteristic for gout:


1. Chronic interstitial nephritis
2. Renal stones
3. Acute renal damage (acute tubular necrosis)
4. Renal amyloidosis

40.23 What are probable mechanisms of development of analgetic nephropathy:


lsmusis.lsmuni.lt/Klausimai/Spausdinti?Length=0?Kalba=EN&KategorijaId=142&Kalbos_input=EN&Kalbos=EN&KategorijaEn_input=Nephrology&Ka… 3/10
3/27/2019 LSMUSIS
1. Direct toxic action of analgetics on medullary layer of kidneys
2. Analgetics can diminish medullary blood flow by inhibiting the production of vasodilator prostaglandins
3. Impairement of oxygenation process in renal cell's membranes
4. Chronic interstitial nephritis due to immune complexes deposition

40.24 What recommendations for patient with diabetic nephropathy you have to give:
1. Good contol of glycemia
2. Medications for reduction of proteinuria
3. Good control of blood pressure
4. To change insulin into peroral hypoglycemic agents

40.25 What is the treatment of myeloma kidney:


1. Treatment of main disease with cytostatic agents
2. Alcalinisation of urine
3. High fluid intake
4. Antibiotics

40.26 Which symptoms are characteristic for myeloma kidney:


1. Normal size kidneys despite chronic renal failure
2. Presence of Bence-Jones protein in urine
3. Signs of tubular dysfunction
4. Gross hematuria
5. Pain in the projection of kidneys

40.27 Which radiologic signs of intravenous urograms are specific for chronic pylonephritis wihout obstruction:
1. Enlargement of kidneys
2. Diminishing of kidneys
3. Advanced dilatation of calyces
4. Deformation of calyces

40.28 Which of radioisotope renography are characteristic for renovascular hypertension:


1. Asymetric renograms
2. Worse function of damaged kidney
3. Prolonged isotope excretion in damaged kidney
4. Shortened isotope excretion in damaged kidney

40.29 Which of diseases listed below is associated with normal size or enlargement of kidneys despite chronic
renal failure:
1. Myeloma kidney
2. Diabetic nephropathy
3. Amyloidosis
4. Chronic pyelonephritis
5. Chronic glomerulonephritis

40.30 Which symptoms are common in renovascular hypertension:


1. Arterial hypertension diagnosed in young (<30 years) and older (>50 years) age
2. Elevated level of renin in venous blood flow from affected kidney
3. Narrowing of renal artery on angiographic investigation
4. Radioizotopinės renogramos asimetrinės. Šlapimo tyrimas normalus

40.31 For which diseases listed below is characteristic decrease of renal size:
1. Analgetic nephropathy
2. Diabetic nephropathy
3. Chronic pyelonephritis
4. Renal amyloidosis
5. Myeloma kidney

IV type tasks. Choose only one best answer

40.32 Man (40 years old) was hospitalized in district hospital due to abdominal pain and vomminting. Surgical
pathology was denied. On examination: arterial blood pressure 160/100 mmHg, diuresis 800ml. Blood analysis: Hb

lsmusis.lsmuni.lt/Klausimai/Spausdinti?Length=0?Kalba=EN&KategorijaId=142&Kalbos_input=EN&Kalbos=EN&KategorijaEn_input=Nephrology&Ka… 4/10
3/27/2019 LSMUSIS
70 g/l, ESR 60 mm/h, urea 50 mmol/l, 800 µmol/l, potassium 4,5 mmol/l. Urinalysis: specific gravity 1,003,
proteinuria 800 mg/l, WBC 8-10/µl, RBC 4-6/µl. What is the diagnosis and what treatment do you prefere:
A. In this case we have patient with latent chronic pyelonephritis and chronic renal failure. We must treat
pyelonephritis wtih the hope of improvement of renal funtion. Reduced amount of protein in food nad
parenteral infusion of 2-3 liters of fluid per day is necessary.
B. In this case we have patient with latent chronic glomerulonephritis and chronic renal failure.There is
urgent need to begin hemodialysis treatment.
C. In this case we have patient with latent chronic glomerulonephritis and chronic renal failure. He needs
such treatment: bed rest, low protein diet, parenteral infusion of 5% solution of glucose (1000 ml), Ringer's
solurion (1000 ml), captopril 0,025 g 3 times daily.

40.33 Our patient man (42 years old) 20 years ago had an episode of acute glomerulonephritis. After that he was
in a good condition, only during visit to doctor arterial hypertension and proteinuria were found. Today at work he
became unconscious and was brought to hospital by ambulance. On admission: patient is pale, periorbital
oedema, nausea, vomiting, blood pressure 200/100 mmHg. Urinalysis: specific gravity 1.010, proteinuria 3200
mg/l, WBC 4-6 /µl, RBC 10-20 /µl. What diagnosis do you suspect and what treatment do you prefere:
A. There is essential arterial hypertension, hypertensive nephropathy, hypertensive urgency. Patient needs
hospitalisation and antihypertensive treatment.
B. There is chronic glomerulonephritis with secondary hypertension. Patient needs hospitalisation and
antihypertensive treatment. There is a need to investigate renal size and function, after these investigations
to decide whether to perform renal biopsy, to specify form of glomerunephritis and possibility of pathogenetic
treatment.
C. There is chronic glomerulonephritis with secondary hypertension. If our antihypertensive treatment will be
successfull (BP ≤ 140/90 mmHg), patient needs ambulatory antihypertensive treatment. No need for
hospitalisation.
D. There is renovascular hypertension. Patient needs hospitalisation, antihypertensive treatment and
angiography of renal arteries with PTA and stenting.

40.34 16 year old schoolboy has complaints: oedema in face and legs during last 3 days. 2 weeks ago he had
fever till 37,5ºC and sore throat. On examination: body temperature 36,6ºC, oedema in face and legs, blood
pressure 160/100 mmHg. Blood analysis: Hb 130 g/l, WBC 5,2×10⁹/l, ESR 30 mm/h. Urinalysis: specific gravity
1,020, proteinuria 2400 mg/l, WBC 8-12/µl, RBC 6-8-10/µl. What diagnosis do you suspect:
A. Acute glomerulonephritis
B. Exacerbation of chronic glomerulonephritis
C. Acute pyelonephritis
D. Exacerbation of chronic pyelonephritis
E. Renal amyloidosis
F. Lupus nephritis
G. Myeloma kidneys
H. Analgetic nephropathy

40.35 48 years old woman was admitted to hospital after findings suggestive for renal failure (in blood analysis:
urea 18 mmol/l, creatinine 250 µmol/l). Four years ago she had low back pain (right side) with macrohematuria.
Nephrolithiasis was suspected, however no stones were found. After this episode no pathological changes in
urinalysis were found, blood pressure was normal. Frequent migraine attacks during more than 20 years. On
examination: diurnal diuresis 1200 ml, nocturnal diuresis 900 ml, specific gravity of urine 1,005-1,009.
Ultrasonography - no renal stones. Blood analysis: Hb 100 g/l, ESR 26 mm/h. Urinalysis: specific gravity 1,008,
traces of protein, WBC 2-3/µl, no RBC. Test for bacteriuria - negative. What diagnosis do you suspect:
A. Acute glomerulonephritis
B. Chronic glomerulonephritis
C. Chronic pyelonephritis
D. Renal amyloidosis
E. Lupus nephritis
F. Myeloma kidney
G. Polyarteritis nodosa
H. Analgetic nephropathy

40.36 48 years old woman was admitted to hospital after findings suggestive for renal failure (in blood analysis:
urea 18 mmol/l, creatinine 250 µmol/l). Four years ago she had low back pain (right side) with macrohematuria.
Nephrolithiasis was suspected, however no stones were found. After this episode no pathological changes in
urinalysis were found, blood pressure was normal. Frequent migraine attacks during more than 20 years. On
examination: diurnal diuresis 1200 ml, nocturnal diuresis 900 ml, specific gravity of urine 1,005-1,009.
lsmusis.lsmuni.lt/Klausimai/Spausdinti?Length=0?Kalba=EN&KategorijaId=142&Kalbos_input=EN&Kalbos=EN&KategorijaEn_input=Nephrology&Ka… 5/10
3/27/2019 LSMUSIS
Ultrasonography - no renal stones. Blood analysis: Hb 100 g/l, ESR 26 mm/h. Urinalysis: specific gravity 1,008,
traces of protein, WBC 2-3/µl, no RBC. Test for bacteriuria - negative. What examination would be helpful in
establishing right diagnosis:
A. ASO titer
B. Glomerular filtration rate (GFR)
C. Repetative urinalysis for bacteriuria
D. Biopsy of rectal mucosa
E. ant-DNA test
F. Blood marrow punction
G. Skin biopsy
H. Anamnesis about analgetic's use

40.37 45 years old woman has 25-years history of chronic pyelonephritis. Now she has polidypsia and poliuria
(diuresis about 2 l), weakness, dyspnea during physical activity. On examination: pale, no oedema. Blood analysis:
Hb 80 g/l, RBC 2,4x1012/l, urea 25 mmol/l, creatinine 350 µmol/l. Urinalysis: specific gravity 1,008, proteinuria 200
mg/l, WBC 10-14 /µl. Urinalysis for bacteriuria - negative. What is your proposal concerning further treatment of
this woman:
A. Conservative treatment will be ineffective, it is better to begin hemodialysis
B. It is better to start treatment with amikacin or gentamycin. Antibacterial treatment can leed to
improvement of renal function
C. It is better to start treatment with cefuroxime
D. Reikia taikyti konservatyvų inkstų nepakankamumo gydymą: riboti suvartojamo baltymo kiekę, vartoti
pakankamai skysčių, patikslinti anemijos priežastį ir ją koreguoti. Antibakterinis gydymas netikslingas.

40.38 18-years old student had a cold and sore throat with fever till 380C two weeks ago. Now he has low back
pain, oliguria, oedema in face and legs. Blood pressure 160/100 mmHg. Blood analysis: urea 12 mmol/l, creatinine
140 µmol/l. Urinalysis: specific gravity 1,026, proteinuria 1g/l , WBC 6/µl , RBC 450/µl. What diagnosis do you
suspect:
A. Acute glomerulonephritis
B. Acute pyelonephritis
C. Exacerbation of chronic glomerulonephritis
D. Exacerbation of chronic pyelonephritis

40.39 18-years old student had a cold and sore throat with fever till 380C two weeks ago. Now he has low back
pain, oliguria, oedema in face and legs. Blood pressure 160/100 mmHg. Blood analysis: urea 12 mmol/l creatinine
140 µmol/l.. Urinalysis: specific gravity 1,026, proteinuria 1g/l, WBC 6/µl, RBC 450/µl. What treatment do you
prefere:
A. Ampicillin 0,5g 4 times per day, usage of a lot of fluids, bed rest. There is no need for hospitalisation
B. Hospitalisation, penicillin 2 mln VV every 8 hours iv (10 days), bed rest, blood pressure control, diet with
reduced NaCl
C. Hospitalisation, hemodialysis
D. Hospitalisation with suspition on renal stones and pyelonephritis

40.40 40 18-year old patient with renal pathology diagnosed 14 years ago, now has such complaints: headache,
oedema and nausea. Arterial blood pressure 170/100 mmHg, diuresis 700 ml. Blood analysis: Hb 68 g/l, WBC
9x109l; urea 50 mmol/l, creatinine 1000 µmol/l, potassium 7,0 mmol/l, ESR 50 mm/h. Urinalysis: specific gravity
1,004; proteinuria 300 mg/l; WBC 3-6/µl; RBC 8-10/µl, hyaline casts. On ultrasound: kidneys small size, no
hydronephrosis. What the treatment should be:
A. There is urgent need to begin hemodialysis, treatment of complications of chronic renal disease, in future
to plan renal transplantation
B. Hemodialysis is contraindicated because of very severe anaemia
C. There is no need for hemodialysis because patient is young and duration of illness is very long

40.41 18-year old patient with renal pathology diagnosed 14 years ago, now has such complaints: headache,
oedema and nausea. Arterial blood pressure 170/100 mmHg, diuresis 700 ml. Blood analysis: Hb 68 g/l, WBC
9x109l; urea 50 mmol/l, creatinine 1000 µmol/l, potassium 7,0 mmol/l, ESR 50 mm/h. Urinalysis: specific gravity
1,004; proteinuria 300 mg/l; WBC 3-6 /µl; RBC 8-10 /µl, hyaline casts. On ultrasound: kidneys small size, no
hydronephrosis. Which analysis shows urgent need to begin hemodialysis:
A. Level of urea in blood
B. Level of potassium in blood
C. Level of creatinine in blood
D. Level of hemoglobin in blood
lsmusis.lsmuni.lt/Klausimai/Spausdinti?Length=0?Kalba=EN&KategorijaId=142&Kalbos_input=EN&Kalbos=EN&KategorijaEn_input=Nephrology&Ka… 6/10
3/27/2019 LSMUSIS
40.42 26-yers old woman has complaints: arthralgias and fever in last four weeks. Treatment with ibuprofen and
voltaren was ineffective. In urinalysis: specific gravity 1,015, proteinuria 2 g/l, WBC 4/µl, RBC 120 /µl. On
examination: butterfly-shaped rash over cheeks and nose, blood pressure 160/100 mmHg, the articulations of
wrists and foot are swollen, systolic II grade murmur on the heart. ESR 60 mm/h. Which diagnosis should be:
A. Renal amyloidosis
B. Acute pyelonephritis
C. Lupus nephritis
D. Exacerbation of chronic glomerulonephritis
E. Myeloma kidney
F. Acute glomerulonephritis
G. Exacerbation of chronic pyelonephritis

40.43 18-years old student was admitted to intensive care unit through two episodes of epileptiformic seizures. He
was healthy in the past. Three weeks ago he had fever, sore throat and was treated with ampicillin for 3 days. Two
days ago headache and nausea appeared. On examination: oedema in face, there are no meningeal symptoms,
arterial blood pressure 200/120 mmHg. Blood analysis: Hb 136 g/l, urea 15 mmol/l, creatinine 300 µmol/l.
Urinalysis: specific gravity 1,030, proteinuria 3 g/l, RBC 1200/ µl. Normal kidney size on ultrasound. Which
diagnosis do you suspect and what treatment should be:
A. Acute glomerulonephritis with eclampsia. Treatment – antihypertensives, diuretics, penicillin 2mln every 8
hours iv.
B. Acute meningitis. For prevention of seizures it is necessary to treat with diazepam, after that patient must
be transfered to the hospital of infectious diseases.
C. Chronic pyelonephritis (very likely with nephrolithiasis), secondary hypertension and hypertensive
emergency. Suggestions for treatment: captopril, nifedipin and lespenefril.
D. Chronic glomerulonephritis, hypertensive emergency and chronic renal insufficiency. Suggestions for
treatment: captopril, nifedipin and lespenefril.

40.44 58-years old woman has disuria. In past – many episodes of cystitis. Blood analysis: Hb 80 g/l, urea 20
mmol/l. Urinalysis: specific gravity 1,010, proteinuria 200 mg/l, WBC full visual field, RBC 1-2/µl, urinary culture is
taken. What diagnosis do you suspect and what treatment do you prefere:
A. Exacerbation of chronic pyelonephritis. Ambulatory treatment (1 week – antibacterial drug, 3 weeks –
herbal tea)
B. Exacerbation of chronic pyelonephritis. Hospitalisation and treatment with gentamycin (0,08g 3 times
intramusculary), after that antibacterial treatment according to susceptibility of urinary flora
C. Exacerbation of chronic pyelonephritis. Hospitalisation and empirical treatment with Cefuroxime 1,5 g
every 8 hours iv, later treatment is corrected according to urinary culture results.
D. Acute pyelonephritis. Hospitalisation and treatment with oxacillin 1g 4 times per day

40.45 37-years old man with a 14 years long history of chronic glomerulonephritis has such complaints: dry mouth,
dizziness, nausea and weakness. Blood pressure 200/110 mmHg, diuresis 500ml. Blood analysis: Hb 60 g/l, urea
50 mmol/l, potassium 4,5 mmol/l, creatinine 900 µmol/l. Urinalysis: specific gravity 1,006, proteinuria 300 mg/l,
WRB 4 /µl , RBC 8/µl. What treatment do you prefere:
A. There is urgent need to initiate hemodialysis
B. There is contraindication for hemodialysis (severe anaemia)
C. Conservative treatment: reduction of diet protein, diuretics, iron, erythropoethin
D. Creation of arteriovenous fistula and initiation of hemodialysis after 4-6 weeks

40.46 16-years old patient has headache, weakness and dyspnea (during last 2 days). He had often episodes of
pharyngitis, last of them – two weeks ago. On examination: oedema in face and legs, arterial blood pressure
160/100 mmHg, diuresis 600 ml. Blood analysis: urea 20 mmol/l, creatinine 150 µmol/l. Urinalysis: specific gravity
1,028, proteinuria 2g/l, WBC 10/µl, RBC 130/µl. On ultrasound kidneys 11x6 cm size, parenchyma 1,9cm. What
diagnosis do you suspect and what treatment do you prefere:
A. Acute pyelonephritis. Hospitalisation, treatment with ampicillin 0,5g four times per day, later – according
to susceptibility of microflora.
B. Chronic glomerulonephritis and exacerbation of chronic renal failure. Hospitalization and treatment with
prednisolon 40 mg per day, furosemide 40 mg per day, lercanidipine 10 mg per day.
C. Chronic glomerulonephritis and exacerbation of chronic renal failure. Hospitalization and treatment with
ciprofloxacin 0,25g two times per day, infusions of isotonic solution.
D. Acute glomerulonephritis. Hospitalization, bed rest, diet with low content of sodium, penicillin 2mln x3
/day, furosemid 80 mg every days, lercanidipine 10 mg per day

lsmusis.lsmuni.lt/Klausimai/Spausdinti?Length=0?Kalba=EN&KategorijaId=142&Kalbos_input=EN&Kalbos=EN&KategorijaEn_input=Nephrology&Ka… 7/10
3/27/2019 LSMUSIS
40.47 35-years old man two months ago had episode of macrohematuria without other complaints. Then he was
hospitalised: on examination arterial blood pressure was 160/100 mmHg, renal ultrasound, cystoscopy of urinary
bladder, intravenous urography, computer tomography - no pathological changes were found. In urinalysis: specific
gravity 1,020, proteinuria 1g/l, WBC 4/µl, RBC 5000/µl. After normalisation of urinalysis (only RBC 2-3/µl) was
discharged from hospital. Now he was hospitalised again with macrohematuria. What diagnosis to you suspect
and what treatment do you prefere:
A. New episode of renal stone disease. Hospitalisation, analgetics, spasmolitics.
B. Exacerbation of chronic pyelonephritis. Hospitalization, treatment with ciprofloxacin 0,25 g two times per
day.
C. Acute glomerulonephritis, in the way to chronic glomerulonephritis. Hospitalization and treatment with
cyclophosphamide 150 mg per day.
D. Chronic glomerulonephritis (possibly IgA nephropathy). Hospitalization, renal biopsy (with
imunofluorescent microscopy). ACEi for treatment of hypertension.

40.48 40-years old women has arterial hypertension (200/100 mmHg), headache, dissiness, temperature 37.30C.
She has history of often tonsilitis and cystitis, episode of acute pyelonephritis during pregnancy, duration of arterial
hypertension 10 years. Urinalysis: specific gravity 1,010; proteinuria-300 mg/l. WBC 400/µl, no RBC, nitrites ++.
What diagnosis do you suspect and what treatment do you prefere:
A. Acute pyelonephritis. Hospitalisation and treatment with ampicillin, later-according to susceptibility of
microflora.
B. Exacerbation of chronic pyelonephritis and secondary arterial hypertension. Ambulatory treatment:
antihypertensives, after taking urinary culture empirical administration of cefuroxime, later treatment
correction according to culture results, then – antirecidival treatment.
C. Chronic latent glomerulonephritis. Hospitalisation and renal biopsy.
D. Hypertensive form of chronic glomerulonephritis. Hospitalisation and treatment with captopril and
azathioprine.
E. Vasorenal hypertension. Hospitalisation and angiography of renal arteries.

40.49 35 -years old man was hospitalised because of severe headache, dissiness and severe hypertension
(240/140 mmHg). In patient's ambulatory card: 3 years ago arterial blood pressure was normal (130/80 mmHg).
Blood and urinalysis - without pathological changes. No family anamnesis of hypertension. What diagnosis do you
suspect and which treatment do you prefere:
A. Vasorenal hypertension. Hospitalisation, radionuclide renograms, renal angiography, PTA and
antihypertensive treatment.
B. Essential hypertension with hypertensive urgency. Hospitalisation, antihypertensive treatment.
C. Essential hypertension with hypertensive urgency. After administration of antihypertensive treatment –
ambulatory treatment.
D. Chronic pyelonephritis with secondary hypertension. Ambulatory treatment of pyelonephritis and
antihypertensive treatment.

40.50 22 years old woman has fever (till 39 0 C), disuria and polyakiuria, low back pain from right side. Arterial
blood pressure 120/80mmHg. Blood analysis: urea 7 mmol/l. Urinalysis: specific gravity 1,025, proteinuria 400
mg/l, WBC 500/µl, RBC 2-3/µl, bacteriuria >10⁵CFU. What diagnosis do you suspect and what treatment do you
prefere:
A. Exacerbation of chronic pyelonephritis. Ambulatory treatment with antibacterial drugs.
B. Acute pyelonephritis. Hospitalisation, treatment with cefuroxime, later – according to susceptibility of
microflora.
C. Latent form of chronic glomerulonephritis. Ambulatory treatment with prednisolone 30 mg per day.
D. Chronic glomerulonephritis. Hospitalization, renal biopsy and treatment according to results of biopsy.

40.51 42-years old worker man has complaints: weakness, nausea, headache. 12 years long history of chronic
glomerulonephritis. Dry mouth and nausea appeared last year. Arterial blood pressure 200/110 mmHg, diuresis 1,5
1. Blood analysis: Hb 98 g/l, potassium 4.2 mmol/l, urea 20 mmol/l, creatinine 210 µmol/l, eGFR 30
ml/min/1.73m2. Urinalysis: specific gravity 1,004, proteinuria 800 mg/l, WBC 2-4/µl, RBC 2-4/µl. What diagnosis do
you suspect and what treatment do you prefere:
A. Chronic glomerulonephritis and chronic renal failure. Hospitalisation, initiatiation of hemodialysis.
B. Chronic glomerulonephritis and end stage renal failure. Hospitalisation and treatment with peritoneal
dialysis.
C. Exacerbation of chronic glomerulonephritis. Hospitalisation and pathogenetic treatment with citostatics
and corticosteroides. Expectation of improvement of renal function.
D. Chronic glomerulonephritis and chronic renal failure. Hospitalisation and conservative treatment:
reduction of protein intake, hyperhydratation (till diuresis 2-2,5l), antianemic, antihypertensive treatment and
lsmusis.lsmuni.lt/Klausimai/Spausdinti?Length=0?Kalba=EN&KategorijaId=142&Kalbos_input=EN&Kalbos=EN&KategorijaEn_input=Nephrology&Ka… 8/10
3/27/2019 LSMUSIS
phospate binders.

40.52 38-year disabled women has complaints: general oedema, weakness. During last 10 years she had
disability because of rheumatoid arthritis. Arterial blood pressure 120/80 mmHg. Blood analysis: Hb 100 g/l, WBC
6x109/l, urea 8 mmol/l, protein 40 g/l, ESR 60 mm/h. Urinalysis: specific gravity 1,018, proteinuria 5000 mg/l, WBC
3-4/µl. What diagnosis do you suspect and what treatment do you prefere:
A. Acute glomerulonephritis with nephrotic syndrome. Hospitalisation, bed rest, diet with reduced amount of
sodium, penicillin 500.000 UE 6 times per day.
B. Reumatoid polyarthritis with secondary renal amyloidosis (AA). Hospitalisation, biopsy of rectal mucosa
or renal biopsy. Treatment with diuretics and protein infusions for nephrotic syndrome.
C. Membranous nephropathy. Hospitalisation, treatment with cyclophosphamide 100 mg per day and
prednisolone 60 mg per day.
D. Chronic insterstitial nephritis due to long-lasting treatment with NSAID’s. Ambulatory treatment: stop
NSAID’s, reduce salt, increase fluids.

40.53 46-years old woman nurse had complaints for long lasting rhinitis and sinusitis in June. In July - cough and
fever (till 380 C). X-ray examination showed infiltration in the lower lobe of right lung – pneumonia was diagnosed
and treatment with various antibiotics in the department of internal diseases in local hospital was started.
Antibacterial treatment had no effect and patient‘s status was only worsening - dyspnoe appeared and infiltration in
both lungs increased. In September patient was transfered to pulmonological department of university hospital,
treatment with reserve antibiotics (meronem) was started, but dyspnoe progressed till the need of oxigenotherapy,
hemoptoe appeared. Blood analysis: Hb 130g/l, RBC 4x1012l, creatinine 400 µmol/l. Urinanalysis: proteinuria
1500 mg/l, WBC 2-3/µl, RBC 15-25/µl diuresis 1600 ml/24h. What diagnosis do you suspect?
A. Acute pneumonia complicated with acute kidney injury.
B. Anti GBM antibodies mediated Goodpasture's syndrome or systemic vasculitis with pulmo-renal
syndrome, rapidly progressive glomerulonephritis syndrome.
C. Acute pneumonia and nephrotoxic injury (of meronem).
D. Atypical acute pneumonia with secondary glomerulonephritis.

40.54 46-years old woman nurse had complaints for long lasting rhinitis and sinusitis in June. In July - cough and
fever (till 380 C). X-ray examination showed infiltration in the lower lobe of right lung – pneumonia was diagnosed
and treatment with various antibiotics in the department of internal diseases in local hospital was started.
Antibacterial treatment had no effect and patient‘s status was only worsening - dyspnoe appeared and infiltration in
both lungs increased. In September patient was transfered to pulmonological department of university hospital,
treatment with reserve antibiotics (meronem) was started, but dyspnoe progressed till the need of oxigenotherapy,
hemoptoe appeared. Blood analysis: Hb 130g/l, RBC 4x1012l, creatinine 400 µmol/l. Urinanalysis: proteinuria
1500 mg/l, WBC 2-3/µl, RBC 15-25/µl, diuresis 1600 ml/24h. Which examination would be helpfull for establishing
the right diagnosis?
A. Cultures of blood, sputum and urine
B. Blood gas analysis
C. Investigation of anti GBM and ANCA antibodies and renal biopsy
D. CT of thorax

40.55 80 years old pensioner man. During visit to family doctor severe anemia (Hb 80 g/l) was found with severe
renal failure (creatinine 370 mol/l). Urinare analysis: specific gravity 1015, no proteinuria, WBC -3-4/ l, RBC – 1-
2/ l. Ultrasonography: kidney size – 8 cm, residual urine in urinary bladder – 30 ml. In past – pneumonia (2 times),
cholecystectomia, myocardial infartion (2 times), stroke (3 years ago), prothesis of left femoral artery (6 years
ago). Arterial hypertension during last 20 years, good control with fosinopril. What diagnosis do you suspect?
A. Hypertensive nephropathy with chronic renal failure
B. Chronic pyelonephritis with chronic renal failure
C. Chronic glomerulonephritis with chronic renal failure
D. Ischemic nephropathy with chronic renal failure

40.56 23-years old man has complaint that during several days severe oedema developed. 3 days before he had
fever till 380C, sore throat. Family doctor found proteinuria 8 g/l in urinalysis and sent patient for consultation to
nephrologist. On examination: severe oedema in the face and legs, BP 120/80 mmHg, pulse 80x/min. Proteinuria
12g/24h, GFR 120 ml/min. Urinanalysis: specific gravity 1,025, proteinuria 8,4 g/l, WBC 4-6/ l, no RBC. Blood
analysis: creatinine 80 mol/l 0,08 mmol/l, albumin 24 g/l, cholesterol 14 mmol/l. What examination will be the
most usefull for diagnosis?
A. ASO titer
B. Proteinogramm of blood
C. Renal biopsy
lsmusis.lsmuni.lt/Klausimai/Spausdinti?Length=0?Kalba=EN&KategorijaId=142&Kalbos_input=EN&Kalbos=EN&KategorijaEn_input=Nephrology&Ka… 9/10
3/27/2019 LSMUSIS
D. Consultation of otorhinolaryngologist
E. Investigation of anti GBM and ANCA antibodies

40.57 23-years old man has complaint that during several days severe oedema developed. 3 days before he had
fever till 380C, sore throat, symptoms lasted for 3 days. Family doctor found proteinuria 8 g/l in urinalysis and sent
patient for consultation to nephrologist. On examination: severe oedema in the face and legs, BP 120/80 mmHg,
pulse 80x/min. Proteinuria 12g/24h, GFR 120 ml/min. Urinanalysis: specific gravity 1,025, proteinuria 8,4 g/l, WBC
4-6 in visual field, no RBC. Blood analysis: creatinine 0,08 mmol/l, albumin 24 g/l, cholesterol 14 mmol/l. What
diagnosis is the most possible?
A. Acute glomerulonephritis
B. Rapidly progressive glomerulonephritis
C. Membranous nephropathy
D. Minimal change nephropathy
E. Renal amyloidosis

40.1 - A 40.2 - B 40.3 - E 40.4 - B 40.5 - D 40.6 - B 40.7 - B 40.8 - B


40.9 - D 40.10 - C 40.11 - D 40.12 - A 40.13 - A 40.14 - A
40.15 40.16 40.17 40.18
1-D 1-B 1-B 1-B
2-C 2-D 2-D 2-D
3-B 3-C 3-A 3-C
4-E 4-A 4-C 4-A
5-A
40.18 (1) 40.19 40.20 40.21 40.22 40.23 40.24 40.25
1 1 2 1 1 1 1 1
2 2 4 3 2 2 2 2
3 3 3 3 3 3
40.26 40.27 40.28 40.29 40.30 40.31
1 2 1 1 1 1
2 4 2 2 2 3
3 3 3 3
4
40.32 - B 40.33 - B 40.34 - A 40.35 - H 40.36 - H 40.37 - D 40.38 - A 40.39 - B
40.40 - A 40.41 - B 40.42 - C 40.43 - A 40.44 - C 40.45 - A 40.46 - D 40.47 - D
40.48 - B 40.49 - A 40.50 - B 40.51 - D 40.52 - B 40.53 - B 40.54 - C 40.55 - D
40.56 - C 40.57 - D

lsmusis.lsmuni.lt/Klausimai/Spausdinti?Length=0?Kalba=EN&KategorijaId=142&Kalbos_input=EN&Kalbos=EN&KategorijaEn_input=Nephrology&K… 10/10
3/27/2019 LSMUSIS

Urology
I type tasks. Choose only one best answer

19.1 First aid in the out-patients be in case of ruptured urethra:


A. Catheterisation
B. Cystostomy
C. Puncture of the urinary bladder
D. Analgesics

19.2 What is the treatment of extraperitoneously ruptured urinary bladder:


A. Cystostomy
B. Suture of the bladder (cystorrhaphia)
C. Cystostomy, cystorrhaphia, paravesical drainage through the obturator foramen
D. Cystostomy, cystorrhaphia, paravesical drainage through the abdominal wall
E. Cystostomy and cystorrhaphia

19.3 What is the treatment of intraperitoneously ruptured urinary bladder:


A. A. Continual catheterisation
B. B. Cystostomy
C. C. Laparotomy and suture of the bladder (cystorrhaphia)
D. D. Laparotomy, cystorrhaphia and cystostomy

19.4 What should the management of acute traumatic scrotal haematoma be:
A. Squeezing bandage
B. Haemostatic drugs
C. Antibacterial drugs
D. Incision and revision of the scrotum

19.5 Renal tumours can manifest with all following signs, except:
A. Enlarged kidney
B. Dysuria
C. Varicocoele
D. Renal colic

19.6 One can use all the following methods for the management of renal colic, caused by stones, except:
A. Analgesics, spasmolytics
B. Blockades with novocaine
C. Thermal procedures
D. Catheterisation of the ureters
E. Extraction of the stone

II type tasks. For each numbered item,selct the one lettered heading that is most closely asssciated with it

19.7 What kind of haematuria is most characteristic to every following disease:


1 - Renal tumour
2 - Tumour of urinary bladder
3 - Stone in the urinary bladder
4 - Acute cystitis
5 - Urethritis
A. Total haematuria
B. Initial haematuria
C. Terminal haematuria

19.8 What are the characteristic kinds of haematuria according to the site of the disease:
1 - Kidney
2 - Ureter
3 - Bottom and cervix of the urinary bladder
4 - Urethra
A. Total haematuria
B. Initial haematuria
C. Terminal haematuria

lsmusis.lsmuni.lt/Klausimai/Spausdinti?Length=0?Kalba=EN&KategorijaId=121&Kalbos_input=EN&Kalbos=EN&KategorijaEn_input=Urology&Kategor… 1/7
3/27/2019 LSMUSIS
19.9 What kind of urinary retention is most characteristic to every following disease:
1 - Renal tumour
2 - Tumour of the urinary bladder
3 - BPH (benign prostatic hyperplasy)
4 - Acute prostatitis
5 - Complete rupture of urethra
A. Partial urinary retention
B. Complete urinary retention

III type tasks. For each question there is one or more correct answers:
A – if correct answers are 1,2,3
B – if correct answers are 1 and 3
C – if correct answers are 2 and 4
D – if correct answer is 4
E – if correct are all answers above

19.10 What examinations give the diagnosis of the urinary retention:


1. Palpation and percussion of urinary bladder
2. Digital rectal examination
3. Catheterisation of urinary bladder
4. Plain urography

19.11 What is the treatment of the urinary retention due to BPH (benign prostatic hyperplasy):
1. Catheterisation with elastic catheter
2. Catheterisation with metal catheter
3. Puncture of urinary bladder
4. Diuretics

19.12 What are the best means in treating secondary obstructive pyelonephritis:
1. Spasmolytics
2. Enforced diuresis
3. Catheterisation of the kidney
4. Aminoglycosides

19.13 What anamnestic data are characteristic to postrenal anuria:


1. Renal colic
2. Blood transfusion
3. Radiotherapy after gynaecological surgery
4. Heavy vomiting

19.14 What anamnestic data are characteristic to prerenal anuria:


1. Heavy vomiting
2. Massive blood loss
3. Long lasting surgery
4. Radiotherapy after gynaecological surgery
5. Crush syndrome

19.15 What examinations are necessary to diagnose the injury of the ureter:
1. Dynamic scintigraphy of kidneys
2. Ultrasonic investigation of kidneys
3. Intravenous urography
4. Renal catheterisation

19.16 What signs permit to suspect intraperitoneal rupture of the urinary bladder:
1. Inability to urinate
2. Abdominal pain
3. Compulsory posture of the patient
4. Patients tossing
5. Urethrorrhagia

19.17 What examinations and signs are valuable in diagnosing intraperitoneal rupture of the urinary bladder:
1. Palpation of rectum
lsmusis.lsmuni.lt/Klausimai/Spausdinti?Length=0?Kalba=EN&KategorijaId=121&Kalbos_input=EN&Kalbos=EN&KategorijaEn_input=Urology&Kategor… 2/7
3/27/2019 LSMUSIS
2. Abdominal percussion
3. Signs of irritated peritoneum
4. Catheterisation of urinary bladder
5. Cystography

IV type tasks. Choose only one best answer

19.18 40 year old male fell ill suddenly: strong pain under the right arch of the ribs and in the right iliac fossa,
nausea, vomiting. The abdomen is “blowed out” and tender under the palpation. Peristalsis is paretic, irritation of
peritoneums absent. The right loin is painful if knocked. Body temperature is normal. Blood count: WBC- 6,9 10* /I,
microhaematuria. What is the preliminar diagnosis:
A. Bowel obstruction
B. Acute appendicitis
C. Gallstones
D. Renal stones

19.19 40 year old male fell ill suddenly: strong pain under the right arch of the ribs and in the right iliac fossa,
nausea, vomiting. The abdomen is “blowed out” and tender under the palpation. Peristalsis is paretic, irritation of
peritoneums absent. The right loin is painful if knocked. Body temperature is normal. Blood count: WBC- 6,9 10* /I,
microhaematuria. Which examination is useless in this case:
A. Plain abdominal X-Ray
B. Ultrasonic investigation of kidneys
C. Chromocystoscopy
D. Isotopic renography

19.20 36 year old male complains of right loin and right iliac fossa pain, “blowed out” abdomen, fecal and gas
retention, increased frequency of urinating. The onset of the disease was a day before, after abundant meal. Right
loin colic spread into the abdomen. Nausea. After taking medicine (?) the pain decreased, but the pain in the small
of the back was still tiring, “blowing out” of the abdomen and inclination for frequent urination appeared. The last
defecation was a day before. Pulse rate: 90, BP - 140/80 mmHg, T - 36,6 C*. Patient is pale, his tongue with thin
white coating. The abdomen is tender, tympanic under the percussion, peristalsis is weak. Kidneys are unpalpable.
The right loin is painful when knocked. A lot of gas can be seen in small and large bowel in plain abdominal X-Ray.
The renal outlines are obscured, no shadows of stones can be seen. There are 3-5 WBC/ml and 1-2-3 RBC/ml,
urates in the urine, which is acid. What is the preliminar diagnosis:
A. Acute gastritis
B. Acute pancreatitis
C. Perforated duodenal ulcer
D. Bowel obstruction
E. Renal stones

19.21 36 year old male complains of right loin and right iliac fossa pain, “blowed out” abdomen, fecal and gas
retention, increased frequency of urinating. The onset of the disease was a day before, after abundant meal. Right
loin colic spread into the abdomen. Nausea. After taking medicine (?) the pain decreased, but the pain in the small
of the back was still tiring, “blowing out” of the abdomen and inclination for frequent urination appeared. The last
defecation was a day before. Pulse rate: 90, BP - 140/80 mmHg, T - 36,6 C*. Patient is pale, his tongue with thin
white coating. The abdomen is tender, tympanic under the percussion, peristalsis is weak. Kidneys are unpalpable.
The right loin is painful when knocked. A lot of gas can be seen in small and large bowel in plain abdominal X-Ray.
The renal outlines are obscured, no shadows of stones can be seen. There are 3-5 WBC/ml and 1-2-3 RBC/ml,
urates in the urine, which is acid. What should the treatment be:
A. Thermal procedures, hot bathing
B. Gastric tube
C. Enema
D. Paranephric novocaine blockade
E. Catheterisation of the ureter

19.22 Female patient noticed blood in the urine, then left loin colic appeared. The pain decreased after thermal
procedures. The next day there was a little of blood in the urine and the out-patient physician prescribed Cystenal
drops and Rutin tablets. What the disease was suspected by the physician:
A. Renal TB
B. Renal stones
C. Pyelonephritis
D. Renal tumour
lsmusis.lsmuni.lt/Klausimai/Spausdinti?Length=0?Kalba=EN&KategorijaId=121&Kalbos_input=EN&Kalbos=EN&KategorijaEn_input=Urology&Kategor… 3/7
3/27/2019 LSMUSIS
19.23 65 years old male complains of burdened and more frequent urinating, tiring pain in spinal column and
sacrum. The neurologist was treating like radiculitis, but the treatment was not succesful. What is the diagnosis:
A. Spondyloarthrosis
B. Prostatic adenoma
C. Prostatic cancer
D. Stones in the urinary bladder
E. Prostatitis

19.24 65 years old male complains of burdened and more frequent urinating, tiring pain in spinal column and
sacrum. The neurologist was treating like radiculitis, but the treatment was not succesful. Which of the following
examinations carries least information for the diagnosis:
A. Cystoscopy
B. Biopsy of the prostate
C. Ultrasonic investigation of kidneys and urinary bladder
D. Columnal X-Ray
E. Palpation of the prostate

19.25 70 years old male complains of urinary retention, weakness, poor appetite. He has headaches in the
mornings and usually is thirsty. Dysuria persists already for two years, the patient used to urinate at night. In recent
time he has to drink some water in the nighttime as well. There were episodes of urinary incontinence several
nights, but their were hesitancy and dribbling in the daytime. Physical examination: pulse rate - 82/min, BP -140/90
mmHg, T - normal. The patient is exhausted, pale, his tongue with brown coating. There is a strange smell coming
from his mouth. Elastic and a little bit painful lump, sized as a childs head, can be palpated above his hip-bone.
There is no pain in the loins when knocked. Prostate is oval, large, elastic, painless. Catheterisation with rubber
catheter failed. What is the diagnosis:
A. Urethral stricture
B. Tumour of the large bowel
C. Cancer of the urinary bladder
D. Prostatic cancer
E. BPH (benign prostatic hyperplasy)

19.26 70 years old male complains of urinary retention, weakness, poor appetite. He has headaches in the
mornings and usually is thirsty. Dysuria persists already for two years, the patient used to urinate at night. In recent
time he has to drink some water in the nighttime as well. There were episodes of urinary incontinence several
nights, but their were hesitancy and dribbling in the daytime. Physical examination: pulse rate - 82/min, BP -140/90
mmHg, T - normal. The patient is exhausted, pale, his tongue with brown coating. There is a strange smell coming
from his mouth. Elastic and a little bit painful lump, sized as a childs head, can be palpated above his hip-bone.
There is no pain in the loins when knocked. Prostate is oval, large, elastic, painless. Catheterisation with rubber
catheter failed. What should the first aid be:
A. Proserine, calimine or other parasympathomymetics
B. Catheterisation of urinary bladder with metal catheter
C. Puncture of urinary bladder (with needle)
D. Puncture of urinary bladder (with troacar)

19.27 51 years old male brought from the place of car accident complains of weakness and pains in the right arm,
perineum and hypogastrium. Due to pains in perineum he is unable to urinate, there were only few drops of blood.
Physical examination : pulse rate - 120/min, BP - 100/55 mmHg. The patient is pale, his abdomen takes part in
breathing. Hypogastrium is tender. Elastic and painful mass, sized as a head of a child, can be palpated above his
hip-bone. What examination is necessary before giving the diagnosis:
A. Catheterisation of the urinary bladder
B. Intravenous urography
C. Isotopic renography
D. Contrast urethrography
E. Cystoscopy

19.28 51 years old male brought from the place of car accident complains of weakness and pains in the right arm,
perineum and hypogastrium. Due to pains in perineum he is unable to urinate, there were only few drops of blood.
Physical examination : pulse rate - 120/min, BP - 100/55 mmHg. The patient is pale, his abdomen takes part in
breathing. Hypogastrium is tender. Elastic and painful mass, sized as a head of a child, can be palpated above his
hip-bone. What should the treatment be:
A. A. Suture of the urethra
B. Resection of urethra 2 - 3 months after the epicystostomy
lsmusis.lsmuni.lt/Klausimai/Spausdinti?Length=0?Kalba=EN&KategorijaId=121&Kalbos_input=EN&Kalbos=EN&KategorijaEn_input=Urology&Kategor… 4/7
3/27/2019 LSMUSIS
C. Catheterisation of the urinary bladder 2 - 3 times a day
D. Antibiotics and continual catheterisation
E. Antibiotics and epicystostomy

19.29 68 years old male complains of weakness, fever with shivers in the evenings, constant pains in the right loin
and right side of the abdomen, radiating to the chest and right hip. The pain is more severe while taking a deep
breath in and stretching his right leg. The onset was approximately 2 weeks ago. A little bit earlier the patient had
cought cold, he was sneezing and coughing. Physical examination : pulse rate - 92/min, BP - 140/80mmHg,
breathing -24/min, T- 38ºC. The patient is pale, sweaty. He tries to lie on his right side with his right leg flexed. The
right side of the diaphragm is higher under the percussion than the left one. The right waist is "plained", the skin is
hyperaemic. The wrinkle of the skin in that region is thicker. The right upper abdominal quadrant and the right hip
are painful under the deep palpation. The pain is more severe when the right leg is stretched. The kidneys are
unpalpable. Blood count: WBC -19 .10 /I, Hb -111 g/l, ESR - 32 mm/h. Plain abdominal X-Ray: the outlines of the
left kidney are clear, neither right kidney, nor right ileopsoas muscle can't be seen. There is some effusion in the
right pleural cavity. What is the preliminar diagnosis:
A. Acute cholecystitis
B. Shielded perforation of the duodenum
C. Subdiaphragmatic abscess
D. Paranephritis
E. Paracolitis

19.30 68 years old male complains of weakness, fever with shivers in the evenings, constant pains in the right loin
and right side of the abdomen, radiating to the chest and right hip. The pain is more severe while taking a deep
breath in and stretching his right leg. The onset was approximately 2 weeks ago. A little bit earlier the patient had
cought cold, he was sneezing and coughing. Physical examination : pulse rate - 92/min, BP - 140/80mmHg,
breathing -24/min, T- 38ºC. The patient is pale, sweaty. He tries to lie on his right side with his right leg flexed. The
right side of the diaphragm is higher under the percussion than the left one. The right waist is "plained", the skin is
hyperaemic. The wrinkle of the skin in that region is thicker. The right upper abdominal quadrant and the right hip
are painful under the deep palpation. The pain is more severe when the right leg is stretched. The kidneys are
unpalpable. Blood count: WBC -19 .10 /I, Hb -111 g/l, ESR - 32 mm/h. Plain abdominal X-Ray: the outlines of the
left kidney are clear, neither right kidney, nor right ileopsoas muscle can't be seen. There is some effusion in the
right pleural cavity. What is the most valuable investigation in this case:
A. Retrogradic investigation of the right kidney
B. Fibrogastroscopy
C. Pleural puncture
D. Intravenous urography

19.31 25 years old single man complains of pains in the right iliac fossa and right scrotum, fever and frequent
micturition. The onset was three days before and the fever appeared yesterday. There was right renal colic 2 years
ago when crystals were found as well. 1,5 month ago there was secretion from the urethra. After the administration
of bicillin the secretion disappeared. Physical examination: pulse rate - 84/min, BP -120/70 mmHg, T - 37,6*C. The
tongue is clean, the abdomen takes part in breathing. Right iliac fossa and inguinal canal are painless under deep
palpation, the kidneys are unpalpable. The right side of scrotum is bigger and the skin over it is reddish. Right
ductus deferens is thicker and painful. The right testis is as big as egg of a hen and extremely painful. The pain is
not as much severe in the anterior surface of the latter. Epididymis is unpalpable. The left testis is of normal
volume and consistence. There is no secretion from urethra, ifs labies are stuck together. What is the diagnosis:
A. Right testicular tumour
B. Right acute epididymitis
C. Right chronic epididymitis
D. Right acute orchoepididymitis
E. Right hydrocoele

19.32 25 years old single man complains of pains in the right iliac fossa and right scrotum, fever and frequent
micturition. The onset was three days before and the fever appeared yesterday. There was right renal colic 2 years
ago when crystals were found as well. 1,5 month ago there was secretion from the urethra. After the administration
of bicillin the secretion disappeared. Physical examination: pulse rate - 84/min, BP -120/70 mmHg, T - 37,6*C. The
tongue is clean, the abdomen takes part in breathing. Right iliac fossa and inguinal canal are painless under deep
palpation, the kidneys are unpalpable. The right side of scrotum is bigger and the skin over it is reddish. Right
ductus deferens is thicker and painful. The right testis is as big as egg of a hen and extremely painful. The pain is
not as much severe in the anterior surface of the latter. Epididymis is unpalpable. The left testis is of normal
volume and consistence. There is no secretion from urethra, ifs labies are stuck together. Which examination
carries less information:
lsmusis.lsmuni.lt/Klausimai/Spausdinti?Length=0?Kalba=EN&KategorijaId=121&Kalbos_input=EN&Kalbos=EN&KategorijaEn_input=Urology&Kategor… 5/7
3/27/2019 LSMUSIS
A. Palpation
B. Exploration of prostatic secretion
C. Intravenous urography
D. Urine culture
E. Exploration of "two portions" of the urine

19.33 59 years old male suffering from hangover complains of abdominal pain, which is most severe in
hypogastrium. He has no inclination for urinating. He can't remember what was the onset of the disease, because
he was too drunk. He remembers only vomiting. Physical examination: pulse rate - 100/min, BP - 110/70 mmHg, T
- 37,6*C. The patient is exhausted, with subicterus. His right chin and arm are scratched. He cant lie down,
because the pain is less severe while sitting. Hypogastrium is most painful under palpation. Guarding and rebound
tenderness of the abdomen are obvious. The liver is enlarged. The peristalsis can be heard. The prostate is a little
bit enlarged, with "plained" median sulcus, but soft. Cavum Douglasi is bulging over prostate. Blood count: Hb -
127 g/l; RBC-3,210 /I;WBC-10,1 10 /I. What is the diagnosis:
A. Urinary retention
B. Anuria
C. Extraperitoneal rupture of the urinary bladder
D. Peritonitis due to cholecystopancreatitis
E. Peritonitis due to intraperitoneal rupture of the urinary bladder

19.34 59 years old male suffering from hangover complains of abdominal pain, which is most severe in
hypogastrium. He has no inclination for urinating. He can't remember what was the onset of the disease, because
he was too drunk. He remembers only vomiting. Physical examination: pulse rate - 100/min, BP - 110/70 mmHg, T
- 37,6*C. The patient is exhausted, with subicterus. His right chin and arm are scratched. He cant lie down,
because the pain is less severe while sitting. Hypogastrium is most painful under palpation. Guarding and rebound
tenderness of the abdomen are obvious. The liver is enlarged. The peristalsis can be heard. The prostate is a little
bit enlarged, with "plained" median sulcus, but soft. Cavum Douglasi is bulging over prostate. Blood count: Hb -
127 g/l; RBC-3,210 /I;WBC-10,1 10 /I. Which examination is the most valuable in this case:
A. Catheterisation of the urinary bladder
B. Ultrasonic investigation of the urinary bladder
C. Intravenous urography
D. Contrast examination of the urinary bladder
E. Cystoscopy

19.35 22 years old male complaining of weakness, poor appetite, dull pain below the left arch of the ribs and in the
left loin. The patient feels sick for 2 months. Earlier he has been absolutely healthy, except the fact, that his left
testis descended only 1 year ago. Physical examination: pulse rate - 60/min, BP - 120/70 mmHg, T - 36,6*C. Pale.
Abdomen is taking part in breathing, painless. Deep below the left arch of the ribs one can palpate a lump with
smooth surface. The left testis is bigger and harder than the right one, painless, with smooth surface. Ductus
deferens and epididymis are normal. What is the diagnosis:
A. Renal tumour
B. Tumour of the large bowel
C. Orchitis
D. Hydrocele
E. Testicular tumour

19.1 - C 19.2 - C 19.3 - D 19.4 - D 19.5 - B 19.6 - E


19.7 19.8 19.9
1-A 1-A 1-A
2-A 2-A 2-A
3-C 3-C 3-B
4-C 4-B 4-B
5-B 5-B
19.10 19.11 19.12 19.13 19.14 19.15 19.16 19.17
1 1 1 1 1 1 1 1
2 3 3 3 2 2 2 2
3 3 3 3 3
4 4
5
19.18 - D 19.19 - C 19.20 - E 19.21 - E 19.22 - B 19.23 - C 19.24 - A 19.25 - E
19.26 - C 19.27 - D 19.28 - A 19.29 - D 19.30 - D 19.31 - B 19.32 - C 19.33 - E
lsmusis.lsmuni.lt/Klausimai/Spausdinti?Length=0?Kalba=EN&KategorijaId=121&Kalbos_input=EN&Kalbos=EN&KategorijaEn_input=Urology&Kategor… 6/7
3/27/2019 LSMUSIS

19.34 - D 19.35 - E

lsmusis.lsmuni.lt/Klausimai/Spausdinti?Length=0?Kalba=EN&KategorijaId=121&Kalbos_input=EN&Kalbos=EN&KategorijaEn_input=Urology&Kategor… 7/7
3/27/2019 LSMUSIS

Toxicology
I type tasks. Choose only one best answer

2.1 Absorption is:


A. Metabolism of poisons
B. Distribution of poisons
C. Process by which a xenobiotic enters the body
D. Excretion of poisons

2.2 Which antidote should be prescribed for treatment of methanol intoxication:


A. Glucose solution
B. Amyl nitrite
C. Activated charcoal
D. Ethanol

2.3 After poisoning by clonidine for patient with hypotensia and bradycardia is recommended:
A. To drink large amounts of water
B. To use laxatives
C. Atropine and infusiontherapy
D. To observe the patient

2.4 In which case gastric lavage is contraindicated:


A. Some blood is presented in vomiting mass
B. Before activated charcoal
C. After ingestion of strong alkali
D. Coma and artifitial lung ventilation

2.5 The methabolites of ethanol are:


A. Oxalic acid
B. Acetaldehyde and acetic acid, later on – carbon dioxide and water
C. Formaldehyde
D. Formic acid

II type tasks. For each numbered item,selct the one lettered heading that is most closely asssciated with it

2.6 Which antidote should be prescribed for each of the following poisonings:
1 - Sodium nitrite
2 - Isoniazid
3 - Ethylene glycol
4 - Phosphoroorganic agents
5 - Opioids
A. Pyridoxine
B. Ethanol
C. Naloxone, nalorphine
D. Methylene blue
E. Atropine

2.7 Which combination of symptoms is usual for each of the following poisonings:
1 - Phosphoroorganic agents
2 - Methanol
3 - Acetic acid
4 - Carbon monoxide
A. Nausea, vimiting, dizziness, visual disturbances, pathologic changes of ocular fundus, severe
methabolic acidosis, hypovolemia
B. Nausea, vomiting, dizziness, oral pain, retrosternal pain, chemical burn of oral cavity, acidosis,
hemolysis
C. Nausea, pulsating hedache, dizziness, weakness, drowsiness, red colour of the skin,
carboxihemoglobinemia
D. Nausea, dizziness, weakness, hypersalivation, watery nasal discharge, lacrimation, increased
bronchial secretion, abdominal pain, dyspnea.

lsmusis.lsmuni.lt/Klausimai/Spausdinti?Length=0?Kalba=EN&KategorijaId=104&Kalbos_input=EN&Kalbos=EN&KategorijaEn_input=Toxicology&Kate… 1/4
3/27/2019 LSMUSIS
III type tasks. For each question there is one or more correct answers:
A – if correct answers are 1,2,3
B – if correct answers are 1 and 3
C – if correct answers are 2 and 4
D – if correct answer is 4
E – if correct are all answers above

2.8 In case of which severe poisoning the urgent hemodialysis is indicated:


1. Amitriptyline
2. Methanole
3. Carbon monoxide
4. Ethylene glycol
5. Aminazine

2.9 Which methods of treatment should be provided for methanol poisoning:


1. Gastric lavage
2. Correction of methabolic acidosis
3. Ethanol
4. Folic acid
5. Hemodialysis

2.10 Which methods of treatment should be provided for ethyleme glycol poisoning:
1. Ethanol
2. Hemodialysis
3. Pyridoxine and thiamine
4. Methylene blue
5. Atropine

2.11 The pathogenetic mechanisms of hydrogen sulfide poisoning are following:


1. Methemoglobinemia
2. Acute renal failure
3. Strong local burning effect
4. Inhibition of tissue enzyme cytochrome oxidase
5. Hemolysis

2.12 Which findings are presented in Amanita phalloides poisoning:


1. 6-48 hours latent period
2. 1-2 hours latent period
3. Hepatotoxicity
4. Hemolysis
5. Irritation of oral mucosa

2.13 Please choose an example of lethal biotransformation (lethal synthesis) among following variants of poisons
metabolism:
1. Methabolism of isopropanol to acetone
2. Methabolism of ethanol to acetaldehyde and acetic acid
3. Methabolism of acetaldehyde to carbon dioxide and water
4. Methabolism of methanol to farmaldehyde and formic acid
5. Absence of carbon monoxide methabolism to other substances

2.14 The antidotes of phosphoroorganic agents are:


1. Ethanol
2. Atropine
3. Oxygen
4. Cholinesterase reactivating agents
5. Pyridoxine

2.15 Which antidotes should be prescribed in case of intoxication due to unknown poisons:
1. Atropine
2. Dimercaprol
3. Naloxone
4. Antidote should not be prescribed
lsmusis.lsmuni.lt/Klausimai/Spausdinti?Length=0?Kalba=EN&KategorijaId=104&Kalbos_input=EN&Kalbos=EN&KategorijaEn_input=Toxicology&Kate… 2/4
3/27/2019 LSMUSIS
5. Prescription of different antidotes every 30 minutes

IV type tasks. Choose only one best answer

2.16 A 50-year-old unconsious man was found at the street. He was admitted to ICU of the nearest hospital. After
physical examination the diagnosis of alcohol intoxication was made. Despite to intensive treatment the positive
dynamic not appeared. Some time later physician began to doubt if diagnosis is correct. After which period of time
physician could doubt:
A. After 10 hours
B. After 8 hours
C. After 1 hour
D. After 3 hours
E. After 12 hours

2.17 A 50-year-old unconsious man was found at the street. He was admitted to ICU of the nearest hospital. After
physical examination the diagnosis of alcohol intoxication was made. Despite to intensive treatment the positive
dynamic not appeared. Some time later physician began to doubt if diagnosis is correct. If diagnosis of alcohol
intoxication is doubtful, which disease or intoxication could be suspect first of all:
A. Methemoglobin-generating agents intoxication
B. Heavy metals intoxication
C. Mushrooms intoxication
D. Methanol or ethylene glycol, neurotropic medications intoxication, cerebral trauma, acute cerebrovascular
disturbances, hypoglycemic coma

2.18 A 40-year-old healthy man was drinking a lot of alcohol which he bought at nearest market during some days.
Severe headache, vomiting, psychomotoric agitation were developed in 2 hours after last drinking. On admission
he complained of visual disturbances. Physical examination revealed drowsiness, meningeal symptomes, absence
of focal neurological signs, dilated pupils, blurred vision, heart rate 110/min., blood pressure 105/70 mm Hg,
coated tongue. The preliminary diagnosis is:
A. Ethanol intoxication
B. Ethanol (with possible phosphoroorganic agent admixture) intoxication
C. Methanol intoxication
D. Tranquilizers intoxication

2.19 A 40-year-old healthy man was drinking a lot of alcohol which he bought at nearest market during some days.
Severe headache, vomiting, psychomotoric agitation were developed in 2 hours after last drinking. On admission
he complained of visual disturbances. Physical examination revealed drowsiness, meningeal symptomes, absence
of focal neurological signs, dilated pupils, blurred vision, heart rate 110/min., blood pressure 105/70 mm Hg,
coated tongue. Which laboratory data can help in estimation of diagnosis:
A. Complete blood count, serum cholesterol
B. Complete blood count, proteins
C. Serum electrolytes and glucose
D. Acid-base balance, ocular fundus examination

2.20 Car driver, reparing the car, ingested accidentally about 50 ml of antifreez. After short period of time he came
to the hospital. Which treatment should be presribed by doctor:
A. Atropine, forced diuresis
B. To drink a lot of water, induced vomiting, laxatives
C. Gastric lavage, ethanol, hemodialysis
D. Dimercaprol, forced diuresis, oculist’s consultation

2.21 Car driver, reparing the car, ingested accidentally about 50 ml of antifreez. After short period of time he came
to the hospital. In the case of incorrect treatment the following complication will develop:
A. Cerebrovascular disturbances
B. Myocardial infaction
C. Acute renal failure
D. Hemolysis

2.22 A 40-year-old man was admitted to the hospital with suspition of sodium nitrite intoxication. Which laboratory
test (additionally to anamnesis and physical examination data) will be helpful in evaluation of severity of
intoxication:
A. Blood carboxyhemoglobin

lsmusis.lsmuni.lt/Klausimai/Spausdinti?Length=0?Kalba=EN&KategorijaId=104&Kalbos_input=EN&Kalbos=EN&KategorijaEn_input=Toxicology&Kate… 3/4
3/27/2019 LSMUSIS
B. Serum alcohol
C. Blood methemoglobin
D. Serum free hemoglobin
E. Serum aminotransferazes

2.23 A 40-year-old man was admitted to the hospital with suspition of sodium nitrite intoxication. Which value of
poison concentration corresponds to severe intoxication:
A. 60 %
B. 5 %
C. 25 %
D. 10 %
E. 30 %

2.24 After ingestion of 50-60 ml of methanol a 35-year-old female was admitted to the hospital. Which
management sould be provided in this case:
A. Laxatives, suportive treatment, observation of patient
B. Gastric lavage, correction of methabolic acidosis and hypovolemia, antidotes (ethanol and folic acid),
hemodialysis, suportive treatment
C. Gastric lavage, forced diuresis
D. Vitamins of B group, infusiontherapy, correction of hypokalemia and acidosis

2.25 Young female was admitted to the hospital 30 minutes after accidental ingestion of acetic acid. Patient
complained about burning oral pain, nausea and weakness. Physical examination revealed hyperemia and mild
edema of oral mucosa. Which management should be provided in this case:
A. Apomorphine hydrochloride for induction of vomiting
B. Alcaline solutions intravenously
C. Narcotic analgesics and gastric lavage with cold water, sodium bicarbonate intravenously, observation of
patient for hemolysis and acute renal failure, suportive treatment
D. A lot of water with sodium bicarbonate and induced vomiting

2.1 - C 2.2 - D 2.3 - C 2.4 - C 2.5 - B


2.6 2.7
1-D 1-D
2-A 2-A
3-B 3-B
4-E 4-C
5-C
2.8 2.9 2.10 2.11 2.12 2.13 2.14 2.15
2 1 1 4 1 4 2 4
4 2 2 3 4
3 3
4
5
2.16 - D 2.17 - D 2.18 - C 2.19 - D 2.20 - C 2.21 - C 2.22 - C 2.23 - A
2.24 - B 2.25 - C

lsmusis.lsmuni.lt/Klausimai/Spausdinti?Length=0?Kalba=EN&KategorijaId=104&Kalbos_input=EN&Kalbos=EN&KategorijaEn_input=Toxicology&Kate… 4/4
3/27/2019 LSMUSIS

Psychiatry
I type tasks. Choose only one best answer

34.1 A female patient is complaining about "bugs" crawling in her skin. She specifies the direction of "bugs'"
movement and tries to pick them out. Her skin is riddled with deep scratch marks. What psychopathological
disturbances would you assign these feelings to?
A. Affective illusions;
B. Visceral hallucinations;
C. Disturbances in psychosensorial synthesis
D. Tactille hallucinations;

34.2 Aura is specific to:


A. Major epileptic seizures;
B. Minor epileptic seizures;
C. Jacksonian convulsive seizures
D. Hysterical seizures;

34.3 Which one of the following psychopathological symptoms does not belong to the volition's pathology?
A. Abulia;
B. Hypobulia;
C. Hyperbulia;
D. Bulimia.

34.4 Severe symptoms of dementia are specific to which type of psychoorganic syndrome?
A. Apathic;
B. Asthenic;
C. Explosive;
D. Euphoric.

34.5 Which function is the first one to be affected by the late Alzheimer’s disease?
A. Speech;
B. Memory;
C. Writing;
D. Practical activity.

34.6 Which of the below statements about pathological drunkenness is incorrect?


A. Appears after small quantity of alcohol;
B. Affective disorders have no influence;
C. More common for people with epilepsy;
D. Can happen one time in a life;

34.7 Symmetrical and narrow eye pupils is one of the most common signs of opiate intoxication. Which one of the
following opiates does not cause the narrowing of eye pupils?
A. Heroin;
B. Codeine;
C. Methadon;
D. Morphine.

34.8 Which one of the following psychopathological symptoms is common to the alcohol delirium syndrome:
A. Fixative amnesia;
B. Pareidolia;
C. Intermethamorphotic delusion;
D. Kinetic automatism.

34.9 The dizziness syndrome is not common to:


A. Hypertonic crisis;
B. Hemorrhagic stroke;
C. Lead poisoning;
D. Brain tumor.

34.10 Define the personality disorder:

lsmusis.lsmuni.lt/Klausimai/Spausdinti?Length=0?Kalba=EN&KategorijaId=136&Kalbos_input=EN&Kalbos=EN&KategorijaEn_input=Psychiatry&Kat… 1/13
3/27/2019 LSMUSIS
A. It is a condition where the consciousness is narrowed as if it had transitioned to a lower state of psychic
functioning;
B. It is a disease where the lack of unity between different psychic processes is apparent: a division of some
sort.
C. It's a union of stable personality traits that affect one's ability to adapt to the environment, causing social
and professional disorders or subjective distress.
D. It is a disorder that causes great distress and the symptoms are unacceptable and alien to the patient,
even though the patient's connection with reality is not disturbed.

34.11 What is the main feature of a dependent personality:


A. Impulsiveness;
B. Passivity;
C. Perfectionism;
D. Suspiciousness.

34.12 Which personality type is most likely to have dissociative disorders?


A. Schizoid;
B. Histrionic;
C. Antisocial;
D. Paranoid.

34.13 Self-harm, suicidal threats and acts are common to which type of personality disorder:
A. Histrionic;
B. Schizoid;
C. Emotionally unstable;
D. Paranoid.

34.14 Define the obsessive-compulsive disorder:


A. Thoughts, images and actions that obsessively and stereotypically return to the patient's mind and cannot
be managed by his or her will. The patient is not critical and accepts these disturbances as his personality
trait.
B. A disorder made apparent by various obsessions emerging against the patient's will, causing intense
stress and emotional tension. The patient is critical regarding the obsessions and tries to get rid of them.
C. A disorder made apparent by obsessive fears emerging against the patient's will of not being able to deal
with a specific life situation. The patient is critical regarding this fear and wants to conquer it.

34.15 Which statement about specific phobia is incorrect?


A. Starts during the childhood or adolescence;
B. Lasts for a short while (up to 1month)
C. The patient avoids situations that cause fear;
D. The fear can reach the level of a panic attack.

34.16 Which of the following models of the disease state that the development of the disease begins when stress
experienced by a person overwhelms his or her ability to adapt to it, causing physiological reactions and disorders
of metabolic processes?
A. Cognitive;
B. Biological;
C. Psychoanalytic;
D. Biopsychosocial;

34.17 What is first prescribed for the treatment of acute psychosis?


A. Benzodiazepines;
B. Normotimics;
C. Antidepressants;
D. Neuroleptics;

34.18 The tasks of primary mental health care centers are:


A. Deliver mental health care, provide social support for mentally ill patients
B. Provide social support and direct the patient towards treatment in a hospital;
C. Provide social support during the cases of acute mental disorders;
D. Provide psychotherapeutic and psychological support.

lsmusis.lsmuni.lt/Klausimai/Spausdinti?Length=0?Kalba=EN&KategorijaId=136&Kalbos_input=EN&Kalbos=EN&KategorijaEn_input=Psychiatry&Kat… 2/13
3/27/2019 LSMUSIS
34.19 A person who is ill with a severe mental illness and refuses hospitalisation may be admitted involuntarily to
the custody of the hospital for the treatment only if:
A. The patient's family members are interested in his treatment, the request for the treatment is supported
by there signature in the medical documentation;
B. The committee of three psychiatrists confirms the need for forced treatment;
C. There is real danger that by his actions he is likely to commit serious harm to his health, life and to the
health, life of others.
D. The forced treatment is not provided in the statute of Republic of Lithuania (violate the human rights).

34.20 Can a patient with mental disorders be treated in inpatient hospital without his/her written consent?
A. Can't;
B. Can, in the emergency department;
C. Consent is always required;
D. Can, but only according to the procedure, specified by laws;

34.21 The diagnosis of mental disorder made according to:


A. Mental state examination, psychiatric medical history, paraclinical and clinical tests;
B. Psychiatric medical history, clinical and biochemical tests;
C. Mental state examination, evaluation of mental processes;
D. Evaluation of mental processes, paraclinical tests.

34.22 What is prescribed for the treatment of resistant schizophrenia?


A. Clozapine;
B. Chlorpromazine;
C. Risperidone;
D. Amisulpiride.

34.23 The overdose of which of these antidepresant group medications is the most life threatening:
A. Selective serotonin reuptake inhibitors;
B. Selective serotonin and noradrenaline reuptake inhibitors;
C. Noradrenaline reuptake inhibitors;
D. Tricyclic antidepressants.

34.24 Indications for the assignment of light therapy:


A. Moderate and severe depression;
B. Prolonged depression;
C. Seasonal (winter) depression;
D. All of the above;

34.25 Which schizophrenic symptoms are described as positive?


A. Poverty of thought content;
B. Delusions;
C. Blunted emotions;
D. Hipobulia;

34.26 Which proposition is correct:


A. Prevalence of schizophrenia - 15-20 percent, manifests more often in middle-aged males;
B. Prevalence of schizophrenia - 1-1,5 percent, manifests more often in young people of both genders
equally;
C. Prevalence of schizophrenia - 20 - 30 percent, manifests more often in young females;
D. Prevalence of schizophrenia - 5-6 percent, manifests more often in artists and scientists.

34.27 A 32 year old woman complains about her sad mood, anxiety, accompanied by a feeling of pressure in her
chest. Objective assessment: sad facial expression, slow movements, crooked and fixed sitting posture; an
interview reveals a weakened memory, a slow down of speech and thought processes. She was diagnosed with
depression. Which medication is the most appropriate for treating this syndrome?
A. Haloperidol
B. Leponex (Clozapin)
C. Coaxil (Tianeptin)
D. Sydnocarbum

34.28 Treatment of moderate and severe depression:

lsmusis.lsmuni.lt/Klausimai/Spausdinti?Length=0?Kalba=EN&KategorijaId=136&Kalbos_input=EN&Kalbos=EN&KategorijaEn_input=Psychiatry&Kat… 3/13
3/27/2019 LSMUSIS
A. Psychotherapy and antidepressants;
B. Only psychotherapy;
C. Only antidepressants;
D. Only benzodiazepines;
E. Antipsychotics and antidepressants.

34.29 Somatic (masked) depression can be diagnosed if:


A. A patient is hiding his bad mood from his relatives and the doctor in order to avoid treatment.
B. Somatic and vegetative symptoms dominate at the clinic of depression;
C. The patient's depression is caused by a severe, incurable disease;
D. Depression occurs as a side effect of the prescribed somatic medications.

34.30 A person intending to commit suicide:


A. Always hides his intentions from others;
B. Usually notifies about his intentions, directly or indirectly;
C. Does it "seriously" and irreversibly;
D. Always expects to be saved

34.31 Which of the following disorders is not associated with sexual disorders?
A. Frotteurism;
B. Paedophilia;
C. Exhibitionism;
D. Dromomania.

34.32 Which statement about nervous anorexia is inappropriate?


A. Common endocrine disorder affects the hypothalamus - hypophysis - gonad axis, asserts for women as
an amenorrhea, for men - as a reduction of libido and sexual potention.
B. Nervous anorexia – although tiresome, is not a dangerous disease and can be cured with a proper diet.
C. Mostly suffered by teenage girls and young women.
D. Nervous anorexia - a chronic mental disorder followed by secondary endocrine and metabolic disorders
that may cause the death of a patient.

34.33 Somnambulism is:


A. An altered state of consciousness where phenomena ascribed to both a state of sleep and wakefullness,
sleepwalking and amnesia show up.
B. One of the delusional syndromes showing up as complex motor acts only during the night time;
C. A sleep disorder usually emerging during the old age, conditioned by severe mental illnesses (for
example: schizophrenia)
D. A type of desire disorder showing up as increased drowsiness during the day and an increased level of
activity during the night time.

34.34 Which statement is inappropriate for the diagnostics of night horror:


A. The main symptom - one or more episodes of awakening, panic screaming, anxiety, irritation, vegetative
hyperactivity;
B. Length of one to ten minutes, onset at the first third of a sleep period;
C. It is easy to wake the patient up and calm him down.
D. The patient has a weak reaction to the effort of others to affect the episode of horror.

34.35 Which of the following disorders is ascribable to the group of sexual disorders?
A. Kleptomania;
B. Pygmalionism;
C. Trichotillomania;
D. Dipsomania.

II type tasks. For each numbered item,selct the one lettered heading that is most closely asssciated with it

34.36 Which group of features is associated with each of the following personality disorders?
1 - Antisocial personality disorder –
2 - Anxious personality disorder-
3 - Schizoid personality disorder –
A. Emotionally blunted, introspective, unable to express feelings, distant;
B. Indifferent to the feelings of others, unable to feel guilt and maintain long-term relationships

lsmusis.lsmuni.lt/Klausimai/Spausdinti?Length=0?Kalba=EN&KategorijaId=136&Kalbos_input=EN&Kalbos=EN&KategorijaEn_input=Psychiatry&Kat… 4/13
3/27/2019 LSMUSIS
C. Regular worries about possible failures, sensitivity to criticism, preference for being alone rather than
feeling rejected, unlikable
D. None of the above.

34.37 To which delusion group is each delusion assigned to?


1 - Delusion of persecution -
2 - Delusion of self-accusation-
3 - Delusion of innocence -
A. Psychogenic delusions group;
B. Affective/mood congruent delusions group;
C. Interpretational delusions group.

34.38 What personality traits appear during an illness:


1 - Alcoholism (third stage of dependence )-
2 - Traumatic head brain disorder (an injury 20 years ago caused a long-lasting disturbance of
consciousness that lasted 3 days)-
3 - Epilepsy (has manifested since 12 years old, seizures of "grand mal" type two to three times per week.
The patient is now 38 years old)-
A. Circumstantiality of thinking, delayed at reaching its goal, susceptible to dysphorea, accumulation of
negative emotions and unexpected bursts of aggression
B. Arrogant, familiar, rather cynical, has a tendency to suspect his sexual partner of infidelity (sometimes
the suspisions grow into delusions of jealousy)
C. Emotionally liable, blunted, has volatile needs, moods and impulses, unable to concentrate on directed
activity for a long time.

34.39 What specific blood alcohol concentration is associated with each of these descriptions:
1 - Euphoria, weakened ability to concentrate, increased talkativeness-
2 - Emotional instability, lack of self - criticism, slowdown of reaction, unbalanced walking, gibberish-
3 - Apathy, strongly unbalanced coordination, irregular consciousness, somnolence progressing into a
coma-
A. More than 2.5 prom.
B. 0.4- 1.5 prom.
C. 1.5- 2.5 prom.

34.40 Choose the right description for each sexual disorder:


1 - Voyeurism-
2 - Fetishism-
3 - Sadism –
4 - Transvestitism –
A. Sexual satisfaction coming from carrying and petting personal stuff of opposite sex;
B. Sexual satisfaction of watching sexual activity;
C. Attraction to wearing clothes of the opposite sex;
D. Sexual satisfaction coming from physical or psychological torment of a sexual partner.

34.41 Which one type of stupor is common to each disease:


1 - Schizophrenia –
2 - Bipolar disorder –
3 - Reactive psychosis –
A. Depressive stupor;
B. Catatonic stupor;
C. Psychogenic stupor.

34.42 Choose the best description for each of the following disorders:
1 - Dissociative amnesia -
2 - Dissociative fugue -
3 - Ganser syndrome -
A. Loss of memory followed by purposeful travel beyond the usual everyday range with a possibility for
simple social contact;
B. Uninterrupted speech and chaos of thoughts appearing after a psychotraumatic situation;
C. Loss of memory, usually concerning important past events and the current event, appearing after a
traumatic life event;
D. None of the above.
lsmusis.lsmuni.lt/Klausimai/Spausdinti?Length=0?Kalba=EN&KategorijaId=136&Kalbos_input=EN&Kalbos=EN&KategorijaEn_input=Psychiatry&Kat… 5/13
3/27/2019 LSMUSIS
34.43 Choose the best description of these disorders:
1 - Hypomnesia -
2 - Hypermnesia -
3 - Amnesia -
4 - Cryptomnesia -
A. Disturbance of memory where the patient is unable to discern what he actually heard, read or saw and
what he imagined himself.;
B. Disturbance of the processes of memorization, storing and recalling of information appearing as a loss
of memory;
C. Morbid intensification of memory;
D. Weakening of memory.

34.44 Choose the best description for each of the following disorders:
1 - Reaction to severe stress-
2 - Adjustment disorders-
3 - Dissociative disorders-
A. A short while after a stressogenic event there's a partial or complete loss of bodily sensations or
control of movement and the normal integration of memories of the past is disturbed;
B. After a stressogenic event the person's field of consciousness is constricted, his or her ability to
comprehend the environment is weakened and he or she becomes "dazed", disoriented. This is followed
by a withdrawal from the surrounding situation or agitation.
C. After a stressogenic event the person becomes worried, distressed and overcome with a depressive
mood. He or she can hardly cope with everyday activities and is unable to plan the future.

34.45 Choose the best description for each clinical type of schizophrenia:
1 - Paranoid schizophrenia –
2 - Simple schizophrenia –
3 - Hebephrenic schizophrenia –
A. A disorder in which there is an insidious but progressive development of oddities of conduct, inability to
meet the demands of society, and decline in total performance.
B. A disorder in which dominated the relatively stable delusions, usually accompanied by hallucinations
and other perceptual disturbances.
C. A disorder in which affective changes are prominent, delusions and hallucinations fleeting and
fragmentary, behaviour irresponsible and unpredictable, and mannerisms common.

III type tasks. For each question there is one or more correct answers:
A – if correct answers are 1,2,3
B – if correct answers are 1 and 3
C – if correct answers are 2 and 4
D – if correct answer is 4
E – if correct are all answers above

34.46 Types of hallucinations:


1. True hallucinations;
2. Pseudohallucinations;
3. Simple hallucinations;
4. Complex hallucinations.

34.47 Atrophic changes during a pathomorphological exam during the Pick's disease are found in which part of the
brain?
1. Frontal lobes;
2. Occipital lobes;
3. Temporal lobes;
4. All parts of the brain degenerate evenly.

34.48 Which symptoms are common to acute intoxication with cocaine:


1. Euphoria;
2. Hypersexuality;
3. Agitation;
4. Tachycardia.

lsmusis.lsmuni.lt/Klausimai/Spausdinti?Length=0?Kalba=EN&KategorijaId=136&Kalbos_input=EN&Kalbos=EN&KategorijaEn_input=Psychiatry&Kat… 6/13
3/27/2019 LSMUSIS
34.49 Which features are common to a paranoid personality disorder:
1. Tendency to be hostile, rejection of apologies for insults and harm;
2. Suspiciousness,tendency to twist the facts interpreting the neutral actions of others as being hostile;
3. An inappropriately strong stance on personal rights considering the actual situation;
4. Inability to maintain long-term relationships without a problem of starting them.

34.50 Which features are common to a anankastic personality disorder:


1. Exaggerated concern for physical attraction;
2. Limited capacity to make decisions without the advice of others;
3. Suggestibility, tendency to be easily influenced by the others and the circumstances;
4. Perfectionism, preventing the completion of tasks.

34.51 Secondary personality disorders (psychopatisation) are:


1. Disorders formed by constitutional and idiopathic causes;
2. Disorders formed under the influence of strong distress.
3. Disorders formed under the influence of consitutional, social and psychological factors;
4. Disorders evoked by another mental disorder or brain disease.

34.52 Which symptom is an inappropriate descriptor of an emotionally unstable personality:


1. Emotional instability;
2. Poor impulse control;
3. Tendency toward emotional crisis;
4. Tendency to flirt.

34.53 What symptoms are common to a schizoid personality:


1. Emotional instability;
2. Emotional coldness;
3. Tendency towards emotional crises;
4. Indifference towards social norms.

34.54 Which statements about panic disorders are correct:


1. Panic attack is unpredictable;
2. Dominating symptoms of a panic attack are sudden changes of the heartbeat, chest pains, dizziness and
the feeling of being strangled.
3. Is almost always followed by a secondary fear of death and fear of loss of self-control;
4. There is a fear of a relapse after the panic attack.

34.55 A panic attack can be evoked by:


1. Emotional stress;
2. Physical exercise;
3. Psychical trauma;
4. Sexual activity.

34.56 Somatoformic autonomous dysfunction has to be differentiated from these disorders:


1. Somatic disorder;
2. Generalized anxiety disorder;
3. Somatisation disorder;
4. Schizophrenia.

34.57 Groups of symptoms of schizophrenia:


1. Volitional disorders;
2. Emotional disorders;
3. Thought disorders;
4. Mood disorders;

34.58 While treating someone with a somatisation disorder one should:


1. Refrain from persuasion that he or she is healthy;
2. Suspend from analysis requested by the patient;
3. Prioritise the control of the patient's disorder rather than use of medications;
4. Compose a schedule of regular visits in advance.

34.59 Common to the course of somatisation disorders are:


1. Periodicity: exacerbation of the disorder transitions into periods of complete recovery.
lsmusis.lsmuni.lt/Klausimai/Spausdinti?Length=0?Kalba=EN&KategorijaId=136&Kalbos_input=EN&Kalbos=EN&KategorijaEn_input=Psychiatry&Kat… 7/13
3/27/2019 LSMUSIS
2. Disorder starts after experiencing stress and, with adequate treatment, goes away within a year.
3. Disorder progresses continuously and ends in a personality defect.
4. A chronic, practically incurable disorder with periods of exacerbation and remission.

34.60 Typical features of the somatoform disorder:


1. Complaints about non-functioning organs, body alterations, abnormal appearance;
2. States that are followed by great vegetative dysfunctions, numbness, pain, fluctuations of blood pressure;
3. States that are followed by constant delusion of becoming ill with an incurable disease;
4. Constant complaints about body sensations and ailments that are hard to describe;

34.61 Mental health services are provided at:


1. Mental health care centres;
2. Addiction treatment centres;
3. Psychiatric hospitals;
4. University hospitals.

34.62 What are the most common symptoms of paranoid schizophrenia?


1. Clear apathy, disorganized speech, inadequate emotions;
2. Catatonic behaviour;
3. Clear affective disorders, hallucinations and delusions;
4. Thought echo, thought insertion, auditory hallucinations, relationship delusions.

34.63 The types of expertises in Forensic Psychiatry:


1. Forensic psychiatric expertise
2. Forensic narcological expertise;
3. Forensic psychological expertise;
4. Forensic expertise of circumstances.

34.64 What could be evaluated during the Forensic Psychiatry expertise process:
1. Responsibility;
2. Non- responsibility;
3. Limited responsibility;
4. Limited non- responsibility.

34.65 During the primary examination of acutely ill patient doctor-psychiatrist have to evaluate:
1. Intensity and severity of psychopathological and somatic symptoms;
2. Possibility of aggressive actions;
3. Possibility of auto-aggressive actions;
4. Characteristic of workplace.

34.66 Typical (classic) antipsychotics compared to atypical (new) more frequently cause these side effects:
1. Increased secretion of prolactin;
2. Drug-induced Parkinson's syndrome;
3. Body mass gain;
4. Tardive dyskinesia.

34.67 Typical features of the neuroleptic malignant syndrome:


1. Sudden onset (24-72 hrs., the first 10 days of starting to use neuroleptics);
2. Hypertonus of all of the muscles in the body (dysphagia, disruption of breathing);
3. Akinetic mutism;
4. Hyperthermia, severe autonomic dysfunction.

34.68 Atypical (new) antipsychotics are different from typical (classic) by these features:
1. More effective in the treatment of positive symptoms (hallucinations, delusions, affect of anger);
2. More effective in the treatment of psychotic disorders in older patients;
3. More effective in the treatment of treatment resistant psychotic disorders;
4. More effective in the treatment of negative symptoms (bluntness of emotions, social distancing, lack of
motivation).

34.69 Indications for prescription of selective serotonin reuptake inhibitors:


1. Depression;
2. Panic disorder;
3. Obsessive–compulsive disorder;
lsmusis.lsmuni.lt/Klausimai/Spausdinti?Length=0?Kalba=EN&KategorijaId=136&Kalbos_input=EN&Kalbos=EN&KategorijaEn_input=Psychiatry&Kat… 8/13
3/27/2019 LSMUSIS
4. Social anxiety disorder.

34.70 Medications belonging to the selective serotonin reuptake inhibitors are:


1. Sertraline;
2. Venlafaxine;
3. Citalopram;
4. Mirtazapine.

34.71 Benzodiazepines are prescribed for:


1. For treatment of anxiety disorders;
2. For short - term treatment of insomnia;
3. Fot treatment of convulsive syndromes and epileptic states;
4. For treatment of spastic conditions of skeletal muscles.

34.72 Medications belonging to the benzodiazepines are:


1. Alprazolam;
2. Bromazepam;
3. Lorazepam;
4. Reboxetine.

34.73 Medications belonging to the mood-stabilizers (normothymics)are:


1. Carbamazepinum;
2. Lithium;
3. Valproic acid derivatives;
4. Donepezil.

34.74 Indications for the assignment of electroconvulsive therapy:


1. Severe depression with high risk of suicide;
2. Treatment resistant depression;
3. Persistent delusional disorder;
4. Catatonic schizophrenia.

34.75 It is uncommon for a schizoaffective disorder:


1. Symptoms of schizophrenia;
2. Symptoms of depression;
3. Symptoms of mania;
4. Symptoms of dementia.

34.76 Which of the following symptoms are common to paranoid schizophrenia?


1. Primary interpretative delusion (delusion of infidelity, invention, persecution);
2. Hallucinations and pseudohallucinations;
3. Illogical and circumstantial thinking;
4. Euphoric, elevated mood;
5. Catatonic stupor.

34.77 Which factors increase the risk of suicide?


1. Loneliness;
2. A breakthrough of a stereotype of life;
3. Previous suicide attempts;
4. Suicides in the family;
5. Psychopathological features: agitation, insomnia and ideas of self-accusation.

34.78 Which of the following drugs can cause a mania-like syndrome:


1. Amphetamines;
2. Tricyclic antidepressants;
3. Corticosteroids;
4. Reserpine.

34.79 During which somatic disorders can a depressive syndrome develop?


1. Pancreatic cancer;
2. Hypertension;
3. Hypothyroidism;
4. Peptic ulcer.
lsmusis.lsmuni.lt/Klausimai/Spausdinti?Length=0?Kalba=EN&KategorijaId=136&Kalbos_input=EN&Kalbos=EN&KategorijaEn_input=Psychiatry&Kat… 9/13
3/27/2019 LSMUSIS
34.80 Which of the following somatic symptoms are common to a typical episode of depression:
1. Loss of appetite;
2. Constipation;
3. Reduced sexual desire;
4. Heart arrhythmia;

34.81 Nervous anorexia has the following characteristics:


1. Body weight at least 15 percent lower than normal;
2. Conscious weight reduction (diet, vomiting, use of laxatives, hard exercising and so on.);
3. Disturbed perception of body shape, dysmorphophobia;
4. Delusion of influence, pseudohallucinations.

IV type tasks. Choose only one best answer

34.82 A 27 year old man is brought into an emergency room by an ambulance accompanied by a couple of
friends. According to his friends, the patient has been using various drugs for about 7 years (heroin, LSD,
amphetamine, cocaine). At the same time the patient abused alcohol drinking for 3-4 days straight. For the last few
days the patient has been apathetic, drowsy, mostly lying in bed smiling inadequately and failing to meaningfully
answer some questions. Occasionally he became restless, excited, claiming strong bone and muscle pain.
Afterwards he would become drowsy again. Friends can't tell how long the apathy and agitation episodes have
lasted. Visiting the friend today they found him lying in bed, breathing jerkily, the face was blue and they failed to
wake him up. Objective examination of the patient: dry scurfy skin, fragile nails, forearms of both hands have
multiple scars, cyanotic face, failure to breathe, no verbal contact, contracted pupils and no pupillary reaction to
light. What is the diagnosis of the disease:
A. Opiate intoxication;
B. Cocaine intoxication
C. Hallucinogenic drugs intoxication;
D. Alcohol intoxication

34.83 A 27 year old man is brought into an emergency room by an ambulance accompanied by a couple of
friends. According to his friends, the patient has been using various drugs for about 7 years (heroin, LSD,
amphetamine, cocaine). At the same time the patient abused alcohol drinking for 3-4 days straight. For the last few
days the patient has been apathetic, drowsy, mostly lying in bed smiling inadequately and failing to meaningfully
answer some questions. Occasionally he became restless, excited, claiming strong bone and muscle pain.
Afterwards he would become drowsy again. Friends can't tell how long the apathy and agitation episodes have
lasted. Visiting the friend today they found him lying in bed, breathing jerkily, the face was blue and they failed to
wake him up. Objective examination of the patient: dry scurfy skin, fragile nails, forearms of both hands have
multiple scars, cyanotic face, failure to breathe, no verbal contact, contracted pupils and no pupillary reaction to
light. What is the treatment approach:
A. Gastric lavage and infusion therapy;
B. Presciption of opioid antagonists;
C. Presiciption of neuroleptics.

34.84 The patient, a 22-year-old man, has an impulsive personality, is easily provoked, his mood is volatile and
has a tendency to get involved in conflicts. Under the influence of alcohol the patient gets out of control, becomes
angry, aggressive and dysphoric. Frequent attacks of aggression and self-aggression. The patient has attempted
to cut his veins several times before. Frequent alcohol abuse and amnesic forms of intoxication. He has been fined
for driving under the influence of alcohol before. He drank about a bottle of vodka and took 10 tab. diazepam, 10
tab. validolum, 8 tab. paracetamol today in order to commit suicide. What is the diagnosis of the disease?
A. Mental and behavioural disorders due to use of alcohol, harmful use;
B. Emotionally unstable personality disorder;
C. Antisocial personality disorder;
D. Histrionic personality disorder.

34.85 A 28 year old patient suffers from headaches and increased fatigue when doing mental work. Routine work
requires lots of physical and mental effort. He feels good in the morning, but after 1 to 2 working hours starts
suffering from growing fatigue, failure to concentrate and headaches. The patient has become sensitive, irritable
and annoyed by the noise of closing doors and playing children. Even the feeling of clothes touching the body has
become irritating. He conflicts with his family on every occasion even for the smallest of reasons and then suffers
for it deeply. The sleeping pattern is disturbed – fails to fall asleep for a long period of time in the evening. What is
the name of the syndrome?
A. Hypochondriac Syndrome;
lsmusis.lsmuni.lt/Klausimai/Spausdinti?Length=0?Kalba=EN&KategorijaId=136&Kalbos_input=EN&Kalbos=EN&KategorijaEn_input=Psychiatry&Ka… 10/13
3/27/2019 LSMUSIS
B. Paranoia Syndrome;
C. Asthenic syndrome;
D. Depression syndrome.

34.86 A 28 year old patient suffers from headaches and increased fatigue when doing mental work. Routine work
requires lots of physical and mental effort. He feels good in the morning, but after 1 to 2 working hours starts
suffering from growing fatigue, failure to concentrate and headaches. The patient has become sensitive, irritable
and annoyed by the noise of closing doors and playing children. Even the feeling of clothes touching the body has
become irritating. He conflicts with his family on every occasion even for the smallest of reasons and then suffers
for it deeply. The sleeping pattern is disturbed – fails to fall asleep for a long period of time in the evening.What is
the treatment approach:
A. Psychopharmacotherapy;
B. Psychotherapy;
C. Electroconvulsive therapy;
D. Combination of psychopharmacotherapy and psychotherapy.

34.87 A 29-year-old man complains that he is afraid to have eye contact with other people, because he is guilty
against humanity, has a “devil inside”, hears the voice of devil in his head. The patient claims that he has had
mental problems for a while now ("I have been running away from myself for my whole life. I need to be nurtured. I
do not understand myself."). According to his household, the inappropriate behaviour was being observed for
about 2 years and his condition has substantially deteriorated during the past 3-4 months. The patient has become
anxious, inadequate, excited, has begun to stay up at night, has told "silly" ideas - that he found O. Bin Laden
hiding in his house or had made significant inventions for humanity. The patient claims that his family has begun to
fear him for they failed to understand his thoughts, unconventional behaviour and have decided on the need for
treatment at a psychiatric clinic. State of mind: conscious. Correct orientation in relation to of place, time and
himself. Good verbal contact. Appearance: messy. Suspicious, strained during a conversation, anxious. Thinking is
incoherent and illogical with elements of tangentiality. He answers the questions after a brief pause in a monotonic
voice and claims that there are many thoughts in his mind, but it is difficult to compile and express them - they are
chaotic and disorderly. Emotions are blunted and his face mimics lack vitality. The patient expresses ideas about
self-accusation (“I am guilty of the greatest of sins - pride and greed”), talks about being possessed (“there is a
devil inside of me and I can't let him out"), claims that he can influence other people (“when I hold the palms of my
hands up in the air, the energy flowing out of them can cure all the diseases in the world”
A. Schizoaffective disorder, manic type;
B. Acute schizophrenia-like psychotic disorder;
C. Delusional disorder;
D. Paranoid schizophrenia.

34.88 A 29-year-old man complains that he is afraid to have eye contact with other people, because he is guilty
against humanity, has a “devil inside”, hears the voice of devil in his head. The patient claims that he has had
mental problems for a while now ("I have been running away from myself for my whole life. I need to be nurtured. I
do not understand myself."). According to his household, the inappropriate behaviour was being observed for
about 2 years and his condition has substantially deteriorated during the past 3-4 months. The patient has become
anxious, inadequate, excited, has begun to stay up at night, has told "silly" ideas - that he found O. Bin Laden
hiding in his house or had made significant inventions for humanity. The patient claims that his family has begun to
fear him for they failed to understand his thoughts, unconventional behaviour and have decided on the need for
treatment at a psychiatric clinic. State of mind: conscious. Correct orientation in relation to of place, time and
himself. Good verbal contact. Appearance: messy. Suspicious, strained during a conversation, anxious. Thinking is
incoherent and illogical with elements of tangentiality. He answers the questions after a brief pause in a monotonic
voice and claims that there are many thoughts in his mind, but it is difficult to compile and express them - they are
chaotic and disorderly. Emotions are blunted and his face mimics lack vitality. The patient expresses ideas about
self-accusation (“I am guilty of the greatest of sins - pride and greed”), talks about being possessed (“there is a
devil inside of me and I can't let him out"), claims that he can influence other people (“when I hold the palms of my
hands up in the air, the energy flowing out of them can cure all the diseases in the world”
A. Carbamazepin;
B. Olanzapin;
C. Clozapine;
D. Escitalopram.

34.89 The patient, a 43 year old woman, has been feeling more nervous, irritable and has had mood swings for
about 6-7 years. At first it became very hard to concentrate at work and the anxiety, tension and tearfulness
became apparent. Her sleep was disturbed and her mood fell. She contacted psychiatrists and Sertraline 50 mg
od per os was prescribed. When she started taking the medicine, her mood improved, but remained inadequate.
lsmusis.lsmuni.lt/Klausimai/Spausdinti?Length=0?Kalba=EN&KategorijaId=136&Kalbos_input=EN&Kalbos=EN&KategorijaEn_input=Psychiatry&Ka… 11/13
3/27/2019 LSMUSIS
Afterwards, she lost her appetite, her body mass dropped, fear of the future, anxiety, tension and anhedonia
appeared, work efficiency fell. She could not concentrate and was fatigued by the onset of bad mood and became
tearful. Because of these complaints the patient was treated at a mental hospital using Citalopram 20 mg od per
os and Amitriptyline 75 mg od per os. She was addressed for continuation of treatment to the out- patient clinic
after her condition improved. The patient continued the treatment taking Escitalopram. During the last month the
patient has become irritable and angry again and her night sleep is disturbed. For 2 weeks the patient has been
acting inappropriately(started spending a lot of money on unnecessary things), experiences elevated mood, feels
excited even aggressive occasionally, her speech is chaotic, creates unreal future plans and doesn’t sleep at night.
What is the diagnosis?
A. Hypomania;
B. Recurrent depressive disorder, current episode moderate;
C. Mania without psychotic symptoms;
D. Bipolar affective disorder, current episode manic

34.90 The patient, a 43 year old woman, has been feeling more nervous, irritable and has had mood swings for
about 6-7 years. At first it became very hard to concentrate at work and the anxiety, tension and tearfulness
became apparent. Her sleep was disturbed and her mood fell. She contacted psychiatrists and Sertraline 50 mg
od per os was prescribed. When she started taking the medicine, her mood improved, but remained inadequate.
Afterwards, she lost her appetite, her body mass dropped, fear of the future, anxiety, tension and anhedonia
appeared, work efficiency fell. She could not concentrate and was fatigued by the onset of bad mood and became
tearful. Because of these complaints the patient was treated at a mental hospital using Citalopram 20 mg od per
os and Amitriptyline 75 mg od per os. She was addressed for continuation of treatment to the out- patient clinic
after her condition improved. The patient continued the treatment taking Escitalopram. During the last month the
patient has become irritable and angry again and her night sleep is disturbed. For 2 weeks the patient has been
acting inappropriately(started spending a lot of money on unnecessary things), experiences elevated mood, feels
excited even aggressive occasionally, her speech is chaotic, creates unreal future plans and doesn’t sleep at
night.What is the treatment approach:
A. Out-patient treatment with antidepressants and psychotherapy;
B. In-patient treatment with antipsychotics and psychotherapy;
C. In-patient treatment with antipsychotics, mood stabilizes and psychotherapy;
D. No need of treatment.

34.1 - D 34.2 - A 34.3 - D 34.4 - A 34.5 - B 34.6 - B 34.7 - C 34.8 - B


34.9 - A 34.10 - C 34.11 - B 34.12 - B 34.13 - C 34.14 - B 34.15 - B 34.16 - B
34.17 - D 34.18 - A 34.19 - C 34.20 - D 34.21 - A 34.22 - A 34.23 - D 34.24 - D
34.25 - B 34.26 - B 34.27 - C 34.28 - A 34.29 - B 34.30 - B 34.31 - D 34.32 - B
34.33 - A 34.34 - C 34.35 - B
34.36 34.37 34.38 34.39 34.40 34.41 34.42 34.43
1-B 1-C 1-B 1-B 1-B 1-B 1-C 1-D
2-C 2-B 2-C 2-C 2-A 2-A 2-A 2-C
3-A 3-A 3-A 3-A 3-D 3-C 3-D 3-B
4-C 4-A
34.44 34.45
1-B 1-B
2-C 2-A
3-A 3-C
34.46 34.47 34.48 34.49 34.50 34.51 34.52 34.53
1 1 1 1 4 4 4 2
2 3 2 2 4
3 3 3
4 4
34.54 34.55 34.56 34.57 34.58 34.59 34.60 34.61
1 1 1 1 1 4 2 1
2 3 2 2 2 4 2
3 3 3 3 3
4 4 4
34.62 34.63 34.64 34.65 34.66 34.67 34.68 34.69
4 1 1 1 2 1 4 1
2 2 2 4 2 2
lsmusis.lsmuni.lt/Klausimai/Spausdinti?Length=0?Kalba=EN&KategorijaId=136&Kalbos_input=EN&Kalbos=EN&KategorijaEn_input=Psychiatry&Ka… 12/13
3/27/2019 LSMUSIS
3 3 3 3 3
4 4
34.70 34.71 34.72 34.73 34.74 34.75 34.76 34.77
1 1 1 1 1 4 1 1
3 2 2 2 2 2 2
3 3 3 3 3 3
4 4 4
5
34.78 34.79 34.80 34.81
1 1 1 1
2 3 2 2
3 3 3
34.82 - A 34.83 - B 34.84 - B 34.85 - C 34.86 - D 34.87 - D 34.88 - B 34.89 - D
34.90 - C

lsmusis.lsmuni.lt/Klausimai/Spausdinti?Length=0?Kalba=EN&KategorijaId=136&Kalbos_input=EN&Kalbos=EN&KategorijaEn_input=Psychiatry&Ka… 13/13
3/27/2019 LSMUSIS

Radiology
I type tasks. Choose only one best answer

47.1 Shadows not visible on anterioposterior chest radiograph:


A. Mammary glands
B. Scapula
C. Posterior ends of ribs
D. Sternum
E. Anterior ends of ribs

47.2 Changing of patients‘ position causes alteration of radiographic appearance in case of


A. Lung cancer
B. Bronchitis
C. Pleuritis
D. Bronchiectasis
E. Pneumonija

47.3 Nonhomogeneous opacification of lung field on a radiograph is typical for


A. Pulmonary athelectasis
B. Pleuritis
C. Interlobitis
D. Pulmonary cirrhosis

47.4 Aortic arch is depicted best of all on


A. Left lateral projection radiograph
B. Right lateral projection radiograph
C. Lift oblique projection radiograph
D. Right oblique projection radiograph
E. Anteroposterior projection radiograph

47.5 Highest effective radiation dose delivered to a patient is in case of


A. Chest CT
B. Chest radiograph
C. Fluoroscopy (roentgenoscopy)of intestines
D. fluoroscopy (roentgenoscopy)of stomach
E. Radiography of intestines

47.6 Contrast medium designed for ultrasound investigations


A. Sol.Magnevist
B. Sol. Sonovue
C. Sol. Hypaque
D. Sol. Omnipaque
E. Sol.Ultravist

47.7 Hazardous effect of ionizing radiation is represented by


A. Equivalent dose
B. Absorbed dose
C. Effective dose
D. Equivallent dose
E. All mentioned above

47.8 Lesser roxic (isoosmolar) radiographic contrast medium


A. Sol. Japamiro
B. Sol. Japamiro
C. Sol. Hypaque
D. Sol. Visipaque
E. Sol. Ultravist

47.9 Barium sulphate (BaSO₄) is not suitable for investigation of


A. Gall-bladder
B. Oesophagus

lsmusis.lsmuni.lt/Klausimai/Spausdinti?Length=0?Kalba=EN&KategorijaId=149&Kalbos_input=EN&Kalbos=EN&KategorijaEn_input=Radiology&Kate… 1/11
3/27/2019 LSMUSIS
C. Stomach
D. Small intestine
E. Large intestine

47.10 Teleroentgenography is applied in


A. Investigation of movements of organs
B. Evalution of small details
C. Transfer of images in long distances
D. Measurement of normal dimensions of organs
E. Evaluation of density of organs

47.11 Radiation exposure is not influenced by


A. Kilovoltage of X-ray unit
B. Resistance of inlet of X-ray unit
C. Output capacity of X-ray unit
D. Distance between object and X-ray source

47.12 Syndrome of spread dissemination in chest radiology is not common for


A. Carcinomatosis
B. Focal pneumonia
C. Pulmonary policitosis
D. Hemosyderosis
E. Pneumoconiosis

47.13 First choice method of investigation in case of suspicion of perforated gastric ulcer
A. Radiography with double - conrast
B. Radiography with contrast medium (supine position)
C. Radiography with contrast medium (upright position
D. Conventional tomography
E. Plain radiograph

47.14 Local filling defect of oesophagus radiographically (roentgenologically) is not common for
A. Cardiac achalasia
B. Benign oesophageal tumor
C. Malignang oesophageal tumor
D. Foreign body
E. Oesophageal varicosis

47.15 Endophytic gastric cancer roentenologically is presented with


A. Filling defect of regular shape
B. Filling defect of irregular shape
C. Circular narrowing of gastric lumen
D. Evident convergation of gastric folds towards filling defect
E. Prominent perifocal inflammatory roll

47.16 Perforated gastric ulcer radiographically (roentgenologically) is presented with


A. Evident convergation of gastric folds towards ulcer
B. Free air under the diaphragm
C. Prominent perifocal inflammatory roll
D. Indistinct perifocal inflammatory roll
E. Sing of big niche

47.17 Egsophytic gastric cancer roentenologically is not presented with


A. Disruption of folds
B. Rigidity of folds
C. Absence of folds
D. Convergation of folds
E. Absence of peristalsis

47.18 Prominence of oesophageal contour on contrast radiograph is not typical for


A. Oesophageal ulcer
B. Bening oesophagel tumor

lsmusis.lsmuni.lt/Klausimai/Spausdinti?Length=0?Kalba=EN&KategorijaId=149&Kalbos_input=EN&Kalbos=EN&KategorijaEn_input=Radiology&Kate… 2/11
3/27/2019 LSMUSIS
C. Ulcrated oesophageal cancer
D. Oesophageal diverticulum

47.19 Calcifications in the liver are not detected radiologicaly in case of


A. Alveolococcosis
B. Echinococcosis
C. Abscess of liver
D. Infarction of liver

47.20 Kidneys are evaluated best using


A. Plain radiograph
B. Intravenous urography
C. Abdominal ultrasound
D. Abdominal fluoroscopy (roentgenoscopy)
E. Retrograde pyelography

47.21 Diffuse (widely spread) narrowing of intestinal lumen is common for


A. Endophytic cancer
B. Ulcerative colitis
C. Egsophytic cancer
D. Bowel obstruction
E. Benign tumor of intestines

47.22 Gall-bladder and bile ducts are evaluated best using


A. Ultrasound
B. Radiography with contrast medium
C. Plain radiography
D. Fluoroscopy (roentgenoscopy)
E. Fluorography

47.23 Abdominal organs of pregnant women or investigation of foetus is performed by means of


A. Abdominal and pelvic CT
B. Abdominal and pelvic ultrasound
C. Abdominal and pelvic radiography
D. abdominal and pelvic fluoroscopy (roentgenoscopy)

47.24 It is possible to detect on plain abdominal radiograph


A. Renal calculi
B. Renal cancer
C. Hydronephrosis
D. Renal cist
E. All mentioned above

47.25 Hyperostosis means


A. Increase of bone density
B. Physiological increase of bone size
C. Bone bulging
D. Local loss of bone structure
E. Pathological thickenning of bone

47.26 Not common for bone tuberculosis


A. Affection of joint
B. Linear reaction of periosteum
C. Focuses of destruction
D. Spongiousl sequesters
E. Affection of bone epiphysis

47.27 Radiographic sign not typical for acute osteomyelitis


A. Affection of diaphysis
B. Focuses of destruction in progress
C. Spongious sequesters
D. Linear reaction of periosteum

lsmusis.lsmuni.lt/Klausimai/Spausdinti?Length=0?Kalba=EN&KategorijaId=149&Kalbos_input=EN&Kalbos=EN&KategorijaEn_input=Radiology&Kate… 3/11
3/27/2019 LSMUSIS
E. Patchy osteoporosis

47.28 Pseudoarthrosis of bone may develop in case of


A. Arthritis
B. Arthrosis
C. Osteomyelitis
D. Former fracture
E. Tuberculosis

47.29 Radiographic sign of periosteal reaction of spikes (thin calcified streaks, oriented perpendicular to the
surface of the bone) is typical for
A. Osteoclastic sarcoma
B. Acute osteomyelitis
C. Relapsing osteomyelitis
D. Chronic osteomyelitis
E. Osteoblastic sarcoma

47.30 Increase of bone size with local reduction of bone tissue is typical for
A. Hypertrophy
B. Osteolysis
C. Hyperostosis
D. Osteoporosis
E. Bone bulging

II type tasks. For each numbered item,selct the one lettered heading that is most closely asssciated with it

47.31 Appearance of a stomach air bubble in patients of different constitutional types


1 - 50.43. Air bubble of a half circle shape
2 - 50.44. Low and flat air bubble, with plane dome
3 - 50.45. Loss of air5 bubble
4 - 50.46. Air bubble of a half vertical ovoid shape
A. Various constitutional type, recumbent position
B. Asthenic
C. Normosthenic
D. Hypersthenic

47.32 Which radiographic alteration of mucosa is typicalmfor diseases mentioned below


1 - Multiple radiotransparent spots
2 - Loss of folds in the area of inflammatory roll
3 - Thickening of mucosal folds
4 - Disruption of folds in the tumoral area
A. Ulcer
B. Inflammation
C. Cancer
D. Oesophageal varices

47.33 Radiographical (roentgenological) alterations of lung pattern, typical for diseases mentioned below
1 - 50.55. Enlarged elements of lung pattern
2 - 50.56. Diminished elements of lung pattern
3 - 50.57. Deformed elements of lung pattern
4 - 50.58. Aberrant elementys of lung pattern
A. Congenital stenosis of a. pulmonalis
B. Pleural adhesions
C. Bronchitis
D. Congenital vitia cordis

47.34 Which type of contoours of round shape shadows of lung field are common for lung diseases mentioned
below
1 - 50.47. Clear and slightly uneven contour
2 - 50.48. Unclear contour
3 - 50.49. Clear and smooth contour
4 - 50.50. Clear and rugged contour

lsmusis.lsmuni.lt/Klausimai/Spausdinti?Length=0?Kalba=EN&KategorijaId=149&Kalbos_input=EN&Kalbos=EN&KategorijaEn_input=Radiology&Kate… 4/11
3/27/2019 LSMUSIS
A. Acute pneumonia
B. Cyst
C. Cancer
D. Tuberculoma

47.35 Which methods of radiographical (roentgenological) investigation are performed in case of below mentioned
diseases
1 - 50.39. Plain abdominal radiograph
2 - 50.40. Double-contrast radiographic investigation of large bowel
3 - 50.41. Polygraphy
4 - 50.42. Endoscopic retrograde chalonagiopancreatography (ERCP)
A. Gallstones of biliary ducts
B. Acute bowel obstruction
C. Cancer ofmlarge bowel
D. Rigidity of gastric walls due to a tumor

47.36 Radiographically (roentgenologically) detectable enlargement of heart chambers, typical for diseases
mentioned below
1 - 50.63. Enlargement of left ventricle
2 - 50.64. Enlargement of right ventricle
3 - 50.65. Enlargement of left atrium
4 - 50.66. Enlargement of ringht atrium
A. Chronic pulmonary diseases
B. Aortic stenosis
C. Isolated tricuspidal stenosis
D. Mitral insufficiency

47.37 Radiographical (roentgenological) signs common for types of gastric ulcer, mentioned below
1 - 50.87. Small, triangle niche
2 - 50.88. Big, quadriangle niche
3 - 50.89. Big, niche with layers of different substances (contrast medium-gastric juice-air)
4 - 50.90. Air beneath the diaphragm
A. Perforated ulcer
B. Acute ulcer
C. Chronic ulcer
D. Chronic penetrating ulcer

47.38 Which radiographic signs are typical for diseases mentioned below
1 - Niche
2 - Tube-shaped stomach
3 - Loss of of haustral markings
4 - Suprastenotic dilatation of lumen
A. Endophytic gastric cancer
B. Ulcerative colitis
C. Ulcer
D. Acid or alkali burn

47.40 Alteration of bone structure, typical for diseses mentioned below


1 - 50.75. Osteoporosis
2 - 50.76 Osteonecrosis
3 - 50.78. Osteolysis
4 - 50.78. Fracture line
A. Osteochondropathy
B. Reino disease
C. Trauma
D. In elderly and exhaustion

47.42 Type of reaction of periosteum, common for diseases, mentioned below


1 - 50.67 Linear
2 - 50.68. Laminar
3 - 50.69. Rugged
4 - 50.70. Spikes
lsmusis.lsmuni.lt/Klausimai/Spausdinti?Length=0?Kalba=EN&KategorijaId=149&Kalbos_input=EN&Kalbos=EN&KategorijaEn_input=Radiology&Kate… 5/11
3/27/2019 LSMUSIS
A. Osteoblastic sarcoma
B. Acute osteomyelitis
C. Reapsing osteomyelitis
D. Osteomyelitis in progress

47.43 Descriptions of standard projections of cardiac radiographical (roentgenological) examinations


1 - 50.59. Patent with his/her right shoulder directed with 45° angle to the screen
2 - 50.60. Patient with his/her back directed to the X-ray tube and his anterior chest wall atached to the
screen/or cassette
3 - 50.61. Patient with his/her left shoulder directed with 45° angle to the screen
4 - 50.62. Patient with his/her left flank directed to the screen
A. Right oblique (first oblique)
B. Left oblique (second oblique)
C. Anteroposterior
D. Left lateral

47.44 Which radiographical (roentgenological) syndromes are typical for lung diseases mentioned below
1 - 50.51. Round-shaped shadow of lung field
2 - 50.52. Limited/local opacification of lung field
3 - 50.53. Spread dissemination
4 - 50.54. Ring shape shadow of lung field
A. Free pleural fluid
B. Encapsulated pleural fluid
C. Pulmonary abscess
D. Acute hematogenic disseminated tuberkulosis

47.45 Typical location of bone destruction, common for diseases mentioned below
1 - 50.79. Metaphysis with spread into diaphysis
2 - 50.80. Short spongious bone
3 - 50.81. Cortical layer of diaphysis
4 - 50.82. Destruction of entire bone mass
A. Syphilis
B. Nonspecific osteomyelitis
C. Tuberculosis
D. Malignant tumours

III type tasks. For each question there is one or more correct answers:
A – if correct answers are 1,2,3
B – if correct answers are 1 and 3
C – if correct answers are 2 and 4
D – if correct answer is 4
E – if correct are all answers above

47.46 Methods of radiological investigations operated with ionizing radiation


1. Fluoroscopy
2. CT
3. Fluoroscopy (roentgenoscopy)
4. Ultrasound
5. MRI

47.47 Negative contrast media


1. Not absorbing X-rays
2. water-soluble, containing iodine
3. Air, CO₂N₂O, and other gasses
4. Water-insoluble, containing barium sulphate
5. Paramagnetic and superparamagnetic

47.48 Interventional procedures performed under ultrasound control


1. Abscess drainage
2. Pancreatic duct drainage
3. Ascites aspiration

lsmusis.lsmuni.lt/Klausimai/Spausdinti?Length=0?Kalba=EN&KategorijaId=149&Kalbos_input=EN&Kalbos=EN&KategorijaEn_input=Radiology&Kate… 6/11
3/27/2019 LSMUSIS
4. Transcutaneous nephrostomy
5. Biopsy of focuses of prostatic

47.49 Possible main side effects after intravenous injection of iodine containing contrast medium
1. Alergic reactions
2. Haematological reactions
3. Nephrotoxic reactions
4. Neurotoxic reactions

47.50 Layers and walls of uterus and ovaries are evaluated best applying
1. Pelvic ultrasound examination
2. Pelvic plain radiography
3. Pelvic MRI
4. Pelvic CT
5. Hysterosalpyngography

47.51 Ring-shape shadow of lung field on chest plain radiographs is common for
1. Tuberculous cavern
2. Pulmonary abscess
3. Periferic lung cancer with destruction
4. Pleuritis
5. Diaphragmatic hernia

47.52 Spread of cancer of urinary bladder of obese patient is best depicted by means of
1. Pelvic radiography
2. Pelvic MRI
3. Pelvic ultrasound
4. Pelvic CT

47.53 Normal heart position in the chest radiograph of an adult


1. 2/3 of heart shadow on the left, l/3 - on the right side of thorax
2. 2/3 of heart shadow on the right, 1/3 - on the left side of thorax
3. Oblique, vertical or horizontal heart position depending upon constitutional type
4. Irregular ball-shaped heart

47.54 Radiographical (roentgenological) signs of enlarged left atrium


1. Elongated and expressed 2nd heart shadow (knob) on the left side on anteroposterior projection
2. Elongated and expressed 3rd heart shadow (knob) on the left side on aneroposterior projection
3. Wall of left atrium prominating towards spinal column,easily visible after application of barium meal in
oesophagus on the left oblique projection
4. Wall of right atrium prominating towards spinal column,easily visible aftear application of barium meal in
oesophagus on the right oblique projection

47.55 Human bone age is approximately detected by means of


1. A radiograph of all skeleton
2. A skull radiograph
3. A radiograph of long bones
4. A radiograph of wrist and hand

47.56 Not common for osteoporosis


1. Bone tissue loss per unit of volume
2. Normal bone elements (fat,bone marrow, blood) appear in place of vanishing trabeculas
3. Complete loss of bone structure, sparing only sharp bone contours
4. Pathological tissue appearing in place of gradually decaying bone

47.57 Upper part of gastrointestinal tract is composed of


1. Pharynx, oesophagus
2. Stomach
3. Duadenum
4. Jejunum

47.58 Radiographic sign of "pointing finger" of stomach is typical for


1. Small plate-shape gastric cancer
lsmusis.lsmuni.lt/Klausimai/Spausdinti?Length=0?Kalba=EN&KategorijaId=149&Kalbos_input=EN&Kalbos=EN&KategorijaEn_input=Radiology&Kate… 7/11
3/27/2019 LSMUSIS
2. Ulcerating gastric cancer
3. Polypoid or fungating type of gastric cancer
4. Gastric ulcer

47.59 Location of Meckel"s diverticulum


1. Oesophagus
2. Duodenum
3. Jejunum
4. Ileum
5. Caecum

47.60 Bovel obstruction radiographicaly is represented by


1. Wide loops of bowels
2. Kloiber"s cups
3. Arches of bowels
4. Horizontal air-fluid surface in the bowel lumen

IV type tasks. Choose only one best answer

47.61 54 year old female, complaining of dyspnea. Long-time smoker, several, times was treated because of
inflammation of veins of the right leg. Plain chest radiograph (erect position): synuses are free. On the left side -
doom of diaphragm is at the anterior ending of 5th rib; on the left lung area - lung pattern is diminished, left hilus is
widened,heart shadow widened to the right as well,heart waist is flattened. Mediastinum is widened on the right.
Suspected pathology:
A. Lung cancer
B. Pulmonary embolism
C. Myocarditis
D. Pneumonia

47.62 27 years old male, complaining of pain in epigastrium, originating after; also indicates waterbrash.
Radiograph of stomach with barium meal: free fluid in an empty stomach,mucosal folds are thickened, in the lower
third of corpus of stomach folds are convergating towards lesser curve where accumulation of contrast medium
("niche") is evident, extending contour line of stomach. Suspeccted pathology.
A. Gastric pathology
B. Gastritis
C. Gastric diverticulum
D. Gastric ulcer

47.63 48 years old female complaning of pain in the knee joints, especially the right one, and decreased flexibility
which is particularly pronounced in the morining. Upon inspection: knee joints: not deformed, crepitation is evident
at flexion-extension. Plain radiograph of knee joints: both articular cavities are unevenly narrowed, subchondral
bone is osteosclerotic, trochanters of intercondyloid eminence of tibias are sharpened, osteophytes on the edges
of articular surfaces. Suspected pathology.
A. Deforming osteoarthrosis
B. Rheumatoid artritis
C. Dissecting osteochondritis
D. Chondromatosis of joints

47.64 66 year old male, long-time smoker, often taking alkoholic beverages. A week ago was ill with flu, now
complaining of hacking cough, shortness of breath increasing during physical excercise; also of faintness and
subfebrility. Plain chest radiograph: pleural synuses free, dooms of diaphragm smooth, lung pattern increased due
to interstitial and peribronchal elements; on the ritht side, inferior lobe (S8-S10) - multiple small focuses, right hilus
widend, of altered structure. Width of heart shadow increased to the left side, wais expressed; aorta diffusely
broadened and intense. Suspected pathology:
A. Focal tuberculosis of the right lung
B. Pneumonia of the right lung
C. Pulmonary embolism
D. Central tumor of the right lung

47.65 31 year old female complaining of general fatique, loss of appetite and dyspepsia. Blood sample revealed
anemia. Radiographic investigation of stomach with barium meal: stomach is shortened, lesser and major curves

lsmusis.lsmuni.lt/Klausimai/Spausdinti?Length=0?Kalba=EN&KategorijaId=149&Kalbos_input=EN&Kalbos=EN&KategorijaEn_input=Radiology&Kate… 8/11
3/27/2019 LSMUSIS
seem evened, shape of stomach resembles "horn", mucosal folds nearly absent, motility is very week, apparent
only in distal part of major curve. Suspected pathology:
A. Gastritis
B. Sclerodermia
C. Gastric ulcer
D. Gastric cancer

47.66 13 year old girl complaining of lowe backpain, general fatigue, subfebrility. Variable backpain, manifesting
with movements, was presented for 3-4 months already. Subfebrility was apparent several times, as well.
Anteroposterior and laateral radiographs of thoracic spine height of Th10 and Th11 vertebra is decreased, bone
structure of vertebral bodies is porotic, articular plates disrupted, with focuses of destruction below. Intervertebral
space spared. In paravertebral region of affected - shadows of thickened soft tissues are present. Suspected
pathology:
A. Tuberculous spondilitis
B. Juvenile osteochondropathy
C. Nonspecific spondilodiscitis
D. Metastases

47.67 34year old female complaining of nausea, vomiting, restraint of gas and stool. A year ago she passed
surgery due to perforated appendicitis. Abdominal plain radiograph: dilated with gas loops of small bowels and
Kloiber"s cups,no gas in large bowel. Suspected pathology:
A. Perforation of large bowel
B. High obstruction of small bowel
C. Low obstruction of small bowel
D. Large bowel obstruction

47.68 59 year old male, fond of spicy food, complains swallowing of thick meal, that is present for one month
already. Radiographic investigation of oesophagus with barium meal:contrast medium passess oesophagus freely;
in the lower third of thoracic part of oesophagus, in a segment of about 3 cm length above the diaphragm the
lumen is slightly narrowed, walls are uneved, rigid, mucosal folds absent,while in the lumen - irregular shape filling
defects of different size. Suspeced pathology:
A. Reflux esophagitis
B. Esophageal varices
C. cancer of oesophagus
D. Achalasia

47.69 53 year old female complaining of stool restraint,occasional looseness,spastic abdominal pain. Expreriences
constant stress. Double-contrast barium enema (DCBE) radiographic investigation: evident segmental haustral
spasms, evacuation incomplete, mucosal,folds normal, during double contrast phase -walls are destensible, no
additional structures in the lumen.Suspeced pathology
A. Diverticulosis
B. Ulcerative colitis
C. Functional alterations of large bowel
D. Crohn"s disease

47.70 Normal 55 year old male, at prophylactic inspection. Occasional findings on plain chest radiograph: incresed
lung pattern, singns of 1 st degree venous stasis. In the middle field of left lung - round about 1.4 cm opacity with
irregular contours, broadened left hilum, with possibe enlarged lymphnodes. Calcified bifurcational and
paratracheal lymph nodes. On the last prophylactic chest radiograph 2 years ago these finding were not present.
Suspected pathology:
A. Abscess
B. Complicated pulmonary cyst
C. Pulmonary infarction
D. Peripheric lung cancer

47.71 34 year old female complaining of pain in metacarpophalangeal and interphalangeal articulations with
swelling, morning numbness lasting longer than an hour. Anteroposterior radiograph of both hands: porosis of
metacarpal and proximal-medial phalangeal epiphyses.Soft tissues around the joints symmetricaly swelled,
several small subchondral cysts and usurations. Suspected pathology:
A. Reumatoid artritis
B. Deforming osteoarthrosis
C. General osteoporosis
lsmusis.lsmuni.lt/Klausimai/Spausdinti?Length=0?Kalba=EN&KategorijaId=149&Kalbos_input=EN&Kalbos=EN&KategorijaEn_input=Radiology&Kate… 9/11
3/27/2019 LSMUSIS
D. Gout

47.72 61 year old male complaining of diffcult defecation, with feeling of incompletion. For 3 monts he spots some
frech bood in the feces. Double-contrast barium enema (DCBE)radiographic investigation: in the middle part of
rectal ampulla deformation and narrowing of lumen of about. 5 cm length; irregular accumulation of deformation.
Large bowel is dilated above the deformation, with much concents in the lumen. Suspected pathology:
A. Crohn"s disease
B. Adhesions
C. Ulcerated cancer
D. Divertuculitis

47.73 18 year old male complaining of pain of left knee joint, especialy intense at night. A month ago he
experienced a mild trauma to the joit. Clinical investigation: in the distal part of hip, dorsal part - palpable induration
of unclear contours, inert and painless. Anteroposterior and lateral radiographs of left knee joint: articular cavity is
intact; in the femoral distal metaepiphysis - irregular bony mass with unclear contours, invading posterior cortical
layer of the femoral bone; in the dorsal side of the femoral bone - spike - form periosteal reaction, perifocal soft
tissue shadow is also intense. Suspected pathology:
A. Malignant bone tumor
B. Bening bone tumor
C. Chronic osteomyelitis
D. Healing fracture

47.74 38 year old female complaining of sharp abdonimal pain, slightly to the right of umbilicus. Two hours ago
she experienced more intense pain on the back. Vomited once, urinated with reddish urine.Plain abdominal
radioograph: close to the right transvers process of L3 vertebra - irregular ovoid, with smooth contours,
radioopaque (intensity of bone), homogeneous structure, about 6 mm in diameter. Suspected pathology:
A. Calculus in the common bile duct
B. Calcylus in the bowels
C. Calculus in the ureter
D. Phlebolithon

47.75 56 year old femele complains of hemoptysis. Blood sample is normal. Plain chest radiograph: lung pattern
appears normal; left hilum tumorously widened, slightly broadened shadow of upper mediastinum on the left as
well. Suspected pathology:
A. Pulmonary embolismtromboembolia of pulmonary artery
B. Tuberculosis
C. Sarcoidosis
D. Central tumor of left lung

47.76 45 Year old male complains of very sharp and sudden onset (as "stabbing With knife") of abdominal pain.
For 8 years already ill with gastric ulcer. Plain abdominal radiograph (erect position): under the right doom of
diaphragm, above the surface of liver - radiotransparent stripe of about 7 mm, on the left side - gastric air bubble
abssent, large bowel dilataed with gass. Suspected pathology:
A. High obstruction of small bowel
B. Large bowel obstruction
C. Perforation of hollow organ of gastrointenstinal tract
D. Subdiaphragmatic abscess

47.1 - D 47.2 - C 47.3 - D 47.4 - A 47.5 - A 47.6 - B 47.7 - C 47.8 - D


47.9 - A 47.10 - D 47.11 - B 47.12 - C 47.13 - E 47.14 - A 47.15 - C 47.16 - B
47.17 - D 47.18 - B 47.19 - D 47.20 - C 47.21 - B 47.22 - A 47.23 - B 47.24 - A
47.25 - E 47.26 - B 47.27 - C 47.28 - D 47.29 - E 47.30 - E
47.31 47.32 47.33 47.34 47.35 47.36 47.37 47.38
1-C 1-D 1-D 1-D 1-B 1-B 1-B 1-C
2-D 2-A 2-A 2-A 2-C 2-A 2-C 2-A
3-A 3-B 3-C 3-B 3-D 3-D 3-D 3-B
4-B 4-C 4-B 4-C 4-A 4-C 4-A 4-D
47.40 47.42 47.43 47.44 47.45
1-D 1-B 1-A 1-B 1-B
2-A 2-C 2-C 2-A 2-C
3-B 3-D 3-B 3-D 3-A
lsmusis.lsmuni.lt/Klausimai/Spausdinti?Length=0?Kalba=EN&KategorijaId=149&Kalbos_input=EN&Kalbos=EN&KategorijaEn_input=Radiology&Kat… 10/11
3/27/2019 LSMUSIS
4-C 4-A 4-D 4-C 4-D
47.46 47.47 47.48 47.49 47.50 47.51 47.52 47.53
1 1 1 1 1 1 2 1
2 3 2 2 3 2 4 3
3 3 3 3
4
5
47.54 47.55 47.56 47.57 47.58 47.59 47.60
2 4 4 1 4 4 1
4 2 2
3 3
4
47.61 - B 47.62 - D 47.63 - A 47.64 - B 47.65 - D 47.66 - A 47.67 - C 47.68 - C
47.69 - C 47.70 - D 47.71 - A 47.72 - C 47.73 - A 47.74 - C 47.75 - D 47.76 - C

lsmusis.lsmuni.lt/Klausimai/Spausdinti?Length=0?Kalba=EN&KategorijaId=149&Kalbos_input=EN&Kalbos=EN&KategorijaEn_input=Radiology&Kat… 11/11
3/27/2019 LSMUSIS

IV type tasks. Choose only one best answer

51.1 57 years old man complains of restriction of full extension of 4th and 5th fingers of the right hand. In the palm
of the hand the two oblong painful formations that have grown to the skin, can be palpated. Movements of 4th
finger's joints are limited about 50% and the 5th finger's - about 80%. Patient have been ill about 10 years. Which
a diagnosis is correct?

A. Osteoarthritis of 4th and 5th fingers


B. Dupuytren's contracture
C. Rheumatic arthritis
D. Scars after injury

51.2 57 years old man complains of restriction of full extension of 4th and 5th fingers of the right hand. In the palm
of the hand the two oblong painful formations that have grown to the skin, can be palpated. Movements of 4th
finger's joints are limited about 50% and the 5th finger’s - about 80%. Patient have been ill about 10 years. What
is the right tactics of the treatment?

lsmusis.lsmuni.lt/Klausimai/Spausdinti?Length=0?Kalba=EN&KategorijaId=156&Kalbos_input=EN&Kalbos=EN&KategorijaEn_input=Orthopedy&Kat… 1/41
3/27/2019 LSMUSIS

A. Arthrodesis of 4th and 5th fingers' joints


B. Arfhroplasty of 4th and 5th fingers' joints
C. Replacement of 4th and 5th fingers' joints (with prostheses)
D. Excisioii of the palmar aponeurosis

51.3 38 years old man complains of severe pain in lumbar spine. Pain tires as in standing, as during the night.
Patient is unable to do standing work. The movements of lumbar spine are limited and painful. One has been ill
about 15 years and illness progresses all the time. Conservative treatment is uneffective. Which a diagnosis is
correct?

A. Tuberculosis of the lumbar vertebrae.


lsmusis.lsmuni.lt/Klausimai/Spausdinti?Length=0?Kalba=EN&KategorijaId=156&Kalbos_input=EN&Kalbos=EN&KategorijaEn_input=Orthopedy&Kat… 2/41
3/27/2019 LSMUSIS
B. Compression fracture of the lumbar vertebra's body.
C. Injury of the intervertebral disks.
D. Spondylolisthesis

51.4 38 years old man complains of severe pain in lumbar spine. Pain tires as in standing, as during the night.
Patient is unable to do standing work. The movements of lumbar spine are limited and painful. One has been ill
about 15 years and illness progresses all the time. Conservative treatment is uneffective. What is the right tactics
of the treatment?

A. Treatment with antituberculosis drugs


B. Spondylodesis
C. Qrthopaedic brace support
D. Conservative treatment in a sanatorium

51.5 18 years old man complains of deformity of both the legs. He has been ill about 8 years and illness had been
progressed. There are deformities of both the knees; distance between medial condyles of both femurs is 8 cm.
Which a diagnosis is correct?

lsmusis.lsmuni.lt/Klausimai/Spausdinti?Length=0?Kalba=EN&KategorijaId=156&Kalbos_input=EN&Kalbos=EN&KategorijaEn_input=Orthopedy&Kat… 3/41
3/27/2019 LSMUSIS

A. Valgus deformity of the knees.


B. Varus deformity of the knees.
C. Recurvatus knees
D. Legg-Perthes disease

51.6 18 years old man complains of deformity of both the legs. He has been ill about 8 years and illness had been
progressed. There are deformities of both the knees; distance between medial condyles of both femurs is 8 cm.
What is thee right tactics of the treatment?

lsmusis.lsmuni.lt/Klausimai/Spausdinti?Length=0?Kalba=EN&KategorijaId=156&Kalbos_input=EN&Kalbos=EN&KategorijaEn_input=Orthopedy&Kat… 4/41
3/27/2019 LSMUSIS

A. Axthrodesis of both the knees.


B. Corrective osteotomy of lower ends of both the femurs.
C. Total both the knees replacement.

51.7 33 years old woman fell down from 6 meters of height, 2 years ago. Doctor had diagnosed the fracture of the
calcaneus with marked displacement of fragments (Bohler's angle of 5 degrees). Foot had been immobilized with
plaster bandage. The fracture bad healed, but Borhler’s angle remained of the same size. Pain had appeared in
the foot soon after the plaster cast had been removed. Now the patient feels the severe pain in the heeland finds
it difficult to walk, so he Is limping. The tenderness in me subtalar joint and longitudinal flatfoot is found upon
clinical examination. Which a diagnosis is correct?

A. Haglund disease.
B. Osteoarthrosis of the talocalcaneal joint.
C. Nonunion of the calcaneus.
D. Avascular necrosis of the talus.

51.8 33 years old woman fell down from 6 meters of height, 2 years ago. Doctor had diagnosed the fracture of the
calcaneus with marked displacement of fragments (Bohler's angle of 5 degrees). Foot had been immobilized with
plaster bandage. The fracture bad healed, but Borhler’s angle remained of the same size. Pain had appeared in
the foot soon after the plaster cast had been removed. Now the patient feels the severe pain in the heeland finds

lsmusis.lsmuni.lt/Klausimai/Spausdinti?Length=0?Kalba=EN&KategorijaId=156&Kalbos_input=EN&Kalbos=EN&KategorijaEn_input=Orthopedy&Kat… 5/41
3/27/2019 LSMUSIS
it difficult to walk, so he Is limping. The tenderness in me subtalar joint and longitudinal flatfoot is found upon
clinical examination. What is the right tactics of the treatment?

A. Arfhrodesis of the subtalar joint.


B. Corrective osteotomy of the tibia.
C. Replacement of the ankle (talocrural) joint
D. Arfhrodesis of the ankle joint

51.9 32 years old man complains of severe pain in the right ankle joint and of his disability to standing work. He
walks with a stick and sometimes - on crutches. He has been ill about 3 years. The closed reduction and
immobilization in plaster bandage were performed due to the fracture of medial malleolus accompanied by
intraarticular fracture of talus. The pain began to increase after me 4 months when me plaster bandage had been
removed. On clinical examination, the movements of the right ankle are limited (dorsal flexion - 5 degrees, plantar
flexion - 3 degrees), me atrophy of the muscles of the right leg - 3 cm. The x-ray findings are following: severe
joint space narrowing, marked osteophyte formation at the joint margins, dense sclerosis of subchondral bone,
and subchondral cysts. Which diagnosis is correct?

A. Tuberculosis of the ankle joint


B. Pseudarthrosis of the medial malleolus
lsmusis.lsmuni.lt/Klausimai/Spausdinti?Length=0?Kalba=EN&KategorijaId=156&Kalbos_input=EN&Kalbos=EN&KategorijaEn_input=Orthopedy&Kat… 6/41
3/27/2019 LSMUSIS
C. Pseudoarthrosis of the lateral malleolus
D. Ostearthrosis of the ankle joint

51.10 32 years old man complains of severe pain in the right ankle joint and of his disability to standing work. He
walks with a stick and sometimes - on crutches. He has been ill about 3 years. The closed reduction and
immobilization in plaster bandage were performed due to the fracture of medial malleolus accompanied by
intraarticular fracture of talus. The pain began to increase after me 4 months when me plaster bandage had been
removed. On clinical examination, the movements of the right ankle are limited (dorsal flexion - 5 degrees, plantar
flexion - 3 degrees), me atrophy of the muscles of the right leg - 3 cm. The x-ray findings are following: severe
joint space narrowing, marked osteophyte formation at the joint margins, dense sclerosis of subchondral bone,
and subchondral cysts. What is the right tactics of the treatment?

A. Osteotomy of the distal end of the tibia


B. Arthrodesis of the ankle joint
C. Arthrodesis of the subtalar joint
D. Scraping off the bone cavities and its filling by biocompatible osteoconductive polimers (BOP) or bone
graft

51.11 62 years old woman complains of lateral deviation of the great toe and also severe pain in the first
metatarsophalangeal joint. Pain usually increases on walking. Patient uses. the footwear of adequate size and.
appropriate shape, but me pain have been progressed. Physical therapy wasn’t effective. The movements of the
firstmetatarsophalangeal joint are limited (amplitude of motion is about 10 degrees) and painful. Patient has been
ill about 30 years. Which a diagnosis is correct?

lsmusis.lsmuni.lt/Klausimai/Spausdinti?Length=0?Kalba=EN&KategorijaId=156&Kalbos_input=EN&Kalbos=EN&KategorijaEn_input=Orthopedy&Kat… 7/41
3/27/2019 LSMUSIS

A. Haglund disease.
B. Hammered (malleus) deformity
C. Osteoblastochondroma of distal end of the Istmetatarsal.
D. Osteoarthrosis of the first metatarsophalangeal joint.

51.12 62 years old woman complains of lateral deviation of the great toe and also severe pain in the first
metatarsophalangeal joint. Pain usually increases on walking. Patient uses. the footwear of adequate size and.
appropriate shape, but me pain have been progressed. Physical therapy wasn’t effective. The movements of the
firstmetatarsophalangeal joint are limited (amplitude of motion is about 10 degrees) and painful. Patient has been
ill about 30 years. What is the right tactics of the treatment?

lsmusis.lsmuni.lt/Klausimai/Spausdinti?Length=0?Kalba=EN&KategorijaId=156&Kalbos_input=EN&Kalbos=EN&KategorijaEn_input=Orthopedy&Kat… 8/41
3/27/2019 LSMUSIS

A. Biopsy of the bone tumor.


B. Arthroplasty (prosthesis) of the first metatarsophalangeal joint.
C. Immobilization of the foot with plaster splint.
D. Conservative treatment in a sanatorium.

51.13 18 aged woman complains of deformity of 2nd right metacarpophalangeal joint and constant pain sensation
in the past 3 years. Objective symptoms: deformity of the joint, painful palpation, walk with a limp on right foot. X-
ray film show flattening, irregularity and multiple exostoses of articular surface of the 2nd metatarsal head. 18
aged woman complains of deformity of 2nd right metacarpophalangeal joint and constant pain sensation in the
past 3 years. Objective symptoms: deformity of the joint, painful palpation, walk with a limp on right foot. X-ray film
show flattening, irregularity and multiple exostoses of articular surface of the 2nd metatarsal head. Which
diagnosis is correct?

lsmusis.lsmuni.lt/Klausimai/Spausdinti?Length=0?Kalba=EN&KategorijaId=156&Kalbos_input=EN&Kalbos=EN&KategorijaEn_input=Orthopedy&Kat… 9/41
3/27/2019 LSMUSIS

A. Kohler’s Il disease
B. Tbc of the 2nd metacarpal bone
C. Hallux valgus
D. Hallux rigidus

51.14 18 aged woman complains of deformity of 2nd right metacarpophalangeal joint and constant pain sensation
in the past 3 years. Objective symptoms: deformity of the joint, painful palpation, walk with a limp on right foot. X-
ray film show flattening, irregularity and multiple exostoses of articular surface of the 2nd metatarsal head. What
is a right tactic of the treatment?

lsmusis.lsmuni.lt/Klausimai/Spausdinti?Length=0?Kalba=EN&KategorijaId=156&Kalbos_input=EN&Kalbos=EN&KategorijaEn_input=Orthopedy&Ka… 10/41
3/27/2019 LSMUSIS

A. Arthrodesis of the metatarsophalangeal joint


B. Osteotomy of the 2nd metatarsal
C. Immobilization by plaster cast
D. Arthroplasty of metatarsophalangeal joint

51.15 59-years - old woman complains of angular deformity of great toe, breadthening of the foot and bunion
pain. Duration of disease -20 years. Objective findings: distal part of the foot is breadthened, great toe adducted,
1-st metatarsal - abducted. Which a diagnosis is correct?

lsmusis.lsmuni.lt/Klausimai/Spausdinti?Length=0?Kalba=EN&KategorijaId=156&Kalbos_input=EN&Kalbos=EN&KategorijaEn_input=Orthopedy&Ka… 11/41
3/27/2019 LSMUSIS

A. Hallux valgus
B. Haglund's disease
C. Digitus maleus
D. Pes equinovarus

51.16 59-years - old woman complains of angular deformity of great toe, breadthening of the foot and bunion
pain. Duration of disease -20 years. Objective findings: distal part of the foot is breadthened, great toe adducted,
1-st metatarsal - abducted. What is a right tactics of the treatment?

A. Physiotherapeutic treatment
B. Skeletal traction of the great toe
C. Immobilization of the foot by plaster cast
lsmusis.lsmuni.lt/Klausimai/Spausdinti?Length=0?Kalba=EN&KategorijaId=156&Kalbos_input=EN&Kalbos=EN&KategorijaEn_input=Orthopedy&Ka… 12/41
3/27/2019 LSMUSIS
D. Corrective osteotomy of metatarsal bone

51.17 57 aged woman complains of deformity of, the 4st toe, pain sensation, increasing on walk. Duration of
disease -13 years. Objective examination: 4st toe flexed on 45° angle in metatarsophalangeal joint and on 90° in
interphalangeal joint. Limited range of motion (10°) of those joints. Corns in the region of metatarsal head and
distal end of proximal phalanx. Which a diagnosis is correct?

A. Hallux valgus
B. Kohler’s disease
C. Digitus maleus
D. Hallux rigidus

51.18 57 aged woman complains of deformity of the 4th toe, pain sensation, increasing on walk. Duration of
disease -13 years. Objective examination: 4st toe flexed on 45° angle in metatarsophalangeal joint and on 90° in
interphalangeal joint. Limited range of motion (10°) of those joints. Corns in the region of metatarsal head and
distal end of proximal phalanx. what is a right tactics of the treatment?

A. Plaster cast
B. Physiotherapeutic treatment
C. Skeletal traction
D. Surgical treatment (Partial resection of the phalanx

51.19 64 aged woman complains of sharp pain in the right knee joint disability, Objective symptoms: Right knee
motions painful and markedly limited: flexion 1O°, extension - 0°. Muscle atrophy of the right thigh. X-ray film
show knee joint line narrowed, sclerosis of femoral and tibial condyles, marginal exostosis on both sides of
articular surfaces. Which a diagnosis is correct?

lsmusis.lsmuni.lt/Klausimai/Spausdinti?Length=0?Kalba=EN&KategorijaId=156&Kalbos_input=EN&Kalbos=EN&KategorijaEn_input=Orthopedy&Ka… 13/41
3/27/2019 LSMUSIS

A. Osteochondropathy of the femur


B. Garre's osteomyelitis
C. Osteoarthritis deformans of the knee joint
D. Osgood - Schlatter disease

51.20 64 aged woman complains of sharp pain in the right knee joint disability, Objective symptoms: Right knee
motions painful and markedly limited: flexion 1O°, extension - 0°. Muscle atrophy of the right thigh. X-ray film
show knee joint line narrowed, sclerosis of femoral and tibial condyles, marginal exostosis on both sides of
articular surfaces. What is a right tactics of the treatment?

lsmusis.lsmuni.lt/Klausimai/Spausdinti?Length=0?Kalba=EN&KategorijaId=156&Kalbos_input=EN&Kalbos=EN&KategorijaEn_input=Orthopedy&Ka… 14/41
3/27/2019 LSMUSIS

A. Total knee joint replacement


B. Sequestrectomy
C. Scraping and autoplasty of the cysts
D. Lateroposition of the patella

51.21 58- years - old man complains of intensive pain in the hip joint during last 15 years. Walks with one crutch.
Rang of motion of the knee joint, abduction — 0’adduction - 100 flexion - l5, extension 0°, internal rotation - 06,
external rotation - 0° X-ray film: hip joint line markedly narrowed, joint surface irregular, sclerosis and
degenerative cysts of acetabulum and femoral bead. Which a diagnosis is correct?

lsmusis.lsmuni.lt/Klausimai/Spausdinti?Length=0?Kalba=EN&KategorijaId=156&Kalbos_input=EN&Kalbos=EN&KategorijaEn_input=Orthopedy&Ka… 15/41
3/27/2019 LSMUSIS
A. Tbc of the femoral head
B. Coxarthrosis deformans
C. Perthes’ disease
D. Slipping of the capital femoral epiphysis

51.22 58- years - old man complains of intensive pain in the hip joint during last 15 years. Walks with one crutch.
Rang of motion of the knee joint, abduction — 0’adduction - 100 flexion - l5, extension 0°, internal rotation - 06,
external rotation - 0° X-ray film: hip joint line markedly narrowed, joint surface irregular, sclerosis and
degenerative cysts of acetabulum and femoral bead. What is a right tactics of the treatment?

A. Total hip joint replacement


B. Scraping of the degenerative cysts
C. Formation of the acetabular roof
D. Plaster immobilization

51.23 25 aged woman complains of painful foot, especially in walking. Objective findings: swelling and
inflammation in the region of insertion of Achilles tendon. Local heat and erythema in the region of heel. Which a
diagnosis is correct?

A. Transverse flatfoot
B. Digitus maleus
C. Haglund’s disease
D. Hallux valgus

51.24 25 aged woman complains of painful foot, especially in walking. Objective findings: swelling and
inflammation in the region of insertion of Achilles tendon. Local heat and erythema in the region of heel. What is a
lsmusis.lsmuni.lt/Klausimai/Spausdinti?Length=0?Kalba=EN&KategorijaId=156&Kalbos_input=EN&Kalbos=EN&KategorijaEn_input=Orthopedy&Ka… 16/41
3/27/2019 LSMUSIS
Eight tactics of the treatment?

A. Immobilization by circular plaster cast


B. Plaster splint
C. Bed rest
D. Surgical treatment

51.25 26- years - old sportsman complains of acute pain in the right knee joint. No trauma history. Duration of
disease -5 months. Objective: limping, swelling, painful palpation, limited range of motion (flexion-500, extension -
0°) in the right knee joint. X-ray film show necrotic tine surrounding a portion (1X2cm) of the joint surface of
medial condyle. Which diagnosis is correct?

A. Morbus Osgood - Schtatter


B. Tumor of the medial condyle of femur
C. Konig's disease
D. Fracture of the medial condyle femur

51.26 26- years - old sportsman complains of acute pain in the right knee joint. No trauma history. Duration of
disease -5 months. Objective: limping, swelling, painful palpation, limited range of motion (flexion-500, extension -
lsmusis.lsmuni.lt/Klausimai/Spausdinti?Length=0?Kalba=EN&KategorijaId=156&Kalbos_input=EN&Kalbos=EN&KategorijaEn_input=Orthopedy&Ka… 17/41
3/27/2019 LSMUSIS
0°) in the right knee joint. X-ray film show necrotic tine surrounding a portion (1X2cm) of the joint surface of
medial condyle. What is a right tactics of the treatment?

A. Immobilization of the knee joint by plaster cast


B. Skin traction of the leg
C. Rentgenotherapy
D. Arthrotomy add scraping out of necrotic bone

51.27 A 39-year old sportsman complains of sharp pains in the region of right knee joint. In the history- contusion
of knee joint 5 years ago. No fractures was found, but after accident walked with limp. Objective findings: slight
swelling, tenderness and crepitus on palpation of patella, which increases with movements of the knee joint.
Radiographs shows incongruity of anterior surface et patella, marginal spurs , degenerative cysts. Which a
diagnosis is correct?

lsmusis.lsmuni.lt/Klausimai/Spausdinti?Length=0?Kalba=EN&KategorijaId=156&Kalbos_input=EN&Kalbos=EN&KategorijaEn_input=Orthopedy&Ka… 18/41
3/27/2019 LSMUSIS

A. Konig's disease
B. Morbus Osgood - Schlatter
C. Genu varum
D. Chondromalatia patellae

51.28 A 39-year old sportsman complains of sharp pains in the region of right knee joint. In the history- contusion
of knee joint 5 years ago. No fractures was found, but after accident walked with limp. Objective findings: slight
swelling, tenderness and crepitus on palpation of patella, which increases with movements of the knee joint.
Radiographs shows incongruity of anterior surface et patella, marginal spurs , degenerative cysts. What is a right
tactics of the treatment?

lsmusis.lsmuni.lt/Klausimai/Spausdinti?Length=0?Kalba=EN&KategorijaId=156&Kalbos_input=EN&Kalbos=EN&KategorijaEn_input=Orthopedy&Ka… 19/41
3/27/2019 LSMUSIS

A. Arthrotomy and removal of marginal spun


B. Resection of patella
C. Skeletal traction of tibia
D. Plaster splint

51.30 16-years-old girl complains of deformityof legs, remarked 2 years ago. Inspection shows both knee joints
closed and calves abducted. In such a position it is impossible to draw together tarsus joints (distance between
both ‘malleolus medialis 5cm). What is a right tactics of the treatment?

lsmusis.lsmuni.lt/Klausimai/Spausdinti?Length=0?Kalba=EN&KategorijaId=156&Kalbos_input=EN&Kalbos=EN&KategorijaEn_input=Orthopedy&Ka… 20/41
3/27/2019 LSMUSIS

A. Reposition and plaster immobilization


B. Total knee joint replacement
C. Corrective osteotomy of proximal tibia
D. Arthrodesis of the knee joint

51.31 Parents of the 7- years-old girl remarked false seat of their daughter. On inspection: curvature of the spine,
level of the scapula 1 cm higher and triangle of waist lesser than on the right side.X - ray film shows lateral
curvature of the thoracic part of spine with anguiafion (19°). which diagnosis is correct?

A. Scoliosis of 1st degree


B. False seat
C. Sprengel’s shoulder
D. Scoliosis of 2nd degree

lsmusis.lsmuni.lt/Klausimai/Spausdinti?Length=0?Kalba=EN&KategorijaId=156&Kalbos_input=EN&Kalbos=EN&KategorijaEn_input=Orthopedy&Ka… 21/41
3/27/2019 LSMUSIS
51.32 Parents of the 7- years-old girl remarked false seat of their daughter. On inspection: curvature of the spine,
level of the scapula 1 cm higher and triangle of waist lesser than on the right side.X - ray film shows lateral
curvature of the thoracic part of spine with anguiafion (19°). What is a right method of the treatment?

A. Guldyti ligonę į gipso lovelę


B. Complex conservative treatment: special gymnastic exercises, massage, electric muscle stimulation,
swimming
C. Operative treatment (spondylodesis)

51.33 13 - years - old boy complains impossibility to raise his right foot. History of acute illness with high
temperature at the age of 3 years. Medicamentous therapy, massage, physiotherapeutic procedures were
unsuccessful. Objective examination: distinct atrophy of right thigh and calf muscles. Right tarsus joint and foot
flail. Active dorsiflexion of the foot is impossible,Passive possible, but after release foot hang. Which a diagnosis
is correct? Kokia ligos diagnozė:

lsmusis.lsmuni.lt/Klausimai/Spausdinti?Length=0?Kalba=EN&KategorijaId=156&Kalbos_input=EN&Kalbos=EN&KategorijaEn_input=Orthopedy&Ka… 22/41
3/27/2019 LSMUSIS

A. Congenital clubfoot (pes equinovarus congenitus dex.)


B. Spastic paralysis
C. Sequela of rachitis .
D. Deformation of the right foot after poliomyelitis (pes equinus paralyticus dex)

51.34 13 - years - old boy complains impossibility to raise his right foot. History of acute illness with high
temperature at the age of 3 years. Medicamentous therapy, massage, physiotherapeutic procedures were
unsuccessful. Objective examination: distinct atrophy of right thigh and calf muscles. Right tarsus joint and foot
flail. Active dorsiflexion of the foot is impossible,Passive possible, but after release foot hang. What is a right tactic
of the treatment?

lsmusis.lsmuni.lt/Klausimai/Spausdinti?Length=0?Kalba=EN&KategorijaId=156&Kalbos_input=EN&Kalbos=EN&KategorijaEn_input=Orthopedy&Ka… 23/41
3/27/2019 LSMUSIS

A. Wear orthopedic shoes


B. Use antispastic remedies
C. Operative treatment (Triple arthrodesis of the foot)
D. To take warm bath

51.35 7-years - old boy complains of lack of control over movements of both legs. Objective symptoms: patient
walks with shuffling gait. Distinct flexion and adduction contractures of hips, flexion contracture of the knee and
equinus position of both feet. Tendon reflexes increased. Which diagnosis is correct?

lsmusis.lsmuni.lt/Klausimai/Spausdinti?Length=0?Kalba=EN&KategorijaId=156&Kalbos_input=EN&Kalbos=EN&KategorijaEn_input=Orthopedy&Ka… 24/41
3/27/2019 LSMUSIS

A. Congenital dislocation of both hip joints


B. Arthrogryposis
C. Spastic paralysis
D. Sequels of rachitis

51.36 7-years - old boy complains of lack of control over movements of both legs. Objective symptoms: patient
walks with shuffling gait. Distinct flexion and adduction contractures of hips, flexion contracture of the knee and
equinus position of both feet. Tendon reflexes increased. What is a right tactics of the treatment?

lsmusis.lsmuni.lt/Klausimai/Spausdinti?Length=0?Kalba=EN&KategorijaId=156&Kalbos_input=EN&Kalbos=EN&KategorijaEn_input=Orthopedy&Ka… 25/41
3/27/2019 LSMUSIS

A. Plaster immobilization
B. Operative treatment (corrective osteotomy of both femur bones)
C. Electric muscle stimulation
D. Operative treatment: tendotomy of adductors of both thighs and neurotomy of motoric radiclesof
n.obturatorius et n.tibialis)

51.37 4- years - old boy complains of pains in the region of hip joint limping. Objective examination: patient walks
with limp. Limitation of internal and external rotation in the right hip joint. Some right quadriceps atrophy. X-ray
films shows widening and irregularity of epiphyseal line with some rarefication of the juxta metaphyseal portion of
the femoral neck, irregularity of articular surface of epiphysisfragmentation of the right femoral bead. Which a
diagnosis is correct?

A. Epiphyseal osteomyelitis of the right femoral head


B. Osteosarcoma of the right femoral head C. Obstetric dislocation of the right hip joint
C. Perthes’ disease

lsmusis.lsmuni.lt/Klausimai/Spausdinti?Length=0?Kalba=EN&KategorijaId=156&Kalbos_input=EN&Kalbos=EN&KategorijaEn_input=Orthopedy&Ka… 26/41
3/27/2019 LSMUSIS
51.38 4- years - old boy complains of pains in the region of hip joint limping. Objective examination: patient walks
with limp. Limitation of internal and external rotation in the right hip joint. Some right quadriceps atrophy. X-ray
films shows widening and irregularity of epiphyseal line with some rarefication of the juxta metaphyseal portion of
the femoral neck, irregularity of articular surface of epiphysisfragmentation of the right femoral bead. What is a
right tactics of the treatment?

A. Active muscles strengthening exercises


B. Sanatorium treatment, long time bed rest
C. Treatment by intraarticular injections of hydrocortisone
D. Immediate operative treatment

51.39 5- years - old boy walks withwaddling gait (“duck’s gait”). Clinical examination reveals limited abduction of
the flexed thighs lumbar lordosis. Both great trochanter lies above Nelaton’s line. Positive Trendelenburg sign. X -
ray film show both femoral heads dislocated laterally and upward. Which a diagnosis is correct?

A. Perthes’ disease
B. Bilateral dislocation of the hip

lsmusis.lsmuni.lt/Klausimai/Spausdinti?Length=0?Kalba=EN&KategorijaId=156&Kalbos_input=EN&Kalbos=EN&KategorijaEn_input=Orthopedy&Ka… 27/41
3/27/2019 LSMUSIS
C. Bilateral epiphysiolysis of the hip
D. Bilateral subluxation of the hip

51.40 5- years - old boy walks withwaddling gait (“duck’s gait”). Clinical examination reveals limited abduction of
the flexed thighs lumbar lordosis. Both great trochanter lies above Nelaton’s line. Positive Trendelenburg sign. X -
ray film show both femoral heads dislocated laterally and upward. What is a right tactics of the treatment?

A. use Baubinas’ distraction device


B. Operative treatment
C. Use the Frejka pillow
D. Use massage of the buttock’s

51.41 12- years - old girl complains of limping. Objective examination: shortening of the right leg, bent of the
pelvis. Standing the right foot upon 5 cm high platform results disappearing of the bent. Which a diagnosis is
correct?

lsmusis.lsmuni.lt/Klausimai/Spausdinti?Length=0?Kalba=EN&KategorijaId=156&Kalbos_input=EN&Kalbos=EN&KategorijaEn_input=Orthopedy&Ka… 28/41
3/27/2019 LSMUSIS

A. Brevitas membri inferioris dextri


B. Brevitas membri inferioris sinistri
C. Sprengel's disease
D. Spastic paralysis

51.42 12- years - old girl complains of limping. Objective examination: shortening of the right leg, bent of the
pelvis. Standing the right foot upon 5 cm high platform results disappearing of the bent. What is a right tactics of
the treatment?

lsmusis.lsmuni.lt/Klausimai/Spausdinti?Length=0?Kalba=EN&KategorijaId=156&Kalbos_input=EN&Kalbos=EN&KategorijaEn_input=Orthopedy&Ka… 29/41
3/27/2019 LSMUSIS

A. Right leg - lengthening at the 18 years age


B. Immediate right leg lengthening
C. Massage of the right leg
D. Electric muscle stimulation of the right leg

51.43 Parents of the 12- days - old newborn remarked he is bandy - footed. On ,inspection: adduction and
supination of the right forefoot, varus and equinos position of heel. Which a diagnosis is correct?

A. Obstetric rupture of the lateral ligaments of midtarsal Joints of the right foot
B. Arthrogryposis
C. Congenital right clubfoot (pes equinovarus congenitus dextrus)
D. Kohler’s I disease

51.44 Parents of the 12- days - old newborn remarked he is bandy - footed. On ,inspection: adduction and
supination of the right forefoot, varus and equinos position of heel. What is a right tactics of the treatment?
lsmusis.lsmuni.lt/Klausimai/Spausdinti?Length=0?Kalba=EN&KategorijaId=156&Kalbos_input=EN&Kalbos=EN&KategorijaEn_input=Orthopedy&Ka… 30/41
3/27/2019 LSMUSIS

A. Operative treatment (Ligamentoplasty)


B. Wedging cast applied periodically
C. No treatment till one months of age
D. To correct deformation by soft Fink-Ettingen's bandages

51.45 Clinical examination of 3- months- old infant reveals asymmetrical skin folds, limited abduction of the flexed
thighs (right SCA, left - 60°). X - ray film shows obliquity of acetabulum (right acetabular angle 40°, left-35°),
irregularity of shenton’s lines and displacement of both femoral epiphysis laterally. Which a diagnosis is correct?

A. Bilateral slipping of the capital femoral epiphysis


B. Proximal epiphyseal osteomyelitis of both thigh bones
C. Bilateral dislocation of the hip due to obstetric trauma
D. Bilateral congenital dysplasia of the’ hip (dysplasia coxae congenita bilateralis)

lsmusis.lsmuni.lt/Klausimai/Spausdinti?Length=0?Kalba=EN&KategorijaId=156&Kalbos_input=EN&Kalbos=EN&KategorijaEn_input=Orthopedy&Ka… 31/41
3/27/2019 LSMUSIS
51.46 Clinical examination of 3- months- old infant reveals asymmetrical skin folds, limited abduction of the flexed
thighs (right SCA, left - 60°). X - ray film shows obliquity of acetabulum (right acetabular angle 40°, left-35°),
irregularity of shenton’s lines and displacement of both femoral epiphysis laterally. What is a right tactics of the
treatment?

A. No special treatment. Do not limit the movements of hip joints


B. Prescribe massage ad exercises of the thighs
C. Plaster cast immobilization
D. Maintain extremities in wide abduction by Frejka pillow

51.47 1- year- old girl is bandy - legged. X -ray film shows varus type curvature of both tibial bones. Which a
diagnosis is correct?

lsmusis.lsmuni.lt/Klausimai/Spausdinti?Length=0?Kalba=EN&KategorijaId=156&Kalbos_input=EN&Kalbos=EN&KategorijaEn_input=Orthopedy&Ka… 32/41
3/27/2019 LSMUSIS

A. Crura valga utriusque


B. Crura vara utriusque
C. Spastic paralysis
D. Scheuerman - Mau disease

51.48 1- year- old girl is bandy - legged. X -ray film shows varus type curvature of both tibial bones. What is a
right tactic of the treatment?

lsmusis.lsmuni.lt/Klausimai/Spausdinti?Length=0?Kalba=EN&KategorijaId=156&Kalbos_input=EN&Kalbos=EN&KategorijaEn_input=Orthopedy&Ka… 33/41
3/27/2019 LSMUSIS
A. Perform corrective osteotomy of both tibial bones
B. Bed rest
C. No special treatment. Control examination after 3 months
D. Use external fixation apparatus

51.49 3-years old boy complains of painful feet and quick tiredness in walk. Clinical examination reveals flat,
pronated feet, loss of longitudinal arch, abduction of fore foot and valgus position of position of heel with toe - out
gait . Which a diagnosis is correct?

A. Bilateral clubfoot (pes equino varus utriusque)


B. Pes calcaneus utriusque
C. Klippel - Feil’s disease
D. Bilateral flatfoot (pes plano valgus utriusque)

51.50 3-years old boy complains of painful feet and quick tiredness in walk. Clinical examination reveals flat,
pronated feet, loss of longitudinal arch, abduction of fore foot and valgus position of position of heel with toe - out
gait. What is a right lactic of the treatment?

A. Operative treatment
B. No treatment
C. Complex conservative treatment muscles of feet strengthening

51.51 3 - year - old boy is wry - necked. His head tilted to left, face turned to right side. The face on left side
flattened and shortened. Which a diagnosis is correct?

lsmusis.lsmuni.lt/Klausimai/Spausdinti?Length=0?Kalba=EN&KategorijaId=156&Kalbos_input=EN&Kalbos=EN&KategorijaEn_input=Orthopedy&Ka… 34/41
3/27/2019 LSMUSIS

A. Congenital left torticollis


B. Congenital right torticollis
C. Myoma of the neck
D. Kohler’s I disease

51.52 3 - year - old boy is wry - necked. His head tilted to left, face turned to right side. The face on left side
flattened and shortened. What is a right tactics of the treatment?

A. Operative treatment at the 3 years age


B. Immediate operative treatment (partial resection of sternocleidomastoid muscle)
C. Massage of the left side of the neck
D. To use a corrective collar

51.53 13 - year - old boy complains of pain in the region of right Calf below knee joint, occurred after football
match. Objective examination: local swelling and painfulness on palpation over right tibial tubercle, particularly
with overextension. X - ray films show density, some fragmentation of the hanging down right tibial apophysis and
soft-tissue swelling shadow. Which a diagnosis is correct?

lsmusis.lsmuni.lt/Klausimai/Spausdinti?Length=0?Kalba=EN&KategorijaId=156&Kalbos_input=EN&Kalbos=EN&KategorijaEn_input=Orthopedy&Ka… 35/41
3/27/2019 LSMUSIS

A. Osteomyelitis of the right tibia


B. Osteoma of the right tibia
C. Kohler's I disease
D. Osgood - Schlatter disease

51.54 13 - year - old boy complains of pain in the region of right Calf below knee joint, occurred after football
match. Objective examination: local swelling and painfulness on palpation over right tibial tubercle, particularly
with overextension. X - ray films show density, some fragmentation of the hanging down right tibial apophysis and
soft-tissue swelling shadow. What is a right tactics of the treatment?

lsmusis.lsmuni.lt/Klausimai/Spausdinti?Length=0?Kalba=EN&KategorijaId=156&Kalbos_input=EN&Kalbos=EN&KategorijaEn_input=Orthopedy&Ka… 36/41
3/27/2019 LSMUSIS

A. Use of antibiotics
B. Rest, avoidance of strenuous activity and physiotherapeutic measures
C. Operative treatment (resectio marginalis tibiae dextrae)
D. Injections of hydrocortizone

51.55 In the region of right upper arm of 13 - year - old girl appeared a painful tumor . In history - contusion of the
arm 2 years ago. On palpation - a hard immovable tumor arising from the anterior surface of right upper arm. X -
ray film shows osseous tumor with sharp apex and wide basis. Which a diagnosis is correct?

lsmusis.lsmuni.lt/Klausimai/Spausdinti?Length=0?Kalba=EN&KategorijaId=156&Kalbos_input=EN&Kalbos=EN&KategorijaEn_input=Orthopedy&Ka… 37/41
3/27/2019 LSMUSIS

A. Care type osteomyelitis of the humerus


B. Osgood - Schlatter disease
C. Callus after the fracture of right humerus
D. Osteoma of the humerus

51.56 In the region of right upper arm of 13 - year - old girl appeared a painful tumor . In history - contusion of the
arm 2 years ago. On palpation - a hard immovable tumor arising from the anterior surface of right upper arm. X -
ray film shows osseous tumor with sharp apex and wide basis. what is a right tactics of the treatment?

lsmusis.lsmuni.lt/Klausimai/Spausdinti?Length=0?Kalba=EN&KategorijaId=156&Kalbos_input=EN&Kalbos=EN&KategorijaEn_input=Orthopedy&Ka… 38/41
3/27/2019 LSMUSIS

A. Plaster cast immobilization


B. Use of antibiotics
C. No treatment
D. Operative treatment: marginal resection of the humerus

51.57 14- year - old boy complains of pain of the right foo increasing after walk. Objective examination: some
swelling and pain on palpation over 2nd metatarsal head. X - ray film shows fattening and irregularity of
articularsurface of the 2nd metatarsal head. Which a diagnosis is correct?

A. Osteosarcoma of the right 2nd metatarsal bone


B. Osteomyelitis of the right 2nd metatarsal bone
C. Fracture of the right 2nd metatarsal bone
D. Koehler's II disease

51.58 14- year - old boy complains of pain of the right foo increasing after walk. Objective examination: some
swelling and pain on palpation over 2nd metatarsal head. X - ray film shows fattening and irregularity of
lsmusis.lsmuni.lt/Klausimai/Spausdinti?Length=0?Kalba=EN&KategorijaId=156&Kalbos_input=EN&Kalbos=EN&KategorijaEn_input=Orthopedy&Ka… 39/41
3/27/2019 LSMUSIS
articularsurface of the 2nd metatarsal head. What is a right tactics of the treatment?

A. Immediate operative treatment


B. Use of antibiotics
C. Use arch support (supinator)
D. Use antituberculotic remedies

51.59 14- year - old girl is hump - backed, walks with the body bent forward. Deformity began 2 years ago and
tend to progress. Radiographs shows Th4 - Th8 vertebral bodies wedgeshaped, its upper and lower margins
irregular, fragmented. Which a diagnosis is correct?

A. Compression fracture of Th.- The vertebral bodies


B. Sprengel’s disease
C. Osteosarcoma
D. Scheuerman - Mau disease

lsmusis.lsmuni.lt/Klausimai/Spausdinti?Length=0?Kalba=EN&KategorijaId=156&Kalbos_input=EN&Kalbos=EN&KategorijaEn_input=Orthopedy&Ka… 40/41
3/27/2019 LSMUSIS
51.60 14- year - old girl is hump - backed, walks with the body bent forward. Deformity began 2 years ago and
tend to progress. Radiographs shows Th4 - Th8 vertebral bodies wedgeshaped, its upper and lower margins
irregular, fragmented. What is a right tactics of the treatment?

A. use of reclinator massage, physical exercises strengthening of dorsal muscles


B. Use a bicycle
C. Operative treatment: endocorrection of the spine
D. Operative treatment: spondylodesis

51.1 - B 51.2 - D 51.3 - D 51.4 - B 51.5 - B 51.6 - B 51.7 - B 51.8 - A


51.9 - D 51.10 - B 51.11 - D 51.12 - B 51.13 - A 51.14 - A 51.15 - A 51.16 - D
51.17 - C 51.18 - D 51.19 - C 51.20 - A 51.21 - B 51.22 - A 51.23 - C 51.24 - D
51.25 - C 51.26 - D 51.27 - D 51.28 - A 51.30 - C 51.31 - A 51.32 - B 51.33 - D
51.34 - C 51.35 - C 51.36 - D 51.37 - C 51.38 - B 51.39 - B 51.40 - B 51.41 - A
51.42 - B 51.43 - C 51.44 - B 51.45 - D 51.46 - D 51.47 - B 51.48 - C 51.49 - D
51.50 - C 51.51 - A 51.52 - B 51.53 - D 51.54 - B 51.55 - D 51.56 - D 51.57 - D
51.58 - C 51.59 - D 51.60 - A

lsmusis.lsmuni.lt/Klausimai/Spausdinti?Length=0?Kalba=EN&KategorijaId=156&Kalbos_input=EN&Kalbos=EN&KategorijaEn_input=Orthopedy&Ka… 41/41
3/27/2019 LSMUSIS

IV type tasks. Choose only one best answer

52.1 28 years old man fell out of the car and felt the pain at the symphysis and in the region of the wing of the
ileum. Casually is unable to walk. He can’t raise his legs up. The movements of the hip joints are slightly painful,
unlimited. The diastasis between the two hip bones is 2cm. Which a diagnosis is correct?

A. Fracture of the pubis.


B. Fracture of the ischium.
C. Fracture of the wing of the ileum.
D. Disruption of the symphysis pubis (symphyseolysis).

52.2 28 years old man fell out of the car and felt the pain at the symphysis and in the region of the wing of the
ileum. Casually is unable to walk. He can’t raise his legs up. The movements of the hip joints are slightly painful,
unlimited. The diastasis between the two hip bones is 2cm. What is the right tactics of the treatment?

A. Surround compression of the pelvis by placing the patient into the hammock.
B. The bed regimen.
C. Coxitic plaster cast
D. Skeletal traction of the femur.

52.3 50 years old man fell off a roof and felt severe pain in the right hip joint. Casualty is unable to walk. The
movements of the right hip joint are limited: abduction - 0 deg., adduction - 5 deg., flection - 0 deg., external
rotation - 0 deg., internalrotation -5 deg. Trauma occurred 3 hours before. Which a diagnosis is correct?

lsmusis.lsmuni.lt/Klausimai/Spausdinti?Length=0?Kalba=EN&KategorijaId=157&Kalbos_input=EN&Kalbos=EN&KategorijaEn_input=Traumatology&… 1/14
3/27/2019 LSMUSIS

A. Fracture of the acetabular fossa with the central displacement of the femur’s head.
B. Fracture of the pubic rami.
C. Fracture of the ischial and pubic rami.
D. Fracture of the neck of the femur.

52.4 50 years old man fell off a roof and felt severe pain in the right hip joint. Casualty is unable to walk. The
movements of the right hip joint are limited: abduction - 0 deg., adduction - 5 deg., flection - 0 deg., external
rotation - 0 deg., internalrotation -5 deg. Trauma occurred 3 hours before. What is the right tactics of the
treatment?

lsmusis.lsmuni.lt/Klausimai/Spausdinti?Length=0?Kalba=EN&KategorijaId=157&Kalbos_input=EN&Kalbos=EN&KategorijaEn_input=Traumatology&… 2/14
3/27/2019 LSMUSIS

A. Coxitic plaster cast


B. Disciplinar tension of the lower extremity.
C. Closed reduction and orthopedic support
D. Open reduction of the head of femur and fragments with internal plate fixation

52.5 27 years old man had drove the car which has struck against the wayside tree. Trauma occurred before 2
hours. The casualty is unable to walk and feels severe pain in the right hip joint The right foot is turned inwards
(rotated). Movements of the right hip joint are limited: abduction - 0 deg., adduction - 10 deg., flexion - 5 deg.,
extension - 0deg., internal rotation - 0 deg., external rotation - 8 deg. Which a diagnosis is correct?

lsmusis.lsmuni.lt/Klausimai/Spausdinti?Length=0?Kalba=EN&KategorijaId=157&Kalbos_input=EN&Kalbos=EN&KategorijaEn_input=Traumatology&… 3/14
3/27/2019 LSMUSIS

A. Coxarthrosis.
B. Fracture of the posterior margin of the acetabular fossa with the subluxation of the femur’s head.
C. Central displacement of the femur.
D. Fracture of the neck of the femur

52.6 27 years old man had drove the car which has struck against the wayside tree. Trauma occurred before 2
hours. The casualty is unable to walk and feels severe pain in the right hip joint The right foot is turned inwards
(rotated). Movements of the right hip joint are limited: abduction - 0 deg., adduction - 10 deg., flexion - 5 deg.,
extension - 0deg., internal rotation - 0 deg., external rotation - 8 deg. What is the right tactics of the treatment?

A. Coxitic plaster cast


B. Closed reduction.
C. Open reduction of the femun head and osteosynthesis of acetabular fossa
lsmusis.lsmuni.lt/Klausimai/Spausdinti?Length=0?Kalba=EN&KategorijaId=157&Kalbos_input=EN&Kalbos=EN&KategorijaEn_input=Traumatology&… 4/14
3/27/2019 LSMUSIS
D. Arthrodesis of the hip joint

52.7 19 years old woman fell from the second floor. Trauma occurred 4 hours before. Casualty feels severe pain
in the right side of the pelvis. Movements of the hip joints are painful, unlimited. Which a diagnosis is correct?

A. Fracture of the wing of the ilium


B. Fracture of the ischial tuberosity.
C. Symphyseolysis
D. Fracture of the pubis

52.8 19 years old woman fell from the second floor. Trauma occurred 4 hours before. Casualty feels severe pain
in the right side of the pelvis. Movements of the hip joints are painful, unlimited. What is the right tactics of the
treatment?

A. Coxitic plaster cast.


B. Open reduction and osteosynthesis.
C. Surround compression of the pelvis by placing the patient into the hammock
D. The bed regimen (in a face-downwards position with slightly flexed leg on the injured side)

52.9 18 years old man fell from the 5 meters height. Casualty is unable to walk and feels severe pain in both
sides of the pubic and the ischial rami. Movements of the hip joint are slightly painful. Trauma occurred 2 hours
before. Which a diagnosis is correct?

lsmusis.lsmuni.lt/Klausimai/Spausdinti?Length=0?Kalba=EN&KategorijaId=157&Kalbos_input=EN&Kalbos=EN&KategorijaEn_input=Traumatology&… 5/14
3/27/2019 LSMUSIS

A. Fracture of the ischial tuberosity.


B. Fracture of both the pubic bones
C. Fracture of the left ischium and the right pubis
D. Fracture of the left pubis and the right isehium

52.10 18 years old man fell from the 5 meters height. Casualty is unable to walk and feels severe pain in both
sides of the pubic and the ischial rami. Movements of the hip joint are slightly painful. Trauma occurred 2 hours
before. What is the right tactics of the treatment?

A. Skeletal traction.
B. Bed regimen in “frogs” position.
C. Coxitic plaster cast
D. Open reduction and osteosynthesis

52.11 22 years old sportsman fell on an outstretched hand in training, felt severe pain in the right shoulder and
arrived to traumatologic aid post. Swelling, bruising, tenderness and deformity were found in medial third of the
clavicle. Pain at the site of injury increased by movement Casualty supports the arm on the injured side at the
elbow. X-ray findings - look at picture. Which a diagnosis is correct?

A. Incomplete dislocation of the right acromioclavicular joint.


B. Displacement of the sternal end of the right clavicle

lsmusis.lsmuni.lt/Klausimai/Spausdinti?Length=0?Kalba=EN&KategorijaId=157&Kalbos_input=EN&Kalbos=EN&KategorijaEn_input=Traumatology&… 6/14
3/27/2019 LSMUSIS
C. Fracture of the right clavicle
D. Abruption of the coracoid process of the right scapula

52.12 22 years old sportsman fell on an outstretched hand in training, felt severe pain in the right shoulder and
arrived to traumatologic aid post. Swelling, bruising, tenderness and deformity were found in medial third of the
clavicle. Pain at the site of injury increased by movement Casualty supports the arm on the injured side at the
elbow. X-ray findings - look at picture. What is the right tactics of the treatment?

A. Closed reduction with subsequent immobilization in figure-of-eight dressing for 4-5 weeks.
B. Open reduction and internal fixation with sequent immobilization of the injured limb for 4-5 weeks.
C. Immobilization in plaster Desault dressing for 3-4 weeks.
D. Use of the sling for 2-3 weeks.

52.13 55 years old woman slipped, fell on an flexed hand, felt severe pain in the left shoulder and arrived to
traumatologic aid post The tenderness, swelling, limitation of movements due to pain in the left shoulder were
found. Crepitation of the fragments wasn't felt upon examination.Circulatory and sensation disorders weren't
found on the injured limb. X-ray findings are shown on the picture.

A. Fractura colli humeri sinistri


B. Fractura colli scapulae sinistrae
C. Abruptio tuberculi majoris humeri sinistri
D. Luxatio humeri sinistri

52.14 55 years old woman slipped, fell on an flexed hand, felt severe pain in the left shoulder and arrived to
traumatologic aid post The tenderness, swelling, limitation of movements due to pain in the left shoulder were
found. Crepitation of the fragments wasn't felt upon examination.Circulatory and sensation disorders weren't
found on the injured limb. X-ray findings are shown on the picture.

A. Immobilization in plaster Desault dressing for 3-5 weeks and, subsequently, judicious exercises
B. Closed reduction with subsequent immobilization in Velpeau dressing for 4-6 weeks.

lsmusis.lsmuni.lt/Klausimai/Spausdinti?Length=0?Kalba=EN&KategorijaId=157&Kalbos_input=EN&Kalbos=EN&KategorijaEn_input=Traumatology&… 7/14
3/27/2019 LSMUSIS
C. Open reduction and internal fixation
D. Skeletal traction by a Kirshner wire through the proximal ulna with the arm in abduction and elevation for
5-6 weeks.

52.15 30 years old man fall down on outstretched hand and felt severe pain in left shoulder joint The flattening of
deltoid region, anterior fullness, and restriction of motiondue to pain were found upon clinical examination. The
nerve and vascular injury is absence. X-ray findings - look to the picture. Which a diagnosis is correct?

A. Fractura colli humeri sinistri


B. Abruptio tuberculi majoris humeri sinistri
C. Fractura processus acromialis scapulae sinistrae
D. Luxatio humeri sinistri

52.16 30 years old man fall down on outstretched hand and felt severe pain in left shoulder joint The flattening of
deltoid region, anterior fullness, and restriction of motiondue to pain were found upon clinical examination. The
nerve and vascular injury is absence. X-ray findings - look to the picture. What is the right tactics of the treatment?

A. Skeletal traction by a Kirshner wire through the proximal ulna with the arm in abduction and elevation for
4 weeks.
B. Open reduction and immobilization in shoulder immobilizer.

lsmusis.lsmuni.lt/Klausimai/Spausdinti?Length=0?Kalba=EN&KategorijaId=157&Kalbos_input=EN&Kalbos=EN&KategorijaEn_input=Traumatology&… 8/14
3/27/2019 LSMUSIS
C. Closed reduction, immobilization in shoulder immobilizer for 1-3 weeks before active motion is begun.
D. Immobilization in plaster spica, with the arm in 30 degrees of external rotation and the elbow flexed to a
right angle, for 3-6 weeks before active motion is permitted.

52.17 40 years old man slipped, fell down and contused the right upper arm, felt the severe pain and was unable
to perform any movements with me injured limb. Tenderness, swelling, bruising, deformity, crepitating and
pamologic mobility of bone ends betweenthe mid and distal thirds of the right humerus were found on clinical
examination. Trauma occurred 3 hours before. The right hand was hanged down and active extension of fingers
and hand was broken down. The radial pulse was palpated. X-ray findings - look to the picture. Which a diagnosis
is correct?

A. Fractura humeri dextra. Laesio n. medianus dextri traumatica


B. Fractura comminutiva humeri dextri. Laesio n. ulnaris dextri
C. Fractura nonconsolidata humeri dextri. Neuropathia n. radialis dextri
D. Fractura humeri dextri. Laesio n. Radialis dextri traumatica

52.18 40 years old man slipped, fell down and contused the right upper arm, felt the severe pain and was unable
to perform any movements with me injured limb. Tenderness, swelling, bruising, deformity, crepitating and
pamologic mobility of bone ends betweenthe mid and distal thirds of the right humerus were found on clinical
examination. Trauma occurred 3 hours before. The right hand was hanged down and active extension of fingers
and hand was broken down. The radial pulse was palpated. X-ray findings - look to the picture. What is the right
tactics of the treatment?

lsmusis.lsmuni.lt/Klausimai/Spausdinti?Length=0?Kalba=EN&KategorijaId=157&Kalbos_input=EN&Kalbos=EN&KategorijaEn_input=Traumatology&… 9/14
3/27/2019 LSMUSIS

A. U-shaped coaptation plaster splint with shoulder immobilizer and dynamic hand supporter until fracture
has healed.
B. Open reduction & internal fixation, nerve revision & suture
C. Transosseous osteosynthesis and nerve suture

52.19 years old woman slipped, fell down on outstretched hand and felt severe pain in the right elbow joint The
local tenderness, swelling in the right elbow and restriction of forearm pronation and supination due to pain were
found on clinical examination. X-ray findings - look to the picture. Which a diagnosis is correct?

A. Fractura capituli humeri dextri

lsmusis.lsmuni.lt/Klausimai/Spausdinti?Length=0?Kalba=EN&KategorijaId=157&Kalbos_input=EN&Kalbos=EN&KategorijaEn_input=Traumatology… 10/14
3/27/2019 LSMUSIS
B. Fractura capitis radii dextri
C. Fractura colli radii dextri
D. Epiphyseolysis proximalis radii dextri

52.20 45 years old woman slipped, fell down on outstretched hand and felt severe pain in the right elbow joint The
local tenderness, swelling in the right elbow and restriction of forearm pronation and supination due to pain were
found on clinical examination. X-ray findings - look to the picture. What is the right tactics of the treatment?

A. Dorsal plaster splint from armpit to knuckles for 3 weeks.


B. Closed reposition and immobilization with plaster splint for 4-5 weeks.
C. Resectio capituli radii
D. Open reposition and transfixation with Kirshner wire through the humerus.

52.21 50 years old woman slipped, fell down on outstretched hand (the wrist in dorsiflexion and the forearm in
pronation) and felt severe pain in the left wrist.. The slighttenderness, insignificant swelling, bruising, “bayonet”
deformity were found on clinical examination. Active movements in left wrist joint are restricted due to pain. X-ray
findings are shown on the picture. Which a diagnosis is correct?

A. Distorsio ligamentorum articulationis carpi sinistri


B. Fractura ossis scaphoidei sinistri
C. Fractura radii sinistri in loco typico
D. Luxatio perilunaris art. carpi sinistri

52.22 50 years old woman slipped, fell down on outstretched hand (the wrist in dorsiflexion and the forearm in
pronation) and felt severe pain in the left wrist.. The slighttenderness, insignificant swelling, bruising, “bayonet”
deformity were found on clinical examination. Active movements in left wrist joint are restricted due to pain. X-ray
findings are shown on the picture. What is the right tactics of the treatment?

lsmusis.lsmuni.lt/Klausimai/Spausdinti?Length=0?Kalba=EN&KategorijaId=157&Kalbos_input=EN&Kalbos=EN&KategorijaEn_input=Traumatology&… 11/14
3/27/2019 LSMUSIS

A. Immobilization in a short-arm cast or volar split for 3-4 weeks.


B. Closed reduction and immobilization for 6 weeks.
C. Open reduction and internal fixation.

52.23 35 years old man carried a sack on one's back, fell down on outstretched left arm and felt marked pain in
upper third of forearm. Swelling, tenderness in the forearm and around the elbow were found. Patient resisted any
motion of the elbow joint. Any neurologic or vascular disorders weren't found. X-ray findings are shown on the
picture. Which a diagnosis is correct?

A. Fractura ulnae et luxatio capitis radii sinistri


B. Fractura radii et luxatio capitis ulnae sinistrae
C. Fractura olecrani et luxatio capitis radii sinistri
D. Luxatio posterioris radii et ulnae sinistri

52.24 35 years old man carried a sack on one's back, fell down on outstretched left arm and felt marked pain in
upper third of forearm. Swelling, tenderness in the forearm and around the elbow were found. Patient resisted any
motion of the elbow joint. Any neurologic or vascular disorders weren't found. X-ray findings are shown on the
picture. What is the right tactics of the treatment?

lsmusis.lsmuni.lt/Klausimai/Spausdinti?Length=0?Kalba=EN&KategorijaId=157&Kalbos_input=EN&Kalbos=EN&KategorijaEn_input=Traumatology… 12/14
3/27/2019 LSMUSIS

A. Internal fixation of the ulna and closed reduction of radial head.


B. Immobilization in a tubular plaster cast with the elbow at a right angle and the arm in full supination.
C. Internal fixation of the radius and open reduction of ulnar head.
D. Immobilization in a tubular plaster cast with the elbow at 110 degrees of flexion for 6 weeks.

52.25 18 years old lad during scrimmage felt severe pain in the thumb of the right hand. Swelling, tenderness,
deformity in proximal third of I metacarpal bone were found on clinical examination. The lad found it difficult to
move thumb. X-ray findings are shown on the picture. Which a diagnosis is correct?

A. Fractura ossis scaphoidei dextri


B. Fractura ossis metacarpalis primi manus dextrae
C. Fractura phalangis proximalis digiti primi manus dextrae

lsmusis.lsmuni.lt/Klausimai/Spausdinti?Length=0?Kalba=EN&KategorijaId=157&Kalbos_input=EN&Kalbos=EN&KategorijaEn_input=Traumatology… 13/14
3/27/2019 LSMUSIS
52.26 18 years old lad during scrimmage felt severe pain in the thumb of the right hand. Swelling, tenderness,
deformity in proximal third of I metacarpal bone were found on clinical examination. The lad found it difficult to
move thumb. X-ray findings are shown on the picture. What is the right tactics of the treatment?

A. Immobilization in plaster cast for 4-6 weeks.


B. Immobilization in dorsal plaster splint for 4-6 weeks.
C. Skeletal traction through the distal phalanx for 4-6 weeks.
D. Open reduction and fixation with Kirshner wire.

52.1 - D 52.2 - A 52.3 - A 52.4 - D 52.5 - B 52.6 - C 52.7 - B 52.8 - D


52.9 - D 52.10 - B 52.11 - C 52.12 - A 52.13 - A 52.14 - A 52.15 - D 52.16 - C
52.17 - D 52.18 - A 52.19 - B 52.20 - C 52.21 - C 52.22 - A 52.23 - A 52.24 - A
52.25 - B 52.26 - D

lsmusis.lsmuni.lt/Klausimai/Spausdinti?Length=0?Kalba=EN&KategorijaId=157&Kalbos_input=EN&Kalbos=EN&KategorijaEn_input=Traumatology… 14/14
3/27/2019 LSMUSIS

Rehabilitation
I type tasks. Choose only one best answer

8.1 When should rehabilitation care of patient begin?


A. After an acute period of the disease
B. Just after the evident onset of the disorder
C. When disablement is stated
D. When medical treatment does not result improvement

8.2 What is the optimum temperature of curative swimming pool water?


A. 26-28ºC
B. 32-36ºC
C. 28-32ºC
D. 36-38ºC

8.4 How would you describe the term “metabolic coefficient”?


A. How many milliliters of oxygen are uptaken per kilogram of body weight per minute
B. How many times more oxygen is uptaken by a subject during the activity than during quiet sitting
C. Now many times more oxygen is uptaken by a subject during the activity than during quiet lying

8.5 What is the lower level of norm of physical working capacity?


A. 100 W
B. 75 W
C. 1.5 w/kg of body mass
D. Ability to climbs stairs up to the second floor without shortness of breath (dyspnea) in accordance with
Lithuanian or American system

8.6 What is the work load for training the patient’s endurance?
A. The load, when heart rate = 220 – patient’s age
B. 60-90% of the threshold load
C. The load, when heart rate = 200 – patient’s age

8.7 What method of physiotherapy is the best to improve resorption of a postoperative infiltrate in the first days
after the surgery?
A. Inductothermy
B. Pulsed current therapy
C. Ultrahigh frequency current therapy
D. Iontophoresis

8.8 What physical factors acts upon the organism when applying amplipulsetherapy?
A. Direct current
B. Electric field
C. Sinusoidal 5000 Hz frequency currents modulated by low frequency
D. Mechanical vibrations

8.9 At what time after irradiation the individual ultraviolet ray biodose is estimated?
A. After 2-4 hours
B. After 4-6 hours
C. After 6-8 hours
D. After 8-12 hours

8.10 What physical agents are affecting the deep thermal effect:
A. Ultrasound, high frequency electromagnetic field therapy.
B. Application of paraffin.
C. Low frequency pulse electromagnetic field.
D. Polarized light.

8.11 What are the properties of „Bioptron“ light:


A. Polarized, polychromatic, non-coherent light, low power.
B. Monochromatic, coherent.
C. Polarized, monochromatic.

lsmusis.lsmuni.lt/Klausimai/Spausdinti?Length=0?Kalba=EN&KategorijaId=110&Kalbos_input=EN&Kalbos=EN&KategorijaEn_input=Rehabilitation&K… 1/6
3/27/2019 LSMUSIS
8.12 Patient’s with myocardial infarction threshold rate-pressure product is 230, metabolic coefficient is 5. What
functional class according to the Criteria Committee of the New York Heart Association classification does this
patient belong to?
A. I
B. II
C. III
D. IV

8.13 When exercise therapy is to be started for the patient with myocardial infarction:
A. From the first days
B. A week after the onset of illness
C. Two weeks after the onset
D. A month after the onset

8.14 What part of the disability reasons consist the chronic diseases:
A. 30-50%
B. 60-70%
C. 80-90%

8.15 How long is the prescription time of the permanent prosthesis?


A. 1-2 years
B. 3-4 years
C. 1-5 years.

8.16 When exercise therapy is to be started after thoracic surgery?


A. Immediately after narcosis time
B. 2-4 hours after the operation
C. 6-12 hours after the operation
D. 24 hours after the operation

8.17 What type of breathing is recommended in early period after abdominal operation?
A. Type of breathing does not matter
B. Thoracic
C. Abdominal

8.18 What are the natural heeling procedures?


A. Electrotherapy, ultrasound
B. Laser therapy.
C. Klimatotherapy, balneotherapy, mud therapy.

8.20 What are the parameters of sauna for patients?


A. to 45-60ºC, relative humidity 40-60%
B. to 70-80ºC, relative humidity 10%
C. to 90-110ºC, relative humidity 5-10%

8.21 What duration procedures in sauna are recommended for the athletes on training days?
A. 30-40 min.
B. 20-30 min.
C. iki 10 min.
D. 10-20 min.

8.22 What criteria are used to establish level of working capacity:


A. Medical
B. Functional
C. Medical, functional, professional
D. Professional

8.24 The patient with ischemic heart diseases has rare attacks of angina pectoris and the first stage of chronic
heart failure. What exercise regimen can be prescribed at sanatorium rehabilitation, if be achieves 75-100 W
threshold charge:
A. Conditioning
B. Conditioning-training
C. Training
lsmusis.lsmuni.lt/Klausimai/Spausdinti?Length=0?Kalba=EN&KategorijaId=110&Kalbos_input=EN&Kalbos=EN&KategorijaEn_input=Rehabilitation&K… 2/6
3/27/2019 LSMUSIS
D. Free

II type tasks. For each numbered item,selct the one lettered heading that is most closely asssciated with it

8.25 How is established the working capacity level of patient (working capacity level is established by percent):
1 - 0 - 25%
2 - 0 - 15%
3 - 30 - 55%
4 - 60 - 100%
A. Partly employable.
B. Employable
C. Disabled

8.26 How is established disability and working capacity of children and adult:
1 - Persons till 18 years.
2 - Persons from 18 years till retired age.
A. Three levels of disability.
B. The level of working capacity established by percent

8.27 What signs indicate the pressure ulcers stages


1-I
2 - II
3 - III
A. Superficial dermal ulceration
B. The ulceration in the junction with the muscles
C. The soft tissue erythema and induration above the bone prominence
D. The ulceration which include the subcutaneus tissue without the muscles

8.28 What are the measures of prophylaxis of following disablements:


1 - Primary
2 - Secondary
A. Hygienic measures, health education, rational nutrition, improvement of primary medical care,
prophylaxis of infectious and chronic noncommunicable diseases, immunization; prevention of various
traumas, unnecessary using of medicines and narcotic drugs; improvement ecological situation
B. Early determination of functional disorders, application of rehabilitation means

8.29 Main reason of disablement:


1 - In developing countries
2 - In developed industrial countries
A. Accidents, somatic diseases, chronic alcoholism, narcotic addiction, results of modern medicine,
increasing number of middle-aged inhabitants
B. Insufficient nutrition, infectious diseases, insufficient perinatal care, accidents

8.30 Which characterization suits best of all to the following exercise tolerance tests:
1 - On bicycle ergometer
2 - Master two-step test
3 - On treadmill
A. Most physiological
B. Most suitable for our conditions
C. Maybe performed on a patient with coxarthrosis
D. Mechanic work corresponds to the amount of expended energy

8.31 Which physiotherapeutical methods are the most suitable for the following illnesses:
1 - Diabetic neurotrophic ulcer
2 - Lumbosacral radiculitis (with dominating pain syndrome)
3 - Slown down the consolidation of bone after fracture
4 - Acute pneumonia
A. Iontophoresis with Novocain (procainehydrochloride)
B. Laser therapy
C. UHF current therapy
D. Ultrasound therapy

lsmusis.lsmuni.lt/Klausimai/Spausdinti?Length=0?Kalba=EN&KategorijaId=110&Kalbos_input=EN&Kalbos=EN&KategorijaEn_input=Rehabilitation&K… 3/6
3/27/2019 LSMUSIS
8.32 What is the basic factor of the following physiotherapeutical methods:
1 - Iontophoresis
2 - Diadynamic current therapy
3 - Diathermy
4 - Ultrasound
A. Interferential current
B. Constant electric current
C. Alternating electromagnetic field of high frequency
D. Mechanical vibration
E. Electric field

8.33 In which cases the following physiotherapeutical procedures are not recommended:
1 - UHF (ultra high frequency) current therapy
2 - Magnetotherapy
3 - Electromagnetic induction therapy
A. Development of adhesions
B. Thyrotoxicosis
C. Significant hypertension

III type tasks. For each question there is one or more correct answers:
A – if correct answers are 1,2,3
B – if correct answers are 1 and 3
C – if correct answers are 2 and 4
D – if correct answer is 4
E – if correct are all answers above

8.34 What is the therapeutic effect of diadynamic current therapy?


1. anesthetic
2. stimulating
3. decreasing inflammation
4. decreasing edema

8.35 What is the therapeutic effect of the UHF (ultrahigh frequency current)?
1. decreasing inflammation
2. improving tissues trophics
3. spasmolytic
4. decreasing regeneration of connective tissue and formation of adhesions

8.36 How does the physical load act upon the function of digestion system?
1. intensive physical load reduces gastric secretion and gastric motoric movements
2. small physical load increases gastric secretion and gastric motoric movements
3. blood circulation improves in pelvic and abdominal organs, vein blood outflow is accelerated
4. load intensiveness has no impact upon the digestion function

8.37 What is the effect to the organism of A and B ultraviolet rays:


1. take part in production of vitamin D
2. destroy vitamin D
3. used for healing psoriasis
4. increase the acidity of gastric juice

8.38 What are adversed effects of thermotherapy:


1. Burns
2. Fainting
3. Bleeding
4. Eye damage.

8.39 What are general indications for physical activity and exercise?
1. acute period of disease
2. when due to disease the function disappears or becomes inadequate
3. arterial blood pressure lower than 90/50 mm Hg
4. when the physical working capacity is less than 1,5 W per kilogram

lsmusis.lsmuni.lt/Klausimai/Spausdinti?Length=0?Kalba=EN&KategorijaId=110&Kalbos_input=EN&Kalbos=EN&KategorijaEn_input=Rehabilitation&K… 4/6
3/27/2019 LSMUSIS
8.40 What are the contraindications for physical exercise in a swimming pool?
1. various skin diseases
2. fever
3. epilepsy
4. respiratory failure

8.41 What means should be used in case of a tendon strain?


1. ice applied locally
2. keep the extremity in a rest state
3. elevation of extremity
4. orthopedic means

8.42 What are the main purposes of mechanotherapy:


1. to restore the range of joint motion
2. to increase the strength of the muscles
3. to increase physical working capacity
4. to reduce pain

8.43 Please describe the influence of the physical exercises to the patients with chronic pulmonary diseases?
1. Improve motivation
2. Improve respiratory function
3. Improve muscle capability
4. Stimulate the bone’s marrow.

8.44 What are the main rehabilitation goals for patients with respiratory disease?
1. To diminish the signs of the disease
2. To improve the physical work capasity
3. To improve functional state
4. To diminish the expences for health care.

8.45 Who has the main responsibility during the children’s rehabilitation process?
1. Physician
2. Psychologist
3. Physical therapist
4. Parent

IV type tasks. Choose only one best answer

8.46 Patient, walking on the even plane road, has no shortness of breath, but climbing upstairs or hill has it. Which
is the functional class of shortness of breath?
A. I
B. II
C. III
D. IV
E. V

8.47 Patient, walking on the even plane road, has no shortness of breath, but climbing upstairs or hill has it. Is the
patient able to work?
A. Able to work
B. Able to work if he (she) works in sitting position
C. Disabled

8.48 A patient with chronic hepatitis arrived at rehabilitation department. What diet would you prescribe to him
(her)?
A. SK↓
B. R ↓
C. B↓

8.49 A patient with chronic hepatitis arrived at rehabilitation department What advise should be given concerning
mineral water drinking?
A. To drink warmed up mineral water 40-45 minutes before meals
B. To drink warmed up to 35º-40ºC mineral water 1-1,5 hours before meals

lsmusis.lsmuni.lt/Klausimai/Spausdinti?Length=0?Kalba=EN&KategorijaId=110&Kalbos_input=EN&Kalbos=EN&KategorijaEn_input=Rehabilitation&K… 5/6
3/27/2019 LSMUSIS
C. To drink cold mineral water 30 minutes before meals

8.50 A 40 year-old patient with bronchial asthma arrived at the rehabilitation department. What tests should be
performed before making individualized rehabilitation program?
A. Assessment of acuteness of the disorder
B. Evalutation of pulmonary function
C. Chest X-ray film

8.51 A patient has shortness of breath during accomplishment of basic self-care activities (washing himself, putting
on his clothes). He has no shortness of breath at rest. Which is the class of the shortness of breath?
A. I
B. II
C. III
D. IV
E. V

8.52 A 50-year-old patient who had recovered from myocardial infarction arrived at the rehabilitation department.
During physical therapy patient starts to feel breathless, dizziness and cardiac arrhythmia. What is the cause of
this phenomenon?
A. Osteochondrosis
B. Myocardial ischemia became deeper
C. Increased blood pressure

8.53 A 50-year-old patient who had recovered from myocardial infarction arrived at the rehabilitation department.
During physical therapy patient starts to feel breathless, dizziness and cardiac arrhythmia. What tests will you
perform first to make the diagnosis exact?
A. X-ray of the cervical part of the spine
B. Blood pressure measurement
C. ECG

8.1 - B 8.2 - C 8.4 - B 8.5 - C 8.6 - B 8.7 - C 8.8 - C 8.9 - D


8.10 - A 8.11 - A 8.12 - B 8.13 - A 8.14 - C 8.15 - C 8.16 - A 8.17 - B
8.18 - C 8.20 - B 8.21 - C 8.22 - C 8.24 - B
8.25 8.26 8.27 8.28 8.29 8.30 8.31 8.32
1-C 1-A 1-C 1-A 1-B 1-C 1-B 1-B
2-C 2-B 2-A 2-B 2-A 2-B 2-A 2-A
3-A 3-D 3-A 3-D 3-C
4-B 4-C 4-D
8.33
1-A
2-C
3-B
8.34 8.35 8.36 8.37 8.38 8.39 8.40 8.41
1 1 1 1 1 2 1 1
2 2 2 3 2 4 2 2
3 3 3 3 3 3
4 4 4 4
8.42 8.43 8.44 8.45
1 1 1 4
2 3 2
3 3
4
8.46 - B 8.47 - B 8.48 - B 8.49 - A 8.50 - B 8.51 - C 8.52 - B 8.53 - C

lsmusis.lsmuni.lt/Klausimai/Spausdinti?Length=0?Kalba=EN&KategorijaId=110&Kalbos_input=EN&Kalbos=EN&KategorijaEn_input=Rehabilitation&K… 6/6
3/27/2019 LSMUSIS

Rheumatology
I type tasks. Choose only one best answer

24.1 What kind of skin affection is typical for dermatomyositis:


A. Facial and upper chest erythema, sensitive to insolation
B. Psoriasis- like scurfy spots
C. Axial and inguinal rashes
D. Pruritic rashes

24.2 What symptom is important for diagnostic of early stage rheumatoid arthritis:
A. Tenderness of Achiles
B. Ulnar deviation
C. Subcutaeus nodules
D. Swelling of proximal interphalangeal joints

24.3 What is typical for affection of joints in cases of systemic lupus erythematosus:
A. Early joint deformations
B. Subcutaneous nodules
C. Superficial erosions of bones, visible on X-rays
D. Nonerosive arthritis

24.4 Which of HLA antigens is associated with seronegative spondyloarthropathies:


A. B8
B. B27
C. DR4
D. DR3

24.5 The most probable cause of low back pain for 56 year-old woman, 50 kg of weigh, 8 years after extirpation of
uterus and adnexes:
A. Osteoporosis
B. Osteochondrosis
C. Sheuermann’s disease
D. Nephrolithiasis

24.6 Which statement about nonsteroidal antiinflammatory drugs is true:


A. Antiinflammatory effect depends on inhibition of cyclooxigenase-1
B. They inhibit synthesis of inflammatory prostaglandins
C. These drugs have stronger antiinflamatory effect than corticosteroids
D. They have proven chondroprotective effect

24.7 For which disease the syndrom of dry eyes is a principally characteristic:
A. Rheumatoid arthritis
B. Sarcoidosis
C. Scleroderma
D. Sjogren’s dissease

24.8 What kind of respiratory affection is mostly common in rheumatoid arthritis:


A. Bronchiolitis obliterans
B. Pulmonary hypertension
C. Fibrosing alveolitis
D. Exudative pleuritis

24.9 How frequently HLA B27 is found in cases of ankylosing spondyloarthritis:


A. More than in 90% cases
B. Frequency depends on population
C. Frequency depends on course of disease
D. Frequency depends on patients age and sex

24.10 Which medication has the mostly favorable rate of inhibition of cyclooxygenases (COX-2/COX-1):
A. Indomethacine
B. Ibuprofen
C. Ketoprofen
lsmusis.lsmuni.lt/Klausimai/Spausdinti?Length=0?Kalba=EN&KategorijaId=126&Kalbos_input=EN&Kalbos=EN&KategorijaEn_input=Rheumatology&… 1/10
3/27/2019 LSMUSIS
D. Celecoxib

24.11 Which pathological type of kidney affection is typical for severe lupus erythematosus with nephritic
syndrome:
A. Focal segmental glomerulosclerosis
B. Mesangial glomerulonephritis
C. Diffuse proliferative glomerulonephritis
D. Amyloidosis

24.12 What duration of antibiotics treatment is recommended in chlamydiosis –related arthritis:


A. One week
B. Four weeks
C. 3 months
D. 2 months

24.13 Which statement about NSAIDs in reactive arthritis is true:


A. Should be used in minimal effective doses
B. Should be used as analgetics, depending on severity of pain
C. Should be used as shortly, as it is possible
D. Should be used until the symptoms of arthritis dissapears

24.14 What is characteristic for dermato/ polymyositis:


A. Pleuritis, pericarditis
B. Amyloidosis
C. Fingers ulcers
D. Dysphagia

24.15 For which vasculitis serological hepatitis B markers are common:


A. Wegener’s granulomatosis
B. Polyarteriitis nodosa
C. Churg-Strauss syndrom
D. Takayassu disease

24.16 Which statement about rheumatoid arthritis is true:


A. Exercises have small influence on musculoskeletal function
B. Infection- related death is more frequent than in health population
C. Severity of dry syndrom is in correlation with arthritis activity

24.17 Which statement about advantages of joints echoscopy is not true:


A. Cartilage can be evaluated better that on X-rays
B. It is possible to differentiate intraarticular and periarticular fluid
C. Tendons and ligaments can be evaluated
D. Blood, pus and inflammatory synovial fluid can be differentiated

24.18 Which side-effect of glucocorticoids is mainly related with duration of treatment:


A. Osteoporosis
B. Peptic ulcers
C. Infection
D. Hyperglycemia

24.19 Which symptom is not characteristic for seronegative spondyloarthropathies:


A. Uveitis
B. Colitis
C. Pericarditis
D. Psoriasis

24.20 Which joints are most commonly involved in septic arthritis:


A. Knee
B. Humeroscapular
C. Wrist
D. Ankle

24.21 Which microorganism is causative for septic arthritis most often:


lsmusis.lsmuni.lt/Klausimai/Spausdinti?Length=0?Kalba=EN&KategorijaId=126&Kalbos_input=EN&Kalbos=EN&KategorijaEn_input=Rheumatology&… 2/10
3/27/2019 LSMUSIS
A. Streptococcus haemolyticus
B. Esherichia coli
C. Staphylococcus aureus
D. Klebsiella pneumoniae

24.22 What dosage of antibiotics should be used for the treatment of septic arthritis:
A. Middle doses
B. Large doses, like for the treatment of sepsis
C. Small doses, to avoid side effects of antibiotics
D. Large doses for one or two days than dosage must be reduced

24.23 Intraarticular corticosteroid injections are contraindicated in case of:


A. Reactive arthritis
B. Rheumatoid arthritis
C. Psoriatic arthritis
D. Septic arthritis

II type tasks. For each numbered item,selct the one lettered heading that is most closely asssciated with it

24.24 What rheumatic disease usualy presents with named signs:


1 - Symetric polyarthritis of matacarpophalangeal joints
2 - Asymetric oligoarthritis of the feet and urethritis
3 - Sacroileitis and symmetric syndesmophits on the x-ray of the spine
4 - Monoarthritis of the knee and febrile body temperature
A. Infectious arthritis
B. Reactive arthritis
C. Rheumatoid arthritis
D. Ankylosing spondylitis

24.25 Which localization of sensoric affection and pain is typical for each named radicular affection:
1 - C3
2 - C6
3 - L4
4 - S1
A. Lateral surface of feet
B. Occiput
C. External hand’s surface, thumb, II finger
D. External leg’s surface, spinal surface of feet

24.26 Which localization of pain is typical for each named affection of knee:
1 - Prepatellar bursitis
2 - B.anserinae bursitis
3 - Beiker’s cyst
4 - Pelegrini-Steda disease
A. Medial part of knee, with descendent radiation
B. Anterior part of patella
C. Back of the knee
D. Medial part of the knee

24.27 Which disease presents with following syndromes:


1 - Arthritis, serositis, dermatitis, nephritis
2 - Induration of the skin, dysphagia, pneumosclerosis
3 - Hemoptoe, sinusitis or rinitis, renal insufficiency
4 - Weakness of proximal muscles, elevation of creatinkinase in blood serum
A. Polymyositis
B. Vegeners granulomatosis
C. Systemic sclerosis
D. Systemic lupus erythematosus

24.28 Which autoantibodies are characteristic to diseases named below:


1 - Autoantibodies to native DNA
2 - Anti-histones

lsmusis.lsmuni.lt/Klausimai/Spausdinti?Length=0?Kalba=EN&KategorijaId=126&Kalbos_input=EN&Kalbos=EN&KategorijaEn_input=Rheumatology&… 3/10
3/27/2019 LSMUSIS
3 - Autoantibodies against neutrophils cytoplasm antigens
4 - Autoantibodies to centromeres
A. Drug-induced lupus
B. Lupus erythematosus
C. Wegeners granulomatosis
D. Scleroderma

24.29 For which disease each named roentgenological symptom is characteristic:


1 - Osteolysis of distal phalangs
2 - “Puncher’s” phenomenon
3 - Subchondral osteosclerosis, osteophytes
4 - Syndesmophites, “rails” phenomenon
A. Ankylosing spondyloarthritis
B. Osteoarthritis
C. Chronic gout
D. Scleroderma

24.30 For which disease each named deformation is characterictic:


1 - “Seals” hand
2 - “Widow’s hump”
3 - “Beggar’s“ posture
4 - “Sausage” digit
A. Rheumatoid arthritis
B. Ankylosing spondylitis
C. Osteoporosis
D. Reiter’s disease

III type tasks. For each question there is one or more correct answers:
A – if correct answers are 1,2,3
B – if correct answers are 1 and 3
C – if correct answers are 2 and 4
D – if correct answer is 4
E – if correct are all answers above

24.31 Which symptoms are not characteristic to ankylosing spondylityis:


1. Low back pain decreasing after activity
2. Mostly females are suffering
3. Onset of disease in young persons
4. Low back pain increasing after activity

24.32 Which X-ray symptoms are typical for ankylosing spondyloarthritis:


1. Widening of sacriiliac joint space
2. Subchondral sclerosis of sacriliac joint
3. “Squarening” of vertebral bodies
4. Syndesmophytes

24.33 Which laboratory changes are characteristic for ankylosing spondyloarthritis only:
1. Elevated ESR
2. HLA B 27 antigen
3. Antinuclear antibodies
4. None of them

24.34 Which X-ray symptoms are characteristic for psoriatic arthritis:


1. Bone erosions
2. Single syndesmophytes
3. Periostitis
4. Osteolysis

24.35 Which symptoms are characteristic to systemic form of rheumatoid arthritis:


1. Peripheric neuropathy
2. Rheumatoid nodules
3. Skin necrotising vasculitis
lsmusis.lsmuni.lt/Klausimai/Spausdinti?Length=0?Kalba=EN&KategorijaId=126&Kalbos_input=EN&Kalbos=EN&KategorijaEn_input=Rheumatology&… 4/10
3/27/2019 LSMUSIS
4. Erythema nodosum

24.36 What are causes of anemia in rheumatoid arthritis:


1. Hemolysis
2. Chronic inflammation
3. Hypersplenism
4. Side effect of medications

24.37 For seronegative spondyloarthropathies are characteristic:


1. Radiological symptoms of sacroiliitis
2. Negative rheumatoid factor
3. Relation to HLA B27
4. Asymetric pattern of arthritis

24.38 Which microorganisms can cause Reiter’s disease or reactive arthritis:


1. Shigella flexneri
2. Yersinia enterocolitica
3. Ureoplasma
4. Chlamidia trachomatis

24.39 In which situations in reactive arthritis sulfasalazyne is suitable for treatment:


1. Polyarthritis
2. Enterocolitis
3. Uroarthritis
4. Protracted arthritis

24.40 How to use glucocorticoids for treatment of reactive arthritis:


1. Maintained treatment in chronic cases
2. Long-term treatment with small per oral doses
3. Starting from the big dose, gradually reducing
4. Intraarticular injection

24.41 Computed tomography is a good method of investigation to confirm diagnosis of:


1. Prolapsus of intervertebral disk
2. Spinal stenosis
3. Spondylodiscitis
4. Osteochondrosis

24.42 Myopathy can be a side effect of:


1. Clofibrate
2. Penicillamine
3. Glucocorticoids
4. Lovastatin

24.43 Antiphospholipid antibodies – the group of heterogenous antibodies:


1. Lupus anticoagulant
2. Antibodies to DNA
3. Antibodies to cardiolipin
4. Antinuclear antibodies

24.44 Which medications are commonly used for the treatment of rheumatoid arthritis:
1. Antimaliarials
2. Methotrexate
3. Azathyoprine
4. Gold salts
5. D-Penicillamin

24.45 Which statement about gout is correct:


1. Morbidity is the same for females and males
2. The first attack of arthritis usually happens in 20-30 years of age
3. Uric acid in the blood is always elevated during arthritis attack
4. Patients usually are in a fever during the arthritis attack

lsmusis.lsmuni.lt/Klausimai/Spausdinti?Length=0?Kalba=EN&KategorijaId=126&Kalbos_input=EN&Kalbos=EN&KategorijaEn_input=Rheumatology&… 5/10
3/27/2019 LSMUSIS
24.46 What is typical for the primary Sjogren’s syndrome:
1. Positive serum rheumatoid factor
2. Lymphoma
3. Peripheric neuropathy
4. Skin induration

24.47 Which statements about glucocorticoid-induced osteoporosis are correct:


1. Reduction of intestinal resorbtion of calcium
2. Increasing of renal excretion of calcium
3. Inhibition of proliferation of osteoblasts
4. Vit.D 400 IU/ day and bisphosphonates are equally effective for the treatment

24.48 Which statements about pregnancy in systemic lupus erythematosus are correct:
1. Lupus usually becomes less active during pregnancy
2. Spontaneous abortions are rare
3. The best treatment – small doses of glucocorticoids
4. anti-SS/A, anti-SS/B are related to risk for fetus affection

24.49 Lupus erythematosus diagnostic criteria are:


1. Ulcers of mucous membrane of mouth
2. Hemolytic anaemia
3. Anti-Sm
4. Negative rheumatoid factor

24.50 What kind of treatment do you choose for the patient with lupus erythematosus and arthritis, serositis,
nephritis:
1. NSAIDS and sulfasalazin
2. NSAIDS and prednisone
3. Low dosage of prednisone and methotrexate
4. Puls therapy with prednisone and cyclophosphamide

24.51 What tests or procedures are of the best diagnostic value for septic arthritis:
1. Blood culture
2. X-ray of the joint
3. Magnetic resonance of the joint
4. Synovial fluid culture

IV type tasks. Choose only one best answer

24.52 74 –year-old woman has fever for 5 weeks, and complains about headache, “claudication” of tongue and
“locking” of jaws , when she is speaking and eating. Obj: T 38ºC, tender and swollen temporal arteries, WBC
9,2x10 /l, Hb 104 g/l, ESR 127 mm/h. Diagnosis:
A. Odontogenic infection
B. Temporal arteriitis
C. Systemic lupus erythematosus
D. Sepsis

24.53 74 –year-old woman has fever for 5 weeks, and complains about headache, “claudication” of tongue and
“locking” of jaws , when she is speaking and eating. Obj: T 38ºC, tender and swollen temporal arteries, WBC
9,2x10 /l, Hb 104 g/l, ESR 127 mm/h. Treatment:
A. Prednisolon 60 mg/day
B. Methylprednisolon 1000 mg i/v
C. Azathyoprine 150 mg/day
D. Penicillin 6 mln.UA /day

24.54 32-year-old female has arthralgia, weakness, and sub-febrile fever for 3 months. Obj.: swelling of proximal
interphalangeal joints, ESR 51 mm/h, CRP – normal. Diagnostic investigations:
A. WBC
B. ANA, anti-DNA
C. Blood culture
D. Cardioechoscopy

lsmusis.lsmuni.lt/Klausimai/Spausdinti?Length=0?Kalba=EN&KategorijaId=126&Kalbos_input=EN&Kalbos=EN&KategorijaEn_input=Rheumatology&… 6/10
3/27/2019 LSMUSIS
24.55 32-year-old female has arthralgia, weakness, and sub-febrile fever for 3 months. Obj.: swelling of proximal
interphalangeal joints, ESR 51 mm/h, CRP – normal. Treatment, if diagnosis is systemic lupus erythematosus:
A. Plaquenil and small dose of prednisolon
B. Azathyoprine
C. Cyclophosphamide puls-therapy
D. Antipyretics

24.56 32-years-old male complains about acute low back pain, radiating to left leg till the knee joint. Obj.:
straighten leg can be raised till 60º, positive test for n.ischiadicus tension, no signs of sensoric affection, muscle
strength and reflexes –normal. Diagnostic investigation:
A. Lumbar spine X-ray
B. WBC and ESR
C. CT of lumbar spine
D. Prostate specific antigen

24.57 32-years-old male complains about acute low back pain, radiating to left leg till the knee joint. Obj.:
straighten leg can be raised till 60º, positive test for n.ischiadicus tension, no signs of sensoric affection, muscle
strength and reflexes –normal. Treatment :
A. Bed regime for 14 days
B. Systemic glucocorticoids
C. Analgetics, myorelaxants, slight dehydration
D. Epidural injection of glucocorticoids

24.58 40-years-old man has complains about the right arm pain, lasted 6 weeks. Pain arise during the active
abduction of shoulder, this movement is limited by pain. Rotation of shoulder is painless. Which diagnosis is mostly
probable:
A. Subacromial bursitis
B. Shoulders rotator cuff syndrome
C. Bicipital tendonitis
D. Fibrotic capsulitis

24.59 65-year-old female, had investigation for bone mineral density (BMD). Lumbar T score was found –2,1,
femoral neck T score –2,2. Lady is in menopause from 46 years . She is generally health, smoking 20 cigarettes
per day, drinking 200-300ml of vine per day. Your recommendation:
A. Hormone replacement therapy
B. Life style changes ( Intake of calcium supplements, exercises, stop to smoke and to drink alcohol)
C. Bisphosphonates for 3 years
D. Natrium fluoride

24.60 A 69 –year- old woman complains of pain in both knees. She can go at on foot without stopping only 500
metres . Physical examination : a patient`s body weight 85 kg, knees are deformed but without swelling, there are
crepitus on movement, x-ray - end stage of osteoarthritis. What treatment would you recommend:
A. Quadriceps strengthening exercises
B. Intra – articular hyaluronic acid
C. Arthroscopy
D. Joint replacement (arthroplasty)

24.61 A 64-year-old man has had seropositive rheumatoid arthritis for 15 years. He is treated with 5 mg of
prednisolon per day and 12,5 mg of methotrexate per week. He also has diabetes mellitus and is treated with
peroral hypoglicemic drugs. Over the last three weeks he has been suffering from the increasing pain in his right
knee. Physical examination: the right knee warm, swollen due to an effusion. The left knee without any signs of
synovitis. Other affected joints (hands, wrists, elbows, feet) are slightly deformed, but without visible signs of
synovitis. What is the most likely diagnosis:
A. Exacerbation of rheumatoid arthritis
B. Gout
C. Septic arthritis
D. Right knee meniscus tear

24.62 A 64-year-old man has had seropositive rheumatoid arthritis for 15 years. He is treated with 5 mg of
prednisolon per day and 12,5 mg of methotrexate per week. He also has diabetes mellitus and is treated with
peroral hypoglicemic drugs. Over the last three weeks he has been suffering from the increasing pain in his right
knee. Physical examination: the right knee warm, swollen due to an effusion. The left knee without any signs of

lsmusis.lsmuni.lt/Klausimai/Spausdinti?Length=0?Kalba=EN&KategorijaId=126&Kalbos_input=EN&Kalbos=EN&KategorijaEn_input=Rheumatology&… 7/10
3/27/2019 LSMUSIS
synovitis. Other affected joints (hands, wrists, elbows, feet) are slightly deformed, but without visible signs of
synovitis. What should be performed?
A. Aspiration of joint fluid and injection of intra-articular steroids
B. Raise the dose of methotrexate up to 15 mg/week and suggest intra-articular sreroid injection
C. Take blood for CRP and leukogram, assign next visit after 2 days
D. Aspirate joint fluid and examine it (microscopic examination, culture and antibioticogram)

24.63 A 66-year –old woman complains of pain in her back, thighs, groins, buttocks, she limps a little. She has had
pain for some years , but before she felt it only after a long period of walk, manual work and it dissapeared after a
rest or after taking a tablet of paracetamol. Which investigation specifys the stage of osteoarthritis:
A. X-ray of the joints
B. Ultrasound of the joint
C. Joint aspiration
D. Clinical evaluation of joint movement amplitude

24.64 A 35-year-old woman has been complaining of knees pain and swelling for some years. Symptom
decreaseses significantly while taking diclofenac, but they come back after the discontinuance of treatment. The
left hip, groin became painful 5 days ago. Physical examination: both knees are swollen and painful, the overlying
skin is warm. A rotation of left hip is painful, there are few elements of psoriasis-like rash on elbows. What is the
diagnosis:
A. Reiter´s disease
B. Psoriatic arthritis
C. Ankylosing spondylitis
D. Gout

24.65 A 35-year-old woman has been complaining of knees pain and swelling for some years. Symptom
decreaseses significantly while taking diclofenac, but they come back after the discontinuance of treatment. The
left hip, groin became painful 5 days ago. Physical examination: both knees are swollen and painful, the overlying
skin is warm. A rotation of left hip is painful, there are few elements of psoriasis-like rash on elbows. What
treatment would you recommend:
A. NSAIDs
B. Methotrexate
C. Cyclosporine
D. Allopurinol

24.66 A 45-year-old woman has had fever for 5 days, she has also noticed skin rash (small red spots) in her lower
legs. Later rash spreaded to the middle of thighs, she had a few episodes of griping pain in her stomach and
sickness after meal. History: she is allergic to citrus, to some washing powder. Two weeks ago she was ill with
tonsillitis, therefore she used penicillin. Physical examination: Temperature 37,9° C, there is palpable purpura on
the skin of legs, from the soles till the middle of shin, left ankle is painful and swollen, during abdominal palpation ,
abdomen was soft, painful in epigastric and mesogastric areas What is the most likely diagnosis:
A. Subacute urticaria
B. Henoch-Schonlein vasculitis
C. Erythema nodosum
D. Polyarteritis nodosa

24.67 A 45-year-old woman has had fever for 5 days, she has also noticed skin rash (small red spots) in her lower
legs. Later rash spreaded to the middle of thighs, she had a few episodes of griping pain in her stomach and
sickness after meal. History: she is allergic to citrus, to some washing powder. Two weeks ago she was ill with
tonsillitis, therefore she used penicillin. Physical examination: Temperature 37,9° C, there is palpable purpura on
the skin of legs, from the soles till the middle of shin, left ankle is painful and swollen, during abdominal palpation ,
abdomen was soft, painful in epigastric and mesogastric areas What treatment would you give:
A. Low dose of prednisolone
B. Antiallergic drugs
C. NSAIDs
D. Medium dose of prednisolone

24.68 A 26-year-old man complains of low back pain, radiating into his buttocks and outer surface of his thighs.
The pain is not very intensive, but it has been present for two years and it is getting worse now. Pain is
exacerbated by rest, he always feels it early in the morning. The pain improves with exercise, after some period of
walking. From history: he had some episodes of eye inflammation, tonsillitis, once pneumonia. Patient`s father has
ulcerative colitis. Physical examination: T 37,2ᵒC, the loss of lumbar lordosis, lumbar muscle spasm, the pain
lsmusis.lsmuni.lt/Klausimai/Spausdinti?Length=0?Kalba=EN&KategorijaId=126&Kalbos_input=EN&Kalbos=EN&KategorijaEn_input=Rheumatology&… 8/10
3/27/2019 LSMUSIS
during palpation over sacrum margins , he can not reach the floor without bending knees. Other joints and internal
organs are without alterations. Laboratory investigations: Leucocytes 8,9×109/l, CRP 27 mg/l. What is the most
likely diagnosis:
A. Osteochondrosis
B. Juvenile osteochondropathy
C. Osteoporosis
D. Ankylosing spondylitis

24.69 A 26-year-old man complains of low back pain, radiating into his buttocks and outer surface of his thighs.
The pain is not very intensive, but it has been present for two years and it is getting worse now. Pain is
exacerbated by rest, he always feels it early in the morning. The pain improves with exercise, after some period of
walking. From history: he had some episodes of eye inflammation, tonsillitis, once pneumonia. Patient`s father has
ulcerative colitis. Physical examination: T 37,2ᵒC, the loss of lumbar lordosis, lumbar muscle spasm, the pain
during palpation over sacrum margins , he can not reach the floor without bending knees. Other joints and internal
organs are without alterations. Laboratory investigations: Leucocytes 8,9×109/l, CRP 27 mg/l. What treatment
would you recommend?
A. NSAIDs
B. Prednisolone given orally
C. NSAIDs and methotrexate
D. NSAIDs and sulphasalazine

24.70 A 57-years-old woman has been having subfebrile temperature, the pain in her wrists and knees, repetitive
sinusitis and she coughs with blood. Recently, signs of renal insufficiency have emerged. The treatment with
antibiotics was ineffective. What investigations would you perform to specify the diagnosis:
A. Rheumatoid factor, antibodys against native DNA
B. Biopsy from mucosa of sinuses, chest X-ray, ANCA antibodys, bronchoscopy
C. Anti CCP antibodys, chest X-ray, knees and wrists X-ray
D. Serum urate, examination of knee joint fluid, X-ray of painful joints

24.71 A 57-years-old woman has been having subfebrile temperature, the pain in her wrists and knees, repetitive
sinusitis and she coughs with blood. Recently, signs of renal insufficiency have emerged. The treatment with
antibiotics was ineffective. What is the most likely diagnosis:
A. Rheumatoid arthritis
B. Gout and its caused chronic renal failure
C. Systemic lupus erythematosus
D. Wegener´s granulomatosis

24.72 A 57-years-old woman has been having subfebrile temperature, the pain in her wrists and knees, repetitive
sinusitis and she coughs with blood. Recently, signs of renal insufficiency have emerged. The treatment with
antibiotics was ineffective. What treatment would you give?
A. NSAIDs
B. Medium doses of prednisolone given orally and methotrexate
C. Intra-articular steroids injections and NSAIDs
D. Pulse therapy with methylprednisolone and cyclophosphamide

24.73 A 75-year-old-man has fever up to 38,5ᵒ C with shiver. The right knee is swollen, warmer and painful during
palpation. 3 days ago a rheumatologist injected 40 mg of kenalog into this joint because of osteoarthritis and
secondary synovitis . What is the preliminary diagnosis and what investigations are the most important to specify
the diagnosis:
A. Gouty arthritis, synovial fluid examination for detection urate crystals with polarising light microscope
B. Septic arthritis, culture of joint fluid
C. Reactive arthritis, CRP, ESR
D. Osteoarthritis, exacerbation of secondary synovitis, X-ray of the right knee

24.1 - A 24.2 - D 24.3 - D 24.4 - B 24.5 - A 24.6 - B 24.7 - D 24.8 - C


24.9 - A 24.10 - D 24.11 - C 24.12 - B 24.13 - D 24.14 - D 24.15 - B 24.16 - B
24.17 - D 24.18 - A 24.19 - C 24.20 - A 24.21 - C 24.22 - B 24.23 - D
24.24 24.25 24.26 24.27 24.28 24.29 24.30
1-C 1-B 1-B 1-D 1-B 1-D 1-A
2-B 2-C 2-A 2-C 2-A 2-C 2-C
3-D 3-D 3-C 3-B 3-C 3-B 3-B
lsmusis.lsmuni.lt/Klausimai/Spausdinti?Length=0?Kalba=EN&KategorijaId=126&Kalbos_input=EN&Kalbos=EN&KategorijaEn_input=Rheumatology&… 9/10
3/27/2019 LSMUSIS
4-A 4-A 4-D 4-A 4-D 4-A 4-D
24.31 24.32 24.33 24.34 24.35 24.36 24.37 24.38
2 1 4 1 1 2 1 2
4 2 2 2 4 2 4
3 3 3 3
4
24.39 24.40 24.41 24.42 24.43 24.44 24.45 24.46
4 4 1 1 1 1 4 1
2 2 3 2 3
3 3 3
4
24.47 24.48 24.49 24.50 24.51
1 4 1 4 4
2 2
3 3
24.52 - B 24.53 - A 24.54 - B 24.55 - A 24.56 - C 24.57 - C 24.58 - B 24.59 - B
24.60 - D 24.61 - C 24.62 - D 24.63 - A 24.64 - B 24.65 - B 24.66 - B 24.67 - D
24.68 - D 24.69 - A 24.70 - B 24.71 - D 24.72 - D 24.73 - B

lsmusis.lsmuni.lt/Klausimai/Spausdinti?Length=0?Kalba=EN&KategorijaId=126&Kalbos_input=EN&Kalbos=EN&KategorijaEn_input=Rheumatology… 10/10
3/27/2019 LSMUSIS

Neurology
I type tasks. Choose only one best answer

36.1 Is atrophy present when peripheral motor neuron is damaged:


A. Yes
B. No

36.2 What is the change in muscle tone when peripheral motor neuron is damaged
A. Increased
B. Decreased
C. No change

36.3 Does tetraplegia occur in case of unilateral internal capsule damage


A. Yes
B. No

36.4 Does pathologic synkinesias develop when peripheral motor neuron is damaged
A. Yes
B. No

36.5 Which group of symptoms describes the lesion of upper motor neuron:
A. Muscle atrophy, atonia, areflexia, fasciculations
B. Spasticity, exaggerated proprioreceptive reflexes, pathologic reflexes and synkinesias, loss of skin
reflexes
C. Intention tremor, scanned speech, nystagmus, muscle hypotonia, ataxia, dysdiadochokinesia
D. Amimia, bradylalia, bradykinesia, rigidity, boxer’s posture, “cog-wheel” phenomenon, micrography

36.6 Which group of symptoms describes the lesion of lower motor neuron:
A. Muscle atrophy, atonia, areflexia, fibrillation, fasciculation
B. Spasticity, exaggerated proprioreceptive reflexes, pathologic reflexes and synkinesias, loss of skin
reflexes
C. Intention tremor, scanned speech, nystagmus, muscle hypotonia, ataxia, dysdiadochokinesia
D. Amimia, bradylalia, bradykinesia, rigidity, boxer’s posture, cog-wheel phenomenon, micrography

36.7 Which group of symptoms describes the lesion of cerebellum:


A. Muscle atrophy, atonia, areflexia, fasciculation
B. Spasticity, exaggerated proprioreceptive reflexes, pathologic reflexes and synkinesias, loss of skin
reflexes
C. Intention tremor, scanned speech, nystagmus, muscle hypotonia, ataxia, dysdiadochokinesia
D. Amimia, bradylalia, bradykinesia, rigidity, boxer’s posture, “cog-wheel” phenomenon, micrography

36.8 Which group of symptoms describes the lesion of paleostriatum:


A. Muscle atrophy, atonia, areflexia, fibrillation, fasciculation, change of conduction in nerves
B. Spastic increase of muscle tone, exaggerated proprioreceptive reflexes, pathologic reflexes and
synkinesias, loss of skin reflexes
C. Intention tremor, scanning speech, nystagmus, muscle hypotonia, “ ataxia, dysdiadochokinesia
D. Amimia, bradylalia, bradykinesia, rigidity, tremor, “cog-wheel” phenomenon, micrography

36.9 Which group of symptoms is typical for dementia syndrome


A. Ataxic, dysdiadochokinesia, intention tremor, scanned speech, nystagmus, muscle hypotonia
B. Amimia, bradylalia, bradykinesia, rigidity, boxer’ posture, fatty face, “cog-wheel” phenomenon,
micrography
C. Acquired, progressive deterioration of more than one of the cognitive functions sufficient to impair of daily
activity

36.10 Which symptom is characteristic for L5 radiculopathy:


A. Weakness of extensors of the big toe
B. Loss of Achille tendon reflex
C. Hypoesthesia in the little toe
D. Weakness of flexors of the big toe
E. Loss of knee-jerk (patellar) reflex

lsmusis.lsmuni.lt/Klausimai/Spausdinti?Length=0?Kalba=EN&KategorijaId=138&Kalbos_input=EN&Kalbos=EN&KategorijaEn_input=Neurology&Kate… 1/11
3/27/2019 LSMUSIS
36.11 Which symptom is characteristic to S1 radiculopathy:
A. Loss of knee-jerk (patellar) reflex
B. Hypoesthesia in the lateral foot area
C. Weakness of extensors of the big toe
D. Weakness of flexors of the big toe
E. Hypoesthesia of the knee-cap area

36.12 Which symptom is characteristic to C7 radiculopathy:


A. Loss of triceps reflex
B. Hypoesthesia in the thumb
C. Loss of biceps reflex
D. Loss of carporadial reflex
E. Hypoesthesia of the little finger

36.13 Which symptom is characteristic for motor aphasia:


A. Impaired comprehension of the speech
B. Impaired recognition of the object
C. Impaired ability to speak
D. Scanning speech

36.68 Which symptom reflects cognitive dtsfunction:


A. Scanning speech
B. Aphasia
C. Bradylalia
D. Dysarthria

II type tasks. For each numbered item,selct the one lettered heading that is most closely asssciated with it

36.14 Which part of the nervous system is impaired in case of the following diseases
E 1 - Multiple sclerosis
C 2 - Lateral amyotrophic sclerosis
D 3 - Syringomyelia
B 4 - Myasthenia
A 5 - Huntington’s chorea
A. Basal ganglia
B. Neuromuscular junction
C. Motor nuclei and pyramidal tract
D. Posterior horns of the spinal cord
E. Myelin sheaths around the axons

36.15 What symptoms are characteristic for these diseases


B 1 - Herpetic encephalitis
A 2 - Parkinson disease
C 3 - Multiple sclerosis
D 4 - Guillain – Barre syndrome
A. Bradykinesia
B. Seizures
C. Optic neuritis
D. Polyneuropathy

36.16 What sign is characteristic for each of these lesions


1 - Lesion of n. radialis
2 - Lesion of n. ulnaris
3 - Lesion of n. medianus
4 - Lesion of n. peroneus communis
5 - Lesion of n. tibialis
A. Weakness of wrist extension
B. Weakness of thumb abduction
C. Weakness of fingers abduction and adduction
D. Weakness of foot extension
E. Weakness of foot plantar flexion

lsmusis.lsmuni.lt/Klausimai/Spausdinti?Length=0?Kalba=EN&KategorijaId=138&Kalbos_input=EN&Kalbos=EN&KategorijaEn_input=Neurology&Kate… 2/11
3/27/2019 LSMUSIS
III type tasks. For each question there is one or more correct answers:
A – if correct answers are 1,2,3
B – if correct answers are 1 and 3
C – if correct answers are 2 and 4
D – if correct answer is 4
E – if correct are all answers above

36.17 Which of drugs are used for the secondary prevention of ischemic stroke:
1. Anticoagulants
2. Antiagregants
3. Statins
4. Antioxidants

36.18 Which symptoms are characteristic for brainstem herniation:


1. Bradycardia
2. Irregular breathing
3. Decreased blood pressure
4. Increased sweating

36.19 What clinical signs are characteristic for subarachnoid hemorrhage:


1. Acute onset of the symptoms
2. Severe headache, vomiting
3. Meningeal signs
4. Psychomotor agitation

36.20 What are the signs of the peripheral facial palsy:


1. Ipsilateral lacrimation, possible impairment of taste
2. Unilateral paralysis of all mimic muscles
3. Hyperacusis
4. Ipsilateral hypoesthesia

36.21 What symptoms and signs are characteristic for acute inflammatory demyelinating neuropathy (Guillain-Bare
syndrome):
1. Polyneuropathic sensory impairment
2. Respiratory weakness
3. Weakness and areflexia in extremities
4. Protein and cell dissociation in the cerebrospinal fluid

36.22 What drugs should be used for neuropathic pain:


1. Antiepileptics
2. Steroids
3. Antidepressants
4. Antibiotics

36.23 What are the clinical signs of amyotrophic lateral sclerosis


1. Generalized motor weakness with atrophies
2. Scars after multiple burns
3. Fasciculations across the back and in all limbs
4. Dissociate segmental disorder of sensation

36.24 Which symptoms are characteristic for dementia:


1. Congenital impairment of cognitive functions
2. Progressive impairment of cognitive functions
3. Impairment of one cognitive function
4. Acquired impairment of cognitive functions

36.25 A patient has dissociated cape-like loss of pain and temperature sensation along the trunk and arms.
Proprioreception is preserved. Where is the lesion:
1. Posterior horn
2. Tractus spinothalamicus
3. Posterior root
4. Commissura anterior
lsmusis.lsmuni.lt/Klausimai/Spausdinti?Length=0?Kalba=EN&KategorijaId=138&Kalbos_input=EN&Kalbos=EN&KategorijaEn_input=Neurology&Kate… 3/11
3/27/2019 LSMUSIS
36.26 Which sign (-s) belong to meningeal irritation:
1. Optic oedema
2. Paresis
3. Babinski sign
4. Kernig’s sign

36.65 What findings are characteristic for thrombotic type of ischemic stroke:
1. Relatively slow development of focal symptoms without impairment of consciousness
2. Atrial fibrillation
3. Previous transient ischaemic attacks
4. Acute beginning, impairment of consciousness, vomiting

IV type tasks. Choose only one best answer

36.27 A 28 year-old woman presents with numbness and tingling in her feet. She was seen in the emergency
department, and after no neurological abnormalities were found, she was reassured and discharged home.Two
days later she returns, unable to walk. She complains of numbness and tingling from her toes to just above her
knees, and also affecting her hands. On examination, she has distal more than proximal weakness in the lower
extremities and subtle weakness in her hands. Ankle and patellar reflexes are absent. She recalls having a viral
illness a couple ow weeks ago. Which of the following is the most likely diagnosis:
A. Multiple sclerosis
B. Stroke
C. Myelopathy
D. Guillain-Barre syndrome

36.28 A 28 year-old woman presents with numbness and tingling in her feet. She was seen in the emergency
department, and after no neurological abnormalities were found, she was reassured and discharged home.Two
days later she returns, unable to walk. She complains of numbness and tingling from her toes to just above her
knees, and also affecting her hands. On examination, she has distal more than proximal weakness in the lower
extremities and subtle weakness in her hands. Ankle and patellar reflexes are absent. She recalls having a viral
illness a couple ow weeks ago.What is the cause of this disease:
A. Autoimmunological reation
B. Endogenous intoxication
C. Thrombosis
D. Autosomic dominant inheritance

36.29 A 31 years old woman complains of the weakness in her legs, ataxic gait. One year ago she had and
episode of double vision which disturbed her reading and writing. The ophthalmologs refered her to neurologist,
but she did not visit him as double vision disappeared soon. Neurological examination: horizontal nystagmus,
shallow nasolabial fold on the right, deviation of the tongue to the left. Intensional tremor and dyscoordination.
Ataxia, spastic gate, urination difficulties. No impairment of sensation. Exagerated reflexes in legs, bilateral
Babinski sign. What is the most probable diagnosis:
A. Myelitis
B. Multiple sclerosis
C. Amyotrophic lateral sclerosis
D. Transitory ischaemic attack

36.30 A 31 years old woman complains of the weakness in her legs, ataxic gait. One year ago she had and
episode of double vision which disturbed her reading and writing. The ophthalmologs refered her to neurologist,
but she did not visit him as double vision disappeared soon. Neurological examination: horizontal nystagmus,
shallow nasolabial fold on the right, deviation of the tongue to the left. Intensional tremor and dyscoordination.
Ataxia, spastic gate, urination difficulties. No impairment of sensation. Exagerated reflexes in legs, bilateral
Babinski sign. Are ophtalmological symptoms characteristic for this disease:
A. Yes
B. No

36.31 A 31 years old woman complains of the weakness in her legs, ataxic gait. One year ago she had and
episode of double vision which disturbed her reading and writing. The ophthalmologs refered her to neurologist,
but she did not visit him as double vision disappeared soon. Neurological examination: horizontal nystagmus,
shallow nasolabial fold on the right, deviation of the tongue to the left. Intensional tremor and dyscoordination.
Ataxia, spastic gate, urination difficulties. No impairment of sensation. Exagerated reflexes in legs, bilateral
Babinski sign. Are cerebellar lesions characteristic to this disease:
lsmusis.lsmuni.lt/Klausimai/Spausdinti?Length=0?Kalba=EN&KategorijaId=138&Kalbos_input=EN&Kalbos=EN&KategorijaEn_input=Neurology&Kate… 4/11
3/27/2019 LSMUSIS
A. Yes
B. No

36.32 A 31 years old woman complains of the weakness in her legs, ataxic gait. One year ago she had and
episode of double vision which disturbed her reading and writing. The ophthalmologs refered her to neurologist,
but she did not visit him as double vision disappeared soon. Neurological examination: horizontal nystagmus,
shallow nasolabial fold on the right, deviation of the tongue to the left. Intensional tremor and dyscoordination.
Ataxia, spastic gate, urination difficulties. No impairment of sensation. Exagerated reflexes in legs, bilateral
Babinski sign. Is the cognitive impairment characteristic for this disease:
A. Yes
B. No

36.33 A 58 year-old man who has not typically get headaches presents to your office. He has a history of
melanoma, which had been successfully treated. His headache has been worsening over the last month and has
been constant. The pain is a deep ache around the left temporal and left frontal regions. He admits to mild
intermittent low-grade fever and slight weight loss. He described one episode in which he had some trouble seeing
out the left eye. He is worried because his mother had “some type of brain tumor”. Which of following is not
considered a “red flag”in evaluation of this headache:
A. Age
B. Family history of brain tumor
C. Fever
D. Patient's history of malignancy

36.34 A 58 year-old man who has not typically get headaches presents to your office. He has a history of
melanoma, which had been successfully treated. His headache has been worsening over the last month and has
been constant. The pain is a deep ache around the left temporal and left frontal regions. He admits to mild
intermittent low-grade fever and slight weight loss. He described one episode in which he had some trouble seeing
out the left eye. He is worried because his mother had “some type of brain tumor”. On further questioning, he
mentions that over the last 3 weeks he has been experiencing acramp in his jaw while chewing and talking. That is
the most likely diagnosis that should be first considered:
A. Brain tumor
B. Cluster headache
C. Temporal arteritis
D. Tension-type headache

36.35 A man aged 38 had acute pain in the lumbosacral region while working in the garden. He was forced to
remain in fixed flexed position with no possibility to change it. The patient is lying in bed with hands and legs
flexed, every movement of the trunk causes pain; paravertebral muscles strained, Lasegue sign positive bilaterally,
no sensory disturbance, urination and defecation not impaired. What is the diagnosis:
A. Radiculopathy
B. Medullar stroke
C. Lumboischialgia
D. Acute lumbalgia

36.36 A man aged 38 had acute pain in the lumbosacral region while working in the garden. He was forced to
remain in fixed flexed position with no possibility to change it. The patient is lying in bed with hands and legs
flexed, every movement of the trunk causes pain; paravertebral muscles strained, Lasegue sign positive bilaterally,
no sensory disturbance, urination and defecation not impaired. Treatment options:
A. NSAIDs
B. Opioids
C. Antiepileptics
D. Miorelaxants.
E. Complex therapy seeking appropriate analgesia and early active rehabilitation.

36.37 A 42-year-old female complains of severe pain (8 score in NRS) in the left neck radiating to the arm and first
two fingers and, lasting intermittently for about 5 years. Now there is exacerbation of disease, it is more severe at
night. On examination: limited movements of cervical part of the spine, paravertebral muscle strain, muscle
strength in the right hand - 4 scores, muscle atrophy absent, decreased carporadial reflex hypoesthesia in the
thumb. Which of the following is the most likely diagnosis:
A. Painful C6 radiculopathy
B. Cervicobrachialgia
C. Medianus neuropathy
lsmusis.lsmuni.lt/Klausimai/Spausdinti?Length=0?Kalba=EN&KategorijaId=138&Kalbos_input=EN&Kalbos=EN&KategorijaEn_input=Neurology&Kate… 5/11
3/27/2019 LSMUSIS
D. Shoulder plexopathy

36.38 A 42-year-old female complains of severe pain (8 score in NRS) in the left neck radiating to the arm and first
two fingers and, lasting intermittently for about 5 years. Now there is exacerbation of disease, it is more severe at
night. On examination: limited movements of cervical part of the spine, paravertebral muscle strain, muscle
strength in the right hand - 4 scores, muscle atrophy absent, decreased carporadial reflex hypoesthesia in the
thumb. Which of the following tests should be obtained to ensure adequate care:
A. X- ray of cervical spine
B. Myelography
C. Cervical spine MRI
D. Shoulder ultrasonography

36.39 A 30-year old woman complains of 4 weeks duration pain in the right buttock , which irradiates to the lateral
surface of the legs down to the big toe, weakness in the feet (“dropping feet”). The pain intermittently lasts for 2
years. This exacerbation is related to a long-lasting trip in a sitting position. On examination: asthenic body
structure, limitation of vertebral movements forward and to the left. Scoliosis I0 (to the right), painful palpation of
L4-5 segment. Lasegue sign of the right at the angle of 600. Extension (dorsal flexion) of the left foot impossible.
Patellar and Achilles tendon reflexes are symmetrical, hypoesthesia on the dorsal medial surface of the left foot
and toe. What is the diagnosis:
A. Lumbalgia
B. N. ischiadicus neuropathy
C. Sacral plexopathy
D. Radiculopathy L5
E. Polyneuropathy

36.40 A 30-year old woman complains of 4 weeks duration pain in the right buttock , which irradiates to the lateral
surface of the legs down to the big toe, weakness in the feet (“dropping feet”). The pain intermittently lasts for 2
years. This exacerbation is related to a long-lasting trip in a sitting position. On examination: asthenic body
structure, limitation of vertebral movements forward and to the left. Scoliosis I0 (to the right), painful palpation of
L4-5 segment. Lasegue sign of the right at the angle of 600. Extension (dorsal flexion) of the left foot impossible.
Patellar and Achilles tendon reflexes are symmetrical, hypoesthesia on the dorsal medial surface of the left foot
and toe. Which investigation is the best for confirmation of diagnosis:
A. Spondylography (X-ray)
B. Electroneuromyography (ENMG)
C. MRI of thoracolumbal spine
D. Myelography

36.41 A 64-yearold man has ischaemic heart disease with arrythmia. He suddenly developed weakness in the left
extremities. On examination: consciousness and orientation preserved heart rate arrhythmic, 120 beats/ min., a.
radialis pulse - 96 beats/min, arterial blood pressure 140/90 mmHg; movements of eyeballs limited to the left, the
right nasolabial fold is shallow. No movements is the left hand, movements of the left leg are minimal, muscle
hypotonia and hyporeflexia in the left extremities, Babinski sign positive on the left, hemihypesthesia, hemianopsia
on the left. What is the diagnosis:
A. Myocardial infarction
B. Hemorrhagic stroke
C. Ischaemic stroke
D. Vascular encephalopathy

36.42 A 64-yearold man has ischaemic heart disease with arrythmia. He suddenly developed weakness in the left
extremities. On examination: consciousness and orientation preserved heart rate arrhythmic, 120 beats/ min., a.
radialis pulse - 96 beats/min, arterial blood pressure 140/90 mmHg; movements of eyeballs limited to the left, the
right nasolabial fold is shallow. No movements is the left hand, movements of the left leg are minimal, muscle
hypotonia and hyporeflexia in the left extremities, Babinski sign positive on the left, hemihypesthesia, hemianopsia
on the left. Which investigation is the most important one:
A. Craniogram (scull radiography)
B. Neuroophthalmologic investigation
C. Doppler investigation
D. CT scan of the brain

36.43 A 54-year old man, general manager of a company, had a sudden severe headache and lost his
consciousness. Recently he has had some stressfull problems. Examination: unconscious, no reaction to external

lsmusis.lsmuni.lt/Klausimai/Spausdinti?Length=0?Kalba=EN&KategorijaId=138&Kalbos_input=EN&Kalbos=EN&KategorijaEn_input=Neurology&Kate… 6/11
3/27/2019 LSMUSIS
stimuli. Arterial blood pressure 240/130 mmHg, heart rate 105 beats/min, arrhythmic, eyeballs “floating”, symptom
of “sail” on the right, Babinski sign bilaterally. What is the most probable diagnosis:
A. Ischaemic stroke
B. Vascular encephalopathy
C. Hemorrhagic stroke
D. Hypertensive crisis

36.44 A 54-year old man, general manager of a company, had a sudden severe headache and lost his
consciousness. Recently he has had some stressfull problems. Examination: unconscious, no reaction to external
stimuli. Arterial blood pressure 240/130 mmHg, heart rate 105 beats/min, arrhythmic, eyeballs “floating”, symptom
of “sail” on the right, Babinski sign bilaterally. Which investigation is most helpful in making diagnosis immediately:
A. Electroencephalography
B. CT scan of the brain
C. Lumbal punction
D. MRI investigation

36.45 A 22-year old female being in a night club started a severe sudden headache (“blow”) in occipital region and
lost consciousness for a while. After recovery, she complained of headache, had nausea and vomiting. On
examination: the patient is conscious, orientated, pale. Arterial blood pressure 150/90 mmHg, heart rate 100
beats/min, rhythmic. Cranial nerves - without changes, pathological reflexes absent, no sensory disturbance.
Nuchal rigidity and bilateral Kernig’s sign are present. What is the most probable diagnosis:
A. Meningitis
B. Encephalitis
C. Subarachnoid hemorrhage
D. Epileptic seizures

36.46 A 22-year old female being in a night club started a severe sudden headache (“blow”) in occipital region and
lost consciousness for a while. After recovery, she complained of headache, had nausea and vomiting. On
examination: the patient is conscious, orientated, pale. Arterial blood pressure 150/90 mmHg, heart rate 100
beats/min, rhythmic. Cranial nerves - without changes, pathological reflexes absent, no sensory disturbance.
Nuchal rigidity and bilateral Kernig’s sign are present. Which investigation should be performed immediately:
A. Magnetic resonance imaging angiography
B. Lumbar punction
C. Electroencephalography
D. CT scan of the brain

36.47 Following a mild upper respiratory flu-like illness, a 47 year old male develops a rapidly ascending paralysis.
A week later he is hospitalized and requires intubation with mechanical ventilation. Lumbar puncture yields clear
CSF under normal pressure with a slightly elevated protein, but no red blood cells and only 3 mononuclear cells.
He gradually improves over the next couple of weeks. He most likely has:
A. Myelitis
B. Acute inflammatory demyelinating polyneuropathy (Guillain-Barre syndrome)
C. Multiple sclerosis
D. Stroke of the spinal cord

36.48 Following a mild upper respiratory flu-like illness, a 47 year old male develops a rapidly ascending paralysis.
A week later he is hospitalized and requires intubation with mechanical ventilation. Lumbar puncture yields clear
CSF under normal pressure with a slightly elevated protein, but no red blood cells and only 3 mononuclear cells.
He gradually improves over the next couple of weeks. What treatment should be used in intensive care unit?
A. Corticosteroids
B. Cyclophosphamide
C. Azathioprine
D. Plasma exchange

36.49 A 75-year old farmer has been working hard yesterday. In the morning he started to feel unsteady, and
weakness in his left extremities. In 2 hours he admitted to emergency department. Last month he had transient
numbness in the left hand and leg. Examination: conscious, well oriented, heart rate 84 beats/min, rhythmic,
arterial blood pressure 1801000 mmHg. Limitation of eyeball movements to the left, shallow left nasolabial fold,
hemiplegia on the left, asymmetry of tendon reflexes (exaggerated on the left), Babinski sign on the left. What is
the most probable diagnosis:
A. Hemorrhagic stroke
B. Ischemic stroke
lsmusis.lsmuni.lt/Klausimai/Spausdinti?Length=0?Kalba=EN&KategorijaId=138&Kalbos_input=EN&Kalbos=EN&KategorijaEn_input=Neurology&Kate… 7/11
3/27/2019 LSMUSIS
C. Vascular encephalopathy
D. Subarachnoid hemorrhage

36.50 75-year old farmer has been working hard yesterday. In the morning he started to feel unsteady, and
weakness in his left extremities. In 2 hours he admitted to emergency department. Last month he had transient
numbness in the left hand and leg. Examination: conscious, well oriented, heart rate 84 beats/min, rhythmic,
arterial blood pressure 1801000 mmHg. Limitation of eyeball movements to the left, shallow left nasolabial fold,
hemiplegia on the left, asymmetry of tendon reflexes (exaggerated on the left), Babinski sign on the left. Treatment
options:
A. Antihypertensive treatment
B. Hemodilution
C. Diuretics
D. Thrombolysis

36.51 A 67-year old man was taken to the doctor by his wife. She told that her husband has changed in last few
months, he became slow, his face lost expression. He has difficulty with walking and balance, getting out from the
chair and turn in the bed. Examination revealed resting tremor at the rate of 3/sec., rigidity in the extremities, ‘cog-
wheel’ phenomenon. What disease will you suspect:
A. Familial (essential) tremor
B. Encephalitis
C. Alzheimer’s disease
D. Parkinson’s disease

36.52 A 67-year old man was taken to the doctor by his wife. She told that her husband has changed in last few
months, he became slow, his face lost expression. He has difficulty with walking and balance, getting out from the
chair and turn in the bed. Examination revealed resting tremor at the rate of 3/sec., rigidity in the extremities, ‘cog-
wheel’ phenomenon. Where is the site of lesion:
A. Cerebellum
B. Nucleus caudatus
C. Motor nucleus of brainstem
D. Substantia nigra

36.53 A 67-year old man was taken to the doctor by his wife. She told that her husband has changed in last few
months, he became slow, his face lost expression. He has difficulty with walking and balance, getting out from the
chair and turn in the bed. Examination revealed resting tremor at the rate of 3/sec., rigidity in the extremities, ‘cog-
wheel’ phenomenon. What drugs should not be used:
A. Anticholinergic
B. Anticholinesterase agents
C. L-dopa derivates
D. Dopamine agonists

36.54 A man, aged 27, woke up in the morning and could not move the left part of his face. Yesterday he started to
feel pain behind his left ear. He also told that the noise from the left side seemed too loud and unpleasant to him.
He had no previous history of hypertension and cerebrovascular disease. On examination: complete facial palsy
on the left side, involving both the upper and lower portions of the face; he excessive tearing from his left eye.
Taste is reduced on the left anterior portion of the tongue. No other abnormalities on neurological examination.
Where is the lesion:
A. In the brainstem
B. At the foramen stylomastoideum
C. In the canalis nervi facialis
D. In the motor nucleus of facial nerve

36.55 A man, aged 27, woke up in the morning and could not move the left part of his face. Yesterday he started to
feel pain behind his left ear. He also told that the noise from the left side seemed too loud and unpleasant to him.
He had no previous history of hypertension and cerebrovascular disease. On examination: complete facial palsy
on the left side, involving both the upper and lower portions of the face; he excessive tearing from his left eye.
Taste is reduced on the left anterior portion of the tongue. No other abnormalities on neurological examination.
What type of paralysis he has:
A. Peripheral (lower motor neuron damage)
B. Central (upper motor neuron damage)

lsmusis.lsmuni.lt/Klausimai/Spausdinti?Length=0?Kalba=EN&KategorijaId=138&Kalbos_input=EN&Kalbos=EN&KategorijaEn_input=Neurology&Kate… 8/11
3/27/2019 LSMUSIS
36.56 25.72. A 26-yearold man complained of “metallic” taste in his mouth, then he lost his consciousness,
generalized convulsions occurred, which lasted 10 minutes, after he did not respond to external stimuli. 5 minutes
later convulsions started again and lasted for 25 minutes. What drug should be administered first of all:
A. Lidocain intravenously
B. Phenytoin intramusculary
C. Diazepam intravenously
D. Adrenalin subcutaneusly
E. Phenobarbital intravenously

36.57 A 51 old man suddenly developed right hemiplegia, dyspnea, inability to speak, his face became red. 2 days
before he had fever, complained of pain in the right shoulder followed by syncopal episode. Pulmonary embolism
was diagnosed and heparin was administered. Examination demonstrated slow and poor spontaneous speech but
good comprehension.. A CT scan performed on the first day of the disease was normal. One month later he
developed clonic convulsions of the right hand without impairment of consciousness. What is the most probable
cause of hemiplegia and language impairment:
A. Subarachnoid hemorrhage
B. Embolism of cerebral arteries
C. Hemorrhage in the region of internal capsule
D. Acute spasm of cerebral arteries
E. Neuroinfection

36.58 A 51 old man suddenly developed right hemiplegia, dyspnea, inability to speak, his face became red. 2 days
before he had fever, complained of pain in the right shoulder followed by syncopal episode. Pulmonary embolism
was diagnosed and heparin was administered. Examination demonstrated slow and poor spontaneous speech but
good comprehension.. A CT scan performed on the first day of the disease was normal. One month later he
developed clonic convulsions of the right hand without impairment of consciousness. How would you call the
disturbance of language in this case:
A. Brocca (motor) aphasia
B. Wernicke (sensory) aphasia
C. Bulbar dysarthria
D. Pseudobulbar dysarthria
E. Motor alalia

36.59 A 51 old man suddenly developed right hemiplegia, dyspnea, inability to speak, his face became red. 2 days
before he had fever, complained of pain in the right shoulder followed by syncopal episode. Pulmonary embolism
was diagnosed and heparin was administered. Examination demonstrated slow and poor spontaneous speech but
good comprehension.. A CT scan performed on the first day of the disease was normal. One month later he
developed clonic convulsions of the right hand without impairment of consciousness. How would you call the
seizures:
A. Generalized tonic-clonic
B. Simple partial
C. Complex partial
D. Absences
E. Choreoathetosis

36.60 A 32-old woman presents to the clinic complaining of double vision of 1-month duration. She also reports
her speech slurs when she talks for prolonged periods of tome, and her eyelids start to droop towards the end of
the day.On examination, she has bilateral ptosis that worsens with sustained upward gaze, she unable to hold air
in her mouth against resistance, and her neck flexors are weak. Which of the following is most likely patients
disease:
A. Myotonic dystrophy
B. Myopathy
C. Myasthenia gravis
D. Proximal acute polyneuropathy
E. Paraneoplastic Lambert-Eaton syndrome

36.61 A 32-old woman presents to the clinic complaining of double vision of 1-month duration. She also reports
her speech slurs when she talks for prolonged periods of tome, and her eyelids start to droop towards the end of
the day.On examination, she has bilateral ptosis that worsens with sustained upward gaze, she unable to hold air
in her mouth against resistance, and her neck flexors are weak. What should you perform first of all:
A. Lumbar punction and administration of corticosteroids
B. Electroneuromyography and muscle biopsy
lsmusis.lsmuni.lt/Klausimai/Spausdinti?Length=0?Kalba=EN&KategorijaId=138&Kalbos_input=EN&Kalbos=EN&KategorijaEn_input=Neurology&Kate… 9/11
3/27/2019 LSMUSIS
C. Test of acetylcholine-esterase inhibitor injection or decremental response in electromyography
D. Call for neurosurgeon

36.62 Eight year-old girl started episodes during which she would stop ongoing acitivities and would have a blank
stare, would not respond to calling by name, seems as if would be thinking of something else. After some moment
she would return to previous activity. A few months ago parents got divorced, the girl became very sensitive. What
is the most probable diagnosis:
A. Attention deficit hyperactivity disorder
B. Depression
C. Cochlear neuritis
D. Epilepsy
E. Somatoform disorder

36.63 25.79. A 53 year-old woman was found comatose in bed. Previously she was rather healthy except mild
arterial hypertension. Recently she had some problems with her work, but did not seem depressed. Examination:
deep coma, woman unresponsive to external stimuli. Blood pressure 90/50 mmHg, pulse irregular 110 /min;
respiration regular, 20/min. Meningeal signs are negative. Pupils are small and not reacting to light. There were no
eye movements with doll’s head maneuver. Tendon reflexes are absent. Babinski sign is negative. Cerebrospinal
fluid is normal. What is the most probable cause of this condition:
A. Atonic epileptic seizure
B. Meningoencephalitis
C. Intoxication
D. Subarachnoid hemorrhage
E. Ischemic stroke

36.64 A 24year-old man complained of slowly progressive weakness in his feet since adolescence. His father has
a similar problem, by the age of 60 he still walks independently but with big difficulty althoug never asked for
consultation. Examination revealed muscle atrophy and moderate muscle weakness in the lower limbs
predominantly in the calves (“champagne-bottle-legs”). Patelar reflex is diminished, ankle jerk is absent bilateraly.
Sensory deficit to touch bilaterally in the feet. Patient has prominent deformity of feet (pes cavus). What is the
most probable diagnosis:
A. Myotonia
B. Charcot - Marie - Tooth polyneuropathy
C. Duchene myopathy
D. Spinal amyotrophy

36.66 A 28-year old woman complains of paroxysmal headaches 1-2 times per month, usually related to menses.
Before the start of headache she feels shimmering lights. Headaches are severe, unilateral, lasting for 1-2 hours,
followed by vomiting. Neurological examination without pathological findings.Which is the most probable diagnosis:
A. N. trigeminus neuralgia
B. Intracranial hypertension
C. Migraine
D. Transientischaemic attacks

36.67 A 28-year old woman complains of paroxysmal headaches 1-2 times per month, usually related to menses.
Before the start of headache she feels shimmering lights. Headaches are severe, unilateral, lasting for 1-2 hours,
followed by vomiting. Neurological examination without pathological findings.What treatment should be given:
A. Carbamazepine
B. Triptans
C. Diuretics
D. Acetylsalicylic acid 100 mg

36.1 - A 36.2 - B 36.3 - B 36.4 - B 36.5 - B 36.6 - A 36.7 - C 36.8 - D


36.9 - C 36.10 - A 36.11 - D 36.12 - A 36.13 - C 36.68 - B
36.14 36.15 36.16
1-E 1-B 1-A
2-C 2-A 2-C
3-D 3-C 3-B
4-B 4-D 4-D
5-A 5-E
36.17 36.18 36.19 36.20 36.21 36.22 36.23 36.24
lsmusis.lsmuni.lt/Klausimai/Spausdinti?Length=0?Kalba=EN&KategorijaId=138&Kalbos_input=EN&Kalbos=EN&KategorijaEn_input=Neurology&Kat… 10/11
3/27/2019 LSMUSIS
1 1 1 1 1 1 1 2
2 2 2 2 2 3 3 4
3 3 3 3 3
4 4 4
36.25 36.26 36.65
4 4 1
3
36.27 - D 36.28 - A 36.29 - B 36.30 - A 36.31 - A 36.32 - A 36.33 - A 36.34 - A
36.35 - D 36.36 - E 36.37 - A 36.38 - C 36.39 - D 36.40 - C 36.41 - C 36.42 - D
36.43 - C 36.44 - B 36.45 - C 36.46 - D 36.47 - B 36.48 - D 36.49 - B 36.50 - D
36.51 - D 36.52 - D 36.53 - B 36.54 - C 36.55 - A 36.56 - C 36.57 - B 36.58 - A
36.59 - B 36.60 - C 36.61 - C 36.62 - D 36.63 - C 36.64 - B 36.66 - C 36.67 - B

lsmusis.lsmuni.lt/Klausimai/Spausdinti?Length=0?Kalba=EN&KategorijaId=138&Kalbos_input=EN&Kalbos=EN&KategorijaEn_input=Neurology&Kat… 11/11
3/27/2019 LSMUSIS

Ophtholmology
I type tasks. Choose only one best answer

17.1 Mature monocular cataract was diagnosed for a newborn baby, your choice:
A. Cataract surgery within 7-8 weeks after birth
B. Cataract surgery at the age of 6-8 months
C. Cataract surgery not recommended because of the possible unfavorable outcome
D. Infants and children are monitored closely, cataract surgery performed at the age of 1 year

17.2 Convergent deviation in a six months old infant:


A. At this age it is normal
B. This type of strabismus is not treated, it will resolve spontaneously up to 12 years of age
C. Immediate ophthalmologist consultation – treatment should be started immediately
D. It is pseudostrabismus because binocular vision development is not complete

17.3 Prophylaxis of accommodative strabismus:


A. The extraocular muscles training
B. Convergence training
C. Accommodation training
D. Complete hyperopic correction

17.4 The main symptoms of myopia:


A. Decreased visual acuity; near objects are blurry and distant objects are clear
B. Decreased visual acuity; distant objects are blurry and near objects are clear
C. Decreased visual acuity; objects are blurry in all distances
D. Tiredness in the eyes, headache, flickering

17.5 The management of open globe injury:


A. Avoid instillation of eye drops and ointments
B. Keep the patient fasted – nothing perorally
C. Tetanus prophylaxis should be administered
D. All answers are correct

17.6 Which chemical penetrates more rapidly and deeply:


A. Acids
B. Irritants (pepper gas, petrol, mace)
C. Alkali
D. Similarly all of them

17.7 The main symptoms of the corneal syndrome:


A. Photophobia, tearing, blepharospasm
B. Corneal vascularization
C. Corneal precipitates
D. Hyperemia of the iris

17.8 Which ophthalmic pathology is considered to be a true (absolute) emergency:


A. Rhegmatogenous retinal detachment
B. Central retinal vein occlusion
C. Chemical burn
D. Vitreous hemorrhage

17.9 The aqueous humor is produced by:


A. Canal of Schlemm
B. Zonular fibers
C. Ciliary processes
D. A flat part of a ciliary body

17.10 Acute bacterial conjunctivitis is treated with:


A. Systemic broad-spectrum antibiotics
B. Topical broad-spectrum antibiotics
C. Topical broad-spectrum antibiotics
D. Topical non-steroid anti-inflammatory drugs
lsmusis.lsmuni.lt/Klausimai/Spausdinti?Length=0?Kalba=EN&KategorijaId=119&Kalbos_input=EN&Kalbos=EN&KategorijaEn_input=Ophtholmology&… 1/9
3/27/2019 LSMUSIS
E. Cycloplegic eye drops

17.11 Types of age-related cataract:


A. Primary, nuclear, hypermature
B. Nuclear, cortical, subcapsular
C. Primary, secondary, late onset
D. Anterior, posterior, intermediate

17.12 Glaucoma progression is defined as:


A. Detected intraocular pressure fluctuations within 15-17 mmHg
B. Increasing optic disc cup and visual field defects
C. Optic disc and visual field damage is stable
D. Decreased optic disc cupping and no visual field defects

17.13 Which investigation helps us to determine the type of glaucoma:


A. Tonometry
B. Visual field testing
C. Gonioscopy
D. Ophthalmoscopy

17.14 Common clinical presentation of age-related cataract:


A. Reduced visual acuity
B. Refractive changes in the eye
C. Obscured ocular fundus view
D. All answers are correct

17.15 Linear - branching (dendritic) epithelial infiltration or ulceration of cornea is common for:
A. Bacterial keratitis
B. Dry eye disease
C. Adenoviral keratoconjunctivitis
D. Herpetic keratitis

17.16 Clinical signs of exudative (wet) age-related macular degeneration:


A. Reduced visual acuity, metamorphopsias, scotoma, rapid progression is possible
B. Subretinal hemorrhage
C. Subretinal exudation
D. All answers are correct

17.17 Blood in vitreous is defined as:


A. Hyphema
B. Hypopyon
C. Hemophthalmos
D. Hemianopsia

17.18 Subretinal fluid in cases of retinal detachment accumulates between:


A. Retinal pigment epithelium and choroid
B. Retinal pigment epithelium and choroid
C. Different layers of neurosensory retina
D. Vitreous and retinal internal limiting membrane

17.19 Types of clinical refraction:


A. Hemeralopia, nyctalopia
B. Myopia, hyperopia, astigmatism, emmetropia
C. Presbyopia, amblyopia
D. Pseudomyopia, asthenopia

17.20 Which examination should not be used for detection and localization of metallic intraocular foreign body:
A. Ultrasonography
B. Computed tomography
C. X-ray examination
D. Magnetic resonance imaging

17.21 Hypotony after ocular contusion is related to:


lsmusis.lsmuni.lt/Klausimai/Spausdinti?Length=0?Kalba=EN&KategorijaId=119&Kalbos_input=EN&Kalbos=EN&KategorijaEn_input=Ophtholmology&… 2/9
3/27/2019 LSMUSIS
A. Intraretinal hemorrhage
B. Hyphema
C. Traumatic glaucoma
D. Cyclodialysis

17.22 Ophthalmoscopic signs of central retinal artery occlusion:


A. Superficial, dot and blot, and/or deep retinal hemorrhages ("blood and thunder appearance"), cotton-wool
spots
B. Cherry-red spot in the central part of the retina, thinned retinal arterioles, segmentation of blood column
in the retinal arterioles, emboli in arterioles might be visible
C. Massive vitreous hemorrhage, macular edema, extensive drusen
D. Microaneurisms, “cotton wool” spots and hard exudates

II type tasks. For each numbered item,selct the one lettered heading that is most closely asssciated with it

17.23 Choose presbyopic optical correction (glasses) for an emmetropic person according to the age:
1 - 40 years old
2 - 45 years old
3 - 50 years old
4 - 60 years old
A. +1,5D
B. +2,0D
C. +1,0D
D. +3,0D

17.24 Choose characteristic signs for each pathology:


1 - Endocrine ophthalmopathy
2 - Orbital cellulitis
A. Painful eye movements, no signs of conjunctival irritation or eyelid swelling
B. Retraction of the upper eyelid, exophthalmos, restricted eye motility
C. Eyelid swelling, erythema, conjunctival chemosis and injection, restricted and painful eye motility, fever

17.25 Choose the main cause for each type of retinal detachment:
1 - Rhegmatogenous
2 - Tractional
3 - Exudative
A. Subretinal fluid transudation
B. Vitreoretinal traction
C. Vitreoretinal traction

17.26 Choose characteristic symptoms for each pathology


1 - Angle-closure glaucoma
2 - Open-angle glaucoma
3 - Congenital glaucoma
A. Enlarged and corneal diameter globe, corneal syndrome
B. Usually asymptomatic
C. Acute onset, frontal headache, nausea, blurred vision, colored halos around lights

17.27 What test is performed for examination of:


1 - Color vision
2 - Visual acuity
3 - Measurement of the quantity of tear fluid (tear secretion)
A. Seidel test
B. Schirmer‘s test
C. Snellen chart
D. Ishihara pseudo-isochromatic plates

17.28 Choose a method of treatment for each pathology


1 - Exudative age-related macular degeneration
2 - Mature age-related cataract
3 - Secondary cataract
A. Nd: YAG laser posterior capsulotomy

lsmusis.lsmuni.lt/Klausimai/Spausdinti?Length=0?Kalba=EN&KategorijaId=119&Kalbos_input=EN&Kalbos=EN&KategorijaEn_input=Ophtholmology&… 3/9
3/27/2019 LSMUSIS
B. Intravitreal injections of anti-vascular endothelial growth factor drugs (anti-VEGF)
C. Surgery - phacoemulsification

17.29 Match characteristic findings with each disease:


1 - Bacterial keratitis
2 - Dendritic keratitis
3 - Lagophthalmic keratitis
A. Corneal central deep infiltrate with superficial ulceration
B. Decreased corneal sensation
C. Punctate epithelial defects in the lower one-third, inadequate blinking, and closure of the eyelids

17.30 The signs common for those lesions:


1 - Bacterial corneal ulcer
2 - Dendritic keratitis
3 - Disciform keratitis
4 - Lagophthalmic keratitis
A. Corneal infiltrates
B. Optic nerve atrophy, central scotoma in the visual field
C. Fever, pain in eyelids and orbit
D. Inadequate closure of the eyelids, corneal epithelial defects, stromal ulceration

17.31 Choose a proper treatment for each disease:


1 - Bacterial keratitis
2 - Adenoviral keratoconjunctivitis
3 - Dendritic keratitis
A. Topical broad-spectrum antibiotics
B. No specific treatment
C. Antiviral drugs

17.32 Choose the location of all the glands mentioned below:


1 - Accessory lacrimal glands
2 - Meibomian glands
3 - Glands of Zeis and Moll
A. Tarsus of the eyelid
B. Fornix of the conjunctiva
C. The ciliary margin of the eyelid

17.33 Match ophthalmoscopic findings with each pathology


1 - Narrow retinal arteries, Gvist, and Salus-Gunn signs, small white spot around the retinal arteries,
hemorrhages, "macula star"
2 - Dilated retinal veins, microaneurysms, hard exudates and variuos hemorrhages
3 - Dilated retinal veins, microaneurysms, hard and cotton-wool exudates, the new vessels growing on the
retina and the optic nerve head, bands of connective tissue growing into vitreous, retinal hemorrhages,
hemophthalmus
A. Non-proliferative diabetic retinopathy
B. Proliferative diabetic retinopathy
C. Hypertonic retinopathy

III type tasks. For each question there is one or more correct answers:
A – if correct answers are 1,2,3
B – if correct answers are 1 and 3
C – if correct answers are 2 and 4
D – if correct answer is 4
E – if correct are all answers above

17.34 Stereo vision test:


1. Titmus
2. Maddox
3. Lang
4. Four Dots

17.35 Symptoms of presbyopia:


lsmusis.lsmuni.lt/Klausimai/Spausdinti?Length=0?Kalba=EN&KategorijaId=119&Kalbos_input=EN&Kalbos=EN&KategorijaEn_input=Ophtholmology&… 4/9
3/27/2019 LSMUSIS
1. Near objects are blurry
2. Near objects become clearer when held further away
3. Ilgiau paskaičius atsiranda nuovargis, kartais skauda galvą
4. Halos around the source of light

17.36 External hordeolum (stye) is:


1. Bacterial inflammation of the conjunctiva
2. Outward turning of the eyelid
3. Eyelid swelling
4. Acute inflammation (abscess) of the gland of Zeiss on the lid margin

17.37 Visual field defects characteristic for papilledema:


1. Narrowing of the visual field from the nasal side
2. Concentric narrowing of the visual field
3. Narrowing of the visual field from the temporal side
4. Broadening of the blind spot

17.38 Symptoms of retrobulbar neuritis:


1. Exophthalmos, flickers in the eyes
2. Increased intraocular pressure
3. Binasal hemianopsia
4. Loss of vision, decreased color vision, central scotoma, orbital pain with eye movements

17.39 Causes of secondary open-angle glaucoma:


1. Pigment dispersion syndrome
2. Pseudoexfoliation syndrome
3. Eye trauma
4. Hypermature cataract

17.40 Contusion cataract is:


1. Morganian
2. Anterior polar
3. Age-related
4. Rosette-shaped

17.41 Signs of acute dacryocystitis:


1. Area over the lacrimal gland is painful
2. Area over the lacrimal gland is red and swelling
3. No external visible signs of inflammation
4. Area over the lacrimal sac is painful, red and swelling

17.42 Symptoms/signs of bacterial corneal ulcer:


1. Red eye, moderate-to-severe ocular pain
2. Photophobia, decreased vision
3. Central greyish and circular infiltrate in the corneal stroma
4. Swelling of the corneal stroma, epithelial defect, mucopurulent discharge
5. Anterior chamber reaction with hypopyon

17.43 Acute management (initial therapeutic strategy) of chemical injury:


1. Immediate irrigation of saline (BSS, ringer lactate solution) using lid retractors or Morgan lens and
infusion system
2. Instillation of topical anesthetics every 15-20 minutes
3. Measurement of the pH of the ocular surface
4. Removal of all solid particles with a cotton-tipped applicator or forceps

17.44 Complications of uveitis:


1. Secondary glaucoma
2. Complicated cataract
3. Cystoid macular edema
4. Age-related macular degeneration

17.45 Uveitis is caused by:


1. Tuberculosis
lsmusis.lsmuni.lt/Klausimai/Spausdinti?Length=0?Kalba=EN&KategorijaId=119&Kalbos_input=EN&Kalbos=EN&KategorijaEn_input=Ophtholmology&… 5/9
3/27/2019 LSMUSIS
2. Toxoplasmosis
3. Ankylosing spondylitis
4. Syphilis

17.46 Characteristic ophthalmoscopic findings of central retinal vein occlusion:


1. Diffuse retinal haemorrhages
2. Cystoid macular edema
3. Dilated, tortuous retinal veins
4. Groups of glistening white dots (“snail track”) in the peripheral retina

17.47 Typical symptoms/signs of central retinal artery occlusion:


1. Painless, acute visual loss
2. Macular drusen, clumps of pigment in the outer retina, and retinal pigment epithelium atrophy
3. Superficial opacification and whitening of the retina in the posterior pole
4. Diffuse retinal hemorrhages in all four quadrants of the retina; dilated, tortuous retinal veins

17.48 Ophthalmoscopic characteristics of nonproliferative diabetic retinopathy:


1. Intraretinal hemorrhages
2. Hard exudates
3. Microaneurysms
4. Vitreous hemorrhage

17.49 Proliferative diabetic retinopathy treatment:


1. Blood sugar level control
2. Topical and oral corticosteroids and angioprotectors
3. Panretinal laser photocoagulation
4. Vitrectomy

17.50 Central retinal vein occlusion is mostly associated with:


1. Primary open-angle glaucoma
2. Diabetes
3. Oral contraceptives
4. Arterial hypertension

IV type tasks. Choose only one best answer

17.51 Mother and her three-year-old child came to consult the ophthalmologist because she noticed that her son
started to squint six months ago. Initially, the convergent misalignment of the right eye was intermittent, but later it
became permanent.What examination (tests) should be performed:
A. Contrast sensitivity, color vision, ocular motility, an angle of deviation, biomicroscopy, retinoscopy
B. Visual acuity, color vision, ocular motility, an angle of deviation, biomicroscopy, ophthalmoscopy
C. Visual acuity, retinoscopy, biomicroscopy, ophthalmoscopy, ocular motility, an angle of deviation
D. Visual acuity, contrast sensitivity, color vision, visual field, an angle of deviation, ocular motility

17.52 Mother and her three-year-old child came to consult the ophthalmologist because she noticed that her son
started to squint six months ago. Initially, the convergent misalignment of the right eye was intermittent, but later it
became permanent.No optic media and fundus pathology was observed. After cycloplegia hyperopia of 6.0 D in
the right eye, and 4.0 D in the left eye was determined. Ocular motility was normal. Primary and secondary angles
of deviation were equal. What is the most probable cause of strabismus:
A. Intraocular pathology
B. VI cranial nerve (n. abducens) palsy
C. Hyperopia, anisometropia
D. Stressful situation

17.53 A sixty-year-old woman noticed, that her left eye became more prominent 6 months ago. She complained of
increasing left eye proptosis and swelling of the eyelids. From case history - the patient didn't suffer from any
infectious diseases, diseases of paranasal sinuses, fever or thyroid diseases. Eye examination results:
exophthalmometry - OD - 15 mm, OS – 22 mm. The left eye motility limited upward, inward and outward, no orbital
pain.The preliminary diagnosis could be:
A. Orbital cellulitis
B. Glaucoma
C. Orbital tumor

lsmusis.lsmuni.lt/Klausimai/Spausdinti?Length=0?Kalba=EN&KategorijaId=119&Kalbos_input=EN&Kalbos=EN&KategorijaEn_input=Ophtholmology&… 6/9
3/27/2019 LSMUSIS
D. Endocrine ophthalmopath

17.54 A sixty-year-old woman noticed, that her left eye became more prominent 6 months ago. She complained of
increasing left eye proptosis and swelling of the eyelids. From case history - the patient didn't suffer from any
infectious diseases, diseases of paranasal sinuses, fever or thyroid diseases. Eye examination results:
exophthalmometry - OD - 15 mm, OS – 22 mm. The left eye motility limited upward, inward and outward, no orbital
pain. What tests could be helpful to prove the diagnosis:
A. Ultrasonography of orbit, computed tomography or magnetic resonance imaging of orbit and brain
B. Anterior segment biomicroscopy
C. External examination and Schirmer's test
D. Ophthalmoscopy and fundus photography

17.55 Paranasal sinusitis was diagnosed for a fifty-five-year-old woman 3 weeks ago. One day ago she noticed
swelling of the eyelids of the right eye and complained of headache, weakness and double vision. Objective data:
The eyelids of the right eye are red, swollen; chemosis and hyperemia of the conjunctiva. Exophthalmometry – OD
– 21 mm, OS -16.5 mm. Proptosis of the right eye, restricted ocular motility in all directions. Body temperature
38.1°C. Blood test: leucocytosis and increased erythrocyte sedimentation rate.The preliminary diagnosis could be:
A. An orbital tumor
B. Orbital cellulitis
C. Endocrine ophthalmopathy
D. Carotid-cavernous sinus fistula

17.56 Paranasal sinusitis was diagnosed for a fifty-five-year-old woman 3 weeks ago. One day ago she noticed
swelling of the eyelids of the right eye and complained of headache, weakness and double vision. Objective data:
The eyelids of the right eye are red, swollen; chemosis and hyperemia of the conjunctiva. Exophthalmometry – OD
– 21 mm, OS -16.5 mm. Proptosis of the right eye, restricted ocular motility in all directions. Body temperature
38.1°C. Blood test: leucocytosis and increased erythrocyte sedimentation rate.What tests could be helpful to
confirm the diagnosis:
A. Ophthalmoscopy and biomicroscopy
B. Computed tomography or magnetic resonance imaging of orbit
C. Probing and irrigation of the lacrimal system.
D. Automated visual field test

17.57 Patient complains of pain in his right eye, right-sided headache, nausea, vomiting, and blurred vision. Slit-
lamp examination: ciliary injection, microcystic corneal edema; shallow anterior chamber and fixed mid-dilated
pupil. The preliminary diagnosis could be:
A. Acute anterior uveitis
B. An acute primary angle-closure glaucoma
C. Endophthalmitis

17.58 Patient complains of pain in his right eye, right-sided headache, nausea, vomiting, and blurred vision. Slit-
lamp examination: ciliary injection, microcystic corneal edema; shallow anterior chamber and fixed mid-dilated
pupil.The most appropriate management is:
A. Topical antibiotics and corticosteroids
B. Topical mydriatic and non-steroid anti-inflammatory agents
C. Systemic hyperosmotic agents and carbonic anhydrase inhibitors, topical β blockers, miotics

17.59 A middle-aged man came for a routine eye check. Objective data: VA-OD=0.4; VA-OS=0.8; Tn OD=28
mmHg; OS=24mmHg. Biomicroscopy: iris atrophy, pigment dispersion in both eyes. Ophthalmoscopy: optic nerve
head sharp-margined, grey, excavated, with a nasal displacement of the vessels in both eyes.What tests should
be performed:
A. Automated visual field test
B. Structural analysis of optic nerve
C. Gonioscopy
D. All mentioned above

17.60 A middle-aged man came for a routine eye check. Objective data: VA-OD=0.4; VA-OS=0.8; Tn OD=28
mmHg; OS=24mmHg. Biomicroscopy: iris atrophy, pigment dispersion in both eyes. Ophthalmoscopy: optic nerve
head sharp-margined, grey, excavated, with a nasal displacement of the vessels in both eyes.According to the
results of the tests, glaucoma was diagnosed. The most appropriate initial treatment is:
A. β blockers
B. Prostaglandin analogs

lsmusis.lsmuni.lt/Klausimai/Spausdinti?Length=0?Kalba=EN&KategorijaId=119&Kalbos_input=EN&Kalbos=EN&KategorijaEn_input=Ophtholmology&… 7/9
3/27/2019 LSMUSIS
C. Surgical treatment
D. Laser photocoagulation

17.61 Patient complains of photophobia, tearing, foreign body sensation and redness of his right eye. He is
constantly wearing contact lenses and did not remove them during the last night. Eye examination: mixed injection
of the right eye; a grey-yellow paracentral corneal infiltrate with one of its margins elevated. Hypopyon observed in
the anterior chamber. What additional tests should be performed:
A. Fluorescein staining
B. Corneal scraping and culture
C. Both tests mentioned above

17.62 Patient complains of photophobia, tearing, foreign body sensation and redness of his right eye. He is
constantly wearing contact lenses and did not remove them during the last night. Eye examination: mixed injection
of the right eye; a grey-yellow paracentral corneal infiltrate with one of its margins elevated. Hypopyon observed in
the anterior chamber. What is the management strategy of the disease:
A. Topical broad-spectrum antibiotics should be started immediately, without culture and Gram stain results
B. Two broad-spectrum fortified antibiotics are used alternating q1h (taking a drop every 30 minutes)
C. Topical cycloplegic agents
D. Antibiotics adjusted based on culture and Gram stain results
E. All the answers are correct

17.63 A thirty-five-year-old man complains of blepharospasm, photophobia, tearing and blurred vision of his right
eye. Three days ago he had caught a cold and now is suffering from a runny nose. Eye examination: a
considerable ciliary injection. In the center of the cornea, there is an irregular defect (lesion) of corneal epithelium.
Fluorescein staining showed that the defect has a shape of a branch of a tree. The corneal sensitivity is absent.
The preliminary diagnosis is:
A. Metaherpetic keratitis
B. Bacterial keratitis
C. Dendritic keratitis
D. Adenoviral keratoconjunctivitis

17.64 A thirty-five-year-old man complains of blepharospasm, photophobia, tearing and blurred vision of his right
eye. Three days ago he had caught a cold and now is suffering from a runny nose. Eye examination: a
considerable ciliary injection. In the center of the cornea, there is an irregular defect (lesion) of corneal epithelium.
Fluorescein staining showed that the defect has a shape of a branch of a tree. The corneal sensitivity is
absent.Initial treatment should be:
A. Specific antiviral treatment (topical and systemic) and cycloplegic agents
B. Broad-spectrum antibiotics and cycloplegic agents
C. No specific treatment
D. Artificial tears and cycloplegic agents

17.65 A sixty-year-old man noticed flashing lights and floaters in his left eye one week ago. Two days ago he
noticed something like a grey curtain moving from below over the field of vision. The vision of his left eye markedly
decreased. Eye examination: VA- OD=1.0; VA OS - light perception without projection (PLI), no color vision. What
preliminary diagnoses should be suspected:
A. Retinal detachment
B. Central retinal artery occlusion
C. Optic neuritis

17.66 A sixty-year-old man noticed flashing lights and floaters in his left eye one week ago. Two days ago he
noticed something like a grey curtain moving from below over the field of vision. The vision of his left eye markedly
decreased. Eye examination: VA- OD=1.0; VA OS - light perception without projection (PLI), no color vision. What
fundoscopic signs could be typical for the preliminary diagnosis:
A. Whitening of the retina, narrow retinal arterioles, segmentation of the blood column in the arterioles, a
cherry-red spot in the center of the macula
B. Undulating, mobile, convex superior retina with corrugated folds, retinal break in the shape of a
horseshoe, “tobacco-dust” in the vitreous
C. Pale, elevated and rigid inferior retina, fixed folds, retinal tear.
D. Swollen optic nerve disc, few peripapillary flame-shaped hemorrhages, posterior vitreous cells

17.67 A seventy-year-old patient complained of reduced vision and flashes of light in his right eye. One week ago
he noticed the dark shadow over his visual field in the right eye. Examination data: arterial blood pressure

lsmusis.lsmuni.lt/Klausimai/Spausdinti?Length=0?Kalba=EN&KategorijaId=119&Kalbos_input=EN&Kalbos=EN&KategorijaEn_input=Ophtholmology&… 8/9
3/27/2019 LSMUSIS
145/90mmHg, normal ocular motility, optic media clear, on fundus examination –gray-brown elevated choroidal
lesion in the posterior pole with overlying orange pigment, and subretinal fluid. What tests could be helpful to
confirm the preliminary diagnosis:
A. Ultrasonography, fundus photography, magnetic resonance imaging
B. Visual field testing and dark adaptation
C. Visual evoked potentials, electroretinography and fluorescein angiography

17.68 A seventy-year-old patient complained of reduced vision and flashes of light in his right eye. One week ago
he noticed the dark shadow over his visual field in the right eye. Examination data: arterial blood pressure
145/90mmHg, normal ocular motility, optic media clear, on fundus examination –gray-brown elevated choroidal
lesion in the posterior pole with overlying orange pigment, and subretinal fluid.What the preliminary diagnosis
should be suspected:
A. Age-related macular degeneration
B. Malignant melanoma of the choroid
C. Vitreous hemorrhage

17.1 - A 17.2 - C 17.3 - D 17.4 - B 17.5 - D 17.6 - C 17.7 - A 17.8 - C


17.9 - C 17.10 - B 17.11 - B 17.12 - B 17.13 - C 17.14 - D 17.15 - D 17.16 - D
17.17 - C 17.18 - B 17.19 - B 17.20 - D 17.21 - D 17.22 - B
17.23 17.24 17.25 17.26 17.27 17.28 17.29 17.30
1-C 1-B 1-C 1-C 1-D 1-B 1-A 1-A
2-A 2-C 2-B 2-B 2-C 2-C 2-B 2-C
3-B 3-A 3-A 3-B 3-A 3-C 3-B
4-D 4-D
17.31 17.32 17.33
1-A 1-B 1-C
2-B 2-A 2-A
3-C 3-C 3-B
17.34 17.35 17.36 17.37 17.38 17.39 17.40 17.41
1 1 4 4 4 1 4 4
3 2 2
3 3
4
17.42 17.43 17.44 17.45 17.46 17.47 17.48 17.49
1 1 1 1 1 1 1 1
2 2 2 2 2 3 2 2
3 3 3 3 3 3 3
4 4 4 4
5
17.50
4
17.51 - C 17.52 - C 17.53 - C 17.54 - A 17.55 - B 17.56 - B 17.57 - B 17.58 - C
17.59 - D 17.60 - B 17.61 - C 17.62 - E 17.63 - C 17.64 - A 17.65 - A 17.66 - B
17.67 - A 17.68 - B

lsmusis.lsmuni.lt/Klausimai/Spausdinti?Length=0?Kalba=EN&KategorijaId=119&Kalbos_input=EN&Kalbos=EN&KategorijaEn_input=Ophtholmology&… 9/9
3/27/2019 LSMUSIS

Intensive therapy
I type tasks. Choose only one best answer

48.1 The compression-to-ventilation ratio for one rescuer adult CPR is:
A. 30:2
B. 15:2
C. 60:2
D. 30:1

48.2 The depth of chest compressions for an adult victim should be at least:
A. 2 cm
B. 4 cm
C. 5 cm
D. 6 cm

48.3 Which part of sternum should be compressed to support circulation in BLS?


A. Manubrium sterni
B. Upper half of sternum
C. Middle part of sternum
D. Lower half of sternum

48.4 Which of the following options lists the correct compression and ventilation rates for two rescuer CPR in the
presence of an advanced airway?
A. Compress at a rate of at least 100 per minute, 1 breath every 6 to 8 seconds
B. Compress at a rate of at least 100 per minute, 2 breaths every 10 seconds
C. Compress at a rate of at least 80 per minute, 1 breaths every 3 to 5 seconds
D. Compress at a rate of at least 80 per minute, 2 breaths every 6 to 8 seconds

48.5 Which mechanism of cardiac arrest is caused by high voltage alternate current?
A. Ventricular fibrillation
B. Pulseless Ventricular tachycardia
C. Asystole
D. Electromechanical dissociation

48.6 Which drug is used during resuscitation in all cases independently on mechanism of cardiac arrest?
A. Lidocaine
B. Epinefrine
C. Atropine
D. Calcium chloride

48.7 In cardiac arrest, what will happen to the end-tidal CO2 when return of spontaneous circulation occurs?
A. Will fall
B. Will rise
C. Will not change
D. Will change only if ventilation rate changes

48.8 Dose of epinefrine during resuscitation for adult?


A. 1 mg every 3-5 min.
B. 1 mg/kg every 3-5 min.
C. 10 mg every 3-5 min.
D. 0.4 mg every 3-5 min.

48.9 Normal value of anion gap (AG):


A. 2-4 mmol/l
B. 4-8 mmol/l
C. 8-16 mmol/l

48.10 What is the management for fat embolism?


A. Prevention, early diagnosis and adequate symptomatic treatment
B. Maintenance of intravascular volume, anticoagulation, glucocorticoids
C. Maintenance of intravascular volume, anticoagulation, oxygen therapy
D. Anticoagulation and adequate symptomatic treatment
lsmusis.lsmuni.lt/Klausimai/Spausdinti?Length=0?Kalba=EN&KategorijaId=150&Kalbos_input=EN&Kalbos=EN&KategorijaEn_input=Intensive+thera… 1/11
3/27/2019 LSMUSIS
48.11 What minimal amount of carbohydrate must be used for 24 h during maintenance infusion therapy to prevent
ketoacidosis and supress protein catabolism?
A. 25–50 g
B. 100-150 g
C. 200–250 g

48.12 How many mmol of potassium is in 40 ml 7,45% KCl?


A. 10 mmol
B. 20 mmol
C. 30 mmol
D. 40 mmol

48.13 What is a daily need potassium requirement per kilo of ideal body weight?
A. 1-1,5 mmol
B. 3,5-5,5 mmol
C. 6,7-7,8 mmol

48.14 30 year old patient suddenly became apneic and unconscious during the meal time. Witnesses stated he
was unable to speak and looked nervous and agitated. The most likely diagnosis is:
A. Pulmonary embolism
B. Airway obstruction
C. Anaphylaxis
D. Asthma

48.15 Normal value of total calcium in human blood serum:


A. 1,25–1,75 mmol/l
B. 2,20–2,60 mmol/l
C. 2,70–3,00 mmol/l

48.16 What disturbance of base – acid balance is characteristic for prolonged shock?
A. Metabolic acidosis
B. Metabolic alkalosis
C. Respiratory acidosis
D. Respiratory alkalosis

48.17 Which of the following management options may be used in the treatment of a non-high risk pulmonary
embolism?
A. Trombolysis, anticoagulation, surgical or catheter embolectomy and adequate symptomatic treatment
B. Trombolysis, anticoagulation, inferior vena cava filter and adequate symptomatic treatment
C. Anticoagulation and adequate symptomatic treatment

48.18 When should an inferior vena cava filter be used to treat venous trombembolism?
A. In patients with pulmonary embolism and deep venous thrombosis who have a contraindication to
anticoagulation
B. In patients where recurrent pulmonary embolism occurs despite optimal anticoagulant therapy
C. All of the above

48.19 26 years old female admitted to ICU with antidepressant overdose. She was unconscious, wide QRS wave
and prolonged interval QT were seen on ECG. Pulmonary edema developed at first day. Its cause could be:
A. Altered pulmonary lymphatic flow
B. Reduced oncotic blood pressure
C. Increased hydrostatic pulmonary wedge pressure
D. Increased permeability of capillary endothelium also alveolic epithelium

48.20 Which is a drug of choice in hypertensive crisis?


A. Ganglioblockers
B. Sodium nitroprusside
C. Nitroglycerin
D. Betaadrenoblockers
E. Magnesium sulphate

48.21 Acute liver failure diagnosis based on:

lsmusis.lsmuni.lt/Klausimai/Spausdinti?Length=0?Kalba=EN&KategorijaId=150&Kalbos_input=EN&Kalbos=EN&KategorijaEn_input=Intensive+thera… 2/11
3/27/2019 LSMUSIS
A. Enlargement in size of liver, more evident jaundice, increased amount of bilirubin also hepatic enzymes in
blood
B. Hypoalbuminemia, disturbance of coagulation system, intensity of jaundice
C. Signs of hepatic encephalopathy

48.22 Patient in hypothermia (core T 30⁰) was admitted to ICU. The ventricular fibrillation was recorded on ECG.
The management should be:
A. Cardioversion
B. Antiarrhythmic drugs
C. Advanced cardiac life support according algorithm of ventricular fibrillation and patient's rewarming

48.23 Which is a first line drug in anaphylaxis management?


A. H1 – receptors blockers
B. H2 – receptors blockers
C. Epinephrin
D. Glucocorticoids

II type tasks. For each numbered item,selct the one lettered heading that is most closely asssciated with it

48.24 In which compartments of the body water you can expect distribution of those solutions?
1 - 20.69. Coloids
2 - 20.70. Electrolytes
3 - 20.71. Dextrose
A. Intravascular
B. Intravascular and interstitial
C. Intravascular, interstitial and intracellular

48.25 The dosage of medications and defibrillation during adult CPR in case of ventricular fibrillation (VF):
1 - 20.25. Defibrillation
2 - 20.26. Epinephrine
3 - 20.27. Amiodarone
A. 300 mg bolus IV after the 3rd shock. Further dose of 150 mg if VF persist
B. 1 mg IV after the 3rd shock. Repeated every 3 – 5 min.
C. Initial dose of 120 – 200 J (biphasic). If unknown use maximum available

48.26 What distance must be estimated during choice of:


1 - 20.30. Oropharyngeal airway
2 - 20.31. Nasopharyngeal airway
A. From the nostrile of nose to the earlobe
B. From the angle of mouth to the entrance of ear
C. From the angle of mouth to the angle of mandible
D. From the nostrils of nose to the angle of mandible

48.27 What is the blood serum electrolytes content?


1 - 20.43. Sodium 120 mmol/l
2 - 20.44. Potassium 6 mmol/l
3 - 20.45. Calcium total 1,5 mmol/l
4 - 20.46. Calcium ionised 0,5 mmol/l
5 - 20.47. Magnesium 0.5 mmol/l
6 - 20.48. Chloride 76 mmol/l
A. Normal
B. Decreased
C. Increased

48.28 What is the acid – base status?


1 - 20.54. pH – 7,4, pCO2 mmHg – 40, BE – 0 mmol/l
2 - 20.55. pH – 7,5, pCO2 mmHg – 30, BE – (-) 1 mmol/l
3 - 20.56. pH – 7,6, pCO2 mmHg – 43, BE – (+) 19,1 mmol/l
4 - 20.57. pH – 7,2, pCO2 mmHg – 33, BE – (-) 9 mmol/l
5 - 20.58. pH – 7,25, pCO2 mmHg – 66, BE – (+) 8,2 mmol/l
A. Normal
B. Metabolic acidosis

lsmusis.lsmuni.lt/Klausimai/Spausdinti?Length=0?Kalba=EN&KategorijaId=150&Kalbos_input=EN&Kalbos=EN&KategorijaEn_input=Intensive+thera… 3/11
3/27/2019 LSMUSIS
C. Metabolic alkalosis
D. Respiratory acidosis
E. Respiratory alkalosis

48.29 What is osmotic pressure?


1 - 20.35. 240 mmol/kg H2O
2 - 20.36. 290 mmol/kg H2O
3 - 20.37. 340 mmol/kg H2O
A. Normal
B. Increased
C. Decreased

48.30 What is the water balance?


1 - 20.32. Intake 2300 ml; output 2600 ml
2 - 20.33. Intake 2600 ml, output 2600 ml
3 - 20.34. Intake 2600 ml; output 2300 ml
A. Normal
B. Positive
C. Negative

48.31 How many percents of male body weight consists of bellow listed waters in the human organism?
1 - 20.38. Total body water
2 - 20.39. Intracellular body water
3 - 20.40. Extracellular body water
4 - 20.41. Interstitial body water
5 - 20.42. Intravascular body water
A. 5%
B. 10%
C. 15%
D. 20%
E. 30%
F. 40%
H. 60%

48.32 What is the main route for the elimination of electrolytes from the body?
1 - 20.49. Sodium
2 - 20.50. Potassium
3 - 20.51. Calcium
4 - 20.52. Magnesium
5 - 20.53. Chloride
A. Kidney
B. GI tract
C. Skin
D. Expired air

48.33 What signs of ECG are seen in:


1 - 20.85. Hyperthermia
2 - 20.85. Hypothermia
A. High peaked T – waves, S-T segments dislocation above base – line
B. Osborn (J) wave
C. No specific sign
D. Decreased amplitude of T –wave and increased one of U wave, S-T segments dislocation below base
– line

48.34 What signs of ECG are seen in:


1 - 20.82. Hyperkalemia -B. High peaked T – waves, S-T segments dislocation above base – line
2 - 20.83. Hypokalemia A. Decreased amplitude of T –wave and increased one of U wave, S-T segments dislocation below base
– line
A. Decreased amplitude of T –wave and increased one of U wave, S-T segments dislocation below base
– line
B. High peaked T – waves, S-T segments dislocation above base – line
C. Osborn (J) wave.
D. No specific sign
lsmusis.lsmuni.lt/Klausimai/Spausdinti?Length=0?Kalba=EN&KategorijaId=150&Kalbos_input=EN&Kalbos=EN&KategorijaEn_input=Intensive+thera… 4/11
3/27/2019 LSMUSIS
48.35 Distribution of named solutions according to their osmotic pressure:
B 1 - 20.72 0.9% NaCl
B 2 - 20.73. 5% Glucose
A 3 - 20.74. 4.2% NaHCO3
A 4 - 20.75. 10% Glucose
C 5 - 20.76. 0.45% NaCl
A 6 - 20.77. 20% Glucose
A. Hypertonic
B. Isotonic
C. Hypotonic

48.36 For which pulmonary embolism following features are most characteristic?
1 - 20.78. Pulmonary edema, acute respiratory failure, cerebral edema, seizures, obstructive shock,
coagulopathy
2 - 20.79. Pulmonary edema, acute respiratory failure, petechial rash, cerebral edema, coma
3 - 20.80. Acute respiratory failure, obstructive shock
4 - 20.81. Acute respiratory failure, obstructive shock, focal neurological symptoms
A. Amniotic fluid embolism
B. Air embolism
C. Fat embolism
D. Pulmonary embolism

48.37 What is the primary mechanism of cardiac arrest in each electrical injury case?
1 - 20.28. High voltage alternate current injury
2 - 20.29. Lightening
A. Ventricular fibrillation
B. Asystolia
C. Electromechanical dissociation

48.38 Which treatment must be used for the correction of acid – base disturbances?
1 - 20.65. Metabolic acidosis
2 - 20.66. Metabolic alkalosis
3 - 20.67. Respiratory acidosis
4 - 20.68. Respiratory alkalosis
A. NaHCO₃IV
B. 7.45% KCl + 0.9% NaCl and 0.1 N HCl IV
C. Mechanical ventilation
D. Increase Carbon dioxide concentration in mixture

48.39 What kind of shock is due to:


1 - 20.86. Burns
2 - 20.87. Atrial flutter
3 - 20.88. Pulmonary embolism
4 - 20.89. Myocardial infarction
A. Hypovolemic
B. Neurogenic
C. Obstructive
D. Septic
E. Cardiogenic

48.40 What does the end-tidal CO2 measurement mean during CPR for adult?
1 - 20.59. 5 mm Hg
2 - 20.60. 35 mm Hg
3 - 20.61. 15 mm Hg
A. Need to improve the quality of chest compressions
B. Good quality of chest compressions
C. May return of spontaneous circulation occurs

48.41 What is a coma severity using Glasgow coma scale (GCS)?


1 - 20.62. GCS 13-15
2 - 20.63. GCS 9-12
3 - 20.64. GCS less than 8
lsmusis.lsmuni.lt/Klausimai/Spausdinti?Length=0?Kalba=EN&KategorijaId=150&Kalbos_input=EN&Kalbos=EN&KategorijaEn_input=Intensive+thera… 5/11
3/27/2019 LSMUSIS
A. Severe
B. Moderate
C. Light

48.42 What adrenoreceptors does adrenomimetic drug act on:


1 - 20.90. Epinephrin
2 - 20.91. Norepinephrin
3 - 20.92. Dopamin
4 - 20.93. Phenylephrin
A. On alpha – adrenoreceptors only
B. On beta - adrenoreceptors only
C. Both on alpha and beta – adrenoreceptors
D. On alpha, beta also dopaminergic receptors

III type tasks. For each question there is one or more correct answers:
A – if correct answers are 1,2,3
B – if correct answers are 1 and 3
C – if correct answers are 2 and 4
D – if correct answer is 4
E – if correct are all answers above

48.43 In which case salts of calcium are used in resuscitation?


1. Hyperkalemia
2. β - blockers toxicity
3. Hypokalcemia
4. Calcium channel blockers toxicity.

48.44 Airway management in foreign body obstruction for conscious adult should be started:
1. To perform finger sweep to remove the object
2. Victim standing or sitting – management by Heimlich maneuvre 5
3. Victim lying – management by Heimlich maneuvre 5
4. Back slaps 5 times and Heimlich maneuver 5 times in rotation

48.45 When electrical defibrillation is indicated in the event of cardiac arrest?


1. Asystole
2. Ventricular fibrillation
3. Pulseless electrical activity
4. Pulseless ventricular tachycardia

48.46 The compositions of immediate post–cardiac arrest care are:


1. Optimal systemic perfusion
2. Normocapnia, normoxemia
3. Therapeutic hypothermia
4. Normothermia

48.47 Which drugs may be injected in the bone marrow during resuscitation?
1. Epinephrine
2. Lidocaine
3. Amiodarone
4. Magnesium sulphate

48.48 Under which conditions electrical stimulation of the heart must be used?
1. Ventricular fibrillation
2. Ventricular tachycardia
3. Electromechanical dissociation
4. Unstable bradycardia

48.49 A patient is in cardiac arrest. Ventricular fibrillation has been refractory to a third shock. Of the following,
which drugs should be administered by the IV/IO route?
1. Sodium bicarbonate
2. Amiodarone
3. Atropine
lsmusis.lsmuni.lt/Klausimai/Spausdinti?Length=0?Kalba=EN&KategorijaId=150&Kalbos_input=EN&Kalbos=EN&KategorijaEn_input=Intensive+thera… 6/11
3/27/2019 LSMUSIS
4. Epinephrine

48.50 During resuscitation epinephrine must be used in case of:


1. Ventricular fibrillation
2. Asystole
3. Pulseless ventricular tachycardia
4. Electromechanical dissociation

48.51 In what situations of hypovolemic shock central venous pressure may be not decreased, but increased in
case of:
1. Pneumothorax
2. Pericardial tamponade
3. Air embolism
4. Heart contusion

48.52 The risk factors for development of deep vein thrombosis in postoperative period are:
1. Obesity
2. Pregnancy
3. Malignancy
4. Oral contraceptives
5. Paralysis

48.53 Prehospital treatment of hemorrhagic shock due to trauma consists of:


1. Supine position
2. Control of hemorrhage
3. Immobilization of fractures
4. Correction of hypovolemia by isotonic saline solution

48.54 What conditions may limit the volume of infusion therapy?


1. Respiratory failure
2. Heart failure
3. Liver failure
4. Renal failure

48.55 The treatment of acute respiratory distress syndrome consists of:


1. Extracorporeal membrane oxygenation
2. Mechanical ventilation
3. Glucocorticoids
4. Nonsteroidal antiimflammatory drugs

48.56 Which drugs use is contraindicated in severe asthmatic attack?


1. Codein
2. Sedatives
3. β - adrenoblockers
4. Theophyllin
5. Prednisolon
6. Ipratropium bromide

48.57 Inappropriate treatment of oligoanuric period of acute renal failure complication – pulmonary edema is:
1. Nitrates
2. Oxygenotherapy
3. Hemodialysis
4. Diuretics

48.58 In eclampsia for management of hypertensia could be used:


1. Sodium nitroprusside
2. Diazoxyde
3. Labetalol
4. Hydralazin
5. Clophelin

48.59 The pulmonary edema has developed in acute myocardial infarction. The correct treatment is:
1. Saluretics
lsmusis.lsmuni.lt/Klausimai/Spausdinti?Length=0?Kalba=EN&KategorijaId=150&Kalbos_input=EN&Kalbos=EN&KategorijaEn_input=Intensive+thera… 7/11
3/27/2019 LSMUSIS
2. Nitrates
3. Inhibitors of fosfodiesterase III
4. Morphine

IV type tasks. Choose only one best answer

48.60 A woman, 64 years old, was hospitalized to intensive care unit because acute myocardial infarction. On the
third day she suddenly becomes unconscious with deep noisy breathing and pulslessness at the carotid artery.
There is registered sinus rhythm on cardio monitor. Your diagnosis is:
A. Syncope
B. Shock cardiogenes
C. Cardiac arrest resulted by electromechanical dissociation
D. Coma

48.61 A woman, 64 years old, was hospitalized to intensive care unit because acute myocardial infarction. On the
third day she suddenly becomes unconscious with deep noisy breathing and pulslessness at the carotid artery.
There is registered sinus rhythm on cardio monitor. What aid must be given?
A. You must begin chest compressions immediately, inject epinephrine 1 mg intravenous, again continue
cardiac compressions and estimate reason of such condition
B. You must begin infusion with dopamine
C. You must apply oxygen therapy and inject epinephrine 10 mg intravenous
D. You must put patient in horisontal position and begin oxygen therapy.

48.62 Patient suddenly became unconscious with deep noisy breathing and pulslessness at the carotid artery. P
waves without QRS complexes are registered on cardio monitor. What mechanism of cardiac arrest will you
diagnose?
A. Ventricular fibrillation
B. Atrial fibrillation
C. Asystole
D. Electromechanical dissociation

48.63 Patient suddenly became unconscious with deep noisy breathing and pulslessness at the carotid artery. P
waves without QRS complexes are registered on cardio monitor. What drugs must you use during resuscitation?
A. Epinephrine, lidocaine
B. Epinephrine
C. Atropine, lidocaine
D. You mustn't use any drugs

48.64 A 30-year-old male is admitted to the hospital after a motorcycle accident that resulted in a fracture of the
right femur. Three days later the patient becomes tachypneic and encephalopatic. A petechial rash is noted over
the chest. Arterial blood gases show pO2 50, pCO2 28 mm Hg, and pH 7.49. The most likely diagnosis is:
A. Pulmonary edema
B. Pulmonary contusion
C. Fat embolism
D. Pulmonary embolism
E. Pneumonia

48.65 A 30-year-old male is admitted to the hospital after a motorcycle accident that resulted in a fracture of the
right femur. Three days later the patient becomes tachypneic and encephalopatic. A petechial rash is noted over
the chest. Arterial blood gases show pO2 50, pCO2 28 mm Hg, and pH 7.49. Which of the following management
options must be chosen?
A. Early diagnosis and adequate symptomatic treatment
B. Maintenance of intravascular volume, anticoagulation, glucocorticoids
C. Maintenance of intravascular volume, anticoagulation, oxygen therapy

48.66 In ICU is hospitalized patient after prolonged starvation. Blood pressure 80/60 mmHg. Hb 170 g/l. Htc 0.65.
Cl- 76 mmol/l, blood serum sodium 115 mmol/l. Osmotic pressure 260 mmol/kg H2O. What type of dehydration is
that?
A. Isotonic
B. Hypotonic
C. Hypertonic

lsmusis.lsmuni.lt/Klausimai/Spausdinti?Length=0?Kalba=EN&KategorijaId=150&Kalbos_input=EN&Kalbos=EN&KategorijaEn_input=Intensive+thera… 8/11
3/27/2019 LSMUSIS
48.67 In ICU is hospitalized patient after prolonged starvation. Blood pressure 80/60 mmHg. Hb 170 g/l. Htc 0.65.
Cl- 76 mmol/l, blood serum sodium 115 mmol/l. Osmotic pressure 260 mmol/kg H2O. Which solution must be used
for the correction of dehydration in this case:
A. 5% Dextrose
B. 0.9% NaCl
C. Isotonic – isoionic electrolyte solution

48.68 Potassium level in the blood serum is 2 mmol/l, chloride 79 mmol/l. What amount of potassium in mmol,
must be added to 1 L of infusion solution and infuzed during 1 h?
A. 20 mmol/l; 10 mmol/h
B. 40 mmol/l; 20 mmol/h
C. 60 mmol/l; 30 mmol/h

48.69 22 years old female was admitted to hospital because of eclampsia with manifesting peripheral edemas.
What are the pathophysiological mechanisms of this condition?
A. Hypernatremia, hyperchloremia, hypervolemia
B. Increased peripheral vessels resistancy and permeability
C. Decreased oncotic blood pressure (hypoalbuminemia)

48.70 22 years old female was admitted to hospital because of eclampsia with manifesting peripheral edemas. For
edema treatment you will:
A. Use saluretics, reduce input of NaCl
B. Correct hypovolemia
C. Correct hypoalbuminemia

48.71 The patient with aspirative pneumonia is in fever of 41.2o C. You manage it with antipyretics also internal
and external cooling measures. Which of the following is the most effective external cooling measure?
A. Ice on the magistral vessels
B. Immersion of body in ice – water
C. To humidify body with warm water and to evaporate with electrical dryers
D. To humidify body with cold water and to evaporate with electrical dryers

48.72 The patient with aspirative pneumonia is in fever of 41.2o C. You manage it with antipyretics also internal
and external cooling measures. Which of the following is the most effective internal cooling measure?
A. I/v infusion with cold fluids
B. Enemas with iced – water
C. Gastric lavage with cold water
D. Urinary blader lavage with cold water

48.73 Patient in severe asthamatic attack is dyspneic. Respiratory rate is 18, heart rate 120 per min. there are
rales present in both sides. Blood gases: pH 7.35, BE (-) 1 mmol/l, pCO2 30 mmHg, pO2 70 mmHg. To manage
this disturbance you must use:
A. Oxygenotherapy, beta2 – agonists, prednisolon, fluids
B. Theophyllin, prednisolon
C. Beta 2 – agonists, atropin, theophyllin, oxygenotherapy

48.74 Patient in severe asthamatic attack is dyspneic. Respiratory rate is 18, heart rate 120 per min. there are
rales present in both sides. Blood gases: pH 7.35, BE (-) 1 mmol/l, pCO2 30 mmHg, pO2 70 mmHg. In course of
treatment respiratory rate increased to 30 per min., pH 7.30, pCO2 40 mmHg, pO2 45 mmHg. You will:
A. Continue the same treatment, intubate the patient and start mechanical ventilation
B. Intubate the patient and start mechanical ventilation, organize bronchial lavage
C. Continue the same treatment with additional dose of prednisolone only

48.75 50 years old hypertensive male was admitted to ICU because of the sharp pain in chest behind sternum.
The patient is pale, heart rate 105 beats per min., blood pressure in right hand is 170/100 mmHg, in left one
225/145 mmHg. There is a diastolic murmur in the point of auscultation of aortic valves, signs of left ventricule
hyperthrophy on ECG. The diagnosis is:
A. Acute myocardial infarction
B. Dissection of aortic aneurysm
C. Hypertensive crisis and acute myocardial infarction
D. Hypertensive crisis

lsmusis.lsmuni.lt/Klausimai/Spausdinti?Length=0?Kalba=EN&KategorijaId=150&Kalbos_input=EN&Kalbos=EN&KategorijaEn_input=Intensive+thera… 9/11
3/27/2019 LSMUSIS
48.76 50 years old hypertensive male was admitted to ICU because of the sharp pain in chest behind sternum.
The patient is pale, heart rate 105 beats per min., blood pressure in right hand is 170/100 mmHg, in left one
225/145 mmHg. There is a diastolic murmur in the point of auscultation of aortic valves, signs of left ventricule
hyperthrophy on ECG. What is the drug of choice to manage the hypertension?
A. Phentolamin
B. Sodium nitroprusside and β-adrenoblockers
C. Diazoxyde
D. Hydralasin

48.78 Potassium level in the blood serum is 2 mmol/l, chloride 79 mmol/l. What is maximal amount of potassium,
that can be infused during 24 h?
A. 100 mmol/24 val.
B. 200 mmol/24 h
C. 300 mmol/24 h.

48.79 In ICU hospitalized agitated patient with shallow respiration and cyanosis of the skin. Blood pressure 170/90
mmHg, pH 7.1, pCO2 80 mmHg, BE 10 mmol/l, pO2 60 mmHg. Define the acid – base balance disturbance.
A. Metabolic acidosis
B. Metabolic alkalosis
C. Respiratory acidosis
D. Repiratory alkalosis

48.80 In ICU hospitalized agitated patient with shallow respiration and cyanosis of the skin. Blood pressure 170/90
mmHg, pH 7.1, pCO2 70 mmHg, BE 10 mmol/l, pO2 60 mmHg. For the correction of this disturbance must be
used:
A. 4.2% or 8.4% Sol. NaHCO3 IV
B. Oxygenotherapy by the mask
C. Endotracheal intubation and mechanicial ventilation

48.1 - A 48.2 - C 48.3 - D 48.4 - A 48.5 - A 48.6 - B 48.7 - B 48.8 - A


48.9 - C 48.10 - A 48.11 - B 48.12 - D 48.13 - A 48.14 - B 48.15 - B 48.16 - A
48.17 - C 48.18 - C 48.19 - C 48.20 - B 48.21 - C 48.22 - C 48.23 - C
48.24 48.25 48.26 48.27 48.28 48.29 48.30 48.31
1-A 1-C 1-C 1-B 1-A 1-C 1-C 1-H
2-B 2-B 2-A 2-C 2-E 2-A 2-A 2-F
3-C 3-A 3-B 3-C 3-B 3-B 3-D
4-B 4-B 4-C
5-B 5-D 5-A
6-B
48.32 48.33 48.34 48.35 48.36 48.37 48.38 48.39
1-A 1-C 1-B 1-B 1-A 1-A 1-A 1-A
2-A 2-B 2-A 2-B 2-C 2-B 2-B 2-E
3-B 3-A 3-D 3-C 3-C
4-B 4-A 4-B 4-D 4-E
5-A 5-C
6-A
48.40 48.41 48.42
1-A 1-C 1-C
2-C 2-B 2-C
3-B 3-A 3-D
4-A
48.43 48.44 48.45 48.46 48.47 48.48 48.49 48.50
1 4 2 1 1 4 2 1
2 4 2 2 4 2
3 3 3 3
4 4 4
48.51 48.52 48.53 48.54 48.55 48.56 48.57 48.58
1 1 1 2 1 1 4 1
2 2 2 4 2 2 2
3 3 3 3 3 3
lsmusis.lsmuni.lt/Klausimai/Spausdinti?Length=0?Kalba=EN&KategorijaId=150&Kalbos_input=EN&Kalbos=EN&KategorijaEn_input=Intensive+ther… 10/11
3/27/2019 LSMUSIS
4 4 4 4
5 5
48.59
1
2
3
4
48.60 - C 48.61 - A 48.62 - C 48.63 - B 48.64 - C 48.65 - A 48.66 - B 48.67 - B
48.68 - B 48.69 - B 48.70 - B 48.71 - C 48.72 - A 48.73 - A 48.74 - A 48.75 - B
48.76 - B 48.78 - B 48.79 - C 48.80 - C

lsmusis.lsmuni.lt/Klausimai/Spausdinti?Length=0?Kalba=EN&KategorijaId=150&Kalbos_input=EN&Kalbos=EN&KategorijaEn_input=Intensive+ther… 11/11
3/27/2019 LSMUSIS

Gynaecology
I type tasks. Choose only one best answer

26.1 What is characteristic of vaginitis:


A. Fever
B. Bleeding
C. Vaginal discharge
D. Amenorrhea

26.2 What is disharge is specific to "candida" vaginitis:


A. Thin, watery discharge
B. Green frothy discharge
C. Thick and viscous discharge
D. Curd like discharge

26.3 How is candidal vaginitis treated:


A. Oral or intravaginal metronidazole
B. Intravaginal tinidazole tablets
C. Oral tetracycline tablets
D. Clotrimazole: vaginal cream or suppositories

26.4 Which is of the following is cosidered appropriate local treatment for bacterial vaginosis:
A. Estrogens
B. Gestagens
C. Vaginal irrigation with low pH solution
D. Clindamycin 2% vaginal cream

26.5 Which of the following vaginal infections is caused by a protozoa:


A. Candida infections
B. Gonnorhea
C. Trichomonal vaginitis
D. Bacterial vaginosis

26.6 Which of the following is determined by 2nd level amenorrhea:


A. Uterine or vaginal aplasia
B. Prolactinoma
C. Anorexia nervosa
D. Gonadal dysgenesis

26.7 Which of the following PID complication requires an emergency laparotomy:


A. Chronic pelvic pain
B. Infertility
C. Rupture of tuboovarian abcess
D. Bilateral hydrosalpinx

26.8 Which of the following would confirm the diagnosis of cervical cancer:
A. Colposcopy
B. Histology
C. Ultrasound
D. Laparoscopy

26.9 Which uterine myoma in your opinion would be the most likely to cause heavy menstrual bleeding:
A. Submucosal myoma
B. Subserous myoma
C. Intramural myoma
D. Intraligamentary myomas

26.10 Which of the following would be most informative for evaluating fallopian tube functioning in cases when we
have infertility:
A. Ultrasound
B. Hysterosalpingography
C. Laparoscopy in additon to the dye test
lsmusis.lsmuni.lt/Klausimai/Spausdinti?Length=0?Kalba=EN&KategorijaId=128&Kalbos_input=EN&Kalbos=EN&KategorijaEn_input=Gynaecology&K… 1/10
3/27/2019 LSMUSIS
D. Sex hormones

26.11 What is menopause:


A. Absence of menses for 3 months
B. Retrospectively indicated date of the last menstrual period 6 months ago
C. Absence of menses for 8 months
D. Retrospectively indicated date of the last menstrual period 12 months ago

26.12 In menopause a deficit in which of the following hormones result in atrophic changes:
A. Gestagens
B. Androgens
C. Estrogens
D. Gonadotropins

26.13 In what cases is not recommended to prescribe estrogen therapy for relieving symptoms of menopause:
A. Diabetes mellitus
B. In cases of breast cancer
C. In cases of osteoporosis
D. In cases when the patient suffers from bronchitis

26.14 Which of the following types of abnormal bleeding is cyclic:


A. Metrorraghia
B. Hypermennorhea
C. Bledding due to cervical cancer
D. Bleeding from a cervical polyp

II type tasks. For each numbered item,selct the one lettered heading that is most closely asssciated with it

26.15 How can we diagnose the reason behind infertility:


1 - Ovulation disorders
2 - Tubal damage
3 - The male factor
4 - Anatomical pecularities of the uterine cavity
A. By ordering a spermogram
B. By determining the level of the sex hormones in the serum
C. By performing a laparoscopy
D. By performing a hysteroscopy and taking an endometrial biopsy
E. By ordering a complete blood count

26.16 Match each of the following ovarian tumors with the sentence that best describes it:
1 - Folliculoma
2 - Dermoid cyst
3 - Endometrioma
4 - Fibroma
A. A mobile tumor with a smooth surface. It is usually filled with fat, hair or cartilaginous tissue
B. An androgen producing tumor
C. An estrogen producing tumor
D. A solid, mobile tumor with distinct borders
E. A fixed tumor enveloped in adhaesions and causing pain during menstruation

26.17 Which of the following symptoms are characteristic of the listed pathologies:
1 - Advanced ovarian cancer (st. III – IV)
2 - Cervical cancer
3 - Endometrial cancer
4 - Cervical dysplasia
A. Irregular bleeding and bleeding after intercourse
B. Pre- and postmenopausal bleeding
C. Ascites
D. Hystologically evaluated intraepithelial cervical neoplasia

26.18 Which of the following factors increases the risk of the particular localisation cancer:
1 - Cervical cancer

lsmusis.lsmuni.lt/Klausimai/Spausdinti?Length=0?Kalba=EN&KategorijaId=128&Kalbos_input=EN&Kalbos=EN&KategorijaEn_input=Gynaecology&K… 2/10
3/27/2019 LSMUSIS
2 - Endometrial cancer
3 - Ovarian epithelial cancer
4 - Breast cancer
A. Anovulation, hyperestraemia
B. Family history, age
C. Infertility, ovulation induction
D. Early sexual activity, a large number of sexual partners

26.19 How does each of the following drugs affect the reproductive system:
1 - Ethynilestradiol
2 - Combined oral contraceptive pills
3 - Clomiphen citrate
4 - Bromocriptin
A. Stimulates ovulation
B. Inhibits prolactin secretion
C. Stimulates regeneration and proliferation
D. Inhibits ovulation
E. Stimulates secretion

26.20 What is characteristic of each of the following forms of amenorrhea:


1 - Primary amenorrhea
2 - Secondary amenorrhea
3 - Amenorrhea of the 1st level
4 - Physiologic amenorrhea
A. Absence of menses during the lactation period
B. Absence of menses until 16 years of age
C. Disappearance of menses for a period longer than 6 months
D. There are cyclic changes in the ovaries, however, there is absence of menses due to anatomic
abnormalities of the genital tract

26.21 What is most appropriate treatment in each of the following cases:


1 - Dysfunctional uterine bleeding in premenopause
2 - Heavy dysfunctional uterine bleeding during in puberty
3 - In cases of incomplete abortion and heavy bleeding
4 - In cases of ectopic pregnancy
A. Hormonal therapy in hemostatic doses
B. Dilation and curretage, followed by histologic examination of the evacuated tissue
C. Laparoscopy and laparatomy
D. Drugs that enhance the coagulative properties of blood
E. Hysteroscopy, dilation and curretage, followed by histologic examination of the evacuated tissue

26.22 Match the symptom that is closely associated with each of the following syndromes:
1 - Climacteric syndrome
2 - Adrenogenital syndrome
3 - Polycystic ovarian syndrome
4 - Hyperplolactinemia
A. Disorders of the menstrual cycle, infertility, enlarged ovaries
B. Galactorrhea, infertility
C. Hirsutism, pseudohermaphrodism
D. Dissorders of the vegetive system, depression
E. Cachexia, enlarges ovaries

26.23 Which treatment os most appropriate for each of the following vaginal infections:
1 - Candidiasis
2 - Trichomoniasis
3 - Bacterial vaginosis
A. Metronidazole or clindamycin
B. Metronidazole or tinidazole
C. Penicillin or gentamycin
D. Flucanozole or clotrimazole

26.24 Point out the types of the vulvovaginal discharge which are typical for each disease, as listed below:
lsmusis.lsmuni.lt/Klausimai/Spausdinti?Length=0?Kalba=EN&KategorijaId=128&Kalbos_input=EN&Kalbos=EN&KategorijaEn_input=Gynaecology&K… 3/10
3/27/2019 LSMUSIS
1 - Candidiasis
2 - Trichomoniasis
3 - Bacterial vaginosis
A. Character: clear or slight milky discharge, pH<4.5. Smear: epithelial cells - dominant, leukocytes - few,
polymorphonuclear clue cells - no, bacteria: Lactobacillus - dominant, Trichomonas vaginalis - no,
Candida - no
B. Character: thin, slight milky pH>4.5. Smear: epithelial cells - dominant, leukocytes - few. Bacteria:
Lactobacillus - decreased, Smear: clue cells, Trichomonas vaginalis - no, Candida - no, KOH - test -
positive
C. Character: thin, greenish - yellow, frothy, pH>4.5. Smear: no clue cells, Trichomonas vaginalis - yes,
Candida - no, KOH test - negative
D. Character: thick, tenacious, cheesy - white, pH<4.5. Smear: clue cells - no, Lactobacillus - dominant,
Candida - blastospore forms, mycelia

26.25 Match the disease with its characteristic symptom:


1 - Acute vulvar infection
2 - Acute vaginitis
3 - Acute salpingitis
4 - Acute endocervitis
A. Foul smelling vaginal discharge
B. A reddened cervix
C. Severe acyclic bleeding
D. A diffuse reddening of the external genitalia
E. Pain in the lower abdomen accompanied by fever

III type tasks. For each question there is one or more correct answers:
A – if correct answers are 1,2,3
B – if correct answers are 1 and 3
C – if correct answers are 2 and 4
D – if correct answer is 4
E – if correct are all answers above

26.26 Which of the following tests help us evaluate ovarian function:


1. Hysteroscopy
2. Basal body temperature
3. Ultrasound
4. Ovarian and pituitary hormones

26.27 Which of the following tests help us evaluate tubal passage:


1. Hysterosalpingography
2. Laparoscopy and dye test
3. Ultrasound
4. Hysteroscopy

26.28 For which of the following conditions is laparoscopy performed:


1. Ectopic pregnancy
2. Ovarian tumor
3. Infertility due tubal occlusion
4. Endometrial cancer

26.29 Ultrasound is useful for the diagnosis of:


1. Uterine myoma
2. Ovarian tumor
3. Ectopic pregnancy
4. Tuboovarian abcess

26.30 Which of the following hormones are produced by the ovaries:


1. Estrogens
2. Progesterone
3. Androgens
4. Gonadotropins

lsmusis.lsmuni.lt/Klausimai/Spausdinti?Length=0?Kalba=EN&KategorijaId=128&Kalbos_input=EN&Kalbos=EN&KategorijaEn_input=Gynaecology&K… 4/10
3/27/2019 LSMUSIS
26.31 Which of the following are clinical singns of ectopic pregnancy:
1. Pain in the lower abdomen
2. Fever
3. Disorders of the menstrual cycle
4. An ultrasound scan showing a gestation sac in the uterine cavity

26.32 Surgical treatment for ectopic pregnancy include:


1. Subtotal hysterectomy
2. Removing of the fallopian tube
3. Bilateral removal of the uterine adnexas
4. Laparoscopy and removal of products of conception from the fallopian tube

26.33 In case of hypomenstrual syndrome, menstruation is:


1. Painful
2. Rare
3. Long
4. Scanty

26.34 Which of the following are considered to be methods of family planning:


1. Intrauterine device
2. Hormonal contraception
3. Barrier methods
4. Elective abortion

26.35 Which of the following tests help us diagnose precancerous lesionsof the cervix:
1. Test with iodine solution
2. Test with acetic acid
3. Colposcopy
4. Hysteroscopy

26.36 Which of the following help us diagnose ovarian tumors:


1. Bimanual and rectovaginal examinations
2. Ultrasound
3. Computer tomography
4. Laparoscopy

26.37 Which of the following symptoms are characteristic of total uterine prolapse:
1. Bulking mass at the introitus
2. Inability to urinate
3. Mucosal sores of the bulking mass
4. Bleeding from the genital tract

26.38 Which of the following are indicated for women who are in menopause complaining of total uterine prolapse
and not planning to have sexual intercourse in the future:
1. Vaginal hysterectomy
2. Amputation of the cervix
3. Operation of uterine lifting and fixing
4. Colpocleisis

26.39 What is characteristic of polycystic ovarian syndrome:


1. Infertility
2. Anovulatory cycles
3. Disorders of the menstrual cycle
4. Hirsutism

26.40 Which of the following symptoms are characteristic of endometriosis:


1. Dark vaginal secretion before and after menstruation
2. Ascites
3. Pain, especially augmented during menstruations
4. Transparent viscous vaginal secretions

IV type tasks. Choose only one best answer

lsmusis.lsmuni.lt/Klausimai/Spausdinti?Length=0?Kalba=EN&KategorijaId=128&Kalbos_input=EN&Kalbos=EN&KategorijaEn_input=Gynaecology&K… 5/10
3/27/2019 LSMUSIS
26.41 An 18 year old patient noticed an increased amount of vaginal discharge 1 week after having casual sex.
She also complained of vaginal soreness and painful urination. The gynecologic examination revealed a thin,
greenish - yellow, foamy vaginal discharge in the posterior vaginal fornix. Upon palpation of the internal genitals,
the uterus and adnexas appeared to be normal.
A. A nonspecific vaginitis
B. Candida vaginitis
C. Trichomonal vaginitis
D. Endometritis

26.42 An 18 year old patient noticed an increased amount of vaginal discharge 1 week after having casual sex.
She also complained of vaginal soreness and painful urination. The gynecologic examination revealed a thin,
greenish - yellow, foamy vaginal discharge in the posterior vaginal fornix. Upon palpation of the internal genitals,
the uterus and adnexas appeared to be normal. What treatment should be prescribed:
A. Antibiotics and physiotherapy
B. Nystatin orally
C. Vaginal douching with a lactoacid
D. Metronidazole orally and locally

26.43 A 6o year old female patient complains of a slight bloody discharge, itching, burning and soreness in the
vagina. Her last menstrual period was 10 years ago. The objective examination revealed a very thin and friable
vaginal epithelium, slight bleeding with a light scrape of the speculum. The uterus and its cervix appeared to be
atrophic. No blood is evident from the cervix. The adnexa are without pathological changes. What is the
diagnosis?
A. Endometrial cancer
B. Endometriosis
C. Vaginitis simplex
D. Postmenopausal vaginitis

26.44 A 6o year old female patient complains of a slight bloody discharge, itching, burning and soreness in the
vagina. Her last menstrual period was 10 years ago. The objective examination revealed a very thin and friable
vaginal epithelium, slight bleeding with a light scrape of the speculum. The uterus and its cervix appeared to be
atrophic. No blood is evident from the cervix. The adnexa are without pathological changes. What treatment
should be prescribed:
A. Wide - spectrum antibiotics
B. Vaginal suppositories with levomycetin
C. Vagina suppositories with estrogen
D. Operative treatment

26.45 A 20 year old woman complains of acute pain in the lower part of abdomen and an increased vaginal
discharge. Her menstrual cysle is normal. Her last menstruation ended 3 days earlier. The objective examination
revealed: fever-38,3°C. The gynecologic examination revealed a purulent vaginal discharge and a reddened
cervix. The uterus and its adnexa are painful to palpation. The pain increases upon moving the cervix. What is the
preliminary diagnosis?
A. Trichomonal vaginitis
B. Pelvic inflammatory disease
C. Ovarian torsion
D. Acute appendicitis

26.46 A 20 year old woman complains of acute pain in the lower part of abdomen and an increased vaginal
discharge. Her menstrual cysle is normal. Her last menstruation ended 3 days earlier. The objective examination
revealed: fever-38,3°C. The gynecologic examination revealed a purulent vaginal discharge and a reddened
cervix. The uterus and its adnexa are painful to palpation. The pain increases upon moving the cervix. What test is
needed to help us make the correct diagnosis:
A. Pap smear
B. Colposcopy
C. Hysterosalpingography
D. Inflammatory response found in the blood tests

26.47 A 20-year-old woman was admitted to a hospital complaining of severe right-sided lower abdominal pain,
nausea and vomiting. The pain was crampy, severe (VAS – 10 pts.). It started suddenly, when jumping in the air
during a sports activity. The symptoms have persisted for 2 hours already. She was experiencing regular menses,
C 4/24. Her LMP was 3 weeks before. On physical examination, her abdomen was painful, tense and with
lsmusis.lsmuni.lt/Klausimai/Spausdinti?Length=0?Kalba=EN&KategorijaId=128&Kalbos_input=EN&Kalbos=EN&KategorijaEn_input=Gynaecology&K… 6/10
3/27/2019 LSMUSIS
peritoneal sign. The patient was virgo intacta. On her ultrasound examination a 5 cm-sized cyst in the right adnexa
was evaluated. A small amount of fluid in the cul-de-sac was detected as well. The uterus and the left adnexa
were without visual abnormalities. Which of the following is the most likely diagnosis:
A. Ovarian cyst
B. Pelvic inflammatory disease
C. Endometriosis
D. Adnexal torsion

26.48 A 20-year-old woman was admitted to a hospital complaining of severe right-sided lower abdominal pain,
nausea and vomiting. The pain was crampy, severe (VAS – 10 pts.). It started suddenly, when jumping in the air
during a sports activity. The symptoms have persisted for 2 hours already. She was experiencing regular menses,
C 4/24. Her LMP was 3 weeks before. On physical examination, her abdomen was painful, tense and with
peritoneal sign. The patient was virgo intacta. On her ultrasound examination a 5 cm-sized cyst in the right adnexa
was evaluated. A small amount of fluid in the cul-de-sac was detected as well. The uterus and the left adnexa
were without visual abnormalities.Which of the following is the most appropriate treatment:
A. Infusion therapy, analgetics
B. GnRh agonists
C. Laparoscopy
D. Laparatomy

26.49 45 years old woman complains of profuse bleeding from genitals. In the past year, her menses were
irregular and very profuse. Ger last menstruation period was 6 weeks before. The vaginal examination: in the
vagina-liquid, dark blood with clots. The cervix is 1,5 cm wide. Uterus is soft upon palpation the uterus is found to
be 8x 6x6cm in size and a soft consistency. The adnexa are without pathological changes. A pregnancy test is
negative. The ultrasound does not show any organic pathology of the uterus. What is the preliminary diagnosis:
A. Endometrial polyp
B. Premenopausal bleeding
C. Impeding abortion
D. Submucosal uterine myoma

26.50 45 years old woman complains of profuse bleeding from genitals. In the past year, her menses were
irregular and very profuse. Ger last menstruation period was 6 weeks before. The vaginal examination: in the
vagina-liquid, dark blood with clots. The cervix is 1,5 cm wide. Uterus is soft upon palpation the uterus is found to
be 8x 6x6cm in size and a soft consistency. The adnexa are without pathological changes. A pregnancy test is
negative. The ultrasound does not show any organic pathology of the uterus. What treatment should be
recommended:
A. Subtotal hysterectomy
B. Hysteroscopy, dilation and curretage and histologic examination of the evacuated tissue
C. Total hysterectomy
D. High dose estrogen treatment

26.51 A 36-year-old woman came to see a doctor complaining of gnawing abdominal pain in lower quadrants. The
pain increased during her periods and sexual intercourse. She was experiencing regular painful menses, C 6 –
7/33. She had spotting before and after periods as well. The patient has given birth twice and has had one
pregnancy termination. LMP was 10 days before. On physical examination, sparse, whitish vaginal discharge
appeared. The uterus was of normal size, teder and immobilized. A tender 6 cm-sized cyst was palpated in the
right adnexa. The left adnexa was without palpable abnormalities. Which of the following is the most likely
diagnosis:
A. Pelvic inflammatory disease
B. Endometriosis
C. Ovarian cancer
D. Ectopic pregnancy

26.52 A 36-year-old woman came to see a doctor complaining of gnawing abdominal pain in lower quadrants. The
pain increased during her periods and sexual intercourse. She was experiencing regular painful menses, C 6 –
7/33. She had spotting before and after periods as well. The patient has given birth twice and has had one
pregnancy termination. LMP was 10 days before. On physical examination, sparse, whitish vaginal discharge
appeared. The uterus was of normal size, teder and immobilized. A tender 6 cm-sized cyst was palpated in the
right adnexa. The left adnexa was without palpable abnormalities. The diagnosis can be confirmed by:
A. Complete blood count
B. Ultrasound examination
C. Diagnostic laparoscopy
lsmusis.lsmuni.lt/Klausimai/Spausdinti?Length=0?Kalba=EN&KategorijaId=128&Kalbos_input=EN&Kalbos=EN&KategorijaEn_input=Gynaecology&K… 7/10
3/27/2019 LSMUSIS
D. Oncogenic markers and sex hormones

26.53 A 46 years old patient comes to the hospital complaining of cramps in the lower abdomen and flanks,
vaginal bleeding, fever until 39°C. She has these complaints for a week, however she did not see a doctor. For two
years her menses havebeen heavy and painful. Her last menstruation was 5 weeks ago. Upon examination: there
are dark, foul smelling vaginal secretions. The cervix is dilated, there is a 5x4 cm necrotic tumor protruding through
the cervix. The body or the uterus is normal, slightly enlarged, the adnexa are normal. What is the diagnosis:
A. Endometrial cancer
B. Polyp of the uterine cervix
C. Cervical pregnancy
D. Pedunculated submucous myoma

26.54 A 46 years old patient comes to the hospital complaining of cramps in the lower abdomen and flanks,
vaginal bleeding, fever until 39°C. She has these complaints for a week, however she did not see a doctor. For two
years her menses havebeen heavy and painful. Her last menstruation was 5 weeks ago. Upon examination: there
are dark, foul smelling vaginal secretions. The cervix is dilated, there is a 5x4 cm necrotic tumor protruding through
the cervix. The body or the uterus is normal, slightly enlarged, the adnexa are normal. What treatment should be
recommended:
A. Subtotal hysterectomy with bilateral adnexectomy
B. Vaginal hysterectomy
C. Torsion of the polyp and fractional uterine curettage
D. Torsion of the myoma and antibiotics

26.55 A 59 years old woman came to the gynecologist complaining of vaginal bleeding. Her last menstruation
period was 7 years ago. She started bleeding a year ago. In the last 2 months the bleeding has intensified,
additionally a foul smelling vaginal discharge also appeared. Upon examination: there are small blood clots in the
vagina, the cervix is normal, the size of the uterus is 9x8x8 cm, soft in consistency. The adnexa are normal. What
is the preliminary diagnosis:
A. Postmenopausal uterine bleeding
B. Endometrial cancer
C. Endometriosis
D. Submucous myoma

26.56 A 59 years old woman came to the gynecologist complaining of vaginal bleeding. Her last menstruation
period was 7 years ago. She started bleeding a year ago. In the last 2 months the bleeding has intensified,
additionally a foul smelling vaginal discharge also appeared. Upon examination: there are small blood clots in the
vagina, the cervix is normal, the size of the uterus is 9x8x8 cm, soft in consistency. The adnexa are normal. What
test is needed to help us make the correct diagnosis:
A. Hysterosalpingography
B. Ultrasound
C. Hysteroscopy, dilation and curretage, histologic examination of the evacuated tissue
D. Laparoscopy

26.57 30 years old woman complains of an increased vaginal discharge. Menstrual cycle is normal. Upon
examination: a 2cm red, plain formation is seen protruding through the cervical canal. Soft tissues are palpated at
its base. The cervix without infiltration. Uterus and adnexas are without pathology. What is the diagnosis:
A. Glandular cyst of the uterine cervix
B. Polyp of the cervical canal
C. A pedunculated cervical myoma
D. Cervicitis

26.58 30 years old woman complains of an increased vaginal discharge. Menstrual cycle is normal. Upon
examination: a 2cm red, plain formation is seen protruding through the cervical canal. Soft tissues are palpated at
its base. The cervix without infiltration. Uterus and adnexas are without pathology. What treatment should be
recommended:
A. Torsion of the polyp and performing a D&C
B. Torsion of the myoma
C. Conisation of the cervix
D. Hormonal treatment with gestagens

26.59 A 30 year old patient come to the emergency room complaining of severe pain in the lower abdomen in
addition to vaginal bleeding. The patient admits that her last normal period was 7 weeks later, in the last 2 weeks

lsmusis.lsmuni.lt/Klausimai/Spausdinti?Length=0?Kalba=EN&KategorijaId=128&Kalbos_input=EN&Kalbos=EN&KategorijaEn_input=Gynaecology&K… 8/10
3/27/2019 LSMUSIS
there had been a "bloody show". The pain is increasing in severity. The pregnancy test is positive. Upon
admission, the patient is stable. Upon examination there is a mild degree of peritioneal irritation. US examination
reveals a small amount of free fluid in the peritoneal cavity, no intrauterine pregnancy is evident. What is
preliminary diagnosis:
A. Torsion of an ovarian cyst
B. Ectopic pregnancy
C. Spontaneous abortion
D. Endometriosis

26.60 30 year old patient come to the emergency room complaining of severe pain in the lower abdomen in
addition to vaginal bleeding. The patient admits that her last normal period was 7 weeks later, in the last 2 weeks
there had been a "bloody show". The pain is increasing in severity. The pregnancy test is positive. Upon
admission, the patient is stable. Upon examination there is a mild degree of peritioneal irritation. US examination
reveals a small amount of free fluid in the peritoneal cavity, no intrauterine pregnancy is evident. What should our
course of action be:
A. Antibiotic therapy and i/v line
B. Uterine d&c
C. Laparascopy
D. Analgetics and a follow up within 1 week

26.61 A 52 year old woman came to the gynecologist complaining of "hot flushes", noght sweating, nervousness,
and an absences of menses for 2 years. Upon vaginal examination: the vaginal mucosa is pink, shiny, there are
pale secretions. The uterus is normal in size and mobile. The adnexa are normal. What is the preliminary
diagnosis:
A. Postcastration syndrome
B. Climacteric syndrome
C. Polycystic ovarian syndrome
D. Androgenital syndrome

26.62 A 52 year old woman came to the gynecologist complaining of "hot flushes", noght sweating, nervousness,
and an absences of menses for 2 years. Upon vaginal examination: the vaginal mucosa is pink, shiny, there are
pale secretions. The uterus is normal in size and mobile. The adnexa are normal. What treatment should be
recommended
A. Androgens
B. Estrogens
C. Estrogens and gestagens
D. Gestagens

26.1 - C 26.2 - D 26.3 - D 26.4 - D 26.5 - C 26.6 - D 26.7 - C 26.8 - B


26.9 - A 26.10 - C 26.11 - D 26.12 - C 26.13 - B 26.14 - B
26.15 26.16 26.17 26.18 26.19 26.20 26.21 26.22
1-B 1-C 1-C 1-D 1-C 1-B 1-E 1-D
2-C 2-A 2-A 2-A 2-D 2-C 2-A 2-C
3-A 3-E 3-B 3-C 3-A 3-D 3-B 3-A
4-D 4-D 4-D 4-B 4-B 4-A 4-C 4-B
26.23 26.24 26.25
1-D 1-D 1-D
2-B 2-C 2-A
3-A 3-B 3-E
4-B
26.26 26.27 26.28 26.29 26.30 26.31 26.32 26.33
4 1 1 1 1 1 2 4
2 2 2 2 3 4
3 3 3 3
4
26.34 26.35 26.36 26.37 26.38 26.39 26.40
1 1 1 1 4 1 1
2 2 2 2 2 3
3 3 3 3 3
4 4

lsmusis.lsmuni.lt/Klausimai/Spausdinti?Length=0?Kalba=EN&KategorijaId=128&Kalbos_input=EN&Kalbos=EN&KategorijaEn_input=Gynaecology&K… 9/10
3/27/2019 LSMUSIS
26.41 - C 26.42 - D 26.43 - D 26.44 - C 26.45 - B 26.46 - D 26.47 - D 26.48 - C
26.49 - B 26.50 - B 26.51 - B 26.52 - B 26.53 - D 26.54 - D 26.55 - A 26.56 - C
26.57 - B 26.58 - A 26.59 - B 26.60 - C 26.61 - B 26.62 - C

lsmusis.lsmuni.lt/Klausimai/Spausdinti?Length=0?Kalba=EN&KategorijaId=128&Kalbos_input=EN&Kalbos=EN&KategorijaEn_input=Gynaecology&… 10/10
3/27/2019 LSMUSIS

Obstetrics
I type tasks. Choose only one best answer

46.1 Positive signs of pregnancy include:


A. Perception of fetal movement
B. Changes in size, shape and consistency of the uterus
C. Detection of fetal movement by the examiner
D. Nausea
E. Three consecutively missed menstrual periods

46.2 What phenomenon is considered to be a true labour:


A. Irregular uterine contraction
B. Bleeding
C. Leakage of amniotic fluid
D. Regular uterine contractions with progressive cervical dilatation

46.3 At what gestational age is ultrasound screening performed:


A. 8 weeks of gestation
B. 6 weeks of gestation
C. 18-20 weeks of gestation
D. 22-24 weeks of gestation

46.4 Which of the following hormones are used in the express diagnosis of pregnancy:
A. Estrogen
B. Human chorionic gonadotropin
C. Placental lactogen
D. Progesterone

46.5 Which of the following test helps mostly in determining the fetal viability at 8 week of gestation:
A. Bimanual examination
B. Hysteroscopy
C. Hysterosalpingography
D. Ultrasound examination

46.6 What treatment is the most appropriate for threatened abortion (9 weeks of gestation):
A. Ethyl alcohol
B. Ethinyl estradiol
C. Intravenous administration of magnesium sulfate
D. Bed rest

46.7 Which gestational age is named as a term pregnancy:


A. 37 - 41 weeks
B. 22 - 28 weeks
C. 22 - 36 weeks
D. 42 weeks

46.8 Which of the following most objectively evaluates the progress of labor:
A. Frequency and intensity of uterine contractions using palpation hand through the abdominal wall
B. Progression of cervical dilation
C. Registration of a transabdominal tocogram
D. Ultrasound examination

46.9 What is the most characteristic for preeclampsia:


A. Hypertension and proteinuria in a previously healthy patient after 20 weeks of gestation
B. Hypertension until pregnancy
C. Proteinuria in the early stages of pregnancy
D. Generalized seizures during pregnancy

46.10 If indicated, when is it most appropriate to perform GTT during pregnancy?


A. 12-14 weeks of gestation
B. 18-20 weeks of gestation
C. 24-28 weeks of gestation
lsmusis.lsmuni.lt/Klausimai/Spausdinti?Length=0?Kalba=EN&KategorijaId=148&Kalbos_input=EN&Kalbos=EN&KategorijaEn_input=Obstetrics&Kate… 1/13
3/27/2019 LSMUSIS
D. 30-32 weeks of gestation

46.11 In case of a normal delivery, when should the amniotic membranes rupture:
A. When the cervical dilation is 1-2 cm
B. At the end of the 1st stage of labour
C. When regular uterine contractions start
D. In the 3rd stage of labour

46.12 What should be administered for Rh isoimmunisation prophylaxis:


A. Oxytocin
B. Antibiotics
C. Vaccine
D. Anti (D) immunoglobulin

46.13 Which of the following conditions are life threatening:


A. 1st degree perineal tear
B. Uterine rupture
C. 2nd degree perineal tear
D. Superficial laceration of the cervix

46.14 Which of the following predispose formation of a vesicovaginal fistula:


A. Laceration of the cervix
B. Rupture of the membranes
C. Arrest in the descent of the fetal head in the pelvis for more than 2 hours

46.15 What is the preferred method of delivery in case when we have a term pregnancy, breech presentation and
placenta previa:
A. Caesarean section
B. If there are no signs of bleeding, vaginal delivery is planned
C. Artificial rupture of the membranes and induction of labor
D. Forceps

46.16 In what case is continuous monitoring of the fetal heart rate indicated:
A. Advanced maternal age
B. A large fetus
C. Placenta is implanted on the posterior uterine wall.
D. Meconium stained amniotic fluid

46.17 How does pregnancy induced hypertension affect the fetus:


A. It increases the chances of fetal infection
B. It increases the chances of fetal malformations
C. It can cause fetal growth restriction
D. It can cause fetal macrosomia

46.18 Which of the following is essential to diagnosing eclampsia:


A. Generalized edema
B. Nausea
C. Convulsions (seizures)
D. Hypertension and proteinuria
E. Visual disturbances

46.19 Which method is the most informative for diagnosing fetal hypoxia during pregnancy:
A. Amnioscopy
B. Counting of fetal movements
C. Auscultation of the fetal heart tones using a stethoscope
D. Dopler velocimetry
E. Biophysical profile

46.20 Which of the following are characteristic for fetal growth restriction?
A. Fetal weight < 10 procentile
B. Lower height
C. Normal weight and a relatively smaller head
D. Normal weight and relatively smaller height
lsmusis.lsmuni.lt/Klausimai/Spausdinti?Length=0?Kalba=EN&KategorijaId=148&Kalbos_input=EN&Kalbos=EN&KategorijaEn_input=Obstetrics&Kate… 2/13
3/27/2019 LSMUSIS
46.21 In which case manual exploration of the uterine cavity necessary in the early postpartum period:
A. No bleeding, but remaining amniotic membranes in the uterus are suspected
B. No bleeding,but patient after previous caesarean section
C. After vacuum extraction of the fetus
D. No bleeding, but we suspect that parts of the placenta have remained in the uterus

46.22 Which of the following antibiotics are contraindicated in pregnancy:


A. Cefazoline
B. Ampicillin
C. Levomycitine
D. Erythromycin

46.23 Which of the following is a prerequisite for applying a forceps:


A. Unruptured membranes
B. Cervical dilation (6-8 cm)
C. The head is engaged
D. Stillbirth
E. Narrow pelvis

46.24 Which of the following is most reliable in determing the presenting part during labor:
A. Amnioscopy
B. Vaginal examination
C. Cardiotocography
D. Leopold’s manoevers

46.25 Which of the following Apgar scores are that of a normal newborn:
A. 0-3
B. 4-6
C. 7
D. 8-10

II type tasks. For each numbered item,selct the one lettered heading that is most closely asssciated with it

46.26 The following drugs are recommended in cases of preterm labor. Please match the drug with indication:
1 - Atosiban
2 - Nifedipin
3 - Antibiotics
4 - Dexamethasone
5 - Betamethazone
A. Maturing of the fetal lungs
B. Tocolysis
C. Prophylaxis for infections
D. Ripening of the cervix

46.27 What average duration of the different phases delivery for primipara:
1 - 1st phase of labor
2 - 2nd phase of labor
3 - 3rd phase of labor
4 - Early postpartum period
A. 2 hours
B. 6-16 hours
C. 30 min-1 hour
D. 24 hours

46.28 Match each clinical condition tp the most closely associated risk factor:
1 - 5 previous deliveries
2 - A previuos caesarean section
3 - A large fetus
4 - Polyhydramnion and a large fetus
A. Cephalopelvic disproportion
B. Uterine atony
C. Rupture of the uterine scar

lsmusis.lsmuni.lt/Klausimai/Spausdinti?Length=0?Kalba=EN&KategorijaId=148&Kalbos_input=EN&Kalbos=EN&KategorijaEn_input=Obstetrics&Kate… 3/13
3/27/2019 LSMUSIS
D. Placenta praevia

46.29 Which formula is most suitable for caclculating the EDD, if we know:
1 - First day of the last menstrual period
2 - Date of conception
3 - Perception of first fetal movements (1st time pregnant woman)
4 - Perception of first fetal movements (for women who had previously given birth)
A. Date+5 months and 1 day
B. Date -3 months + 7days
C. Date +4 months and 18 days
D. Date -3 months -7 days

46.30 Match the most appropriate tactic in case we have a pathologic 3rd phase of labor for each of the following:
1 - 30 min. after delivery, there are no signs of placental separation (the patient is not bleeding)
2 - There are retained parts of the membranes in the uterus (the patient starts to bleed)
3 - There are retained parts of the placenta in the uterus (the patient is not bleeding)
4 - The patient starts to bleed after delivery of the placenta
5 - There are signs of placenta accreta (the patient is bleeding profusely)
A. Uterotonic agents nad manual exploration of the uterus
B. Manual exploration of the uteris
C. Hysterectomy
D. Manual removal of the placenta

46.31 Match the risk factor that is most closely related to the clinical situation:
1 - Preeclampsia
2 - Rupture of the membranes in the presence of a transverse fetal lie
3 - Cephalo - pelvic disproportion
4 - Infection during labor
A. Fetal growth restriction
B. Infection of the neonate
C. Uterine rupture
D. Umbilical cord prolapse

46.32 Match the obstetric case with the sentence that best describes it:
1 - A 21 week gestation with rupture of the membranes
2 - A 23 week gestation and regular uterine contractions and cervical contraction
3 - A 33 week gestation with irregular uterine contractions
4 - A 38 week gestation with rupture of the membranes and no uterine contractions
A. Threatening preterm labor
B. Preterm rupture of the membranes
C. A progressing spontaneous abortion
D. Preterm labor

46.33 Please match the most informative diagnostic method for each of the specific clinical situations named:
1 - Fetal hypoxia
2 - Fetal congenital malformation
3 - Evaluation of amount of amniotic fluid
4 - Fetal growth restriction
5 - Fetal anemia
A. Ultrasound
B. Doplerometry of the fetal circulation
C. Evaluation the amniotic fluid for surfactant
D. Auscultation of the fetal heart tones using a stethoscope
E. Biophysical profile and doplerometry

46.34 Choose the most common route for delivery for a term singleton pregnancy with a normal pelvis:
1 - Occiput posterior
2 - Sinciput anterior
3 - Face presentation
4 - Transverse lie
5 - Oblique lie
A. Spontaneous vaginal delivery
lsmusis.lsmuni.lt/Klausimai/Spausdinti?Length=0?Kalba=EN&KategorijaId=148&Kalbos_input=EN&Kalbos=EN&KategorijaEn_input=Obstetrics&Kate… 4/13
3/27/2019 LSMUSIS
B. Cesarean section
C. Forceps
D. Internal podalic version

III type tasks. For each question there is one or more correct answers:
A – if correct answers are 1,2,3
B – if correct answers are 1 and 3
C – if correct answers are 2 and 4
D – if correct answer is 4
E – if correct are all answers above

46.35 Which of the following drugs are used for tocolysis:


1. Magnesium sulfate
2. Oxytocin
3. Beta blockers
4. Calcium channel blockers

46.36 Which of the following are characteristic symptoms of severe preeclampsia:


1. Systolic Pressure ≥ 160 mmHg or Diastolic Pressure ≥ 110 mmHg
2. Proteinuria (5g/24h)
3. Oliguria (400ml/24 val.)
4. CNS symptoms and visual disturbances
5. Headache

46.37 Which of the following symptoms are characteristic for placenta praevia:
1. A larger fundal height than usual
2. Irregular uterine contractions
3. Painful uterus
4. Sudden onset of bleeding

46.38 Which of the following prove fetal hemolytic disease:


1. Incompatibility of the parents' rhesus factors
2. Unfavorable serologic anamnesis
3. Presence of antibodies in the patient
4. Fetal anemia

46.39 Which of the following obstetric situations predispose for uterine atony:
1. Polyhydramnion
2. Multiple pregnancy
3. A large fetus
4. Breech presentation
5. Preterm labor

46.40 Which of the following complications are characteristic of prolonged labor:


1. Malpresentation
2. Fetal hypoxia
3. Polyhydramnion
4. Birth trauma to the fetus

46.41 For palpating the uterus externally, we usually apply Leopold`s maneuvers. What does the 1st manoever
allow us to evaluate:
1. The position of the fetus
2. The fetal lie
3. The presenting part of the fetus
4. The gestational age approximately

46.42 For palpating the uterus externally, we usually apply Leopold`s maneuvers. What does the 3rd manoever
allow us to evaluate:
1. The position of the fetus
2. The fetal lie
3. The presenting part of the fetus
4. Presenting part ingagement
lsmusis.lsmuni.lt/Klausimai/Spausdinti?Length=0?Kalba=EN&KategorijaId=148&Kalbos_input=EN&Kalbos=EN&KategorijaEn_input=Obstetrics&Kate… 5/13
3/27/2019 LSMUSIS
46.43 Which of the following methods are used to diagnosis fetal hypoxia during pregnancy:
1. Cardiotocography
2. Calculation of Fetal Movement
3. Dopplerometry of the fetal circulation
4. Biophysical profile

IV type tasks. Choose only one best answer

46.44 On May 11, a pregnant patient came to the hospital complaining of irregular uterine contractions that there
increasing in intensity. This is her 3rd pregnancy and 2nd delivery. Her menstruation cycle used to be regular (C 4-
5/24). Her LMP was on August 10. Her perception of the first fetal movements were on December 16. Objective
tests: The fundal height is 33 cm, the membranes are intact, the cervical dilation is 3 cm.What is the gestational
age:
A. 40 weeks
B. 36 weeks
C. 39 weeks
D. 42 weeks

46.45 On May 11, a pregnant patient came to the hospital complaining of irregular uterine contractions that were
increasing in intensity. This is her 3rd pregnancy and 2nd delivery. Her menstrual cycle used to be regular (C4-
5/24). Her LMP was on August 10. Her perception of the first fetal movements were on December 16. Objective
tests: The fundal height is 33 cm, the membranes are intact, the cervical dilation is 3 cm.What is the obstetric
diagnosis:
A. Threatening preterm labor
B. Term pregnancy and false labor
C. Postterm pregnancy and true labor

46.46 On May 11, a pregnant patient came to the hospital complaining of irregular uterine contractions that were
increasing in intensity. This is her 3rd pregnancy and 2nd delivery. Her menstrual cycle used to be regular (C4-
5/24). Her LMP was on August 10. Her perception of the first fetal movements were on December 16. Objective
tests: The fundal height is 33 cm, the membranes are intact, the cervical dilation is 3 cm.What is our next course of
action:
A. Suppress the delivery
B. Monitor the progress of labor and fetal state

46.47 A patient pregnant for the 3rd time comes to the outpatient department complaining of vaginal bleeding.
According to the amenorrhea, she should be 9 weeks pregnant. Her obstetric history includes 2 cases of
spontaneous abortion (10 and 12 weeks). Objective tests: the uterine size corresponds to 12 weeks of pregnancy,
the uterus is soft, there are bloody secretions in the vagina, the cervix appears to be normal. The patient is
admitted to the gynecology department suspecting that she has a hydatidiform mole.Which of the following tests
will help us confirm the preliminary diagnosis:
A. Amnioscopy
B. Vaginal examination
C. Ultrasound
D. Transabdominal amniocentesis

46.48 A patient pregnant for the 3rd time comes to the outpatient department complaining of vaginal bleeding.
According to the amenorrhea, she should be 9 weeks pregnant. Her obstetric history includes 2 cases of
spontaneous abortion (10 and 12 weeks). Objective tests: the uterine size corresponds to 12 weeks of pregnancy,
the uterus is soft, there are bloody secretions in the vagina, the cervix appears to be normal. The patient is
admitted to the gynecology department suspecting that she has a hydatidiform mole.How should this patient be
treated after we confirm the diagnosis:
A. D&C
B. Chemotherapy
C. A laparotomy and hysterectomy

46.49 A patient (1st time pregnant) came to the gynecologist complaining of abdominal cramps, and the presence
of bloody vaginal secretions. It is the 14th week of gestation. The ultrasound shows the fetus to be 14 weeks, and
the placenta is implanted on the anterior wall. What is the obstetric diagnosis:
A. Threatening preterm labor
B. Threatening abortion
C. Partial abruption of a normally implanted placenta
lsmusis.lsmuni.lt/Klausimai/Spausdinti?Length=0?Kalba=EN&KategorijaId=148&Kalbos_input=EN&Kalbos=EN&KategorijaEn_input=Obstetrics&Kate… 6/13
3/27/2019 LSMUSIS
46.50 A patient (1st time pregnant) came to the gynecologist complaining of abdominal cramps, and the presence
of bloody vaginal secretions. It is the 14th week of gestation. The ultrasound shows the fetus to be 14 weeks, and
the placenta is implanted on the anterior wall. How should this patient be treated further?
A. Administration human chorionic gonadotropin
B. D&C
C. Tocolysis and fetal lung maturation
D. Bed rest

46.51 A patient (pregnant for the first time) in her 30th week of gestation came to the doctor because she suspects
that her water broke (a thin and watery vaginal discharge appeared 10 hours earlier). Her obstetric history includes
2 spontaneous abortions, treatment for bacterial vaginosis. At present she is running a fever, no uterine
contractions are evident, the fetus is in occiput presentation, vertical lie. The vaginal examination reveals a cervical
dilation of 3 cm, a foul smelling vaginal discharge. The amniotic membranes are found to be ruptured. What is the
obstetric diagnosis:
A. A preterm labor had started 10 hours earlier in the presence of chorioamnionitis
B. A preterm rupture of the membranes complicated with an intrauterine infection.

46.52 A patient (pregnant for the first time) in her 30th week of gestation came to the doctor because she suspects
that her water broke (a thin and watery vaginal discharge appeared 10 hours earlier). Her obstetric history includes
2 spontaneous abortions, treatment for bacterial vaginosis. At present she is running a fever, no uterine
contractions are evident, the fetus is in occiput presentation, vertical lie. The vaginal examination reveals a cervical
dilation of 3 cm, a foul smelling vaginal discharge. The amniotic membranes are found to be ruptured.What should
our course of action be:
A. Artificial maturation of the fetal lungs according to the quick scheme in addition to antibiotics
B. Tocolysis, maturation of the fetal lungs, and antibiotics
C. Induction of labor and antibiotics

46.53 A 25 year old patient is delivering her 2nd baby. The gestational age is 38 weeks. Regular uterine
contractions commenced 3 hours ago. At present, there is a uterine contraction every 3 min, and lasting for 34-40
seconds. The fetal lie is longitudinal, and there is occiput presentation. The vaginal examination reveals the cervix
to be fully effaced, dialtion 5 cm, unruptured membranes, the amniotic fluid is transparent, and the fetal heart tones
are 140-152 beats/min.How would you describe the obstetric situation:
A. There is a normal progression of preterm labor
B. Preterm labor and primary insufficency in the powers
C. The active phase of a term delivery
D. Term labor and secondary insufficiency of the powers

46.54 A 25 year old patient is delivering her 2nd baby. The gestational age is 38 weeks. Regular uterine
contractions commenced 3 hours ago. At present, there is a uterine contraction every 3 min, and lasting for 34-40
seconds. The fetal lie is longitudinal, and there is occiput presentation. The vaginal examination reveals the cervix
to be fully effaced, dialtion 5 cm, unruptured membranes, the amniotic fluid is transparent, and the fetal heart tones
are 140-152 beats/min. What should our next course of action be?
A. Tocolysis and prolongation of the pregnancy
B. Induction of labor
C. Further follow up of a normal labor, which at the moment is progressing well

46.55 A patient delivering her first baby goes into labor (39 weeks of gestation). A vaginal examinations reveals
that the cervix is 4 cm dilated, membranes are intact, the amniotic fluid is clear, there is occiput presentation, the
presenting part is not engaged, close to it the examiner can feel a pulsating loop of the umbilical cord. What is the
obstetric diagnosis:
A. Beginning of the 1st phase and cephalopelvic disproportion
B. Beginning of the 1st phase and vasa previa
C. Beginning of the 1st phase and umbilical cord prolapse

46.56 A patient delivering her first baby goes into labor (39 weeks of gestation). A vaginal examinations reveals
that the cervix is 4 cm dilated, membranes are intact, the amniotic fluid is clear, there is occiput presentation, the
presenting part is not engaged, close to it the examiner can feel a pulsating loop of the umbilical cord. Which of the
following recommendations is most appropriate:
A. To decrease the risk of umbilical cord prolapse, and under favorable conditions then we should perform
podalic version and extraction
B. An emergency caesarean section

lsmusis.lsmuni.lt/Klausimai/Spausdinti?Length=0?Kalba=EN&KategorijaId=148&Kalbos_input=EN&Kalbos=EN&KategorijaEn_input=Obstetrics&Kate… 7/13
3/27/2019 LSMUSIS
C. Continue on with the natural delivery, however in the event that fetal hypoxia becomes evident then a
caesarean section should be done.

46.57 A 25 year old pregnant woman (delivering for the first time) came to the doctor complaining of irregular and
painful uterine contractions. The gestational age according to the last menstrual period is 35 weeks.+3 d.,
according to the perception of first fetal movementsus vaisiaus judesius (doubtful) 37 weeks. MC is regular (C 3-
5/28). Objekt. tyrimai: The fetal lie is longitudinal, occiput presentation. CTG -normal. Vaginal examination: there
are thin watery secretions in the vagina, the cervix is shortened, soft, and 2 cm dilated. Amnioscopy: the
membranes are intact and the amniotic fluid is clear. What is the preliminary diagnosis:
A. False labor and a term gestation
B. A premature rupture of the membranes and a threatening preterm labor
C. Threatening preterm labor
D. 1st phase of labor in a term pregnancy

46.58 A 25 year old pregnant woman (delivering for the first time) came to the doctor complaining of irregular and
painful uterine contractions. The gestational age according to the last menstrual period is 35 weeks.+3 d.,
according to the perception of first fetal movementsus vaisiaus judesius (doubtful) 37 weeks. MC is regular (C 3-
5/28). Objekt. tyrimai: The fetal lie is longitudinal, occiput presentation. CTG -normal. Vaginal examination: there
are thin watery secretions in the vagina, the cervix is shortened, soft, and 2 cm dilated. Amnioscopy: the
membranes are intact and the amniotic fluid is clear. Which of the following tests is essential for determining our
choice of treatment:
A. Ultrasound
B. Arborisation phenomenon
C. A culture for Group B streptococcus colonisation

46.60 A lady delivered her baby in the ambulance. The newborn weighed 4100 g. The 3 rd phase was normal. The
patient was brought to the hospital 15 min later. Upon examining the patient, a perineal tear extending to the anus
was discovered. The patient is unable to contract her anal sphincter. After closer examination, the anal sphincter
was found to be also torn. What is the obstetric diagnosis?
A. A rectovaginal fistula
B. 2nd degree perineal tear
C. 3rd degree perineal tear
D. A hematoma of the perineal region

46.61 A lady delivered her baby in the ambulance. The newborn weighed 4100 g. The 3 rd phase was normal. The
patient was brought to the hospital 15 min later. Upon examining the patient, a perineal tear extending to the anus
was discovered. The patient is unable to contract her anal sphincter. After closer examination, the anal sphincter
was found to be also torn. What should our next course of action be:
A. If the patient is not bleeding then, we should treat her conservatively
B. Immediately repair the defect under local anesthesia
C. Repair the defect after the post partum period has passed
D. Immmediately repair the defect under general anesthesia

46.62 A 25 year old patient pregnant for the 1st time (30 weeks of gestation) comes to the doctor for a visit. Her
BP 150/100 mmHg (before pregnancy it was 120/80 mmHg), porteinuria was also found. Until now she had gained
12 kg of weight. The patient feels quite well. The fetal state is good. What is the preliminary diagnosis:
A. Hypertensive disease
B. Mild form of preeclampsia
C. Kidney disease
D. Vegetodystonia

46.63 A 25 year old patient pregnant for the 1st time (30 weeks of gestation) comes to the doctor for a visit. Her
BP 150/100 mmHg (before pregnancy it was 120/80 mmHg), porteinuria was also found. Until now she had gained
12 kg of weight. The patient feels quite well. The fetal state is good. Which test is essential to determine the
correct diagnosis:
A. A complete blood count
B. Amount of protein in a 24 hour urine sample
C. Tests for evaluating the liver function
D. All of the above

46.64 A pregnant lady for the 3rd time, and who has not seeked medical advice during her pregnancy came to the
hospital on June 19 upon the onset of regular uterine contractions. The menstrual cycle is not regular (C 5-6/26-

lsmusis.lsmuni.lt/Klausimai/Spausdinti?Length=0?Kalba=EN&KategorijaId=148&Kalbos_input=EN&Kalbos=EN&KategorijaEn_input=Obstetrics&Kate… 8/13
3/27/2019 LSMUSIS
56). The last menstrual period was on August 30. The first fetal movements were on January 23. Physical
examination: fundal height-33 cm, cervical dilation 3 cm, membranes are intact. Ultrasound: Biparietal diameter is
9.4 cm. How should this obstetric situation be described?
A. 1st phase of labor (36 weeks of gestation)
B. 1st phase of labor (39 weeks of gestation)
C. Threatening preterm labor
D. 1st phase of labor of a postterm pregnancy

46.65 A 20 year old patient is pregnant for the 1st time. Towards the end of the 1st phase the patient had tonic
clonic seizures. Physical examination: BP - 180/110 mmHg, edema, uterine contractions every 2 min., and last for
50 sec., full cervical dilation, membranes are ruptured, the presenting part is at +2, fetal heart tones - 100-80
beats/min. How should this obstetric situation be described?
A. Preeclampsia in the end of the 1st phase of labor
B. Preeclampsia in the 2nd stage of labor and fetal hypoxia
C. Eclampsia in the end of the 1st phase of labor
D. Eclampsia in the 2nd stage of labor and fetal hypoxia

46.66 A 20 year old patient is pregnant for the 1st time. Towards the end of the 1st phase the patient had tonic
clonic seizures. Physical examination: BP - 180/110 mmHg, edema, uterine contractions every 2 min., and last for
50 sec., full cervical dilation, membranes are ruptured, the presenting part is at +2, fetal heart tones - 100-80
beats/min. What should our next course of action be:
A. General anethesia and caesarean section
B. Augmentation of labor
C. General anesthesia and forceps delivery

46.67 A woman who is pregnant for the 3rd time and has not seeked medical advice during this pregnancy
delivered a term newborn 3500 g. Her Rh factor is negative. After examining the newborn: his skin is yellow, the
liver and spleen are enlarged. A complete blood count of the baby showed a Hb - 140 g/l, RBC.-35•1012 /l, and
bilirub.-96 mmol/l. What is the diagnosis:
A. Hydrops fetus universalis
B. Hemolytic disease of the newborn (anemic form)
C. Severe neonatal jaundice
D. Physiologic state

46.68 A woman who is pregnant for the 3rd time and has not seeked medical advice during this pregnancy
delivered a term newborn 3500 g. Her Rh factor is negative. After examining the newborn: his skin is yellow, the
liver and spleen are enlarged. A complete blood count of the baby showed a Hb - 140 g/l, RBC.-35•1012 /l, and
bilirub.-96 mmol/l. What should our course of action be:
A. Iron replacement therapy
B. Phototherapy
C. Blood transfusion
D. Observation

46.69 The patient is 29 year old patient pregnant for the 1st time. We found her blood pressure to be 185/110
mmHg, she has generalized edemas, and proteinuria was in her urine. The patient is nauseous and complaining of
a headache. There are spasms of her retinal vessels. Until her pregnancy the patient was healthy, and her blood
pressure at the beginning of pregnancy was 110/70 mmHg. During this pregnancy the patient gained 20 kg,
however the results of ultrasound examination lead us to believe that the fetus suffers from fetal growth restriction.
What is the obstetric diagnosis:
A. Hypertensive disease
B. Eclampsia
C. Severe preeclampsia
D. Obesity and hypertensive disease

46.70 The patient is 29 year old patient pregnant for the 1st time. We found her blood pressure to be 185/110
mmHg, she has generalized edemas, and proteinuria was in her urine. The patient is nauseous and complaining of
a headache. There are spasms of her retinal vessels. Until her pregnancy the patient was healthy, and her blood
pressure at the beginning of pregnancy was 110/70 mmHg. During this pregnancy the patient gained 20 kg,
however the results of ultrasound examination lead us to believe that the fetus suffers from fetal growth restriction.
What should our course of action be:
A. Antihypertensive medication, close observation of the fetal state until term is reached
B. Immediate termination of pregnancy
lsmusis.lsmuni.lt/Klausimai/Spausdinti?Length=0?Kalba=EN&KategorijaId=148&Kalbos_input=EN&Kalbos=EN&KategorijaEn_input=Obstetrics&Kate… 9/13
3/27/2019 LSMUSIS
C. Antihypertensive treatment, and if there is no response to treatment then delivery

46.71 A 24 year old patient (2nd time pregnant) arrived in the hospital an hour ago, already in labor. Her obstetrical
history included a caesarean section for fetal hypoxia. The newborn did not survive. This is a term pregnancy, the
estimated fetal weight is 3300 gr. Physical examination: Longitudinal lie, occiput presentation, the membranes are
intact, the cervix is 1 cm long, 2 cm dilation, the fetal heart tones are 140 beats/min. CTG-normal. What is the
obstetric diagnosis:
A. Unfavorable obstetrics history
B. Dystocia
C. Unfavorable obstetric history and a normal pelvis

46.72 A 24 year old patient (2nd time pregnant) arrived in the hospital an hour ago, already in labor. Her obstetrical
history included a caesarean section for fetal hypoxia. The newborn did not survive. This is a term pregnancy, the
estimated fetal weight is 3300 gr. Physical examination: Longitudinal lie, occiput presentation, the membranes are
intact, the cervix is 1 cm long, 2 cm dilation, the fetal heart tones are 140 beats/min. CTG-normal. What should our
course of action be:
A. A trial of vaginal delivery and close follow up of the maternal and fetal states
B. Caesarean section
C. Discharge from the hospital until her membranes rupture

46.73 A woman complained of leakage of amniotic fluid and irregular uterine contraction (33 weeks of gestation).
Occiput presentation, the fetal state is good. Vaginal examination: the cervix is 2 cm long, 2 cm dilation, the
membranes are ruptured. What is the obstetric diagnosis?
A. Progressing preterm labor
B. Premature rupture of the membranes
C. Preterm rupture of the membranes
D. Primary debility of labor

46.74 A woman complained of leakage of amniotic fluid and irregular uterine contraction (33 weeks of gestation).
Occiput presentation, the fetal state is good. Vaginal examination: the cervix is 2 cm long, 2 cm dilation, the
membranes are ruptured. What should our course of action be:
A. Tocolysis, fetal lung maturation, and close maternal observation for possible infection
B. Augmentation of labor
C. Tocolysis and close observation for possible infection

46.75 After inspecting the placenta, on the maternal side no defects were evident. The blood vessels on the
surface of the amniotic membranes, abruptly end at the edge of the membranes. The woman lost 200 ml of blood
in total and is not bleeding any further. What is the obstetric diagnosis:
A. Retained parts of amniotic membranes in the uterus
B. Retained parts of the placenta in the uterus
C. Retained accessory placenta in the uterus
D. Norm

46.76 After inspecting the placenta, on the maternal side no defects were evident. The blood vessels on the
surface of the amniotic membranes, abruptly end at the edge of the memebranes. The woman lost 200 ml of blood
in total and is not bleeding any further. What should our course of action be:
A. Manual exploration of the uterus under intravenous anesthesia
B. Conservative treatment with uterotonic agents
C. Laparatomy and hysterectomy under general anesthesia
D. Active observation for possible bleeding

46.77 A 28 year old who is pregnant for the 3rd time (36 weeks of gestation) came to the hospital complaining that
at night her bed sheets got wet, later she started feeling painful contractions. Upon examination: longitudinal lie,
vertex presentation, fetal state is good, cerviacal dilation 4-5 cm, fern test is positive. A culture for B group
Streptococcus was positive at 35 weeks of gestation. What is the obstetric diagnosis?
A. Threatening preterm labor
B. Preterm rupture of the membranes and preterm labor
C. Progressing preterm labor and fetal hypoxia
D. Threatening preterm labor and vaginitis

46.78 A 28 year old who is pregnant for the 3rd time (36 weeks of gestation) came to the hospital complaining that
at night her bed sheets got wet, later she started feeling painful contractions. Upon examination: longitudinal lie,

lsmusis.lsmuni.lt/Klausimai/Spausdinti?Length=0?Kalba=EN&KategorijaId=148&Kalbos_input=EN&Kalbos=EN&KategorijaEn_input=Obstetrics&Kat… 10/13
3/27/2019 LSMUSIS
vertex presentation, fetal state is good, cerviacal dilation 4-5 cm, fern test is positive. A culture for B group
Streptococcus was positive at 35 weeks of gestation. What should our course of action be:
A. Maturing the fetal lungs and prolongation of pregnancy while prescribing antibiotics
B. Follow up the progress of labor and prescribe penicillin
C. Tocolysis and antibiotics

46.79 A multipara delivered twins at term. Within 15 delivered the whole placenta however directly after she
started to bleed. Blood is gushing from the uterus. Upon examination: the birth tract is intact, the uterus is
enlarged, soft, the heart rate is 80 beats/min, the BP is 110/75 mmHg. What is the obstetric diagnosis?
A. Placenta accreta
B. Uterine rupture
C. Uterine atony
D. Coagulation disorders

46.80 A multipara delivered twins at term. Within 15 delivered the whole placenta however directly after she
started to bleed. Blood is gushing from the uterus. Upon examination: the birth tract is intact, the uterus is
enlarged, soft, the heart rate is 80 beats/min, the BP is 110/75 mmHg. What should our course of action be:
A. Laparotomy and hysterectomy
B. A curretage of the uterus and uterotonic agents
C. Manual exploration of the uterus and uterotonic agents
D. Uterotonic agents and blood replacement

46.81 A patient went into labor, currently there is a painful uterine contraction every 3 minutes. The contraction
lasts for 40 seconds. These complaints have been present for 10 hours. The membranes are intact. Vaginal
examination: cervical dilation – 10 cm. Occiput presentation, the head lies parallel to left oblique diameter, the
small fontanelle is closer to the symphysis. How should this obstetric situation be described:
A. 1st phase of labor
B. 2nd phase of labor

46.82 A patient went into labor, currently there is a painful uterine contraction every 3 minutes. The contraction
lasts for 40 seconds. These complaints have been present for 10 hours. The membranes are intact. Vaginal
examination: cervical dilation – 10 cm. Occiput presentation, the head lies parallel to left oblique diameter, the
small fontanelle is closer to the symphysis. What conclusion do you reach after you do the vaginal examination:
A. Right occiput anterior
B. Left occiput anterior
C. Left occiput posterior
D. Right sinciput posterior

46.83 A patient pregnant for the 1st time (39 weeks) came to the hospital with regular uterine contractions
thatstarted 4 hours ago. The uterus is contracting every 5 min. An hour ago the membranes ruptured. Vaginal
examination: the cervix is centered, effaced, cervical dilation 4 cm, occiput presentation, the head lies parallel to
right oblique diameter, the small fontanelle is closer to the symphysis. How should this obstetric situation be
described:
A. 1st phase of labor and preterm rupture of the membranes
B. 2nd phase of labor and preterm rupture of the membranes
C. 1st phase of labor and preterm rupture of the membranes
D. 1st phase of labor and rupture of the membranes

46.84 A patient pregnant for the 1st time (39 weeks) came to the hospital with regular uterine contractions
thatstarted 4 hours ago. The uterus is contracting every 5 min. An hour ago the membranes ruptured. Vaginal
examination: the cervix is centered, effaced, cervical dilation 4 cm, occiput presentation, the head lies parallel to
right oblique diameter, the small fontanelle is closer to the symphysis. What conclusion do we make after the
vaginal examination:
A. Sinciput presentation, Left sinciput posterior
B. Sinciput presentation, right sinciput posterior
C. Vertex presentation, right occiput anterior
D. Vertex presentation, left occiput anterior

46.85 A 21 year old patient pregnant for the 1st time (40 weeks of gestation), went into labor 8 hours earlier. After
the membranes ruptured spontaeously, vaginal examination revealed that the cervix is fully effaced, cervical
dilation is 8 cm, the presenting part is at station (-3). The fetal heart tones are rhythmic - 144 beats/min. The
uterine contractions recur every 3-4 min. and last for 40 sec. What is the obstetric diagnosis:

lsmusis.lsmuni.lt/Klausimai/Spausdinti?Length=0?Kalba=EN&KategorijaId=148&Kalbos_input=EN&Kalbos=EN&KategorijaEn_input=Obstetrics&Kat… 11/13
3/27/2019 LSMUSIS
A. Debility of the power
B. Premature rupture of the membranes
C. Preterm rupture of the membranes
D. Normal 1st phase of labor

46.86 A 21 year old patient pregnant for the 1st time (40 weeks of gestation), went into labor 8 hours earlier. After
the membranes ruptured spontaeously, vaginal examination revealed that the cervix is fully effaced, cervical
dilation is 8 cm, the presenting part is at station (-3). The fetal heart tones are rhythmic - 144 beats/min. The
uterine contractions recur every 3-4 min. and last for 40 sec. What should our course of action be:
A. Augmentation of labor
B. Observation of the progress of labor and fetal state
C. Antibiotic prophylaxis
D. Tocolysis

46.59 A 25 year old pregnant woman (delivering for the first time) came to the doctor complaining of irregular and
painful uterine contractions. The gestational age according to the last menstrual period is 35 weeks.+3 d.,
according to the perception of first fetal movementsus vaisiaus judesius (doubtful) 37 weeks. MC is regular (C 3-
5/28). Objekt. tyrimai: The fetal lie is longitudinal, occiput presentation. CTG -normal. Vaginal examination: there
are thin watery secretions in the vagina, the cervix is shortened, soft, and 2 cm dilated. Amnioscopy: the
membranes are intact and the amniotic fluid is clear.What should our next course of action be:
A. Observation of the progress of delivery
B. Tocolysis
C. Concomitant administration of tocolytic agents and gliucocorticoid administration for maturing the fetal
lungs
D. Concomintant administration of tocolytic agents, antibiotics and gliucocorticoids

46.1 - C 46.2 - D 46.3 - C 46.4 - B 46.5 - D 46.6 - D 46.7 - A 46.8 - B


46.9 - A 46.10 - C 46.11 - B 46.12 - D 46.13 - B 46.14 - C 46.15 - A 46.16 - D
46.17 - C 46.18 - C 46.19 - D 46.20 - A 46.21 - D 46.22 - C 46.23 - C 46.24 - B
46.25 - D
46.26 46.27 46.28 46.29 46.30 46.31 46.32 46.33
1-B 1-B 1-B 1-B 1-D 1-A 1-C 1-E
2-B 2-A 2-C 2-D 2-B 2-D 2-D 2-A
3-C 3-C 3-A 3-C 3-B 3-C 3-A 3-A
4-A 4-D 4-B 4-A 4-A 4-B 4-B 4-A
5-A 5-C 5-B
46.34
1-A
2-A
3-B
4-B
5-B
46.35 46.36 46.37 46.38 46.39 46.40 46.41 46.42
4 1 4 4 1 2 2 1
2 2 4 4 3
3 3
4
5
46.43
1
2
3
4
46.44 - C 46.45 - B 46.46 - B 46.47 - C 46.48 - A 46.49 - B 46.50 - D 46.51 - B
46.52 - C 46.53 - C 46.54 - C 46.55 - B 46.56 - B 46.57 - C 46.58 - B 46.60 - C
46.61 - D 46.62 - B 46.63 - D 46.64 - B 46.65 - D 46.66 - C 46.67 - C 46.68 - C
46.69 - C 46.70 - B 46.71 - A 46.72 - A 46.73 - C 46.74 - A 46.75 - C 46.76 - A
46.77 - B 46.78 - B 46.79 - C 46.80 - C 46.81 - B 46.82 - A 46.83 - C 46.84 - D

lsmusis.lsmuni.lt/Klausimai/Spausdinti?Length=0?Kalba=EN&KategorijaId=148&Kalbos_input=EN&Kalbos=EN&KategorijaEn_input=Obstetrics&Kat… 12/13
3/27/2019 LSMUSIS
46.85 - D 46.86 - B 46.59 - B

lsmusis.lsmuni.lt/Klausimai/Spausdinti?Length=0?Kalba=EN&KategorijaId=148&Kalbos_input=EN&Kalbos=EN&KategorijaEn_input=Obstetrics&Kat… 13/13
3/27/2019 LSMUSIS

Pediatrics
I type tasks. Choose only one best answer

22.1 Up to 1 year old infant must get:


A. Breast milk and cereal porridge
B. Breast milk and meat or fish
C. Breast milk or infant formulae, vegetable porridge, cereal porridge, meat, fish, fruits
D. Curds, yougurt, mashed fruits

22.2 Breast milk production is stimulated by:


A. Hearb tea, chocolade
B. Motivation to breastfeed infant, demand breastfeeding, physical and psychological rest, positive emotions
of mother
C. Hospitalization of newborn to separate ward and day rest of mother
D. Vitamins taken by mother

22.3 Breast milk advantages for baby:


A. Breast milk has lactoferin, lizozyme, bifidus factor, interferon, antistaphylococcus factor,
docosaheacsinoic and arahidonice acid and other bioactive factors
B. Breast milk has more proteins and NaCl than cows milk
C. Breast milk has greater amount of proteins, fat, Ca, P than cows milk
D. Breast milk has greater amount of B-lactoglobulin than cows milk

22.4 Baby sits only when being held, he catches hold of a rattl and puts it into his mouth. He can overturn if laid on
the spine. He smiles and laughs when spoken to. How old is the baby:
A. 2 months
B. 4 months
C. 6 months
D. 8 months
E. 10 months

22.5 What phenomena are typical for gastric motility of stomach in newborns and first months infants:
A. The pylorus is atonic and cardia is spastic
B. The pylorus spastic and the cardia is atonic
C. The pylorus and cardia are atonic
D. The pylorus and cardia are spastic

22.6 What is not characteristic of baby‘s water balance:


A. Relatively there‘s more water in blood and intercellular fluid than in adults
B. Dehydration (due to pathological loss of fluid) manifests itself rapidly
C. Dehydration due to pathological loss of fluid manifests itself slowly
D. Fluid needs per kg of the body weight are greater than for adult
E. Baby looses more water per kg of the body weight during perspiration than adult

22.7 Which of the following signs is not a characteristic of dehydration of the body:
A. Dry muccous membranes and lips
B. Tachycardia, tachypnoea
C. Oliguria
D. Vomiting
E. Sunken fontanelle
F. Fatigue, drowsiness and dilated left pupil

22.8 Atopic dermatitis management consist of:


A. Antiallergic diet, decongestants, antibiotics
B. Corticosteroid ointment, emollients, B2-agonists
C. Fe medications, cyclosporine, emollients
D. Antiallergic diet, emollients, corticosteroid ointments, antihistamines, antibiotics

22.9 Which medications are used for long term asthma control:
A. Inhaled CS, inhaled B2 agonists, I generation antihistamines
B. Inhaled CS, inhaled B2 agonists, antileucotrienes

lsmusis.lsmuni.lt/Klausimai/Spausdinti?Length=0?Kalba=EN&KategorijaId=124&Kalbos_input=EN&Kalbos=EN&KategorijaEn_input=Pediatrics&Kate… 1/15
3/27/2019 LSMUSIS
C. Inhaled CS, inhaled B2 agonists, antibiotics

22.10 What diagnosis could be suspected in a child with heartburn, retrosternal pain, chronic cough at night and
hoarseness in the morning:
A. Peptic ulcer
B. Gastroesophageal reflux disease
C. Asthma
D. Irritable bowel syndrome

22.11 Which of proposition is correct for functional dyspepsia in children:


A. Nausea and headacke for 1 day
B. Lower abdominal pain
C. Dyspepsia only during diarrhea episode
D. Dyspepsia and no signs of organic disease

22.12 What are the most important clinical features for exacerbation of chronic gastroduodenitis:
A. Pain and neurovegetative disorders
B. Pain and dyspepsia
C. Pain and gastrointestinal bleeding
D. Dyspepsia and neurovegetative disorders

22.13 Which symptoms is not typical for functional infant colic:


A. Pain in the face
B. Abdominal distension
C. Increased gas per rectum
D. Blood in faeces

22.14 In which of the following conditions ventilatory respiratory failure develops due to depresion of CNS:
A. Acute viral obstructive bronchitis
B. Bronchopneumonia
C. Attack of bronchial asthma
D. Poisoning with traquillizers
E. Acute laryngitis

22.15 Which has to be your second action after opening the airway in I months old baby who developed cardiac
arrest after choking with milk:
A. External chest compression (cardiac massage)
B. Mouth to mouth breathing
C. Mouth to nose and mouth breathing
D. Adrenaline i.v.

22.16 24.17. At what age you must start giving complementory feeding for infants:
A. At 1 mo
B. At 6 mo
C. At 10 mo
D. At 3 mo
E. At 12 mo

22.17 In the evaluation of a 2 year old child with recurrent pneumonia all of the following tests would be indicated
to perform routinely, except:
A. Sweat test
B. Imunoglobulin analysis
C. Pulmonary function tests (spirometry)
D. Complete blood count
E. Sputum culture

22.18 At home pneumonia is diagnosed to a previously healthy 9 year old boy. Which oral antibiotic is the most
suitable:
A. Cefuroxim
B. Amoxicillin
C. Clarithromycin
D. Penicillin

lsmusis.lsmuni.lt/Klausimai/Spausdinti?Length=0?Kalba=EN&KategorijaId=124&Kalbos_input=EN&Kalbos=EN&KategorijaEn_input=Pediatrics&Kate… 2/15
3/27/2019 LSMUSIS
E. Biseptol (Trimethoprim-Sulfamethoxazole)

22.19 A 3 year old girl presents with a history of recurrent pneumonia. On physical examination, wheezing and
crackles are heard, and digital clubbing is evident. The most likely diagnosis is:
A. Pulmonary hypoplasia
B. Pneumocystic pnemonia
C. Pulmonary cystic fibrosis
D. Asthma
E. Laryngomalacia (congenital stridor)

22.20 What is the best indicator of cystic fibrosis:


A. A positive family history of cystic fibrosis
B. The presence of digital clubbing
C. A sweat test with a chloride concentration of 30 mmol/l
D. A sweat test with a chloride concentration of 70 mmol/l
E. Bronchiestasis on ar chest x-ray

22.21 What asthma controller medication, having less side effects is the first choice for children under 7 y:
A. Fluticasone (Flixotide)
B. Na montelucaste (Singulaire, Monkasta)
C. Salbutamol or formoterol
D. Prednisone p/o

22.22 Management of the acute asthma attack at home schould be started with:
A. Opening of the window and letting breathe in fresh air
B. Inhalation of 50 mcg flixotide
C. Inhalation of salbutamol 2-4 puffs every 20 minutes up to 3 times per hour
D. Taking tablet of singulaire (monkasta)

22.23 All of the following statements about asthma are correct except:
A. Asthma is disease with remissions and exacerbations
B. It is a disease of airway hyperresponsiveness
C. The presence of breathlessness is main diagnostic symptom
D. It can be triggered by viral infections, exercise or emotions
E. Inhaled β2 agonists are effective

22.24 Severe pulmonary artery stenosis is characterized by all of the following features except:
A. Increased right ventricular pressure
B. Dyspnea and fatigability on exertion
C. A right to left shunt
D. A loud pulmonary component of the second heart sound (S2)
E. Right ventricular hypertrophy on ECG

22.25 Which of the following features is NOT a characteristic of tetralogy of Fallot:


A. Cyanosis on exertion
B. Paroxysmal loss of consciousness
C. ECG shows right-axis deviation
D. Increased pulmonary vascularity
E. Impaired physical development Growth retardation

22.26 Which of the following laboratory tests the most informative in the begining of acute hepatic failure in
children:
A. Blood urea
B. Serum protein
C. Platelet count
D. Blood glucose
E. Prothrombin time

22.27 Which of the following medications is the most appropriate for prevention of hemarthrosis in child with
hemophilia A:
A. Cryoprecipitate
B. Epsilon-aminocapronic acid

lsmusis.lsmuni.lt/Klausimai/Spausdinti?Length=0?Kalba=EN&KategorijaId=124&Kalbos_input=EN&Kalbos=EN&KategorijaEn_input=Pediatrics&Kate… 3/15
3/27/2019 LSMUSIS
C. Factor VIII concentrate
D. Prednisolone

22.28 Methylene blue (Methylenum coeruleum) is the antidote of one group of the following poisons; which:
A. Neuroleptics
B. Salicylates
C. Poisons that oxidize hemoglobin to methemoglobin (nitrates, nitrites, naphtaline, etc.)

22.29 Renal ultrasonography can provide useful information in the diagnosis of all of the following renal disorders
except:
A. Wilm‘s tumor
B. Glomerular injury in acute tubular necrosis
C. Polycystic kidney disease
D. Ureteropelvic junction obstruction
E. Nephrocalcinosis

22.30 Which of the following statements best characterizes the diagnosis of urinary tract infection in children:
A. The presence of fever localizes the infection to the renal parenchyma
B. The diagnosis is likely if there is pyuria with more than 10 white blood cells per high-power field
C. The diagnosis is likely if shows 105 organisms of a single species
D. Recurrent abdominal pain
E. Recurrent vomiting

22.31 What is the most frequent cause of acute renal failure in babies:
A. Acute poststreptococcal glomerulonephritis
B. Acute pyelonephritis
C. Acute dehydration of the II-III degree
D. Drug poisoning
E. Acute heart failure

22.32 What is the most appropriate therapy for the patient with meningococcal infection:
A. Third generation cefalosporins
B. Erythromycin
C. Ampicilin
D. Laevomycetin / Chloraphenicol
E. Gentamicin

22.33 What therapy predetermines the outcome of fulminant meningococcal septic shock (Waterhouse-
Friderichsen syndrome):
A. Oxygen
B. Antibiotics
C. Fluids
D. Inotropic and vasoactive drugs
E. Corticosteroids
F. Fresh frozen plasma (FFP)
G. All the drugs, mentioned above, administered according to the international guidelines for management

22.34 What is the reason of rapid decrease of blood pressure in case of fulminant meningococcal septic shock:
A. Haemorrhage into skin
B. Adrenal haemorrhage
C. Renal haemorrhage
D. Cerebral haemorrhage
E. Myocardial haemorrhage

22.113 What clinical signs do not support diagnosis of bacterial tonsillitis


A. Runny nose and cough
B. Enlarged regional lymph nodes
C. Purulent plaque on the tonsils
D. Hyperthermia

II type tasks. For each numbered item,selct the one lettered heading that is most closely asssciated with it

lsmusis.lsmuni.lt/Klausimai/Spausdinti?Length=0?Kalba=EN&KategorijaId=124&Kalbos_input=EN&Kalbos=EN&KategorijaEn_input=Pediatrics&Kate… 4/15
3/27/2019 LSMUSIS
22.35 What treatment should be prescribed in the diseases below:
C 1 - Irritable bowel syndrome
A 2 - Crohn's disease
B 3 - Gluten- sensitive enteropathy
A. Immunosuppressive treatment
B. Gluten free diet
C. Diet with high - fiber foods

22.36 What is the mechanism of drug action in treatment of status asthmaticus:


1 - Salbutamol (albuterol)
2 - Corticosteroids
3 - I/v fluids
A. Mucolitic action, relief of cough
B. Broncholitic action: relaxation in large and small airways
C. Inhibits plasma exudation and mucus secretion. Increases expression of airway β2 receptors

22.37 What symptomes are typical for the diseases below:


1 - Gluten- sensitive enteropathy (celiac sprue)
2 - Peptic ulcer
3 - Ulcerative colitis
A. Melena
B. Bloody diarrhea
C. Foul smelling diarrhea

22.38 What symptoms are common for appropriate types of nephritis:


1 - Acute poststreptococcal nephritis
2 - IgA nephropathy
3 - Nephrotic syndrome (minimal change disease)
A. Hematuria, acute onset
B. Acute onset, edema, hematuria, hypertension
C. Nephrotic syndrom, no hypertension, no azothemia

22.39 Find the most suitable group for the diseases below:
1 - Gluten- sensitive enteropathy
2 - Lactose intolerance
3 - Hepatitis A
4 - Irritable bowel syndrome
A. Fermentopathy
B. Immune enteropathy
C. Infection disease
D. Functional gastrointestinal disease

22.40 What symptoms are tipical for appropriate renal disease:


1 - Poststreptococcal acute nefritis
2 - Nephrotic syndrome (minimal schange diseases)
3 - Acute pielonephritis
4 - Acute cystitis
A. Frequent, painful urination
B. Edema, hematuria, hyperestesy
C. Leucocituria, bacteriuria
D. Edema, severe proteinuria

22.41 What is the best way of treatment of following renal diseases:


1 - Acute poststreptococcal nefritis
2 - Acute pielonephritis
3 - Acute cystitis
4 - Nephrotic syndrome (minimal change disease)
A. Antibiotics
B. Restrictive phisical activity, no-salt diet, penicillin, treatment of symptoms
C. Short time treatment with furagin or trimetoprim
D. Diet, corticosteroids

lsmusis.lsmuni.lt/Klausimai/Spausdinti?Length=0?Kalba=EN&KategorijaId=124&Kalbos_input=EN&Kalbos=EN&KategorijaEn_input=Pediatrics&Kate… 5/15
3/27/2019 LSMUSIS
22.42 What are the most typical symptoms for the following blood diseases:
1 - Acute lympholeucosis
2 - Iron deficiency anaemia
3 - Purpura trombocytopenica
A. Normal amount of Hb and of the red blood cells (except for post bleading condition). Platelet count is
reduced. Periferial blood (white cells) is normal, appropriate to age.
B. Decrease amount of Hb, RBC and WBC. High sedimentation rate. Lymphocytosis, lymphoblastosis
and „hiatus leucemicus“.
C. Small count of Hb, the mean corpuscular volume (MCV), mean corpuscular haemoglobin (MCH) and
ferritin are decreased.

22.43 What changes of the cerebrospinal fluid are typical for each type of meningitis:
1 - Purulent meningitis
2 - Aseptic meningitis (viral)
3 - Tuberculous meningitis
A. Cerebrospinal fluid is clear, proteins – 500mg/l, cell count 200x106/l (prevalence of lymphocytes),
glucose - normal.
B. Cerebrospinal fluid is clear, proteins – 1100mg/lg, cell count 200x 106/l (prevalence of lymphocytes),
very low quantity of glucose
C. Cerebrospinal fluid is cloudy, proteins – 1100mg/lg, cell count 3000x 106/l (prevalence of neutrophils),
low quantity of glucose.

22.44 Neuroinfection can be coursed by various microorganisms. What treatment is correct in each case:
1 - Encephalitis coursed by herpes viruses
2 - Tick – borne encephalitis
3 - Neuroborreliosis coursed by Borrelia burgdorferi
A. Suppression of replication of the virus (acyclovir)
B. Antibiotics
C. Symptomic treatment (antipyretics, anticonvulsants ect.)

22.45 What is the best definition of appropriate physical finding:


1 - Crepitation
2 - Flaring of nasal wings
3 - Inspirational stridor
A. The feature of increased airway resistance
B. Partial obstruction of trachea and larynx
C. The new breathing sound during inspiration after renewing bypassing of airway

22.46 What findings of auscullation is most typical for:


1 - Defect of septum intraventricularis
2 - Defect of septum intraatrialis
3 - Ductus arterious apertus
4 - Aortic stenosis
A. Systolic murmur in all cardiac area. The best point for auscultation – right IV intercostal space near
sternum. Murmur spreads to the neck. The II tone is weakened in the right intercostal space.
B. Systolic-diastolic murmur. The best point for auscultation is II left intercostal space. Accentuated the II
tone
C. Harsh systolic murmur in III-IV intercostal space with spreading either to all point of auscultation and to
the neck. The accentuation of II tone.
D. III systolic murmur. The best point for auscultation is the left edge of sternum. Splited and accentuated
the II tone.

22.47 Which infectious disease has the following type of rash:


1 - Maculopapupar rash starting from head and spreading to the trunk and extremities
2 - Blisters, turning into crusts later
3 - Maculopapular rash together with enlargement of occipital lymph nods
4 - The rash begins as erythema on the cheeks
A. Rubella
B. Scarlet fewer
C. Chicken pox
D. Measles

lsmusis.lsmuni.lt/Klausimai/Spausdinti?Length=0?Kalba=EN&KategorijaId=124&Kalbos_input=EN&Kalbos=EN&KategorijaEn_input=Pediatrics&Kate… 6/15
3/27/2019 LSMUSIS
22.48 What X-ray features are typical for appropriate congenital heart disease:
1 - Atrial septal defect
2 - Tetralogy of Fallot
3 - Aortic stenosis
4 - Patent ductus arterious.
A. Hypertrophy of the left ventrice and dilated ascendent aorta
B. Pulmonary vascularity is increased. Enlargement of cardiac diameter, di;ated conus arteriosus, high
fourth arch in the ordinaly frontal chest x-ray. In the oblique view – hypertrophy of left atrium and ventricle.
C. Pulmonary vascularity is increased. Cardiac enlargement is‘ best appreciated on the lateral view:
enlargement of the right ventricle and atrium.
D. The pulmonary vascular markings are diministed. The heart size is normal. The apex is uptilted, and a
concavity is noted in the pulmonary segment, which gives the heart the appearance of a boot.

22.49 What is the most tipical symptoms for following heart diseases:
1 - Cyanosis begins on the first day of life
2 - Different pulse tense in a radialis and a gemoralis
3 - Systolic-diastolic murmur in the left II intercostal area near sternum
4 - Attack of cyanosis and dyspnea
A. Tetralogy of Fallot
B. Patent ductus arteriosus
C. Transposition of the great arteries
D. Coarctation of the aorta

22.50 The specific prophylactic – vaccination against various infectious diseases in children is started in different
age. What is right time for vaccination for mentioned infectious diseases:
1 - On the first days of life
2 - During the first year of life
3 - On the second year of life
A. Whooping cough, diphtheria, poliomyelitis, tetanus
B. Tuberculosis and hepatitis B
C. Measles, rubella, parotitis

III type tasks. For each question there is one or more correct answers:
A – if correct answers are 1,2,3
B – if correct answers are 1 and 3
C – if correct answers are 2 and 4
D – if correct answer is 4
E – if correct are all answers above

22.51 What a the reasons of infant regurgitation in the 1-6 month of age without pylorostenosis or other organic
disease:
1. Eat fast
2. Aerophagia
3. Anatomical peculiarity (Hiss angle)
4. Immature gastronintesinal motility

22.52 What are typical symptoms of Rotaviral diarrhea:


1. Vomiting
2. Fever
3. Watery diarrhea
4. Dehydration

22.53 Food allergy can affect:


1. Skin
2. Respiratory tract
3. Gastrointestinal tract
4. Can cause febrile fever

22.54 In which products you can find vit. D:


1. Milk, butter
2. Fish oil

lsmusis.lsmuni.lt/Klausimai/Spausdinti?Length=0?Kalba=EN&KategorijaId=124&Kalbos_input=EN&Kalbos=EN&KategorijaEn_input=Pediatrics&Kate… 7/15
3/27/2019 LSMUSIS
3. Yolk of egg
4. Green peas, cabbage

22.55 What are the factors predisposing rickets:


1. Praematurity and lack of fresh weather
2. Incorrect feeding and digestion problems
3. Liver and kidneys failure
4. Too much protein in food.

22.56 What is the metabolism of vitamin D:


1. Synthesized in skin under the influence of 253-312 nm ultraviolet rays
2. Is resorbed by the small intestine under the influence of ferments of pancreas.
3. Is resorbed by the small intestine under the influence of bile acids.
4. Is reabsorbed by kidney, when phosphatemia 0,65-1,29 mmol/l.

22.57 What are characteristic signs of rickets:


1. Shoemaker‘s chest, „caput quadratum“, hyperextensibility of joints, „O“ form leg bowing (genu varum)
2. Hyperirritability and hyperperspiration, baldness of the back of the head, craniotabes,
3. Rosary, braslets, „X“ form leg bowing (genu valgum), hypotomy of abdomenal muscles
4. Fragility of bones, short and thick extremities

22.58 What cause hypophosphatemia during calcium methabolic disorders:


1. Hipoparathyroidism
2. Vitamin D deficiency rickets
3. Chronic renal failure
4. De Toni-Debre-Fankoni syndromme

22.59 What are the symptoms of laryngitis:


1. Dry cough, wheezes
2. Inspiratory stridor
3. Expiratory stridor
4. Hoarse voice, barking cough, stridor when breathing

22.60 What are the complication of infant pneumonia:


1. Atelectasis
2. Obstruction syndrome
3. Cardiovascular failure
4. Pleural empyema

22.61 Characteristic signs of epiglottitis:


1. A dull voice / voiceless
2. High fever (>38ºC)
3. Sudden onset
4. Dysphagia

22.62 What are the complications of children asthma:


1. Atelectasis
2. Breathing arrest
3. Pneumonia
4. Pleural empyema

22.63 Which statements confirm the diagnosis of bronchial asthma:


1. Mother‘s father was ill with bronchial asthma
2. Wheezing episodes are recurent from 1,5 years of age
3. Inhalant β2 agonists are effective
4. Hoarse voice and inspiratory stridor are heard from the distance

22.64 What are the most frequent complications of cystic fibrosis:


1. Cor pulmonale
2. Chronic respiratory failure
3. Digital clubbing
4. Mental retardation

lsmusis.lsmuni.lt/Klausimai/Spausdinti?Length=0?Kalba=EN&KategorijaId=124&Kalbos_input=EN&Kalbos=EN&KategorijaEn_input=Pediatrics&Kate… 8/15
3/27/2019 LSMUSIS
22.65 What ammount of fluids must be given to the dehydrated child during 24 hours:
1. The child can drink as much as he wants
2. The ammount of fluids in 24 hours must be equal to diuresis
3. The ammount of fluids must be equal to 100 ml/kg of body mass
4. The ammount of fluids must be equal to the physiological fluid requirement plus the lost fluids

22.66 Which of clinical signs show the dehydration of infant:


1. Dry mucous membranes and lips
2. Hollow fontanel
3. Tachycardia, tachypnoe
4. Oliguria

22.67 What are the shortcomings of intravenal urography:


1. Allergic reactions as well as anaphylaxis may occur because of intravenal injection of contrastic material
2. Renal pelvis may not be seen in patients with disorders of glomerular filtration function
3. The procedure may cause acute renal failure
4. The dose of radiation during this procedure is greater when during izotopic renography

22.68 What is characteristic of child with edema in case of nephrotic syndrome:


1. Severe edemas in the sohole body
2. A child may loose blood plasma, although the total quantity of body water is sufficient
3. The occurece of edemas is directly related with protein concentration in blood serum
4. The child must be treated actively with diuretics and fluid restriction

22.112 What are symtoms of hypocalcemic tetanus:


1. Hyperexcitability of nervous sistem
2. Serum calsium level decreases
3. QT prolonged time
4. Laryngospazm

IV type tasks. Choose only one best answer

22.69 The 5 months old infant, who was born normaly, 6100g of weight, was fed with diluted cow‘s milk, didn‘t
receive vit. D. He was taken outdoors on suny day. The same day in the evening convulsions began. Objective
findings: baldness of the back of the head, softness of the bones of the skull, convulsions are recurent. What is the
cause of convulsions:
A. Epilepsy
B. Meningitis
C. Encephalitis
D. Hypocalcemic tetanus

22.70 The 5 months old infant, who was born normaly, 6100g of weight, was fed with diluted cow‘s milk, didn‘t
receive vit. D. He was taken outdoors on suny day. The same day in the evening convulsions began. Objective
findings: baldness of the back of the head, softness of the bones of the skull, convulsions are recurent. What blood
finding is necessary to confirm the diagnosis:
A. Peripheral blood test
B. Serum Ca level
C. Serum urea level
D. Serum Na level

22.71 The 5 months old infant, who was born normaly, 6100g of weight, was fed with diluted cow‘s milk, didn‘t
receive vit. D. He was taken outdoors on suny day. The same day in the evening convulsions began. Objective
findings: baldness of the back of the head, softness of the bones of the skull, convulsions are recurent. What
findings confirm the diagnosis of hypocalcemic tetanus:
A. Ultrasound examination
B. X-Ray
C. Electrocardiography
D. Electroencephalography

22.72 Healthy 6,5 months old child, who was breastfed till 5,5 month and later barley and wheat porridges has
been introduced into his diet. The infant became anxious for the last 4 weeks, he lost appetite, did not gain weight

lsmusis.lsmuni.lt/Klausimai/Spausdinti?Length=0?Kalba=EN&KategorijaId=124&Kalbos_input=EN&Kalbos=EN&KategorijaEn_input=Pediatrics&Kate… 9/15
3/27/2019 LSMUSIS
started to have episodic diarrhea. Objective findings: pale skin, big bulging abdomen, liver and spleen of normal
size. What is the preliminary diagnosis:
A. Infectious colitis
B. Cystic fibrosis
C. Crohn's disease
D. Gluten- sensitive enteropathy

22.73 A healthy breastfed 6,5 months old child who was breasfed till,5,5 month and later barley and wheat
porridges has been intoduced into his diet. The infant became anxious for the last 4 weeks, he lost appetite, did
not gain weight, started to have episodic diarrhea. Objective findings: pale skin, big bulging abdomen, liver and
spleen of normal size. What treatment should be started first:
A. Large doses oral enzymes
B. Oral antibiotics
C. Regulation of the diet by changing inappropriate meals into proper ones
D. Antiinflammatory treatment

22.74 Infant of 6 months old had rhynitis, a cough, temperature at first was subfebrile. On the 2-3 day the
temperature became higher – 38,8⁰C, his state worsened, short of breath ness appeared, anxilliary muscles were
involved in breathing. In both lungs rales and individual wheezes were heard. Respiratory rate – 80/min. X-Ray –
hyperinflation of both lungs without infiltration. 1-2 days later obstruction syndroms almost disappeared, body
temperature became normal. What is the diagnosis of the disease:
A. Acute pneumonia
B. Bronchiolitis
C. Bronchitis
D. Pertusis (whooping cough)
E. Asthma

22.75 Infant of 6 months old had rhynitis, a cough, temperature at first was subfebrile. On the 2-3 day the
temperature became higher – 38,8⁰C, his state worsened, expiratory stridor appeared, anxilliary muscles were
involved in breathing. In both lungs rales and individual wheezes were heard. Respiratory rate – 80/min. X-Ray –
hyperinflation of both lungs without infiltration. 1-2 days later obstruction syndroms almost disappeared, body
temperature became normal. In what way should the patient be treated. All statements are right, except:
A. The room should be ventilated and the air should be moistened
B. Antibiotics should be given
C. Adequat hidration is necessary
D. Adequat oxigen therapy
E. Patient‘s peace should not be disturbed by various manipulations

22.76 7 months age infant fell ill at home. He is in a fever already for 5 days (>380C). He is coughing and moaning
when breathing, Circumoral cyanosis, nasal flaring and intercostal muscles retractions on inspiration are seen.
Auscultation: rales (crepitations) in the apex of the left lung are heard. Blood test – leukocytosis with a shift to the
left. CRP-50 mg/l. X-Ray – small, somewhere blending infiltrates in the apex of the left lung. What is the diagnosis
of the disease:
A. Pneumonia
B. Bronchitis
C. Bronchiolitis
D. Asthma
E. Laringitis

22.77 7 months age infant fell ill at home. He is in a fever already for 5 days (>380C). He is coughing and moaning
when breathing, Circumoral cyanosis, nasal flaring and intercostal muscles retractions on inspiration are seen.
Auscultation: rales (crepitations) in the apex of the left lung are heard. Blood test – leukocytosis with a shift to the
left. CRP-50 mg/l. X-Ray – small, somewhere blending infiltrates in the apex of the left lung. In what way should
the patient be treated? All the statements are right, except:
A. The room should be cool
B. The position of the child should be often changed
C. Little amounts of liquids should be given frequently
D. Penicillins should be given
E. Cephalosporins should be given

22.78 6 months old infant was brought to the emergency room. 3 months ago he was ill with pneumonia. Got sick
3 days ago: rhinitis, cough and temperature (37,2-3,7,5⁰C) appeared. Last night dyspnea appeared. Objective
lsmusis.lsmuni.lt/Klausimai/Spausdinti?Length=0?Kalba=EN&KategorijaId=124&Kalbos_input=EN&Kalbos=EN&KategorijaEn_input=Pediatrics&Kat… 10/15
3/27/2019 LSMUSIS
findings: not intoxicated, his voice is hoarse when he cries, temperature 37,1⁰C, inspiratory stridor, respiratory rate
– 48/min, heart rate – 140/min. Hyperemia of throat. Visceral organs – normal. Which diagnosis is the most
convincing:
A. Epiglottitis
B. Diphtheria
C. Bronchitis
D. Viral laryngitis

22.79 6 months old infant was brought to the reception 3 months ago. He was ill with pneumonia. Fell ill 3 days
ago: rhinitis, cough and temperature (37,2-3,7,5⁰C) appeared. Last night dyspnea appeared. Objective findings:
not toxicated, his voice is hoarse when he cries, temperature 37,1⁰C, inspiratory stridor, respiratory rate – 32/min,
heart rate – 140/min. Hyperemia of throat. Visceral organs – normal.What is the tactics of treatment:
A. Adrenalin by inhalation and corticosteroides
B. Penicillin i/v
C. Cephalosporins i/v
D. Physiotherapeutic procedures in the neck area and cought assistant

22.80 One year old boy was hospitalized in the intensive care unit. He was healthy before and fever up to 39,50C
started just 6 hours ago. He became letargic, vomited twice. Objective findings: symptoms of intoxication, t0
37,50C, the skin of extremities are cold, petechial (hemorrhagic) rash on the abdomen and tights, bulging fontanel.
Heart rate-200/min. Blood pressure – 60/30 mmHg. What is preliminary diagnosis:
A. Meningitis
B. Meningococcal sepsis and meningitis
C. Meningococcal sepsis
D. Exanthem subitum (Roseola infantum)

22.81 4 year old child felt ill suddenly: t-39⁰C, sore throat, refuses to eat and drink. Objectively: tonsiles enlartged,
red, with purple plaque. Leucocytosis with shift to the left. CRP - 100 mg/l. What is a preliminary diagnosis:
A. Laryngitis
B. Infectious mononucleosis
C. Laryngeal diphtheria
D. Purulent tonsillitis

22.82 4 year old child fell ill suddenly: t-39⁰C, sore throat, refuses to eat and drink. Objectively: tonsills enlarged,
red, with purple plaque. Increased neck lymph nodes. Blood test: leucocytes 15x10⁹/l, CRP - 100 mg/l. What is the
first line treatment?
A. Penicillin, paracetamol (acetaminophen), fluids
B. Claritromycin, ibuprofen, fluids
C. Cefuroxim, paracetamol, fliuds
D. Vancomycin, ibuprofen, fluids

22.83 The attack of heart beating began to 2,5 years old boy, who had WPW phenomemon in ECG. ECG findings
during the attack: heart rate – 280/min., QRS duration – 60 ms. What is the tactics of treatment:
A. Digoxin orally
B. Infusion of KCl
C. Adenosinum i/v
D. Defibrilation

22.84 3 years old boy was hospitalized due to breathlessness. On examination - prolonged expiration, wheezing,
crackles and rhales can be heard. Cough is dry. T-37,5 C. Sneezing. What disease might be suspected:
A. Laryngitis
B. Pertusis
C. Pneumonia
D. Asthma

22.85 3 years old boy was hospitalized due to breathlessness. On examination - prolonged expiration, wheezing,
crackles and rhales can be heard. Cough is dry. T-37,5 C. Sneezing. What treatment should be given?
A. Penicillin
B. Adrenalin (Epinefrin) inhaliations
C. Salbutamol 4 puffs every 20 min 3 time
D. Tavegil (Clemastin) 1/2 tab twice per day

lsmusis.lsmuni.lt/Klausimai/Spausdinti?Length=0?Kalba=EN&KategorijaId=124&Kalbos_input=EN&Kalbos=EN&KategorijaEn_input=Pediatrics&Kat… 11/15
3/27/2019 LSMUSIS
22.86 5 years old girl fell ill with influenza. She was treated with biseptol. Suddenly pallor, jaundice and pain under
left costal margin occured. Father and father‘s father had splenectomy in childhood. Objective findings: jaundiced
scleras, mucous membranes and skin, big painful and hard spleen.What is the preliminary diagnosis:
A. Hepatitis
B. Congenital spherocytic hemolytic anemia
C. Hemolytic-uremic syndrome
D. Hepatopsis – (Gilbert sindrome)

22.87 5 years old girl fell ill with influenza. She was treated with biseptol. Suddenly pallor, jaundice and pain under
left costal margin occured. Father and father‘s father had splenectomy in childhood. Objective findings: jaundiced
scleras, mucous membranes and skin, big painful and hard spleen. What should be the typical sings of peripheral
blood erythrocytes?
A. Anisocytosis
B. Poikilocytosis
C. Sferocytosis

22.88 6 years old boy was brought to the hospital because of continuing bleeding after the extraction of baby
tooth. Mather‘s father had died because of bleeding after injury. Objective findings: thickened left knee joint,
movements are limited, permanemt bleeding from the place of tooth extraction.What is the cause of the bleeding:
A. Hypoplastic anemia
B. Imune trombocytopenic purpura
C. Acute leukosis
D. Hemophilia

22.89 6 years old boy was brought to the hospital because of continuing bleeding after the extraction of baby
tooth. Mather‘s father had died because of bleeding after injury. Objective findings: thickened left knee joint,
movements are limited, permanemt bleeding from the place of tooth extraction. Which blood test is the main for
diagnose:
A. Peripheral blood
B. Coagulograme
C. Test of clotting factors
D. Liver enzymes

22.91 13 years old child has severe episodes of paroxysmal cough for a week. His 1 month old brother became ill
as well. Mother noticed and the infant cough was intensive with the episodes of facial cyanosis. Family doctor sent
the baby to Intensive care unit. What was diagnosed by family doctor:
A. Whooping cough
B. Upper respiratory tract infection
C. Pneumonia

22.92 13 years old child has severe episodes of paroxysmal cough for a week. His 1 month old brother became ill
as well. Mother noticed and the infant cough was intensive with the episodes of facial cyanosis. Family doctor sent
the baby to Intensive care unit. Why the infant was sent to the hospital:
A. Because of apnoe which can be life threatening.
B. Because he must be treated with antibiotics
C. Because he must be treated with antispasmotics

22.93 13 years old child has severe episodes of paroxysmal cough for a week. His 1 month old brother became ill
as well. Mother noticed and the infant cough was intensive with the episodes of facial cyanosis. Family doctor sent
the baby to Intensive care unit. What measures must be used when apnoe begins:
A. Oxygen therapy and/or ventilation
B. Medicine, stimulating centre of breathing
C. Atropine

22.94 8 years old girl was accepted to hospital for suspected viral hepatitis. She had fever and sore throat for one
week, yesterday the jaundice appeared. While examining was seen: swollen tonsils with a lot of pus like material,
swollen lymph nodes on the neck and hepatosplenomegaly. Complete blood count test: Hb-110 g/l, WBC
-20x109/l, Eo. - 2%, Gran. 18%, Lymph. 70%, atypical lymphocytes - 10%, sedimentation rate 20 mm/h. Bilirubin –
30 μmol/l. Hepatic enzymes are elevated moderately. What is the preliminary diagnosis:
A. Infectious viral hepatitis
B. Infectious mononucleosis
C. Leukemia

lsmusis.lsmuni.lt/Klausimai/Spausdinti?Length=0?Kalba=EN&KategorijaId=124&Kalbos_input=EN&Kalbos=EN&KategorijaEn_input=Pediatrics&Kat… 12/15
3/27/2019 LSMUSIS
22.95 8 years old girl was accepted to hospital for suspected viral hepatitis. She had fever and sore throat for one
week, yesterday the jaundice appeared. While examining was seen: swollen tonsils with a lot of pus like material,
swollen lymph nodes on the neck and hepatosplenomegaly. Complete blood count test: Hb-110 g/l, WBC
-20x109/l, Eo. - 2%, Gran. 18%, Lymph. 70%, atypical lymphocytes - 10%, sedimentation rate 20 mm/h. Bilirubin –
30 μmol/l. Hepatic enzymes are elevated moderately. Which test will confirm the diagnosis:
A. Heterophile antibody test and/or specific antibody blood test
B. Bone marrow-puncture
C. HBs antigens in blood test

22.96 8 years old girl was accepted to hospital for suspected viral hepatitis. She had fever and sore throat for one
week, yesterday the jaundice appeared. While examining was seen: swollen tonsils with a lot of pus like material,
swollen lymph nodes on the neck and hepatosplenomegaly. Complete blood count test: Hb-110 g/l, WBC
-20x109/l, Eo. - 2%, Gran. 18%, Lymph. 70%, atypical lymphocytes - 10%, sedimentation rate 20 mm/h. Bilirubin –
30 μmol/l. Hepatic enzymes are elevated moderately. Which medicine must be given:
A. Penicillin
B. Ampicillin
C. Symptomic treatment, corticosteroids in severe cases

22.97 6 year old boy was not vaccinated because of the severe asthmatic bronchitis. He became ill, fever was
390C, he had sore throat and he started vomiting. He was treated at home by parents with acetaminophen
(Paracetamol) for 3 days. The doctor was called only when the boy‘s neck was badly swollen. When examining,
doctor saw thin and gray membrane extending from the tonsil to the soft and hard palate. The boy was
immediately carried to Intensive care department. What is the preliminary diagnosis:
A. Toxical pharyngeal diphtheria
B. Streptococcal pharyngitis
C. Infectious mononucleosis

22.98 6 year old boy was not vaccinated because of the severe asthmatic bronchitis. He became ill, fever was
390C, he had sore throat and he started vomiting. He was treated at home by parents with acetaminophen
(Paracetamol) for 3 days. The doctor was called only when the boy‘s neck was badly swollen. When examining,
doctor saw thin and gray membrane extending from the tonsil to the soft and hard palate. The boy was
immediately carried to Intensive care department. What complications are threatening for life:
A. Diffuse miocarditis
B. Nephrosis
C. Polyradiculoneuritis

22.99 6 year old boy was not vaccinated because of the severe asthmatic bronchitis. He became ill, fever was
390C, he had sore throat and he started vomiting. He was treated at home by parents with acetaminophen
(Paracetamol) for 3 days. The doctor was called only when the boy‘s neck was badly swollen. When examining,
doctor saw thin and gray membrane extending from the tonsil to the soft and hard palate. The boy was
immediately carried to Intensive care department. Which test will confirm the diagnosis:
A. Bacterioscopic and bacteriologic pharyngeal smear test
B. Specific antibody blood test
C. Complete blood count test, CRP

22.100 4 years old boy, who was not vaccinated because of allergic dermatitis, became ill with epidemic parotitis
(mumps). He began vomiting and the headache started 6days later. Which is the preliminary diagnosis:
A. Meningitis
B. Pancreatitis
C. Orchitis

22.101 4 years old boy, who was not vaccinated because of allergic dermatitis, became ill with epidemic parotitis
(mumps). He began vomiting and the headache started 6days later. What symptoms resemble meningitis:
A. Meningeal signs
B. Paralysis of cranial nerves
C. Abdomen tenderness

22.102 4 years old boy, who was not vaccinated because of allergic dermatitis, became ill with epidemic parotitis
(mumps). He began vomiting and the headache started 6days later. Which tests will confirm the diagnosis:
A. Examination of cerebrospinal fluid
B. Blood test
C. Isolation of the virus and serological tests

lsmusis.lsmuni.lt/Klausimai/Spausdinti?Length=0?Kalba=EN&KategorijaId=124&Kalbos_input=EN&Kalbos=EN&KategorijaEn_input=Pediatrics&Kat… 13/15
3/27/2019 LSMUSIS
22.103 9 year old boy, who was in the hospital because of joint pain, become ill: fever 37,50C, some red papules
which became vesicles in two hours, were seen on his trunk. Two more patients were treated in the same room.
What is the preliminary diagnosis:
A. Varicella
B. Herpes zoster
C. Pyodermia

22.104 9 year old boy, who was in the hospital because of joint pain, become ill: fever 37,50C, some red papules
which became vesicles in two hours, were seen on his trunk. Two more patients were treated in the same room.
When must be isolated these two patients, who were in contact:
A. At once
B. 9 days after the contact
C. The isolation is not necessary

22.105 9 year old boy, who was in the hospital because of joint pain, become ill: fever 37,50C, some red papules
which became vesicles in two hours, were seen on his trunk. Two more patients were treated in the same room.
How many days 9 year old boy must be isolated:
A. 5 days after the last rash appears
B. 10 days after the last rash appears
C. 14 days from the beginning of the disease

22.106 15 year old girl became ill: temperature 37,50C, maculopapular rash is seen on skin. Family doctor saw red
soft and hard palates, enlarged occipital lymph nodes and rubella was diagnosed. Which tests will confirm the
diagnosis:
A. PCR and immunological test
B. Bacteriologic pharyngeal smear test
C. Complete blood count test, CRP

22.107 15 year old girl became ill: temperature 37,50C, maculopapular rash is seen on skin. Family doctor saw red
soft and hard palates, enlarged occipital lymph nodes and rubella was diagnosed. Which tests will confirm the
diagnosis: For whom contact with rubella is most dangerous:
A. For 1 year old girls
B. For teenagers
C. For pregnant women

22.108 25 years old teacher from kindergarten with fever and sore throat was consulted by family doctor. Tonsillitis
was diagnosed and ampicillin tablets were prescribed for treatment. At once few children from the kindergarten
became ill: fever, sore throat, vomiting and red, fine, punctuated skin rash was seen. What is the disease of these
children:
A. Scarlet fever
B. Measles
C. Rubella

22.109 25 years old teacher from kindergarten with fever and sore throat was consulted by family doctor. Tonsillitis
was diagnosed and ampicillin tablets were prescribed for treatment. At once few children from the kindergarten
became ill: fever, sore throat, vomiting and red, fine, punctuated skin rash was seen. What caused the infection of
teacher and children:
A. A group B-hemolytic streptococci
B. Staphylococcus aureus
C. Meningococcus

22.110 2 year old boy, not vaccinated because of allergic dermatitis, became ill. He had fever, dry cough, runny
nose, inflamed eyes, and sensitivity to light. Doctor diagnosed nasopharyngitis and prescribed ampicillin and
acetaminophen tablets. Next day mother saw rash on the boy‘s face. The fever started again and the rash was
spreading. The boy was hospitalized. The emergency room doctor saw conjunctivitis, rash on face and neck, tiny
white spots inside the mouth on the inner lining of the cheek, called Koplik's spots. He diagnosed measles. Which
symptoms helped the doctor diagnose measles:
A. Fever
B. Nazopharyngitis
C. Koplik spots and rash

lsmusis.lsmuni.lt/Klausimai/Spausdinti?Length=0?Kalba=EN&KategorijaId=124&Kalbos_input=EN&Kalbos=EN&KategorijaEn_input=Pediatrics&Kat… 14/15
3/27/2019 LSMUSIS
22.111 . 2 year old boy, not vaccinated because of allergic dermatitis, became ill. He had fever, dry cough, runny
nose, inflamed eyes, and sensitivity to light. Doctor diagnosed nasopharyngitis and prescribed ampicillin and
acetaminophen tablets. Next day mother saw rash on the boy‘s face. The fever started again and the rash was
spreading. The boy was hospitalized. The emergency room doctor saw conjunctivitis, rash on face and neck, tiny
white spots inside the mouth on the inner lining of the cheek, called Koplik's spots. He diagnosed measles. How
long he has to be isolated:
A. 6 days from the beginning of the disease
B. 14 days from the beginning of the disease
C. 5 days after the rash appeared

22.1 - C 22.2 - B 22.3 - A 22.4 - B 22.5 - B 22.6 - C 22.7 - D 22.8 - D


22.9 - B 22.10 - B 22.11 - D 22.12 - B 22.13 - D 22.14 - D 22.15 - C 22.16 - B
22.17 - C 22.18 - B 22.19 - C 22.20 - D 22.21 - B 22.22 - C 22.23 - C 22.24 - D
22.25 - D 22.26 - E 22.27 - C 22.28 - C 22.29 - B 22.30 - C 22.31 - C 22.32 - A
22.33 - G 22.34 - B 22.113 - A
22.35 22.36 22.37 22.38 22.39 22.40 22.41 22.42
1-C 1-B 1-C 1-B 1-B 1-B 1-B 1-B
2-A 2-C 2-A 2-A 2-A 2-D 2-A 2-C
3-B 3-A 3-B 3-C 3-C 3-C 3-C 3-A
4-D 4-A 4-D
22.43 22.44 22.45 22.46 22.47 22.48 22.49 22.50
1-C 1-A 1-C 1-C 1-D 1-C 1-C 1-B
2-A 2-C 2-A 2-D 2-C 2-D 2-D 2-A
3-B 3-B 3-B 3-B 3-A 3-A 3-B 3-C
4-A 4-B 4-B 4-A
22.51 22.52 22.53 22.54 22.55 22.56 22.57 22.58
1 1 1 1 1 1 1 2
2 2 2 2 2 3 2 4
3 3 3 3 3 3
4 4
22.59 22.60 22.61 22.62 22.63 22.64 22.65 22.66
2 1 1 1 1 1 4 1
4 2 2 2 2 2 2
3 3 3 3 3 3
4 4 4
22.67 22.68 22.112
1 1 1
2 2 2
3 3 3
4 4
22.69 - D 22.70 - B 22.71 - C 22.72 - D 22.73 - C 22.74 - B 22.75 - B 22.76 - A
22.77 - E 22.78 - D 22.79 - A 22.80 - B 22.81 - D 22.82 - A 22.83 - C 22.84 - D
22.85 - C 22.86 - B 22.87 - C 22.88 - D 22.89 - C 22.91 - A 22.92 - A 22.93 - A
22.94 - B 22.95 - A 22.96 - C 22.97 - A 22.98 - A 22.99 - A 22.100 - A 22.101 - A
22.102 - A 22.103 - A 22.104 - B 22.105 - A 22.106 - A 22.107 - C 22.108 - A 22.109 - A
22.110 - C 22.111 - C

lsmusis.lsmuni.lt/Klausimai/Spausdinti?Length=0?Kalba=EN&KategorijaId=124&Kalbos_input=EN&Kalbos=EN&KategorijaEn_input=Pediatrics&Kat… 15/15
3/27/2019 LSMUSIS

Family medicine
I type tasks. Choose only one best answer

13.1 Which sign is not common for cauda equine syndrome?


A. Increasing weakness in the legs
B. Fever
C. Bladder dysfunction
D. Sensation loss in perineum area

13.2 List the stages of the process of death according to E. Akubler-Roos in correct order?
A. Denial; anger; negotiation; depression; acceptance
B. Depression; anger; denial; negotiation; acceptance
C. Anger; depression; denial; negotiation; acceptance
D. Negotiation; denial; anger; depression; acceptance

13.4 Which symptom is not common for influenza?


A. Acute onset of the disease
B. Fever above 38
C. Muscle ache and headache
D. Runny nose in the beginning stage of the disease

13.5 Which agent causes tonsillitis most often?


A. A group beta hemolytic streptococcus
B. Moraxella catharalis
C. Haemophilus influenza
D. Mycoplasma pneumonia

13.7 Polypharmacy is described as:


A. The use of five or more prescribed and/or non-prescribed drugs.
B. The use of five or more prescribed drugs
C. The use of five or more non-prescribed drugs.

13.8 Which type of Family physician consultation is considered to be the most successful?
A. Doctor centered consultation
B. Patient centered consultation
C. Collaboration

13.9 What would you call an unsuccessful consultation?


A. Lack of time
B. Disturbance from other people during the consultation
C. Verbal-nonverbal mismatch
D. Doctor is unable to identify real causes of the consultation
E. Noncompliance of the patient

13.10 Which range of the increase of the temperature is defined as subfebrile?


A. Up to 38°C
B. 38-39°C
C. 39-41°C
D. > 41°C

13.11 Which type of fever is called long term?


A. a. Fever which lasts more than 7 days
B. Fever which lasts more than 10 days
C. Fever which lasts more than 3 weeks
D. Fever which lasts more than 2 weeks

13.12 What percentages of the whole health issues that patients seek in the health care system should be covered
by primary health care:
A. 90 %.
B. 50-70 %
C. 30 %

lsmusis.lsmuni.lt/Klausimai/Spausdinti?Length=0?Kalba=EN&KategorijaId=115&Kalbos_input=EN&Kalbos=EN&KategorijaEn_input=Family+medicine… 1/8
3/27/2019 LSMUSIS
13.36 The influence of peer group is expressed mostly during:
A. Early teenage years
B. Middle teenage years
C. Late teenage period
D. The influence amongst people of the same age group is low during all stages of the teen years

13.37 According to the World Health Organisation teenagers are defined as:
A. 12-18 years old
B. 14-16 years old
C. 10-19 years old
D. 13-21 years old

13.44 Which symptom is not common for non specific pain in lumbar area (reflex syndrome)?
A. Pain in lumbar and sacrum area.
B. Weakened knee reflex
C. Muscle tension along the spine (paravertebral area)
D. Decreased pain with activity

13.45 What does Primary Health Care (PHC) financing principal of capitation mean?
A. Health care system covers a fixed amount of patient’s visits at PHC centers
B. Health care covers fixed amount of registered citizens at PHC centers
C. Health care covers the costs of service quality and quantity provided

13.46 What type of diagnostic tests Family physician is able to perform if angina is suspected?
A. ECG examination during exercise and ECG at rest
B. Echocardiography and ECG at rest
C. Chest X-ray and exercise ECG
D. ECG at rest and if possible ECG during the onset of angina

II type tasks. For each numbered item,selct the one lettered heading that is most closely asssciated with it

13.13 What symptoms are the most common for the possible causes of lumbar pain?
1 - Oncological diseases (metastasis in the spine at lumbar level)
2 - Sacroileitis
3 - Fracture of the vertebra (compression)
A. Previous medical history which includes trauma, osteoporosis and localized pain in spinal area
B. Unexplained weight loss, subfebrile fever, changes in blood proteins, previous oncological history.
C. Pain in the buttock area and at the back of the thigh which decreases with rest and increases with
activity; increased erythrocyte sedimentation rate

13.15 What is the first choice drug which Family physician should choose for the pain management according to
it’s intensity:
1 - Moderate pain
2 - Severe pain
3 - Weak pain
A. Morphine
B. Tramadol
C. Diclophenac

13.16 Physician in emergency room has a patient with chest pain. Choose characteristic changes in the ECG for
every pathology:
1 - Angina pectoris
2 - Uncomplicated acute posterior wall myocardial infarction
3 - Embolism of large pulmonary artery branches
A. ECG - T wave high, symmetrical, tapered or negative
B. ECG leads III pathological Q, AVR and V1, RSR, ST elevated and turning into a negative T
C. ECG III, AVF pathological Q, ST elevated and turning into positive T.

13.17 Choose the maximum daily dose of the drug that can be prescribed for an adult pain management.
1 - Morphine
2 - Acetaminophen
3 - Tramadol

lsmusis.lsmuni.lt/Klausimai/Spausdinti?Length=0?Kalba=EN&KategorijaId=115&Kalbos_input=EN&Kalbos=EN&KategorijaEn_input=Family+medicine… 2/8
3/27/2019 LSMUSIS
A. 400 mg. a day
B. There is no maximum daily dose
C. 1000 mg. a day
D. 3000 - 4000 mg. a day

13.38 What stages of adolescence are characterized by following aspects?


1 - Early adolescence
2 - Middle adolescence
3 - Late adolescence
A. Increased interest in appearance
B. Future planning
C. Increased likelihood of risky behavior

13.39 What developmental aspects of adolescence are characterized by following issues?


1 - Physical development
2 - Cognitive development
3 - Psychosocial development
A. Striving for independency
B. Signs of puberty
C. Abstract thinking

13.47 Which antibiotic should Family physician choose for the certain agent:
B 1 - Group A β hemolytic streptococcus
A 2 - Mycoplasma pneumoniae
C 3 - Haemophilus influenzae
A. Clarithromycin
B. Phenoxymethylpenicillin
C. Amoxicillin

13.48 Which test is used by the Family physician to confirm certain disease:
1 - Ig M antibodies against the pathogen
2 - Normal white blood cell count
3 - CRP> 50 mg / l
A. Otitis caused by bacteria
B. Mycoplasma infection
C. Acute obstructive viral bronchitis

13.49 Choose the correct sentence for each statement:


1 - Multimorbidity at patient level could cause
2 - Multimorbidity at national health care system level could cause
3 - Multimorbidity at Family physician’s work could cause
A. Frequent and sometimes not rational use of health care services; use of time; the need of holistic
approach.
B. High costs of the health care system for the compensated drugs, prolonged hospital stay and rehab
demands
C. Social isolation, psychological tension, disability and cognitive functions disorders

13.50 Which term best describes appropriate communication?


1 - Treat every patient equally
2 - Valuation of the patient’s opinion
3 - Sympathy and compassion for the patient
4 - Acceptance and understanding of the patient’s problem
A. Reflection
B. Partnership
C. Justification
D. Respect

13.51 Which words best describe the concepts of Family physician’s consultation:
1 - Open ended questions
2 - Generalization
3 - Encouragement
4 - Specification

lsmusis.lsmuni.lt/Klausimai/Spausdinti?Length=0?Kalba=EN&KategorijaId=115&Kalbos_input=EN&Kalbos=EN&KategorijaEn_input=Family+medicine… 3/8
3/27/2019 LSMUSIS
A. What was your health like before?
B. "Continue", "yes, I understand", "OK", "Yeah" "m-m”...
C. "You're telling me that the pain decreases with certain medications, tell me more..."
D. “You are complaining of a headache and you are using medications, we will perform tests, is there
anything else you could add?...”

13.52 Assign adequate groups of diseases that can cause long term fever:
1 - Lyme disease
2 - Lymphoma
3 - Lupus erythematous
4 - Sarcoidosis
A. Infections
B. Systematic connective tissue disease
C. Oncological diseases
D. Granuliomic diseases

13.53 What symptoms are present in conjunction with long term fever in the diseases listed below:
1 - Lymphadenopathy
2 - Skin rashes
3 - Cough and respiratory failure
4 - Nausea and vomiting
A. Trichinelosis
B. Tuberculosis
C. Infectious mononucleosis
D. Lyme disease

III type tasks. For each question there is one or more correct answers:
A – if correct answers are 1,2,3
B – if correct answers are 1 and 3
C – if correct answers are 2 and 4
D – if correct answer is 4
E – if correct are all answers above

13.18 Which of the following Family physician’s consultation stages are correct?
1. Initiation of the consultation
2. Setting the reason of the referral
3. An objective evaluation of the patient
4. Discussion about the condition and analysis of the situation
5. Further treatment and testing specifications
6. End of the consultation

13.19 What are the tasks of the Family physician during the consultation:
1. Include the patient in to the problem solving as much as possible, assess patient's thoughts and
expectations
2. Allow the right amount of time for the patient.
3. Find out the link between different problems and identify the main ones
4. Redirect patient's attention from unimportant topics

13.20 What systemic infections can cause long-term fever?


1. Tuberculosis
2. Infective endocarditis
3. Unexplained bacteremia
4. Viral infection

13.21 In which case you could foreknow problems in a new family:


1. Two young people are from very different genetically roots and cultural traditions (religion, education,
nationality, age, and social status)
2. There are big problems in the families of the young couple (divorce, remarriage, and separation)
3. The girl gets pregnant before the marriage
4. The young couple is in conflict with their families

13.22 Adolescent are in increased risk of suicide if he (she):


lsmusis.lsmuni.lt/Klausimai/Spausdinti?Length=0?Kalba=EN&KategorijaId=115&Kalbos_input=EN&Kalbos=EN&KategorijaEn_input=Family+medicine… 4/8
3/27/2019 LSMUSIS
1. Recently experienced important personal loss
2. Long time are troubled by problems in families
3. Had suicidal attempts in the past
4. Use alcohol and drugs

13.23 A 30 years old man complains of lumbar pain lasting for three days which occurred when lifting heavy
weight. There is muscle tension present along the spine, flexion and extension of the lumbar spine are limited.
There is no muscle strength loss or sensory disturbances in the legs. During the straight leg raise test the patient
complains of pain arising from the abdomen when the leg is raised up to 20 ° from the horizontal surface. What is
the treatment strategy for this patient?
1. A few days of rest until the acute pain subsides
2. Prescription of a short-acting non-steroidal anti-inflammatory drugs for 5-7 days
3. Prescription of muscle relaxants for 1-week
4. Referal to physiotherapist for procedures after acute pain is reduced

13.24 What are the possible causes of lumbar pain caused by non-vertebral pathology?
1. Kidney tumor
2. Chronic prostitis
3. Descendent kidney
4. Salivary gland stones

13.25 What are the likely causes of oncological patient’s cough?


1. ACE inhibitors
2. Throat tumor
3. V. Cava superior syndrome
4. Ascites

13.26 According to the WHO recommendations influenza vaccination is particularly recommended for:
1. Residents of care and maintenance treatment facilities
2. Patients with chronic obstructive pulmonary disease
3. Older people (> 65 years)
4. Infants up to 6 months who are born in "influenza season”

13.27 Typical symptoms of viral respiratory tract infection are:


1. Runny nose
2. Cough
3. Fever
4. Hoarseness of the voice

13.28 The health care system which is focused at the Primary Health Care (PHC) is effective to deal with problems
associated with multimorbidity because:
1. Family physician coordinates patient's care
2. PHC is continuous and all-encompassing
3. PHC is based on the principles of teamwork
4. PHC focuses not only on the treatment of diseases but also on their prevention

13.40 What is the impact of the peer group during the development of a teenager?
1. Promotes greater integration into the family of a teenager
2. Promotes involvement into risky behavior
3. Strengthens values formed in the family
4. A favorable effect on a self-esteem

IV type tasks. Choose only one best answer

13.29 Family physician visited the patient with the advanced pancreatic cancer. The oncological hospital
recommended palliative care. Patient complains of nausea, pain in the central part of abdomen and the
assessment with VAS (visual analog scale) - 9 points. What should be the family physician’s actions attending the
patient on the first day after hospital treatment?
A. Referral to hospital
B. Prescription of paracetamol and metoclopramide
C. Prescription of morphine and metoclopramide.
D. Prescription of ketoprofen or diclofenac. Treatment effect to be evaluated after 1 day

lsmusis.lsmuni.lt/Klausimai/Spausdinti?Length=0?Kalba=EN&KategorijaId=115&Kalbos_input=EN&Kalbos=EN&KategorijaEn_input=Family+medicine… 5/8
3/27/2019 LSMUSIS
E. Treatment of tramadol and acetaminophen, assessment of effectiveness in 24 hours

13.30 Patient has lung cancer which has grown over to the main bronchi. Only palliative treatment is applied.
Relatives called family physician because the patient is complaining of shortness of breath. There is no sign of
cyanosis on the patient’s lips, auscultation of lungs is symmetrical on both sides, no crackles audible. Patient is
restless, confused, complains of pain in the lumbar area, where the diagnosis of L4 metastasis was previously
made. Pain intensity according to VAS - 5 points. Patient is treated with morphine. What are the correct actions of
the Family physician?
A. Prescription of angiotensin-converting enzyme inhibitor and beta-adrenoreceptor antagonists, add dioxin
if necessary.
B. Treatment with oxygen and salbutamol inhalations, coughing-enhancing drugs.
C. Increased doses of morphine, prescription of glucocorticoids, oxygen if treatment is ineffective.
D. Treatment with oxygen, bronchodilators, and benzodiazepines.

13.31 A 45 years old man comes to the family physician. He complains of runny nose, severe nasal congestion,
and feeling of general weakness, fatigue and headaches for three days. He has no fever. During physical
examination there is visible redness of the nasopharynx. Patient has a history of sinusitis. What would be the
further Family physician’s tactics?
A. Immediate prescription of amoxicillin and systemic decongestants
B. Prescription of systemic decongestants, perform blood count and CRP tests, decide the prescription of
antibiotics according to the test results
C. Immediate referal to otorhinolaryngology medical consultation because the patient has previously had a
history of sinusitis

13.32 The mother came to Family physician with a five year old boy, who is coughing for 1 week. Body
temperature was less than 38OC and only on the day of the visit morning rose up to 38.8 ° C, the boy became
sluggish, irritable. After physical examination family doctor found redness of the nasopharynx, heard crackles in
the right lower lung. What is the likely diagnosis and further Family physician’s tactics?
A. The right lung pneumonia; prescription of ciprofloxacin, perform complete blood count, CRP and lung X-
ray
B. The right lung pneumonia; prescription of amoxicillin and antipyretics, perform complete blood count,
CRP and lung X-ray
C. Acute upper respiratory tract infection, symptomatic treatment

13.33 88-year-old woman is living alone and suffers from multiple chronic diseases (polyarthrosis, ischemic heart
disease, arterial hypertension, heart failure, type II diabetes). It is difficult for her to independently look after herself
and purchase groceries. Because of that she became more agitated lately and her sleep is disturbed, she laments
more. What should be a family physician’s tactics any time soon?
A. Evaluate the treatment of chronic diseases and their control and send the patient to a cardiologist,
endocrinologist consultation
B. Evaluate patient for possible mental health problems and initiate social assistance
C. Advise patient to visit primary health care center more often.

13.34 75 years old patient is suffering from multiple chronic diseases: chronic obstructive bronchitis, arterial
hypertension, benign prostatic hyperplasia, and gout. He still smokes and is overweight. Takes his medication
regularly. Lately his general condition is stable and there are no new complaints. What should be the best family
doctor's tactics for 12 months of health care planning to this patient?
A. Teach the patient about the elimination of risk factors and practice adequate control of chronic diseases.
Evaluate his medications and their possible side effects and interactions. Investigate his mental health.
B. Refer the patient for planned testing in the second-level hospital at least once a year.
C. Prescribe referrals to specialist doctors to assess the chronic non-infective disease control.

13.35 The 16 years old teenager came to Family physician for the medical certificates required to deliver to the
school. During the conversation, the teenager tells doctor that he hates school. The doctor discovers that he is
being bullied for several years in the classroom and in recent years it is particularly intensified. What should be
further Family physician’s tactics?
A. Evaluation of the physical examination and the test results; give a medical certificate for the school.
B. Evaluation of the physical examination and the test results; give a medical certificate for the school. Offer
teenager to talk about bullying with his parents
C. Evaluation of the physical examination and the test results; give a medical certificate for the school.
Collect adolescent’s psychosocial history and identify potential suicide risk.

lsmusis.lsmuni.lt/Klausimai/Spausdinti?Length=0?Kalba=EN&KategorijaId=115&Kalbos_input=EN&Kalbos=EN&KategorijaEn_input=Family+medicine… 6/8
3/27/2019 LSMUSIS
13.42 Patient is 72 years old and has fever up to 38.5 ° C. He is coughing with sputum for more than a month.
Shortness of breath intensified a week ago. He is a smoker since youth. Suffers from chronic obstructive
pulmonary disease. During the auscultation of the lungs wet crackles are heard on the left side of the lung. Tests:
White blood cells - 13.67 10 x 109 / l; CRP 114 mg / l; ESR - 98 mm / h; chest x-ray - infiltration in the left lung.
Family physician treated suspected pneumonia with amoxicillin for 10 days, but fever continued and chest X-ray
image unchanged. What kind of disease should family doctor suspect and what should be further treatment
tactics?
A. Left lung pneumonia; change to the other antibiotic therapy.
B. Referral to a hospital for the specification of diagnosis because of suspected prolonged pneumonia,
suspected malignancy process or pulmonary tuberculosis.
C. Referral for hospitalization due to prolonged pneumonia.

13.43 16 years old adolescent girl address family physician. She would like to get the prescription for oral
contraceptives as she is sexually active for 6 months. Previously she used spermicides that caused local side
effects. She has not contraindications to use oral contraceptives. Adolescent girl ask physician not to talk with her
parents about this consultation. What should be the strategy of family physician in this situation?
A. To prescribe oral contraceptives and to inform about that girl’s parents.
B. To prescribe oral contraceptives and to explain how to use condoms, not to inform parents.
C. Not to prescribe contraceptives, but explain how to use condoms, not to inform parents.

13.54 Patient is 58 years old and came to the family doctor complaining of severe pain in the left side of the 7th-
8th rib area on axillar line. The pain occurred 3 days ago. Patient has fever up to 37.5oC and dry cough for one
week. Pain worsens when coughing, weakens when he lies down on the right side. Vesicular breath is heard
during the auscultation without crackles. What kind of illness would you suspect and what would be your
investigation tactics?
A. Pneumonia of the left side; preform pulmonary X-ray, CRP and complete blood count;
B. Dry pleuritis; preform pulmonary X-ray, CRP and complete blood count;
C. Intercostal neuralgia evaluation of intercostal pain, skin condition (rashes); perform CRP and complete
blood count.

13.55 48-year-old man is complaining of severe pain in the abdomen lasting for 3 days. The pain is spreading in to
his right leg. The calf muscle cramps are present at night. The patient begins to complain of pain in the right leg
and right buttock during the left raised leg test when the leg is raised up to 30 °. Which statement about the
patient's condition is true?
A. Positive raised leg test shows pressure on the nerve roots.
B. Possible diagnosis of cauda equine syndrome.
C. Rekomenduojama skubi neurochirurginė operacija.
D. Recommended urgent consultation of neurologist

13.1 - B 13.2 - A 13.4 - D 13.5 - A 13.7 - A 13.8 - C 13.9 - D 13.10 - B


13.11 - C 13.12 - A 13.36 - B 13.37 - C 13.44 - B 13.45 - B 13.46 - D
13.13 13.15 13.16 13.17 13.38 13.39 13.47 13.48
1-B 1-B 1-A 1-B 1-A 1-B 1-B 1-B
2-C 2-A 2-C 2-D 2-C 2-C 2-A 2-C
3-A 3-C 3-B 3-A 3-B 3-A 3-C 3-A
13.49 13.50 13.51 13.52 13.53
1-C 1-D 1-A 1-A 1-C
2-B 2-B 2-D 2-C 2-D
3-A 3-A 3-B 3-B 3-B
4-C 4-C 4-D 4-A
13.18 13.19 13.20 13.21 13.22 13.23 13.24 13.25
1 1 1 1 1 1 1 1
2 3 2 2 2 2 2 2
3 3 3 3 3 3 3
4 4 4 4
5
6
13.26 13.27 13.28 13.40
1 1 1 2
2 2 2 4

lsmusis.lsmuni.lt/Klausimai/Spausdinti?Length=0?Kalba=EN&KategorijaId=115&Kalbos_input=EN&Kalbos=EN&KategorijaEn_input=Family+medicine… 7/8
3/27/2019 LSMUSIS
3 3 3
4 4
13.29 - C 13.30 - C 13.31 - B 13.32 - B 13.33 - B 13.34 - A 13.35 - C 13.42 - B
13.43 - B 13.54 - B 13.55 - A

lsmusis.lsmuni.lt/Klausimai/Spausdinti?Length=0?Kalba=EN&KategorijaId=115&Kalbos_input=EN&Kalbos=EN&KategorijaEn_input=Family+medicine… 8/8
3/27/2019 LSMUSIS

Preventive medicine
I type tasks. Choose only one best answer

43.1 Which wording best describes the population strategy in preventive medicine?
A. Health check-ups involving entire population
B. Promoting healthy lifestyles and healthy environment in entire population
C. Health education through mass media
D. Primary prevention measures only among people with risk factors for non-communicable diseases
E. Health education of patients attending health care services

43.2 Which wording best describes the high-risk strategy in preventive medicine:
A. Early detection of non-communicable diseases
B. Early detection and control of risk factors
C. Annual screenings of adult population
D. Risk factor detection
E. Follow-up of patients with non-communicable diseases

43.3 Which preventive strategy – population or high-risk – is more effective in reducing morbidity and mortality due
to non-communicable diseases and why:
A. High-risk strategy, because the resources are used more efficiently for health care
B. Population strategy, because the measures used help to detect all patients with non-communicable
diseases
C. High-risk strategy, because the preventive measures are used for those having the highest risk for non-
communicable diseases development
D. Population strategy, because the preventive measures are applied for entire population, preventing the
risk factor development and decreasing the risk of non-communicable diseases
E. Population strategy, because all people with risk factors are detected

43.4 Which of the following is not recommended for cancer prevention:


A. To quit smoking and avoid alcohol
B. To avoid excessive exposure to the sun
C. To keep normal weight
D. To use vitamins in pills
E. To eat fruits and vegetables every day

43.5 Which wording best describes a screening:


A. Periodical health examination of patients with certain diseases
B. Annual health examination of persons with risk factors
C. Health examination, by which unrecognised diseases or risk factors are identified by tests that can be
applied rapidly on large scale
D. Health examination of persons from certain social groups

43.6 What is the most important reason of increasing prevalence of obesity:


A. Heredity
B. Excess of energy due to unhealthy nutrition and low physical activity
C. Increasing prevalence of endocrine abnormalities
D. Growing consumption of hormones and othermedicine inducing weight gain

43.7 The factor mostly related to the increase of serum cholesterol level is:
A. Dietary cholesterol
B. Saturated fatty acids
C. High energy intake
D. Total fat intake

43.8 Which statement is false:


A. Regular usage of food supplements decreases the risk of coronary heart disease and cancer
B. Obesity is a risk factor for endometrium cancer
C. High salt intake is associated with hypertension
D. High red meat intake increases the risk of colon cancer

43.9 Obesity is not associated with following cancers:

lsmusis.lsmuni.lt/Klausimai/Spausdinti?Length=0?Kalba=EN&KategorijaId=145&Kalbos_input=EN&Kalbos=EN&KategorijaEn_input=Preventive+medi… 1/7
3/27/2019 LSMUSIS
A. Colon
B. Endometrium
C. Stomach
D. Renal
E. Postmenopausal breast cancer

43.10 Which of the measures listed below is the most effective in decreasing alcohol consumption in Lithuania:
A. Health education and social alcohol counter advertising
B. Increasing the price of alcohol
C. Total prohibition of alcohol
D. The set of legislative measures aimed at regulation of alcohol sale and consumption in combination with
health education
E. Specialised medical care

43.11 What is the main strategy for hypertension control in community:


A. Organisation of mass screening campaigns
B. Primary prevention of hypertension
C. Education of health professionals
D. Education of patients

43.12 The most effective method to decrease the smoking prevalence is:
A. Health education and support to smokers for quitting
B. The set of legislative, medical-education and public intervention measures.
C. Total ban on tobacco advertising and restriction of smoking in public places
D. Implementation of smoking prevention programmes in schools

43.13 The most important condition for successful smoking cessation is:
A. A proper course of nicotine replacement therapy
B. Support provided by friends and close relatives
C. High motivation for quitting
D. Arranging follow-up visits (prevention of relapse)

43.14 Which screening test is recommended for middle aged people:


A. Haemoglobin
B. Urinalysis for bacteriuria
C. Blood cholesterol
D. Blood urea
E. Blood potassium

43.39 What level of blood pressure (BP) is considered to be elevated:


A. Systolic BP >=140 mmHg and/or diastolic BP >= 90 mmHg
B. Systolic BP >=160 mmHg and/or diastolic BP >= 95 mmHg
C. Systolic BP >= 140 mmHg and/or diastolic BP >= 95 mmHg
D. Systolic BP >= 170 mmHg and/or diastolic BP >= 100 mmHg
E. Systolic BP >=175 mmHg and/or diastolic BP>=105 mmHg

II type tasks. For each numbered item,selct the one lettered heading that is most closely asssciated with it

43.15 What is the effect of the following nutrients on the level of low density lipoprotein cholesterol:
1 - Trans fatty acids
2 - Dietary cholesterol
3 - n-6 polyunsaturated fatty acids
A. Increasing
B. Decreasing
C. No effect

43.16 What is the effect of the following factors on BP:


1 - Weight gain
2 - Reduced sodium intake
3 - High intake of sugar
4 - Reduced alcohol consumption
5 - Reduced physical activity

lsmusis.lsmuni.lt/Klausimai/Spausdinti?Length=0?Kalba=EN&KategorijaId=145&Kalbos_input=EN&Kalbos=EN&KategorijaEn_input=Preventive+medi… 2/7
3/27/2019 LSMUSIS
A. Increasing
B. Decreasing
C. No effect

43.17 The risk of which non-communicable diseases (matched in numbers) could be decreased by controlling the
risk factors matched in letters:
1 - Stroke
2 - Colon cancer
3 - Diabetes
4 - Chronic bronchitis
5 - Dental caries
A. Sugar consumption
B. Smoking
C. Obesity
D. Hypertension

43.18 Which drug group belongs each of the following substances:


1 - Benzodiazepines
2 - Hallucinogens
3 - Opiates
4 - Stimulants
5 - Volatile substances
A. Codeine
B. Acetone
C. Diazepam
D. LSD (Lysergic acid diethylamide)
E. Cocaine hydrochloride

43.19 Indicate, please, which level of hypertension prevention these activities correspond to:
1 - Early hypertension detection
2 - Rehabilitation of patients with stroke
3 - Prevention of obesity
4 - Risk factor management among hypertensive patients
A. Primary prevention
B. Secondary prevention
C. Tertiary prevention

43.20 What diseases are caused by the following substances:


1 - Carbon monoxide
2 - Tar
3 - Nicotine
4 - Volatile irritating substances
5 - Radioactive elements
6 - Aromatic carbohydrates
7 - Nitrosamines
A. Ischaemic heart disease
B. Cancer
C. Chronic bronchitis

43.21 The food pyramid assists in the selection of the food groups required for healthy nutrition. Put the following
food groups to the appropriate layer of the pyramid starting from the base:
1 - Fruit and vegetables
2 - Meat, milk and diary products
3 - Fats and sugars
4 - Grain products and potatoes
A. Base of the pyramid
B. The middle part of the pyramid
C. Top of the pyramid

43.43 How to change the diet in order:


1 - The intake of fat
2 - The intake of saturated fatty acids
lsmusis.lsmuni.lt/Klausimai/Spausdinti?Length=0?Kalba=EN&KategorijaId=145&Kalbos_input=EN&Kalbos=EN&KategorijaEn_input=Preventive+medi… 3/7
3/27/2019 LSMUSIS
3 - The intake protein
4 - The intake of folic acid
5 - The intake of fibres
A. Too low
B. Too high
C. Adequate

43.44 What is the effect of the following factors on BP:


1 - Weight gain
2 - Reduced sodium intake
3 - High intake of sugar
4 - Reduced alcohol consumption
5 - Reduced physical activity
A. Increasing
B. Decreasing
C. No efect

43.45 Please point out which of the two types of tobacco dependence is indicated by the following specific
symptoms:
1 - Smoking mostly with friends or while being among smoking people
2 - Smoking shortly after awakening, before breakfast
3 - Smoking few cigarettes and different brands
4 - Often being unaware of lighting up a cigarette
5 - Smoking heavily and independent of situation
6 - Difficulty to refrain from smoking for several hours
7 - Irregular smoking, dependent on situation
8 - Being very much aware of the fact when not smoking
A. Pharmacological (physical) dependence
B. Psychological (psychosocial) dependence

III type tasks. For each question there is one or more correct answers:
A – if correct answers are 1,2,3
B – if correct answers are 1 and 3
C – if correct answers are 2 and 4
D – if correct answer is 4
E – if correct are all answers above

43.22 What activities correspond to the primary prevention:


1. Immunization
2. Control of tobacco
3. Promotion of healthy nutrition
4. Early detection of diseases
5. Control of risk factors among patients with coronary heart disease (CHD)

43.23 What is recommended for CHD prevention:


1. Risk of CHD assessment
2. Control of hypercholesterolaemia
3. Hypertension treatment
4. Prescription of aspirin
5. Promotion of physical activity

43.24 What measures listed below are recommended for cancer prevention?
1. Restriction of alcohol consumption
2. To increase of fresh vegetables and fruits intake
3. Screening of cervical cancer
4. Avoidance of foods containing additives
5. To improve quality of life of patients with cancer

43.25 The possible explanations of positive effect of physical activity on reducing risk of coronary hearth disease
are:
1. Increase in the level of high density lipoproteins

lsmusis.lsmuni.lt/Klausimai/Spausdinti?Length=0?Kalba=EN&KategorijaId=145&Kalbos_input=EN&Kalbos=EN&KategorijaEn_input=Preventive+medi… 4/7
3/27/2019 LSMUSIS
2. Decrease in BP level
3. Increase in coronary blood flow
4. Reducing of thrombosis

43.26 For which of the diseases listed below obesity is the risk factor:
1. Coronary heart disease
2. Breast cancer
3. Type 2 diabetes
4. Osteoporosis
5. Stomach cancer

43.27 What are nutrition recommendations for the prevention of CHD:


1. Replace fatty meat and meat products with beans, legumes, poultry or lean meat
2. Consume fish as a main dish 1-2 times a week
3. Eat a variety of vegetables and fruits (at least 400 g per day)
4. Avoid consumption of hard margarine
5. Use vegetable oil in cooking

43.28 Which of the following conditions is an indication for the treatment with ethyl alcohol?
1. Coronary heart disease
2. Threatened miscarriage
3. Hypothermia
4. Methyl alcohol intoxication
5. Alcoholic psychosis

43.29 The best indicator for changes in prevalence of alcohol dependence is:
1. The number of alcohol-related accidents
2. Increase in number of alcoholics
3. Alcohol-related crimes
4. Changes in incidence of alcoholic psychosis

43.30 Secondary prevention of hypertension includes:


1. Promotion of healthy lifestyle
2. Early hypertension detection
3. Early diagnostic and treatment of congestive heart failure
4. Effective treatment of patients with hypertension

43.31 Non-pharmacological hypertension treatment includes:


1. Weight control
2. Restriction of dietary sodium
3. Increasing of physical activity
4. Smoking control
5. Reduction of alcohol intake

43.32 What are the most efficacious approaches to the primary prevention of hypertension:
1. Weight loss
2. Reduced alcohol consumption
3. Reduced sodium intake
4. Stress management
5. Calcium and magnesium pill supplementation

43.33 What is the most adequate treatment for a nicotine dependent patient who failed to quit on his own:
1. Reducing gradually the number of cigarettes smoked
2. Mouth cavity rinsing with a 0,25% solution of silver nitrate
3. Prescribing psychostimulating drugs
4. Prescribing varenicline or bupropion, or nicotine replacement therapy (nicotine chewing gum or patches)

43.34 Which of the following diseases is the most causal-related to smoking:


1. Lip and mouth cancer
2. Coronary heart disease
3. Chronic bronchitis and emphysema
4. Lung cancer

lsmusis.lsmuni.lt/Klausimai/Spausdinti?Length=0?Kalba=EN&KategorijaId=145&Kalbos_input=EN&Kalbos=EN&KategorijaEn_input=Preventive+medi… 5/7
3/27/2019 LSMUSIS
43.35 Which combination of the risk factors increases the risk of CHD mostly:
1. Smoking, overweight, alcohol
2. Hypercholesterolemia, overweight, stress
3. Physical inactivity, hypercholesterolemia, smoking
4. Smoking, hypertension, hypercholesterolemia
5. Hypertension and alcohol

43.36 What test should be performed in screening in order to assess the risk of non-communicable disease:
1. BP measurement
2. Urinanalysis
3. Height and weight measurement
4. Chest X-ray

43.37 The most important condition for solving patient’s alcohol problems is:
1. The help of qualified psychologist
2. Health education
3. Effective pharmacological treatment
4. The understanding of the problem and willingness to solve it

43.38 What are WHO recommendations for adult physical activity:


1. At least 150 minutes of moderate-intensity aerobic physical activity or at least 75 minutes of vigorous-
intensity aerobic physical activity throughout the week
2. Aerobic activity should be performed in bouts of at least 10 minutes duration.
3. Muscle-strengthening activities should be done involving major muscle groups on 2 or more days a week
4. 4. Daily morning exercises lasting 5 min. are recommended
5. Every day walking at least 1 km.

43.1 - B 43.2 - B 43.3 - D 43.4 - D 43.5 - C 43.6 - B 43.7 - B 43.8 - C


43.9 - C 43.10 - D 43.11 - B 43.12 - B 43.13 - C 43.14 - C 43.39 - A
43.15 43.16 43.17 43.18 43.19 43.20 43.21 43.43
1-A 1-A 1-D 1-C 1-B 1-A 1-A 1-B
2-A 2-B 2-C 2-D 2-C 2-B 2-B 2-B
3-B 3-C 3-C 3-A 3-A 3-A 3-C 3-C
4-B 4-B 4-E 4-B 4-C 4-A 4-A
5-A 5-A 5-B 5-B 5-A
6-B
7-B
43.44 43.45
1-A 1-B
2-B 2-A
3-C 3-B
4-B 4-B
5-A 5-A
6-A
7-B
8-A
43.22 43.23 43.24 43.25 43.26 43.27 43.28 43.29
1 1 1 1 1 1 4 4
2 2 2 2 2 2
3 3 3 3 3 3
4 4 4
5 5
43.30 43.31 43.32 43.33 43.34 43.35 43.36 43.37
2 1 1 4 4 4 1 4
4 2 2 3
3 3
4
5
43.38
1
lsmusis.lsmuni.lt/Klausimai/Spausdinti?Length=0?Kalba=EN&KategorijaId=145&Kalbos_input=EN&Kalbos=EN&KategorijaEn_input=Preventive+medi… 6/7
3/27/2019 LSMUSIS
2
3

lsmusis.lsmuni.lt/Klausimai/Spausdinti?Length=0?Kalba=EN&KategorijaId=145&Kalbos_input=EN&Kalbos=EN&KategorijaEn_input=Preventive+medi… 7/7
3/27/2019 LSMUSIS

Occupational medicine
I type tasks. Choose only one best answer

33.1 Basic categories of occupational hazards are:


A. physical, biological, radiation, dust;
B. physical, biological, social
C. noise, vibration, food additives, radiation
D. physical, biological, chemical, psychosocial, ergonomics

33.2 Which of these occupational health service models has highest competence in occupational health and fits to
a large enterprise?
A. group service
B. primary health care unit model
C. big industry model (in-plant) service
D. hospitals outpatient clinics

33.3 Occupational diseases that are increasing worldwide:


A. angiosarcoma of the liver
B. noise- related diseases, occupational hepatitis
C. pneumoconioses, poisonings
D. immunoallergic disorders, neurological problems associated with chemicals at work, stress

33.4 Acute silicosis develops within


A. 3-5 years of exposure to any dust
B. 10-15 years of exposure to silica dust
C. 3-5 years of exposure to silica dust
D. more than 15 years of exposure to silica dust

33.5 Pleural plaques are associated with exposure to


A. coal dust
B. microwaves
C. asbestos
D. all mentioned above

33.6 A rupture of the eardrum may occur as a result of exposure to noise at a level above
A. 140 dB
B. 105 dB
C. 85 dB
D. 75 dB

33.7 „White finger“ phenomenon is associated with exposure to


A. electromagnetic fields
B. noise
C. vibration
D. ionizing radiation
E. asbestos

33.8 Exposure to coal dust is associated with


A. lung cancer, pleural mesothelioma ,ostesarcoma
B. asbestosis, hepatitis, pleural mesothelioma
C. asbestosis, lung cancer, pleural and peritoneal mesotheliomas
D. bronchitis, pneumoconiosis

33.9 Occupational asthma it’s


A. irreversible bronchospasm that follows exposure to isocyanates, nickel, chromium, platinum salts, cotton
and wood dust
B. bronchitis and lung fibrosis that follows exposure to mould hay, wheat flour, mushroom compost, pigeon
droppings, silica dust
C. reversible bronchospasm that follows exposure to isocyanates, nickel, chromium, platinum salts, cotton
and wood dust
D. irreversible bronchospasm that follows exposure to flour, grain dust

lsmusis.lsmuni.lt/Klausimai/Spausdinti?Length=0?Kalba=EN&KategorijaId=135&Kalbos_input=EN&Kalbos=EN&KategorijaEn_input=Occupational+m… 1/3
3/27/2019 LSMUSIS
33.10 Sensoneural peripheral neuropathy, parasthesias, numbness and tingling in hands can be associated with
exposure to
A. asbestos
B. microwaves
C. noise
D. vibration
E. all mentioned above

33.11 Symptoms of acute hypersensitivity pneumonitis are


A. acute febrile 48-hour pneumonia
B. fibrotic changes in lungs without any symptoms
C. acute bronchospastic reactions, “Monday fever”

33.12 Chronic noise -induced hearing loss occur as a consequence of exposure to noise level above
A. 140 dB
B. 105 dB
C. 85 dB
D. 75 dB

33.13 Exposure to asbestos is associated with


A. lung cancer, pleural mesothelioma, ostesarcoma
B. asbestosis, hepatitis, pleural mesothelioma
C. asbestosis, lung cancer, pleural and peritoneal mesotheliomas
D. asthma, lung cancer, pleural mesothelioma, silicosis

33.14 Occupational exposure that is not associated with asthma


A. pollens
B. chromium
C. asbestos
D. flour, grain dust

III type tasks. For each question there is one or more correct answers:
A – if correct answers are 1,2,3
B – if correct answers are 1 and 3
C – if correct answers are 2 and 4
D – if correct answer is 4
E – if correct are all answers above

33.15 Common causes of occupational bronchial asthma


1. nickel, chromium
2. flour
3. toluene diizocyanate
4. vanadium
5. silica dust

33.16 Occupational disease it is


1. a disease linked directly to the occupation, including diseases of accidental origin and accidents at work
2. a disease that often have their origin in a combination of factors, in which the working environment plays
only a partial role
3. the consequence of exposure for a certain amount of time to a risk which exists during the normal
exercise of an occupation
4. a worker’s disease, that was diagnosed during the normal exercise of an occupation

33.17 Hypersensitivity pneumonitis occurs as a result of exposure to


1. mouldy hay, wheat flour
2. pigeons proteins
3. mushroom compost
4. coal dust
5. asbestos

33.18 For hypersensitivity pneumonitis is characteristic


1. fever
lsmusis.lsmuni.lt/Klausimai/Spausdinti?Length=0?Kalba=EN&KategorijaId=135&Kalbos_input=EN&Kalbos=EN&KategorijaEn_input=Occupational+m… 2/3
3/27/2019 LSMUSIS
2. bibasilar crackles
3. specific antibodies
4. eosonophilia

33.19 Occupational factors associated with lung cancer


1. silica dust
2. coal dust
3. cadmium
4. grain dust
5. toluene diizocyanate

33.20 Which of these occupational processes belong(s) to the 1st group of carcinogens according IARC?
1. Furniture and cabinet making
2. Dye production
3. Shoemaking
4. Car production

33.1 - D 33.2 - C 33.3 - D 33.4 - C 33.5 - C 33.6 - A 33.7 - C 33.8 - D


33.9 - C 33.10 - D 33.11 - A 33.12 - C 33.13 - C 33.14 - C
33.15 33.16 33.17 33.18 33.19 33.20
1 1 1 1 1 1
2 3 2 2 3 2
3 3 3 3

lsmusis.lsmuni.lt/Klausimai/Spausdinti?Length=0?Kalba=EN&KategorijaId=135&Kalbos_input=EN&Kalbos=EN&KategorijaEn_input=Occupational+m… 3/3
3/27/2019 LSMUSIS

Social medicine
I type tasks. Choose only one best answer

7.1 Method of standardization is used for:


A. Estimating significance of the difference of the rates
B. Calculating standard deviation
C. Calculating confidence interval
D. Adjusting (standardizing) rates to permit comparison of data sets

7.2 Crude death rate in two regions, which are different according to age structure was: in region A - 8.0‰, in
region B - 9.5‰. Conclusion: crude death rate in region B is higher than that in region A. Is this conclusion right?
A. Right
B. Not right, because death rates by age are not presented
C. Not right, because structure of causes of death is not estimated
D. Not right, because adjusted (standardized) rates were not estimated

7.3 The arithmetic mean of the height of 130 (n) girls is 140 cm, standard error of the mean is ± 0.3 cm. Indicate
the confidence interval with the confidence level of 95%.
A. 140 ± 0.3
B. 140 ± 1.96 × 0.3
C. 140 ± 2.58 × 0.3
D. 140 ± 3.31 × 0.3

7.4 The difference between arithmetic means of two populations will be statistically significant if sample size is
bigger than 30, confidence level 95%, and t criterion:
A. < 1.96
B. >1.96

7.5 For the estimation of the effectiveness of vaccine two groups of children were formed: there were 400
vaccinated children, among them 2.0 ± 0.7% became ill during a year; 600 children were not vaccinated, among
them with the same disease became ill 4.5 ± 0.8% during a year; z criterion 2.2. Is vaccine effective:
A. No
B. Yes (the probability of this statement is 95%)

7.6 By which method the birth rate is estimated:


A. ((Live births + stillbirths) × 100)/ Average annual population number
B. (Number of live births × 1000) / Average population number in the beginning of the year
C. (Number of live births × 1000) / Average population number in the end of the year
D. (Number of live births × 1000) / Average annual population number

7.7 By which method the crude death rate is estimated:


A. (Number of deaths × 1000) / Average annual population number
B. (Number of deaths × 1000) / Average population number in the beginning of the year
C. (Number of deaths of 1 year of age and older × 1000) / Average annual population number
D. (Number of deaths of 1 year of age and older × 1000) / Number of population of 1 year and older

7.8 By which method infant mortality rate is most commonly estimated:


A. (Number of deaths of infants × 1000) / Number of live births
B. (Number of deaths of infants × 100) / Total number of deaths during a year
C. (Number of deaths of infants × 1000) / (1/3 of live births during previous year + 2/3 of live births during
the same year)
D. (Number of deaths of infants × 1000) / (Number of live births + number of stillbirths)
E. (Number of deaths of infants × 1000) / (Average annual population number)

7.9 If the out-patient treatment is needed after the discharge from a hospital, physician together with the head of
the department can give a sick-leave before patient must arrive to health care institution or be visited by the
physician at home, but no longer than:
A. 5 calendar days
B. 7 calendar days
C. 10 calendar days
D. 14 calendar days

lsmusis.lsmuni.lt/Klausimai/Spausdinti?Length=0?Kalba=EN&KategorijaId=109&Kalbos_input=EN&Kalbos=EN&KategorijaEn_input=Social+medicine… 1/6
3/27/2019 LSMUSIS
7.10 What does the life expectancy at birth mean:
A. Average age of the deceased
B. Average age of the population
C. The number of years of life a particular generation would live on average, if current age-specific mortality
rates were to continue

7.11 Which cause of death of Lithuanian males under the age of 45 years is the most common:
A. Cardiovascular diseases
B. Cancers
C. Accidents, poisonings and injuries
D. Respiratory diseases

7.12 Which cause of death of Lithuanian adult population is the most common:
A. Cardiovascular diseases
B. Cancers
C. Accidents, poisonings and injuries
D. Respiratory diseases

7.13 Basic principles of total quality management are:


A. Orientation to clients, continuous development, everyone involvement
B. Optimal management, overcoming competitors, meeting clients needs
C. Competitive price, good technique, good quality medications

7.14 The main elements of quality in health care:


A. Patient quality, professional quality, management quality
B. The quality of pharmaceuticals, qualified doctors, affordable price
C. A pleasant atmosphere, attention to the patient, low price

7.15 Health care quality costs consist of:


A. Medication prices, salaries, maintenance of the premises prices
B. External costs, internal costs, quality measurement and assessment costs, improvement costs
C. Money, contributed by patients and patietns insurance instiutions

7.16 Professional quality is assessed according to:


A. Patients claims
B. Conclusions of medical audit
C. Quality costs
D. Amount of the services

7.19 How quality of doctors’ work is measured in Lithuania?


A. According standard medical practice
B. According rules of good medicine practice
C. According standard set by the law
D. According the criteria of maximum efforts

7.20 What kind of duty has doctor in Lithuania when he is not working and not in the place of his work?
A. The same as when he is working
B. To provide full emergency care
C. To provide part of emergency care, e.g. first aid
D. None

7.21 When health care institution is allowed to cancel provision of health care services to patient?
A. Never
B. When patient refuses to pay for additional health care services, not reimbursed by the state
C. When patient in the process of his treatment refuses to cooperate with the doctor
D. When patient gets involved in personal conflicts with the doctor and writes claims against him

7.22 Patient, who wants to file a complaint for damage done to his health compensation, has to submit it:
A. To court
B. To health care institution
C. To commission for allocating damage done to the patients health
D. To Lithuanian health care ministry

lsmusis.lsmuni.lt/Klausimai/Spausdinti?Length=0?Kalba=EN&KategorijaId=109&Kalbos_input=EN&Kalbos=EN&KategorijaEn_input=Social+medicine… 2/6
3/27/2019 LSMUSIS
7.23 What statement about Lithuanian model of compensation of the damage done to patients’ health is not true?
A. Lithuania has pretrial institution who investigates patient complaints for damage compensation
B. Damage is reimbursed according to state's set methodology for every damage case
C. In Lithuania you have to prove that doctor was negligent in providing medical care
D. Maximum amount of reimbursed damage is not regulated by the law

7.24 Information about patients health can be gathered:


A. So much as is necessary in treatment of the patient
B. So much as is necessary according doctors opinion
C. So much as it is necessary according patients opinion
D. So much as it is necessary according the internal rules of health care institution

7.25 When health care institution can not disclose information about patient health without his written agreement?
A. When patient is deceased and information is sought by his hereditary by law or testament
B. When information is sought by commission which is investigating claim for damage done to patients
health
C. When information is sought by insurance company which insured health care institution for damage done
to patients health
D. When information is sought by police which is acting under claim from patients relative, who want to get
information about patients health

7.26 When is disagreement between minor patient and his legal representative about the treatment, doctor should:
A. Act according minor patient will
B. Act according legal representatives will
C. Arrange consillium of the doctors to make a decision according minor patients will
D. Arrange consillium of the doctors to inquire whether legal representatives will is not against patient
interests

7.27 How often doctor in Lithuania after he obtained medical license has to renew his license?
A. After 2 years
B. After 5 years
C. After 3 years
D. After 4 years

7.28 In Lithuania civil responsibility for the damage done to the patients health goes to:
A. Doctor
B. Health care institution
C. State
D. Patients insurance fund

7.36 What sort of information doctor is not obliged to tell patient:


D. Information about doctors experience
B. Information about doctors qualification
C. Information about cost of health care services
A. Information about patients involvement in medical students teaching process

7.39 What are the main public health functions performed by municipal public health bureau:
A. Public health monitoring, health promotion and education
B. Public health and safety control
C. Public health monitoring, health promotion and education, health, and safety control

7.40 What are the main public health functions are carried out by public health centers:
A. Public health monitoring, health and safety control
B. Public health monitoring, health and safety control, communicable disease control, emergency
management
C. Public health monitoring, health and safety control, emergency management

7.41 Ageing rate is:


A. Proportion of population under 18 years old
B. Proportion of population 19-49 years old
C. Proportion of population over 50 years old
D. Proportion of population over 65 years old

lsmusis.lsmuni.lt/Klausimai/Spausdinti?Length=0?Kalba=EN&KategorijaId=109&Kalbos_input=EN&Kalbos=EN&KategorijaEn_input=Social+medicine… 3/6
3/27/2019 LSMUSIS
E. Proportion of population over 75 years old

7.42 Natural growth rate is estimated:


A. Birth rate x death rate
B. Death rate - birth rate
C. Birth rate / death rate
D. Birth rate - death rate
E. Birth rate + death rate
F. Number of births - number of deaths

7.43 Which of these actions cannot be named as decentralization in health care governance?
A. Deconcentration
B. Deflation
C. Devolution
D. Delegation

7.44 Which definition of decentralization in health care governance is correct?


A. The transfer of authority, from higher to lower levels of government
B. The transfer of authority, from lower to higher level of government
C. The transfer of authority, from national level to international
D. The transfer of authority, from political to non-political level

7.45 Which model of health care financing is insurance based?


A. Semachko
B. Bismarck
C. Beveridge
D. All of them

7.46 What is the size of Satutory health insurance premium, which is paid by employed person in Lithunia?
A. 3% of personal incomes which was defined by the Law on State Social Insurance
B. 6% of personal incomes which was defined by the Law on State Social Insurance
C. 9% of personal incomes which was defined by the Law on State Social Insurance
D. 12% of personal incomes which was defined by the Law on State Social Insurance

II type tasks. For each numbered item,selct the one lettered heading that is most closely asssciated with it

7.29 When patients sick with the indicated diseases and having sick-leave should be sent to Disability and
Working Capacity Assessment Office:
1 - Stomach cancer
2 - Tuberculosis
3 - Hypertonic disease
A. No later than after 122 calendar days
B. No later than after 182 calendar days
C. No later than after 244 calendar days

7.30 Which part of the health of the population is determined by:


1 - Lifestyle
2 - Environment
3 - Health care
A. 10%
B. 50%
C. 20%

7.31 For what period of time sick-leave can be given in each case:
1 - Pregnancy and delivery
2 - Additionally after the complicated delivery
3 - After the induced abortion
A. 28 calendar days
B. No longer than 14 calendar days
C. No longer than 2 calendar days
D. 126 calendar days

lsmusis.lsmuni.lt/Klausimai/Spausdinti?Length=0?Kalba=EN&KategorijaId=109&Kalbos_input=EN&Kalbos=EN&KategorijaEn_input=Social+medicine… 4/6
3/27/2019 LSMUSIS
7.32 How each indicator is estimated:
1 - Crude death rate
2 - Lethality (case fatality)
3 - In-patient lethality (case fatality)
A. (Number of deaths × 1000) / (Population in the beginning of the year)
B. (Number of deaths × 1000) / (Average population)
C. (Number of deaths from a particular disease × 100) / (Number of cases of that disease)
D. (Number of in-patient deaths × 100) / (Number of dicharges + number of in-patient deaths)
E. (Number of in-patient deaths × 100) / (Average population)

III type tasks. For each question there is one or more correct answers:
A – if correct answers are 1,2,3
B – if correct answers are 1 and 3
C – if correct answers are 2 and 4
D – if correct answer is 4
E – if correct are all answers above

7.17 Methods of the assessment of patient quality:


1. Surveys of the patients
2. Inerviews of the patients
3. Patients claims
4. Conclusions of medical audit
5. Quality costs

7.18 Management quality is assessed according to:


1. Conclusions of medical audit
2. Patients claims
3. Quality costs
4. Surveys of the patients

7.33 In which cases sick-leaves can not be given:


1. For persons who are able to work during the period of their health check-up
2. For persons in case of self-inflicted harm to their or their dependants health with the purpose to avoid
work or other responsibilities or pretending (simulating) to be ill
3. For persons, if the direct cause of temporal disability was consumption of alcohol, toxic or narcotic
substances
4. For persons sick with mental disorders, in case of self-inflicted harm to their health

7.34 What are hospital performance indicators:


1. Bed occupancy
2. Average length of stay
3. Bed turnover
4. Long-term monitoring of patients by disease classes per 1,000 population

7.35 What includes health care:


1. Preventive health care
2. Curative health care
3. Medical rehabilitation, nursing, social services
4. Personal health expertise

7.37 When patient asks, he has right to look at and check his medical records, except:
1. When this can seriously detriment his health
2. When this can influence his desire to get well
3. When this can cause danger to his life
4. When patient is willing to file a complaint against health care institution

7.38 Expenditures of State social insurance are:


1. Pensions social insurance
2. Sickness and motherhood social insurance
3. Unemployment social insurance
4. Charity for hospitals

lsmusis.lsmuni.lt/Klausimai/Spausdinti?Length=0?Kalba=EN&KategorijaId=109&Kalbos_input=EN&Kalbos=EN&KategorijaEn_input=Social+medicine… 5/6
3/27/2019 LSMUSIS
7.1 - D 7.2 - D 7.3 - B 7.4 - B 7.5 - B 7.6 - D 7.7 - A 7.8 - C
7.9 - C 7.10 - C 7.11 - C 7.12 - A 7.13 - A 7.14 - A 7.15 - B 7.16 - B
7.19 - D 7.20 - B 7.21 - C 7.22 - C 7.23 - B 7.24 - A 7.25 - D 7.26 - D
7.27 - B 7.28 - B 7.36 - D 7.39 - A 7.40 - B 7.41 - D 7.42 - D 7.43 - B
7.44 - A 7.45 - B 7.46 - C
7.29 7.30 7.31 7.32
1-A 1-B 1-D 1-B
2-B 2-C 2-B 2-C
3-A 3-A 3-C 3-D
7.17 7.18 7.33 7.34 7.35 7.37 7.38
1 1 1 1 1 1 1
2 3 2 2 2 3 2
3 3 3 3 3
4

lsmusis.lsmuni.lt/Klausimai/Spausdinti?Length=0?Kalba=EN&KategorijaId=109&Kalbos_input=EN&Kalbos=EN&KategorijaEn_input=Social+medicine… 6/6
3/27/2019 LSMUSIS

Disaster medicine
I type tasks. Choose only one best answer

16.1 Who is responsible for medical care at the disaster (accident) site:
A. Police
B. Civile defence
C. Medical crew (nurse or physician), first arrived to the disaster site
D. Director of the nearest hospital
E. Minister of health
F. The most experienced physician at the disaster site

16.2 Triage is based on:


A. Sorting of casualties by the severity of injuries
B. Sorting of casualties by localisation of injuries
C. Sorting of casualties by priorities of emergency care

16.3 Which of the following patients will have the first priority:
A. Closed fracture of femur
B. Femoral neck fracture
C. Amputation of the right hand

16.4 Which of the following patients will have the first priority at the hospital:
A. Amputation of the 2nd and 3rd finger of the left hand
B. Closed fracture of the right humerus
C. Luxation of the right humerus

16.5 Which of the following patients will have the first priority at the hospital:
A. Open fracture of the left tibia and closed fracture of the right tibia
B. Contusion wound of the waist. Systolic blood pressure 90 mm Hg
C. Closed fracture of the right femur

16.6 Which of the following patients will have the first priority at the hospital:
A. Unconcious. 3x4 cm wound in the right temporal area, bleeding. Breathing with interruptions. Systolic
blood pressure 80 mm Hg
B. Luxation of the right humerus
C. Closed fracture of the left tibia

16.7 Which of the following patients will have the first priority:
A. Scratches of the left cheek and both hands. Teared wound of the right shin
B. Open fracture of the left forearm. Arterial bleeding
C. Luxation of the left elbow

16.8 Which condition is the most dangerous for patient:


A. Bilateral fracture of mandibula. Unconscious
B. Traumatical amputation of the left hand
C. Closed fracture of the right femur

16.9 Which of the following patients will have the first priority:
A. Closed fracture of the hip. Puls - 100 x min. Systolic blood pressure 70 mm Hg
B. Contusion of the right knee
C. Closed fracture of the left femur

16.10 Action of organophosphates


A. Block of cytochromoxidasis
B. Denaturation of proteins
C. Block of cholinestherasis

16.11 Action of cyanides:


A. Block of cytochromoxidasis
B. Denaturation of proteins
C. Block of cholinestherasis

lsmusis.lsmuni.lt/Klausimai/Spausdinti?Length=0?Kalba=EN&KategorijaId=118&Kalbos_input=EN&Kalbos=EN&KategorijaEn_input=Disaster+medicin… 1/5
3/27/2019 LSMUSIS
16.12 Radiated patient at the admission department. No radioactive contamination:
A. No decontamination needed
B. Patient needs degasation
C. Patient needs decontamination

16.13 Radioactive contaminated patient at the admission department.


A. No decontamination needed.
B. Patient needs partial decontamination
C. Patient needs full decontamination

16.14 Which of the following substances is the best for chloracetophenon (lacrimator) decontamination:
A. Water
B. Acid solutions
C. Alkaline solutions

16.15 Removing of radioactive, chemical and biological substances from body surface:
A. Decontamination
B. Degasation
C. Desinfection
D. Desactivation

16.16 Removing of radioactive substances from various contaminated objects:


A. Degasation
B. Desactivation
C. Desinfection
D. Deratisation

16.17 Removing of chemical substances from various contaminated objects:


A. Degasation
B. Desactivation
C. Desinfection

16.18 "CS" substance (irritating gas, used as self-protection device) acts like:
A. Lacrimator
B. Sternite
C. Lacrimator and sternite

16.19 Which emergency procedures you can delay in mass casualties situation:
A. All emergency procedures for 1/2 of patients.
B. Emergency procedures, which delay will not affect health conditions of the patients at this moment.
C. All emergency procedures for 1/3 of patients for 1 hour.

16.20 Cyan acid (HCN) contaminated patient at the accident site:


A. Needs partial decontamination
B. Needs full decontamination
C. No decontamination needed

16.21 Critical situation in the health care facilities will be:


A. When number of patients will exceed hospital treatment capacity
B. 10 emergency patients entering the emergency department at the same time
C. When number of patients will be less than hospital treatment capacity

16.31 Young man came to the out-patient department complaining on eye pain and tearing. Self-protective spray
was spraied directrly into his face approximately half an hour ago. What substance should be used for eye
irrigation?
A. Tap water
B. 5% borax solution
C. 2% baking soda

II type tasks. For each numbered item,selct the one lettered heading that is most closely asssciated with it

16.22 Optimal time for emergency procedures


1 - For the first aid in case of mechanical injuries
lsmusis.lsmuni.lt/Klausimai/Spausdinti?Length=0?Kalba=EN&KategorijaId=118&Kalbos_input=EN&Kalbos=EN&KategorijaEn_input=Disaster+medicin… 2/5
3/27/2019 LSMUSIS
2 - For surgery in case of mechanical injuries
3 - For advanced emergency procedures in case of acute poisoning
A. 0,5-1 hour
B. 1-3 hours
C. Till 6 hours

16.23 Make colour marking for these patients:


1 - Unconcious patien. Bleeding lacerated wound 3x4cm of the left temporal area. Breathing uneven,
32/min. Systolic blood pressure 80mmHg
2 - Superficial wound of the right shin.
3 - Open fracture of the left tibia
A. Red
B. Yellow
C. Green

16.24 Make colour marking for these patients:


1 - Closed fracture of the II-IVth left ribs
2 - 2x3 cm wound of the left hand
3 - Closed fracture of the left femur
4 - Thrusted thoracic wound, forced sitting position, choking
5 - Open fracture of the left and closed fracture of the right shin. Profound bleeding
6 - Scrape in the left lumbal area
7 - Open fracture of the right femur
A. Red
B. Yellow
C. Green

III type tasks. For each question there is one or more correct answers:
A – if correct answers are 1,2,3
B – if correct answers are 1 and 3
C – if correct answers are 2 and 4
D – if correct answer is 4
E – if correct are all answers above

16.25 Mark the first priority


1. Closed rib fracture in the left midle axilar area
2. Cutten wound of forearm, arterial bleeding
3. Fracture of the neck of left femur
4. Punctured chest wound, forced siting position (shortness of breath)

16.26 Which patient should be marked "red" in triage procedure


1. Blunt abdominal trauma
2. Dislocation of the right elbow
3. Amputation of the right hand and lacerated wound of right humerus
4. Open fracture of the right forearm

16.27 Which patient should be marked "red" in triage procedure


1. Scrape wound of forehead
2. Crushed left foot and shin
3. Closed fracture of the left humerus
4. Open fracture of the right femur, arterial bleeding
5. Closed fracture of the right femur
6. Dislocation of the right shoulder

16.28 Wich of the following substances are organophosphates


1. Sarin
2. Dichlofos (Cobra)
3. Soman
4. Iprit

16.29 Which of the following industrial substances has a potential risk to mass intoxication in Kaunas city
1. Sarin
lsmusis.lsmuni.lt/Klausimai/Spausdinti?Length=0?Kalba=EN&KategorijaId=118&Kalbos_input=EN&Kalbos=EN&KategorijaEn_input=Disaster+medicin… 3/5
3/27/2019 LSMUSIS
2. Ammoni
3. Cianides
4. Chlorine

16.30 Number of casualties in case of chemical disaster at the factory, located in densily populated area will
depend on:
1. Day time
2. Meteorological conditions
3. Density of the population
4. Characteristics of the chemical substance

16.32 Rose or pink collor of skin and mucosa suggests intoxication with following substances
1. Organophosphates
2. Cyanides
3. Chlorine
4. Carbon monoxide

16.33 Kind of decontamination:


1. Removing of radioactive substances
2. Removing of chemical substances
3. Removing of biological substances
4. Removing of contamination from body surface

16.34 Typical injuries in nuclear accidents


1. Acute radiation sickness
2. Burns
3. Mechanical trauma
4. Hypothermia

16.35 Typical injuries in marine disasters and floods


1. Hypothermia
2. Mechanical injuries
3. Drowning
4. Smoke and gas intoxication
5. Termal and chemical burns

16.36 Typical injuries in railway disasters


1. Mechanical injuries
2. Hypothermia
3. Burns
4. Hyperthermia

16.37 Typical lesions in all types of big scale disasters


1. Psychological injury
2. Common cold
3. Infection diseases
4. Burns

16.38 Typical injuries in aviation accidents


1. Mechanical trauma
2. Burns
3. Poisoning
4. Frostbite

16.39 Characteristics, typical to any kind of disaster


1. Unexpected event
2. Life and health threatening factors
3. Dead or injuried persons are expected
4. Urgent demand for emergency care to many patients

16.40 Medical consequences of disasters are:


1. Injuried persons
2. Higiene and epidemic problems at the disaster site
lsmusis.lsmuni.lt/Klausimai/Spausdinti?Length=0?Kalba=EN&KategorijaId=118&Kalbos_input=EN&Kalbos=EN&KategorijaEn_input=Disaster+medicin… 4/5
3/27/2019 LSMUSIS
3. Damage of local health care facilities
4. Damage of communication system
5. Damage of electricity supply

16.1 - C 16.2 - C 16.3 - C 16.4 - A 16.5 - B 16.6 - A 16.7 - B 16.8 - A


16.9 - A 16.10 - C 16.11 - A 16.12 - A 16.13 - C 16.14 - C 16.15 - A 16.16 - B
16.17 - A 16.18 - C 16.19 - B 16.20 - C 16.21 - A 16.31 - A
16.22 16.23 16.24
1-A 1-A 1-C
2-C 2-C 2-B
3-B 3-B 3-B
4-A
5-A
6-C
7-B
16.25 16.26 16.27 16.28 16.29 16.30 16.32 16.33
2 1 2 1 2 1 2 1
4 3 4 2 4 2 4 2
3 3 3
4 4
16.34 16.35 16.36 16.37 16.38 16.39 16.40
1 1 1 1 1 1 1
2 2 3 2 2 2 2
3 3 3 3 3 3
4
5

lsmusis.lsmuni.lt/Klausimai/Spausdinti?Length=0?Kalba=EN&KategorijaId=118&Kalbos_input=EN&Kalbos=EN&KategorijaEn_input=Disaster+medicin… 5/5
3/27/2019 LSMUSIS

Environmental medicine
I type tasks. Choose only one best answer

32.1 Water hardness depends upon:


A. Calcium and magnium hydrocarbonates
B. Natrium chloride
C. Carbohydrates
D. Organic compounds

32.2 Which toxic material concentration in drinking water is limited due to the potential for dental and bone
damage?
A. Selenium
B. Cadmium
C. Fluoride
D. Nitrate

32.3 The main drinking water treatment stages are:


A. Mechanical, biological treatment, disinfection
B. Chemical treatment
C. Biological treatment, deodorization

32.4 Worldwide generally used drinking water disinfection method is:


A. Biological treatment
B. Ozonation
C. Chlorination
D. Boiling

32.7 What is permissible nitrate concentration in drinking water?


A. No more than 5 mg/l
B. No more than 10 mg/l
C. No more than 200 mg/l
D. No more than 50 mg/l

32.12 Which statement about the layout of ward is invalid?


A. Ward beds are built in rows parallel to the wall with windows;
B. Spacing between adjacent beds should be no more than 10 cm.;
C. The space between the bed and wall with windows must be not less than 0,9 m.;
D. Spacing between bed ends must not be less than 1,6 m.

32.14 What are the main parts of air composition:


A. N - 78,09 %, O2 - 20,95 %, CO2 - 0,03 %, Ar - 0,93 % and inertic gases
B. N - 79 %, O2 - 15,4 %, H2O - 1,0 %, CO2 - 3,4 %
C. N - 60 %, O2 - 30 %, CO2 - 8 %, Ar - 1 % and inertic gases

32.15 What are the optimal parameters of temperature, humidity and air movement to mental and light physical
work in room adapted for such work: Air temperature Relative humidity Air movement
A. 18-21⁰C 60-80 % 0,2 m/s
B. 16-18⁰C 20-40 % 0,3 m/s
C. 18-20⁰C 40-60 % 0,1 m/s

32.16 What is the natural lighting coefficient:


A. Decreased lighting of window glasses expressed by percents
B. Proportion of indoor and outdoor lighting expressed by percents
C. Proportion of window’s and floorspace expressed by percents
D. Angle of falling rays to the room

32.17 Which of mentioned air pollutants is responsible for the ,,greenhouse effect’’:
A. Ozone
B. SO₂
C. CO₂
D. N₂O₅

lsmusis.lsmuni.lt/Klausimai/Spausdinti?Length=0?Kalba=EN&KategorijaId=134&Kalbos_input=EN&Kalbos=EN&KategorijaEn_input=Environmental+… 1/5
3/27/2019 LSMUSIS
32.18 Calculate the cooling size (H) estimate the employees of mental work thermal comfort feelings, if the data of
katathermometer are – katathermometer constanta (K) – 593 cal/cm2, time (t) katathermometer cooling from the
390 C to 350C – 100 seconds. (H=K/t)
A. Human feels thermal comfort
B. He feels hard cold.
C. He feels hot.
D. He feels coolness.

32.19 Where the meteorological conditions are optimal if work load is very hard if cooling size of indoor air is equal
to - :
A. 8,8 cal.cm²/s
B. 5,9 cal.cm²/s
C. 3,1 cal.cm²/s
D. 11,9 cal.cm²/s

32.23 Which physiological process defines child development?


A. Organs and functional systems biological potential;
B. Qualitative differentiation body systems;
C. Quantitative differentiation body systems;
D. Acceleration process

32.25 Which meteopathology syndrome characterized by heart ailments (pain, arrhythmias, etc.)?
A. cerebral
B. cardiac;
C. asthmatic;
D. the nerves and blood vessels

32.32 The recommended daily energy requirement from protein, fats and carbohydrates are:
A. From proteins 20-30%, fats 35-40%, and carbohydrates 30-45%;
B. From proteins 30-40%, fats 20-30%, and carbohydrates 30-50%;
C. From proteins 10-15%, fats 28-30%, and carbohydrates 55-62%.

32.33 The daily energy requirement depends on:


A. Gender, age, weight, height and physical activity.
B. Gender, height, and weight.
C. Weight and physical activity.

32.34 Body mass index is calculated:


A. Weight (kg) multiply by height (m²)
B. Weight (kg) divided by height (m²)
C. Weight (kg) divided by height (cm²)

32.35 For calculation the quantity of calories which are assimilated and used for the energetically purposes in our
body (Netto calorificity) we used these coefficients:
A. For animal food - 95%, for vegetable food - 85% and for various food - 90%.
B. For animal food - 90%, for vegetable food - 80% and for various food - 90%.
C. For animal food - 85%, for vegetable food - 95% and for various food - 90%.

II type tasks. For each numbered item,selct the one lettered heading that is most closely asssciated with it

32.37 What should be the temperature level at?


1 - school classroom,
2 - the doctor's room,
3 - the preschool classroom,
4 - the gym.
A. 18-19⁰ C;
B. 20-21 °C
C. 21-22⁰C
D. 15-17⁰C

32.38 Please, divide the basic nutrients by releasing calories content.


1 - 1g of fats take to our organism

lsmusis.lsmuni.lt/Klausimai/Spausdinti?Length=0?Kalba=EN&KategorijaId=134&Kalbos_input=EN&Kalbos=EN&KategorijaEn_input=Environmental+… 2/5
3/27/2019 LSMUSIS
2 - 1g of proteins and 1g of carbohydrates take to our organism
3 - 1 g of alcohol take to our organism
A. 7,2 kcal.
B. 4,1 kcal.
C. 9,3 kcal.

32.39 Please, each pathogen assign for appropriately illness.


1 - E. Coli
2 - Fish tapeworm
3 - Salmonella
A. Salmonellosis
B. Teniosis
C. Gastroenteritis

32.40 Please, assign for each claim the diagnosis.


1 - The most common cause, eating pork or wild boar.
2 - Enters the body through canned foods. Multiply and produce toxin under anaerobic conditions.
3 - Fall into the food from the throat, skin abscesses.
A. Staphylococcal intoxication
B. Trichinosis
C. Botulism

32.41 Please remember nonbacterial food poisoning and assign their individual groups respective representatives.
1 - Mycotoxins
2 - Alkaloids.
3 - Cyanogenic glucosides.
A. Solonine
B. Patuline.
C. Amigdaline

32.42 Please, choose the amount of carbohydrates in these products.


1 - 40-46 %.
2 - 70-79 %.
3 - 4,7 %
A. Milk
B. Bread
C. Flour

32.43 Assign the weather in terms of beneficial for health


1 - daily atmospheric pressure varies less than 6 hPa, relative humidity up to 85%, and the wind speed is >6
m/s
2 - daily atmospheric pressure change >4hPa, relative humidity >90%, and wind speed >6 m/s
3 - daily atmospheric pressure changes >10 hPa, wind speed >8 m/s and rainfall greater than 1 mm.
A. Medically favorable weather
B. Medically adverse weather
C. Medically disadvantaged weather

III type tasks. For each question there is one or more correct answers:
A – if correct answers are 1,2,3
B – if correct answers are 1 and 3
C – if correct answers are 2 and 4
D – if correct answer is 4
E – if correct are all answers above

32.5 Which indicators show drinking water contamination by human excrements:


1. Ammonia nitrogen
2. Dissolved oxygen
3. Amount of E.coli in water
4. PH of water

32.6 Why nitrates and nitrites are harmful to human:


1. They form nitrozamines in stomach
lsmusis.lsmuni.lt/Klausimai/Spausdinti?Length=0?Kalba=EN&KategorijaId=134&Kalbos_input=EN&Kalbos=EN&KategorijaEn_input=Environmental+… 3/5
3/27/2019 LSMUSIS
2. They decrease resorbtion of vitamines in organism
3. They cause methemoglobinemia
4. The promote development of nephrolithiasis

32.8 Which methods can be used for drinking water disinfection?


1. Fluoridation
2. Chlorination
3. Degassing
4. Boiling

32.9 What are the main factors contributing to the development of nosocomial infections?
1. The breach of hygienic – anti-epidemic regime the health care institution;
2. Invasive procedures – violation of natural human anatomical barriers;
3. The damage of patient microflora (treatment with antibiotic or cytostatic drugs)
4. Weak instrument sterilization control.

32.10 What are the most important measures to prevent nosocomial infections?
1. Determination of the source of infection, isolation and treatment;
2. A disruption of the spread of infection;
3. Protection of patients that are susceptible to nosocomial infections;
4. Rational prescription of vitamins and microelements to patients

32.11 What settings of the harmful to the health environment should be taken into account while planning the
construction of health care building?
1. Environmental noise;
2. The distance from the major transport units;
3. The chemical pollution of atmosphere;
4. Annual rainfall in the area.

32.13 Which statements about health care institution’s interior design, the materials used, are correct?
1. Walls can be painted in latex paint;
2. Floor coverings should be made of antistatic material;
3. Floor coverings seams must be welded or soldered;
4. All rooms should have windows with unbreakable glass.

32.20 What processes are going on, acting sun ultra-violet rays:
1. Synthesis of melanin is suppressed
2. Synthesis of vit. D is stimulated
3. Ca-P metabolism is getting disturbed
4. Denaturation of skin proteins
5. Human’s psychic activity gets more active.

32.21 What measures are the mains for air pollution control:
1. Dilute in the atmosphere
2. Prevent formation
3. Reduce the quantity
4. Apply air-cleaning technologies

32.22 To evaluate and to express the cooling size using catathermometer it is possible cumulative affect of
meteorological conditions to human:
1. Air temperature
2. Humidity
3. Air movement
4. Atmospheric pressure

32.24 What are the possible causes of acceleration?


1. Dietary changes;
2. Genetic factors;
3. Improved hygienic conditions;
4. Improved treatment of diseases technology

32.26 Which meteorological factors are characterizing weather?


1. temperature
lsmusis.lsmuni.lt/Klausimai/Spausdinti?Length=0?Kalba=EN&KategorijaId=134&Kalbos_input=EN&Kalbos=EN&KategorijaEn_input=Environmental+… 4/5
3/27/2019 LSMUSIS
2. humidity
3. air velocity
4. pressure
5. air ionization

32.27 What properties do have fish oil?


1. There are fats soluble vitamins in them.
2. Fish oil reduces triglycerides in the human blood serum.
3. They are the main source of long-chain polyunsaturated fatty acids.
4. They are liquid and spoil quickly.

32.28 Why did curd is recommended for the nutrition in elderly:


1. This is the main source of calcium and phosphorus.
2. It stimulates guns peristaltic.
3. There are all essential amino acids.
4. There are a lot of water-soluble vitamins.

32.29 What kind of meat is not dangerous for trichineliosis?


1. From beef.
2. From venison.
3. From rabbit.
4. From wild boar.

32.30 What products you can recommend for vitamin A hypovitaminosis treatment?
1. Bread.
2. Liver.
3. Wheat bran
4. Carrots

32.31 What products are the main sources of dietary fibres?


1. Fish, eggs.
2. Milk and dairy products.
3. Meat and meat products
4. Wheat brain, cereals

32.1 - A 32.2 - C 32.3 - A 32.4 - C 32.7 - D 32.12 - B 32.14 - A 32.15 - C


32.16 - B 32.17 - C 32.18 - A 32.19 - A 32.23 - B 32.25 - B 32.32 - C 32.33 - A
32.34 - B 32.35 - A
32.37 32.38 32.39 32.40 32.41 32.42 32.43
1-A 1-C 1-C 1-B 1-B 1-B 1-A
2-B 2-B 2-B 2-C 2-A 2-C 2-B
3-C 3-A 3-A 3-A 3-C 3-A 3-C
4-D
32.5 32.6 32.8 32.9 32.10 32.11 32.13 32.20
1 1 2 1 1 1 1 2
3 3 4 2 2 2 2 4
3 3 3 3
4
32.21 32.22 32.24 32.26 32.27 32.28 32.29 32.30
1 1 1 1 1 1 1 2
2 2 2 2 2 3 2 4
3 3 3 3 3 3
4 4 4
5
32.31
4

lsmusis.lsmuni.lt/Klausimai/Spausdinti?Length=0?Kalba=EN&KategorijaId=134&Kalbos_input=EN&Kalbos=EN&KategorijaEn_input=Environmental+… 5/5
3/27/2019 LSMUSIS

Surgery
I type tasks. Choose only one best answer

1.1 The most common cause of acute abdomen syndrom is:


A. Abdominal and chest trauma.
B. Obstruction of gastrointestinal tract, billiary ducts or blood vessels.
C. Inflammatory diseases of intraabdominal organs.
D. Rupture of intraperitoneal organs, tumours and cysts.

1.2 Which of the symptoms listed below always present in case of acute surgical problem in the peritoneal cavity?
A. Symptoms of peritoneal
B. Weakening or strengthening of the bowel sounds.
C. Local or diffuse muscle distension.
D. Pain

1.3 The prevalent symptoms of the terminal peritonitis:


A. Not localized abdominal pain
B. Intoxication and hypovolemia.
C. Diffuse muscle distension and peritoneal irritation.
D. Swelling of the abdomen and absence of peristalsis.

1.4 Which of the symptoms is pathognomic in case of the penetrating abdominal trauma?
A. Intense bleeding from the wound.
B. Muscle distension and symptoms of peritoneal irritation in the vicinity of the wound.
C. Eventration.
D. Weak peristalsis at auscultation

1.5 The most informative investigation in case of traumatic rupture of the mesenterium is:
A. Sonoscopy.
B. Abdominal paracentesis.
C. Laparoscopy.
D. Computed tomography.

1.6 What is the management of extraperitoneal bladder rupture?


A. Laparotomy, bladder suture and catheterization
B. CT guided drainage of the urine leak followed by repeat cystogram to confirm closure of the injury
C. Catheterization for 7-10 days followed by repeat cystogram to confirm closure of the injury
D. Cystoscopy and edoscopic wound closure

1.7 Pringle manoeuvre for a temporary stopping of bleeding is used in case of bleeding from:
A. Bleeding peptic ulcer
B. Liver
C. Spleen
D. Colon
E. Brachial artery

1.8 Shock caused by injury to the spinal cord would be called?


A. Hypovolemic
B. Hypernatrimic
C. Neurogenic
D. Spinalgenic

1.9 The sequence of action when the first aid is rendered for a trauma patient:
A. Establishment of the adequate airway and breathing function, stopping of the external bleeding,
maintenance of the cardiac function and circulation, analgesia, immobilization.
B. Establishment of the adequate airway and breathing function, analgesia, stopping of the external
bleeding, maintenance of the cardiac function and circulation, immobilization.
C. Stopping of the external bleeding, immobilization, establishment of the adequate airway and breathing
function , maintenance of the cardiac function and circulation, analgesia

1.10 Diagnostic laparoscopy is indicated for:

lsmusis.lsmuni.lt/Klausimai/Spausdinti?Length=0?Kalba=EN&KategorijaId=103&Kalbos_input=EN&Kalbos=EN&KategorijaEn_input=Surgery&Kateg… 1/29
3/27/2019 LSMUSIS
A. .Patient with a blunt abdominal trauma with stable haemodynamics, at sonography and CT a small or
medium amount of liquid is found in abdominal cavity.
B. Patient with a blunt abdominal and cranial trauma, to whom an elevated intracranial pressure is
suspected , at sonography and CT a small or medium amount of liquid is found in abdominal cavity.
C. Patient with a blunt abdominal trauma with stable haemodynamics, at sonography and CT no liquid is
found in abdominal cavity.

1.11 In case of spleen injury, the splenectomy is performed, if:


A. There is more than 6 hours after trauma
B. Subcapsular hematoma of the spleen
C. The rupture of spleen is in upper pole
D. Multiple ruptures of the spleen are present
E. In all above mentioned cases.

1.12 When contusion of the pancreas or the superficial injury without the duct injury is found at the time of
laparotomy, the action will follow:
A. Distal pancreatectomy
B. Pancreaticojejunostomy by Roux
C. Stopping of bleeding and external drainage
D. Pancreatoduodenectomy
E. Endoscopic retrograde cholangiopancreatography

1.13 Which statement is not true about the mechanism of duodenal injuries?
A. Most of the duodenal injuries are penetrating
B. Blunt injuries are equally distributed in incidence in the four parts of duodenum
C. Second part of duodenum is the most common site of injury
D. Motor vehicle accidents are the most common form of blunt duodenal injuries

1.14 Which of the below mentioned symptoms is absent in case of small bowel obstruction:
A. Abdominal distension
B. Nausea
C. Diarrhoea
D. Leucocytosis
E. Thirst

1.15 The most rare cause of the small bowel obstruction is:
A. Incarcerated hernia
B. Adhesions
C. Crohn’s disease
D. Tumours
E. Invagination

1.16 The immediate intervention is not necessary when the diagnosis is?
A. Total small bowel obstruction due to adhesions
B. Bowel strangulation
C. Colon cancer, causing total large bowel obstruction
D. Adhesions in the peritoneal cavity
E. Intestinal intussusception

1.18 The most common cause of destructive ileus is previous:


A. Gastrectomy
B. Appendectomy
C. Hysterectomy
D. Cholecystectomy

1.19 In the routine small bowel follow-through series, regular barium sulfate ordinarily reaches the large intestine
within:
A. 1-2 h
B. 4-6 h
C. 8-12 h
D. Up to 24 h
E. After 24 h

lsmusis.lsmuni.lt/Klausimai/Spausdinti?Length=0?Kalba=EN&KategorijaId=103&Kalbos_input=EN&Kalbos=EN&KategorijaEn_input=Surgery&Kateg… 2/29
3/27/2019 LSMUSIS
1.20 The most common cause of destructive ileus is:
A. Intestinal strangulation
B. Total adhesive process in the peritoneum
C. Intestinal intussusception
D. Solitary adhesion
E. Tumor

1.21 The crucial role for the development destruction of bowel wall in case of strangulation plays:
A. Disappearance of intestinal peristalsis
B. Disturbance of blood supply
C. Disturbance of the immune function
D. Systemic toxic effect
E. Disturbance of absorption

1.22 Which investigation is not right when complete large bowel obstruction is suspected?
A. Plain abdomen x-ray
B. Ultrasound investigation
C. Barium follow through
D. Barium enema
E. Lower GI endoscopy

1.23 The most common cause of large bowel obstruction:


A. Adhesions
B. Incarcerated hernia
C. Tumors
D. Strangulation
E. Obturation

1.24 The bowel peristalsis is being suppressed by:


A. Sympatic nervous system
B. Parasympatic nervous system
C. Alcohol
D. Coffee
E. Sleep

1.25 The „golden standard“ investigation when mesenteric artery occlusion is suspected is:
A. Gastroscopy
B. Laparoscopy
C. Angiography
D. Ultrasound scan
E. Computed tomography

1.26 Which feature is the most common in appendicitis?


A. Abdominal distension
B. Leucocytosis
C. Shooting pains in the abdomen
D. Repeated vomiting
E. Sweating

1.27 Which factor is the most important in development of acute appendicitis?


A. Hypertrophy of lymphatic tissue
B. Spasms of the smooth muscles
C. Obstruction of the lumen of appendix
D. Neurohumoral factors
E. Helminths

1.28 Which of the symptoms is absent in acute appendicitis?


A. Leucocytosis
B. Fever
C. Haematuria
D. Nausea
E. Tachycardia

lsmusis.lsmuni.lt/Klausimai/Spausdinti?Length=0?Kalba=EN&KategorijaId=103&Kalbos_input=EN&Kalbos=EN&KategorijaEn_input=Surgery&Kateg… 3/29
3/27/2019 LSMUSIS
1.29 What investigations you would recommend when acute appendicitis is suspected? Which answer is not right?
A. Laparoscopy
B. Ultrasound scan
C. Urine analysis
D. Plain abdominal X-ray

1.30 Which investigation is least informative to diagnose an acute appendicitis?


A. Common blood test
B. Ultrasound scan
C. Per-rectal exam
D. Plain abdominal X-ray
E. Laparoscopy

1.31 Which statement concerning infiltrative appendicitis is wrong?


A. Infiltrative appendicitis develops in all cases after more than 5 days fromdisease start
B. Surgical procedure can be the choice of treatment of infiltrative appendicitis
C. Infiltrative appendicitis may be treated conservatively
D. Infiltrative appendicitis may cause bowel obstruction

1.32 Immediate appendectomy is performed, when the diagnosis is:


A. Acute appendicitis
B. Acute infiltrative appendicitis
C. Intraabdominal mass
D. Periappendicular abscess
E. Adhesions of the appendix

1.33 The most appropriate type of anaesthesia for appendectomy is:


A. Local anaesthesia
B. General anaesthesia
C. Epidural anaesthesia
D. Spinal anaesthesia
E. General anaesthesia is applicable for children

1.34 Why it is recommended to perform hernia repair as an elective surgery procedure?


A. Rate of complications and mortality increases when incarceration occurs
B. To prevent the development of fibrosis in the hernia neck
C. To prevent formation of non reducible hernia
D. To avoid the procedures in night time
E. Inflammation of the sac develops in case of incarcerated hernia

1.35 Why it is not recommended to restore the incarcerated hernia?


A. To avoid an injury of hernia neck
B. To prevent extrusion of the liquid from the hernia sac into abdominal cavity
C. To prevent the injury of the hernia content
D. To avoid the loss of the incarcerated organ in the abdominal cavity
E. Conservative treatment is the method of choice in case of incarcerated hernia

1.36 Which of the symptoms is not the criterion of the vitality of incarcerated intestine?
A. Pulsation of the mesenteric vessels
B. Absence of peristalsis
C. Bright pink serous layer
D. Time of the incarceration is more than 6 hours

1.37 Which of the incarcerated hernias usually does not cause the symptoms of obstruction?
A. Fecal incarceration
B. Richter’s hernia
C. Sliding hernia
D. Postoperative midline hernia

1.38 Serious risks during inguinal hernia repair:


A. Bleeding
B. Bowel damage

lsmusis.lsmuni.lt/Klausimai/Spausdinti?Length=0?Kalba=EN&KategorijaId=103&Kalbos_input=EN&Kalbos=EN&KategorijaEn_input=Surgery&Kateg… 4/29
3/27/2019 LSMUSIS
C. Spermatic cord damage
D. Bladder damage
E. All above mentioned

1.39 Which of the investigations is most valuable for the diagnosis of the incarceration of obturator hernia?
A. Colposcopy
B. Ultrasound
C. Laparoscopy
D. Computed tomography

1.40 Which factor is the main in the pathogenesis of acute pancreatitis, caused by gallstones?
A. Obstruction of the pancreatic and bile ducts with a stone and disturbance of the evacuation of the
pancreatic juice
B. Disturbed evacuation of the bile from the gallbladder
C. Disturbance of the metabolism of the body
D. Excessive drinking
E. Obesity

1.41 Which of the causes of acute pancreatitis is most common in females?


A. Gallstones
B. Alcohol
C. Hyperlipidemia
D. Surgical procedure

1.42 Endoscopic retrograde cholangiography (ERCP) and papilospincterotomy (PST) are performed in emergent
order for a patient with:
A. Alcoholic acute pancreatitis
B. Acute pancreatitis caused by gallstones and common bile duct obstruction
C. Idiopatic (not clear origin) acute pancreatitis
D. Acute pancreatitis caused by abdominal trauma
E. Iatrogenic (after ERCP) acute pancreatitis

1.43 What investigation will allow to confirm the diagnosis of acute necrotizing pancreatitis?
A. Abdominal plain X-ray
B. Abdominal ultrasound scan
C. Investigation of serum amylase level
D. Contrast enhanced CT scan
E. Common blood test

1.44 Which investigation allows to differentiate between sterile and infected necrosis in severe acute pancreatitis?
A. Abdominal ultrasound
B. Endoscopic retrograde cholangiography (ERCP)
C. Image-guided fine needle aspiration (FNA)
D. Blood CRP level
E. Abdominal plain X-ray

1.45 The optimal time for surgery from the onset of acute necrotic pancreatitis?
A. First 5 days.
B. Second week.
C. Fourth week
D. First week.

1.46 Which type of intervention must be performed in case of acute necrotizing pancreatitis with sterile necrosis of
the tail of the pancreas confirmed with CT scan?
A. Distal pancreatectomy
B. Pacreatoduodenectomy
C. Pancreatectomy
D. Minimally invasive retroperitoneal necrosectomy.
E. None

1.47 The main processes taking place in early stage of acute pancreatitis
A. Local inflammation, infection, necrosis

lsmusis.lsmuni.lt/Klausimai/Spausdinti?Length=0?Kalba=EN&KategorijaId=103&Kalbos_input=EN&Kalbos=EN&KategorijaEn_input=Surgery&Kateg… 5/29
3/27/2019 LSMUSIS
B. Local inflammation, necrosis, SIRS
C. Necrosis, infection, SIRS
D. Necrosis, infection, sepsis
E. Local inflammation, necrosis, sepsis

1.48 Which of the causes of acute pancreatitis is most common for males?
A. Trauma
B. Surgery
C. Alcohol
D. Gallstones

1.49 The main environmental factors increasing the risk of pancreatic cancer
A. Alcohol consumption
B. Smoking
C. Spicy food
D. Long lasting biliary disorders

1.50 Extraductal obstruction of bile ducts is usually caused by


A. Cholangiocarcinoma
B. Periampullary tumors
C. Common bile duct stones
D. Primary sclerosing cholangitis
E. Biliary strictures

1.51 The first investigation of choice in case of clinically suspected mechanical jaundice?
A. Abdominal ultrasound
B. Computed tomography
C. ERCP
D. . Duodenoscopy with EUS

1.52 Which of changes in blood tests allow to confirm mechanical jaundice?


A. Elevation of AST and ALT
B. Elevation of alkaline phosphatase and gamma-glutamyltransferase
C. Isolated elevation of gamma-glutamyltransferase
D. Bilirubinemia

1.54 What is the first choice of treatment, when autonomous toxic adenoma is diagnosed?
A. Thyroid hormone therapy
B. Antithyroid medications
C. Hemithyroidectomy
D. Thyroidectomy
E. Subtotal resection of the thyroid gland

1.55 Which is the Grave’s disease?


A. Diffuse euthyroid goiter
B. Diffuse nodular toxic goiter
C. Diffuse thyrotoxic goiter
D. Autonomous adenoma of the thyroid

1.56 Which of the investigations is the best to diagnose the Thyrotoxic adenoma?
A. Barium swallow
B. Ultrasound scan
C. Thyroid scintigraphy
D. FNA of the node
E. Free thyroxine investigation

1.57 Which changes in blood tests allow to suspect hyperparathyroidism?


A. Uremia
B. Hypocalciemia
C. .Hypercalciemia
D. Hypercalciemia, hypophosphatemia

1.58 Which of the forms of the thyroid carcinoma has the most favourable prognosis?
lsmusis.lsmuni.lt/Klausimai/Spausdinti?Length=0?Kalba=EN&KategorijaId=103&Kalbos_input=EN&Kalbos=EN&KategorijaEn_input=Surgery&Kateg… 6/29
3/27/2019 LSMUSIS
A. Follicular carcinoma
B. Papillary carcinoma
C. Medullary carcinoma
D. Anaplastic carcinoma
E. Thyroid lymphoma

1.60 The most radical thyroid operation is:


A. Hemithyroidectomy
B. Subtotal resection of the thyroid
C. Thyroidectomy
D. Hemithyroidectomy and resection of the opposite side
E. Isthmusectomy and subtotal resection

1.61 What is the most frequent cause of the primary hyperparathyroidism?


A. Carcinoma of the parathyroid gland
B. Solitary parathyroid adenoma
C. Multiple parathyroidadenoma
D. Hyperplasia of the parathyroid glands
E. Hypoplasia of the parathyroid glands

1.62 Which investigation would you recommend suspecting the benign breast disease for a young woman?
A. Mammography
B. Ultrasound scan
C. Excisional biopsy
D. Computed tomography
E. Endoscopic investigation

1.63 The most common symptoms of the breast carcinoma are:


A. Constant induration, spontaneous excretion from the nipple, changes of breast skin
B. Constant induration, breast edema, enlarged axillary lymphnodes
C. Pain of the breast, induration, green discharges from the nipple at palpation
D. Changes of the breast form, induration, involvement of the nipple
E. Family history of breast carcinoma, microcalcifications at mammography

1.64 Which of the symptoms is not characteristic for the breast carcinoma?
A. Pain of the breast
B. Induration in the breast
C. Spontaneous excretion from the nipple
D. Microcalcifications
E. Involvement of the skin of the breast

1.65 Urgent surgery is necessary when we diagnose:


A. Breast carcinoma of 5 cm size
B. Purulent mastitis
C. Galactoductectasy and periductal mastitis
D. Fibrocystic mastopathy
E. Fibroadenoma of the breast

1.66 When portal hypertension is diagnosed indications for splenectomy are:


A. Ascitis
B. Varices of esophageal veins
C. Splenomegaly
D. Hypersplenismus
E. Hepatic coma

1.67 Indications for surgical treatment of chronic pancretitis are:


A. Diabetes mellitus
B. Insufficiency of exocrine pancreatic function
C. Disabling medically intractable pain
D. Pancreatic duct is enlarged up to 7 mm
E. Steatorrhea

lsmusis.lsmuni.lt/Klausimai/Spausdinti?Length=0?Kalba=EN&KategorijaId=103&Kalbos_input=EN&Kalbos=EN&KategorijaEn_input=Surgery&Kateg… 7/29
3/27/2019 LSMUSIS
1.68 Which investigation predicts the treatment strategy of tumor of the pancreas?
A. Abdominal ultrasound
B. Contrast enhanced CT scan
C. Serum bilirubin level
D. ERCP
E. US guided biopsy

1.69 The US or CT guided biopsy of hepatocellular carcinoma must be avoided because of


A. high risk of bleeding
B. seeding of the tumor
C. high risk of false negative test
D. high risk of biliary fistula

1.70 Most common site for pancreatic cancer:


A. Head
B. Body
C. Tail
D. Uncinate process

1.71 What are the typical symptoms of advanced cancer of the pancreas?
A. Weight loss, anorexia, weakness
B. Jaundice, pain
C. Motion disorders
D. Enlarged pancreas head and gallbladder
E. Cholangitis, diabetes mellitus

1.72 Which tumor marker is most informative to diagnose hepatocellular carcinoma?


A. CA 19-9
B. CEA
C. Ca-125
D. AFP
E. Ca 15-3

1.73 What is the most effective investigation to diagnose liver tumour location and growth character
A. Ultrasound scan
B. Selective angiography
C. Magnetic resonance imaging (MRI)
D. Computed tomography (CT)
E. Puncture biopsy

1.74 Heller procedure is:


A. Esophagocardiomyotomy
B. Gastrofundoplication
C. Resection of esophageal diverticulus
D. Endoscopic oesophageal dilation
E. Baloon pneumodilation

1.75 What is the most common reason of upper gastrointestinal tract bleeding?
A. Ulcer disease
B. Oesophageal varices
C. Erosive gastritis
D. Mallory-Weiss syndrome
E. Haemobilia

1.76 Which investigation you start from, if stomach or duodenum ulcer perforation is suspected?
A. Abdominal ultrasound scan
B. Upper gastrointestinal tract endoscopy
C. Barium meal test
D. Abdomen and chest x-ray
E. Laparoscopy

1.77 Indications for urgent surgery of peptic ulcer. Which answer is wrong?

lsmusis.lsmuni.lt/Klausimai/Spausdinti?Length=0?Kalba=EN&KategorijaId=103&Kalbos_input=EN&Kalbos=EN&KategorijaEn_input=Surgery&Kateg… 8/29
3/27/2019 LSMUSIS
A. Active bleeding, failure to achieve hemostasis endoscopically
B. Perforation
C. Hematemesis
D. Recurrent bleeding despite more than two endoscopic attempts at achieving hemostasis

1.78 The risks for bleeding peptic ulcer are:


A. Elderly age
B. Medical history of ulcer
C. Known H.pylori infection
D. Prolonged use of non steroid antiinflammatory medications
E. All above mentioned

1.79 The most effective grop of medications for peptic ulcer treatment
A. H2 blockers
B. Proton pump inhibitors
C. Prokinetics agents
D. Almagel
E. Sucralfat

1.80 Most common surgery procedure for perforated peptic ulcer:


A. Distal gastrectomy
B. Proximal gastrectomy
C. Ulcer suturing
D. Gastrojejunostomy
E. Gastric bypass

1.81 Which investigation is the most sensitive to detect bleeding site in the upper gastrointestinal tract?
A. Barium meal test
B. Diagnostic laparotomy
C. Upper gastrointestinal tract endoscopy
D. Angiography
E. Gastrointestinal bleeding scintigraphy

1.82 Patient is more likely to get gastric cancer if he/she:


A. Has a family history of gastric cancer
B. Had a polyp larger than 2 centimeters in his stomach
C. Has inflammation and swelling of the stomach for a long time (chronic atrophic gastritis)
D. Has an infection of the stomach by bacteria called Helicobacter pylori
E. All above mentioned answers are right

1.83 The most important factor which influences on 5 year survival after radical surgery for stomach cancer:
A. Number of affected lymphatic nodes
B. Age
C. Sex
D. Adjuvant chemotheraphy
E. Tumour location

1.84 The most common malignant gastric neoplasm is:


A. Carcinoid
B. Gastrointestinal stromal tumor (GIST)
C. Sarcoma
D. Planocellular carcinoma
E. Adenocarcinoma

1.85 What are the symptoms of acute paraproctitis?


A. Pain in anus, fever
B. Bleeding from the rectum
C. Anal itch

1.86 What is the most common surgical procedure in treatment of acute paraproctitis?
A. Incision
B. Incision and lateral sphincterotomy

lsmusis.lsmuni.lt/Klausimai/Spausdinti?Length=0?Kalba=EN&KategorijaId=103&Kalbos_input=EN&Kalbos=EN&KategorijaEn_input=Surgery&Kateg… 9/29
3/27/2019 LSMUSIS
C. Radical excision of inner hole
D. Preservation of anal sphincter

1.87 What is the treatment of acute inflammatory haemorrhoids ?


A. Always only conservative
B. Urgent operation m. Milligan –Morgan’s
C. The method depends on the number of inflammed haemorrhoids
D. First three days -thrombectomy, later-conservative
E. Overpressing rubber rings

1.88 What indicates the degree of internal haemorrhoids?


A. Bleeding
B. Size
C. Pain
D. Prolapse
E. Constipation

1.89 What is the most precise diagnostic method of anorectal fistula?


A. Investigation with the probe
B. Anoscopy
C. MRI
D. Fibrocolonoscopy

1.90 What is the most frequent symptom of anal fissure?


A. Anal pain in anus after defecation
B. Bleeding from the rectum at the end of defecation
C. Anal itch and mucous secretions from anus
D. Constipation

1.91 Indicate the wrong answer. What is typical for hereditary large bowel nonpolypous cancer syndrome?
A. At least 3 direct relatives must be diagnosed and histologically proved cancer of the large bowel.
B. Should not be family adenomatous polyposis
C. The disease has to recur in 2 sequent generations
D. Genetic testing is not necessary to confirm diagnosis
E. Cancer must be diagnosed untill the age of 50y

1.92 Risk factors of large bowel cancer are, except:


A. Congenital polyposis of large bowel
B. Nonspecific ulcerative colitis.
C. Inherited syndrom of large bowel nonpolypose family cancer.
D. Constipations longer than 1 week.
E. The patient had large bowel cancer before.

1.93 What procedure would you choose in case for surgery of the patient with mechanical bowel obstruction
caused by sigmoid cancer and perforated ascendent colon ?
A. Right hemicolectomy.
B. Drainage of the abdomen and colostomy.
C. Subtotal colectomy
D. The extraperitonisation of the perforated bowel and delayed resection.
E. Sigmoid resection and protective ileostomy

1.94 What is the optimal tactics in case of mechanical bowel obstruction caused by rectal cancer 10 cm from anus
?
A. Loop colostomy and delayed resection
B. Urgent radiotherapy and delayed anterior resection.
C. Endoscopic stenting and anterior resection
D. The extirpation of the rectum.
E. The resection of the rectum by Hartmann.

1.95 Diverticulitis is best diagnosed by:


A. CT scan
B. Fibrocolonoscopy

lsmusis.lsmuni.lt/Klausimai/Spausdinti?Length=0?Kalba=EN&KategorijaId=103&Kalbos_input=EN&Kalbos=EN&KategorijaEn_input=Surgery&Kate… 10/29
3/27/2019 LSMUSIS
C. Anmnestic data and clinical investigation
D. Barium enema

1.96 What is the optimal treatment of non-complicated acute diverticulitis?


A. Laparascopic sigmoid resection
B. Antibioticotherapy
C. .Open sigmoid resection m. Hartmann
D. Laparascopic revision and drainage

1.97 Bleeding from the anus at the end of defecation is typical for:
A. Anal fissure
B. Haemorrhoids
C. Diverticular disease
D. Rectosigmoid cancer

1.98 What are the symptoms of pelviorectal paraproctitis?


A. Anal pain, fever, pararectal suppuration
B. Anal pain with fever
C. Fever, weakness, intoxication
D. Mucous secretions from anus at the end of defecation

1.99 Which test best evaluates the anorectal fistula tract?


A. Dye (color) test
B. Checking the fistular canal by means of a probe
C. MRI
D. Fistulogramme

1.100 Fistular discision is the optimal surgical method in treatment of:


A. Intrasphincteric fistula
B. Extrasphincteric fistula
C. Transsphincteric fistula

1.101 What is your tactics if Dye test (color) of anorectal fistula does not show the localization of internal opening?
A. Leave it for healing
B. Opening the fistular canal by means of a probe to the closest crypt
C. MRI
D. Excision of fistula together with the closest crypt

1.185 What are indications for rubber band ligation of haemorrhoids (Barron’s procedure)?
A. Degree III haemorrhoids
B. When sclerotherapy is not effective
C. Degree II haemorrhoids, when conservative treatment is not effective

1.186 How to apply appropriate first aid to a burned patient (on site of injury)?
A. To give drink a lot of liquids
B. Remove burned layers as quick as possible
C. Inject antibiotics intravenously
D. Cool the burn wound with running water
E. Prescribe intravenous fluid therapy

1.187 How to treat superficial dermal (2A degree) burns?


A. Apply atraumatic dressings untill the wound epithelizes
B. Perform sharp debridement
C. Perform Excision and skin grafting
D. Perform immediate fasciotomy
E. Cover the wound with skin substitute till necrotic tissue will come out

1.188 Which material is considered standard to dress the burn wounds?


A. Buckthorn (hippophae)-plant oil
B. Silver sulfadiazine ointment
C. Penicillin solution
D. Cotton gauze soaked with sterile water
E. Gentamycin ointment
lsmusis.lsmuni.lt/Klausimai/Spausdinti?Length=0?Kalba=EN&KategorijaId=103&Kalbos_input=EN&Kalbos=EN&KategorijaEn_input=Surgery&Kate… 11/29
3/27/2019 LSMUSIS
1.189 How to treat the donor site of the skin graft?
A. Dress twice a day with cotton gauze
B. Cover the wound with skin substitute till necrotic tissue will come out
C. Apply porcine skin graft (xenograft)
D. Dress at the time of surgery, leave dressings on the wound unchanged later
E. Dress once a day using antibiotic ointment.

II type tasks. For each numbered item,selct the one lettered heading that is most closely asssciated with it

1.102 Which of the symptoms characteristic of these diseases?


1 - Thyrotoxicosis
2 - primary hyperparathyroidism
3 - pheochromocytoma
4 - hypothyroidism
5 - insulinoma
A. muscle pain
B. dry skin
C. perspiration
D. hypertensive crisis
E. hypoglycemic coma

1.103 What measures are being taken to stop bleeding from the sources listed below
1 - Temporary homeostasis of the common carotid artery
2 - Temporary homeostasis of the popliteal artery
3 - Temporary homeostasis of calf varicose vein
4 - Temporary homeostasis of the facial artery
5 - Temporary homeostasis of the superficial temporal artery
A. apply a bandage
B. press with a finger against the vertebral column
C. press with a finger against the zygomatic bone
D. apply a screw
E. press with a finger near the mandibular body

1.104 Which of the symptoms characteristic of these diseases?


1 - chronic hemorrhoid
2 - acute inflammatory hemorrhoid
3 - acute paraproctitis
4 - anal fissure
5 - rectal prolapsus
A. pain and fever
B. protrusion and pain
C. bleeding and fever
D. pain and bleeding
E. protrusion and bleeding

1.105 Which of the symptoms characteristic of these diseases?


1 - acute cholecystitis
2 - nephrolithiasis
3 - ileus
4 - mesenteric vessels thrombembolism
5 - rupture of aortal aneurism
A. cardiac arrhytmia
B. spastic abdominal pain
C. local pain in the projection of organ
D. increase in dimensions of the abdomen
E. shock

1.106 Which of the symptoms characteristic of these diseases?


1 - acute appendicitis
2 - bleeding peptic ulcer
3 - pelvic inflammatory disease
4 - chronic appendicitis
lsmusis.lsmuni.lt/Klausimai/Spausdinti?Length=0?Kalba=EN&KategorijaId=103&Kalbos_input=EN&Kalbos=EN&KategorijaEn_input=Surgery&Kate… 12/29
3/27/2019 LSMUSIS
5 - kidney stones
A. “coffee ground” vomiting
B. local pain in the projection of organ
C. diffuse abdominal pain
D. colicky pain
E. lower abdominal pain

1.107 Which is the most appropriate test to diagnose below listed diseases?
1 - upper gastrointestinal endoscopy
2 - ultrasound scan
3 - angiography
4 - abdominal x-ray test
5 - transilumination
A. irreponible scrotal hernia
B. acute appendicitis
C. oesophageal hiatus hernia
D. acute mesenteric ischaemia
E. perforated duodenal ulcer

1.108 Which of the below mentioned signs is characteristic of the diseases?


1 - Cushing syndrome
2 - pheochromocytoma
3 - Conn’s syndrome
4 - adrenal tumor
5 - MEN I syndrome
A. hirsutism
B. hypocalemia
C. hypercalcemia
D. hypertensive crisis
E. osteoporosis

1.109 Which is the most appropriate test to diagnose below listed diseases?
1 - Angiography
2 - upper gastrointestinal endoscopy
3 - colonoscopy
4 - barium follow throw
5 - ultrasound scan
A. large bowel obstruction
B. acute appendicitis
C. small bowel obstruction
D. acute mesenteric ischaemia
E. Mallory-Weiss syndrome

1.110 Which symptoms are the most common in case of these perianal region diseases?
1 - anal fissure
2 - chronic hemorrhoids
3 - acute paraproctitis
4 - inflammed bleeding haemorrhoid
A. pain near the anus with fever
B. anal pain after defecation
C. bleeding at the end of defecation
D. painful tenderness near anus with bleeding

1.111 Which is the most appropriate test to diagnose below listed diseases?
1 - inspection
2 - palpation
3 - rectoscopy
4 - anoscope
A. acute paraproctitis
B. rectal cancer
C. chronic I degree hemorrhoid
D. anal fissure
lsmusis.lsmuni.lt/Klausimai/Spausdinti?Length=0?Kalba=EN&KategorijaId=103&Kalbos_input=EN&Kalbos=EN&KategorijaEn_input=Surgery&Kate… 13/29
3/27/2019 LSMUSIS
1.112 Which type of colon preparation is the most appropriate before treatment?
1 - chronic hemorrhoid
2 - acute paraproctitis
3 - rectal carcinoma
4 - carcinoma of the transverse colon
A. no preparation
B. phosphate enema
C. per oral intestinal cleansing medications
D. per oral gastrografin

1.114 Which of the symptoms characteristic of these diseases?


1 - anal fissure
2 - hemorrhoid
3 - rectal carcinoma
4 - anorectal fistula
A. wetness near the anus
B. anal pain after defecation
C. bleeding at the end of defecation
D. defecating thin faeces

1.115 Which of the symptoms characteristic of these diseases?


C 1 - nonspecific ulcerative colitis
D 2 - Crohn’s disease
B 3 - left side colon cancer
A 4 - diverticulitis
A. pain in the left lower abdomen
B. progressive obstruction symptoms
C. diarrhea with bloody and mucous mixture
D. anal fissure and fistula

1.116 Which type of treatment is the most appropriate for these diseases?
1 - anal fissure
2 - intrasphincteric anorectal fistula
3 - acute paraproctitis
4 - anal papilloma
A. dissection
B. incision
C. sphicterotomy
D. excision

1.117 Which of the symptoms characteristic of these diseases?


1 - small bowel obstruction
2 - acute appendicitis
3 - perforated duodenal ulcer
4 - acute mesenteric ischemia
5 - ectopic pregnancy
A. diffusive muscle guarding
B. Kocher symptom
C. enlargement of the abdomen size
D. weakness
E. diffusive visceral abdominal pain

1.118 Which is the most appropriate test to diagnose below listed diseases?
1 - Angiography
2 - Ultrasound scan
3 - Upper gastrointestinal endoscopy
4 - Abdomen X - ray
5 - Computed tomography (CT)
A. Acute appendicitis
B. Acute pancreatitis
C. Acute mesenteric ischemia
D. Bowel obstruction
lsmusis.lsmuni.lt/Klausimai/Spausdinti?Length=0?Kalba=EN&KategorijaId=103&Kalbos_input=EN&Kalbos=EN&KategorijaEn_input=Surgery&Kate… 14/29
3/27/2019 LSMUSIS
E. Gastrointestinal bleeding

1.119 Which of the symptoms characteristic of these diseases?


1 - Cholecystitis
2 - Bowel obstruction
3 - Acute mesenteric ischemia
4 - Gastrointestinal bleeding
5 - Incarcerated hernia
A. Acute diffusive pain of the abdomen
B. Hematemesis
C. Painful mass in the inquinal region
D. Active high tone peristalsis
E. Colicky pain episode

1.120 Which is the most appropriate test to diagnose below listed diseases?
1 - Urgent angiography
2 - Urgent upper gastrointestinal endoscopy
3 - Urgent ultrasound scan
4 - Urgent surgery
5 - Abdomen X - ray
A. Blunt abdominal trauma
B. Acute mesenteric ischemia
C. Mallory – Weiss syndrome
D. Bowel obstruction
E. Stab wound in the heart projection

1.121 Which type of treatment is the most appropriate for these diseases?
E 1 - Laparoscopic adhesiot
D 2 - Large bowel resection
B 3 - Large bowel fixation
C 4 - Small bowel resection
A 5 - Conservative treatment
A. Crohn’s disease without complications
B. Large bowel volvulus
C. Intussusception due to polypus
D. Carcinoma of the large bowel
E. Obstruction of the bowel caused by adhesions

1.122 Which of the symptoms characteristic of these diseases?


1 - Prostatitis
2 - Obstruction of the large bowel
3 - Acute appendicitis
4 - Mallory – Weiss syndrome
5 - Kidney stones
A. Colicky pain
B. Pain during defecation
C. Local pain
D. Melena
E. Inability to pass gas and stool

1.123 Which of the symptoms characteristic of these diseases?


1 - Small bowel obstruction
2 - Ovarian apoplexy
3 - Duodenal ulcer
4 - Portal vein thrombosis
5 - Acute mesenteric ischemia
A. Fast fluid accumulation into abdominal cavity
B. Diffusive abdominal pain
C. Acute weakness
D. Melena
E. Vomiting

lsmusis.lsmuni.lt/Klausimai/Spausdinti?Length=0?Kalba=EN&KategorijaId=103&Kalbos_input=EN&Kalbos=EN&KategorijaEn_input=Surgery&Kate… 15/29
3/27/2019 LSMUSIS
1.124 Which is the most appropriate test to diagnose below listed diseases?
1 - Barium follow throw test
2 - Colonoscopy
3 - Ultrasound scan
4 - Magnetic resonance imaging (MRI)
5 - Computed tomography (CT)
A. Acute appendicitis
B. Bowel obstruction
C. Acute pancreatitis
D. Soft tissue sarcoma
E. Crohn`s disease

1.190 Which of the symptoms characteristic of these diseases?


1 - Rupture of colon ascendens between mesocolon peritoneal leaflets
2 - Rupture of posterior wall of the stomach
3 - Deep rupture of the diaphragmatic surface of liver
4 - Broad rupture of jejunum
5 - Subcapsular spleen rupture
A. Pain in left upper quadrant of the abdomen
B. Abdominal muscles guarding, Blumberg symptom
C. Anemia, pain in the neck
D. Free air under right site of the diaphragm
E. Subcutaneous emphysema in the abdomen

1.191 Which statements are correct for the below mentioned diseases?
1 - Incarcerated femoral hernia
2 - Incarcerated omentum
3 - Retrograde hernia incarceration
4 - Richter’s incarcerated hernia
5 - Incarcerated inquinal hernia
A. Symptoms without bowel obstruction
B. More frequent among men rather than women
C. Impossible to palpate
D. More frequent among women rather than men
E. Palpation of the incarcerated hernia is not painful

1.192 Which of the symptoms characteristic of these diseases?


1 - Bleeding stomach ulcer
2 - Mallory – Weiss syndrome
3 - Cirrhosis of the liver
4 - Diverticulosis of the large bowel
A. Profuse vomiting, subsequent bleeding
B. There is no blood in duodenum during the bleeding
C. The pain can decrease after start of the bleeding
D. Splenomegaly is a common finding.

III type tasks. For each question there is one or more correct answers:
A – if correct answers are 1,2,3
B – if correct answers are 1 and 3
C – if correct answers are 2 and 4
D – if correct answer is 4
E – if correct are all answers above

1.125 Diffuse abdominal pain can be caused by:


1. Uremia
2. Diffuse peritonitis
3. Acute bowel obstruction
4. Acute mesenteric ischemia

1.126 Acute appendicitis can be suspected if:


1. Pain is localized in the epigastric area

lsmusis.lsmuni.lt/Klausimai/Spausdinti?Length=0?Kalba=EN&KategorijaId=103&Kalbos_input=EN&Kalbos=EN&KategorijaEn_input=Surgery&Kate… 16/29
3/27/2019 LSMUSIS
2. Local guarding present in the right lower quadrant of the abdomen
3. Abdomen is much distended
4. Presence of Kocher sign
5. Loud peristalsis waves

1.127 Postoperative acute appendicitis complications are:


1. Wound infection
2. Bleeding into abdominal cavity
3. Intraperitoneal abscess
4. Pylephlebitis
5. Fistula

1.128 Class of hemorrhagic shock depends on:


1. Pulse rate
2. Blood pressure
3. Diuresis
4. Circulating blood volume
5. Hb, Ht

1.129 In case of liver injury, bleeding can be stopped using these methods:
1. Electrocoagulation
2. Suturing
3. Packing
4. Common hepatic artery ligation
5. Liver resection

1.130 In case of retroperitoneal rectal injury, colosomy is mandatory when:


1. Time since injury > 6 hours
2. Systolic blood pressure > 100 mmHg
3. Blood serum Hb < 80g/l
4. Gas and stool appear through the wound

1.131 In case of subcapsular spleen rupture, clinical signs during the first hours are:
1. Hemorrhagic shock
2. Decrease of Hb and erythrocytes in the blood
3. Rebound tenderness in the left abdomen side
4. Pain in the trauma area

1.132 Indications for urgent surgery in case of abdominal trauma are:


1. Internal bleeding causing hemorrhagic shock
2. Symptoms of diffuse peritonitis
3. Protrusion of visceral organs
4. 100 ml of fluid was found in the abdominal cavity on ultrasound scan examination 4 hours after injury

1.133 Polytrauma patient management:


1. Release airway
2. Maintain circulation
3. Maintain breathing
4. Cool the body
5. Perform laparoscopy

1.134 Bowel vitality in case of incarcerated hernia shows:


1. Color
2. Gloss of the serosa
3. Pulsation of the mesenteric blood vessels
4. Incarceration time
5. General patient status

1.135 What (which) kind of tissue plastic method (methods) you will choose to repair recurrent hernia?
1. Autoplastic
2. Homoplastic
3. Alloplastic

lsmusis.lsmuni.lt/Klausimai/Spausdinti?Length=0?Kalba=EN&KategorijaId=103&Kalbos_input=EN&Kalbos=EN&KategorijaEn_input=Surgery&Kate… 17/29
3/27/2019 LSMUSIS
4. Heteroplastic
5. Ksenoplastic

1.136 What pathology will you differentiate with, if you have incarcerated femoral hernia:
1. V. saphena magna thrombophlebitis
2. Rozen – Miuler lymphatic node
3. “Cold” abscess
4. Bowel obstruction caused by adhaesions
5. Richter’s hernia

1.137 Inguinal hernia incarceration predisposes:


1. Elderly age of patient
2. Chronic bronchitis
3. Indirect hernia
4. Constipation
5. Good condition of abdominal wall muscles

1.138 Mesenteric artery embolism must be suspected if:


1. Elderly patient
2. Cardiac arythmias was or is present
3. Acute abdominal pain
4. Flatulence of abdomen
5. Melena is present

1.139 True about the role of medical therapy in Barrett's esophagus:


1. There is direct evidence that medical treatment prevents development of Barrett's esophagus in patients
with GERD
2. Prokinetic agents must be prescriber if high grade dysplasia is diagnosed
3. COX 2 inhibitors have a role in prevention of adenocarcinoma in patients with Barrett's esophagus
4. The end point of treatment in Barrett's esophagus is control of reflux

1.140 Symptoms which are characteristic for thrombosis of v.portae:


1. Shock
2. Ascites
3. Diarrhea
4. Jaundice
5. Dysuria

1.141 Reasons that can cause bowel obstruction:


1. Hernias
2. Tumors
3. Malrotation
4. Foreign bodies
5. Strictures of intestine

1.142 A 50year old female undergoes a laparotomy for acute small bowel obstruction due to adhaesions. On day 6
postoperatively she develops small bowel fistula throw the wound without diffuse peritonitis. Management?
1. relaparotomy, small bowel resection with anastomosis
2. relaparotomy, small bowel resection, ileostomy
3. local enterocutaneous fistula suturing
4. conservative management including antibiotics, active fistula production aspiration
5. all above mentioned methods are possible

1.143 When volvulus of the large bowel is diagnosed, recommendations are:


1. perform bowel resection if there is necrosis of the bowel
2. restore the bowel passing by blowing the air under x-ray control
3. derotate the large bowel endoscopicaly and fix it, if there is no mucosa necrosis
4. perform an phosphate enema
5. start conservative therapy (i/v fluids) and monitor the patient

1.144 When large bowel obstruction, caused by cancer is suspected, it is necessary to:
1. perform barium enema test

lsmusis.lsmuni.lt/Klausimai/Spausdinti?Length=0?Kalba=EN&KategorijaId=103&Kalbos_input=EN&Kalbos=EN&KategorijaEn_input=Surgery&Kate… 18/29
3/27/2019 LSMUSIS
2. perform colonoscopy
3. perform endoscopic recanalisation of large bowel
4. operate immediately
5. perform angiography

1.145 Laparoscopic adhesiolysis in case of small bowel obstruction is recommended:


1. when patient has a history of previous surgery
2. when patient suffers from diarrhea
3. when abdomen bloating is frequent
4. when the only surgery before is appendectomy
5. when abdominal pain episodes are frequent

1.146 Before surgery due to small bowel obstruction is necessary:


1. To perform decompression of small bowel if it‘s possible
2. To maintain homeostasis correction
3. To start wide spectrum antibiotics
4. To administer spasmolytics
5. To perform an enema

1.148 Which does not predispose to Carcinoma stomach?


1. Low fat and protein diet
2. Salted meat and fish
3. Smoking
4. Low nitrate consumption
5. Prior gastric surgery

1.149 Radical gastrectomy with extended lymphonodectomy is the choice when the stomach cancer is staging as:
1. T1 N2 M1
2. T4 N1 M0
3. T1 N0 M1
4. T2 N0 M0
5. T1 N3 M1

1.150 Most common methods of treatment for cardiac achalasia are:


1. Baloon dilation
2. Botuline injection
3. Heller‘s operation
4. Torec‘s operation
5. Gastrostomy

1.151 Invasive nonoperative treatment of haemorrhoids are:


1. Injective sclerotherapy
2. Ambulatory trombectomy
3. Clipsing with rubber ring
4. Milligan - Morgan‘s operation
5. Dilatation of anal sphincters

1.152 Operative treatment of anorectal fistula:


1. lateral sphincterotomy
2. fistula excision
3. incision
4. free ligation of fistula

1.153 Surgical treatment for chronic anal fissure:


1. lateral closed sphincterotomy
2. lateral closed sphincterotomy with tear excision
3. open posterior sphincterotomy with tear excision
4. mucosa membrane excision
5. incision

1.154 Which of these propositions describes the anal fissure:


1. atony of anal sphincters

lsmusis.lsmuni.lt/Klausimai/Spausdinti?Length=0?Kalba=EN&KategorijaId=103&Kalbos_input=EN&Kalbos=EN&KategorijaEn_input=Surgery&Kate… 19/29
3/27/2019 LSMUSIS
2. bleeding with mucous
3. only conservative treatment
4. doesn‘t heal because of trophic disorders

1.155 Acute paraproctitis treatment:


1. incision and washout
2. incision and drainage
3. incision and free ligature
4. incision and elastic ligature
5. Gabriel‘s operation

1.156 For chronic haemorrhoids is characteristic:


1. bleeding with bright blood
2. anorectal pain between passages
3. haemorrhoids prolapse in time of passage
4. only surgical treatment

1.157 Treatment of chronic haemorrhoids is:


1. diet
2. relief of passage
3. sedentary baths
4. vasotonics
5. rectal suppository

1.158 Invasive treatment of haemorrhoids:


1. sclerotherapy
2. infrared coagulation
3. band ligation
4. sacroiliac blockade

1.159 For acute haemorrhoids are characteristic:


1. Severe bleeding
2. anorectal pain
3. only surgical treatment
4. diagnosed on medical examination
5. rectoscopy is mandatory

1.160 For rectal cancer is characteristic:


1. bleeding at the end of passage
2. blood in faeces
3. pain at the end of passage
4. passage with linear excrements

1.161 Urgent surgery procedure due to nonspecific ulcerative colitis is:


1. hemicolectomy
2. proctocolectomy
3. ileostomy
4. colectomy

1.162 Procedures to treat rectal cancer are:


1. rectal resection
2. extirpation of the rectum
3. posterior proctectomy and tumor excision
4. transanal tumor excision

1.163 Operations due to left side large bowel cancer are:


1. left hemicolectomy
2. transversosigmostomy
3. transversostomy
4. endoscopic tumor excision

1.164 Operations due to right side large bowel cancer are:


1. right hemicolectomy
lsmusis.lsmuni.lt/Klausimai/Spausdinti?Length=0?Kalba=EN&KategorijaId=103&Kalbos_input=EN&Kalbos=EN&KategorijaEn_input=Surgery&Kate… 20/29
3/27/2019 LSMUSIS
2. ileotransversostomy
3. ileostomy
4. left hemicolectomy

1.193 The most frequent spread ways of large bowel cancer metastasis are:
1. Brain
2. Lungs
3. Kidneys
4. Liver

1.194 Complications of nonspecific ulcerative colitis are:


1. Toxic large bowel dilation
2. Large bowel perforation
3. Profuse lower gastrointestinal tract bleeding
4. Bowel obstruction

1.195 Noninvasive treatment of hemorrhoids are:


1. Rectal suppository
2. Scleroterapy
3. Bath with KMnO4
4. Milligan-Morgan haemorrhoidectomy

1.196 For deep dermal (2B degree) burn is specific:


1. Wound with blisters
2. Wound infection
3. Wound covered by thick layer of necrotic tissues with visible subcutaneous thrombotic vessels
4. Wound covered by thin layer of necrotic tissue

1.197 First aid for burns is:


1. Cool the burn wound with ice or snow
2. Cover burned area with sea buckthorn oil
3. Apply artificial skin
4. Cool the burn wound with running water

IV type tasks. Choose only one best answer

1.165 64 year old female was brought to Emergency complaining of pain in epigastrium and right subcostal area,
chills and fever. The duration of the pain is one day. The pain is persistent. The temperature was spiking up to
38,9°C with chills. On examination: body temperature 37.5, pulse rate 84 beats per minute, blood pressure 146/75
mm Hg. Skin and sclera are yellow. The abdomen is not blown, soft, slight tender in right upper quadrant. Bowel
sounds present. Your preliminary diagnosis:
A. Viral hepatitis;
B. Acute cholecystitis;
C. Mechanical jaundice with cholangitis;
D. Liver tumor.

1.166 64 year old female was brought to Emergency complaining of pain in epigastrium and right subcostal area,
chills and fever. The duration of the pain is one day. The pain is persistent. The temperature was spiking up to
38,9°C with chills. On examination: body temperature 37.5, pulse rate 84 beats per minute, blood pressure 146/75
mm Hg. Skin and sclera are yellow. The abdomen is not blown, soft, slight tender in right upper quadrant. Bowel
sounds present. What you will do first of all?

A. Plain abdominal X-ray;


B. ERCP;
C. Abdominal ultrasound;
D. Blood tests including serum bilirubin level, liver enzymes analysis, CRP.

1.167 54 year old male with history of viral hepatitis C and liver cirrhosis was on follow up under
gastroenterologist. At the time of routine examination AFP test was made. The result was 200 ng/ml. Abdominal
ultrasound, contrast enhanced CT, MRI didn’t revealed any tumors in the liver. What investigation plan will be the
best option for this patient?
A. PET scan

lsmusis.lsmuni.lt/Klausimai/Spausdinti?Length=0?Kalba=EN&KategorijaId=103&Kalbos_input=EN&Kalbos=EN&KategorijaEn_input=Surgery&Kate… 21/29
3/27/2019 LSMUSIS
B. Conventional angiography
C. Repeated abdominal ultrasound and AFP every 3 months
D. ERCP
E. Laparoscopy

1.168 46 year old male was admitted to gastroenterology department due to jaundice and weight loss. The patient
have noticed jaundice 2 weeks ago. There was no abdominal pain, fever and chills. Last two weeks feces became
acholic, urine is dark brownish. On examination: body temperature normal. Skin and sclera of the eyes yellow.
Abdomen is soft, without tenderness. Serum bilirubin level 160mmol/L, direct bilirubin - 98 μmol/L, alkaline
phosphatase – 1120 IU/L and gamma-glutamyltransferase – 798 IU/L. Abdominal ultrasound: common bile duct 13
mm Ø, there are no stones in gall-bladder. ERCP revealed stricture of the distal CBD. The stricture was
recanalysed with plastic endoprothesis. Contrast enhanced CT scan didn’t revealed any tumors in pancreas. Your
main preliminary diagnosis:
A. Choledocholithiasis
B. Benign biliary stricture
C. Cholangiocarcinoma of distal CBD
D. Duodenum tumor

1.169 46 year old male was admitted to gastroenterology department due to jaundice and weight loss. The patient
have noticed jaundice 2 weeks ago. There was no abdominal pain, fever and chills. Last two weeks feces became
acholic, urine is dark brownish. On examination: body temperature normal. Skin and sclera of the eyes yellow.
Abdomen is soft, without tenderness. Serum bilirubin level 160mmol/L, direct bilirubin - 98 μmol/L, alkaline
phosphatase – 1120 IU/L and gamma-glutamyltransferase – 798 IU/L. Abdominal ultrasound: common bile duct 13
mm Ø, there are no stones in gall-bladder. ERCP revealed stricture of the distal CBD. The stricture was
recanalysed with plastic endoprothesis. Contrast enhanced CT scan didn’t revealed any tumors in pancreas. What
kind of surgery is necessary for the patien? A. ; B. ; C. ; D. .
A. Resection of the extrahepatic biliary ducts
B. Biliodigestive bypass
C. Pancreatoduodenectomy
D. Percutaneous transhepatic stenting

1.170 62-year-old female is under investigation due to recent episode of low back pain and known kidney stones.
Her estimated calcium intake is 800mg/day and she takes no vitamins. Her physical examination is unremarkable.
Spinal x-ray reveals osteopenia and a compression fracture of the second lumbar vertebra (L2). Laboratory tests:
serum calcium – 13.0mg/dL (n 8.5-10.5), phosphorus – 2,3mg/dL (n 2.5-4.5), albumin- 4.4 g/dL (n 3.2-5.5), intact
PTH- 72pg/mL (n 11-54), and 24-hour urine calcium – 312mg (n 100-300). What is the most likely diagnosis?
A. Multiple endocrine neoplasia II syndrome
B. Familial hypocalciuric hypercalciemia
C. Solitary parathyroid adenoma
D. Parathyroid carcinoma

1.171 62-year-old female is under investigation due to recent episode of low back pain and known kidney stones.
Her estimated calcium intake is 800mg/day and she takes no vitamins. Her physical examination is unremarkable.
Spinal x-ray reveals osteopenia and a compression fracture of the second lumbar vertebra (L2). Laboratory tests:
serum calcium – 13.0mg/dL (n 8.5-10.5), phosphorus – 2,3mg/dL (n 2.5-4.5), albumin- 4.4 g/dL (n 3.2-5.5), intact
PTH- 72pg/mL (n 11-54), and 24-hour urine calcium – 312mg (n 100-300). What is the most informative
investigation?
A. Computed tomography of the thorax and neck
B. Scintigraphy with Tc99-MIBI
C. Plain chest x-ray
D. Abdominal ultrasound

1.172 65 years old male had a long lasting constipation, but the last two months his bowel habits became normal.
One week ago he noticed bloody stools with clots. What disease you will suspect?
A. Colon cancer
B. Ulcerative colitis
C. Diverticulitis
D. Haemorrhoids

1.173 65 years old male had a long lasting constipation, but the last two months his bowel habits became normal.
One week ago he noticed bloody stools with clots. Which test you start from to define the problem?
A. Colonoscopy
lsmusis.lsmuni.lt/Klausimai/Spausdinti?Length=0?Kalba=EN&KategorijaId=103&Kalbos_input=EN&Kalbos=EN&KategorijaEn_input=Surgery&Kate… 22/29
3/27/2019 LSMUSIS
B. CT scan of pelvis
C. Flexible sigmoidoscopy
D. Plain abdominal X-ray

1.174 Institutionalized 85 years old female has had constipations for 40 years. She is complaining of a mild
bleeding from the anus, fecal and gas incontinence, and a tumor in the perineum now. These complaints occured
2 months ago. What disease you will suspect first?
A. Haemorrhoids (piles)
B. Rectal cancer
C. Rectal prolapse
D. Rectocele (proctocele)

1.175 Institutionalized 85 years old female has had constipations for 40 years. She is complaining of a mild
bleeding from the anus, fecal and gas incontinence, and a tumor in the perineum now. These complaints occured
2 months ago. Which test will help you to determine the proper diagnosis?
A. Inspection of the anus and perineum
B. Per-rectal examination
C. Proctosigmoidoscopy
D. Anorectal manometry

1.176 50 years old male was suffers from ulcerative colitis for more than 30 years. He used medications irregularly.
Complains of flatulence, fever (38,6 degrees of Celsius) now. He has opened bowels 3-4 times per day until this
exacerbation. He is evacuating once a day now. History of disease - frequent colitis exacerbation. An objective
examination have revealed: puls rate - 108 beats per minute, blood pressure - 100/74 mm Hg. The abdomen is
moderately blown, soft, and slightly tender in the upper region and in the right iliac fossa. Signs of rebound
tenderness are absent. Per-rectal examination - traces of bloody feces. WBC count - 18,0X109/l, Hb - 88 g/l,
packed cell volume - 0,34. What disease you will suspect first?
A. Acute mechanical bowel obstruction
B. Toxic colon dilation
C. Non-specific colitis exacerbation
D. Colon perforation

1.177 50 years old male was suffers from ulcerative colitis for more than 30 years. He used medications irregularly.
Complains of flatulence, fever (38,6 degrees of Celsius) now. He has opened bowels 3-4 times per day until this
exacerbation. He is evacuating once a day now. History of disease - frequent colitis exacerbation. An objective
examination have revealed: puls rate - 108 beats per minute, blood pressure - 100/74 mm Hg. The abdomen is
moderately blown, soft, and slightly tender in the upper region and in the right iliac fossa. Signs of rebound
tenderness are absent. Per-rectal examination - traces of bloody feces. WBC count - 18,0X109/l, Hb - 88 g/l,
packed cell volume - 0,34. Which test you will perform first?
A. Ultrasound scan of the abdomen
B. Double-contrast barium enema
C. Colonoscopy
D. Plain abdominal X-ray
E. CT scan

1.178 18 year old female feels herself unwell for some years. She had multiple episodes of bloody diarrhea in the
past. She had an appointment with GP, anemia was diagnosed and iron tablets were prescribed. Her mother and
sister died from colon cancer in their youth. What disease will you suspect?
A. Non-specific colitis
B. Crohn's disease
C. Familial adenomatous polyposis
D. Colon diverticulosis

1.179 18 year old female feels herself unwell for some years. She had multiple episodes of bloody diarrhea in the
past. She had an appointment with GP, anemia was diagnosed and iron tablets were prescribed. Her mother and
sister died from colon cancer in their youth. Which test you will do to define the precise diagnosis?
A. Barium enema
B. Plain abdominal X-ray
C. Ultrasound scan of the abdomen
D. Colonoscopy

lsmusis.lsmuni.lt/Klausimai/Spausdinti?Length=0?Kalba=EN&KategorijaId=103&Kalbos_input=EN&Kalbos=EN&KategorijaEn_input=Surgery&Kate… 23/29
3/27/2019 LSMUSIS
1.180 67 year old female suddenly had profuse bloody diarrhea with clots. Before the episode she had
constipations for many years and suffered from pain in the left iliac fossa area. What disease you will suspect?
A. Colon cancer
B. Ulcerative colitis
C. Diverticulitis
D. Haemorrhoids

1.181 67 year old female suddenly had profuse bloody diarrhea with clots. Before the episode she had
constipations for many years and suffered from pain in the left iliac fossa area. Which test you will do to define the
diagnosis?
A. Proctosigmoidoscopy
B. CT scan pelvis
C. Angiography
D. Plain abdominal X-ray

1.182 76 years old male has arrived to Emergency Department, complaining of sharp unlocalized pain in the
abdomen, which started 20 minutes ago. He didn’t have nausea or vomiting, but was sick and had heartburn for
some days before. On examination: lips are cyanotic and tongue is wet and clean. Blood pressure - 100/60 mm
Hg. Puls irregular, rate 94 beats per minute. There is crepitating in the lower part of the lungs. The abdomen is
slightly distended; there are enlarged veins of the abdomen wall. The abdomen is soft, peristalsis is active. Body
temperature – 36.2 degrees of Celsius. What disease you will suspect first of all?
A. Destructive appendicitis
B. Perforated duodenal ulcer
C. Acute pancreatitis
D. Kidney stones
E. Acute mesenteric ischemia

1.183 76 years old male has arrived to Emergency Department, complaining of sharp unlocalized pain in the
abdomen, which started 20 minutes ago. He didn’t have nausea or vomiting, but was sick and had heartburn for
some days before. On examination: lips are cyanotic and tongue is wet and clean. Blood pressure - 100/60 mm
Hg. Puls irregular, rate 94 beats per minute. There is crepitating in the lower part of the lungs. The abdomen is
slightly distended; there are enlarged veins of the abdomen wall. The abdomen is soft, peristalsis is active. Body
temperature – 36.2 degrees of Celsius. Which test you will perform first to confirm the preliminary diagnosis?
A. Plain abdominal X-ray
B. Ultrasound of the abdomen
C. CT scan
D. Angiography
E. Laparoscopy

1.184 76 years old male has arrived to Emergency Department, complaining of sharp unlocalized pain in the
abdomen, which started 20 minutes ago. He didn’t have nausea or vomiting, but was sick and had heartburn for
some days before. On examination: lips are cyanotic and tongue is wet and clean. Blood pressure - 100/60 mm
Hg. Puls irregular, rate 94 beats per minute. There is crepitating in the lower part of the lungs. The abdomen is
slightly distended; there are enlarged veins of the abdomen wall. The abdomen is soft, peristalsis is active. Body
temperature – 36.2 degrees of Celsius. What laboratory findings of this patient you will expect?
A. Anemia
B. Raised WBC count
C. Hypoglycemia
D. Uremia
E. Bilirubinemia

1.198 36 years old female was ill for 16 hours. She came to the Emergency Department complaining of flatulence,
constant abdominal pain, vomiting, retention of feces and gas. Such abdominal pain attack has occurred for the
first time. Medical history consists from surgery due to ectopic pregnancy few years ago. She lifted heavy thing
before this attack and then griping pain started. She vomited several times. On examination: tongue is coated.
Blood pressure - 140/80 mm Hg, puls rate is 96 beats per minute, regular. Chest sounds are clear. Abdomen is
increased in size, blown more in the lower region. Bowel sounds absent; there is fluid collection in the intestine.
Abdomen is tender, more on the right side, where painful mass is palpable. Signs of rebound tenderness are
present. What disease you will suspect?
A. Acute pancreatitis
B. Acute mesenteric ischaemia
C. Intestinal obstruction caused by adhesions
lsmusis.lsmuni.lt/Klausimai/Spausdinti?Length=0?Kalba=EN&KategorijaId=103&Kalbos_input=EN&Kalbos=EN&KategorijaEn_input=Surgery&Kate… 24/29
3/27/2019 LSMUSIS
D. Destructive appendicitis
E. Diverticulitis

1.199 36 years old female was ill for 16 hours. She came to the Emergency Department complaining of flatulence,
constant abdominal pain, vomiting, retention of feces and gas. Such abdominal pain attack has occurred for the
first time. Medical history consists from surgery due to ectopic pregnancy few years ago. She lifted heavy thing
before this attack and then griping pain started. She vomited several times. On examination: tongue is coated.
Blood pressure - 140/80 mm Hg, puls rate is 96 beats per minute, regular. Chest sounds are clear. Abdomen is
increased in size, blown more in the lower region. Bowel sounds absent; there is fluid collection in the intestine.
Abdomen is tender, more on the right side, where painful mass is palpable. Signs of rebound tenderness are
present. Which test can assist in defining preliminary diagnosis?
A. Angiography
B. Barium follow throw test
C. Plain abdominal X-ray
D. Colonoscopy
E. Laparoscopy

1.200 36 years old female was ill for 16 hours. She came to the Emergency Department complaining of flatulence,
constant abdominal pain, vomiting, retention of feces and gas. Such abdominal pain attack has occurred for the
first time. Medical history consists from surgery due to ectopic pregnancy few years ago. She lifted heavy thing
before this attack and then griping pain started. She vomited several times. On examination: tongue is coated.
Blood pressure - 140/80 mm Hg, puls rate is 96 beats per minute, regular. Chest sounds are clear. Abdomen is
increased in size, blown more in the lower region. Bowel sounds absent; there is fluid collection in the intestine.
Abdomen is tender, more on the right side, where painful mass is palpable. Signs of rebound tenderness are
present. What laboratory findings of this woman you do not expect?
A. Hyperkalemia
B. Hypoalbuminemia
C. Leucocytosis
D. Hypernatremia
E. Moderate uremia

1.201 36 years old female was ill for 16 hours. She came to the Emergency Department complaining of flatulence,
constant abdominal pain, vomiting, retention of feces and gas. Such abdominal pain attack has occurred for the
first time. Medical history consists from surgery due to ectopic pregnancy few years ago. She lifted heavy thing
before this attack and then griping pain started. She vomited several times. On examination: tongue is coated.
Blood pressure - 140/80 mm Hg, puls rate is 96 beats per minute, regular. Chest sounds are clear. Abdomen is
increased in size, blown more in the lower region. Bowel sounds absent; there is fluid collection in the intestine.
Abdomen is tender, more on the right side, where painful mass is palpable. Signs of rebound tenderness are
present. What method of treatment will you suggest?
A. Active observation of the patient and conservative treatment
B. Preparation for possible operation, management of homeostasis, continuing investigations; when the
diagnosis is clear - operation
C. Immediate preparation for operation and operation
D. Colonoscopy and restoration of the intestinal lumen, if it fails - operation
E. Call for senior consultant

1.202 58 years old male presented in clinic with complains on inconstant flatulence, constipations, anorexia,
unintentional weight loss. He felt sick for 3 months. He couldn't open his bowels for last two days, and felt constant
abdominal pain during that time. For these reasons he decided to apply to a doctor. The patient haven`t had
previous surgery. Peptic ulcer disease was diagnosed a while ago. On examination: overweight male (BMI-42),
tongue is coated, wet, the abdomen is increased in size, bloated, soft. Bowel sounds active; no signs of fluids
collection in the intestine. Suspition of distended colon on palpation. No palpable mass in the abdomen. Per rectal
examination - the ampule of the rectum is empty. Body temperature is normal. What disease will you suspect?
A. Large bower obstruction
B. Diverculitis
C. Volvulus
D. Intussusception
E. Crohn's disease

1.203 58 years old male presented in clinic with complains on inconstant flatulence, constipations, anorexia,
unintentional weight loss. He felt sick for 3 months. He couldn't open his bowels for last two days, and felt constant
abdominal pain during that time. For these reasons he decided to apply to a doctor. The patient haven`t had
lsmusis.lsmuni.lt/Klausimai/Spausdinti?Length=0?Kalba=EN&KategorijaId=103&Kalbos_input=EN&Kalbos=EN&KategorijaEn_input=Surgery&Kate… 25/29
3/27/2019 LSMUSIS
previous surgery. Peptic ulcer disease was diagnosed a while ago. On examination: overweight male (BMI-42),
tongue is coated, wet, the abdomen is increased in size, bloated, soft. Bowel sounds active; no signs of fluids
collection in the intestine. Suspition of distended colon on palpation. No palpable mass in the abdomen. Per rectal
examination - the ampule of the rectum is empty. Body temperature is normal. What investigations will you
suggest?
A. CT scan
B. Colonoscopy and/or barium enema
C. Laparascopy
D. Barium follow throw test
E. Ultrasound scan of the abdomen

1.204 58 years old male presented in clinic with complains on inconstant flatulence, constipations, anorexia,
unintentional weight loss. He felt sick for 3 months. He couldn't open his bowels for last two days, and felt constant
abdominal pain during that time. For these reasons he decided to apply to a doctor. The patient haven`t had
previous surgery. Peptic ulcer disease was diagnosed a while ago. On examination: overweight male (BMI-42),
tongue is coated, wet, the abdomen is increased in size, bloated, soft. Bowel sounds active; no signs of fluids
collection in the intestine. Suspition of distended colon on palpation. No palpable mass in the abdomen. Per rectal
examination - the ampule of the rectum is empty. Body temperature is normal. What laboratory findings could be
expected?
A. Leucocytosis, dehidration, hypokalemia
B. Dehidration, hypokalemia, hypoalbuminemia
C. Anemia, leucocytosis, hyperkalemia, uremia
D. Dehidration, hyponatremia, hypoalbuminemia
E. The findings will be in a normal range

1.205 58 years old male presented in clinic with complains on inconstant flatulence, constipations, anorexia,
unintentional weight loss. He felt sick for 3 months. He couldn't open his bowels for last two days, and felt constant
abdominal pain during that time. For these reasons he decided to apply to a doctor. The patient haven`t had
previous surgery. Peptic ulcer disease was diagnosed a while ago. On examination: overweight male (BMI-42),
tongue is coated, wet, the abdomen is increased in size, bloated, soft. Bowel sounds active; no signs of fluids
collection in the intestine. Suspition of distended colon on palpation. No palpable mass in the abdomen. Per rectal
examination - the ampule of the rectum is empty. Body temperature is normal. What tactics will you choose?
A. Immediate laparotomy
B. Antibiotics, if peritonitis occurs - operation
C. Laparoscopic bowel resection with anastomosis
D. Laparoscopic untwisting and fixation
E. Endoscopic restoration of the lumen of the colon, planned bowel resection

1.206 18 year old male undervent urgent surgery for perforated duodenal ulcer. A laparoscopic closure of the ulcer
was performed. On postoperative day 7 flatulence occurred. There was griping pain in the abdomen, nausea.
Nasogastric tube was inserted and 1000ml of green gastric content was evacuated. Body temperature remained
normal. On examination: tongue is coated. Abdomen is bloated, peristalsis sluggish. There is fluids collection in
the intestine. Abdomen is soft on palpation. Signs of rebound tenderness are absent. A plain abdominal X-ray
have revealed an increased amount of air in the stomach, gas - fluid surfaces, small amount of gas in the colon.
What postoperative complication you will suspect?
A. Pneumonia
B. Suture leakage
C. Intestinal obstruction
D. Peritoneal abscess
E. Duodenal stenosis

1.207 18 year old male undervent urgent surgery for perforated duodenal ulcer. A laparoscopic closure of the ulcer
was performed. On postoperative day 7 flatulence occurred. There was griping pain in the abdomen, nausea.
Nasogastric tube was inserted and 1000ml of green gastric content was evacuated. Body temperature remained
normal. On examination: tongue is coated. Abdomen is bloated, peristalsis sluggish. There is fluids collection in
the intestine. Abdomen is soft on palpation. Signs of rebound tenderness are absent. A plain abdominal X-ray
have revealed an increased amount of air in the stomach, gas - fluid surfaces, small amount of gas in the colon.
What are the next steps?
A. Ultrasound scan of the abdomen, drainage of the abscess under CT or ultrasound guidance
B. Gastroscopy, insertion of nasojejunal feeding tube
C. Gastrografin insertion throw the nasogastric tube into the stomach, gastrografin follow throw test
D. Repeat urgent laparoscopy
lsmusis.lsmuni.lt/Klausimai/Spausdinti?Length=0?Kalba=EN&KategorijaId=103&Kalbos_input=EN&Kalbos=EN&KategorijaEn_input=Surgery&Kate… 26/29
3/27/2019 LSMUSIS
E. Chest plain X-ray

1.208 68 year old female suffers from severe abdomen pain, which has started suddenly 6 hours ago after heavy
lifting. She has cardiac arrhythmia and uses painkillers due to backache for several years. She has had an
episode of pneumonia recently as well. On examination: general state is satisfactory. Pulse rate - 100 beats per
minute, arrhytmia present. Blood pressure - 130/80 mm Hg. Abdomen is increased in size because of obesity and
tender. Muscular tension and the signs of rebound tenderness are absent. There is soft, painless, relatively
immobile tumour in the epigastrium, 2X2 cm in size. Peristalsis absent. On auscultation - the breathing sounds are
slightly diminished. What disease you will suspect?
A. Incarcerated epigastrial hernia
B. Renal colic
C. Acute mesenteric ischaemia
D. Severe chest infection

1.209 68 year old female suffers from severe abdomen pain, which has started suddenly 6 hours ago after heavy
lifting. She has cardiac arrhythmia and uses painkillers due to backache for several years. She has had an
episode of pneumonia recently as well. On examination: general state is satisfactory. Pulse rate - 100 beats per
minute, arrhytmia present. Blood pressure - 130/80 mm Hg. Abdomen is increased in size because of obesity and
tender. Muscular tension and the signs of rebound tenderness are absent. There is soft, painless, relatively
immobile tumour in the epigastrium, 2X2 cm in size. Peristalsis absent. On auscultation - the breathing sounds are
slightly diminished.
A. Laparoscopy
B. Plain chest and abdominal X-ray
C. Cystoscopy
D. CT abdomen (contrast) or angiography

1.210 32 year old male had the traffic accident 4 days ago. 9-10 ribs in the posterior axillary line were broken and
left thigh was compressed at the time of the accident. The man had troublesome cough and increasing pain on the
left side of the chest at the time of coughing during these days, though general state remained stable. One hour
ago weakness have occured suddenly, he collapsed, lost consciousness for few seconds. Weakness still present.
On examination: body temperature 37.5, pulse rate 100 beats per minute, blood pressure 110/65 mm Hg. There is
hematoma on the external surface of the left thigh, 12X10 cm in size. The breathing is superficial, 22 times per
minute, there is significant pain in the projection of 9-10 ribs. The abdomen is not blown, soft, tender in left upper
quadrant. Peristalsis present. Blood test Hb 100 g/l, WBC 13.2, otherwise normal. Your preliminary diagnosis?
A. Right pneumothorax
B. Posthaemorrhagic anaemia because of thigh hematoma
C. Rupture of the spleen
D. Posttraumatic chest infection

1.211 32 year old male had the traffic accident 4 days ago. 9-10 ribs in the posterior axillary line were broken and
left thigh was compressed at the time of the accident. The man had troublesome cough and increasing pain on the
left side of the chest at the time of coughing during these days, though general state remained stable. One hour
ago weakness have occured suddenly, he collapsed, lost consciousness for few seconds. Weakness still present.
On examination: body temperature 37.5, pulse rate 100 beats per minute, blood pressure 110/65 mm Hg. There is
hematoma on the external surface of the left thigh, 12X10 cm in size. The breathing is superficial, 22 times per
minute, there is significant pain in the projection of 9-10 ribs. The abdomen is not blown, soft, tender in left upper
quadrant. Peristalsis present. Blood test Hb 100 g/l, WBC 13.2, otherwise normal. What investigation must be
done?
A. Plain chest and abdominal X-ray
B. X-ray of the right thigh
C. Blood test including CRP
D. Ultrasound of the peritoneal cavity

1.212 You have visited the patient at home. 74 year old male has right inguinal hernia of a fist in size for some
years. He underwent course of a conservative treatment for gastric ulcer last year. Sudden episode of pain started
in the abdomen, hernia became incarcerated and tender after coughing 3 hours ago. The patient felt sick, vomited
once with dark content. Half an hour ago the pain settled and disappeared. On examination: body temperature
37.2 degrees of Celsius, pulse rate 86 beats per minute. The abdomen is soft, slightly tender around the umbilicus
and in the rihgt iliac fossa region. The inguinal hernia reducible, the external ring of inguinal canal is enlarged,
painful on palpation. Bowel sounds weak. Your preliminary diagnosis?
A. Perforated and covered gastric ulcer
B. Bleeding gastric ulcer
lsmusis.lsmuni.lt/Klausimai/Spausdinti?Length=0?Kalba=EN&KategorijaId=103&Kalbos_input=EN&Kalbos=EN&KategorijaEn_input=Surgery&Kate… 27/29
3/27/2019 LSMUSIS
C. Incarcerated inguinal hernia
D. Restored indirect inguinal hernia

1.213 You have visited the patient at home. 74 year old male has right inguinal hernia of a fist in size for some
years. He underwent course of a conservative treatment for gastric ulcer last year. Sudden episode of pain started
in the abdomen, hernia became incarcerated and tender after coughing 3 hours ago. The patient felt sick, vomited
once with dark content. Half an hour ago the pain settled and disappeared. On examination: body temperature
37.2 degrees of Celsius, pulse rate 86 beats per minute. The abdomen is soft, slightly tender around the umbilicus
and in the rihgt iliac fossa region. The inguinal hernia reducible, the external ring of inguinal canal is enlarged,
painful on palpation. Bowel sounds weak. Your decision?
A. Send the patient for gastroscopy immediately
B. Recommend ellective hernia repair
C. Send the patient to Emergency Department for admission to the hospital
D. Recommend to call Ambulance, if the pain comes again

1.1 - C 1.2 - D 1.3 - B 1.4 - C 1.5 - C 1.6 - C 1.7 - B 1.8 - C


1.9 - A 1.10 - A 1.11 - D 1.12 - C 1.13 - B 1.14 - C 1.15 - E 1.16 - D
1.18 - B 1.19 - B 1.20 - D 1.21 - B 1.22 - C 1.23 - C 1.24 - A 1.25 - C
1.26 - B 1.27 - C 1.28 - C 1.29 - D 1.30 - D 1.31 - A 1.32 - A 1.33 - B
1.34 - A 1.35 - D 1.36 - D 1.37 - B 1.38 - E 1.39 - C 1.40 - A 1.41 - A
1.42 - B 1.43 - D 1.44 - C 1.45 - C 1.46 - E 1.47 - B 1.48 - C 1.49 - B
1.50 - B 1.51 - A 1.52 - B 1.54 - D 1.55 - B 1.56 - C 1.57 - D 1.58 - B
1.60 - C 1.61 - B 1.62 - B 1.63 - A 1.64 - A 1.65 - B 1.66 - D 1.67 - C
1.68 - B 1.69 - B 1.70 - A 1.71 - B 1.72 - D 1.73 - C 1.74 - A 1.75 - A
1.76 - D 1.77 - C 1.78 - E 1.79 - B 1.80 - C 1.81 - C 1.82 - E 1.83 - A
1.84 - E 1.85 - A 1.86 - B 1.87 - C 1.88 - D 1.89 - A 1.90 - A 1.91 - D
1.92 - D 1.93 - C 1.94 - C 1.95 - A 1.96 - B 1.97 - B 1.98 - B 1.99 - B
1.100 - A 1.101 - A 1.185 - C 1.186 - D 1.187 - A 1.188 - B 1.189 - D
1.102 1.103 1.104 1.105 1.106 1.107 1.108 1.109
1-C 1-B 1-E 1-C 1-B 1-C 1-E 1-D
2-A 2-D 2-B 2-B 2-A 2-B 2-D 2-E
3-D 3-A 3-A 3-D 3-E 3-D 3-B 3-A
4-B 4-E 4-D 4-A 4-B 4-E 4-A 4-C
5-E 5-C 5-E 5-E 5-D 5-A 5-C 5-B
1.110 1.111 1.112 1.114 1.115 1.116 1.117 1.118
1-B 1-D 1-B 1-B 1-C 1-C 1-C 1-C
2-C 2-A 2-A 2-C 2-D 2-A 2-B 2-A
3-A 3-B 3-C 3-D 3-B 3-B 3-A 3-E
4-D 4-C 4-C 4-A 4-A 4-D 4-E 4-D
5-D 5-B
1.119 1.120 1.121 1.122 1.123 1.124 1.190 1.191
1-E 1-B 1-E 1-B 1-E 1-B 1-E 1-D
2-D 2-C 2-D 2-E 2-C 2-E 2-D 2-A
3-A 3-A 3-B 3-C 3-D 3-A 3-C 3-E
4-B 4-E 4-C 4-D 4-A 4-D 4-B 4-C
5-C 5-D 5-A 5-A 5-B 5-C 5-A 5-B
1.192
1-C
2-A
3-D
4-B
1.125 1.126 1.127 1.128 1.129 1.130 1.131 1.132
1 2 1 1 1 1 4 1
2 4 2 2 2 2 2
3 3 3 3 3 3
4 4 4 4 4
5 5 5
lsmusis.lsmuni.lt/Klausimai/Spausdinti?Length=0?Kalba=EN&KategorijaId=103&Kalbos_input=EN&Kalbos=EN&KategorijaEn_input=Surgery&Kate… 28/29
3/27/2019 LSMUSIS
1.133 1.134 1.135 1.136 1.137 1.138 1.139 1.140
1 1 1 1 2 1 4 2
2 2 3 2 4 2 4
3 3 3 3
4
5
1.141 1.142 1.143 1.144 1.145 1.146 1.148 1.149
1 4 1 1 4 1 4 2
2 3 2 2 4
3 3 3
4
5
1.150 1.151 1.152 1.153 1.154 1.155 1.156 1.157
1 1 2 1 4 1 1 1
2 3 4 2 2 3 2
3 3 3 3
4
5
1.158 1.159 1.160 1.161 1.162 1.163 1.164 1.193
1 2 2 4 1 1 1 2
2 4 4 2 2 2 4
3 3 3 3
4 4
1.194 1.195 1.196 1.197
1 1 4 4
2 3
3
1.165 - C 1.166 - D 1.167 - C 1.168 - C 1.169 - C 1.170 - C 1.171 - B 1.172 - A
1.173 - A 1.174 - C 1.175 - A 1.176 - B 1.177 - D 1.178 - C 1.179 - D 1.180 - C
1.181 - D 1.182 - E 1.183 - D 1.184 - B 1.198 - C 1.199 - C 1.200 - B 1.201 - C
1.202 - A 1.203 - B 1.204 - B 1.205 - E 1.206 - C 1.207 - C 1.208 - C 1.209 - D
1.210 - C 1.211 - D 1.212 - D 1.213 - C

lsmusis.lsmuni.lt/Klausimai/Spausdinti?Length=0?Kalba=EN&KategorijaId=103&Kalbos_input=EN&Kalbos=EN&KategorijaEn_input=Surgery&Kate… 29/29
3/27/2019 LSMUSIS

General surgery
I type tasks. Choose only one best answer

14.1 Not used for the sterilization of surgical instruments:


A. Ethylene oxide
B. Gamma ionizing radiation
C. 2% glutaraldehide
D. Autoclave

14.2 Which solution should be used for the surgical wound washing:
A. 0,05 % spirituous chlordexidine solution
B. 20 % chlordexidine bigluconate water solution
C. S4 solution
D. 0.9 % NaCl solution

14.3 Which temperature kills HIV and infectious hepatitis virus:


A. 40oC
B. 56oC
C. 100oC
D. Above 100oC

14.4 Characteristic changes in the organism during bleeding:


A. Phlebectasia (Vein dilatation)
B. Widespread arteriole dilatation
C. Spleen contraction
D. A decrease in the blood volume released from the heart
E. Decreased aldosterone secretion

14.5 To the 1st degree hypovolemic shock is characteristic:


A. A. It develops upon losing over 40% of circulating blood volume
B. B. Tachycardia
C. C. Decrease in pulse pressure is recorded
D. The amount of urine significantly decreases
E. Slowed down filling of capillaries

14.6 Which is the main purpose of pre-operative patient’s preparation:


A. The establishment of diagnosis
B. The determination of indications for the surgery as well as its time and extend
C. The selection of the most appropriate manner of anesthesia
D. Psychological patient’s preparation for a surgery
E. The minimization of surgical risk as well as the possibility of post-operative complications

14.7 Which factor determines surgical (operation) risk:


A. General patient’s state, method of anesthesia and surgical trauma
B. Insufficient patient’s preparation for the surgery at the presence of vital indications
C. Patient’s age, character of disease, operation and adjacent diseases
D. The functional state of vital organs (heart, lungs, kidneys, liver)
E. Haemostatic disorders

14.8 To local anesthesia is characteristic:


A. Local anesthetics primarily influence larger nervous fibers
B. Adrenaline (epinephrine) accelerates the resorbtion of the anesthetic and extends the time of its effect
C. The toxic effect of local anesthetics most heavily manifests itself in the cardiovascular and central
nervous systems
D. When fingers are operated on its safer to use the mixture of anasthetic and adrenaline
E. Lidocaine has longer anaesthetical duration than bupivicaine

14.9 Which test has greatest importance on the reduction of surgical risk in pre-operative period:
A. ECG
B. Fluorogram of chest organs
C. Extensive anamnesis

lsmusis.lsmuni.lt/Klausimai/Spausdinti?Length=0?Kalba=EN&KategorijaId=116&Kalbos_input=EN&Kalbos=EN&KategorijaEn_input=General+surger… 1/10
3/27/2019 LSMUSIS
D. Patient’s inspection, palpation, percussion and auscultation
E. Biochemical blood test

14.10 When an abscess must be operated on:


A. When pain is felt
B. When pain, swelling, redness of the skin and the febrile state occur
C. Only in case of fluctuation
D. Upon abscess rupture and purulence discharge

14.11 Which tests are the most important for the diagnosis of superfitial abscess:
A. General laboratory tests
B. Bacteriological tests
C. Physical tests
D. Anamnesis
E. X-ray and other special tests

14.12 In which period does mastitis most often develop:


A. Childhood
B. Pregnancy
C. Post-delivery period
D. Climacteric period

14.13 Which is the proper method of treatment of mastitis after incision:


A. Focus drainage via contraperture and primary disecated suture
B. Elastic tamponade of the focus cavity
C. Rinsing of the focus cavity with antiseptics and drainage
D. Tight suturation of the wound

14.14 What does predetermine intoxication in case of tetanus:


A. Putrific infection (putrifying bacteria)
B. Staphylococcus
C. Clostridical toxins
D. Acid products of metabolism

14.15 Which drug is used for a passive prevention of tetanus:


A. Cephalasporines
B. Sulphanilamides
C. Tetanus toxoid
D. Anti tetanus immunoglobuline

14.16 What does predetermine the development of anaerobic microorganisms:


A. Necrotic tissues
B. The decreasing of immunity
C. Tissue acidosis
D. Aerobic infection
E. All above listed factors

14.17 What causes anaerobic infection:


A. Escherichia coli
B. Proteus sp.
C. Klebsiella sp.
D. Bacteroides fragilis

14.18 Which symptom of sepsis is the most important:


A. Primary focus
B. Tachycardia over 100 k/min.
C. Splenomegaly
D. Rising temperature over 38oC
E. All above listed symptoms

14.19 Patient’s leg suddenly became cold, white, started heavily aching and a pulse has disappeared. What is the
disease diagnosis:
A. Neuritis
lsmusis.lsmuni.lt/Klausimai/Spausdinti?Length=0?Kalba=EN&KategorijaId=116&Kalbos_input=EN&Kalbos=EN&KategorijaEn_input=General+surger… 2/10
3/27/2019 LSMUSIS
B. Vein thrombosis
C. The inflammation of soft tissues of the leg
D. Acute occlusion of arteries

14.20 A middle-aged woman of good nutrition with the leg vein varicosis found a pigmented spot with tissue
defects in the center on a median surface of her left calf. What is the disease diagnosis:
A. Erysipelas
B. Diabetic foot
C. Osteomyelitis
D. Varicose trophic ulcer
E. Eczema

14.21 The healing process of an open tibia fracture was very slow. After 2 months the temperature rose, the place
of fracture turned red and started aching. In addition, there opened a cavity and purulent secret starting going out.
What is the preliminary diagnosis of the disease:
A. Thrombophlebitis
B. Abscess
C. Trophic sore
D. Osteomyelitic fistula

14.22 An elderly patient has a chronic disease, is continuously staying in bed, does not move much and
sometimes suffers from bedwetting. In sacroiliac area a palm-sized red spot with still darker center has appeared.
What is the diagnosis of the disease complication:
A. Post-injection medical abscess
B. A forming-up bedsore
C. Phlegmona
D. Paraproctitis
E. Eczema
F. Dermatitis

14.23 Which case is indicative for primary surgical management of the wound:
A. Clean wound
B. Contaminated wound
C. Infected wound
D. Suppurative wound

14.24 Which drainage method, is the best in aseptic and antiseptic aspect for the management of surgical
infection:
A. Passive closed
B. Passive open
C. Active open
D. Active closed
E. Prophylactic

14.25 Which suturing material can cause greatest inflammatory reaction:


A. Prolen
B. Silk
C. Ketgut
D. Vicryl

14.26 A patient, without any obvious cause runs a fever, is sweating and weakening. What is the preliminary
diagnosis of the disease:
A. Typhus or other infectious disease
B. Pneumonia or tonsillitis
C. Erysipelas
D. Deep abscess

14.27 Which abscess is healed by in closed method:


A. Anaerobic phlegmon
B. Post-injectional gluteal abscess
C. Felon
D. Osteomyelitis

lsmusis.lsmuni.lt/Klausimai/Spausdinti?Length=0?Kalba=EN&KategorijaId=116&Kalbos_input=EN&Kalbos=EN&KategorijaEn_input=General+surger… 3/10
3/27/2019 LSMUSIS
14.28 The consequence of which disease is axillary lymphadenitis:
A. Hand and finder fester
B. Sepsis
C. Hydradenitis
D. Blood disease

14.29 Dorsal surface of the finger and hand have been swollen. What is the preliminary diagnosis of the disease:
A. The fester of the dorsal surface of the finger and hand
B. Thrombophlebitis of the finger and hand
C. The fester of the palmar surface of finger and hand
D. Allergy

14.30 Which part of clothing of the surgeon during operation must be sterile:
A. A mask
B. Underwear
C. A cap
D. A gown
E. An apron

14.31 In which method producers sterilized surgical sutures:


A. By autoclave
B. By gamma rays
C. By formalin steam
D. By ethyl oxide
E. By ethyl spirit

14.32 Which action depends on mechanical antisepsis:


A. Wound rinsing with 0,9 % NaCl solution
B. Instrument washing with a brush
C. Primary surgical management of the wound
D. Floor washing in an operation-room
E. Wound washing with hydrogen peroxide

14.33 Which action belongs to physical antiseptics:


A. Ultraviolet irradiation of an operation-room
B. Renewal of wound edges
C. Instrument sterilization with hot air
D. Physiotherapic procedures in the wound area
E. Instrument sterilization with hot steam under increased pressure

14.34 In how many days from the disease onset is it possible to detect roentgenologic symptoms of osteomyelitis:
A. After 10-14 days
B. After one day
C. After 30 days
D. After 8-9 days

14.35 What does cause haematogenous osteomyelitis:


A. Egzogenic infection
B. Super infection
C. Egzogenic and endogenic infection
D. Endogenic infection

14.36 Which purulent disease should be cured with hyperbaric oxygenation:


A. Erysipelas
B. Hydradenitis
C. The furunculosis of entire body
D. Anaerobic phlegmona

14.63 A patient runs a fever, and often and painfully evacuates (tenesmes). What is the disease diagnosis:
A. Dysentery
B. Hemorrhoids
C. Paraproctitis

lsmusis.lsmuni.lt/Klausimai/Spausdinti?Length=0?Kalba=EN&KategorijaId=116&Kalbos_input=EN&Kalbos=EN&KategorijaEn_input=General+surger… 4/10
3/27/2019 LSMUSIS
D. Rectal cancer

II type tasks. For each numbered item,selct the one lettered heading that is most closely asssciated with it

14.37 When does manifest itself post-operational fever caused by below enumerated:
1 - Wound infection
2 - Atelectasis, increased metabolism, endocrinic changes, transfusive reactions, infection dissemination
3 - Lung complications
4 - Thrombophlebitis, urinary infection
5 - Abscess in peritoneum cavity
A. On the 3rd day and later
B. Immediately after the operation
C. After 36 hours
D. On the 5th-7th day
E. After 3 days

14.38 To which disease is characteristic each of below symptoms:


1 - Crepitation
2 - Rigidity of cervical muscles (opistotonus)
3 - Local pain sensation
4 - Splenomegaly
A. Tetanus
B. Sepsis
C. Gaseous gangrene
D. Osteomyelitis

14.40 Which type of operation should be chosen in case of each disease below:
1 - Acute appendicitis
2 - Free inguinal hernia
3 - Traumatic spleen rupture
4 - Acute suppurative mastitis
5 - Gastric cancer
6 - Varicosis of superfitial leg veins
7 - Decompensated phylorostenosis
A. Extra operation
B. Urgent operation
C. Elective operation

14.64 Which patient examination method is the most suitable in each of below indicated pathological cases:
1 - Bleeding from varicose veins of oesophagus
2 - Perforated gastric ulcer
3 - Mechanical intestinal obstruction
4 - Traumatic rupture of urinary bladder
5 - Abscess of off excavatio.rectovesicalis (rectouterina)
7 - Traumatic rupture of the spleen
A. Plain abdominal X-ray
B. Fibroesophagogastroduodenoscopy
C. Urine test
D. Blood test
E. Echoscopy
F. Rectum palpation
G. Urinary bladder catheterization
H. Laparoscopy

III type tasks. For each question there is one or more correct answers:
A – if correct answers are 1,2,3
B – if correct answers are 1 and 3
C – if correct answers are 2 and 4
D – if correct answer is 4
E – if correct are all answers above

lsmusis.lsmuni.lt/Klausimai/Spausdinti?Length=0?Kalba=EN&KategorijaId=116&Kalbos_input=EN&Kalbos=EN&KategorijaEn_input=General+surger… 5/10
3/27/2019 LSMUSIS
14.41 Ultrasound examination is to be used:
1. To diagnose breast tumor in women under 35 years
2. To determine the causes of pain in the upper abdominal quadrant in women above 45 years
3. When suspecting an under-diaphragm abscess in post-operational period
4. To detect possible bone metastases in men above 70 years

14.42 Which symptoms facilitate the timely diagnosis of many post-operative complications:
1. Patient’s fever on the 3rd-6th day after operation can be caused by urinary infection
2. The most frequent cause of patients’ fever in the first days after operation is pulmonary complications
3. The infection of post-operative wound can cause fever on the 5-8th day after the operation
4. The most effective way to prevent thromboembolic complications in post-operational period – patient’s
enema on the 2nd-3rd day after operation
5. On the 3rd day after operation the organism accumulates sequestrated liquids and, therefore, it’s
necessary to adjust the water balance – to additionally transfuse haemo-correctors

14.43 To which disease is related the greatest surgical risk when planning large scale operations on elderly
people:
1. Bronchial asthma
2. Lung cancer
3. Pneumosclerosis
4. Chronic obstructive bronchitis
5. Prostatic adenoma

14.44 Which symptoms are characteristic to tendinous felon:


1. Redness of the finger
2. Finger contracture (painful)
3. Finger swelling
4. Pulsatory finger pain
5. Serous-purulent abscesses of epidermis

14.45 Which microorganisms most frequently cause hypodermic abscesses:


1. Staphylococci
2. Pseudomona aeroginosa
3. Mixed flora
4. Anaerobs
5. Pneumococci and gonococci

14.46 How should be treated fluctuating hypodermic abscess:


1. By immediate incision
2. By the immobilization of the affected part and application of warm compresses
3. By puncture, drainage with troacar, washing with antiseptics via drains
4. Treatment with antibiotics i/m, i/v, into artery and endolymphaticaly

14.47 How are characterized vazotrophic ulcers:


1. These are various ulcers caused by chronic venous failure
2. These are various ulcers caused by chronic arterial failure
3. Ulcers having occurred because of lymphatic circulation disorders
4. Ulcers having occurred because of peripheral nervous system disorders
5. Sores caused by various dermal diseases

14.48 Which disorders are characteristic to a diabetic foot:


1. Macroangiopathy
2. Microangiopathy
3. Neuropathy
4. Soft tissue inflammations
5. Foot bone osteomyelitis

14.49 Which symptoms are characteristic to a congenital fistula:


1. Secretion is continuously or with intervals running out of the fistula
2. It’s possible to diagnose fistula immediately after birth or somewhat later
3. The duct of fistula is covered with epithelium
4. The fistula walls are covered with granulation

lsmusis.lsmuni.lt/Klausimai/Spausdinti?Length=0?Kalba=EN&KategorijaId=116&Kalbos_input=EN&Kalbos=EN&KategorijaEn_input=General+surger… 6/10
3/27/2019 LSMUSIS
5. Fistula forms up because of some diseases

IV type tasks. Choose only one best answer

14.50 A surgeon forgot to bring his glasses, went out of the operation-room into the corridor of surgical department
for a while, where to his glasses were brought and put onto his face without any contact with his sterile clothing.
What should the surgeon do upon returning to the operation-room:
A. Change his gown, gloves, footwear protection (bakhiles), mask and cap
B. Change only sterile clothing
C. Change only footwear protection (bakhiles)
D. Prepare his hands anew, change the gown, gloves, footwear protection (bakhiles), mask and cap

14.51 An incidental person stabbed the patient with a knife. A small, slightly bleeding wound has emerged on the
right of the umbilicus. The patient’s heart is beating heavily, he is in a cold sweat, thirsty and complains of
weakness. Which phenomenon is developing:
A. Peritoneum inflammation (peritonitis) after the injury of hollow organ
B. Traumatic shock after the injury
C. Acute anemia caused by internal bleeding
D. The reaction of nervous system because of fright

14.52 A 43-year-old patient was hospitalized after 20 hours from the onset of a disease. Upon arrival he
complained of the swelling of his right leg and pain in his calf, especially when walking. It appeared that 26 hours
ago he had fallen down from a 3-meter height and stubbed his right calf. Object. check-up: a 16x15 cm
hypodermic bruising on the back surface of the right calf and small bruisings around medial and lateral ankles are
located, while the right hip is by 8 cm thicker than the left one, while the calf by 9 cm thicker than the left one, the
calf muscles are very painful, the skin above them is tensed and shining, and, a. femoralis and a. poplitea pulse is
normal, T- 37,8oC, the patient is pale, his face is sweaty. Lab. tests: blood - Hb-86 g/l, leuc.-10,7.109/l, urea-16
mmol/l, in urine test - leucocyte 3-5 r.l., erythrocite - 4,5.1012/l, hial. cylinder. 3-4 r.l., X ray does not show bone
fractures, after performing the puncture of the thigh bruising, a small amount of serohaemorrhagic discharge was
obtained, and serous discharge from the place of the puncture is heavy. What is the diagnosis of this disease:
A. Deep phlebitis
B. Haematoma of soft tissues of the thigh
C. Crush syndrome
D. Traumatic v. femoralis rupture
E. Phlegmona of the thigh

14.53 There are several painful red-bluish infiltrates in woman’s right armpit, two of which are fluctuating, while the
third – has ruptured and discharges liquid purulence. What is the diagnosis of this disease:
A. Abscess
B. Lymphadenite
C. Thrombophlebitis
D. Hydradenitis
E. Cancerous metastases (breast)

14.54 A 42-year old woman has a 4x5 cm diameter painful infiltrate and a fistula in the upper lateral quadrant of
her right breast which produce sparse purulent discharge. The skin above the infiltrate is reddened and hot. What
is the diagnosis of this disease:
A. Purulent mastitis
B. Breast actinomycosis
C. Dermal tuberculosis
D. Hypodermic abscess
E. Complicated, decaying breast cancer

14.55 When playing in a yard, a schoolboy strained leg ligaments. In the evening the leg got swollen and very
painful, the inguinal lymphatic node has enlarged, the patient started running high fever and suddenly the leg
function got disturbed. The greatest pain was felt in the medial one third of the calf. What is preliminary diagnosis
of this disease:
A. Bone fracture
B. Ligament strain
C. Tetanus
D. Osteomyelitis

lsmusis.lsmuni.lt/Klausimai/Spausdinti?Length=0?Kalba=EN&KategorijaId=116&Kalbos_input=EN&Kalbos=EN&KategorijaEn_input=General+surger… 7/10
3/27/2019 LSMUSIS
14.56 When playing basketball one player had an open fracture of tibias. The wound was bandaged and the
patient taken to hospital, where reposition was made, the wound was sutured and the calf was put in plaster. After
two days, the patient started feeling great pain in the traumatized leg, ABP decreased, tachycardia has developed
and the patient became unquiet. The antibacterial and detoxicating treatment was started, however his state has
not improved. Because of this the plastic bandage was removed. The leg was heavily swollen but there was no
hyperemia around the wound and no purulent discharge. The patient’s temperature rose to 37ºC. A surgeon once
again dressed the wound, put the leg in plaster and increased antibacterial treatment. The patient’s state rapidly
worsened and after one day from the state worsening the patient died. What is the diagnosis of this disease:
A. Sepsis
B. Tetanus
C. Too heavy compression of the leg with plastic bandage
D. Gaseous gangrene
E. Haematogenic osteomyelitis
F. Osseal tuberculosis

14.57 Several furuncles emerged on a schoolgirl’s face. Before going to the school the girl squeezed out the
purulence and put on some cream. While being at school, the girl’s face got swollen. Upon returning home, the
patient’s temperature rose to 40ºC, tachycardia and tachypnoe have emerged and she started feeling cold. The
girl was taken to hospital, where upon detecting leucocytosis the antibacterial treatment was started. Since her
condition has not improved and the signs of intoxication have intensified, splenomegaly, hepatomegaly, subicterus
and anemia have developed, and related medical consultation was held. What is the conclusion of this medical
consultation:
A. Meningitis
B. Hepatitis C
C. Osteomyelitis
D. Sepsis
E. Gaseous gangrene
F. Tetanus

14.58 In the place of former necrotic erysipelas in the patient’s calf there emerged a black spot, whose skin is hard
and insensitive. What is the diagnosis of this disease:
A. Trophic ulcer
B. Reoccurring erysipelas
C. Dermal necrosis
D. Phlegmona
E. Thrombophlebitis

14.59 A elderly man having smoked for many years suffers from diabetes melitus, general aterosclerosis and leg
vein varicose. Presently, at nights he feels great pain in his foot, there emerged inflammatory expressions and the
toes V and IV have blackened. What is the diagnosis of this disease:
A. Diabetic foot
B. Aterosclerotic – arterial foot gangrene of occlusive origin
C. Gangrene caused by vein varicose
D. Foot phlegmona
E. Osteomyelitis of foot bones

14.60 The operation scar of the patient, who several years ago had a laparatomy, got swollen and reddened and
there emerged small cavities in it discharging turbid liquid. What is the diagnosis of this disease:
A. Post-operational hernia
B. Tissue inflammation because of infection
C. Ligature fistula is forming up
D. Phlegmona of the anterior abdominal wall
E. Eventeratio

14.61 A patient has had an insulin-dependent diabetes for 20 years and he can hardly walk. Now intermittent
lameness is developing. The distance of lameness - 150 m. Recently the toes V and IV of the left foot have
reddened and started aching. The toe tips have blackened. The body temperature rose to 38,5ºC. What is the
diagnosis of this disease:
A. Tissue nutrition disorders of arterial occlusion origin
B. Gout
C. Felon
D. Osteomyelitis
lsmusis.lsmuni.lt/Klausimai/Spausdinti?Length=0?Kalba=EN&KategorijaId=116&Kalbos_input=EN&Kalbos=EN&KategorijaEn_input=General+surger… 8/10
3/27/2019 LSMUSIS
E. Diabetic foot

14.62 A year ago, a young patient had paraproctitis which was successfully removed during surgery. Presently,
every several weeks, a painful nut-sized formation forms up around the anus in the place of former operational
scar. Then the body temperature rises to 39oC. When the formation ruptures and bloody liquid discharges from it
the patient’s state improves. What is the diagnosis of this disease
A. Gluteal abscess
B. Anorectal fistula
C. Hemorrhoids
D. Rectum prolapse
E. Tumor
F. Paraproctitis

14.65 In a 66-year-old patient the ascites of cardiac origin is progressing (12 years ago she had myocardial
infarction which preconditioned the development of chronic auricular fibrillation). In the area of former umbilical
hernia the skin got perforated and ascites started running. After unsuccessful attempts to suture the skin defect,
and after related consultations, the patient was operated on (local novocaine anesthesia), with the aim to make
hernioplastics. A skin patch together with the umbilicus were removed and around 3 liters of ascites were
released. When performing the abdominal wall plastics the patient got in clod sweat, she felt giddiness and big
weakness. Objective analysis: tachycardia (pulse 108-112 k/min.), At the beginning arterial blood pressure jumped
to 140/85 mmHg, while afterwards started falling and decreased to 80/50 mmHg, ECG-clear tachycardia and
auricular fibrillation. How to describe the patient’s state:
A. The above appearances are related to her reaction to pain because of insufficient anesthesia
B. Suddenly released ascites decreased intraabdominal pressure, and the blood that flew into abdominal
organs, decreased cerebral blood circulation
C. Novocaine-intoxication syndrome has developed
D. Ischemic stroke has developed
E. Anaphylactic shock has developed

14.66 In a 66-year-old patient the ascites of cardiac origin is progressing (12 years ago she had myocardial
infarction which preconditioned the development of chronic auricular fibrillation). In the area of former umbilical
hernia the skin got perforated and ascites started running. After unsuccessful attempts to suture the skin defect,
and after related consultations, the patient was operated on (local novocaine anesthesia), with the aim to make
hernioplastics. A skin patch together with the umbilicus were removed and around 3 liters of ascites were
released. When performing the abdominal wall plastics the patient got in clod sweat, she felt giddiness and big
weakness. Objective analysis: tachycardia (pulse 108-112 k/min.), At the beginning arterial blood pressure jumped
to 140/85 mmHg, while afterwards started falling and decreased to 80/50 mmHg, ECG-clear tachycardia and
auricular fibrillation. How should a surgeon behave in such case:
A. Terminate the surgery and take the patient to intensive care unit
B. Additionally infiltrate the tissues with novocaine not exceeding the total dose of 0,8 g
C. Change the anesthetic and continue the surgery
D. Stop the surgery, raise the foot of the operation table, make antihistaminic injection, and prescribe
cardiac, oxygen, infusion. Upon occurrence of convulsions assign barbiturates and upon breathing stoppage
start artificial lung ventilation
E. Make a promedol injection, give some oxygen to breathe in and continue operation after 15 minutes.

14.1 - C 14.2 - D 14.3 - D 14.4 - D 14.5 - B 14.6 - E 14.7 - C 14.8 - C


14.9 - C 14.10 - B 14.11 - C 14.12 - C 14.13 - A 14.14 - C 14.15 - D 14.16 - E
14.17 - D 14.18 - E 14.19 - D 14.20 - D 14.21 - D 14.22 - B 14.23 - B 14.24 - D
14.25 - B 14.26 - D 14.27 - B 14.28 - A 14.29 - C 14.30 - D 14.31 - D 14.32 - C
14.33 - D 14.34 - A 14.35 - D 14.36 - D 14.63 - C
14.37 14.38 14.40 14.64
1-A 1-C 1-A 1-B
2-B 2-A 2-C 2-B
3-C 3-D 3-A 3-A
4-E 4-B 4-A 4-G
5-D 5-B 5-F
6-C 7-E
7-B
14.41 14.42 14.43 14.44 14.45 14.46 14.47 14.48
lsmusis.lsmuni.lt/Klausimai/Spausdinti?Length=0?Kalba=EN&KategorijaId=116&Kalbos_input=EN&Kalbos=EN&KategorijaEn_input=General+surger… 9/10
3/27/2019 LSMUSIS
1 1 4 2 1 1 1 1
2 2 4 2 3 2 2
3 3 3 3 3
14.49
1
2
3
14.50 - D 14.51 - C 14.52 - C 14.53 - D 14.54 - E 14.55 - D 14.56 - D 14.57 - D
14.58 - C 14.59 - A 14.60 - C 14.61 - E 14.62 - B 14.65 - C 14.66 - D

lsmusis.lsmuni.lt/Klausimai/Spausdinti?Length=0?Kalba=EN&KategorijaId=116&Kalbos_input=EN&Kalbos=EN&KategorijaEn_input=General+surge… 10/10
3/27/2019 LSMUSIS

Cardiac surgery
I type tasks. Choose only one best answer

11.1 Which cannulation sites are used for extracorporeal circulation during combined CABG and mitral valve repair
procedure:
A. Aorta, superior and inferior caval veins.
B. Femoral artery, inferior caval vein.
C. Aorta, inferior caval vein.
D. Femoral artery, femoral vein.

11.2 Patient has 75 proc. narrowing of left main stem on coronary angiography, preserved left ventricular function
and recurrent angina pectoris. Which method of treatment do you suggest:
A. CABG procedure.
B. PTCA
C. Drug treatment
D. Observation

11.3 Narrowing of renal artery induces:


A. Proteinuria
B. Azotemia
C. Hypertension
D. Pyelonephritis

11.4 Fallot's tetrad:


A. Is a non cyanotic heart defect, consisting of pulmonary stenosis, ventricular septal defect, coarctation of
aorta and hypertrophy of left ventricle.
B. Is a cyanotic heart defect, consisting of pulmonary stenosis, ventricular septal defect, aorta dextroposition
and hypertrophy of right ventricle.
C. Is a shunting heart defect, consisting of pulmonary stenosis, atrial septal defect, hypertophy of right
ventricle and insufficiency of bicuspid valve.

11.5 Which type of heart defects (according classification of M.Abbott) aorta coarctation is:
A. No shunting heart defects.
B. Shunting non cyanotic heart defects.
C. Shunting cyanotic heart defects.

11.6 70 years old patient with severe calcified aortic stenosis, preserved left ventricle function and sinus rhythm.
Which method of treatment do you suggest?
A. Implantation of mechanical aortic valve prosthesis.
B. Implantation of aortic valve bioprosthesis
C. Aortic valve repair
D. Drug treatment

11.7 Which operation requires circulatory arrest:


A. Implantation of aortic valve prosthesis.
B. Replacement of ascending aorta.
C. Implantation two or more heart valves.
D. Replacement of aortic arch

11.8 Which complication after heart surgery might be suspected in case of increased venous pressure, decreased
arterial blood pressure and decreased diuresis:
A. Internal bleeding
B. Heart tamponade
C. Pneumothorax
D. Pneumomediastinum

11.9 Treatment of the ascending aortic dissection:


A. Drug treatment
B. Aortic endograft insertion
C. Urgent replacement of ascending aorta

11.10 Treatment in case of acute severe bicuspid insufficiency because of ruptured papillary muscle:
lsmusis.lsmuni.lt/Klausimai/Spausdinti?Length=0?Kalba=EN&KategorijaId=113&Kalbos_input=EN&Kalbos=EN&KategorijaEn_input=Cardiac+surgery… 1/6
3/27/2019 LSMUSIS
A. Drug treatment
B. Urgent repair (prosthesis) of bicuspid valve.
C. Repair (prosthesis) of bicuspid valve after 2 months

11.40 Balloon of the intra-aortic balloon pump introduced into:


A. Left ventricular cavity through aortic valve
B. Left subclavian artery
C. A proximal part of the descending aorta
D. Inferior caval vein at the inlet of kidney veins

11.41 Balloon of the intra-aortic balloon pump inflates:


A. During ventricular systole
B. During ventricular diastole

II type tasks. For each numbered item,selct the one lettered heading that is most closely asssciated with it

11.11 Which operation could be done:


1 - Stenosis of bicuspidal aortic valve and dilatation at proximal part of aorta.
2 - Stenosis more than 70% of left main artery.
3 - Aortic valve stenosis in 55 years old patient.
A. Mechanical valve implantation
B. Aortic valve and proximal aortic prosthesis
C. CABG or PCI and stenting
D. Valve replacement with biological valve

11.12 Which vessels needs to be cannulated before initiating cardiopulmonary bypass in order to:
1 - Perform CABG operation
2 - Perform mitral valve plasty
3 - Perform tricuspidal valve plasty
4 - Perform surgical closure of atrial septal defect
A. Aorta, superior and inferior caval veins
B. Aorta, right atrium and inferior caval vein (double stage venous cannula)

11.13 Which palliative heart operations should be performed regarding the below mentioned congenital heart
diseases:
1 - Tetrada Fallot
2 - In case of multiple ventricular septal defects and increased pulmonary artery blood pressure.
3 - In case of great arteries transposition
4 - In case of hypoplastic right ventricle
A. Narrowing of pulmonary artery
B. Blacock-Taussing operation
C. Shunting of Right ventricle
D. Atrial septectomy or septostomy

11.14 Choose the proper operation to treat following congenital heart lesionss
1 - Coarction of aorta
2 - Patent ductus arteriosus
3 - Defectus septi ventriculorum
4 - Tetrada Fallot
A. Septoplastic and infundibulektomia
B. Septoplastica
C. Ligation
D. Resection

11.15 Which operation could be done for patient if


1 - Aneurysm of ascending aorta and aortic valve III regurgitation (MArfan syndrome)
2 - Mitral valve III regurgitation because of mitral valve mixoidal degeneration
3 - Hypertrophic cardiomyopathy and subaortic stenosis
A. Valve prothesis (replacement)
B. Prothesis of ascending aorta with preserving aortic valve (Operation mode David)
C. Plastic operation of valve
D. Myectomy

lsmusis.lsmuni.lt/Klausimai/Spausdinti?Length=0?Kalba=EN&KategorijaId=113&Kalbos_input=EN&Kalbos=EN&KategorijaEn_input=Cardiac+surgery… 2/6
3/27/2019 LSMUSIS
11.16 Choose a proper surgical treatmant to treat following diseases:
1 - Carotid stenosis
2 - Aneurysm of aortae abdominalis
3 - Leriche syndrome (aortoiliac occlusive disease)
4 - Long occlusion of the superficial femoral artery
A. Prosthesis
B. Prosthetic bifurcation shunt
C. Endarterectomy
D. Autovenous shunting

11.17 Acute artery occlusion causes various categories of ischemia. Choose proper symptom specific to category
of ischemia:
1 - I category
2 - IIB category
3 - III category
A. Sensory loss, muscle weakness
B. Numbres, palenes
C. Paralysis (rigor)

III type tasks. For each question there is one or more correct answers:
A – if correct answers are 1,2,3
B – if correct answers are 1 and 3
C – if correct answers are 2 and 4
D – if correct answer is 4
E – if correct are all answers above

11.18 Which symptoms are essential for surgical treatment in acute arterial insufficenty:
1. Pain in extremity
2. Sensitivity disorders
3. Numbness
4. Lack of movements
5. Low skin temperature

11.19 Main signs in varicosis etiology and pathogenesis are:


1. Venous hypertension
2. Hyperlipidaemia
3. Displastic venous valves
4. Malnutrition
5. Hyperglycemia

11.20 Which signs show necessity of critical aortic stenosis surgical correction:
1. Shortnees breath at physical exertion
2. Rhytm dysturbances
3. Faintings
4. Cardiomegaly
5. Focal damage of parenchymal organs

11.21 Please, select congenital heart lesions with shunt from following list:
1. Patent ductus arteriosus
2. Defectus septi ventricularis
3. Tetrada Fallo
4. Stenosis trunci pulmonalis
5. Coarctatio aortae

11.22 There are pulmonary circulation disorders caused by heart septal defects. Select from following list the
degree of pulmonary circulation disorders, which is a indication for surgical closure of defect:
1. Pulmonary hypervolemia
2. Maximal pulmonary hypertension
3. Cianotic heart disease
4. Initial pulmonary hypertension

11.23 Pericardial drainage is indicated:


lsmusis.lsmuni.lt/Klausimai/Spausdinti?Length=0?Kalba=EN&KategorijaId=113&Kalbos_input=EN&Kalbos=EN&KategorijaEn_input=Cardiac+surgery… 3/6
3/27/2019 LSMUSIS
1. Supurative pericarditis
2. Pericardial effusion > 2cm
3. IThe presens of echocardiographic signs of tamponade
4. Constrictive pericarditis

11.24 Which symptoms show chronic mesenteric ischemia?


1. Postprandial abdominal pain, loss of body weight
2. Not digested muscle fiber in captogram
3. Systolic murmur in epigastric region
4. Diarrhea

11.25 What should be endarteritis obliterans (thromboangiitis) conservative tratment:


1. Smoking cessation
2. Antispasmotic drugs
3. Enhancement of microcirculation
4. Prevention of tissue damage and infection

11.26 What symptoms are typical of endarteriitis obliterans?


1. Claudicatio intermittens
2. Thrombophlebitis migrans
3. Damage of the smal1 arteries
4. Hyperlipidaemia

11.27 Which changes in mitral valve requires valve replacement:


1. Calcinosis of the fibrous ring and valve
2. Shortening of the chorda tendinea
3. Fibrosis of the cusps
4. Widening of the fibrous ring

IV type tasks. Choose only one best answer

11.28 6 years old boy, active, has no complains. Murmur is audible since the birth. Clinical investigation revealed
aortic valve stenosis. Choose proper tactic of cardiologist:
A. Surgical correction
B. Child observation
C. Heart catheterisation
D. Heart ECHO

11.29 3 days after birth, 30 years old women suddenly feels pain in chest, shortness of breath and lost conscious.
Massive pulmonary embolism was suspected. Which method of investigation is not necessary in this situation:
A. Chest X-ray
B. EKG
C. Cardiac sonography
D. Chest CT
E. Contrast pulmoangiography
F. Sonography og abdomen

11.30 40m. the patient was diagnosed with severe symptomatic aortic valve insufficiency (aortic valve tricuspid)
and an enlarged ascending aorta up to 55 mm wide. What kind of operation would you recommend:
A. Aortic valve prosthesis with mechanical prosthesis
B. Aortic valve prosthesis with biological prosthesis
C. Ascending aortae prosthesis and aortic valve prosthesis
D. Ascending aortae prosthesis and aortic valve repair
E. Ascending aortae prosthesis

11.31 Mitral valve was replaced for 33 years old woman 4 years ago. After doing well for one year, she noticed
short attacks of legs parasthesia, swelling of the legs, headache. Choose preferable method of investigation:
A. Computed tomography of the head
B. Angiogram of the legs
C. Chest X-ray
D. Echocardiography
E. Computed tomography of the chest

lsmusis.lsmuni.lt/Klausimai/Spausdinti?Length=0?Kalba=EN&KategorijaId=113&Kalbos_input=EN&Kalbos=EN&KategorijaEn_input=Cardiac+surgery… 4/6
3/27/2019 LSMUSIS
11.32 Which diagnostic method would be the most informative in diagnosis of cardiac stab wound:
A. Measurement of arterial blood pressure
B. Hb, hematocrite
C. Echocardiography
D. ECG
E. Chest X-ray

11.33 Which symptom is not characterising the heart tamponade?


A. Distension of neck's veins
B. Cyanosis
C. Liver enlargement
D. Lowered central venous pressure
E. Muffled heart sounds

11.34 55-year-old man after a heart-team consultation was suggested to perform aortic valve prosthetic surgery in
a planned manner. What investigation is not necessary before the intended operation:
A. Chest X-ray
B. Echocardiography
C. Coronary Artery Angiography
D. Dentist consultation on focal infection
E. Chest computed tomography

11.35 74 years old man had a myocardial infarction 5 years ago. He started to experience a chest pain despite the
optimal medicamental treatment. Which diagnostic test would you perform?
A. ECG
B. Veloergometry
C. Echocardiography
D. Coronarography
E. All listed diagnostic tests

11.36 Coronary angiography revealed diffuse sclerosis of circumflex branch and 90% stenoses of LDA and RCA.
Ejection fraction less 0,3. Choose proper treatment:
A. Aortocoronary venous bypass grafts to LDA and RCA
B. Thoracicoronary graft toLDA, venous graft to RCA

11.37 Eighty years old women 3 days feels severe pain in rigth leg. This extremity is cool and paralysed, arterial
pulse is absent, sensitivity of foot is lost, crural muscles are hard, it is rigor of talocrural joint. Which is category of
ischemia:
A. I°
B. II°A
C. II°B
D. III°

11.38 Eighty years old women feels severe pain in rigth leg for 3 days. This extremity is cool and paralysed,
arterial pulse is absent, sensitivity of foot is lost, crural muscles are hard, it is rigor of talocrural joint. Choose
proper method of treatment:
A. Urgent embolectomy
B. Amputation of extremity
C. Delayed embolectomy and fasciotomy
D. Conservative treatment with heparin

11.39 65 years old man complaints of syncopal attacks during 2 weeks, he feels pain in thorax and dyspnoea. He
note progressing legs swelling. Which method of investigation is not urgent?
A. Pulse rate measurement
B. Measurement of arterial blood pressure
C. ECG
D. Chest X-ray
E. Encephalography

11.42 A 53-year-old man with terminal heart failure due to dilated caromyopathy 9 months ago was listed as a
potential heart recipients. Over the past 12 months for the fourth time is hospitalized due to exacerbated heart

lsmusis.lsmuni.lt/Klausimai/Spausdinti?Length=0?Kalba=EN&KategorijaId=113&Kalbos_input=EN&Kalbos=EN&KategorijaEn_input=Cardiac+surgery… 5/6
3/27/2019 LSMUSIS
failure. Cyanotic, peripheral edema in legs and bones. Respiratory rate 24 / min. Heart rate 110 rpm. Arterial blood
pressure 90/50 mmHg. He was on optimal medication for heart failure. During this hospitalisation treatment tactics:
A. Increase dosage of diuretics, reduce heart rate, initiate cardiac glycosides
B. Implant a ventricular assisting system
C. Implant a ventricular assisting system
D. Continue outpatient treatment until heart transplantation

11.43 70 years old man suffered from MI, which lead to left ventricle aneurysm development and cardiac function
impairment. Coronarography revealed stenoses of right coronary artery 80 and circumflex branch 70 with
occlusion of RIVA with retrograde filling. Ejection fraction of left ventricle – 0,25. What treatment is optimal?
A. Aortocoronary bypass grafting
B. Aortocoronary bypass grafting and aneurysmectomy of LV
C. Conservative treatment

11.1 - A 11.2 - A 11.3 - C 11.4 - B 11.5 - A 11.6 - B 11.7 - D 11.8 - B


11.9 - C 11.10 - B 11.40 - C 11.41 - B
11.11 11.12 11.13 11.14 11.15 11.16 11.17
1-B 1-B 1-B 1-D 1-B 1-B 1-B
2-C 2-A 2-A 2-C 2-C 2-A 2-A
3-A 3-A 3-D 3-B 3-D 3-D 3-C
4-A 4-C 4-A 4-C
11.18 11.19 11.20 11.21 11.22 11.23 11.24 11.25
2 1 1 1 4 1 1 1
4 2 2 2 2 2 2
3 3 3 3 3 3
4 4
11.26 11.27
1 1
2 3
3
11.28 - D 11.29 - F 11.30 - D 11.31 - D 11.32 - C 11.33 - D 11.34 - E 11.35 - E
11.36 - B 11.37 - D 11.38 - B 11.39 - E 11.42 - C 11.43 - B

lsmusis.lsmuni.lt/Klausimai/Spausdinti?Length=0?Kalba=EN&KategorijaId=113&Kalbos_input=EN&Kalbos=EN&KategorijaEn_input=Cardiac+surgery… 6/6
3/27/2019 LSMUSIS

Pediatric Surgery
I type tasks. Choose only one best answer

12.1 At what age children should be operated for the funnel chest:
A. At 3 - 5 years
B. At 6-12 years
C. At 13-14 years
D. At 15-16 years
E. At 16-18 years

12.2 When usually the first symptoms of congenital pyloric stenosis appear:
A. After first feeding attempts
B. On the 3-4 day of life
C. On the 2-4 week of life
D. At the 4-8 month of life
E. After introduction of formula feeding

12.3 For which disease the symptom of visibel gastric peristaltic waves ("sandglass") is characteristic:
A. Congenital stenosis of the oesophagus
B. Atresia of the oesophagus
C. High congenital intestinal obstruction
D. Low congenital intestinal obstruction
E. Congenital pyloric stenosis

12.4 When the newborn baby can be feeded after uncomplicated pyloromyotomy:
A. After 24 hours
B. After 18 hours
C. After 12 hours
D. After 6 hours
E. After 3 hours

12.5 What is vitelline duct:


A. Junction between vitelline sack and v. portae
B. Junction between vitelline sack and urine bladder
C. Junction between vitelline sack and primary gut
D. Junction between vitelline sack and embrionic peritoneal cavity
E. Junction between vitelline sack and umbilical artery

12.6 What is the usual Meckel's diverticulus localization:


A. Sigmoid colon
B. Ascending colon, 30 cm from the cecum
C. Duodenum
D. Ileum, 20-70 cm from the cecum
E. Rectum

12.7 Which disorders of pregnancy lead to esophageal atresia in newborns:


A. Nephropathy in pregnancy
B. Preeclampsia
C. Polyhydramnion
D. Untimely discharge of stirred fetal amniotic fluid
E. Threatening premature delivery

12.8 When must the diagnosis of the esophageal atresia be given:


A. During the first 6 hours
B. During the first 12 hours
C. The first twenty-four hours
D. The first week
E. Up to the first feeding

12.9 Which from these disease is not bounded with intestinal malrotation:
A. Umbilical hernia

lsmusis.lsmuni.lt/Klausimai/Spausdinti?Length=0?Kalba=EN&KategorijaId=114&Kalbos_input=EN&Kalbos=EN&KategorijaEn_input=Pediatric+Surge… 1/25
3/27/2019 LSMUSIS
B. Congenital volvulus
C. High retroperitoneal localisation of the vermiform appendix
D. Ladd's syndrome
E. Incarcerated internal hernia of mesocolon

12.10 What is the most frequent cause of the meconial intestinal ileus in children:
A. Anular pancreas
B. Atresia of the small intestine
C. Disorders of embrional intestinal rotation
D. Cystic degeneration of pancreas
E. Congenital stenosis of duodenum

12.11 Which part of intestine the abnormally viscous meconium fills and blocks mostly in case of meconium ileus:
A. Duodenum
B. Jejunum
C. Ileum
D. Cecum
E. Colon sigmoideum

12.12 What form of high congenital ileus is the Ladd's syndrome:


A. Intestinal nonrotation
B. Intestinal malrotation
C. Pathology of omphaloenteric duct
D. Volvulus of midgut and pressure of duodenum because of highly existing cecum
E. Disorder of bowel innervation

12.13 Two horizontal gas - fluid surfaces in the plain x-ray is characteristic to:
A. Atresia of large bowel
B. Acute form of Hirshprung's disease
C. Atresia of pyloric canal
D. Atresia of duodenum
E. Pylorostenosis

12.14 At what age children are the most susceptible to intussusception:


A. Newborns
B. 2-3 months of age
C. 4-9 months of age
D. 1-3 years old
E. 6-10 years old

12.15 What complex of symptoms is the most characteristic to the intestinal intussusception during the intervals
between the attacks of pain:
A. The painful lateral region of the abdomen with clear signs of peritoneum irritation
B. There is not passive muscular rigidity of the abdominal wall. The mobile and painful tumor is felt to the
righjt of the umbilicus
C. The abdomen is distended. Theres is not passive muscular rigidity and very intensive pain during
palpation
D. The abdomen is distended. glittered. with clear passive muscular rigidity
E. Passive muscular rigidity, hard, very painful tumor in the right lateral abdominal region, clear symptoms of
the irritation of peritoneum

12.16 What is the treatment tactics if symptoms of intussusception occured again next day after conservative
desinvagination:
A. Repeated conservative desinvagination with air
B. Conservative desinvagination with barium enema
C. Urgent operation
D. Planned operation after retrograd contrast x-ray of large bowel
E. Observation and medicamental therapy

12.17 These named clinical signs are characteristic to the peritonitis in the newborn, except:
A. Repeating vomiting with an admixture of bile
B. Painful palpation of the abdomen

lsmusis.lsmuni.lt/Klausimai/Spausdinti?Length=0?Kalba=EN&KategorijaId=114&Kalbos_input=EN&Kalbos=EN&KategorijaEn_input=Pediatric+Surge… 2/25
3/27/2019 LSMUSIS
C. Rigidity of the muscles of abdomen wall
D. Distended abdomen
E. Pastosity (oedematous) of the inferior part of abdomen and genitalia

12.18 What test would determinate the tactics of newborn's peritonitis treatment:
A. Abdominal ultrasonography
B. Fibrogastroscopy
C. Blood culture
D. Plain chest-abdominal x-ray
E. Laparoscopy

12.19 Pseudofurunculosis in newborns is:


A. Purulent inflammation of the bulb of hair
B. Purulent inflammation of the sweat glands
C. Purulent necrotic inflammation of the subcutis and sebaceous glands
D. Purulent inflammation of epidermis
E. Purulent inflammation of the lymphatic glands

12.20 What tissue is the primary affected in case of newborn's necrotic phlegmona:
A. Skin
B. Subskin layers
C. Muscles
D. Superficial lymph ducts
E. Periphereal vessels

12.21 What is the commonest cause of newborn's surgical sepsis:


A. Omphalitis
B. Peritonitis
C. Abscess of the skin
D. Bacterial destructive pneumonia
E. Maceration of the skin

12.22 Which complication of pediatric surgical sepsis is the most dangerous:


A. Syndrom of hypertermy
B. Disorder of water-electrolyte balance
C. Destructive pneumonia
D. Disorder of acid-alcaline balance
E. Septic shock

12.23 Indicate the most frequent cause of pyothorax in children:


A. Mediastinitis
B. Rib osteomyelitis
C. Bacterial destructive pneumonia
D. Subdiaphragmatic abscess
E. Chest injury

12.24 What additional test can help to choose the best treatment for the child with bacterial right side lung
destruction, diagnosed clinically and rentgenologically, which has complicated into pyopneumothorax:
A. Bronchoscopy
B. Pleural punction in the second intercostal space
C. Pleuroscopy
D. Pleural punction in the sixth or the seventh intercostal space
E. Echoscopy of the thorax

12.25 Choose the most correct treatment of children's total pleural empyema:
A. Pleural punction
B. Intensive medicamentic therapy
C. Passive pleural drainage
D. Pleural drainage with active aspiration
E. Bronchoscopy

12.26 Which part of the bone is affected most often in case of acute hematogenic osteomyelitis:

lsmusis.lsmuni.lt/Klausimai/Spausdinti?Length=0?Kalba=EN&KategorijaId=114&Kalbos_input=EN&Kalbos=EN&KategorijaEn_input=Pediatric+Surge… 3/25
3/27/2019 LSMUSIS
A. Diaphysis
B. Articular surface
C. Epiphysis
D. Metaphysis
E. Articular capsule

12.27 What part of thigh bone is more often damaged in case of acute haematogenic osteomyelitis:
A. Femur neck
B. Proximal metaphysis
C. Diaphysis
D. Distal epiphysis
E. Proximal epiphysis

12.28 What part of tibia is affected more often in case of acute haematogenic osteomyelitis:
A. Proximal epiphysis
B. Distal epiphysis
C. Diaphysis
D. Proximal metaphysis
E. Distal metaphysis

12.29 When the x-ray signs are visible in case of acute haematogenic osteomyelitis:
A. When the symptoms are present
B. Without antibioticotherapy
C. After 10-12 days
D. After 1 months
E. In case of local form

12.30 What is the reason for spreading of infection from the metaphysis to the epiphysis and adjacent joint in case
of acute haematogenic osteomyelitis in children under 2 years:
A. Immunodeficiency
B. Insufficient barrier of epiphyseal cartilage
C. Weakness of macroorganism
D. Bacterial virulence
E. Influence of predisposing factors

12.31 Choose the most reliable method of early diagnosis of acute haematogenic osteomyelitis:
A. X-ray
B. Phlebography
C. Bone scintigraphy
D. Diagnostic punction of soft tissues
E. Blood test

12.32 What is the morfological basis of Hirschsprung disease:


A. Hyperplasia of mesenteric nerve plexus
B. Ganglioneuromatosis
C. Adenomatosis of large intestine
D. Agangliosis of plexus of the large intestine
E. Hamartoma of the large intestine

12.33 Which part of bowel is affected in children with Hirschprung's disease:


A. Duodenum
B. Jejunum
C. Distal part of ileum
D. Distal part of the large intestine
E. Cecum

12.34 When does reveal the clinical symptoms of severe form of Hirschprung's disease:
A. In the first 6 months
B. In the first year of life
C. In the first month after birth
D. In the first days after birth
E. When child starts to be feeded with the solid food

lsmusis.lsmuni.lt/Klausimai/Spausdinti?Length=0?Kalba=EN&KategorijaId=114&Kalbos_input=EN&Kalbos=EN&KategorijaEn_input=Pediatric+Surge… 4/25
3/27/2019 LSMUSIS
12.35 What causes obstipation and gaseous distension in case of Hirschprung’s disease:
A. Impaired water resorbtion in the aganglionic segment
B. Absence of peristalsis in the aganglionic segment
C. Colonic dilatation and atonicity
D. Defecation disturbance

12.36 What is the prompt treatment action after diagnosis of Hirschprung's disease:
A. Colostomy operation
B. Operation - resection of aganglionic zone and anastomosis
C. Conservative treatment with medicine, helping evacuation of large bowel contents
D. Operation - resection of aganglionic zone and colostomy

12.37 What clinical symptom enables the earliest suspection for the rectum atresia:
A. The abdomen of a newborn is asymetric
B. A newborn is vomiting with bile admixture
C. A newborn is defecating a small quantity of meconium
D. Meconium is not evacuated during the first day of life

12.38 What test helps to measure distance between rectum atgresia and anus:
A. Rectomanoscopy
B. Irigoscopy (x-ray with barium enema)
C. Contrast passage
D. Invertogram
E. Plain x-ray of abdomen in vertical position

12.39 What treatment tactics is the most efficient in case of anus atresia with a fistula in the perineum:
A. Urgent operation, performing proctoplasty
B. Urgent colostomy
C. Conservative treatment, postponing the operation for the later period
D. Vangensteen's probe and treatment according to its results

12.40 Vesicoureteral reflux is active, if:


A. Contrast fills only distal part of ureter
B. Contrast fills distented ureter and renal pelvis
C. It is seen during urination
D. It is seen only ofter filling urine bladder with contrast
E. It is seen in horizontal position

12.41 What way of examination is the most efficient for the diagnosing of vesicouretheral reflux in children:
A. Echoscopy
B. Excretory urography
C. Cystography
D. Ascending pyelography

12.42 Which is the right approach treating a I-II grade vesicoureteral reflux:
A. Ureterocystoneostomy (reimplantation of the ureters)
B. Plastics of the pyeloureteric segment
C. Epicystostomy and antimicrobic treatment
D. Medical treatment of the urinary tract infection

12.43 Which pathogenic cause of pyelonephritis is the most important in children:


A. Immature immune system
B. Increased tissue sensibilization due to infection
C. Congenital urinary tract obstruction or dysfunction of the neurogenic bladder both causing impaired
urinary passage
D. Infection spreading in the urinary tract by haematogenic way

12.44 Vesicoureteric reflux is:


A. Urine flow from bladder to urethra
B. Urine flow from bladder to ureters

12.45 Paraphymosis is:


A. The narrow foreskin
lsmusis.lsmuni.lt/Klausimai/Spausdinti?Length=0?Kalba=EN&KategorijaId=114&Kalbos_input=EN&Kalbos=EN&KategorijaEn_input=Pediatric+Surge… 5/25
3/27/2019 LSMUSIS
B. Ectopy of uretral orifice
C. Separation of corpore cavernosa of the penis
D. Incarceration of glans penis in the narrow orifice of the foreskin

12.46 Which is the right approach treating the “acute scrotum” syndrome:
A. Puncture of the testis coats
B. Antibiotics and spirituous bandage
C. Urgent surgical exploration of the inguinal channel
D. Urgent surgery opening coats of the testis
E. Bed regimen, suspensor and physiotherapy

12.47 What is called “varicocele”:


A. Fluid accumulation under testicular coats
B. Fluid accumulation in the spermatic cord
C. Sack like dilatation of the distal part of the ureter
D. Nodes-like varicosity of the veins of the pampiniform plexus

12.48 What kind of voiding disorders is characteristic to epispadia:


A. Painful voiding (dysuria)
B. Urine retention
C. Nocturnal enuresis
D. Urine incontinention

12.49 What disorder of urination is characteristic to the children suffering from hypospadias:
A. Painful urination
B. Difficult urination
C. Imperative urination
D. Incontinence of urine
E. Noctural incontinence

12.50 When is the most appropriate time for the operation on cryptorchidism:
A. Immediately after the diagnosis was established
B. Till 6 months of age
C. At 0,5-2 years of age
D. At 3-5 years of age
E. At puberty

12.51 What age is the best for hydrocele operation:


A. After the diagnosing
B. 6-12 mouths
C. 1-2 years
D. 3-5 years
E. School age

12.52 What is the main cause of inguinal hernia:


A. "Weakness" of aponeurosis and muscles in the groin region
B. Wide external inguinal ring
C. Persistense of processus vaginalis
D. Increased intraabdominal pressure
E. Frequent respiratory diseases

12.53 What is the most appropriate examination choosing the tactics of treatment, in case of hematuria after blunt
abdominal trauma:
A. Plain-film radiography
B. Ultrasonography
C. Radioisotopic urography
D. CT
E. Cystography

12.54 What is the reason of urgent operation in case of teratoma in children:


A. Danger of suppuration
B. Pressing of the nearest organs

lsmusis.lsmuni.lt/Klausimai/Spausdinti?Length=0?Kalba=EN&KategorijaId=114&Kalbos_input=EN&Kalbos=EN&KategorijaEn_input=Pediatric+Surge… 6/25
3/27/2019 LSMUSIS
C. Danger of malignisation
D. Danger of bleeding
E. Cosmetical defects

12.55 At what age would you recommend to ablate the teratoma in children:
A. l month of age
B. 3-6 months of age
C. Over 6 months of age
D. At once, after diagnosis was confirmed
E. 1 year of age

12.56 Which tumour of the retroperitoneal space is the most common in children:
A. Neuroblastoma
B. Nephroblastoma
C. Teratoblastoma
D. Angiosarcoma

12.57 What is the localisation of lateral neck cyst or fistula:


A. By the angle of lower mandibula
B. By the anterior edge of m.sternocleidomastoideus
C. By the posterior edge of m.sternocleidomastoideus
D. Between the pedicles of m.sternocleidomastoideus

12.58 Which localisation of the gastrointestinal tract duplicature is the most common:
A. Esophagus
B. Stomach
C. Small intestine
D. Large intestine
E. Rectum

II type tasks. For each numbered item,selct the one lettered heading that is most closely asssciated with it

12.59 What treatment is necessary to each pediatric urologic disease:


1 - Varicocele
2 - Hydrocele testis
3 - Torsio testis
4 - Funiculocele communicans
5 - Paraphymosis
6 - Hypospadia penilis
A. Urgent operation
B. Planned operation

12.60 What features of rectal bleeding is characteristic to each disease:


1 - Intussusception
2 - Rectal polyp
3 - Anal fissure
4 - Meckel's diverticulum
5 - Newborn's haemorrhagic syndrom
A. "Black currant jelly" stool
B. Dark, liquid stool
C. Stool with small amount of bright blood
D. Bright blood or blood clots
E. Mixed bright and dark blood in the stool

12.61 What is treatment tactics in following postoperative ileus cases:


1 - Dynamic (paralitic) ileus
2 - Early adhaesional ileus
3 - Late adhaesional ileus
A. Urgent operative treatment
B. Conservative treatment for at least 6 hours, then it unsuccessful - operation
C. Conservative treatment for 2-3 hours, if unsuccessful - operation
D. Only conservative treatment

lsmusis.lsmuni.lt/Klausimai/Spausdinti?Length=0?Kalba=EN&KategorijaId=114&Kalbos_input=EN&Kalbos=EN&KategorijaEn_input=Pediatric+Surge… 7/25
3/27/2019 LSMUSIS
12.62 What x-ray symptoms are characteristic to each form of oesophageal atresia:
1 - Upper part ended blindly, lower has connection with trachea
2 - Both parts end blindly
3 - Upper part is connected with trachea, lower part ends blindly
4 - Both ports can are connected with trachea
A. Contrast is in the upper part. There is no air in the stomach and bowels
B. Contrast is in upper part, trace - in respiratory tract. There is no air in the stomach and bowels
C. Contrast is in the upper part. Theres is air in the stomach and bowels
D. Contrast is in the upper part, trache-in the respiratory tract. There is air in the stomach and bowels

12.63 What disorders of urination are characteritic to each malformation of urethra:


1 - Hypospadias
2 - Epispadias
3 - Phymosis
A. Difficult urination
B. Painful urination
C. Incontinence
D. Imperative urination

12.64 What x-ray signs is characteristic to each form of congenital ileus:


1 - High congenital ileus
2 - Low congenital ileus
A. 2 horizontal air - fluid surfaces and darkened abdominal cavity
B. One big horisontal air- fluid surface (stomach air buble) and small amount of gas in the intestines
C. Many horizontal air-fluid surfaces

12.65 What age of the child is the best for surgical correction of each urogenital malformations:
1 - Epispadias
2 - Cryptorchidism
3 - Hydrocele
A. Till 1 month of age
B. Till 6 months of age
C. At 0,5-2 years of age
D. At 3-7 years of age

12.66 What symptoms are characteristic to each form of congenital diaphragmatic hernia:
1 - Vomiting
2 - Cyanosis
3 - Haemorrhagic syndrom
4 - Weak breathing sound in the left side of the chest
5 - Heart tones are stronger in the right side
A. Posterolateral diaphragmatic hernia (Bochdalek)
B. Hernia hiatus oesophagi

12.67 What symptom is the most characteristic to:


1 - Pylorospasmus
2 - Pyloric stenosis
3 - Bradyesophagus
4 - Low congenital ileus
5 - Cerebral injury during delivery
A. Vomiting with bile admixtures
B. Vomiting without bile admixtures
C. Vomiting with blood admixtures

12.68 What in the most characteristic symptom to each of these diseases:


1 - Rectum stenosis
2 - Anal stenosis
3 - Fissura ani
4 - Rectal polyps
A. Constipation
B. Painful defecation
C. Incontinence of stool
lsmusis.lsmuni.lt/Klausimai/Spausdinti?Length=0?Kalba=EN&KategorijaId=114&Kalbos_input=EN&Kalbos=EN&KategorijaEn_input=Pediatric+Surge… 8/25
3/27/2019 LSMUSIS
D. Stool with blood

12.69 What method of bowel contents evacuation is more acceptable for each disease:
1 - Atonic constipation
2 - Hirschprung's disease
3 - Postoperative paresis of bowel
4 - Adhaesional ileus
A. Washing enema
B. Hypertonic enema
C. Siphon enema
D. Laxatives

12.70 What are the causes of each congenital ileus forms:


1 - High position of cecum
2 - Annular pancreas
3 - Atresia of ileum
4 - Meconial ileus
5 - Atresia of duodenum
6 - Hirshrung's disease
A. Low congenital ileus
B. High congenital ileus

12.71 What are the reasons of mentioned diseases in pediatric urology:


1 - Stenosis of pyelouretheral segment
2 - Vesicoureteral reflux
3 - Addional artery of the kidney
4 - High position of ureter in the renal pelvis
5 - Ureterocele
6 - Stenosis in the intramural part of ureter
A. Hydronephrosis
B. Ureterohydronephrosis

12.72 What symptoms are characteristic to this diseases (in the age group under 3 years):
1 - Acute onset of the disease
2 - Subfebrile temperature
3 - Tenderness and rigidity of abdominal wall
4 - Vomiting
5 - Soft, relaxed abdomen
A. Acute appendicitis
B. Acute respiratory infection

12.73 What features of umbilical cord coverings is characteristic to each form of umbilical cord hernias:
1 - Coverings are transparent and unruptured
2 - Coverings are ruptured
3 - Coverings are untranspared with necrotic tissue and fibrin
4 - Coverings are ruptured, part of viscera - eventrated
A. Uncomplicated umbilical cord hernia
B. Complicated umblical cord hernia

12.74 What diaphragmatic hernias are "true" and what "untrue":


1 - Anterior diaphragmatic hernia
2 - Hernia hiatus esophagi
3 - Posterolateral diaphragmatic hernia. (Bochdalek)
4 - Phrenico-pericardial hernia
5 - Pathologic bending of diaphragma
A. True diaphragmatic hernia
B. Untrue diaphragmatic hernia

III type tasks. For each question there is one or more correct answers:
A – if correct answers are 1,2,3
B – if correct answers are 1 and 3
C – if correct answers are 2 and 4
lsmusis.lsmuni.lt/Klausimai/Spausdinti?Length=0?Kalba=EN&KategorijaId=114&Kalbos_input=EN&Kalbos=EN&KategorijaEn_input=Pediatric+Surge… 9/25
3/27/2019 LSMUSIS
D – if correct answer is 4
E – if correct are all answers above

12.75 Which clinical symptoms become more apparent with age in a child with a funnel chest:
1. Progressing deformity
2. Spinal column deformity
3. Paradoxal breathing
4. Circulation disturbance in the small circle of blood circulation
5. Retarded physical maturation

12.76 What criteria is the most important to the operative treatment of chest wall deformity:
1. Age of the patient
2. Degree of deformity
3. Duration of the disease
4. Functional changes of chest organs
5. Concomitant diseases

12.77 What symptoms are characteristic to pylorostenosis:


1. Projectile vomiting
2. Vomiting with bile
3. Vomiting without bile
4. Belch with food
5. Cyanosis and dyspnea after feeding

12.78 Which symptoms are the most reliable diagnosing exicosis in infants with congenital pyloric stenosis:
1. Sunken fontanel
2. Oliguria
3. Infrequent defecation
4. Increased hematocrit
5. Increased hemoglobin level

12.79 Which symptoms of contrast roentgenography are the most common for the pyloric stenosis:
1. Contrast staying in the cardia
2. Retarded evacuation from the stomach
3. Pylorus filling deffect
4. Narrowed antrum pylori
5. More apparent relief of pyloric mucosa

12.80 What are the Meckel's diverticulum complications:


1. Inflammation
2. Intestinal volvulus
3. Intestinal bleeding
4. Intestinal intususception
5. Internal intestinal incarceration

12.81 What combination of clinical symptoms is characteristic to esophageal atresia:


1. Foamy secretion from the month and the nose
2. Cianosis
3. Moist rales in the lungs
4. Positive Elephant probe
5. When probing the esophagus, at the distance of 10-12 cm an obstacle is traced

12.82 What are the most important causes determining succesful treatment of newborns with esophageal atresia:
1. The level of maturity of a newborn
2. The distance between the segments of esophagus
3. The time of diagnosing abnormality
4. Associated congenital anomalies
5. Techniques of operation and postoperation care

12.83 What clinical symptoms are characteristic in newborns with inborn posterolateral diaphragmatic hernia:
1. Cyanosis
2. Weaker respiration in the left side of the thorax

lsmusis.lsmuni.lt/Klausimai/Spausdinti?Length=0?Kalba=EN&KategorijaId=114&Kalbos_input=EN&Kalbos=EN&KategorijaEn_input=Pediatric+Surg… 10/25
3/27/2019 LSMUSIS
3. Stronger heart tones, heard in the righ side of the thorax
4. Distended abdomen
5. Vomiting after meals

12.84 What complex of x-ray symptoms is characteristic to congenital posterolateral diaphragmatic hernia:
1. Changes in the diaphragma line
2. Various "darke" and "light" areas in the left side of the chest
3. Dislocation of the mediastinum organs to the right
4. Dislocation of the mediastinum organs to the left
5. Multiplex horizontal surfaces in the abdomen

12.85 Why a child with hernia of the esophageal hiatus may vomit with blood:
1. Because of hemorrhagic vasculitis
2. Because of traumatisation of the stomach wall
3. Because of gastric ulcerative disease
4. Because of peptic esophagitis
5. Because of venostasis in the stomach wall

12.86 What are the most important symptoms of congenital high ileus:
1. Vomiting after the first feeding
2. Quick loss of weight
3. "Sunked" and not painful abdomen
4. Absence of meconium
5. Obstructive peristaltic waves

12.87 What are the most characteristic symptoms in case of low congenital ileus:
1. Vomitting, during the second or third day of life
2. Absence of meconium
3. Distended, not very painful abdomen
4. Obstructive peristalsis
5. Tension of the abdominal muscles, rebound tenderness

12.88 What combination of clinical symptoms are characteristic to intussusception:


1. Periodical pain
2. Vomiting
3. Stool with blood
4. Abdominal distention
5. Kocher's symptom

12.89 What combination of the symptoms of acute appendicitis is the most valuable:
1. Vomiting
2. Rigidity of abdominal muscles
3. Diarrhea
4. Local tenderness and pain in the right lateral area of abdomen
5. Rebound tenderness (Blumberg's symptom)

12.90 What are the commonest preoperative complications of pediatric appendicitis:


1. Sepsis
2. Appendicular mass
3. Paralytic ileus
4. Purulent peritonitis
5. Interintestinal abscess

12.91 What complex of tests confirms postoperative diagnosis of yersiniosis:


1. Blood test
2. Histology of mesenteric lymphnode
3. Culture of peritoneal fluid
4. Serologic tests
5. Histology of appendix

12.92 What are the causes for umbilicus to become moist in newborns:
1. Calcinosis of the umbilicus

lsmusis.lsmuni.lt/Klausimai/Spausdinti?Length=0?Kalba=EN&KategorijaId=114&Kalbos_input=EN&Kalbos=EN&KategorijaEn_input=Pediatric+Surg… 11/25
3/27/2019 LSMUSIS
2. Omphalitis
3. Granuloma of the umbilical bottom
4. Fistula of the umbilicus

12.93 What are the complications of newborn's omphalitis:


1. Sepsis
2. Phlegmone of abdominal wall
3. Peritonitis
4. Liver abscess
5. Osteomyelitis

12.94 What x-ray symptoms let us to suspect pyopneumothorax:


1. Darkened lower part of the lung
2. Light and without structure near the wall of the chest
3. Horizontal light - dark line
4. Darkened upper part of the lung
5. Darkened central part of the lung with the little light zones

12.95 What changes of blood test are the earliest and the most characteristic in case of pediatric surgical sepsis:
1. Trombocytopenia
2. Anemia
3. Leucocytosis
4. Neutropenia
5. Eosinophylia

12.96 What are characteristic changes in the periferial blood in case of acute hematogenic osteomyelitis:
1. Leucocytosis
2. High erythrocytes sedimentation rate
3. Trombocytopenia
4. Anemia
5. Shift to the left in the blood formula

12.97 What roentgenological symptoms are the most characteristic for the acute hematogenic osteomyelitis in
adolescence:
1. Foci of the osteoporosis in the metaphysis
2. Enlargement of the joint space
3. Ossificative periostitis
4. Fragmentation of the epiphysis
5. Bone thickening and sclerosis with the spines of the periost

12.98 Which roentgenologic signs are characteristic to acute haematogenic osteomyelitis in newborns, infants and
little children (1-2 years age):
1. Periostis ossificans
2. Destruction in metaphyseal area
3. Thickness of the bone and foci of osteosclerosis with spiculas of periosteum
4. Widening of the joint space
5. Epiphyseal fragmentation (foci of destruction in the metaepiphysis)

12.99 The most important signs of Hirschsprung's disease are:


1. Constipation
2. Increasing of abdomen's volume
3. Meteorism
4. Painful evacuation
5. Melena

12.100 What test can confirm the Hirschprung's disease diagnosis:


1. Anorectal manometry
2. Irigoscopy and irigography
3. Neurohistochemical test of the large bowel mucosa
4. Barium contrast x-ray
5. Fibrocolonoscopy

lsmusis.lsmuni.lt/Klausimai/Spausdinti?Length=0?Kalba=EN&KategorijaId=114&Kalbos_input=EN&Kalbos=EN&KategorijaEn_input=Pediatric+Surg… 12/25
3/27/2019 LSMUSIS
12.101 In what cases of rectal or anal atresia urgent colostomy is indicated:
1. Atresia with fistula to urinary tract
2. In case of diagnosis delay, if the condition of the newborn is poor
3. Atresia with uteral fistula
4. In case of poor newborn's condition due to associated anomalies
5. In case of high rectal atresia

12.102 What are late complications after reconstructive operations of anus and rectum malformations:
1. Deformation of anus and stenosis of anus
2. Insufficiention of anal sphincter and stool incontinence
3. Recurrent fistula between rectum and nearest organs
4. Urine incontinence

12.103 Which clinical symptoms or laboratory data may give a suspition about a urinary tract disease:
1. Leucocyturia
2. Disorders of micturition
3. Abdominal pain
4. Subfebrile fever
5. Hematuria

12.104 What changes of urine test are the most common in case of pediatric urological diseases:
1. Urobilinuria
2. Haematuria
3. Pollakiuria
4. Piuria
5. Proteinuria

12.105 Which tests should be carried for a 4-year-old girl admitted for abdominal pain and micturition disturbance,
after pyuria was confirmed:
1. Izotopic renography
2. Renal ultrasound
3. Cystoscopy
4. Bacteriological urine assay
5. Vaginal discharge assay

12.106 What are the most often phimosis complications:


1. Necrosis of the glans of the penis
2. Balanopostitis
3. Deformation of the penis
4. Infravesical obstruction

12.107 Choose the appropriate treatment methods of paraphymosis:


1. Circumcision
2. Reposition of glans penis under the narcosis
3. Foreskin resection
4. Dorsal incision of the foreskin ring
5. Bath of desinfectants

12.108 Which are the most common causes of the urethral obstruction in children:
1. Sclerosis of the urinary bladder neck
2. Phimosis
3. Urethral valve
4. Ureteric stenosis
5. Diverticulum of the urinary bladder

12.109 What combination of clinical symptoms is characteristic to the inguinal hernia in children:
1. Unstable, limited swelling in the inguinal area
2. Thickened spermatic cord
3. Widened superficial ring of the inguinal canal
4. Absent testis in the side of suspected disease
5. Swollen scrotum veins of that side

lsmusis.lsmuni.lt/Klausimai/Spausdinti?Length=0?Kalba=EN&KategorijaId=114&Kalbos_input=EN&Kalbos=EN&KategorijaEn_input=Pediatric+Surg… 13/25
3/27/2019 LSMUSIS
12.110 When conservative reposition of incarcerated hernia is possible in boy:
1. If he has no viral infection
2. If he has cryptorchidism
3. If the duration of incarceration is not longer 6 hours
4. If the duration of incarceration is not longer 12 hours
5. If there is no symptoms of ileus

12.111 When conservative ways of treatment are not recomended for the reposition of incarcerated inguinal hernia
in children:
1. A boy's incarcerated inguinal hernia
2. A premature boy’s incarcerated inguinal hernia
3. 10 hours passed from the incarceration
4. Inguinal hernia is incarcerated to a boy with cryptorchidism of the same side
5. A child with pneumonia

12.112 What are the most common reasons of the blood in the stool in children:
1. Anal fissure
2. Varices haemorrhoidales
3. Intestinal polyps
4. Intestinal duplication with the ulcer
5. Dysentery

12.113 What is your tactics if there is some blood in a patient’s feces after otherwise normal defecation:
1. Prescription of rectal suppositories
2. Examination of the anus with anal dilators
3. Prescription of a warm bath with KMnO in sitting position
4. Rectal examination and rectosigmoscopy
5. Sending to an infectionist

12.114 Choose the most efficient ways of treating hemangioma in children:


1. Intralesional steroids
2. Surgical extirpation
3. Sclerous therapy
4. Diathermocoagulation
5. Cryotherapy

12.115 What could be the symptoms in case of alimentary tract duplications:


1. Hemorrhage into the alimentary tract
2. Mechanical ileus
3. Tumor in the abdomen
4. Inflammation in the abdominal cavity
5. Any symptoms

IV type tasks. Choose only one best answer

12.116 A 3-week-old bay presented with plentiful belching or projectile vomiting. The poured off content consisted
of half digested milk free of bile. Amount of thrown up fluid was greater than amount of primarily eaten milk. The
boy also developed obstipation and was urinating as frequently as 4-5 times per 24 hours. Examination revealed a
sick child with dry wrinkled skin, the epigastrium was bulging, peristalsis waves were visible. The elastic mass
palpable in the epigastrium to the right from midline. What is the preliminary diagnosis:
A. Esophageal atresia
B. High congenital intestinal obstruction
C. Low congenital intestinal obstruction
D. Congenital pylorostenosis
E. Pylorospasm

12.117 A 3-week-old bay presented with plentiful belching or projectile vomiting. The poured off content consisted
of half digested milk free of bile. Amount of thrown up fluid was greater than amount of primarily eaten milk. The
boy also developed obstipation and was urinating as frequently as 4-5 times per 24 hours. Examination revealed a
sick child with dry wrinkled skin, the epigastrium was bulging, peristalsis waves were visible. The elastic mass
palpable in the epigastrium to the right from midline. Which investigation could be helpful establishing the
diagnosis:

lsmusis.lsmuni.lt/Klausimai/Spausdinti?Length=0?Kalba=EN&KategorijaId=114&Kalbos_input=EN&Kalbos=EN&KategorijaEn_input=Pediatric+Surg… 14/25
3/27/2019 LSMUSIS
A. Computer tomography
B. Abdominal ultrasound
C. Retrograde irigoscopy

12.118 A 3-week-old bay presented with plentiful belching or projectile vomiting. The poured off content consisted
of half digested milk free of bile. Amount of thrown up fluid was greater than amount of primarily eaten milk. The
boy also developed obstipation and was urinating as frequently as 4-5 times per 24 hours. Examination revealed a
sick child with dry wrinkled skin, the epigastrium was bulging, peristalsis waves were visible. The elastic mass
palpable in the epigastrium to the right from midline. What is the most efficient treatment of this disease:
A. Resection of the pylorus
B. Gastroenterostomy
C. Pyloromyotomy
D. Atropinisation

12.119 A 4-week –old boy presented with severe vomiting and weight loss. Peristalsis waves were visible while the
stomach was examined. An obvious hypertrophy of the pyloric muscle was diagnosed by ultrasound. What is your
tactics:
A. Atropinisation and observation
B. Pyloromyotomy
C. Pyloroplastic
D. Gastroduodenostomy
E. Resection of the pylorus

12.120 12 hours after birth a male newborn was admited to the department of Poediatric Surgery. The baby was
delivered from the second pregnancy during which polyhydramnion was diagnosed for the mother. Birth weight of
the child was 3000g, now it is 2950g. The newborn is cyanotic, a foamy discharge is coming from his mouth and
nose. At the first feeding attempts the patient starts choking, coughing, gets bluish, fever rises up to 38,5ºC. The
skin is dry, wrinkled, the eyeballs and fontanel is sunken, lungs auscultation reveals wet crepitation.What is the
preliminary diagnosis:
A. Congenital diaphragmatic hernia
B. Congenital emphysema of the lung lobe
C. Congenital esophageal atresia
D. Congenital duodenal atresia
E. Congenital pyloric stenosis

12.121 12 hours after birth a male newborn was admited to the department of Poediatric Surgery. The baby was
delivered from the second pregnancy during which polyhydramnion was diagnosed for the mother. Birth weight of
the child was 3000g, now it is 2950g. The newborn is cyanotic, a foamy discharge is coming from his mouth and
nose. At the first feeding attempts the patient starts choking, coughing, gets bluish, fever rises up to 38,5ºC. The
skin is dry, wrinkled, the eyeballs and fontanel is sunken, lungs auscultation reveals wet crepitation.How the
newborn should be prepared for the operation:
A. Placed into a heated incubator, supplied with oxygen, with a catheter in the esophagus for periodical
suction of mucous discharge, getting vit.K and antibiotics supply
B. Intubated and being under regulated ventilation
C. Undergoing curable bronchoscopy, getting fluids intravenously for correction of the homeostasis
D. Getting direct 20-30 ml blood transfusion

12.122 Newborn started to breath rapidly and dyspnea occured after 6 hours after the birth and feeding.
Tachycardia - 180 b/min. Dyspnea is intensive in the position on the right side. The left part of the chest is bigger
and the movements of this side are disproportional. Percussion: the normal percusion sound in the right and
changing areas of blunt and tympanic sounds is the left. Auscultation: heart beats are not clear in the typical
points. There is no lung sound in the left, sometimes some crepitation is hearing. What is preliminary diagnosis:
A. Congenital lobar emphisema
B. Congenital lung cyst
C. Esophageal atresia
D. Diaphragmatic hernia
E. Pneumothorax

12.123 Newborn started to breath rapidly and dyspnea occured after 6 hours after the birth and feeding.
Tachycardia - 180 b/min. Dyspnea is intensive in the position on the right side. The left part of the chest is bigger
and the movements of this side are disproportional. Percussion: the normal percusion sound in the right and
changing areas of blunt and tympanic sounds is the left. Auscultation: heart beats are not clear in the typical
lsmusis.lsmuni.lt/Klausimai/Spausdinti?Length=0?Kalba=EN&KategorijaId=114&Kalbos_input=EN&Kalbos=EN&KategorijaEn_input=Pediatric+Surg… 15/25
3/27/2019 LSMUSIS
points. There is no lung sound in the left, sometimes some crepitation is hearing. What test could help with the
diagnosis:
A. Ultrasonography of chest and abdomen
B. Brochoscopy
C. Punction of pleura
D. X-ray of chest and abdomen
E. Laparoscopy

12.124 Newborn started to breath rapidly and dyspnea occured after 6 hours after the birth and feeding.
Tachycardia - 180 b/min. Dyspnea is intensive in the position on the right side. The left part of the chest is bigger
and the movements of this side are disproportional. Percussion: the normal percusion sound in the right and
changing areas of blunt and tympanic sounds is the left. Auscultation: heart beats are not clear in the typical
points. There is no lung sound in the left, sometimes some crepitation is hearing. What must be treatment tactics:
A. Pleural drenage
B. Pleural punction
C. Siphon enema
D. Urgent operation
E. Elective surgery after 3 months

12.125 A newborn started to vomit with bile after the first feeding. He vomited 3 times the first day. Next day
vomiting became more intensive, newborn became sluggish, adinamic, had weight loss of 250 g. He had stool with
small quantity of meconium and urinated 6 times. During examination: skin and mucosas are dry, fontanel is
"falled": Hypogastrium is distented, it becomes less distented after vomiting. Abdomen is soft. Blood test: creatinin
- 3,2 mmol/l; urea 92 mmol/l; sodium - 116 mmol/; Ht 0,55. Two horizontal fluid surfaces is seen in plain x-ray, no
air in the bowels.What is the preliminary diagnosis:
A. Congenital pyloric stenosis
B. Congenital high ileus
C. Oesophageal atresia
D. Hirschprung's disease
E. Congenital low ileus

12.126 A newborn started to vomit with bile after the first feeding. He vomited 3 times the first day. Next day
vomiting became more intensive, newborn became sluggish, adinamic, had weight loss of 250 g. He had stool with
small quantity of meconium and urinated 6 times. During examination: skin and mucosas are dry, fontanel is
"falled": Hypogastrium is distented, it becomes less distented after vomiting. Abdomen is soft. Blood test: creatinin
- 3,2 mmol/l; urea 92 mmol/l; sodium - 116 mmol/; Ht 0,55. Two horizontal fluid surfaces is seen in plain x-ray, no
air in the bowels.Are additional tests necessary to confirme the diagnosis:
A. Yes
B. No

12.127 A newborn started to vomit with bile after the first feeding. He vomited 3 times the first day. Next day
vomiting became more intensive, newborn became sluggish, adinamic, had weight loss of 250 g. He had stool with
small quantity of meconium and urinated 6 times. During examination: skin and mucosas are dry, fontanel is
"falled": Hypogastrium is distented, it becomes less distented after vomiting. Abdomen is soft. Blood test: creatinin
- 3,2 mmol/l; urea 92 mmol/l; sodium - 116 mmol/; Ht 0,55. Two horizontal fluid surfaces is seen in plain x-ray, no
air in the bowels.What is treatment tactics:
A. Long lasting atropinisation
B. Permanent gastric tube
C. Peridural anaesthesia
D. Urgent operation
E. Siphon enema

12.128 A 5-months- old boy presented with history of severe abdominal pain followed by repeated vomiting. The
symptoms appeared 3 hour ago. Pain was lasting for several minutes and than disappeared. There were also
retention of urine and feces. The abdomen was soft, the patient was not reacting for palpation. An oblique movable
mass was palpated at the right side of the abdomen, that area was tender and the child was reacting by crying.
Rectal examination showed the empty ampulla, but there was a trace of blood on glove.What is the preliminary
diagnosis:
A. Nephroblastoma
B. Intussusception
C. Acute appendicitis
D. Volvulus
lsmusis.lsmuni.lt/Klausimai/Spausdinti?Length=0?Kalba=EN&KategorijaId=114&Kalbos_input=EN&Kalbos=EN&KategorijaEn_input=Pediatric+Surg… 16/25
3/27/2019 LSMUSIS
E. Coprostasis

12.129 A 5-months- old boy presented with history of severe abdominal pain followed by repeated vomiting. The
symptoms appeared 3 hour ago. Pain was lasting for several minutes and than disappeared. There were also
retention of urine and feces. The abdomen was soft, the patient was not reacting for palpation. An oblique movable
mass was palpated at the right side of the abdomen, that area was tender and the child was reacting by crying.
Rectal examination showed the empty ampulla, but there was a trace of blood on glove.Which investigation may
be helpful establishing the diagnosis:
A. Plain chest and abdominal X-ray
B. Laparoscopy
C. Contrast passage through intestines on serial x-rays
D. Abdominal ultrasound
E. Rectoscopy

12.130 A 5-months- old boy presented with history of severe abdominal pain followed by repeated vomiting. The
symptoms appeared 3 hour ago. Pain was lasting for several minutes and than disappeared. There were also
retention of urine and feces. The abdomen was soft, the patient was not reacting for palpation. An oblique movable
mass was palpated at the right side of the abdomen, that area was tender and the child was reacting by crying.
Rectal examination showed the empty ampulla, but there was a trace of blood on glove.What is the choise of
treatment:
A. Immediate surgery
B. Urgent surgery after 1-2 hours of preparation
C. Elective surgery after an enema and rectoscopy
D. Irigoscopy with an increased air pressure
E. Siphonic enema

12.131 An- 8- year-old boy presented with intussusception. The patient has been ill for 10 hours. What is the
choice of treatment:
A. Elective surgery
B. Conservative reposition
C. Deep palpation of the abdomen under general anesthesia
D. Observation for 4-6h
E. Urgent surgery

12.132 Yesterday, about 10 P.M. a 5-month-old baby suddenly started crying and got cyanotic. 20 min. later he
vomited with half digested milk. The mother gave him an enema after which the baby passed normal stools,
became quiet and feel asleep. Half an hour later he woke up, started crying and vomited again with yellowish fluid.
All over the night was crying, vomited once more. In the morning defecated with bloody stools. At 9 A.M. was
brought to emergency room. The examination revealed that the patient had no fever, but his general condition was
poor- the baby was intoxicated and irritated. There were no pathological lungs sounds, heart beating was
rhytmical, heart rate – 100 beats per min. The abdomen was distended, soft, the elastic oblique, painful,
unmovable mass was palpated in the right side of the abdomen.To which department the baby must be admited:
A. Dept, of Paediatric Infectious diseases
B. Dept.of Paediatrics
C. Dept.of Paediatric Surgery

12.133 Yesterday, about 10 P.M. a 5-month-old baby suddenly started crying and got cyanotic. 20 min. later he
vomited with half digested milk. The mother gave him an enema after which the baby passed normal stools,
became quiet and feel asleep. Half an hour later he woke up, started crying and vomited again with yellowish fluid.
All over the night was crying, vomited once more. In the morning defecated with bloody stools. At 9 A.M. was
brought to emergency room. The examination revealed that the patient had no fever, but his general condition was
poor- the baby was intoxicated and irritated. There were no pathological lungs sounds, heart beating was
rhytmical, heart rate – 100 beats per min. The abdomen was distended, soft, the elastic oblique, painful,
unmovable mass was palpated in the right side of the abdomen.What is preliminary diagnosis:
A. Mesenterial cyst
B. Renal tumour
C. Intussusception
D. Intestinal volvulus
E. Dysenteria

12.134 Yesterday, about 10 P.M. a 5-month-old baby suddenly started crying and got cyanotic. 20 min. later he
vomited with half digested milk. The mother gave him an enema after which the baby passed normal stools,
lsmusis.lsmuni.lt/Klausimai/Spausdinti?Length=0?Kalba=EN&KategorijaId=114&Kalbos_input=EN&Kalbos=EN&KategorijaEn_input=Pediatric+Surg… 17/25
3/27/2019 LSMUSIS
became quiet and feel asleep. Half an hour later he woke up, started crying and vomited again with yellowish fluid.
All over the night was crying, vomited once more. In the morning defecated with bloody stools. At 9 A.M. was
brought to emergency room. The examination revealed that the patient had no fever, but his general condition was
poor- the baby was intoxicated and irritated. There were no pathological lungs sounds, heart beating was
rhytmical, heart rate – 100 beats per min. The abdomen was distended, soft, the elastic oblique, painful,
unmovable mass was palpated in the right side of the abdomen.Which investigation could be helpful establishing
the precise diagnosis:
A. Rectoromanoscopy
B. Plain abdominal X-ray
C. Contrast investigation with Barium
D. Abdominal ultrasound

12.135 Yesterday, about 10 P.M. a 5-month-old baby suddenly started crying and got cyanotic. 20 min. later he
vomited with half digested milk. The mother gave him an enema after which the baby passed normal stools,
became quiet and feel asleep. Half an hour later he woke up, started crying and vomited again with yellowish fluid.
All over the night was crying, vomited once more. In the morning defecated with bloody stools. At 9 A.M. was
brought to emergency room. The examination revealed that the patient had no fever, but his general condition was
poor- the baby was intoxicated and irritated. There were no pathological lungs sounds, heart beating was
rhytmical, heart rate – 100 beats per min. The abdomen was distended, soft, the elastic oblique, painful,
unmovable mass was palpated in the right side of the abdomen.Reponating an intussusception under X-ray
control, it moved down to the ileocaecal region, but the air did not pass into the small bowel. The patient has been
ill for 18 hours. What is the choice of treatment:
A. Observation for 2-3 hours
B. Elective surgery
C. Deep palpation of the abdomen under general anesthesia
D. Repeated contrast X-ray with Barium
E. Urgent operation

12.136 14 days after conservative reposition of intussusception an 8-months-old baby developed clinical
symptoms of intussusception again. What is the choice of treatment:
A. Consultations of the pediatrics
B. Serial x-rays with contrast
C. Reposition with air enema
D. Elective surgery
E. Observation

12.137 A 2-year-old boy was admitted to the clinic of Pediatric Surgery because of severe onset of pain. The
symptoms started 12 hours ago. The small bowel intussusceptions was diagnosed. What is the choice of
treatment:
A. Contrast intestinal X-ray
B. Observation at the hospital
C. Urgent surgery
D. Elective surgery
E. Conservative reposition

12.138 A child is complaining of severe abdominal pains. Examination revealed a postoperative scar in the
abdominal wall. What is the preliminary diagnosis:
A. Renal stones
B. Pancreatitis
C. Intussusceptions
D. Spastic intestinal obstruction
E. Intestinal obstruction due to adhesions

12.139 A 10-year-old boy was operated half a year ago for diffuse peritonitis of appendicular origin. Present onset
of pains started suddenly. The child was repeatedly vomiting with food remnants, later – with green fluid. The
mother has given a no-spa tablet, but it didn’t really help. The patient was admitted to the hospital 6 hours after the
pains started. The boy was laying pale, lips dry, knees drowned up to abdomen. The abdomen was distended, soft
and tender, there was no muscle guarding. Sometimes a loud peristalsis could be auscultated.What is the
preliminary diagnosis:
A. Renal stones
B. Dynamic (paralytic) intestinal obstruction
C. Intestinal obstruction due to adhesions
lsmusis.lsmuni.lt/Klausimai/Spausdinti?Length=0?Kalba=EN&KategorijaId=114&Kalbos_input=EN&Kalbos=EN&KategorijaEn_input=Pediatric+Surg… 18/25
3/27/2019 LSMUSIS
D. Incarcerated inguinal hernia
E. Acute pancreatitis

12.140 A 10-year-old boy was operated half a year ago for diffuse peritonitis of appendicular origin. Present onset
of pains started suddenly. The child was repeatedly vomiting with food remnants, later – with green fluid. The
mother has given a no-spa tablet, but it didn’t really help. The patient was admitted to the hospital 6 hours after the
pains started. The boy was laying pale, lips dry, knees drowned up to abdomen. The abdomen was distended, soft
and tender, there was no muscle guarding. Sometimes a loud peristalsis could be auscultated.Which investigation
could be helpful establishing the diagnosis:
A. Abdominal ultrasound
B. Izotopic renography
C. Plain abdominal X-ray
D. Laparoscopy

12.141 A 10-year-old boy was operated half a year ago for diffuse peritonitis of appendicular origin. Present onset
of pains started suddenly. The child was repeatedly vomiting with food remnants, later – with green fluid. The
mother has given a no-spa tablet, but it didn’t really help. The patient was admitted to the hospital 6 hours after the
pains started. The boy was laying pale, lips dry, knees drowned up to abdomen. The abdomen was distended, soft
and tender, there was no muscle guarding. Sometimes a loud peristalsis could be auscultated.What is the choice
of treatment:
A. Immediate operation
B. 3-5 hours of medical treatment: evacuation of stomach content, siphon enema, intravenous drip – after
that surgery
C. 12h observation, infusotherapy with spasmolitics
D. Peridural anesthesia
E. Stimulation of bowel peristalsis with prozerin

12.142 A 4-month-old boy is ill with bacterial pneumonia. 2 days ago the general condition started worsening: the
abdomen got distended, there was no flatus, the patient was not passing stools, there were a few onsets of
vomiting with green content. Examination revealed fever up up 38.4ºC, the abdomen was distended tender,
without obvious muscle guarding, there was no peristalsis. The dilated bowel filled up with air was seen on the
plain abdominal X-ray.What is the preliminary diagnosis:
A. Peritonitis
B. Paralytic (dinamic) intestinal obstruction
C. Pleural empyema
D. Appendicitis
E. Intussusception

12.143 A 4-month-old boy is ill with bacterial pneumonia. 2 days ago the general condition started worsening: the
abdomen got distended, there was no flatus, the patient was not passing stools, there were a few onsets of
vomiting with green content. Examination revealed fever up up 38.4ºC, the abdomen was distended tender,
without obvious muscle guarding, there was no peristalsis. The dilated bowel filled up with air was seen on the
plain abdominal X-ray.Which assays could be helpful choosing the appropriate treatment:
A. Hematocrit, acid-base balance, K in a blood
B. Hemaglobin, WBC and RBC
C. Protein and sugar in a blood
D. Amilasis, transaminases and alkaline phosphatasis in a blood

12.144 A 4-month-old boy is ill with bacterial pneumonia. 2 days ago the general condition started worsening: the
abdomen got distended, there was no flatus, the patient was not passing stools, there were a few onsets of
vomiting with green content. Examination revealed fever up up 38.4ºC, the abdomen was distended tender,
without obvious muscle guarding, there was no peristalsis. The dilated bowel filled up with air was seen on the
plain abdominal X-ray.What is the choice of treatment:
A. Injection of papaverin or analgin, oxygen supply
B. Bed regimen, fasting
C. Electrolytes infusion, prozerin injection, constant gastric tube, siphonic enema
D. Infussion of proteases inhibitors, fasting

12.145 What must to doctor after examination of irritated 2,5 years old boy with suspection of appendicitis if he
haven'y found enough symptoms, confirming diagnosis of acute appendicitis:
A. Let go home
B. Admit to the hospital
lsmusis.lsmuni.lt/Klausimai/Spausdinti?Length=0?Kalba=EN&KategorijaId=114&Kalbos_input=EN&Kalbos=EN&KategorijaEn_input=Pediatric+Surg… 19/25
3/27/2019 LSMUSIS
C. Let go home and call to district pediatrician for observation
D. Let go home and call to district surgeon for observation
E. Let go home and ask to return back in case of worsening

12.146 3 years old boy became irritable after dinner. The sleep in the night was not good, he had vomiting one
time. In the morning he had softer stool 2 times. Examination: He has running nose, lips are dry. heart rate is 94
b/min. T 37,5 C. Boy layes on the right side, huddled up in bed. Abdomen is mild distented; painful paltation of the
right side; here is involuntary muscular guarding. Digital examination - rectum is empty.What is the preliminary
diagnosis:
A. Dysenteria
B. Intussusception
C. Acute appendicitis
D. Acute pancreatitis
E. Obstructive ileus

12.147 3 years old boy became irritable after dinner. The sleep in the night was not good, he had vomiting one
time. In the morning he had softer stool 2 times. Examination: He has running nose, lips are dry. heart rate is 94
b/min. T 37,5 C. Boy layes on the right side, huddled up in bed. Abdomen is mild distented; painful paltation of the
right side; here is involuntary muscular guarding. Digital examination - rectum is empty.What is the treatment
tactics:
A. Urgent operation
B. Operation after preparation with i/v infusion for 2 hours
C. Intensive antibioticotherapy without fluid limitation
D. Parenteral nutrition, antibiotics, observation
E. Enemas

12.148 A 3-year-old girl is ill for 8 hours, she is irritable, has running nose and fever 38ºC. The patient was
complaining for the abdominal pain, vomiting, does not allow to be examined by a doctor. The girl is crying,
resisting to palpation, tensing abdominal muscles during the investigation. What tactic must choose pediatric
surgeon if investigation is impossible? How a doctor should behave in this situation:
A. To admit patient to the hospital for observation
B. To prescribe antibiotics and discharge home for ambulatory observation
C. To ask a pediatrician to see the patient
D. To arrange the abdominal X-Ray
E. To prescribe an enema

12.149 A 3-year-old girl is ill for 8 hours, she is irritable, has running nose and fever 38ºC. The patient was
complaining for the abdominal pain, vomiting, does not allow to be examined by a doctor. The girl is crying,
resisting to palpation, tensing abdominal muscles during the investigation. What tactic must choose pediatric
surgeon if investigation is impossible? How the patient should be examined:
A. Examination in spite of the child’s reluctance
B. Examination while the parents are absent
C. Examination after the patient is put into sleep by 3 percent chloralhydrat enema
D. Examination under general anesthesia

12.150 The infiltrative appendicitis is diagnosed to the boy 12 years old. During the course of conservative
treatment abdomen pain and signs of peritoneum irritation occured. Temperature - 38,7º C. What is your tactics:
A. Operation - abscessotomy
B. Increasing of antibiotic's dose
C. Changing of antibiotics
D. Cold upon the abdomen and observation
E. Operation - Appendectomy

12.151 A 13-year-old boy is suspected for the appendicular mass without an abscess (the patient has been ill for 3
weeks). Your tactics:
A. Urgent surgery without preparation
B. Elective surgery in the nearest future
C. Active treatment with antibiotics, physiotherapy, observation
D. Observation at the Intensive Care Unit, no medical treatment
E. Ambulatoric observation

lsmusis.lsmuni.lt/Klausimai/Spausdinti?Length=0?Kalba=EN&KategorijaId=114&Kalbos_input=EN&Kalbos=EN&KategorijaEn_input=Pediatric+Surg… 20/25
3/27/2019 LSMUSIS
12.152 A 6-year-old boy has been ill at home for 3 days. The symptoms started with pain in the epigastrium. Later
the pain shifted to the right side of the abdomen. The patient repeatedly vomited with green content. The parents
treated the boy themselves with some tablets. After admission to the hospital general condition of the patient was
poor: fever 39ºC, the boy was pale, the heart rate was 140 /min. The lips were dry, the tongue with white coating.
The abdominal wall was motionless in the process of the breathing. The whole abdomen was tender, there was
muscle guarding and rebound tenderness. There were a few erythrocytes in the urine sample and leucocytosis in
the blood sample – 15x10 /l.What is the preliminar diagnosis:
A. Acute appendicitis
B. Peritonitis
C. Intestinal volvulus
D. Acute pancreatitis
E. Renal stones

12.153 A 6-year-old boy has been ill at home for 3 days. The symptoms started with pain in the epigastrium. Later
the pain shifted to the right side of the abdomen. The patient repeatedly vomited with green content. The parents
treated the boy themselves with some tablets. After admission to the hospital general condition of the patient was
poor: fever 39ºC, the boy was pale, the heart rate was 140 /min. The lips were dry, the tongue with white coating.
The abdominal wall was motionless in the process of the breathing. The whole abdomen was tender, there was
muscle guarding and rebound tenderness. There were a few erythrocytes in the urine sample and leucocytosis in
the blood sample – 15x10 /l.What is the choice of treatment:
A. Prescribtion of analgetics and putting a child into a warm bath
B. Urgent laparotomy
C. Urgent surgery via appendicectomy incision
D. Preoperative preparation with intravenous infusion depending on degree of the dehidratation and surgery
after preparing

12.154 A red spot without clear limits was noticed in the skin of newborn. He is irritated, has fever (39º C). Skin
around the spot is infiltrated, warmer.What is preliminary diagnosis:
A. Erysipelas
B. Adiponecrosis
C. Osteomyelitis
D. Necrotic phlegmona
E. Sclerema

12.155 A red spot without clear limits was noticed in the skin of newborn. He is irritated, has fever (39º C). Skin
around the spot is infiltrated, warmer.What must be treatment tactics:
A. Prescribe antibiotics and warm localy, next day - examination
B. Prescribe antibiotics and warm procedures, next day - consultation of pediatric surgeon
C. Urgent admition to pediatric surgery department
D. Urgent admition to the department of infection diseases

12.156 2 years old boy had viral infection, then pneumonia of the right lung was diagnosed, and the boy was
admitted to the pediatric department. Suddenly the condition became worse: cyanosis, dyspnea were observed.
Breathing rate - 60 t/min; heart rate - 140 t/min; there in no breathing sound in the right side of the chest, bronchial
breathing sound in the left. Tympanic percusion sound is in the right side.What is the diagnosis of the pneumonia
complication:
A. Lung atelectasis
B. Lung abscess
C. Pyothorax
D. Pyopneumothorax

12.157 2 years old boy had viral infection, then pneumonia of the right lung was diagnosed, and the boy was
admitted to the pediatric department. Suddenly the condition became worse: cyanosis, dyspnea were observed.
Breathing rate - 60 t/min; heart rate - 140 t/min; there in no breathing sound in the right side of the chest, bronchial
breathing sound in the left. Tympanic percusion sound is in the right side.What urgent test would be the most
helpful with diagnosis:
A. Chest ultrasound
B. Bronchoscopy
C. Pleural punction
D. Chest x-ray

lsmusis.lsmuni.lt/Klausimai/Spausdinti?Length=0?Kalba=EN&KategorijaId=114&Kalbos_input=EN&Kalbos=EN&KategorijaEn_input=Pediatric+Surg… 21/25
3/27/2019 LSMUSIS
12.158 Right side pneumonia was diagnosed for a 9-month-old baby. The general condition was not improving
after intensive antibacterial treatment. The intoxication accompanied by high fever was increasing and breathing
insufficiency was getting worse. Auscultation data: the breathing sound is weaker on the right side, the percussion
sound is hollow.What is the preliminary diagnosis of the complication after pneumonia:
A. Lung atelectasis
B. Lung abscess
C. Pyopneumothorax
D. Pyothorax

12.159 Right side pneumonia was diagnosed for a 9-month-old baby. The general condition was not improving
after intensive antibacterial treatment. The intoxication accompanied by high fever was increasing and breathing
insufficiency was getting worse. Auscultation data: the breathing sound is weaker on the right side, the percussion
sound is hollow.Which urgent investigation could confirm the diagnosis:
A. Chest ultrasound
B. Bronchoscopy
C. Pleural puncture
D. Chest X-ray

12.160 37.223 A 3-week-old newborn started running fever up to 39oC. There also was apetitis and sleep regimen
disturbance The mother noticed the the baby starts crying while being bathed, changed clothes and especially
touched on the right leg. There was noticeble enlargement of the articular space.. What is the preliminary
diagnosis:
A. Thigh cellulitis (phlegmone)
B. Acute hematogenic osteomyelitis
C. Necrotic cellulitis (phlegmone) of newborns
D. Osteoplastic sarcoma of the thigh-bone
E. Osteidosteoma

12.161 District paediatrician has observed a mobile, unpainful mass in the abdomen above the symphysis for a 6-
year-old boy. Constipation, meteorysmus, viral infections, retardation of physical development was noticed in the
medical history. Now the boy is pale, thin, abdomen is distended; mobile, unpainful mass is palpable above the
symphysis.What is preliminary diagnosis:
A. Hydronephrosis
B. Splenomegalia
C. Nephroblastoma
D. Hirschsprung's disease
E. Renal polycystosis

12.162 District paediatrician has observed a mobile, unpainful mass in the abdomen above the symphysis for a 6-
year-old boy. Constipation, meteorysmus, viral infections, retardation of physical development was noticed in the
medical history. Now the boy is pale, thin, abdomen is distended; mobile, unpainful mass is palpable above the
symphysis.What test would help with diagnosis:
A. Abdominal ultrasound
B. Rectoromanoscopy
C. Cystography
D. Excretion urography
E. Retrograde contrast x-ray intestinal investigation

12.163 A 5-years-old girl is suffering from recurrent pyuria and lumbar pain. Ultrasonography and excretory
urography showed ureterohydronephrosis in the left side. During the cystoscopy chronic cystitis and open orifice of
left ureter were diagnosed.What of the tests can help with diagnosis:
A. Retrograde pyelography
B. Dynamic scintigraphy with isotopes
C. Cystography
D. Aorthography

12.164 A 5-years-old girl is suffering from recurrent pyuria and lumbar pain. Ultrasonography and excretory
urography showed ureterohydronephrosis in the left side. During the cystoscopy chronic cystitis and open orifice of
left ureter were diagnosed.
A. Urgent operation - ureterocystoneostomy
B. Operation - plastic of pyeloureteral segment
C. To perform epicystostomy and to treat with anti-inflammatory drugs.
lsmusis.lsmuni.lt/Klausimai/Spausdinti?Length=0?Kalba=EN&KategorijaId=114&Kalbos_input=EN&Kalbos=EN&KategorijaEn_input=Pediatric+Surg… 22/25
3/27/2019 LSMUSIS
D. To prescribe drugs for the sanation of the urinary tract

12.165 A 3-year-old boy started complaining that he wants to micturate but he is not able. The mother has given a
tablet of no-spa, but the child still could not void. The patient was brought to the emergency room. Examination
revealed the irritable crying child in pain. He had no fever, heart rate was 82 beats per minute. The lower part of
the abdomen was distended, just above the oss pubis one could palpate a round shaped elastic not movable a bit
painful mass. At the site of external genitalia – no pathological findings, just a tip of the penis looks redish and
there comes a bit of yellow puss under a slight pressure. What is the preliminary diagnosis:
A. Cystitis
B. Stones in the urinary bladder
C. Paraphimosis
D. Urethral valve
E. Balanopostitis

12.166 A 3-year-old boy started complaining that he wants to micturate but he is not able. The mother has given a
tablet of no-spa, but the child still could not void. The patient was brought to the emergency room. Examination
revealed the irritable crying child in pain. He had no fever, heart rate was 82 beats per minute. The lower part of
the abdomen was distended, just above the oss pubis one could palpate a round shaped elastic not movable a bit
painful mass. At the site of external genitalia – no pathological findings, just a tip of the penis looks redish and
there comes a bit of yellow puss under a slight pressure. What is the choice of treatment:
A. Transurethral resection of the valve
B. Surgical correction of paraphimosis
C. Cleaning of the preputium, warm antiseptic baths, warm enema
D. Puncture of the urinary bladder
E. Lythotripsy

12.167 A 3-year old boy presented with an incarcerated inguinal hernia, it happened 2 hours ago. Which is the
right approach:
A. Urgent surgery
B. Manual reposition of the hernia
C. Put cold on the heria for necrosis prevention
D. Prescription of some medicine hoping that the hernia will reponate spontaneously
E. Reposition under short term anesthesia

12.168 A 2-year-old boy suddenly started to cry, suffocate, to vomit, became cyanotic. Grandmother knocked
several times to the back. The boy became better, cyanosis disappeared. Emergency came and paediatrician
stated that child is frightened; breath rate is 32 times/minute, vesicular breathing in the both sides of breast: in the
right lower parts - just a little weaker. Heart sounds clear, abdomen - without changes.What is the preliminary
diagnosis:
A. Pneumonia
B. Foreign body in the respiratory tract
C. Intoxicatio alimentaris
D. Foreign body in the digestive tract
E. Bronchitis

12.169 A 2-year-old boy suddenly started to cry, suffocate, to vomit, became cyanotic. Grandmother knocked
several times to the back. The boy became better, cyanosis disappeared. Emergency came and paediatrician
stated that child is frightened; breath rate is 32 times/minute, vesicular breathing in the both sides of breast: in the
right lower parts - just a little weaker. Heart sounds clear, abdomen - without changes.What is your tactics:
A. To observe at home
B. To send to the hospital
C. To prescribe antibiotics and to observe
D. To send to the district paediatrician

12.170 A 2-year-old boy suddenly started to cry, suffocate, to vomit, became cyanotic. Grandmother knocked
several times to the back. The boy became better, cyanosis disappeared. Emergency came and paediatrician
stated that child is frightened; breath rate is 32 times/minute, vesicular breathing in the both sides of breast: in the
right lower parts - just a little weaker. Heart sounds clear, abdomen - without changes.What additional investigation
is the most informative:
A. Fibroesophagoscopy
B. Bronchoscopy
C. Laryngoscopy
lsmusis.lsmuni.lt/Klausimai/Spausdinti?Length=0?Kalba=EN&KategorijaId=114&Kalbos_input=EN&Kalbos=EN&KategorijaEn_input=Pediatric+Surg… 23/25
3/27/2019 LSMUSIS
D. Blood test

12.1 - B 12.2 - C 12.3 - E 12.4 - E 12.5 - C 12.6 - D 12.7 - C 12.8 - E


12.9 - A 12.10 - D 12.11 - C 12.12 - D 12.13 - D 12.14 - C 12.15 - B 12.16 - A
12.17 - C 12.18 - D 12.19 - B 12.20 - B 12.21 - A 12.22 - E 12.23 - C 12.24 - D
12.25 - D 12.26 - D 12.27 - A 12.28 - D 12.29 - C 12.30 - B 12.31 - C 12.32 - D
12.33 - D 12.34 - D 12.35 - B 12.36 - B 12.37 - D 12.38 - D 12.39 - C 12.40 - C
12.41 - C 12.42 - D 12.43 - C 12.44 - B 12.45 - D 12.46 - D 12.47 - D 12.48 - D
12.49 - B 12.50 - C 12.51 - C 12.52 - C 12.53 - D 12.54 - C 12.55 - D 12.56 - B
12.57 - B 12.58 - C
12.59 12.60 12.61 12.62 12.63 12.64 12.65 12.66
1-B 1-A 1-D 1-C 1-A 1-A 1-C 1-B
2-B 2-D 2-B 2-A 2-C 2-C 2-C 2-A
3-A 3-C 3-C 3-B 3-A 3-C 3-B
4-B 4-B 4-D 4-A
5-A 5-E 5-A
6-B
12.67 12.68 12.69 12.70 12.71 12.72 12.73 12.74
1-A 1-A 1-D 1-B 1-A 1-B 1-A 1-A
2-B 2-A 2-C 2-B 2-B 2-A 2-B 2-A
3-C 3-B 3-B 3-A 3-A 3-A 3-B 3-B
4-A 4-D 4-C 4-A 4-A 4-A 4-B 4-B
5-A 5-B 5-B 5-B 5-A
6-A 6-B
12.75 12.76 12.77 12.78 12.79 12.80 12.81 12.82
1 2 1 1 2 1 1 1
2 4 3 2 4 2 2 2
3 3 3 3 3
4 4 4 4 4
5 5 5 5 5
12.83 12.84 12.85 12.86 12.87 12.88 12.89 12.90
1 1 2 1 1 1 2 2
2 2 4 2 2 2 4 4
3 3 3 3 3
12.91 12.92 12.93 12.94 12.95 12.96 12.97 12.98
2 1 1 1 1 1 1 2
4 2 2 2 2 2 3 4
3 3 3 3 3
4 4 4
5 5
12.99 12.100 12.101 12.102 12.103 12.104 12.105 12.106
1 1 1 1 1 2 2 2
2 2 2 2 2 4 4 4
3 3 3 3 3
4 4 4
5 5
12.107 12.108 12.109 12.110 12.111 12.112 12.113 12.114
2 1 1 2 4 1 2 1
4 2 2 4 3 4 2
3 3 3
4
5
12.115
1
2
3
4

lsmusis.lsmuni.lt/Klausimai/Spausdinti?Length=0?Kalba=EN&KategorijaId=114&Kalbos_input=EN&Kalbos=EN&KategorijaEn_input=Pediatric+Surg… 24/25
3/27/2019 LSMUSIS
5
12.116 - D 12.117 - B 12.118 - C 12.119 - B 12.120 - C 12.121 - A 12.122 - D 12.123 - D
12.124 - D 12.125 - B 12.126 - B 12.127 - D 12.128 - B 12.129 - D 12.130 - D 12.131 - B
12.132 - C 12.133 - C 12.134 - D 12.135 - E 12.136 - C 12.137 - E 12.138 - E 12.139 - C
12.140 - C 12.141 - B 12.142 - B 12.143 - A 12.144 - C 12.145 - B 12.146 - C 12.147 - A
12.148 - A 12.149 - C 12.150 - A 12.151 - C 12.152 - B 12.153 - D 12.154 - D 12.155 - C
12.156 - D 12.157 - D 12.158 - D 12.159 - D 12.160 - B 12.161 - D 12.162 - E 12.163 - C
12.164 - D 12.165 - E 12.166 - C 12.167 - D 12.168 - B 12.169 - B 12.170 - B

lsmusis.lsmuni.lt/Klausimai/Spausdinti?Length=0?Kalba=EN&KategorijaId=114&Kalbos_input=EN&Kalbos=EN&KategorijaEn_input=Pediatric+Surg… 25/25
3/27/2019 LSMUSIS

Thoracic surgery
I type tasks. Choose only one best answer

41.1 45.1. A cavitary lesion of 3 cm in diameter located in second segment of right lung was revealed in 55-year
old male on his prophylactic examination. Which one of the following tests is most likely to establish the diagnosis:
A. Bronchography
B. Computed tomography
C. Bronchoscopy and biopsyt
D. Pulmonoangiography
E. Imunologic skin test

41.2 Tension (valvular) pneumothorax may cause all of the following pathologic syndromes except one:
A. Hypoxia
B. Acidosis
C. Decreased venous return to the heart
D. Decreased cardiac output
E. Dulness to percusion on the side of the pneumothorax

41.3 All of following symptoms and signs are indicative for tension pneumothorax except one:
A. Chest pain
B. Shortness of breath
C. Absence of breath sound unilaterally
D. Shifting of the mediastinum towards the side of the pneumothorax
E. Hypotension

41.4 45.4. A patient underwent a left scalene node biopsy to rule out carcinoma of the lung. One hour later the
patient became cyanotic and dyspneic. Clinical examination revealed tachycardia and decreased breath sounds
on the left side of the chest. What is the maneuver that most likely to improve the patient’s condition:
A. Blood transfusion
B. Insertion of a right subclavian catheter and administration of intravenous fluids
C. Endotracheal intubation
D. Drainage of the left pleural cavity
E. Reexploration of the wound

41.5 45.5. A 60-year-old man has a peripheral solitary pulmonary node (coin lesion). All of the following diagnosis
are likely except one:
A. Bronchogenic carcinoma
B. Tuberculosis
C. Metastatic colon cancer
D. Hamartoma
E. Sarcoidosis

41.6 Which of the following findings indicates unresectablity of the lung cancer:
A. Pneumonia
B. Sputum cytology positive for cancer
C. Shortness of breath
D. Left vocal cord paralysis
E. Clubbing of the digits

41.7 45.7. Which of the following procedures would be the most appropriate measure in the management of a
patient with a spontaneous pneumothorax:
A. Observation with daily chest roentgenograms
B. Aspiration of air with a needle followed by observation
C. Thoracotomy and biopsy
D. Chest tube drainage of the pleural space
E. Thoracotomy and pleurectomy to prevent recurrence

41.8 45.8. A 45-year-old man underwent chest X-ray and CT. A peripheral lung tumor located in seventh segment
of his right lung was revealed. However, bronchoscopy and brush biopsy from seventh segment of right lung did
not confirm suspicion of lung cancer. What methods for diagnosis and treatment are most suitable in this case:
A. Follow-up using chest X-ray

lsmusis.lsmuni.lt/Klausimai/Spausdinti?Length=0?Kalba=EN&KategorijaId=143&Kalbos_input=EN&Kalbos=EN&KategorijaEn_input=Thoracic+surgery… 1/7
3/27/2019 LSMUSIS
B. Radical radiotherapy
C. Lover lobectomy of right lung
D. Explorative thoracotomy
E. Chemotherapy

41.9 45.9. Hoarseness secondary to bronchogenic carcinoma is usually caused by extension of the tumor into
mediastinum. Which one of the following organs is involved in this case:
A. Vocal cord
B. Superior laryngeal nerve
C. Left recurrent laryngeal nerve
D. Right vagus nerve
E. Larynx

41.10 45.10. The lower part of the esophagus was perforated during esophagoscopy. A barium swallow done
immediately after erforation demonstrates a small amount of barium is leaking into the left pleural space. What is
the most suitable method of treatment:
A. Observation alone
B. Observation and antibacterial therapy
C. Insertion of a left chest tube
D. Prolonged esophageal intubation
E. Urgent surgical repair of esophageal injury

41.11 45.11. A 65-year-old man can swallow only liquid meal. Esophageal carcinoma of 4-th clinical group was
diagnosed. In such case there are some acceptable methods of treatment except one:
A. Radiation therapy
B. Laser recanalization of the esophageal lumen
C. Bougierage or pneumodilatation
D. Gastrostomy
E. Radical resection and reconstruction of the esophagus
F. Bypass esophago-gastric anastomosis

41.12 A 30-year-old woman drinked by mistake some vinegar (acetic acid). The bougierage of the esophagus was
not used. During the last 5 days complete dysphagia developed. What method of treatment is unfit in this case:
A. Parenteric (intravenous) nutrition
B. Gastrostomy
C. Bougierage of the esophagus
D. Resection and reconstruction of the esophagus

II type tasks. For each numbered item,selct the one lettered heading that is most closely asssciated with it

41.13 What is the main and most often cause of patients mortality in following urgent situations:
1 - Tension pneumothorax
2 - Intensive pulmonary haemorrhage
3 - Injury of the heart
4 - Thoracoabdominal injury of the liver
5 - Purulent mediastinitis
A. Loss of blood
B. Asphyxia
C. Acute respiratory and cardiovascular insufficiency
D. Sepsis
E. Intrapericardial cardiac tamponade

41.14 What is the most acceptable method for treatment of the following chest injuries:
1 - Multiple fracture of the ribs
2 - Rupture of the diaphragm
3 - Injury to the lung presenting with pneumothorax
4 - Injury to the heart
5 - Rupture of the esophagus
A. Immobilisation of chest wall using adhesive plaster
B. Urgent surgical intervention
C. Drainage of the pleural cavity
D. Local anesthesia of intercostal nerves
lsmusis.lsmuni.lt/Klausimai/Spausdinti?Length=0?Kalba=EN&KategorijaId=143&Kalbos_input=EN&Kalbos=EN&KategorijaEn_input=Thoracic+surgery… 2/7
3/27/2019 LSMUSIS
E. Vago-symphatetic novocaine blockade

41.15 What is the most acceptable method for treatment of chest trauma, presenting with:
1 - Haemothorax
2 - Pneumothorax
3 - Chylothorax
4 - Clotted hemothorax
5 - Unstable chest wall (flail chest)
A. Thoracotomy
B. Local anesthesia of intercostal nerves
C. Drainage of the pleural cavity
D. Bandage of the chest wall
E. Osteosynthesis and fixation of ribs

41.16 What is the method of patient’s examination is most likely to establish the diagnosis in following situations:
1 - Traumatic hemo-pneumothorax
2 - Spontaneous pneumothorax
3 - Foreign body in airways
4 - Erlargement of paratracheal lymphnodes
5 - Atelectasis of middle lobe of the right lung
A. Explorative thoracotomy
B. Bronchoscopy
C. Percutaneous needle biopsy
D. Mediastinoscopy
E. Thoracoscopy

41.17 The left-side diaphragmatic relaxation was established 6 years after chest trauma. What is the most
acceptable method of treatment in following situations:
1 - Diaphragmatic relaxation of I-II°
2 - Diaphragmatic relaxation of III°
A. Surgical intervention – diaphragmatic plication
B. Artificial pulmonary ventilation
C. Tracheostomy
D. Kinesitherapy and diaphragmatic electrostimulation
E. Phrenic nerve repair

41.18 What anatomical structures of the chest are injured in cases of following traumas:
1 - Superficial chest wounds
2 - Blunt chest trauma
3 - Penetrating chest trauma
4 - Mixed chest trauma (polytrauma)
5 - Combined chest trauma
A. Injuries to the chest wall, at least parietal pleura
B. Any chest wall layer except parietal pleura
C. Any anatomical structure except skin
D. Injury to the chest and other anatomical regions
E. Injury caused by several traumatic factors: mechanical, thermal, chemical, radiation etc.

III type tasks. For each question there is one or more correct answers:
A – if correct answers are 1,2,3
B – if correct answers are 1 and 3
C – if correct answers are 2 and 4
D – if correct answer is 4
E – if correct are all answers above

41.19 45-year-old man complains of dysphagia during the period of 10 last years. The diagnosis of cardiac
axalasia is established. What methods of treatment are acceptable:
1. Esophagomyotomy (Heller procedure)
2. Pyloroplasty
3. Pneumodilatation
4. Gastrostomy

lsmusis.lsmuni.lt/Klausimai/Spausdinti?Length=0?Kalba=EN&KategorijaId=143&Kalbos_input=EN&Kalbos=EN&KategorijaEn_input=Thoracic+surgery… 3/7
3/27/2019 LSMUSIS
41.20 What are the indication for resection of esophageal Zenker diverticulum:
1. Retention of the contrast material in diverticulum sac
2. Diverticulitis
3. Aspiration during eating and drinking
4. Dysphagia

41.21 When it is necessary to start with kinesitherapy:


1. 1 hours after operation
2. During the first postoperative day
3. At any time after operation
4. Early as possible

41.22 What is the optimal duration of one séance of kinesitherapy


1. 1 hour
2. 30 min.
3. About 15 min.
4. Until patient’s mild fatique occurs

41.23 How often is rationally to repeat the seances of kinesitherapy:


1. Every 15 min.
2. Every 30 min.
3. 3 times pro day
4. Every 1-2 hours
5. Mornings and evenings

41.24 What are methods used for local management (drainage) of lung abscesses:
1. Transsternal
2. Transthoracic (percutaneous)
3. Transdiaphragmatic
4. Endobronchial

41.25 What conditions are required for treatment of lung abscess using percutaneous (transthoracic) drainage
(Monaldi method):
1. Abscess is localized near the chest wall (peripherial lung abscess)
2. Patient’s general condition is satisfactory
3. Around the abscess are formed pleural adhesions
4. Abscess wall is quite thick

41.26 What are the most important signs of chronic lung abscess:
1. Abscess wall are thick
2. Recurrence of inflammatory process
3. Duration of intensive treatment is more than 2 months
4. Patients age more than 50 years

41.27 What surgical maneuvers should be performed in case of recurrent spontaneous pneumothorax:
1. Suture of lung tears of bullas
2. Decortication and biopsy of the lung
3. Pleurectomy
4. Thoracoplasty
5. Pleuropulmonectomy

41.28 What surgical maneuvers should be performed in case of chronic pleural empyema :
1. Decortication of the lung
2. Thoracoplasty
3. Pleurectomy
4. Pleurostomy

41.29 What surgical maneuvers should be performed in case of clotted traumatic hemothorax:
1. Decortication of the lung
2. Hemostasis
3. Removing of blood clots
4. Inflation of pulmonary atelectasis

lsmusis.lsmuni.lt/Klausimai/Spausdinti?Length=0?Kalba=EN&KategorijaId=143&Kalbos_input=EN&Kalbos=EN&KategorijaEn_input=Thoracic+surgery… 4/7
3/27/2019 LSMUSIS
41.30 What is the reaction of lower esophageal sphincter during swallowing in patients with achalasia:
1. Contraction (spasm)
2. Relaxation
3. Coutmetion and relaxation
4. Lower esophageal sphincter does not react to swallowing

41.31 What are early symptoms or syndromes of esophageal cancer:


1. Complete dysphagia caused by esophageal obstruction
2. Intermittent dysphagia
3. Aspiration
4. Predysphagia

41.32 What is the main methods for percutaneous (transthoracic) drainage of lung abscess:
1. Puncture of abscess cavity
2. Abscessoscopy
3. Drainage of abscess cavity
4. Catheterisation of abscess cavity (Monaldi method)

41.33 What are endobronchial methods for treatment drainage of an acute lung abscess and bronchectasis:
1. Bronchoscopy
2. Bronchial catheterization
3. “Microtracheostomy”
4. Bronchial brushing

41.34 What maneuvers are performed during the procedure of local treatment of lung abscess or bronchiectasis:
1. Aspiration of sputum
2. Endobronchial lavage (washing)
3. Endobronchial injection of fibrinolytic and broncholytic drugs
4. Endobronchial injection of antibiotics

41.35 Complete relaxation of left hemidiaphragm and paralysis of phrenic nerve developed due to penetrating
neck injury. What methods could be effective to restore diaphragmatic function:
1. Double-layered diaphragmatic plication
2. Repair of phrenic nerve
3. Lung ventilation
4. Kinesitherapy and phrenic nerve pacing
5. Tracheostomy

41.36 What statements concerning Barret’s esophagus are correct:


1. Squamous esophageal epithelium is replaced by cylindric gastric epithelium
2. Almost always it is malignant lesion
3. As usually distal portion of the esophagus is involved
4. Pneumodilatation is used for treatment

IV type tasks. Choose only one best answer

41.37 10-year-old boy yesterday accidentally aspirated an plastic foreign body. After this accident boy had severe
cough and dyspnea. Today he feels better. Chest roentgenograms revealed the emphysema of upper and middle
lobe and atelectasis of lower lobe of the right lung. Which bronchus is the foreign body in:
A. Right upper lobar bronchus
B. Intermediary bronchus
C. Middle lobar bronchus
D. Right lower lobar bronchus
E. Left main bronchus

41.38 10-year-old boy yesterday accidentally aspirated an plastic foreign body. After this accident boy had severe
cough and dyspnea. Today he feels better. Chest roentgenograms revealed the emphysema of upper and middle
lobe and atelectasis of lower lobe of the right lung. An attempt to remove this foreign body using
fibrobronchoscope was not successful. What other method should be used first in this situation:
A. Transnasal bronchial catheterization
B. Rigid bronchoscopy
C. Thoracotomy and bronchotomy

lsmusis.lsmuni.lt/Klausimai/Spausdinti?Length=0?Kalba=EN&KategorijaId=143&Kalbos_input=EN&Kalbos=EN&KategorijaEn_input=Thoracic+surgery… 5/7
3/27/2019 LSMUSIS
D. Tracheostomy
E. Mediastinoscopy

41.39 Using roentgenoscopy was revealed solitary asymptomatic small (1,5 cm in diameter), peripherial mass in
48-year-old man What diseases can be suspected:
A. Inflammatory infiltrative
B. Sarcoidosis
C. Foreign body
D. Pulmonary carcinoma

41.40 Using roentgenoscopy was revealed solitary asymptomatic small (1,5 cm in diameter), peripherial mass in
48-year-old man. Wich of the following tests is most informative to verify diagnosis:
A. Dynamic roentgenologic observation
B. Bronchoscopy
C. Bronchial catheterization
D. Percutaneous needle aspiration biopsy
E. Explorative thoracotomy

41.41 Using roentgenoscopy was revealed solitary asymptomatic small (1,5 cm in diameter), peripherial mass in
48-year-old man. Squamous pulmonary cancer without involvement to tracheobronchial lymphnodes was
diagnosed. What is the tactics of treatment:
A. Radiation therapy
B. Segmentectomy
C. Lobectomy
D. Chemiotherapy

41.42 Using roentgenoscopy was revealed solitary asymptomatic small (1,5 cm in diameter), peripherial mass in
48-year-old man. What would be optimal tactics for treatment, if on operation pulmonary tuberculoma was verified:
B. Enucleation of tuberculoma with subsequent antituberculous drug therapy
C. Lobectomy with subsequent antituberculous drug therapy
D. Pulmonectomy with subsequent antituberculous drug therapy
E. Antituberculous drug therapy alone

41.43 Using roentgenoscopy was revealed solitary asymptomatic small (1,5 cm in diameter), peripherial mass in
48-year-old man. What is the most acceptable method for treatment of pulmonary hamartoma:
A. Enucleation
B. Segmentectomy
C. Lobectomy
D. Pulmonectomy
E. Radiation therapy

41.44 60-year-old man complains of severe cough. During the last 6 months he was ill 3 times with pneumonia.
Treatment undertaken for pneumonia was intensive. Blood-streaked expectoration was observed few times. Chest
roentgenography revealed that overshadow in third segment of right lung decreased in size. What diseases should
be suspected at first:
A. Pneumonia
B. Pulmonary tuberculosis
C. Lung cancer
E. Chronic bronchitis

41.45 60-year-old man complains of severe cough. During the last 6 months he was ill 3 times with pneumonia.
Treatment undertaken for pneumonia was intensive. Blood-streaked expectoration was observed few times. Chest
roentgenography revealed that overshadow in third segment of right lung decreased in size. Which of the following
tests is most likely to establish the final diagnosis:
A. Computed tomography
B. Thoracocoscopy
C. Bronchoscopy
D. Mediastinascopy
E. Immunologic skin test

41.1 - C 41.2 - E 41.3 - D 41.4 - D 41.5 - E 41.6 - D 41.7 - D 41.8 - D


41.9 - C 41.10 - E 41.11 - E 41.12 - D
lsmusis.lsmuni.lt/Klausimai/Spausdinti?Length=0?Kalba=EN&KategorijaId=143&Kalbos_input=EN&Kalbos=EN&KategorijaEn_input=Thoracic+surgery… 6/7
3/27/2019 LSMUSIS
41.13 41.14 41.15 41.16 41.17 41.18
1-C 1-D 1-C 1-E 1-D 1-B
2-B 2-B 2-C 2-E 2-A 2-C
3-E 3-C 3-C 3-B 3-A
4-A 4-B 4-A 4-D 4-D
5-D 5-B 5-E 5-B 5-E
41.19 41.20 41.21 41.22 41.23 41.24 41.25 41.26
1 1 4 4 4 2 1 1
3 2 4 3 2
3 3
4
41.27 41.28 41.29 41.30 41.31 41.32 41.33 41.34
1 1 1 4 2 4 1 1
3 3 2 4 2 2
3 3 3
4 4
41.35 41.36
2 1
4 3
41.37 - D 41.38 - B 41.39 - D 41.40 - E 41.41 - C 41.42 - B 41.43 - A 41.44 - C
41.45 - C

lsmusis.lsmuni.lt/Klausimai/Spausdinti?Length=0?Kalba=EN&KategorijaId=143&Kalbos_input=EN&Kalbos=EN&KategorijaEn_input=Thoracic+surgery… 7/7
3/27/2019 LSMUSIS

Neurosurgery
I type tasks. Choose only one best answer

44.1 The most characteristic sign of increased intracranial pressure is:


A. Equililibrium instability
B. Morning headaches with nausea et vomiting
C. Sleepness
D. Impaired visual aquity
E. Evening headaches

44.2 What sign is the most reliable indicator of the side location of intracranial haematoma:
A. Dilated pupil is usually on haematoma side
B. Signs of central paralysis in limbs are usually on haematoma side
C. Dilated pupil is usually contralateral to haematoma
D. Bilateral pyramidal signs of central paralysis in limbs
E. Ptosis is usually contralateral to haematoma

44.3 Which is the most important sign of the brain compression:


A. Bradycardia
B. Anisocoria
C. Severe headache
D. Deteriorating of consciousness
E. Hemiplegia

II type tasks. For each numbered item,selct the one lettered heading that is most closely asssciated with it

44.4 Which operation is most appropriate for the treatment of following brain tumours:
B 1 - Stereotactic aspiration of cyst.
E 2 - Subtotal extirpation of the tumour.
D 3 - Transsphenoidal removal of the tumour.
C 4 - CSF shunting operation
A 5 - Radical extirpation of the tumour
A. Meningioma.
B. Cystic craniopharyngioma
C. Brainstem glioma with obstructive hydrocephalus
D. Pituitary adenoma
E. Glioblastoma in the frontal lobe of the brain.

44.5 What is an appropriate surgical procedure for the treatment of following diseases:
E 1 - Arterial extra-intracranial bypass
D 2 - Thrombintimectomia
A 3 - Endovascular embolisation
B 4 - Clipping of aneurism.
C 5 - Craniotomia and evacuation of haematoma.
A. Carotido-cavernous fistula.
B. Saccular aneurism of cerebral artery
C. Lateral haemorrhagic stroke with intracerebral haematoma
D. Thrombosis of carotid artery in the neck
E. Thrombosis of MCA (medial cerebral artery).

44.6 Which clinical signs are characteristic for different brain tumors:
D 1 - Shwanoma (neurinoma) of VIII-th cranial nerve
A 2 - Craniopharyngioma
B 3 - Glioma of optical chiasm
C 4 - Cerebellar medulloblastoma
E 5 - Glioblastoma
A. Hipopituitarism
B. Disturbances of eye fields and vision
C. Cerebellar ataxia, hipertensive headaches
D. Hearing disturbances
E. Disturbances of orientation, motor and speech deficit
lsmusis.lsmuni.lt/Klausimai/Spausdinti?Length=0?Kalba=EN&KategorijaId=146&Kalbos_input=EN&Kalbos=EN&KategorijaEn_input=Neurosurgery&K… 1/5
3/27/2019 LSMUSIS
44.7 What is the level of spinal cord damage, according to the following neurological symptomes:
B 1 - Symmetrical disappearance of motor activity and sensation in legs, urinary incontinence
D 2 - Motor activity and sensation preserved in shoulders but absent in arms and legs, retention of urine
A 3 - Assymetrical disorders of movements and sensations, pain in legs with urinary retention
C 4 - Movement and sensation of arms are preserved but they are absent in legs urinary retention
E 5 - Tetraplegia with anesthesia, urinary retention and disorders of respiration
A. Cauda equina
B. Conus, epiconus
C. Thoracic level
D. Lower cervical spine
E. Upper cervical spine

III type tasks. For each question there is one or more correct answers:
A – if correct answers are 1,2,3
B – if correct answers are 1 and 3
C – if correct answers are 2 and 4
D – if correct answer is 4
E – if correct are all answers above

44.8 What is the management of acute intracranial haematoma with signs of brain compression:
1. Urgent intubation and hyperventilation
2. Bolus of mannitol
3. Cardiovascular examination to evaluate the risk of anesthesia
4. Urgent craniotomy with evacuation of the haematoma
5. Ventriculodrainage for decrease of intracranial pressure

44.9 What are the most appropriate diagnostic methods for differentiation between intracranial haematoma and
brain oedema:
1. Angiography
2. Lumbar puncture
3. Electroencephalography
4. CT or MRI scan
5. Transcranial dopplerography

44.10 What can help in diagnostics vestibulocochlear neurinoma:


1. Hearing test (audiogramm)
2. MRI scan
3. Brainstem evoked potentials
4. Eye fundus examination
5. Transcranial dopplerography

44.11 In what cases of intervertebral disk herniation urgent surgical treatment is indicated:
1. Disorders of function of pelvic organs
2. The patient is in compulsary position, he cannot sleep due to pain
3. Foot drop paresis
4. There is little therapeutic effect during a weak treatment from the beginning of radiculopathy
5. Severe scoliosis, even in the horizontal position

44.12 What are the characteristic features of subdural haematoma on CT:


1. Hyperdense zone beneath the skull bone
2. Semilunar zone under the skull bone
3. Lateral shift of ventricular system and midline structures of the brain
4. Isodense zone under the skull bone
5. Hypodense zone under the skull bone

44.13 Which pathologic processes can cause malabsorbtion of CSF and hydrocephalus:
1. Subarachnoidal haemorrhage
2. Meningoencephalitis
3. Severe traumatic brain injury
4. Cerebral atrophy
5. Lacunar ischemic stroke

lsmusis.lsmuni.lt/Klausimai/Spausdinti?Length=0?Kalba=EN&KategorijaId=146&Kalbos_input=EN&Kalbos=EN&KategorijaEn_input=Neurosurgery&K… 2/5
3/27/2019 LSMUSIS
44.14 When one can perform stereotactic operations on the brain:
1. For treatment of extrapyramidal hyperkinesis
2. For treatment of pain
3. For biopsy of of tumors deeply located in the brain
4. For removal of cerebral AVM
5. For removal of pituitary and other tumors in sellar region

IV type tasks. Choose only one best answer

44.15 53 year old gentleman complains on episodic jerks of the right hand of 10-15 minute duration, with
simultaneous difficulty to speak, headache, but clear consciousness during event. Seizures began 2 weeks ago.
It's difficult to patient to concentrate himself, memory is impaired. Morning headaches and fatigue after night sleep
used to be often during last 4 months. He has suffered motorcycle accident earlier with loss of consciousness.
Examination revealed weakness of his right arm, shallowness of the right nasolabial fold, normal lower limb
strength, tendon hyperreflexia on the right side, positive Babinski sign on the right. What disease can be
suspected:
A. Focal posttraumatic epilepsy
B. Arteriovenous malformation
C. Frontotemporal glioma
D. Chronic subdural haematoma
E. Obstruction of the internal carotid artery in the neck

44.16 53 year old gentleman complains on episodic jerks of the right hand of 10-15 minute duration, with
simultaneous difficulty to speak, headache, but clear consciousness during event. Seizures began 2 weeks ago.
It's difficult to patient to concentrate himself, memory is impaired. Morning headaches and fatigue after night sleep
used to be often during last 4 months. He has suffered motorcycle accident earlier with loss of consciousness.
Examination revealed weakness of his right arm, shallowness of the right nasolabial fold, normal lower limb
strength, tendon hyperreflexia on the right side, positive Babinski sign on the right. Which is the most valuable
diagnostic method in this situation:
A. Lumbar puncture
B. Electroencephalography
C. MRI scan
D. Transcranial dopplerography
E. Brainstem evoked potentials

44.17 Diagnosis of lumbar radiculitis has been established for 41 year old gentleman 10 years ago. There were
episodes of pain, irradiating to one or both legs, slight bilateral feet weakness has developed. Patient was treated
in rehabilitation department, when he slipped on his way to the swimming pool, fell down, beated his buttocks.
Severe lumbar and leg pain together with urination disturbances has developed. Examination revealed positive
Lasegue signs and absent Achilles' tendon reflexes bilaterally, hypaesthesia of the dorsolateral surface of the left
and posterolateral surface of the right feet. What is the most probable disease:
A. Epidural haematoma of lumbar region
B. Acute posttraumatic coccygeal pain (coccygodynia)
C. Bilateral sciatic neuritis
D. Lumbar medial disc herniation
E. Lumbar myositis

44.18 Diagnosis of lumbar radiculitis has been established for 41 year old gentleman 10 years ago. There were
episodes of pain, irradiating to one or both legs, slight bilateral feet weakness has developed. Patient was treated
in rehabilitation department, when he slipped on his way to the swimming pool, fell down, beated his buttocks.
Severe lumbar and leg pain together with urination disturbances has developed. Examination revealed positive
Lasegue signs and absent Achilles' tendon reflexes bilaterally, hypaesthesia of the dorsolateral surface of the left
and posterolateral surface of the right feet. Which method is optimal for acurate diagnosis:
A. Electomyography
B. Myelography
C. Lumbar CT scan
D. Lumbar X-ray examination
E. Somatosensory evoked potentials

44.19 60 year old gentleman was found lying near his broken bicycle on the suburban road. Conscious, doesn't
remembering anything about the event. He was unable to move his legs as well as urinate. Examination revealed:

lsmusis.lsmuni.lt/Klausimai/Spausdinti?Length=0?Kalba=EN&KategorijaId=146&Kalbos_input=EN&Kalbos=EN&KategorijaEn_input=Neurosurgery&K… 3/5
3/27/2019 LSMUSIS
BP-120/80 mm Hg, PR-72/min, complete anaesthesia downwards from umbilicus, absence of patellar and Achilles'
tendon reflexes bilaterally, no Babinski signs or plantar reflex as well. What injury is mostly suspected:
A. Lower cervical spine fracture
B. Fracture of sacrum
C. Upper cervical spine fracture
D. Lumbar spine fracture
E. Middle thoracic spine fracture

44.20 60 year old gentleman was found lying near his broken bicycle on the suburban road. Conscious, doesn't
remembering anything about the event. He was unable to move his legs as well as urinate. Examination revealed:
BP-120/80 mm Hg, PR-72/min, complete anaesthesia downwards from umbilicus, absence of patellar and Achilles'
tendon reflexes bilaterally, no Babinski signs or plantar reflex as well. What is the most important diagnostic
method to determine the tactics of treatment:
A. Somatosensory evoked potentials
B. Lumbar puncture
C. Spinal CT scan
D. Myelography
E. Lumbar puncture with CSF-dynamic tests

44.21 58 year old lady complains of gradually impairing hearing of her right ear. She heard buzzing, whistling in
this ear for some period before. Equilibrium is disturbed with sudden movements of the head; she feels a sort of
"traction" to the right, while going. A story of purulent otitis was in childhood, but she doesn't remember which ear
was sick. Examination revealed total deafness of the right ear, perforation of the tympanum, caloric tests couldn't
be performed therefore; asymetria of the right side of the face, especially while smiling. No other neurologic deficit.
What disease is most probable:
A. Tumour (neurinoma) of VIII nerve
B. Acoustic neuritis
C. Chronic otitis
D. Tumour of cerebellar hemispher
E. Pontine tumour

44.22 58 year old lady complains of gradually impairing hearing of her right ear. She heard buzzing, whistling in
this ear for some period before. Equilibrium is disturbed with sudden movements of the head; she feels a sort of
"traction" to the right, while going. A story of purulent otitis was in childhood, but she doesn't remember which ear
was sick. Examination revealed total deafness of the right ear, perforation of the tympanum, caloric tests couldn't
be performed therefore; asymetria of the right side of the face, especially while smiling. No other neurologic deficit.
What diagnostic method is most valuable in this situation:
A. Electroencephalography
B. Facial electromyography
C. Brainstem evoked potentials
D. Investigation of vestibular function
E. MRI scan

44.23 5 years girl during 2 last months presented with intensive tremor in her right arm. She looked neglectful to
her parents, she often harmed herself when going through the door or through the other narrow space in the room.
The girl started to run clumsy this month as well as she complained with morning headaches last 2-3 weeks.
Severe hedaches with nausea and vomiting began this morning. On admission in the hospital she was sluggish.
Nausea and vomiting have repeated once more. There was stiffness of the neck, impaired coordination with the
intensive tremor in the right arm and leg unstability of equilibrium. What disease can you suspect in this case?
A. Tumour of the left hemispherium of the brain.
B. Sclerosis disseminate
C. Tumour of the cerebellar hemisphere.
D. Hydrocephalus.
E. Meningitis.

44.24 5 years girl during 2 last months presented with intensive tremor in her right arm. She looked neglectful to
her parents, she often harmed herself when going through the door or through the other narrow space in the room.
The girl started to run clumsy this month as well as she complained with morning headaches last 2-3 weeks.
Severe hedaches with nausea and vomiting began this morning. On admission in the hospital she was sluggish.
Nausea and vomiting have repeated once more. There was stiffness of the neck, impaired coordination with the
intensive tremor in the right arm and leg unstability of equilibrium. Which are the most informative diagnostic
methods in this case:
lsmusis.lsmuni.lt/Klausimai/Spausdinti?Length=0?Kalba=EN&KategorijaId=146&Kalbos_input=EN&Kalbos=EN&KategorijaEn_input=Neurosurgery&K… 4/5
3/27/2019 LSMUSIS
A. SPECT.
B. Lumbar puncture.
C. Electromyography.
D. Head CT or MRI scan.
E. BERA

44.25 A man of approximately 30 years old was found lying in the meadow near the road. A broken motorcycle
was near him. He was conscious, but dezoriented, restless. After some period he complained with headache and
started vomiting and became drowsy again. During the 1 and half an hour he was transferred to the hospital. On
admission the patient ws unconsciuous already, with loosed contact, dilated left pupil, both eyes directed to the
left, defensive movements with the right arm to the painful stimulus. Bradicardia. Normal breathing. What kind of
head injury could be diagnosed:
A. Commotion of the brain
B. Diffuse axonal injury
C. Acute traumatic epidural haemorrhage in the left temporoparietal region
D. Fracture of the scull base in the middle fossa.
E. Acute subdural haemorrhage on the right cerebral hemisherium.

44.26 A man of approximately 30 years old was found lying in the meadow near the road. A broken motorcycle
was near him. He was conscious, but dezoriented, restless. After some period he complained with headache and
started vomiting and became drowsy again. During the 1 and half an hour he was transferred to the hospital. On
admission the patient ws unconsciuous already, with loosed contact, dilated left pupil, both eyes directed to the
left, defensive movements with the right arm to the painful stimulus. Bradicardia. Normal breathing. What methods
of treatment can you propose:
A. Urgent surgery: craniotomy and evacuation of epidural haematoma.
B. Urgent external ventricular drainage.
C. Bifrontal decompressive cranioectomy.
D. Bolus of manitol and further conservative treatment.
E. Urgent intubation of the patient, sedative, analgetics, myorelaxants; monitoring of vital functions..

44.1 - B 44.2 - A 44.3 - D


44.4 44.5 44.6 44.7
1-B 1-E 1-D 1-B
2-E 2-D 2-A 2-D
3-D 3-A 3-B 3-A
4-C 4-B 4-C 4-C
5-A 5-C 5-E 5-E
44.8 44.9 44.10 44.11 44.12 44.13 44.14
4 4 1 1 1 1 1
2 2 2 2 2
3 3 3 3 3
4
5
44.15 - C 44.16 - C 44.17 - D 44.18 - C 44.19 - E 44.20 - C 44.21 - A 44.22 - E
44.23 - C 44.24 - D 44.25 - C 44.26 - A

lsmusis.lsmuni.lt/Klausimai/Spausdinti?Length=0?Kalba=EN&KategorijaId=146&Kalbos_input=EN&Kalbos=EN&KategorijaEn_input=Neurosurgery&K… 5/5
3/27/2019 LSMUSIS

Anesthesiology
I type tasks. Choose only one best answer

5.1 Drug, administered for premedication before anaesthesia in order to decrease secretion of saliva:
A. Promedol
B. Diazepam
C. Atropine
D. Midazolam

5.2 Using fentanyl, most common complication is:


A. Asystolia
B. Depression of ventilation
C. Tachycardia
D. Allergic reaction

5.3 Muscular relaxation of the shortest duration (3-5 min) is produced by:
A. Atracurium (Tracrium)
B. Rocuronium (Esmeron)
C. Succinylcholine
D. Pipecuronium (Arduan)

5.4 Which physiologic function should be optimised in the first order following anaesthesia :
A. Consciuosness
B. Motor function
C. Breathing

5.5 The indication for the use of succinylcholine is:


A. Muscular relaxation during the operation
B. Treatment of bronchospasm during anaesthesia
C. Treatment of laringeal spasm during anaesthesia
D. Traetment of malignant hiperthermia

5.6 Action of non-depolarizing muscular relaxants is antagonized by:


A. Naloxone
B. Proserine
C. Atropine

5.7 The role of atropine in decurarisation:


A. To potentiate the action of proserine
B. To avoid muscarinic effects of proserine
C. To avoid depression of ventilation

5.8 Which effect is characteristic for the intravenous anaesthetic ketamine:


A. Arterial hypotension
B. Analgesia
C. Muscular relaxation

5.9 Which effect is unusual for the opioid fentanyl:


A. Depression of ventilation
B. Nausea and vomiting
C. Significant cardiovascular depression

5.10 the maximal concentration of nitrous oxide in mixture with oxygen:


A. 50%.
B. 70%
C. 90%

5.11 Which intravenous anaesthetic is of the shortest duration:


A. Thiopental
B. Propofol
C. Ketamine

lsmusis.lsmuni.lt/Klausimai/Spausdinti?Length=0?Kalba=EN&KategorijaId=107&Kalbos_input=EN&Kalbos=EN&KategorijaEn_input=Anesthesiology&… 1/4
3/27/2019 LSMUSIS
5.12 Which intravenous anaesthetic is the agent of choice for anaethesia induction in patients with cardiovascular
disease:
A. Etomidate
B. Ketamine
C. Propofol
D. Thiopental

5.13 What indicator is best used to assess the tracheal intubation risk:
A. ASA class
B. Dimensions of the oral cavity
C. Mallampati class

5.14 During regional anaesthesia the sensations are blocked in the following order:
A. Vasodilation (autonomic fibers ); temperature; pain; deep pressure sensations; motor fibers.
B. Temperature; vasodilation; deep pressure; pain; motor fibers.
C. Vasodilation; motor fibers; deep pressure; temperature; pain.

5.15 Specific bet rare complication of spinal anaesthesia is:


A. Depression of ventilation
B. Postoperative nausea and vomiting
C. Headache
D. urinary retention

5.16 The safest approach for spinal anaesthesia (a patient of any age) is the intervertebral space between:
A. Th12-L1
B. L1-L2
C. L2-L3
D. L3-L4

5.17 The most common cause of hypotension during spinal anaesthesia is:
A. Cardiotoxic effect of local anaesthetic agent
B. Arteriolar and venular dilation
C. Depression of cardiac contractility by local anaesthetic agent
D. Hypoventilation

5.18 Main advantage of epidural over spinal anaesthesia is:


A. Simplicity of technique
B. Dura mater is not punctured
C. Less negative effect on cardiovascular system
D. Possibility to produce analgesia at any level of the spinal cord

5.19 Local anaesthetic agent of the greatest potency and longest duration of action is:
A. Lidocaine
B. Bupivacaine
C. Ropivacaine
D. Procaine

II type tasks. For each numbered item,selct the one lettered heading that is most closely asssciated with it

5.20 Role of these drugs in premedication:


1 - Atropine
2 - Diazepam
3 - 0ndasetron
4 - Morphine
A. Reduces secretion of saliva
B. Induces analgesia
C. Reduces nausea and vomiting
D. Reduces anxiety

5.21 Which anaesthetic agent should be avoided in the following clinical situations:
C 1 - Hypertonic disease
A 2 - Status asthmaticus
lsmusis.lsmuni.lt/Klausimai/Spausdinti?Length=0?Kalba=EN&KategorijaId=107&Kalbos_input=EN&Kalbos=EN&KategorijaEn_input=Anesthesiology&… 2/4
3/27/2019 LSMUSIS
C 3 - Eclampsia
B 4 - Hyperkalemia
A. Isoflurane
B. Succinylcholine
C. Ketamine

5.22 Which side effect is common for these anaesthetic agents:


1 - Fentanyl
2 - Propofol
3 - Ketamine
A. Bradycardia
B. Depression of ventilation
C. Hallucinations and motor agitation

III type tasks. For each question there is one or more correct answers:
A – if correct answers are 1,2,3
B – if correct answers are 1 and 3
C – if correct answers are 2 and 4
D – if correct answer is 4
E – if correct are all answers above

5.23 Role of atropine in premedication:


1. Reduction of secretion of saliva
2. Potentiation of action of anaesthetic agents
3. Inhibibition of autonomic reflexes
4. Prevention of nausea and vomiting

5.24 Ketamine is contraindicated in:


1. Bronchial asthma
2. Arterial hypertension
3. Hepatic insufficiency
4. Eclampsia

5.25 Side effects of ketamine anaesthesia:


1. Systolic hypertension
2. Tachycardia
3. Motor agitation
4. Depression of ventilation

5.26 The side effect common during i/v fentanyl injection:


1. Hypotension
2. Arrhythmia
3. Allergic reaction
4. Depression of ventilation
5. Seizures

5.27 Depression of ventilation is caused by:


1. Thiopental
2. Propofol
3. Fentanyl
4. Diazepam
5. Ketamine

5.28 Drugs administered for prevention of postoperative nausea and vomiting are:
1. Metoclopramide
2. Droperidol
3. Ondansetron
4. Diazepam
5. Morphine

IV type tasks. Choose only one best answer

lsmusis.lsmuni.lt/Klausimai/Spausdinti?Length=0?Kalba=EN&KategorijaId=107&Kalbos_input=EN&Kalbos=EN&KategorijaEn_input=Anesthesiology&… 3/4
3/27/2019 LSMUSIS
5.29 Fifty-year old male patient is to undergo stomach (ventricular) resection. Arterial blood pressure is 160/90
mmHg. Which anaesthetic agent for the induction of anaesthesia should be avoided:
A. Propofol
B. Ketamine
C. Diazepam
D. Midazolam

5.30 The patient is to undergo emergency surgery on his leg. Ten years ago this patient had undergone cardiac
surgery for mitral valve prothesis. Since then he is on anticoagulant medications. Presently, his cloting capacity is
decreased 2,5 times below normal. What anaesthesia method would be contraindicated for this patient:
A. General anaesthesia with tracheal intubation
B. Intravenous anaesthesia
C. Epidural anaesthesia

5.31 A female patient aged 37 is scheduled for varicectomia. She also suffers myasthenia. A method of
anaesthesia which should be avoided is:
A. Inhalational anaesthesia with sevoflurane under spontaneous breathing
B. Endotracheal anaesthesia under muscular relaxation
C. Spinal anaesthesia

5.32 Forty-five year old male patient is to undergo change of wound dressings under general anaesthesia. In
addition, the patient had been treated in psychiatric hospital 2 years ago. Which anaesthetic agent should be
avoided during anaesthesia
A. Propofol
B. Fentanyl
C. Ketamine
D. Thiopental

5.33 A patient is operated on. Anaesthesia - endotracheal. Nitrous oxide and sevoflurane are used for
maintenance, fentanyl - for analgesia. In 15 minutes after intubation arterial blood pressure decreased from 130/80
to 70/50 mmHg. The first action to be taken is:
A. Increase speed of intavenous fluid infusion
B. Decrease the inhalational concentration of sevoflurane
C. Administer vasopressors

5.34 A patient is operated on for occlusion of femoral artery. He received spinal anaesthesia. In 20 minutes after
intrathecal injection of 20 mg bupivacaine (0,5 proc) hypotension developed, arterial blood pressure decreased
from 110/70 to 60/40 mm/Hg. The medicine of choice for the prompt correction of hypotension is:
A. Natrium nitropruside
B. Dopamine
C. Ephedrine
D. Epinephrine (Adrenaline)

5.1 - C 5.2 - B 5.3 - C 5.4 - C 5.5 - C 5.6 - B 5.7 - B 5.8 - B


5.9 - C 5.10 - B 5.11 - B 5.12 - A 5.13 - C 5.14 - A 5.15 - C 5.16 - D
5.17 - B 5.18 - D 5.19 - B
5.20 5.21 5.22
1-A 1-C 1-B
2-D 2-A 2-A
3-C 3-C 3-C
4-B 4-B
5.23 5.24 5.25 5.26 5.27 5.28
1 2 1 4 1 1
3 4 2 2 2
3 3 3
5.29 - B 5.30 - C 5.31 - B 5.32 - C 5.33 - A 5.34 - C

lsmusis.lsmuni.lt/Klausimai/Spausdinti?Length=0?Kalba=EN&KategorijaId=107&Kalbos_input=EN&Kalbos=EN&KategorijaEn_input=Anesthesiology&… 4/4
3/27/2019 LSMUSIS

Mix
I type tasks. Choose only one best answer

21.1 Which blood vessels must be ligatured in a case of extra-uterine pregnancy and a rupture of the Fallopian
tube:
A. a. uterina ir a. vaginais
B. a. ovarica ir a. obturatoria
C. a. vaginalis ir a. pudenda interna
D. a. uterina ir a. ovarica

21.2 Into which lymph nodes does lymph flow from the lateral part of the thorax:
A. Nodi pectorales
B. Nodi pulmonales
C. Nodi intercostales
D. Nodi tracheobronhiales

21.3 Which statement about the accessory pancreatic duct is incorrect:


A. Accessory duct starts in the head of the pancreas
B. In some cases the accessory duct may be a branch of the main pancreatic duct
C. Accessory pancreatic duct is shorter than the main pancreatic duct
D. Accessory duct opens as the papilla duodeni minor
E. The end of the accessory pancreatic duct joins the bile duct, ductus choledochus

21.4 Gases such as oxygen and carbon dioxide cross the plasma membrane by:
A. Primary active transport
B. Passive diffusion through the lipid bilayer
C. A specific carrier protein
D. Secondary active transport
E. Facilitated diffusion

21.5 Dietary fat, after being processed, is extruded from the mucosal cells of the gastrointestinal tract into
lymphatic ducts in the form of
A. Monoglycerides
B. Diglycerides
C. Triglycerides
D. Chylomicrons
E. Free fatty acids

21.6 Nissl bodies are composed of:


A. Synaptic vesicles and acetylcholine
B. Polyribosomes and rough endoplasmic reticulum
C. Lipoprotein and melanin
D. Neurofilaments and microtubules
E. Smooth endoplasmic reticulum and mitochondria

21.7 Which of the following statements about the perineurium is true:


A. It is a fascia surrounding many bundles of nerve fibers
B. It consists in part of epithelioid cells that surround a bundle (fascicle) of nerve fibers
C. It is a thin layer of reticular fibers covering individual nerve fibers
D. It is a fascia that excludes macromolecules and forms the external coat of nerves

21.8 Which of the following statements apply to metastases:


A. They are histologically different from the primary tumor
B. They spread from the primary site to only one other site
C. They may occur with either malignant or benign neoplasms
D. They indicate a poorer prognosis than when they have not developed

21.9 All of the following are manifestations of end-stage renal failure EXCEPT:
A. Congestive heart failure
B. Hypokalemia
C. Pulmonary edema

lsmusis.lsmuni.lt/Klausimai/Spausdinti?Length=0?Kalba=EN&KategorijaId=123&Kalbos_input=EN&Kalbos=EN&KategorijaEn_input=Mix&Kategorija… 1/38
3/27/2019 LSMUSIS
D. Anemia

21.10 Which of the following statements is TRUE for creatine kinase:


A. Brain and skeletal muscle cells have the same isoenzyme of creatine kinase
B. Active creatine kinase is tetrameric enzyme
C. Heart cells have not creatine kinase
D. The presence of creatine kinase in the plasma of healthy individuals basically is due to release of
enzyme from skeletal muscle cells

21.11 Which of the following hormones suppresses reabsorbtion of both water and Na ions in kidney:
A. Vasopressin
B. Aldosterone
C. Natriuretic peptide
D. Somatotropin

21.12 What clinical syndromes are associated with Helicobacter pylori infection:
A. Haemolytic uremic syndrome
B. Bloody - diarrhea
C. Food borne intoxication
D. Cancer of large intestine
E. Gastric and duodenal ulcers

21.13 Why Clostridium difficile cause diarrhea after antibiotic therapy:


A. Antibiotics can be contaminated with spores of Clostridium difficile.
B. Antibiotics activate tox-genes of bacteria
C. Antibiotics injury epithelium of intestines and increase sensitivity to bacterial toxin
D. Antibiotics disrupt the normal gut flora and permit the overgrowth of Clostridium difficile
E. Non of the above is correct, because Clostridium difficile don’t cause antibiotic-associated diarrhea

21.14 The most common side effect of amlodipine is:


A. Myocardium depression
B. Dry cough
C. AV Heart block
D. Constipation
E. Peripheral oedema

21.15 Which proposition describes drug development time and costs best:
A. 100.000 EUR/0.2% sales/1 month
B. 1.000.000 EUR/0.6% sales/6 month
C. 10.000.000 EUR/6% sales/6 years
D. 900.000.000 EUR/14-16% sales/14 years

21.16 Bioavailability is a term that defines amount of medicine that appears into:
A. Stomach
B. Liver
C. Systemic circulation
D. All mentioned places
E. None of mentioned

21.17 In which phase and which subsequent division does the crossing-over occur:
A. Metaphase II
B. Prophase I
C. Prophase II
D. Metaphase I

21.18 Penetrance is defined as


A. Intensity of a trait determined by a gene or genotype.
B. Percentage of individuals with a given genotype who exhibit the corresponding trait.
C. Repeated mutations of the same locus producing three or more alleles.
D. Two, three or more phenotypic effects of a single gene.

21.19 Part of general air pollution devoted to transport pollution is:


A. 20-30 %
lsmusis.lsmuni.lt/Klausimai/Spausdinti?Length=0?Kalba=EN&KategorijaId=123&Kalbos_input=EN&Kalbos=EN&KategorijaEn_input=Mix&Kategorija… 2/38
3/27/2019 LSMUSIS
B. 60-70 %
C. 40-50 %
D. 10-30 %

21.20 What must be school class lighting level from the luminescence lamps:
A. Not less than 100 lx
B. Not less than 150 lx
C. Not less than 200 lx
D. Not less than 300 lx
E. Not less than 400 lx

21.21 Capillary pulsation is seen in:


A. Aortic hypertension
B. Raynaud‘s phenomenon
C. Aortic regurgitation
D. Healthy people

21.22 Vesicular breath sounds are:


A. Best heard on inspiration and on the beginning of expiration
B. Best heard on expiration and on the beginning of inspiration
C. Evenly heard on inspiration and expiration
D. Better heard on expiration than on inspiration

21.23 Which of the following definitions of allergic rhinitis is correct:


A. Allergic rhinitis is a symptomatic disorder of the nose induced by and T cell mediated inflammation after
allergen exposure of the membranes of the nose.
B. Allergic rhinitis is a symptomatic disorder of the nose induced by and IgE mediated inflammation after
allergen exposure of the membranes of the nose.
C. Allergic rhinitis is a symptomatic disorder of the nose inflammation induced by the irritants.
D. Allergic rhinitis is a symptomatic disorder of the nose inflammation induced by bacteria and allergens.

21.24 What is characteristic for sarcoidosis stage IV:


A. Bilateral hilar adenopathy
B. B. Bilateral hilar adenopathy and parenchymal involvement
C. Pulmonary fibrosis
D. Parenchymal involvement with shrinking adenopathy

21.25 Which patients belong to the high risk group for infective endocarditis:
A. Having atrial septal defect
B. Having mitral valve’s prolaps without hemodinamical defects
C. Having artificial valves
D. Having hypertrophic cardiomyopathy

21.26 What symptoms aren’t typical acute myocardium infarction with ST elevation:
A. ECG without dynamics
B. Elevated troponin I
C. Prolonged angina pectoris type pain
D. Akynetic zones

21.27 Which one of following criteria is most relevant for chronic hepatitis C:
A. Prothrombin time
B. antiHCV (+)
C. Ascites
D. HBsAg (+)

21.28 Which drug combination can be used in the management of chronic active hepatitis C:
A. Prednisolone and Azathioprine
B. Interferon and Lamivudine
C. Pegilated interferon and Ribavirin
D. Prednisolone and Ribavirin

21.29 51.32. Which clinical situation is the most typical for hypertensive nephropathy:

lsmusis.lsmuni.lt/Klausimai/Spausdinti?Length=0?Kalba=EN&KategorijaId=123&Kalbos_input=EN&Kalbos=EN&KategorijaEn_input=Mix&Kategorija… 3/38
3/27/2019 LSMUSIS
A. 70-year – old male with badly controlled arterial hypertension during last 30 years. 10 years ago -
myocardial infarction, 6 years ago – stroke. BP – 200/100 mmHg, urinanalysis – no pathological changes,
serum creatinine 240 µmol/l, on sonography – length of both kidney – 8 cm.
B. 34-years old male with history of arterial hypertension since childhood (for this reason was released from
army), from childrood on antihypertensives. BP – 180/110 mmHg. ECG – hypertrophy of left ventricle, grade
II hypertensive retinopathy, urinanalysis – proteinuria 2 g/l, RBC – 15-20 in h.p.f. Serum creatinine – 120
µmol/l. Ultrasonography- length of both kidney – 10 cm.
C. 56-years-old female with history of badly controled arterial hypertension during last 36 years. In past –
viral hepatitis, pneumonia, many times – cystitis. BP – 160/100 mmHg. ECG – hypertrophy of left ventricle.
Grade II hypertensive retinopathy. Urinanalysis – protein-trace, WBC -10-15-20. Serum creatinine – 80
µmol/l. Ultrasonography – lengh of right kidney – 8 cm, left – 11 cm.
D. 60-years-old male with history of badly controled arterial hypertension during last 25 years. BP- 220/120
mmHg. ECG – hypertrophy of left ventricle. Grade II hypertensive retinopathy. ECG- hypertrophy of left
ventricle. Urinanalysis – proteinuria 0,5 g/l. Serum creatinine 180 µmol/l. Ultrasonography – length of both
kidney – 10 cm.

21.30 Which symptoms are helpful in diagnosis of acute renal failure (in differential diagnosis with chronic renal
failure):
A. Higher levels of serum creatinine and urea.
B. Presence of dyspeptic syndrome (nausea, vomiting)
C. Absence of anemia and normal size of kidney on ultrasonography.
D. Absence of hematuria and proteinuria in urinanalysis.

21.31 A leukocyte count and differential was 20x109/l. There were 10% neutrophils, 88% lymphocytes, 2%
monocytes. This represents:
A. Absolute neutrophilia
B. Agranulocytosis
C. Relative lymphocytosis
D. Absolute lymphocytosis

21.32 Which statement about chronic myelocytic leukemia is wrong:


A. Course of disease can be divited in chronic, accelerated and blastic phases
B. hiladelphia chromosome is usually absent in cytogenetic test
C. Disease usually is effectively cured with imatinib
D. Myeloid/erythroid precursors ratio in bone marrow is usually increased

21.33 Which symptom is not characteristic for acromegaly:


A. Hands, foots increase
B. Hypertension
C. Glucose intolerance or diabetes mellitus
D. Rugged face lineament
E. Ophthalmopathy

21.34 What medication should be used in treatment of prolactinoma:


A. Thiamazol
B. Bromcriptin
C. Hydrocortisone
D. Levothyroxin
E. Metformin

21.35 51.38. Which blood test is the most informative for diagnosis of acromegaly:
A. Growth hormone (STH or GH), insulin-like growth factor 1 (IGF 1)
B. Thyroid – stimulating hormone (TSH) , thyroxine ( FT4)
C. Adrenocorticotropin hormone (ACTH)
D. Potassium, sodium
E. Prolactin

21.36 Which test is the best to assess iron reserve in organism?


A. Haemoglobin
B. Iron concentration in blood serum
C. Plasma capacity to bind iron
D. Ferritin concentration

lsmusis.lsmuni.lt/Klausimai/Spausdinti?Length=0?Kalba=EN&KategorijaId=123&Kalbos_input=EN&Kalbos=EN&KategorijaEn_input=Mix&Kategorija… 4/38
3/27/2019 LSMUSIS
E. Transferrin concentration

21.37 Drug of first choise to treat delirium in older patients:


A. Phenobarbital
B. Haloperidol
C. Lorazepam
D. Diazepam

21.38 Normal PSA serum concentration:


A. <4 ng/ml
B. 4-10 ng/ml
C. 10-20 ng/ml
D. >20 ng/ml

21.39 Median risk prostate cancer is, when:


A. T1c, Gleason score of ≤6,serum PSA <10 ng/ml
B. T3a, Gleason score of ≥6, serum PSA >20 ng/ml
C. T2b, Gleason score of 7, serum PSA 10-20 ng/ml
D. T1c, Gleason score of 9, serum PSA >20 ng/ml
E. T4, Gleason score of ≤6, serum PSA 10-20 ng/ml

21.40 Prostate cancer risk factors:


A. Smoking
B. Excessive drinking
C. Genetic predisposition
D. Prostate benign hyperplasy

21.41 Which of the following indicators is most valuable in evaluation of alcohol consumption level in the
community:
A. Per capita consumption of pure alcohol per year
B. Alcohol sales in litters
C. Amount of confiscated illegal alcohol
D. Amount of alcohol taxes paid to country budget

21.42 WHO experts recommend to limit daily salt intake to:


A. Less than 5 g per day not including the processed food
B. Less than 10 g per day not including the processed food
C. Less than 5 g per day including the processed food
D. Less than 10 g per day including the processed food

21.43 Weight of adult person is considered as normal when:


A. BMI 18 – 22 kg/m2
B. BMI 18,5 – 24,9 kg/m2
C. BMI 20 – 25 kg/m2
D. BMI 20,5 – 24,9 kg/m2

21.44 Clinical signs of children GORD:


A. Oesophageal symptoms
B. Chronic unproductive cough and wheezing
C. Erosion of dental enamel
D. Fe deficit anaemia
E. All mentioned signs

21.45 Unusual symptom in children ulcerative colitis:


A. Bloody diarrhea
B. Inflammation in colon
C. Inflammation in small intestine
D. Arthralgia
E. Anaemia

21.46 Which of the following neurological symptoms is not focal:


A. Hemiparesis
B. Impairment of consciousness
lsmusis.lsmuni.lt/Klausimai/Spausdinti?Length=0?Kalba=EN&KategorijaId=123&Kalbos_input=EN&Kalbos=EN&KategorijaEn_input=Mix&Kategorija… 5/38
3/27/2019 LSMUSIS
C. Hemihypesthesia
D. Aphasia
E. Homonymous hemianopia

21.47 Which of the following symptoms is uncommon for cerebrovascular accident in the anterior circulation:
A. Hemiparesis
B. Aphasia
C. Symptoms of cranial nerves’ dysfunction
D. Homonymous hemianopia

21.48 A positive sign of Wickham’s striae is characteristic for:


A. Bullous pemphigoid
B. Contact allergic dermatitis
C. Psoriasis
D. Lichen planus

21.49 The causative agent for herpes genitalis is (are):


A. Herpes simplex virus-1
B. Chlamydia trachomatis
C. Herpes simplex virus-2
D. Herpes simplex virus-1 and Herpes simplex virus-2

21.50 What is ASA (American Society of Anesthesiologists) class:


A. Classification of cardiovascular function of the patient
B. Classification of the of the patient’s physical status, according to which kind of surgery is selected
C. Classification of the of the patient’s physical status by American Society of Anesthesiologists, according
to which the risk of perioperative complications is anticipated.
D. Classification of the of the patent’s airway patency.

21.51 Which maneuver protects from regurgitation in case of full stomach:


A. Breathing 100% oxygen for 5 min
B. Tilting head down
C. Sellick’s maneuver (pulling cricoid cartilage down )
D. Pulling the mandible up

21.52 Antimicrobial prophylaxis is most effective (optimal) when given i/v


A. More than 2 hours prior to the initial incision
B. 30 to 60 minutes prior to the initial incision
C. 30 minutes after to the initial incision
D. The day prior to the initial incision

21.53 Which of the following statements is true regarding the diagnosis of chronic constrictive pericarditis:
A. Pulsus paradoxus is more common in the setting of atrial fibrillation.
B. The most common cause of chronic constrictive pericarditis is previous cardiac surgery.
C. Abdominal distension is the most common symptom.
D. Catheterization demonstrates characteristic early rapid rise in RV diastolic pressure, followed by a rapid
fall to a plateau (square-root sign).
E. CT scan is useful to demonstrate the thickened pericardium.

21.55 Which of the following is characteristic to asymptomatic bacteriuria:


A. Dysuria
B. Bacteriuria
C. Fever with chills
D. Flank plain
E. Leukocyturia

21.56 Which of the following is not characterisitc to ectopic pregnancy:


A. Disorders of the menstrual cycle
B. Fever
C. Pain in the lower part of the pelvis
D. Vaginal bleeding

21.57 Mechanism of Carboanhydraze inhibitors in decreasing of IOP is:


lsmusis.lsmuni.lt/Klausimai/Spausdinti?Length=0?Kalba=EN&KategorijaId=123&Kalbos_input=EN&Kalbos=EN&KategorijaEn_input=Mix&Kategorija… 6/38
3/27/2019 LSMUSIS
A. Cause myosis
B. Decrease IOP by increasing trabecular outflow
C. Decrease IOP by suppressing production of aqueous humour
D. Release spasm of ciliary muscle
E. Decrease IOP by increasing uveoscleral outflow

21.58 Which is the pathognomonic sign of the ocular allergy:


A. Conjunctival chemosis
B. Tearing
C. Itching
D. Blepharospasm
E. Decreased visual acuity

21.59 Public institutions of health care are mainly financed from:


A. Assignations from the State budget;
B. Assignations from the municipalities budget;
C. Compulsory health insurance fund.

21.60 Social insurance is:


A. Part of the social security system, which covers health care expenditure for the persons insured by the
State;
B. Part of the health care system, which covers health care expenditure for the insured persons;
C. Part of the social security system, which quarantees income of the insured persons in cases of incapacity
for work;
D. Part of the social security system, which guarantees social assistance for the persons insured by the
State from the State budget.

21.61 The name of most frequent value of random variable is:


A. Mode
B. Median
C. Average
D. Frequency

21.62 What do not characterizes the variability of data:


A. Median
B. Coefficient of variation
C. Variance
D. Standard deviation

21.63 Patient underwent percutaneous needle biopsy due to peripherial tumor of the left lung. One hour later
patient feels severe dyspnea. Clinical examination revealed tachycardia, decreased breath sounds and
hyperresonance to percussion on the left side of the chest. What treatment method is most suitable in this case:
A. Thoracotomy
B. Insertion of a right subclavian catheter and administration of intravenous fluids
C. Endotracheal intubation
D. Drainage of the left pleural cavity
E. Surgical exploration of the punction place in the chest wall

21.64 What does it mean methemoglobinemia:


A. Hemoglobin binding with carbon monoxide (CO)
B. Hemoglobin binding with oxygen
C. The changes of hemoglobin iron atomicity from bivalent to trivalent
D. The hemoglobin release from eritrocytes

21.65 Whitch metabolite is the toxic metabolite of paracetamol:


A. Glucuronide
B. N-acetylbenzoquinoneimine
C. Benzoylecgonine
D. Formaldehyde

21.66 51.86. Which pathogen does not cause acute bacterial sinusitis, otitis and typical pneumonia:
A. Moraxella catarrhalis

lsmusis.lsmuni.lt/Klausimai/Spausdinti?Length=0?Kalba=EN&KategorijaId=123&Kalbos_input=EN&Kalbos=EN&KategorijaEn_input=Mix&Kategorija… 7/38
3/27/2019 LSMUSIS
B. Streptococcus pneumoniae
C. Haemophilus influenzae
D. Mycoplasma pneumoniae

21.68 Which of the following specialists confirms the diagnosis of occupational disease:
A. State Labour Inspectorate representative;
B. Any doctor which suspected disease
C. Occupational hygienist;
D. Occupational medicine doctor;
E. All of the professionals;

21.69 Occupational lung cancer risk factors:


A. silica dust
B. ammonia fumes
C. coal dust
D. toluene diisocyanate
E. all listed

21.70 Main active material of contrast media which are used in ultrasound examinations for detection of enhancing
effect
A. Atoms of iodine
B. atoms of nickel
C. Microgranules of D-Galactosis
D. 18 FDG (2-(F-18) fluoro-2-deoxy-D-glucosis
E. Atoms of Gadolinium

21.71 Effective dose of ionizing radiation in grays (Gy) is highest in case of:
A. Abdominal CT
B. Chest CT
C. Cerebral CT
D. Stomach radioscopy
E. Intestinal radioscopy

21.72 Sarin, a chemical warfare, is:


A. A nerve agent
B. A choking agent
C. A blister agent
D. An irritant

21.73 How many proteins need a person per day:


A. 28 proc.
B. 14 proc.
C. 55 proc.

21.74 How many fats need a person per day:


A. 30 proc.
B. 60 proc.
C. 15 proc.

21.75 How many carbohydrates need a person per day:


A. 15 proc.
B. 56 proc.
C. 30 proc.

21.76 The degradation of intracellular organelles through the process in which autosomes combine with primary
lysosomes to form autophagolysosomes is called:
A. Autophagy
B. Heterophagy
C. Heteroplasmy
D. Homophagy
E. Endocytosis

21.77 Which traumatic laesions of cervical spine could be visualisated on spondylogramms


lsmusis.lsmuni.lt/Klausimai/Spausdinti?Length=0?Kalba=EN&KategorijaId=123&Kalbos_input=EN&Kalbos=EN&KategorijaEn_input=Mix&Kategorija… 8/38
3/27/2019 LSMUSIS
A. Fractures of vertebra and subluxationes
B. Ligamental ruptures
C. Local haematomas
D. Contusio of cervical spine
E. Protrusion of cervical intervertebral disc

21.78 Which way is usual for metastases into the brain from other malignancies:
A. Straight invasion of neoplasm
B. Haematogenic pathway
C. CSF pathway
D. By intraspinal veins
E. Leptomeningeal spread

21.79 Which symptom is typical for hip osteoarthritis:


A. Groin pain
B. Pain spread to low back
C. Swelling of hip region
D. Subfebrile fever

21.80 Which symptom is typical for osteoarthritis:


A. “Mechanical” type of pain ( at start and after physical loading)
B. Swelling and synovial fluid
C. Pain of metacarpophalangeal joints
D. Spasmodic, short duration joint swelling

II type tasks. For each numbered item,selct the one lettered heading that is most closely asssciated with it

21.81 Innervating nerves for:


1 - Glandula parotis
2 - Musculus deltoideus
3 - Musculus stapedius
4 - Musculus levator palpebrae
5 - Musculus mylohyoideus
A. Nervus facialis
B. Nervus mandibularis
C. Nervus axillaris
D. Nervus oculomotorius
E. Nervus glossopharyngeus

21.82 What is characteristic for each disease:


1 - Diverticulitis
2 - Gall bladder stones
3 - Acute appendicitis
4 - Embolism of the mesenteric vessels
A. Spasmodic pain in the abdomen
B. Continuous abdominal pain
C. Gnawing in the epigastric region
D. Severe abdominal pain

21.83 The common feature for each disease:


1 - Diabetic retinopathy
2 - Carotido-cavernous fistula
3 - Glaucoma
A. Thinning of the retinal nerve fiber layer
B. Spontaneous haemophthalmus
C. Pulsating exophthalmos

21.85 What is characteristic for these root syndromes:


1 - Root C₆
2 - Root C₇
3 - Root L₅
4 - Root S₁
lsmusis.lsmuni.lt/Klausimai/Spausdinti?Length=0?Kalba=EN&KategorijaId=123&Kalbos_input=EN&Kalbos=EN&KategorijaEn_input=Mix&Kategorija… 9/38
3/27/2019 LSMUSIS
A. Sensory signs and symptoms (pain) on the lateral leg, the dorsomedial foot, and the large toe;
depressed extension of the large toe.
B. Sensory disturbances on the little toe, lateral foot, and most of the sole of the foot; depressed foot
dorsiflexion; depressed the Achilles reflex.
C. Sensory signs and symptoms (pain) on the lateral forearm, lateral hand, and the first and second
digits; the biceps reflex may be depressed or absent.
D. Sensory signs and symptoms (pain) on the lateral forearm, lateral hand, and the third and fourth digits;
the triceps reflex may be depressed or absent.

21.86 What is characteristic for these anaesthesia methods:


1 - Epidural anaesthesia
2 - Endotracheal anaesthesia
3 - Spinal anaesthesia
A. Limited duration of anaesthesia
B. Possibility to use postoperatively
C. Possibility to use muscle relaxants

21.87 Choose correct answer for each disease:


1 - Gilbert’s syndrome
2 - Chronic hepatitis
3 - Hepatic cirrhosis
4 - Choledocholithiasis
A. Increase in unconjugated bilirubin, normal activity of transaminases (ALT, AST)
B. Decreased level of serum albumin, prothrombin, increased concentration of unconjugated and
conjugated bilirubin
C. Increase in conjugated bilirubin and AP
D. Increase in unconjugated and conjugated bilirubin, elevated transaminases

21.88 Which therapeutic option is the best for managing of following diseases:
1 - Primary biliary cirrhosis Pirminė bilijinė cirozė
2 - Chronic viral hepatitis C Lėtinis virusinis C hepatitas
3 - Autoimmune hepatitis Autoimuninis hepatitas
4 - Decompensated hepatic cirrhosis Dekompensuota kepenų cirozė
A. Ursodeoxycholic acid
B. Prednisolone
C. Interferon and ribavirine
D. Diurethic

21.89 Which treatment would you choose for following diseases and bleeding:
1 - Hemophylia A
2 - Hemophylia B
3 - Congenital factor VII deficiency
4 - Acute liver insufficiency
5 - Overdosage of varfarin
A. Prothrombin complex concentrate, fresh frozen plasma
B. VII clotting factor concentrate
C. IX clotting factor concentrate, fresh frozen plasma
D. Prothrombin complex concentrate, fresh frozen plasma, vitamin K
E. VIII clotting factor concentrate, cryoprecipitate

21.90 Which treatment would you choose for following diseases:


1 - Chronic myelocytic leukemia
2 - Chronic lymphocytic leukemia
3 - Acute myeloblastic leukemia
4 - Multiple myeloma
5 - Hodgkin‘s disease
A. Cytosar and daunorubycin 7+3 chemotherapy, allogenic bone marrow transplantation
B. ABVD or BEACOPP type chemotherapy, radiation therapy
C. Melphalan , talidomide, prednisolone or bortezomib and dexamethasone, following autologous bone
marrow
D. Imatinib or interferon alfa, allogenic bone marrow transplantation
E. Chlorambucil or rituksimab, fludarabine and cyclophosphamide chemotherapy
lsmusis.lsmuni.lt/Klausimai/Spausdinti?Length=0?Kalba=EN&KategorijaId=123&Kalbos_input=EN&Kalbos=EN&KategorijaEn_input=Mix&Kategorij… 10/38
3/27/2019 LSMUSIS
21.91 Which of the following treatment is recomended when is:
1 - Congenital aortic valve stenosis and of ascedence part aortic aneurysm
2 - In the left coronary artery trunk 95 percent. stenosis
3 - 75 year-old male with a critical mitral valve stenosis
4 - Ebštein anomaly
A. Tricuspidal valve and atrial septum plastic
B. Biological valve prosthesis implantation
C. Aorta-coronary connection formation
D. Bental operation

21.92 Treatment you prefere for each disease:


1 - Ocular allergic reactions
2 - Chalasion
3 - Retinal neovascularisation because of central retinal vein thrombosis.
A. Laser photocoagulation
B. Artificial tears, vasoconstrictors, antihistaminic, mast-cell stabilizing eye drops
C. Kenalog injection and in inefficient result – surgical treatment

21.93 What ASA(American Society of Anesthesiologists) class do these patients belong:


1 - I class
2 - III class
3 - VI class
A. A patient with severe systemic disease that results in functional limitation
B. A patient whose organs are being removed for donor purposes
C. A normal healthy patient

21.94 What peculiarities of lividity are observed in each of following cases:


1 - Asphyxia
2 - Carbon monoxide toxicity
3 - Cyanide toxicity
4 - Nitrate toxicity
A. Brown color
B. Dark red color
C. Bright, intense bluish purple color
D. Bright red color

21.95 Which test are using for differential diagnosis of follow diseases:
1 - Adrenal insufficiency
2 - Cuching‘s disease
3 - Insulinoma
4 - Diabetic nephropathy
A. Captopril test
B. Dexamethasone test
C. Synacthen test
D. Fasting test

21.96 What symptoms are associated with different causes of low back pain:
1 - Malignant disease (multiple myeloma), metastatic disease
2 - Abdominal aortic aneurysm
3 - Spine fracture (compression fracture)
A. History of trauma, osteoporosis, localized pain over spine
B. Unexplained weight loss, fever, abnormal serum protein electrophoresis pattern, history of malignant
disease
C. Inability to find position of comfort, back pain not relieved by rest, pulsate mass in abdomen

21.97 What sexually transmitted diseases do we treat with these medications:


1 - Benzathine penicillin
2 - Valacyclovir
3 - Podophyllotoxin
4 - Permethrin
A. Pediculosis pubis
B. Syphilis
lsmusis.lsmuni.lt/Klausimai/Spausdinti?Length=0?Kalba=EN&KategorijaId=123&Kalbos_input=EN&Kalbos=EN&KategorijaEn_input=Mix&Kategorija… 11/38
3/27/2019 LSMUSIS
C. Herpes genitalis
D. Genital warts

21.98 What is the mechanism of action of each of the following:


1 - Combined contraceptive pills
2 - Low dose oral progestins
3 - Intra-uterine spiral
4 - Spermicidal gel
A. Affect the properties of the cervical mucosa
B. Inhibition of sperm movement and its viability
C. Inhibition of both follicle maturation and ovulation
D. Disorders of implanation

21.99 What fracture occurs in each case of injury to the long bone:
1 - Bending of the bone
2 - Pressure on the bone
3 - Twisting of the bone
4 - Abrupt strain of the tendon
A. Impacted fracture in metadyaphysis
B. Disjunction of a piece of bone tissue
C. Fragmented fracture with a large fragment in the zone of trauma.
D. Spiral fracture

21.100 Which is the main diagnostic method for determination of the surgical method of the treatment:
1 - Ischemic mitral insufficiency
2 - Aortic disecatin
3 - Ischemic heart disease
4 - Suspected, the increased amount of fluid in the thorax
A. Coronary angiography
B. Echocardioskopy
C. Reconstructive computer tomography
D. X-ray of the thoracic organs

21.101 Which symptom is common for each of the diseases:


B 1 - Myasthenia gravis
A 2 - Cerebral infarct
D 3 - Alzheimer’s disease
C 4 - Parkinson disease
A. Hemiparesis/hemihypesthesia
B. Easy fatigability of voluntary skeletal muscle
C. Unilateral tremor
D. Progressive impairment of memory

21.102 To what type of anemia it refers:


1 - May be caused by chemotherapy
2 - Jaundice most likely to occur
3 - Can be caused by atrophic gastritis
4 - Can be caused by chronic blood loss
A. Hemolytic anemia
B. Iron deficiency
C. Aplastic anemia
D. Vit. B12 deficiency anemia

21.103 Make the links between the cell types and cell structures:
1 - Plant cell
2 - Animal cell
3 - Bacterial cell
A. Mesosome
B. Central vakuole
C. Glycocalyx

21.104 To which group belong medications for the tobacco addiction treatment:

lsmusis.lsmuni.lt/Klausimai/Spausdinti?Length=0?Kalba=EN&KategorijaId=123&Kalbos_input=EN&Kalbos=EN&KategorijaEn_input=Mix&Kategorij… 12/38
3/27/2019 LSMUSIS
1 - Nicotine medications
2 - Nortriptiline
3 - Varenicline tartras
4 - Bupropione
A. Medications under prescription
B. Medications without prescription

21.105 Which of the below mentioned diseases is presented with the following radiographic alterations of lung
roots:
1 - Venous stasis
2 - Fibrosis and scar tissue formation
3 - Enlargement of lymph nodes
4 - Calcification of lymphnodes
A. Lymphoma
B. Tuberculosis
C. Pneumofibrosis
D. Pulmonary hypertension

21.106 Which diagnostic tests are characteristic for following diseases:


1 - Vasospasm
2 - Cavernoma
3 - AVM
4 - Fractures of spinal vertebra
5 - Symptomatic epilepsy
A. MRI scan
B. TCD (transcranial doplerography)
C. Cerebral angiography
D. CT scan
E. EEG

21.107 Which clinical signs are characteristic for these types of immunodeficiency:
C 1 - Antibody deficiency in a case of common variable immunodeficiency
D 2 - T cell deficiency in AIDS
A 3 - Complement deficiency because of a lack of C1 esterase
B 4 - Phagocytic cells deficiency in a case of chronic granulomatous disease
A. Repeated attacks of angioedema
B. Deep organ and skin absceses, granuloma formation.
C. Repeated bacterial respiratory system infections
D. Pneumocystis jirovecii infection

21.108 In case of which clinical symptoms Family Physician should suspect the following disease:
1 - Sore throat, fever up to 39°C, tonsilar hyperemia, enlarged lymphatic nodes of the neck
2 - Fever up to 37,5°C, throaty voice, coryza, dry cough
3 - Sudden onset of the symptoms, head and muscle pain, fever up to 40°C; later dry cough, hyperemia of
mouth and nose
A. Influenza
B. Acute viral upper respiratory tract infection
C. Acute streptococcical tonsillitis

21.109 Match medication to the corresponding clinical situation:


C 1 - Prostate carcinoma
D 2 - Benign prostatic hyperplasia
A 3 - Nethrolithiasis
B 4 - Bladder carcinoma
A. Citrates
B. BCG vaccine
C. LH-RH agonists
D. 5-alfa reductase inhibitors

21.110 What the diseases are caused by these microorganisms


1 - Pneumonia
2 - Dysentery
lsmusis.lsmuni.lt/Klausimai/Spausdinti?Length=0?Kalba=EN&KategorijaId=123&Kalbos_input=EN&Kalbos=EN&KategorijaEn_input=Mix&Kategorij… 13/38
3/27/2019 LSMUSIS
3 - Carbuncul
4 - Scarlet fever
A. Streptococcus pyogenes
B. Streptococcus pneumoniae
C. Staphylococcus aureus
D. Shigella fleksneri

21.111 Which changes of heart sounds are characteristic for or may be found in the conditions listed below:
1 - Ejection sound
2 - Diastolic pathologic S3
3 - Click
4 - I pitched decomposition
A. The prolapsed mitral valve
B. Right bundle branch block
C. Aortic stenosis
D. Aortic regurgitation

21.112 Which heart murmur is characteristic for conditions listed below:


1 - Presystolic crescendo murmur at the cardiac apex
2 - Protodiastolic crescendo murmur in the aortic area, the Erb‘s area, and over mitral area
3 - Holosystolic plateau murmur over mitral area
4 - Midsystolic crescendo-decrescendo murmur over the aortic area, the Erb‘s area, and the mitral area
A. Mitral regurgitation
B. Aortic stenosis
C. Mitral stenosis
D. Aortic regurgitation

21.113 For which part of the special sense organs is characteristic the following structures:
1 - A vascularized pseudostratified epithelium
2 - The photoreceptor layer
3 - The highly vascular loose connective tissue which contains many melanocytes
A. The stria vascularis in cochlear duct
B. The choroids of the eye
C. The retina of the eye

21.114 Wich kind of pleurae punctatus characterises these symptoms:


1 - Effusion
2 - Exudate
3 - Haemathorax
4 - Chylothorax
A. >50% of blood haematocrit level
B. Triglyceridus > 110 mg/dl
C. 33 g/l proteins and 2 x 109/l leucocytes
D. Lactate dehydrogenase ratio in pleurae punctatus with blood 0,5

21.115 What is the most acceptable method for treatment of the following chest injuries:
1 - Uncomplicated fractures of the ribs
2 - Rupture of the diaphragm
3 - Traumatic pneumothorax
4 - Injury to the heart
5 - Clotted (coagulated) heamothorax
A. Osteosynthesis and fixation of ribs
B. Urgent thoracotomy
C. Drainage of the pleural cavity
D. Local anesthesia of intercostal nerves
E. Ellective thoracotomy

21.116 Which of the following histological characteristics is suitable to the genital ducts of the male reproductive
system:
1 - Labirinthine plexus of anastomosing channels lined by a simple cuboidal epithelium; many of cells
possess a single luminal flagellum

lsmusis.lsmuni.lt/Klausimai/Spausdinti?Length=0?Kalba=EN&KategorijaId=123&Kalbos_input=EN&Kalbos=EN&KategorijaEn_input=Mix&Kategorij… 14/38
3/27/2019 LSMUSIS
2 - They are lined by a simple epithelium composed of alternating clusters of nonciliated cuboidal cells and
ciliated columnar cells
3 - This structure is lined by a pseudostratified columnar epithelium, which contains the following two cell
types: basal cells and principal cells, which are columnar in shape and possess nonmotile stereocilia
A. Ductus epididymidis
B. Ductuli efferentes
C. The rete testis

21.117 What lab. tests results are characteristic for the following diseases:
1 - Subacute thyroiditis
2 - Chronic autoimmune thyroiditis
3 - Graves’desease
4 - Postoperative hypothyroidism
A. Increased FT4, decreased TSH blood level, TSH receptor antibodies
B. Increased FT4, decreased TSH blood level, elevated ESR
C. Increased TSH, decreased FT4 blood level, antithyroperoxidase antibodies
D. Increased TSH, decreased FT4 blood level hypercholesterolaemia

21.118 Which mean of toxicokinetic mechanisme (the passing through biomembrane) match to proposition below:
1 - Passive diffusion
2 - Filtration
3 - Active transport
4 - Pinocytosis
A. Hydrophilic substances pass trough biomembrane pores with circulating water flow
B. The vesicules with toxic substances penetrate biomembrane
C. The substances pass trough by concentration gradient from the more to less concentrated
environment or to environment whithout such substances
D. The mechanisme when the energy is needed for transportation and the active toxines transporting
systems (enzymes adenosinetriphosphatases) are present and the toxines can pass against
concentration gradient (from environment with less concentration to more concentrated)

21.119 Which variation of pathogenesis match to poisoning:


1 - Opioides
2 - Organophosphates
3 - Amanitines
4 - Carbon monoxide
5 - Hydrogenium sulphide
A. Binde to hemoglobin replacing oxygen and compose carboxyhemoglobin
B. Block DNA trascription to RNA and interupt proteine synthesis in the hepatocytes and induce
lipodystrophia of liver and the necrosis of liver parenchyma.
C. Acethylcholine accumulation in the synaptic gap and cholinomimetic effects because of the inhibition
of cholinesterase.
D. The respiratory center depression because of acting on the CNS opiod receptors.
E. The oxygen transport interruption in to the cells because of cytochromoxydase binding to trivalent iron.

21.120 Which indicator characterizes each of the following statements on diseases of the circulatory system:
1 - During a year 47/1000 population new cases of diseases of the circulatory system were diagnosed
2 - Morbidity of diseases of circulatory system among smokers was 1.6 times higher than that among non-
smokers
3 - Diseases of the circulatory system composed 55% in the overall structure of causes of death during a
year
A. Proportion
B. Incidence rate
C. Mortality
D. Lethality (case fatality)
E. Rate ratio

21.121 Which symptom is specific for each of the following child rheumatic diseases:
1 - Positive rheumatoid factor
2 - Enthesitis
3 - Iridocyclitis
4 - Gottron papule
lsmusis.lsmuni.lt/Klausimai/Spausdinti?Length=0?Kalba=EN&KategorijaId=123&Kalbos_input=EN&Kalbos=EN&KategorijaEn_input=Mix&Kategorij… 15/38
3/27/2019 LSMUSIS
A. Dermatomyositis
B. RF positive polyarthritis
C. Oligoarthritis type 1
D. Oligoarthritis type 2 [HLA-B27]

21.122 Which symptom is specific for each mentioned congenital heart disease:
1 - A grade to 4/6 (“machinery”) murmur at the left infraclavicular area
2 - A grade 2 to 5/6 systolic murmur at the lower left sternal border
3 - A widely split and fixed S2
4 - Week pulse in legs
A. Ventricular septal defect
B. Ductus arteriosus
C. Coarctation of aorta
D. Atrial septal defect

21.123 What phenomena of light correspond to following observations:


1 - Beam of light bends when passing the sharp edge of plate
2 - Colourful film of oil on a surface of water
3 - Beam of white light splits into different colours after passing the glass prism
4 - Beam of light travelling through two perpendicularly crossed prisms of Island spars crosses the first
prism but disappears in the second one.
A. Dispersion
B. Diffraction
C. Polarization
D. Interference

21.124 Which of the following statements applies to diabetes mellitus Type I or Type II:
1 - It is an autoimmune disorder
2 - There is absolute insulin deficiency
3 - Individuals with the disease are usually obese
4 - The onset of manifestations is gradual
5 - Onset is usually in childhood and adolescence
6 - Individuals with this disorder are often still synthesizing insulin
A. Type I
B. Type II

21.125 In which of these cases involving electric and magnetic fields the following phenomena are observed:
1 - The Hall effect in flowing electrolytes
2 - The orientation of polar particles along the field lines
3 - The change of dielectric polarization (displacement current)
4 - Electromagnetic induction
A. Constant electric (electrostatic) field
B. Oscillating electric field
C. Constant magnetic field
D. Oscillating magnetic field

21.126 Mass poisonings depend of the following physical conditions of the toxic substance:
1 - Low boiling temperature substances
2 - Volatile substances
3 - Substances with high relative density of vapors
A. Will be toxic, if their volatility in the air will be higher than minimal toxic concentration;
B. Will persist in low landscape and basements of buildings;
C. Will faster reach toxic concentration in the air, but their persistence is shorter

21.127 Point the characteristic occupational exposure for each occupational disease:
1 - Bronchial asthma
2 - Angiosarcoma of liver
3 - Mesothelioma
4 - Ototoxic damage
A. Styrene
B. Vinilchloride
C. Izocyanates
lsmusis.lsmuni.lt/Klausimai/Spausdinti?Length=0?Kalba=EN&KategorijaId=123&Kalbos_input=EN&Kalbos=EN&KategorijaEn_input=Mix&Kategorij… 16/38
3/27/2019 LSMUSIS
D. Asbestos

21.128 Point the appropriate occupational risk factor for each disease:
1 - Mould fungi
2 - Asbestos
3 - Grain dust
4 - Benzene
A. Leukemia
B. Professional asthma
C. Adhesive pleuritis
D. Hypersensitivity pneumonitis

21.129 Match, please, dietary risk factors with cancers of different location:
1 - Stomach
2 - Kidney
3 - Liver
A. Obesity
B. Salt, salted food
C. Aflatoxin

21.130 These are main antidotes in case of poisoning:


1 - Nerve agents - organophosphates
2 - Cyanides
3 - Iprite
A. Amilnytrite, methylene blue
B. Atropine
C. No specific antidote

21.131 Match the most closely associated description of a cardiovascular disease process:
1 - Large, destructive vegetations involving the aortic valve
2 - Myxomatoous degeneration of the mitral valve
3 - Fragmentation of elastic fibers in the media of the aortic arch
A. A 30-year-old male with long-standing essential hypertension collapses after the sudden onset of
severe chest pain. On physical examination, his radial pulses are weak but femoral pulses are normal. A
chest radiograph reveals widening of the mediastinum.
B. A 35-year-old asymptomatic female is found to have a mid-systolic click heart murmur during a routine
physical examination for an insurance policy.
C. A 35-year-old male with a history of intravenous drug abuse is brought to the hospital emergency room
for evaluation of acute onset fever and altered mental status. A head CT scan reveals evidence of a rigth
parietal, 3-cm abscess.

21.132 Structures passing through formina:


1 - Arteria ophthalmica
2 - Arteria meningea media
3 - N. petrosus profundus
4 - N.maxillaris
5 - Radix spinalis nervi craniali XI
A. Foramen spinosum
B. Canalis pterygoideus
C. Canalis opticus
D. Foramen rotundum
E. Foramen magnum

21.133 For energy values listed below select the approximate amount of energy released after complete oxidation
of nutrients:
1 - Approximate amount of energy in kcal after complete oxidation of 1 g of fat
2 - Approximate amount of energy in kcal after complete oxidation of 1 g of proteins
A. 9 kcal
B. 4 kcal
C. 7,4 kcal
D. 100 kcal

lsmusis.lsmuni.lt/Klausimai/Spausdinti?Length=0?Kalba=EN&KategorijaId=123&Kalbos_input=EN&Kalbos=EN&KategorijaEn_input=Mix&Kategorij… 17/38
3/27/2019 LSMUSIS
III type tasks. For each question there is one or more correct answers:
A – if correct answers are 1,2,3
B – if correct answers are 1 and 3
C – if correct answers are 2 and 4
D – if correct answer is 4
E – if correct are all answers above

21.134 The suprarenal arteries may arise from the:


1. Aorta
2. Renal artery
3. Inferior phrenic artery
4. Superior mesenteric artery

21.135 The corneal reflex is absent in the lesion of:


1. n. supraorbitalis
2. n. oculomotorius
3. n. maxillaris
4. n. ophthalmicus

21.136 The respiratory center


1. Is situated in the hypothalamus
2. Sends out regular bursts of impulses to the inspiratory muscles during quiet respiration
3. Sends out regular bursts of impulses to the expiratory muscles during quiet respiration
4. Is inhibited during swallowing

21.137 Insulin:
1. Stimulates the release of free fatty acids from adipose tissue
2. Facilitates entry of glucose into skeletal muscle
3. Secretion tends to raise the plasma potassium level
4. Facilitates entry of amino acids into skeletal muscle

21.138 Cells of the fundic glands of the stomach:


1. Mucous neck cells
2. Parietal cells
3. Chief (zymogenic) cells, which secrete pepsinogen
4. Paneth cells

21.139 Why does osteodystrophy develop in chronic renal failure:


1. There is decreased renal activation of vitamin D
2. There is decreased intestinal absorption of calcium
3. Osteoclasts are stimulated by increased parathyroid hormone secretion
4. Decreased serum levels of phosphate inhibit osteoblastic activity
5. There is increased renal excretion of calcium

21.140 What compensatory mechanisms are initiated in the initial stage of hypovolemia:
1. Increased secretion of antidiuretic hormone
2. Stimulation of the sympathetic nervous system
3. Increased secretion of rennin
4. Generalized vasodilatation

21.141 Which of the following processes can detoxify ammonia in tissues:


1. Transamination of amino acids
2. Formation of amides of amino acids
3. Decarboxylation of amino acids
4. Urea synthesis

21.143 What statements concerning cytomegalovirus virus are correct:


1. Can pass placenta and cause congenital infection
2. Predominant cause inapparent infections and activation of virus in
3. Virus can be detected in urine and other secretions of infected person
4. Virus can be transmitted in blood transfusions and tissue transplants
5. Virus belongs to Herpesviruses
lsmusis.lsmuni.lt/Klausimai/Spausdinti?Length=0?Kalba=EN&KategorijaId=123&Kalbos_input=EN&Kalbos=EN&KategorijaEn_input=Mix&Kategorij… 18/38
3/27/2019 LSMUSIS
21.144 What statements concerning apie Molluscum contagiosum viruses are correct:
1. Belongs to Poxviridae family
2. Cause skin lesions and genitalia epithelium not malignant proliferations
3. They are monopathogenic
4. They are parasites
5. They are RNA viruses

21.145 What are the differences between antidepressants of SSRI group and tricycle antidepressants (TCAs):
1. Less common urinary retentions in patients with adenoma of prostate
2. Less risk of overdose
3. Less risk of arrhythmias
4. Can be used for prevention of depression
5. Greater efficacy

21.147 Contraindications for the use of ACE inhibitors include:


1. Hyperkalemia
2. Hypersensitivity to the drug components
3. Both a. renalis stenosis
4. Previously therapy of diuretics
5. Renal failure

21.148 Characteristics of Klinefelter syndrome:


1. Tall stature
2. Sterility
3. Breast development
4. Hairy body
5. Short stature

21.149 Characteristics of Turner syndrome:


1. Short stature
2. No menstruation
3. Shield-shaped thorax
4. Excessive menstruation
5. Tall stature

21.150 What is impact of sun ultraviolet rays to human organism:


1. Activate metabolic system
2. Stimulate production of vitamine D
3. Suppression of synthesis of sex hormones
4. Stimulate skin pigmentation

21.151 For inflammatory arthritis are typical:


1. Joint crepitation
2. Joint swelling
3. Joint redness
4. Bouchard‘s nodes

21.152 For degenerative arthropathy are typical:


1. Joint redness
2. Increased joint space on X-ray
3. Symmetrical involvement of joints
4. Heberden‘s nodes

21.153 The most common community acquired pneumonia pathogens are:


1. Streptococcus pneumoniae
2. Klebsiella pneumoniae
3. Haemophilus influenzae
4. Pseudomonas aeruginosa

21.154 How is treated stable chronic obstructive lung disease with rare exacerbations:
1. Short acting bronchodilators
2. Inhaled steroids

lsmusis.lsmuni.lt/Klausimai/Spausdinti?Length=0?Kalba=EN&KategorijaId=123&Kalbos_input=EN&Kalbos=EN&KategorijaEn_input=Mix&Kategorij… 19/38
3/27/2019 LSMUSIS
3. One or more long acting bronchodilators
4. Oral steroids

21.155 Which analysis should be done when arterial hypertension is suspected:


1. Urine analysis
2. Blood potassium
3. Total cholesterol and triglyceride levels
4. Serum glucose

21.156 What signs suggest obstructive jaundice:


1. Increase in AP, y-GT
2. Conjugated hyperbilirubinemia
3. Stone in the common bile duct on a sonograph
4. Unconjugated hyperbilirubinemia

21.157 Which one of following diseases can cause hepatocellular jaundice:


1. Acute viral hepatitis
2. Carcinoma of the head of the pancreas
3. Hepatic cirrhosis
4. Choledocholithiasis

21.158 You want to calculate glomerular filtration rate (GFR). You need:
1. 24 h diuresis
2. Amount of creatinine in 24 h urine
3. Serum creatinine level
4. Specific gravity of morning urine

21.159 Acute pyelonephritis more often is caused by:


1. Streptococci
2. Stafilococci
3. Pseudomones
4. E coli

21.160 When one should diagnose severe hemophilia A:


1. When factor VIII activity is more than 2%
2. When factor IX activity is less than 1%
3. When factor IX activity is more than 2%
4. When factor VIII activity is less than 1%

21.161 Which complications are characteristic of multiple myeloma:


1. Acute renal insufficiency
2. Transformation to acute myeloblastic leukemia
3. Spontaneous bone fractures
4. Development of aplastic anemia

21.162 Symptoms and signs characteristic for prolactinoma:


1. Headaches
2. Disturbed gonadal function
3. Galactorrhoea
4. Osteoporosis

21.163 Causes of hyperprolactinaemia are:


1. Macroadenoma compressing stalk
2. Prolactinomas
3. Pregnancy
4. Primary hypothyroidism

21.164 The clinical features of Cushing’s syndrome are:


1. Weight loss
2. Hypertension
3. Hypoglycaemia
4. Osteoporosis

lsmusis.lsmuni.lt/Klausimai/Spausdinti?Length=0?Kalba=EN&KategorijaId=123&Kalbos_input=EN&Kalbos=EN&KategorijaEn_input=Mix&Kategorij… 20/38
3/27/2019 LSMUSIS
21.165 Which symptoms are characteristic to delirium
1. Fluctuating course
2. Inattention
3. Disorientation
4. Language disturbance

21.166 Which of the listed measures are the most effective in reduction of total alcohol consumption in population:
1. Alcohol price increase
2. Reduction of number of outlets
3. Restriction on hours of sale
4. Age restrictions

21.167 Persons are at high risk of fatal cardiovascular disease within the next 10 years, if they have:
1. Blood pressure > 180/110 mmHg
2. Diabetes mellitus with microalbuminuria
3. Total cholesterol > 8 mmol/l
4. High density lipoprotein cholesterol > 1 mmol/l

21.168 What are the symptoms of child with acute poststreptococcal glomerulonephritis:
1. The first symptoms mostly are hematuria and edema
2. Gross hematuria occurs in about one-third of patients
3. Serum urea nitrogen and creatinine levels increase
4. Child should be treated with fluid and salt restriction, diuretics

21.170 What is characteristic for pemphigus vulgaris:


1. Positive Nikolsky’s sign
2. First blisters often develop in the mouth
3. Primary skin lesion – blister
4. The acantholytic cells in smears from blisters are identified using cytological test

21.171 Methods used in the sterilisation of surgical equipment include:


1. Ethylene oxide
2. Gamma irradiation
3. Autoclave
4. 2% Gluteraldehyde
5. 2% Chlorhexidine

21.172 What complications are likely to follow a haemorrhoidectomy:


1. Narrowing of the anal canal
2. Formation of an epithelial fistula
3. Bleeding
4. Faecal incontinence

21.173 Which cases regarding complications of acute appendicitis indicate a necessity of an urgent surgery:
1. In case of an emergence of the periappendicular abscess
2. In case of a leakage of the Vermiform appendix
3. In case of peritonitis
4. During the infiltrational inflammation of the Vermiform appendix

21.174 Which of the following statements is true regarding the anatomy and pathophysiology of post-infarct left
ventricular aneurysm:
1. The inner wall of the aneurysm retains its trabeculations.
2. Mural thrombus is uncommon.
3. Most posterior aneurysms are true aneurysms.
4. Calcification may occur in the adherent pericardium.
5. Most patients with left ventricular aneurysms have single vessel LAD disease.

21.175 Which of the following statements is true regarding Tetralogy of Fallot morphology:
1. RV Hypertrophy
2. RVOT obstruction (infundibular stenosis)
3. Aorta dextraposition, overrides VSD
4. Patent ductus arteriosus

lsmusis.lsmuni.lt/Klausimai/Spausdinti?Length=0?Kalba=EN&KategorijaId=123&Kalbos_input=EN&Kalbos=EN&KategorijaEn_input=Mix&Kategorij… 21/38
3/27/2019 LSMUSIS
5. Left ventricular hypertrophy

21.176 Which treatment methods are recomended in the treatment of Fournier gangrene:
1. Antibiotics
2. Orchectomy
3. Necrectomy
4. Bladder catherisation
5. Cystostomia

21.177 Which operation(s) are performed to removed kidney pelvis tumors:


1. Resection of kidney pelvis
2. Kidney resection
3. Nephrectomy
4. Nephroureterectomy
5. Endoscopic fidney pelvis resection

21.178 Characteristic signs of peritonitis of the appendicular origin in childhood are:


1. Intense signs of the peritoneum irritation
2. Tensed muscles of the abdominal wall
3. Abdominal distension
4. Painful palpation in any parts of the abdomen
5. Multifold vomiting

21.179 Complications of the undiagnosed appendicitis arte:


1. Purulent peritonitis
2. Sepsis
3. Appendicular mass
4. Paralytic ileus
5. Pneumonia

21.180 What are the possible fetal complications of a pregnancy complicated with dibetes mellitus:
1. Cardiac malformations
2. Macrosomia
3. Fetal death in early pregnancy
4. Hyrops fetus universalis

21.181 Which of the following are included in the differantial diagnosis of pelvic inflammatory disease:
1. Ectopic pregnancy
2. Endometriosis
3. Acute appendicitis
4. Rupture of an ovarian cyst
5. Torsion of an ovarian cyst

21.182 Diagnostic criteria of open angle glaucoma:


1. Abnormalities of the visual field
2. Optic nerve disc changes
3. Thinning of the retinal nerve fiber layer
4. Blood glutamat concentration
5. Decreased number of the corneal endothelial cells.

21.183 Follicular reaction is typical for conjuntivitis caused by:


1. Adenoviral infection
2. Chlamydial infection
3. Epidemic adenoviral keratoconjunctivitis
4. Diphtherial conjunctivitis
5. Staphylococcal infection

21.184 Compulsory health insurance fund covers:


1. Preventive health care
2. Curative health care
3. Medical rehabilitation, nursing and social services
4. Sickness and maternity benefits

lsmusis.lsmuni.lt/Klausimai/Spausdinti?Length=0?Kalba=EN&KategorijaId=123&Kalbos_input=EN&Kalbos=EN&KategorijaEn_input=Mix&Kategorij… 22/38
3/27/2019 LSMUSIS
21.185 Correlation coefficient indicates:
1. Linear relationship between two variables
2. The strength of relationship between two variables
3. Relationship positive or negative
4. Causation

21.186 What are the indications for surgical treatment of pharyngoesophageal (Zenker) diverticulum:
1. Large diverticulum
2. Diverticulitis
3. Aspiration during eating and drinking
4. Dysphagia

21.187 In which compounde is toxic carbon monoxide concentration:


1. Autocar exhaust gas
2. Fire smoke
3. Domestic heater‘s fume in incomplete burning process
4. Drain gas
5. Main gas

21.188 In amanita phalloides poisoning is typical:


1. Latent period 6 – 48 hours
2. Liver damage symptoms
3. Markedly elevated liver enzymes
4. Markedly reduction of protrombine time (SPA)
5. Nausea, womiting, diarrhoea

21.189 Dry cough is very typical symptom in case of which disease:


1. Acute bronchitis
2. Influenza
3. Atypical pneumonia
4. Acute streptococcical tonsillitis
5. Acute sinusitis

21.190 A 38-year old woman complains of 2 days of low back pain, after she did some heavy lifting. Limitation of
full flexion and rotation, paravertebral muscle tenderness are present. Motor and sensory examinations of her legs
are normal. In the straight-leg-raising test, you can raise each of her legs (with the knee in extension) 20°with
causing radicular pain. Which of the following should you recommend:
1. Strict bed rest for 2-3 days before radicular pain relief
2. Nonsteroidal antiinflamatory drugs for 5-7 days
3. Daily muscle relaxant for 1 week
4. Physical therapy after pain relief

21.191 How diagnosis of occupational diseases is established:


1. By symptoms, medical documentation
2. Symptoms related to occupational disease
3. Workplace occupational risk assessment data
4. Preventive health inspection data
5. Social insurance sertificate accounts

21.192 Syndromes characteristic for long term exposure to vibration:


1. Vertigo
2. Sensorineural polyneuropathy
3. Angiospastic syndrome
4. Reiter‘s syndrome
5. Asthenopy

21.193 In radiographic investigations of skeleton it is important to perform radiographs in:


1. Three projections
2. Single projection
3. Two projections (usually axial and oblique)
4. In two perpendicular projections (usually anteroposterior and lateral)

lsmusis.lsmuni.lt/Klausimai/Spausdinti?Length=0?Kalba=EN&KategorijaId=123&Kalbos_input=EN&Kalbos=EN&KategorijaEn_input=Mix&Kategorij… 23/38
3/27/2019 LSMUSIS
21.194 Bone age of a patient is detected from
1. Epiphyseal lines
2. Ossification nuclei
3. Radiographs of hands and (or) wrists
4. Radiographs of pelvic bones

21.195 Which of these events is critical according to official emergency criteria:


1. Illegal attendance at the nuclear object
2. Theft of the radioactive material
3. Unattended radioactive material
4. Any event at the nuclear object requiring physical interference

21.196 Which of these events is critical according to official emergency criteria:


1. One single case of high contagious disease
2. Increase of morbidity of contagious disease
3. Outbreak (two and more interrelated cases of high contagious disease)
4. Epidemics or pandemic related to high mortality

21.197 Histologic sections of a follicular carcinoma of the thyroid would characteristically reveal:
1. An amyloid stroma intermixed with neoplastic C cells
2. Undifferentiated anaplastic cells with occasional giant cells
3. Undifferentiated anaplastic
4. Blood vessel and capsular invasion by malignant thyroid follicles

21.198 Which pathologic processes can cause malabsorbtion of CSF and hydrocephalus:
1. Subarachnoidal haemorrhage
2. Meningoencephalitis
3. Severe traumatic brain injury
4. Cerebral atrophy
5. Lacunar ischemic stroke

21.199 What signs indicate, that the wound is the entrance of the gunshot:
1. The wound is of irregular form and uneven margins, no tissue defect
2. The wound is linear with smooth borders, no abrading of the skin
3. The wound is linear with smooth borders, with injury to the bone
4. The wound is round or oval with smooth borders, and defect of tissue, surrounded by a margin of
abraded skin and stained wound edges

21.200 Which proposition concerning gout are correct:


1. Gout is inflammatory arthropathy of unknown etiology
2. Gout is induced by tissue deposition of urates
3. Young females are the most part of patients
4. Hyperuricemia is typical for idiopathic and for secondary gout

21.201 Indicate causes of hyperuricemia:


1. Lymphoproliferative diseases
2. Dietary excess of purins
3. Chronic renal failure
4. Diuretics lowering excretion of urates

21.202 For treatment of acute gouty arthritis are recommended:


1. Nonsteroidal anti-inflammatory drugs
2. Colchicin
3. Glucocorticoids
4. Allopurinol

21.203 What is characteristic for chronic (tophaceus) gout:


1. Development after several years of intermittent arthritis
2. Development after the first 2-3 episodes of acute gouty arthritis
3. Typical clinical feature is chronic polyarthritis
4. Typical duration of arthritis is 3-7 days

21.204 Acite monoarthritis is typical for:


lsmusis.lsmuni.lt/Klausimai/Spausdinti?Length=0?Kalba=EN&KategorijaId=123&Kalbos_input=EN&Kalbos=EN&KategorijaEn_input=Mix&Kategorij… 24/38
3/27/2019 LSMUSIS
1. Septic arthritis
2. Rheumatoid arthritis
3. Podagra
4. Rheumatic fever

IV type tasks. Choose only one best answer

21.205 Mr. Johnson, a 68-year-old retired auto mechanic develops fever, myalgias, and non-productive cough.
You diagnose influenza, prescribe rest and analgesics, and after three days he is feeling better. He then develops
a sudden shaking chill and a cough producing rusty sputum. He calls your office because he is coughing so much
he can't smoke any more. What do you tell him:
A. You‘ve wanted him to quit smoking for years
B. Drive to a pharmacy and buy a cough suppressant
C. Come to the hospital/office for laboratory tests
D. Drive to a pharmacy and ask him to pick up the antibiotics

21.206 Mr. Johnson, a 68-year-old retired auto mechanic develops fever, myalgias, and non-productive cough.
You diagnose influenza, prescribe rest and analgesics, and after three days he is feeling better. He then develops
a sudden shaking chill and a cough producing rusty sputum. He calls your office because he is coughing so much
he can't smoke any more. The laboratory reports his sputum gram stain as gram + diplococci. Culture will be
available tomorrow and antibiotic-sensitivity the next day. What should your initial therapy be:
A. Penicillin
B. Cephalosporins
C. Quinolones
D. Amantadine

21.207 A two-year-old boy was brought to the Emergency Room in November because of vomiting and diarrhea.
He was admitted to the hospital because of fever (39.5) and dehydration. He had a slight tachycardia and
respirations problem. Initial tests showed that there were no fecal leukocytes. Microscopic examination of a fecal
smear revealed no indication of parasites. Routine stool cultures were done and were all negative. What is the
significance of the fact that no fecal leukocytes were found:
A. He does not have an infection
B. He has a viral diarrhea
C. He has diarrhea caused by a parasite
D. He has a noninflammatory or secretory diarrhea

21.208 40 - year old man has a dilated cardiomyopathy, phlebitis of the profound veins of the left leg and failure of
both parts of the heart. Pulmonary embolism repeated for several times. After seven days therapy with
thrombolysis and heparin 40000 IU/day administration clinical status improved shortly but episodes of pulmonary
embolism repeats again. Lab of activated partial thromboplastin time (aPTT) started to decrease from 79 seconds
to 22 seconds despite increased dose of Heparin. How can we improve treatment:
A. Increase the dose of heparin till 60-80.000 IU per day
B. Add warfarin therapy
C. Add antithrombine III and fresh frozen plasma
D. Repeat thrombolysis and add aspirin 150 mg per day

21.209 Two women, one of blood types AB;M, the other of B;MN claim a baby of blood types O;M as their child.
Indicate: The child genotype:
A. Iº Iº LM LM
B. Iº Iº LM LN
C. IA Iº LM LM
D. IB Iº LM LM

21.210 Two women, one of blood types AB;M, the other of B;MN claim a baby of blood types O;M as their child.
Indicate: The first woman:
A. Could not be the mother
B. Could be if her genotype was IA IB LM LM
C. Could be if her genotype was IA IB LM LN
D. Could be if either genotype

21.211 Two women, one of blood types AB;M, the other of B;MN claim a baby of blood types O;M as their child.
Indicate: The second woman:

lsmusis.lsmuni.lt/Klausimai/Spausdinti?Length=0?Kalba=EN&KategorijaId=123&Kalbos_input=EN&Kalbos=EN&KategorijaEn_input=Mix&Kategorij… 25/38
3/27/2019 LSMUSIS
A. Could not be the mother
B. Could be if her genotype was IB IB LM LN
C. Could be if her genotype was IB Iº LM LN
D. Could be if either genotype

21.212 Two women, one of blood types AB;M, the other of B;MN claim a baby of blood types O;M as their child.
Indicate: The father blood types:
A. B; M
B. B; MN
C. 0; M
D. 0; MN
E. All of above

21.213 A man has a rare skin disease, ichthyosis caused by the X linked recessive allele. The main feature of the
disease, dry and scaley skin is not apparent in infancy. In the man family, both the daughter and daughter’s
offspring: the teenage daughter, and the newborn son appear to be healthy. Draw the family pedigree and explain:
What proportion of the daughter would be the carrier:
A. 0%
B. 25%
C. 50%
D. 100%

21.214 A man has a rare skin disease, ichthyosis caused by the X linked recessive allele. The main feature of the
disease, dry and scaley skin is not apparent in infancy. In the man family, both the daughter and daughter’s
offspring: the teenage daughter, and the newborn son appear to be healthy. Draw the family pedigree and explain:
What proportion of the grandson would be affected:
A. 0%
B. 25%
C. 50%
D. 100%

21.215 Evaluation of microclimate of living room reveals such data: air temperature 20 0C, relative humidity - 25
%, speed of air movement – 0,1 m/s. Which parameter do not correspond to normal value:
A. Air temperature
B. Relative humidity
C. Speed of air movement
D. All parameters

21.216 Performing chicken product microbiological investigation, detected, that chicken is measly… For patients
stated acute intestine infection. The disease manifested with diarrhea, fever, cramps and head pains. What kind of
the disease is it:
A. Bacterial genesis
B. Not bacterial genesis
C. Invasive genesis

21.217 Performing chicken product microbiological investigation, detected, that chicken is measly… For patients
stated acute intestine infection. The disease manifested with diarrhea, fever, cramps and head pains. What is the
agent of the disease:
A. Salmonella type bacteria
B. Aflatoksines
C. Trichinella spiralis

21.218 The 24-yr-old female presented with frequent micturition, the color of urine similar to rust, fever, nausea,
and permanent pain in the lumbar region.. Three weeks ago the patient suffered from a sore throat, fever up to
39ºC. She used antibiotics, and did not refer to a doctor. The results of physical and laboratory examination:
temperature 37,8 ºC, periorbital swellings, arterial blood pressure 134/88 mm Hg, increased serum ASO titer,
reduced serum complement C3 concentration, in urine – 4 leucocytes/hpf, 96 red blood cells/hpf, specific gravity
1,020. Which syndrome do you suspect according to the indicated symptoms:
A. Nephrotic
B. Nephritic
C. Acute renal failure
D. Renal hypertension

lsmusis.lsmuni.lt/Klausimai/Spausdinti?Length=0?Kalba=EN&KategorijaId=123&Kalbos_input=EN&Kalbos=EN&KategorijaEn_input=Mix&Kategorij… 26/38
3/27/2019 LSMUSIS
21.219 The 24-yr-old female presented with frequent micturition, the color of urine similar to rust, fever, nausea,
and permanent pain in the lumbar region.. Three weeks ago the patient suffered from a sore throat, fever up to
39ºC. She used antibiotics, and did not refer to a doctor. The results of physical and laboratory examination:
temperature 37,8 ºC, periorbital swellings, arterial blood pressure 134/88 mm Hg, increased serum ASO titer,
reduced serum complement C3 concentration, in urine – 4 leucocytes/hpf, 96 red blood cells/hpf, specific gravity
1,020. Which laboratory marker may confirm the diagnosis:
A. Hypercreatininemia
B. RBC casts
C. Proteinuria
D. Reduced urine pH

21.220 38 years old patient during accident hit right side of the chest, suddently felt the intencive dyspnoe .
Objective signs: respiratory rate 30 times/min., heart rate 120 times/min., blood pressure 120/60 mmHg, absent
breath sounds in the right. Preliminal diagnosis:
A. Pneumonia
B. Acute bronchitis
C. Pneumothorax
D. Pulmonary embolism

21.221 38 years old patient during accident hit right side of the chest, suddently felt the intencive dyspnoe .
Objective signs: respiratory rate 30 times/min., heart rate 120 times/min., blood pressure 120/60 mmHg, absent
breath sounds in the right. Which investigation confirms diagnosis:
A. 2D heart echo
B. Chest radiography
C. Pleural echoscopy
D. Chest computer tomography

21.222 38 years old patient during accident hit right side of the chest, suddently felt the intencive dyspnoe .
Objective signs: respiratory rate 30 times/min., heart rate 120 times/min., blood pressure 120/60 mmHg, absent
breath sounds in the right. Treatment:
A. Pleural puncture
B. Pleural cavity drainage
C. Antibioc therapy
D. Thrombolysis

21.223 25 years old male, non-smoker, get sick suddenly. He has fever 39-40o C, productive cough, chest pain.
Physical examination: Hyperthermia fever 39°C, auscultation –rales in the right lover part of the chest. What is
preliminary diagnosis:
A. Exacerbation of chronic obstructive lung disease.
B. Pneumonia of right lower lobe.
C. Acute upper airway infection.

21.224 25 years old male, non-smoker, get sick suddenly. He has fever 39-40o C, productive cough, chest pain.
Physical examination: Hyperthermia fever 39°C, auscultation –rales in the right lover part of the chest. What
studies have to be you performed to confirm the diagnosis:
A. Chest plain and right side X-ray, microscopic and microbiological
B. Cytological evaluation of sputum, blood test, gas diffusion test.

21.225 A 17 years old woman suffers from syncope. Torsades de pointes have been registered 3-4 times a year
during the attack. Physical examination normal, ABP – 120/70 mmHg. ECG on admission a sinus rhythm 70 bpm,
PR – 0,18 s, Q-TC – 0,46 s. What disease should be suspected:
A. Myocarditis
B. Congenital heart deffect
C. Congenital long QT interval syndrome
D. WPW syndrome

21.226 A 17 years old woman suffers from syncope. Torsades de pointes have been registered 3-4 times a year
during the attack. Physical examination normal, ABP – 120/70 mmHg. ECG on admission a sinus rhythm 70 bpm,
PR – 0,18 s, Q-TC – 0,46 s. Which of these treatments is contraindicated for the patient:
A. Amiodarone
B. Beta-adrenoblockers
C. Cardiac pacing

lsmusis.lsmuni.lt/Klausimai/Spausdinti?Length=0?Kalba=EN&KategorijaId=123&Kalbos_input=EN&Kalbos=EN&KategorijaEn_input=Mix&Kategorij… 27/38
3/27/2019 LSMUSIS
D. Implantation of cardioverter-defibrillator

21.227 Female, 36 years old. Complains: Feeling of indigestion, early satiety, nausea, upper abdominal bloating.
Symptoms are more than 10 years, episodes of worsening 5-8 times per year, with 1-2 months duration.
Exacerbations are caused by stress, irregular nutrition. Several times there were investigations performed
(Esophagogastroduodenoscopy, abdominal ultrasound, colonoscopy): without significant pathology. Different
medications in short courses administered, not always effective. Last esophagogastroduodenoscopy performed 5
months ago – without pathology, Helicobacter pylori not found. In the past patient used to use antidepressants, last
dose 6 months ago. Physical examination – normal, body mass – stable. Blood investigations – normal. What is
the preliminary diagnosis:
A. Exacerbation of peptic ulcer disease
B. Exacerbation of chronic gastritis
C. unctional dyspepsia (dysmotility-like variant)
D. Gastroesophageal reflux disease

21.228 Female, 36 years old. Complains: Feeling of indigestion, early satiety, nausea, upper abdominal bloating.
Symptoms are more than 10 years, episodes of worsening 5-8 times per year, with 1-2 months duration.
Exacerbations are caused by stress, irregular nutrition. Several times there were investigations performed
(Esophagogastroduodenoscopy, abdominal ultrasound, colonoscopy): without significant pathology. Different
medications in short courses administered, not always effective. Last esophagogastroduodenoscopy performed 5
months ago – without pathology, Helicobacter pylori not found. In the past patient used to use antidepressants, last
dose 6 months ago. Physical examination – normal, body mass – stable. Blood investigations – normal. Which
treatment could be best for this patient:
A. Antidepressants and neuroleptics
B. Procinetics (metoklopramide, domperidone)
C. Proton pump inhibitors
D. Antibacterial medications

21.229 21 year old female was admitted to the hospital because of bloody diarrhoea up 14 times per day lasting
for 3 weeks, fever -38.0 C, nausea, weakness. Patient was taking ciprofloxacin orally for 5 days, because she
thought having infection from food poisoning. Therapy was not effective. In the hospital investigations were done:
Hb – 97 g/l, CRB -35 mg/l, stool culture: no pathogenic flora detected. Sigmoidoscopy: erithema, oedema,
granularity, spontaneous bleeding, oval ulcers. What is your diagnosis:
A. Severe Crohn’s disease
B. Acute infectious colitis of unknown ethiology
C. Severe attack of ulcerative colitis
D. Ischemic colitis

21.230 21 year old female was admitted to the hospital because of bloody diarrhoea up 14 times per day lasting
for 3 weeks, fever -38.0 C, nausea, weakness. Patient was taking ciprofloxacin orally for 5 days, because she
thought having infection from food poisoning. Therapy was not effective. In the hospital investigations were done:
Hb – 97 g/l, CRB -35 mg/l, stool culture: no pathogenic flora detected. Sigmoidoscopy: erithema, oedema,
granularity, spontaneous bleeding, oval ulcers. What therapy you will prescribe:
A. Prednisolone 1mg/kg i/v, broad spectrum antibiotics, electrolyte solutions i/v
B. Broad spectrum antibiotics, Smecta, blood transfusion
C. Azathioprine, mesalazine, electrolyte solutions i/v
D. Methotrexate i/m

21.231 30-years –old teacher (woman) was sent for nephrologist consultation by family doctor. Her complaints –
during last 6 months e tibial oedema and exertional dyspnea appeared. Family doctor on examination found
arterial hypertension (160/105 mmHg) and proteinuria (≥ 3,0 g/l). In past – appendectomia, pulmonary
tuberculosis, 18 years ago I type of diabetes was diagnosed and till now she is on insulin therapy. On examination
– severe tibial oedema. BP -160/100 mmHg. Proteinuria 5 g/l, WBC – 4-6 per h.p.f., RBC -1-2 per h.p.f. Hb – 120
g/l, serum urea 10 mmol/l, serum creatinine - 100 µmol/l, serum albumin 24g/l. Your preliminary diagnosis, further
examination and treatment:
A. First type diabetes, diabetic nephropathy, nephrotic syndrome. We must measure 24 h proteinuria and
glomerular filtration rate. Good glycemic control, intensive antihypertensive therapy (BP < 120/80 mmhg)
with ACEI or/and ARB, reduction of protein intake till 0,8 g/kg, NaCl till 4 g/24h, loop diuretics.
B. Chronic glomerulonephritis, nephrotic syndrome (minimal change nephropathy? focal segmental
glomerulosclerosis? membranous nephropathy? membranoproliferative glomerulonephritis?). Renal biopsy
and pathogenetic therapy (if possible). Antihypertensive drugs, loop diuretics.

lsmusis.lsmuni.lt/Klausimai/Spausdinti?Length=0?Kalba=EN&KategorijaId=123&Kalbos_input=EN&Kalbos=EN&KategorijaEn_input=Mix&Kategorij… 28/38
3/27/2019 LSMUSIS
C. Renal amyloidosis (secondary, probably due to pulmonary tuberculosis). Chest rentgenograms, chest CT,
renal biopsy. In case of active pulmonary tuberculosis and renal amyloidosis, treatment of tuberculosis by 3-
4 drugs. Antihypertensive drugs, loop diuretics.

21.232 66-year-old retired book-keeper after intensive vomiting and diarea with fever till 380 C was admitted to
hospital due to oliguria. On examination: dry mouth and skin, bad turgor of skin. Urinalysis – no urine – anuria. Hb
– 70g/l, serum potassium – 7,5 mmol/l, serum sodium – 158 mmol/l, serum creatinin - 400µmol/l, urea 32 mmol/l.
Ultrasonography: length of right kidney – 8 cm , parenchyma – 0,8 cm; length of left kidney – 9,6 cm, parenchyma
– 0,7 cm. Diagnosis and treatment:
A. Acute renal failure due to severe hypovolemia. Because we have anuria with severe dehidratation – we
must start fluids intravenously. If anuria will persist – urgent hemodialysis.
B. We have a case of end stage renal failure (anemia, small kidney), so we can conclude, that vomiting and
diarea are due to uremia. We must start urgent hemodialysis, create arteriovenous fistula and patient will be
on hemodialysis all the remaining life.
C. We have a case of chronic renal failure (anemia, small kidney), but we en suspect that possible food
infection (fever, vomiting, diarea) worsened renal function. We must puncture central vein, measure central
venous pressure, correct volemia by intravenous fluids. Big possibility of need for urgent hemodialysis, but
in future it is possible, that renal function can improve.

21.233 18 years old man was seen in family doctor office due to swollen and painfull left knee joint and bleeding
from gums. Without any obvious reason patient is bleeding in various joints every two – three weeks since
childhood. Patient admitted that his cousin from childhood has “bleeding disorder”. What is preliminary diagnosis:
A. Acute leukemia
B. Rheumatoid arthritis
C. Hemophilia A or B
D. Aplastic anemia

21.234 18 years old man was seen in family doctor office due to swollen and painfull left knee joint and bleeding
from gums. Without any obvious reason patient is bleeding in various joints every two – three weeks since
childhood. Patient admitted that his cousin from childhood has “bleeding disorder”. Which test is most usefull to
make diagnosis:
A. Bone marrow aspiration
B. Left knee joint punction
C. Platelet aggregation tests
D. Activated partial thromboplastin time, factor VIII and IX activity assays

21.235 70 years old women was seen in family doctor office due to bleeding from nose and dark stool. 2 months
ago she was diagnosed with deep vein thrombosis in left leg. From that time she is taking“blood thinning” drug.
Patient admitted no personal or family bleeding tendency. Laboratory tests results are: Hb 130 g/l, eryt.
5,4x1012/1, platelets 280x109 /1, leukocytes 5,8x109/1, leukocytes differential is normal, ESR 10 mm/hour.
Activated partial thromboplastin time is 80 seconds, prothrombin complex activity (SPA) 1%, INR – 8,0, fibrinogen
4,0 g/l. What is preliminary diagnosis:
A. Hemophilia A or B
B. Acute leukemia
C. Congenital factor VII deficiency
D. Overdosage of varfarin

21.236 70 years old women was seen in family doctor office due to bleeding from nose and dark stool. 2 months
ago she was diagnosed with deep vein thrombosis in left leg. From that time she is taking“blood thinning” drug.
Patient admitted no personal or family bleeding tendency. Laboratory tests results are: Hb 130 g/l, eryt.
5,4x1012/1, platelets 280x109 /1, leukocytes 5,8x109/1, leukocytes differential is normal, ESR 10 mm/hour.
Activated partial thromboplastin time is 80 seconds, prothrombin complex activity (SPA) 1%, INR – 8,0, fibrinogen
4,0 g/l. Which treatment must be administered:
A. Transfusions of platelets
B. Desmopressin
C. Prothrombin complex concentrate, vitamin K i/v
D. Cryoprecipitate

21.237 A 82 yrs old alert woman is hospitalised with pneumonia. On the 2nd day of hospitalisation she complains
of feeling tired and does not want to talk much. She is distracted by the voices in the hallway, so that questions
must be repeated several times. She slurs her words and has significant difficulty with word finding. What is
diagnosis?
lsmusis.lsmuni.lt/Klausimai/Spausdinti?Length=0?Kalba=EN&KategorijaId=123&Kalbos_input=EN&Kalbos=EN&KategorijaEn_input=Mix&Kategorij… 29/38
3/27/2019 LSMUSIS
A. Alzheimer's disease
B. Dementia with Lewy bodies
C. Delirium
D. Depression

21.238 A 82 yrs old alert woman is hospitalised with pneumonia. On the 2nd day of hospitalisation she complains
of feeling tired and does not want to talk much. She is distracted by the voices in the hallway, so that questions
must be repeated several times. She slurs her words and has significant difficulty with word finding. What is the
treatment?
A. Levodopa
B. Haloperidol
C. Antidepressants
D. Diazepam

21.239 A 70 yrs old man has been complaining of not having enough energy. He has slowed down over the last
two years. Examination reveals lead pipe rigidity of all four limbs (worse on one side) and slowness of repetitive
finger and foot movement. What is the diagnosis?
A. Alzheimer's disease
B. Dementia with Lewy body
C. Parkinson's disease
D. Osteoarthrosis
E. Stroke

21.240 4 year old boy became ill at home. Temperature is 390C, he refused drinking and eatin g, was vomiting
twice. Objective findings: red soft and hard palate, tonsils with pus, enlarged lymph nodes in the neck, no rash on
the skin, no changes on lung auscultation and abdominal palpation. Blood examination; WBC-16,5.109/l, shift to
the left, CRP-52,3 mg/l. What is the diagnosis of the disease:
A. Virus pharyngitis
B. Bacterial pharyngotonzillitis
C. Infectious mononucleosis
D. Scarlet fever

21.241 4 year old boy became ill at home. Temperature is 390C, he refused drinking and eatin g, was vomiting
twice. Objective findings: red soft and hard palate, tonsils with pus, enlarged lymph nodes in the neck, no rash on
the skin, no changes on lung auscultation and abdominal palpation. Blood examination; WBC-16,5.109/l, shift to
the left, CRP-52,3 mg/l. What treatment should be given:
A. Fluids i/v, antipyretics
B. Penicillin, antipyretics, fluids i/v
C. Fluids i/v, H2 receptor antagonists
D. Antipyretics, gargling of the throat, inhaled dezynfecting fluids for the throat

21.242 A 29 years old female patient complaints of easy fatigability, has to have a rest frequently; after the rest
feels much stronger. These complaints appeared after the delivery. During the last week, intermittent double vision
appeared, mostly in the afternoon and in the evening. Sometimes there is some difficulty with swallowing,
sometimes – some shortness of the breath. Becomes tired when speaking, the voice fades and becomes nasal,
therefore avoids to communicate, became sensitive, depressed. Diazepam had just a negative influence on the
symptoms. What disease would you suspect:
A. Postpartum depression
B. Myasthenia gravis
C. Lyme borreliosis
D. Multiple sclerosis

21.243 A 29 years old female patient complaints of easy fatigability, has to have a rest frequently; after the rest
feels much stronger. These complaints appeared after the delivery. During the last week, intermittent double vision
appeared, mostly in the afternoon and in the evening. Sometimes there is some difficulty with swallowing,
sometimes – some shortness of the breath. Becomes tired when speaking, the voice fades and becomes nasal,
therefore avoids to communicate, became sensitive, depressed. Diazepam had just a negative influence on the
symptoms. What investigation (consultation) is needed as a first choice:
A. Blood sample for antibodies against b. burgdorferi
B. Consultation of psychiatrist
C. Pyridostigmine test
D. Cerebral MRI
lsmusis.lsmuni.lt/Klausimai/Spausdinti?Length=0?Kalba=EN&KategorijaId=123&Kalbos_input=EN&Kalbos=EN&KategorijaEn_input=Mix&Kategorij… 30/38
3/27/2019 LSMUSIS
21.244 A 35 years old man complaints of progressive weakness of the limbs, which started in the lower extremities
and evolved to the upper ones, and of numbness of the foots and hands. The patient had a fever 2 weeks ago and
was diagnosed as having acute upper respiratory infection. On examination, the brachioradialis and the ankle
reflexes were found absent, and “gloves” and “stockings” type impairment of sensations was elucidated. What
disease would you suspect:
A. Cerebrovascular accident
B. Encephalitis
C. Acute inflammatory polyneuropathy
D. Acute myopathy

21.245 A 35 years old man complaints of progressive weakness of the limbs, which started in the lower extremities
and evolved to the upper ones, and of numbness of the foots and hands. The patient had a fever 2 weeks ago and
was diagnosed as having acute upper respiratory infection. On examination, the brachioradialis and the ankle
reflexes were found absent, and “gloves” and “stockings” type impairment of sensations was elucidated. What
investigation would confirm your diagnosis:
A. CT scan
B. Lumbar puncture
C. Electroneuromyogrphy and lumbar puncture
D. Electroneuromyography and muscle biopsy

21.246 Fifty-year-old patient is treated in the intensive care unit for sepsis and multiple organ failure. Artificial
ventilation of the lungs is applied. Hemodynamics is supported by vasopressors and fast fluid infusion. The patient
is going to be operated on for acute abdomen. What ASA(American Society of Anesthesiologists) class does this
patient belongs:
A. I class
B. III class
C. V class
D. VI class

21.247 Twenty-six-year-old patient suffering myastenia gravis is prepared for thymectomy operation. The standard
monitoring- ECG, non-invasive blood pressure, SpO2, tempereture, ET CO2- is going to be used. What additional
monitoring would you choose for this patient:
A. Invasive blood-pressure
B. Monitoring of the depth of anaesthesia
C. Monitoring of the neuromuscular blocade

21.248 A 70-year-old man has undergone anterior resection for carcinoma of the rectum. He is extubated in the
operating room (OR). In the recovery room, he is found to be restless with an HR of 136 bpm and a BP of 144/80
mm Hg. ABG analysis on room air reveals pH, 7.24; PCO2, 60 mm Hg; PO2, 54 mm Hg; HCO3, 25 mEq/L; and
SaO2, 90%. The physiologic status can best be described as which of the following:
A. Respiratory alkalosis
B. Respiratory acidosis
C. Metabolic acidosis
D. Metabolic alkalosis
E. Combined respiratory and metabolic acidosis

21.249 Coronarography revealed 80 percent narrowing in the left and right main coronary arteries of patient of 70
years of age. What is the recommended method of treatment of the patient:
A. Conservative
B. Angioplasty
C. Stenting
D. Coronary bypass

21.250 Patient complains with stranguria frequency and nicturia 3x per night. Residual urine volume 72ml, IPSS
18. Prostate volume 42ml, PSA 1,4ng/ml. What treatment modality would you choose:
A. Transurethral prostate resectio
B. Retropubic adenectomy
C. α blockers
D. Fitotherapy
E. α blockers in combinatio with su 5- α reductase inhibitors
F. Observation

lsmusis.lsmuni.lt/Klausimai/Spausdinti?Length=0?Kalba=EN&KategorijaId=123&Kalbos_input=EN&Kalbos=EN&KategorijaEn_input=Mix&Kategorij… 31/38
3/27/2019 LSMUSIS
21.252 The 8 years old boy became sick around 7 hours before. The disease begins with abdominal pain and
nausea. He doesn’t sleep during the night. Primary care doctor found: skin pallor, dry lips, temperature 37.3oC,
heart rate 100 /min. Local abdominal pain in the right lower abdominal quadrant and tensed abdominal muscles
were found during examination. What is your preliminary diagnosis:
A. Gastritis acuta
B. Pancreatitis acuta
C. Coprostasis
D. Appendicitis acuta
E. Cholecystitis acuta

21.253 A 25 year old primipara came to the doctor complaining of a fever with chills (t- 39ºC), sudden appearance
of flank pain and nausea. Gestational age is 30 weeks. The patient has good perception of fetal movements. What
is the preliminary diagnosis:
A. Threatening preterm labor
B. Acute peylonephritis
C. Viral infection
D. Asymptomatic bacteriuria

21.254 A 25 year old primipara came to the doctor complaining of a fever with chills (t- 39ºC), sudden appearance
of flank pain and nausea. Gestational age is 30 weeks. The patient has good perception of fetal movements. What
is our further course of action:
A. Tocolysis and maturing of the fetal lungs
B. Immediate termination of the pregnancy
C. Antibiotic therapy and assuring urine outflow
D. Antiviral and antipyretic drugs

21.255 A 30 year old patient came to the emergency room complaining of severe pain in the lower abdomen in
addition to vaginal bleeding. The patient admits that her last normal period was 7 weeks earlier, in the last 2 weeks
there had been a „bloody show“. The pain is increasing in severity. The pregnancy test is positive. Upon
admission, the patient is stable. Upon examination there is a mild degree of peritioneal irritation. US examination
reveals a small amount of free fluid in the peritoneal cavity, no intrauterine pregnancy is evident. What is the
preliminary diagnosis:
A. Torsion of an ovarian cyst
B. Ectopic pregnancy
C. Spontaneous abortion
D. Endometriosis

21.256 A 30 year old patient came to the emergency room complaining of severe pain in the lower abdomen in
addition to vaginal bleeding. The patient admits that her last normal period was 7 weeks earlier, in the last 2 weeks
there had been a „bloody show“. The pain is increasing in severity. The pregnancy test is positive. Upon
admission, the patient is stable. Upon examination there is a mild degree of peritioneal irritation. US examination
reveals a small amount of free fluid in the peritoneal cavity, no intrauterine pregnancy is evident. What should our
course of action be:
A. Antibiotic therapy and i/v line
B. Uterine d & c
C. Laparoscopy
D. Analgetics and a follow up within 1 week

21.257 A 24 year old woman noticed worsenning of the vision in the left eye two days ago. Vision is getting worse.
It hurts behind the eyeball, moving the eyes. She uses peroral contraceptives. Examination data: V-OS=0,1 not
correctable, V-OD=1,0; OU optical media transparent. Ophthalmoscopy - OS OND temporal margin unclear, at the
6th hour small flameshape hemorrhage. OD fundus is normal. Preliminary diagnosis:
A. Thrombosis of the central retinal vein
B. Optic neuritis (papillitis)
C. Retrobulbar tumour

21.258 A 24 year old woman noticed worsenning of the vision in the left eye two days ago. Vision is getting worse.
It hurts behind the eyeball, moving the eyes. She uses peroral contraceptives. Examination data: V-OS=0,1 not
correctable, V-OD=1,0; OU optical media transparent. Ophthalmoscopy - OS OND temporal margin unclear, at the
6th hour small flameshape hemorrhage. OD fundus is normal. What is the most suspected cause of disease:
A. Use of peroral contraceptives
B. Multiple sclerosis
lsmusis.lsmuni.lt/Klausimai/Spausdinti?Length=0?Kalba=EN&KategorijaId=123&Kalbos_input=EN&Kalbos=EN&KategorijaEn_input=Mix&Kategorij… 32/38
3/27/2019 LSMUSIS
C. Intoxication with unknown neurotoxic agent
D. Temporal arteriitis.

21.259 A 24 year old woman noticed worsenning of the vision in the left eye two days ago. Vision is getting worse.
It hurts behind the eyeball, moving the eyes. She uses peroral contraceptives. Examination data: V-OS=0,1 not
correctable, V-OD=1,0; OU optical media transparent. Ophthalmoscopy - OS OND temporal margin unclear, at the
6th hour small flameshape hemorrhage. OD fundus is normal. What additional examination may confirm the
diagnosis:
A. Dark adaptation, ESR, C reactive protein, coagulation tests
B. Fluoresceine angiography, ultrasonic B scan and Dopplerography
C. Perimetry, CT scan, MRI

21.260 Parents noticed that the left eye of their 1 year old son sometimes squints and the pupil is shining white.
Examination data: position of the eyes is normal, intermittent squint of the left, with deviation to the medial side.
Ophthalmoscopic examination in the left eye fundus paracentrally pearly pink colour lesion with promination with
fine blood vessels on the surface, resembles cottage cheese. Preliminary diagnosis:
A. Retinoblastoma
B. Primature retinopathy (retrolental fribroplasia)
C. Juvenile retinoshysis

21.261 Parents noticed that the left eye of their 1 year old son sometimes squints and the pupil is shining white.
Examination data: position of the eyes is normal, intermittent squint of the left, with deviation to the medial side.
Ophthalmoscopic examination in the left eye fundus paracentrally pearly pink colour lesion with promination with
fine blood vessels on the surface, resembles cottage cheese. What additional examination may confirm the
diagnosis:
A. Perimetry, optical conherent tomography, laser scanning ophthalmoscopy
B. Ultrasonic B scan, diaphanoscopy, CT scan, MRI.
C. Dark adaptation, fluorescence angiography

21.262 Hospital reported that they had the following bed occupancy rates in selected departments: surgery –
318.6 days; therapeutics – 321.5 days; obstetrics – 210.2 days; rehabilitation – 410 days. In which department
there are surplus of beds:
A. Surgery
B. Therapeutics
C. Obstetrics
D. Rehabilitation

21.263 Hospital reported that they had the following bed occupancy rates in selected departments: surgery –
318.6 days; therapeutics – 321.5 days; obstetrics – 210.2 days; rehabilitation – 410 days. In which department
there are lack of beds:
A. Surgery
B. Therapeutics
C. Obstetrics
D. Rehabilitation

21.264 Yesterday patient accidentally aspirated a foreign organic body. Today he feels better. Chest
roentgenograms revealed atelectasis of the right lung lower and middle lobes. Which bronchus is the foreign body
in:
A. Right upper lobar bronchus
B. Intermediary bronchus
C. Middle lobar bronchus
D. Right lower bronchus
E. Left main bronchus

21.265 Yesterday patient accidentally aspirated a foreign organic body. Today he feels better. Chest
roentgenograms revealed atelectasis of the right lung lower and middle lobes. An attempt to remove this foreign
body using fibrobronchoscope was not successful. What other method should be used in this situation:
A. Bronchial catheterization
B. Rigid bronchoscopy
C. Thoracotomy and resection of atelectatic lobes
D. Tracheostomy
E. Mediastinoscopy

lsmusis.lsmuni.lt/Klausimai/Spausdinti?Length=0?Kalba=EN&KategorijaId=123&Kalbos_input=EN&Kalbos=EN&KategorijaEn_input=Mix&Kategorij… 33/38
3/27/2019 LSMUSIS
21.266 The young woman admited to the hospital. Clinical examination had shown: unconscious, face cyanosis,
Kussmaul type breathing 30 times per minute, tachycardic hart rate, arterial pressure – 100/70 mmHg, no signes
of trauma externaly. Leading persones told that the patient ingested not known amount of not verificated chemical
substance similar to salt with intention to suicide. In the blood sampling blood had brown, chocolate like color.
What poisoning we must think about at the first:
A. Opioides poisoning
B. Carbon monoxide poisoning
C. Nitrates (nitrat salt) poisoning
D. Organophosphates poisoning

21.267 The young woman admited to the hospital. Clinical examination had shown: unconscious, face cyanosis,
Kussmaul type breathing 30 times per minute, tachycardic hart rate, arterial pressure – 100/70 mmHg, no signes
of trauma externaly. Leading persones told that the patient ingested not known amount of not verificated chemical
substance similar to salt with intention to suicide. In the blood sampling blood had brown, chocolate like color.
What laboratory test let us to estimate final diagnosis:
A. Carboxyhemoglobin (COHb) concentration assay
B. Methemoglobin (MetHb) concentration assay
C. Urine test for narcotic substances assay
D. Haptoglobine assay

21.268 One person ingested by mistake little amount of concentrated acetic acid during the party. The doctor was
in the place of accident. What very important method of the first – aid should be done by the doctor:
A. Wash up the face of patient with cold water
B. To lay the patient on the left side
C. To give sedative for the patient
D. To give to drink 200 – 300 ml of fresh water for the patient

21.269 Mother visited Family Physician with her 11 year old boy. Child is sick for approximately 1 week. The boy
had fever up to 38°C for 2 days in the beginning of disease. Later body temperature become normal, but dry
cough and nasal congestion appeared and still present. Physician detected redness of the mucosa of mouth and
nose and wheezing in the lungs during physical examination. Body temperature of the boy 36,7°C; child is bouncy
and lively. What diagnosis should be considered:
A. Acute streptococcical tonsillitis
B. Acute upper respiratory tract infection complicated with acute bronchitis
C. Influenza

21.270 Mother visited Family Physician with her 11 year old boy. Child is sick for approximately 1 week. The boy
had fever up to 38°C for 2 days in the beginning of disease. Later body temperature become normal, but dry
cough and nasal congestion appeared and still present. Physician detected redness of the mucosa of mouth and
nose and wheezing in the lungs during physical examination. Body temperature of the boy 36,7°C; child is bouncy
and lively. What Family Physician should do
A. Should send the patient to the nearest emergency room, because there is strong need to perform chest
X-ray and to decide about hospitalisation
B. Prescribe symptomic treatment: cough and nasal congestion relievers, bronchodilatators, fluids.
C. Prescribe Amoxicilline 500 mg three times per day and symptomic treatment: cough and nasal
congestion relievers, bronchodilatators, fluids.

21.271 You are examining a 50-years old man who has been experiencing lower back pain and right leg sciatica
for 2 weeks. With his left knee in extension, you raise his left leg to 40°. This causes the patient to complain of pain
in his lower back that radiates into his right leg and calf. Which of the following statements is true:
A. A positive straight-leg-raising test is more specific for nerve-root compression on the contralateral side
than in the ipsilateral side.
B. It indicates the presence of the cauda equine syndrome.
C. It indicates the need for urgent neurosurgical intervention.

21.272 Middle-aged male was found unconscious on the street. Circumstances unknown. At inspection – facial
hemorrhages. Skull radiography of two projections – in the projection of occipital bone – an irregular uneven sharp
radiotransparent line of 8 cm length. Suspected pathology:
A. Tumour of occipital bone
B. Free air in cranial cavity
C. Fracture of occipital bone
D. Osteomyelitis of occipital bone
lsmusis.lsmuni.lt/Klausimai/Spausdinti?Length=0?Kalba=EN&KategorijaId=123&Kalbos_input=EN&Kalbos=EN&KategorijaEn_input=Mix&Kategorij… 34/38
3/27/2019 LSMUSIS
21.273 Second Thursday of September. 8:30AM. Head-to-head collision of bus and truck on the road from City A
to City B (15 km from A. Distance to City B – 45km.). The bus went down the road and rolled over. There were 38
passengers. The truck was loaded with food products. 8.32 AM a bus passenger called 112 and reported this
accident. 8:33AM a driver, passing the accident site called the 112 service and reported many injured in this
accident. City A has 300 beds hospital and ambulance service with 4 ambulance teams day time. City B has level-
II trauma centre and ambulance service with 9 ambulances day time. 135 km from accident site there is level-I
trauma center at the university hospital. You are of-duty physician, passing the accident site 8:35AM. What shall
you do first:
A. Take care about safety, evaluate situation, call emergency, start first aid to injured.
B. Start first aid, take care about safety, evaluate situation, call emergency.
C. Call emergency. Start first aid, take care about safety, evaluate situation.

21.274 Second Thursday of September. 8:30AM. Head-to-head collision of bus and truck on the road from City A
to City B (15 km from A. Distance to City B – 45km.). The bus went down the road and rolled over. There were 38
passengers. The truck was loaded with food products. 8.32 AM a bus passenger called 112 and reported this
accident. 8:33AM a driver, passing the accident site called the 112 service and reported many injured in this
accident. City A has 300 beds hospital and ambulance service with 4 ambulance teams day time. City B has level-
II trauma centre and ambulance service with 9 ambulances day time. 135 km from accident site there is level-I
trauma center at the university hospital. You are on-duty physician, arriving with the firs ambulance to the accident
site at 8:50AM. What shall you do first:
A. Immediately start assessment and emergency care of the injured.
B. Contact responsible police and rescue officers to get detailed information and instructions if it is safe to
act at the accident site.
C. Contact ambulance dispatcher and get information about the next arriving ambulances.

21.275 11 PM. Emergency department of the local hospital received a message from 112-center: car accident
involving a truck with a tank of chemicals. UN identifiers unreadable. Nine inhabitants of suburbs near the road
found dead. Rescuers reported 30 intoxicated people with breathing and vision problems. Decontamination in the
field is complicated due to low temperature (-5 degrees below zero Celsius). Is decontamination of these patients
obligatory in the emergency department:
A. Yes
B. No
C. Partial decontamination

21.276 11 PM. Emergency department of the local hospital received a message from 112-center: car accident
involving a truck with a tank of chemicals. UN identifiers unreadable. Nine inhabitants of suburbs near the road
found dead. Rescuers reported 30 intoxicated people with breathing and vision problems. Decontamination in the
field is complicated due to low temperature (-5 degrees below zero Celsius). Protective equipment needed for
personnel of emergency department:
A. Surgical mask, gown, protective goggles, latex gloves
B. Gas mask, protective suit, chemical resistant gloves
C. No protection equipment needed

21.277 11 PM. Emergency department of the local hospital received a message from 112-center: car accident
involving a truck with a tank of chemicals. UN identifiers unreadable. Nine inhabitants of suburbs near the road
found dead. Rescuers reported 30 intoxicated people with breathing and vision problems. Decontamination in the
field is complicated due to low temperature (-5 degrees below zero Celsius). You will need for decontamination:
A. Alkaline water solution
B. Acid water solution
C. Soap water solution

21.278 During examination of drinking water in the well, was found increased nitrate levels. Environment near the
well in bad order, 10 meters away from the well is cattle shed and compost pit. What are the main sources of
nitrate in well water:
A. Inorganic and organic fertilizers
B. Household waste
C. Rain water

21.279 During examination of drinking water in the well, was found increased nitrate levels. Environment near the
well in bad order, 10 meters away from the well is cattle shed and compost pit. Which population group is most
vulnerable to the effects of nitrates:
A. Elderly people
lsmusis.lsmuni.lt/Klausimai/Spausdinti?Length=0?Kalba=EN&KategorijaId=123&Kalbos_input=EN&Kalbos=EN&KategorijaEn_input=Mix&Kategorij… 35/38
3/27/2019 LSMUSIS
B. Infants
C. Adolescents

21.280 5 years girl during 2 last months presented with intensive tremor in her right arm. She looked neglectful to
her parents, she often harmed herself when going through the door or through the other narrow space in the room.
The girl started to run clumsy this month as well as she complained with morning headaches last 2-3 weeks.
Severe hedaches with nausea and vomiting began this morning. On admission in the hospital she was sluggish.
Nausea and vomiting have repeated once more. There was stiffness of the neck, impaired coordination with the
intensive tremor in the right arm and leg unstability of equilibrium. What disease can you suspect in this case:
A. Tumour of the left hemispherium of the brain
B. Sclerosis disseminate
C. Tumour of the cerebellar hemisphere
D. Hydrocephalus
E. Meningitis

21.281 5 years girl during 2 last months presented with intensive tremor in her right arm. She looked neglectful to
her parents, she often harmed herself when going through the door or through the other narrow space in the room.
The girl started to run clumsy this month as well as she complained with morning headaches last 2-3 weeks.
Severe hedaches with nausea and vomiting began this morning. On admission in the hospital she was sluggish.
Nausea and vomiting have repeated once more. There was stiffness of the neck, impaired coordination with the
intensive tremor in the right arm and leg unstability of equilibrium. Which are the most informative diagnostic
methods in this case:
A. SPECT
B. Lumbar puncture
C. Electromyography
D. Head CT or MRI scan
E. BERA

21.282 A man of approximately 30 years old was found lying in the meadow near the road. A broken motorcycle
was near him. He was conscious, but dezoriented, restless. After some period he complained with headache and
started vomiting and became drowsy again. During the 1 and half an hour he was transferred to the hospital. On
admission the patient ws unconsciuous already, with loosed contact, dilated left pupil, both eyes directed to the
left, defensive movements with the right arm to the painful stimulus. Bradicardia. Normal breathing. What kind of
head injury could be diagnosed:
A. Commotion of the brain
B. Diffuse axonal injury
C. Acute traumatic epidural haemorrhage in the left temporoparietal region
D. Fracture of the scull base in the middle fossa
E. Acute subdural haemorrhage on the right cerebral hemisherium

21.283 A man of approximately 30 years old was found lying in the meadow near the road. A broken motorcycle
was near him. He was conscious, but dezoriented, restless. After some period he complained with headache and
started vomiting and became drowsy again. During the 1 and half an hour he was transferred to the hospital. On
admission the patient ws unconsciuous already, with loosed contact, dilated left pupil, both eyes directed to the
left, defensive movements with the right arm to the painful stimulus. Bradicardia. Normal breathing. What methods
of treatment can you propose:
A. Urgent surgery: craniotomy and evacuation of epidural haematoma
B. Urgent external ventricular drainage
C. Bifrontal decompressive cranioectomy
D. Bolus of manitol and further conservative treatment
E. Urgent intubation of the patient, sedative, analgetics, myorelaxants; monitoring of vital functions

21.284 A corps of 56 years-old-man was found sitting in the car in the garage. On inspection no external injuries
are noticed. Lividity is of bright red color, the lividity remains during pressure on the posterior surfaces of the body.
During autopsy it was determined that blood in the heart chambers is fluid, without clots and brightly red. No
pathological changes were observed in the internal organs. Muscles and other organs were brightly red. What is
preliminary cause of death:
A. Sudden death
B. Carbon monoxide toxicity
C. Acute arrhythmia
D. Acute ethanol intoxication

lsmusis.lsmuni.lt/Klausimai/Spausdinti?Length=0?Kalba=EN&KategorijaId=123&Kalbos_input=EN&Kalbos=EN&KategorijaEn_input=Mix&Kategorij… 36/38
3/27/2019 LSMUSIS
21.1 - D 21.2 - A 21.3 - E 21.4 - B 21.5 - D 21.6 - B 21.7 - B 21.8 - D
21.9 - B 21.10 - D 21.11 - C 21.12 - E 21.13 - D 21.14 - E 21.15 - D 21.16 - C
21.17 - B 21.18 - B 21.19 - B 21.20 - D 21.21 - C 21.22 - A 21.23 - B 21.24 - C
21.25 - C 21.26 - A 21.27 - B 21.28 - C 21.29 - D 21.30 - C 21.31 - D 21.32 - B
21.33 - E 21.34 - B 21.35 - A 21.36 - D 21.37 - B 21.38 - A 21.39 - C 21.40 - C
21.41 - A 21.42 - C 21.43 - B 21.44 - E 21.45 - C 21.46 - B 21.47 - C 21.48 - D
21.49 - D 21.50 - C 21.51 - C 21.52 - B 21.53 - E 21.55 - B 21.56 - B 21.57 - C
21.58 - C 21.59 - C 21.60 - C 21.61 - A 21.62 - A 21.63 - D 21.64 - C 21.65 - B
21.66 - D 21.68 - D 21.69 - A 21.70 - C 21.71 - A 21.72 - A 21.73 - B 21.74 - A
21.75 - B 21.76 - A 21.77 - A 21.78 - B 21.79 - A 21.80 - A
21.81 21.82 21.83 21.85 21.86 21.87 21.88 21.89
1-E 1-B 1-B 1-C 1-B 1-A 1-A 1-E
2-C 2-A 2-C 2-D 2-C 2-D 2-C 2-C
3-A 3-C 3-A 3-A 3-A 3-B 3-B 3-B
4-D 4-D 4-B 4-C 4-D 4-A
5-B 5-D
21.90 21.91 21.92 21.93 21.94 21.95 21.96 21.97
1-D 1-D 1-B 1-C 1-C 1-C 1-B 1-B
2-E 2-C 2-C 2-A 2-D 2-B 2-C 2-C
3-A 3-B 3-A 3-B 3-B 3-D 3-A 3-D
4-C 4-A 4-A 4-A 4-A
5-B
21.98 21.99 21.100 21.101 21.102 21.103 21.104 21.105
1-C 1-C 1-B 1-B 1-C 1-B 1-B 1-D
2-A 2-A 2-C 2-A 2-A 2-C 2-A 2-C
3-D 3-D 3-A 3-D 3-D 3-A 3-A 3-A
4-B 4-B 4-D 4-C 4-B 4-A 4-B
21.106 21.107 21.108 21.109 21.110 21.111 21.112 21.113
1-B 1-C 1-C 1-C 1-B 1-C 1-C 1-A
2-A 2-D 2-B 2-D 2-D 2-D 2-D 2-C
3-C 3-A 3-A 3-A 3-C 3-A 3-A 3-B
4-D 4-B 4-B 4-A 4-B 4-B
5-E
21.114 21.115 21.116 21.117 21.118 21.119 21.120 21.121
1-D 1-D 1-C 1-B 1-C 1-D 1-B 1-B
2-C 2-B 2-B 2-C 2-A 2-C 2-E 2-D
3-A 3-C 3-A 3-A 3-D 3-B 3-D 3-C
4-B 4-B 4-D 4-B 4-A 4-A
5-E 5-E
21.122 21.123 21.124 21.125 21.126 21.127 21.128 21.129
1-B 1-B 1-A 1-C 1-C 1-C 1-D 1-B
2-A 2-D 2-A 2-A 2-A 2-B 2-C 2-A
3-D 3-A 3-B 3-B 3-B 3-D 3-D 3-C
4-C 4-C 4-B 4-D 4-A 4-A
5-A
6-B
21.130 21.131 21.132 21.133
1-B 1-C 1-C 1-A
2-A 2-B 2-A 2-B
3-C 3-A 3-B
4-D
5-E
21.134 21.135 21.136 21.137 21.138 21.139 21.140 21.141
1 4 2 2 1 1 1 2
2 4 4 2 2 2 4
3 3 3 3
21.143 21.144 21.145 21.147 21.148 21.149 21.150 21.151
lsmusis.lsmuni.lt/Klausimai/Spausdinti?Length=0?Kalba=EN&KategorijaId=123&Kalbos_input=EN&Kalbos=EN&KategorijaEn_input=Mix&Kategorij… 37/38
3/27/2019 LSMUSIS
1 1 1 1 1 1 2 1
2 2 2 2 2 2 4 2
3 3 3 3 3 3 3
4
5
21.152 21.153 21.154 21.155 21.156 21.157 21.158 21.159
4 1 1 1 1 1 1 4
3 3 2 2 3 2
3 3 3
4
21.160 21.161 21.162 21.163 21.164 21.165 21.166 21.167
4 1 1 1 2 1 1 1
3 2 2 4 2 2 2
3 3 3 3 3
4 4 4 4
21.168 21.170 21.171 21.172 21.173 21.174 21.175 21.176
1 1 1 1 1 4 1 1
2 2 2 3 2 2 3
3 3 3 3 3
4 4
21.177 21.178 21.179 21.180 21.181 21.182 21.183 21.184
4 1 1 1 1 1 1 1
2 3 2 2 2 2 2
3 3 3 3 3 3
4 4
5 5
21.185 21.186 21.187 21.188 21.189 21.190 21.191 21.192
1 1 1 1 1 1 1 1
2 2 2 2 2 2 2 2
3 3 3 3 3 3 3 3
4 4 4 4
5 5
21.193 21.194 21.195 21.196 21.197 21.198 21.199 21.200
4 1 1 1 4 1 4 2
3 2 2 2 4
3 3 3
4 4
21.201 21.202 21.203 21.204
1 1 1 1
2 2 3 3
3 3
4
21.205 - C 21.206 - B 21.207 - B 21.208 - C 21.209 - A 21.210 - A 21.211 - C 21.212 - E
21.213 - D 21.214 - C 21.215 - B 21.216 - A 21.217 - A 21.218 - B 21.219 - B 21.220 - C
21.221 - B 21.222 - B 21.223 - B 21.224 - A 21.225 - C 21.226 - A 21.227 - C 21.228 - B
21.229 - C 21.230 - A 21.231 - A 21.232 - C 21.233 - C 21.234 - D 21.235 - D 21.236 - C
21.237 - C 21.238 - B 21.239 - C 21.240 - B 21.241 - B 21.242 - B 21.243 - C 21.244 - C
21.245 - C 21.246 - C 21.247 - C 21.248 - B 21.249 - C 21.250 - E 21.252 - D 21.253 - B
21.254 - C 21.255 - B 21.256 - C 21.257 - B 21.258 - B 21.259 - C 21.260 - A 21.261 - B
21.262 - C 21.263 - D 21.264 - B 21.265 - B 21.266 - C 21.267 - B 21.268 - D 21.269 - B
21.270 - B 21.271 - A 21.272 - C 21.273 - A 21.274 - B 21.275 - A 21.276 - B 21.277 - C
21.278 - A 21.279 - B 21.280 - C 21.281 - D 21.282 - C 21.283 - A 21.284 - B

lsmusis.lsmuni.lt/Klausimai/Spausdinti?Length=0?Kalba=EN&KategorijaId=123&Kalbos_input=EN&Kalbos=EN&KategorijaEn_input=Mix&Kategorij… 38/38

You might also like